Download as pdf or txt
Download as pdf or txt
You are on page 1of 563

Завантажено з сайту https://тестування.

укр/ - онлайн тестування КРОК

ТЕСТУВАННЯ.УКР Бази тестів

Буклет 2005 року

Цей тест можна пройти в режимі онлайн тестування на сайті https://тестування.укр/testkrok/studing/568

Це офіційні тести з сайту Центру тестування https://www.testcentr.org.ua/

1. The physician must undertake measures for primary prophylaxis of iron deficiency anemia.
Which of the following categories of patient are subject to such primary prophylactic
measures?
A. Pregnant women*
B. Patients after 60
C. All children
D. Patients after operation
E. Workers of industrial enterprises

2. A patient with unstable angina pectoris was given the following complex treatment:
anticoagulants, nitrates, /a/pha-adrenoblockers. However on the third day of treatment the
pain still romains. Which in vestigation shoud be carried out to establish diagnosis?
A. Coronarography*
B. Stress-echocardiogram
C. Test with dosed physical exercises
D. Esophageal electrocardiac stimulator
E. Myocardial scintigraphy

3. The 28 y.o. woman applied to doctor because of limited loss of the hair. In the anamnesis - she
had frequent headache indisposition, arthromyalgia, fever, irregular casual sexual life, drug
user. RW is negative. What examination must be done first?
A. Examination for HIV*
B. Examination for neuropathology
C. Examination for gonorrhea
D. Examination for fungi
E. Examination for trichomoniasis

4. What methods of the collecting of the information is preferable for study of housing conditions
of students of medical HIGH SCHOOL for a training period?
A. Questioning*
B. Interviewing
C. Selecting of materials
D. A method of the directed selection
E. Statistical

5. Choose a method of a graphic representation of monthly information about number of the


registered cases of acute intestinal infection and their comparisons to the average monthly
values, obtained for 5 previous years:
A. The linear diagram*
B. The radial diagram

1
Завантажено з сайту https://тестування.укр/ - онлайн тестування КРОК

C. The sector diagram


D. The figured diagram
E. The curvilinear disgram

6. Define the basic registration document at the profound study of a case rate with temporary
lost labor ability at the industrial enterprise:
A. A card of the personal account of a case rate*
B. "The Report on reasons of a temporary lost labor ability"
C. The sick-leave certificate
D. A ambulatory medical card
E. The inpatient medical record

7. A patient, aged 25, complains of pain in the I finger on the right hand. On examination: the
finger is homogeneously hydropic, in bent position. On attempt to unbend the finger the pain
gets worse. Acute pain appears on touching with the probe in ligament projection. What
decease is the most likely?
A. Thecal whitlow (ligament panaritium)*
B. Subcutaneous panaritium
C. Articular (joint) panaritium
D. Bone panaritium
E. Paronychia

8. A sample of milk was taken for testing from a 5 ton milk batch. Lab analysis showed the
following: fat content 2%, specific density- 1,04 g/cm3, acidity 210 C, reductase probe - weak
positive. What way the product is to be used in? What would you advise?
A. Sell but inform customers about milk quality*
B. Write the product off for animal feeding
C. Utilize technically
D. Sell without limitations
E. Annihilate the product

9. Patient with thyrotoxicosis is in the 2-beds hospital ward of therapeutic department. The area
of the ward is 18 m2, height 3 m, ventilation rate 2,5/hr. Air temperature - 200C, relative
humidity 45%, air movement velocity 0,3 m/s, light coefficient 1/5, noise level 30 dB. Make
hygienic evaluation of the conditions.
A. Discomfortable microclimate*
B. Non-effective ventilation
C. Poor lighting
D. High level of noise
E. All conditions are OK

10. 33 y.o. woman works as the secretary. Her diet contains 150 g of protein (including 100 g of
animal protein), 200 g of fat, 600 g of carbohydrates. What pathology can develop from this
diet?
A. Obesity*
B. Schizophrenia

2
Завантажено з сайту https://тестування.укр/ - онлайн тестування КРОК

C. Paradontosis
D. Common cold
E. Uterine fibromyoma

11. A 9 y.o. girl has an average height and harmonic growth development. She was ill with acute
respiratory infection for five times. Define the group of her health.
A. 2nd group*
B. 1st group
C. 3rd group
D. 4th group
E. 5th group

12. The child is 11 m.o. He suffers from nervous-arthritic diathesis. The increased synthesis of
what acid is pathogenic at nervous-arthritic diathesis?
A. Uric acid*
B. Acetic acid
C. Phosphoric acid
D. Hydrochloric acid
E. Sulfuric acid

13. A 35 y.o. woman was admitted to thoracic surgery department with fever up to 400 C, onset of
pain in the side caused by deep breathing, cough with considerable quantity of purulent
sputum and blood with bad smell. What is the most likely diagnosis?
A. Abscess of the lung*
B. Complication of liver echinococcosis
C. Bronchiectatic disease
D. Actinomycosis of lungs
E. Pulmonary tuberculosis

14. A 52 y.o. man has recurrent transient ischemic attacks. Auscultation of the carotid arteries
detects murmur. What diagnostic method is necessary to apply first?
A. Ultrasound dopplerography*
B. CTof the brain
C. MRI of the brain
D. Cerebral angiography
E. Electroencephalography

15. A 67 y.o. patient complains of palpitation, dizziness, noise in ears, feeling of shortage of air.
Objectively: pale, damp skin. Vesicular respiration, respiratory rate- 22 per min, pulse- 200
bpm, AP-100/70 mm Hg. On ECG: heart rate-200 bmp, ventricular complexes are wi- dened,
deformed, location of segments ST and of wave T is discordant. The wave P is not changed,
superimposes QRST, natural conformity between P and QRS is not present. What kind of
arrhythmia is present?
A. Paroxismal ventricular tachycardia*
B. Sinus tachycardia
C. Atrial flutter

3
Завантажено з сайту https://тестування.укр/ - онлайн тестування КРОК

D. Ventricular extrasystole
E. Atrial tachycardia

16. Ambulance was called to a 48 y.o. man. From the words of relatives he has had three episodes
of lost consciousness and attacks during the day. On examination: the following fit is observed:
patient is unconscious, fell on the floor, tonic and then clonic convuesions of trunk and
extremities happened. The attack lasted for 4 minute, ended by involuntary urination. What
type of attack was observed?
A. Major epileptic seizure*
B. Vegetatic crisis
C. Absence
D. Episode of hysteria
E. Fainting

17. A 2 y.o. girl has been ill for 3 days. Today she has low-grade fever, severe catarrhal symptoms,
non-abundant maculopapular rash on her buttocks and enlarged occipital glands. What is your
diagnosis?
A. Rubella*
B. Scarlet fever
C. Measles
D. Adenoviral infection
E. Pseudotuberculosis

18. Male 30 y.o., noted growing fingers and facial scull, changed face. Complains of poor eyesight,
weakness, skin darkening, loss of body weight. X-ray shows broadening of sella turcica,
thinning of tuberculin sphenoidale, signs of increased intracranial pressure. What diagnosis
can you make?
A. Adenoma of hypophysis*
B. Encephalitis of truncus
C. Optico - hiasmatic arachnoiditis
D. Adrenal gland tumor
E. Tumor of pondo-cerebellar corner

19. A patient complains of a tormental (agonizing) cough with expectoration of up to 600 ml/daily
purulent chocolatecolor sputum with a decay smell. Onset of illness was abrupt, t0- 390C,
fever of irregular type. There is the area of darkening with a cavity in a center on X-ray film,
with irregular contours and level of liquid. What disease is the question?
A. Gangrene of lung*
B. Tuberculosis
C. Bronchiectatic illness
D. Pneumonia complicated by an abscess
E. Lobar pneumonia

20. A 24 y.o. patient complains of nausea, vomiting, headache, shortness of breath. He had an
acute nephritis being 10 y.o. Proteinuria was found out in urine. Objectively: a skin is grey-
pale, the edema is not present. Accent of II tone above aorta. BP 140/100-180/100 mm Hg.

4
Завантажено з сайту https://тестування.укр/ - онлайн тестування КРОК

Blood level of residual N2- 6,6 mmol/L, creatinine- 406 mmol/L. Day’s diuresis-2300 ml,
nocturia. Specific density of urine is 1009, albumin- 0,9 g/L, WBC- 0-2 in f/vis. RBC.- single in
f/vis., hyaline casts single in specimen. Your diagnosis?
A. Chronic nephritis with violation of kidney function*
B. Feochromocitoma
C. Hypertensive illness of the II degree
D. Nephrotic syndrome
E. Stenosis of kidney artery

21. A 33 y.o. male patient was admitted to a hospital. A patient is pale, at an attempt to stand up
he complains of strong dizziness. There was vomiting like coffee-grounds approximately hour
ago. BP- 90/60 mm Hg., pulse- 120 b/min. In anamnesis, a patient has suffered from ulcer of
the stomach, painless form during 4 years. An ulcer was exposed at gastrofi-beroscopy. Your
diagnosis:
A. Ulcer of stomach, complicated with bleeding*
B. Ulcer of duodenum, complicated with bleeding
C. Erosive gastritis
D. Acute pleurisy
E. Acute myocardial infarction, abdominal form

22. A 40 y.o. patient of rheumatic heart disease complains of anorexia, weakness and loss of
weigth, breathless and swelling of feet. On examination: t°- 39°C*, pulse is 100/min. As
ucultation: diastolic murmur in the mitral area. Petechical lesion a round clavicle; spleen was
palpable, tooth extraction one month ago.
A. Subacute bacteria endocarditis*
B. Recurrence of rheumatic fever
C. Thrombocytopenia purpure
D. Mitral stenosis
E. Aortic stenosis

23. A patient with nosocomial pneumonia has signs of collapse. Which of the following pneumonia
complication is the most likely to be accompanied with collapse?
A. Septic shock*
B. Exudative pleuritis
C. Bronchial obstruction
D. Toxic hepatitis
E. Emphysema

24. A 27 y.o. man complains of pain in epigastrium which is relieved by food intake. EGDFS shows
antral erosive gastritis, biopsy of antral mucous presents Helicobacter Pylori. What can be
diagnosed in this case?
A. Gastritis of type B*
B. Gastritis of A type
C. Reflux - gastritis
D. Menetrier’s disease
E. Rigid antral gastritis

5
Завантажено з сайту https://тестування.укр/ - онлайн тестування КРОК

25. A 62 y.o. patient with DM-2. Diabetes is being compensated by diet and Manini-lum. Patient
has to undergo an operation for inguinal hernia. What the tactics of hypoglycemic therapy
should be used?
A. Prescribe fast-acting insulin*
B. Give Glurenorm in place of Maninilum
C. Continue with the current therapy
D. Prescribe long-acting insulin
E. Prescribe guanyl guanidines

26. A 32 y.o. patient complains of severe weakness, tremor of extremities. On physical


examination, there is loss of body weight, wet and warm skin. The thyroid gland is enlarged up
to the 3rd degree, painless, elastic. Ps- 108 bpm. BP- 160/55 mm Hg. The rest is in norm. What
can be diagnosed?
A. Diffuse toxic goitre of the 3rd degree, thyrotoxicosis of the average degree*
B. Diffuse euthyroid goitre of the 3rd degree
C. Chronic autoimmune thyroiditis, hypertrophic type
D. Chronic fibrous thyroiditis
E. Toxiferous adenoma of the thyroid gland

27. A 30 y.o. female with rheumatoid arthritis of five years duration complains of pain in the frist
three fingers of her right hand over past 6 weeks. The pain seems especially severe at night
often awakening her from sleep.The most likelly cause is?
A. Carpal tunnel syndrome*
B. Atlanto-axial sublaxation of cervical spine
C. Sensory peripheral neuropathy
D. Rheumatoid vasculitis
E. Rheumatoid arthritis without complication

28. A 38 y.o. patient was urgently admitted to the hospital with complaints of sudden weakness,
dizziness, loss of consciousness, body weight loss, nausea, vomiting, severe pain in epigastric
area, diarrhea, skin hyperpigmentation. What is the most probable diagnosis?
A. Addisonic crisis*
B. Acute gastroenteritis
C. Meningoencephalitis
D. Scleroderma
E. Pellagra

29. An unconscious patient presents with moist skin, shallow breathing. There are signs of
previous injection on the shoulders and hips. BP- 110/70 mm Hg. Tonus of skeletal muscles
and reflexes are increased. Cramps of muscles of the extremities are seen. What is the most
likely disorder?
A. Hypoglycemic coma*
B. Hyperglycemic coma
C. Hyperosmolar coma
D. Hyperlactacidotic coma
E. Stroke

6
Завантажено з сайту https://тестування.укр/ - онлайн тестування КРОК

30. 7 y.o. boy with chronic sinusitis and rercurent pulmonary infections has chest X-ray
demonstrating a right-sided cardiac silhouette. What is the most likely diagnosis?
A. Kartagener syndrome*
B. Cystic fibrosis (mucoviscidosis)
C. Bronchiolitis obliterans
D. Laryngotracheomalacia
E. o-antitrypsin deficiency

31. A patient was admitted to the hospital on the 7th day of the disease with complaints of high
temperature, headache, pain in the muscles, especially in calf muscles. Dermal integuments
and scleras are icteric. There is hemorrhagic rash on the skin. Urine is bloody. The patient was
fishing two weeks ago. What is the most likely diagnosis?
A. Leptospirosis*
B. Yersiniosis
C. Salmonellosis
D. Brucellosis
E. Trichinellosis

32. A 2,9-kg term male infant is born to a mother who developed polyhydramnios at 34 weeks’
gestation. At birth, the Apgar scores were 9 and 9. The infant develops choking and cyanosis
with the first feed. In addition, is unable to place a nasogastric tube. What is the most likely
diagnosis?
A. Esophageal atresia*
B. Choanal atresia
C. Laryngomalacia
D. Tracheal atresia
E. Respiratory distress syndrome

33. Full term newborn has developed jaundice at 10 hours of age. Hemolytic disease of newborn
due to Rh-incompatibility was diagnosed. 2 hours later the infant has indirect serum bilirubin
level increasing up to 14 mmol/L. What is most appropriate for treatment of
hyperbilirubinemia in this infant?
A. Exchange blood transfusion*
B. Phototherapy
C. Phenobarbital
D. Intestinal sorbents
E. Infusion therapy

34. At term of a gestation of 40 weeks height of standing of a uterine fundus is less then assumed
for the given term. The woman has given birth to the child in weight of 2500 g, a length of a
body 53 cm, with an assessment on a scale of Apgar of 4-6 points. Labor were fast. The cause
of such state of the child were:
A. Chronic fetoplacental insufficiency*
B. Delay of an intra-uterine fetation
C. Placental detachment
D. Infection of a fetus
E. Prematurity

7
Завантажено з сайту https://тестування.укр/ - онлайн тестування КРОК

35. A 42 y.o. man died in a road accident after the haemorrhage on the site, due to acute
hemorrhagic anaemia. What minimum percentage volume of the total blood loss would result
in death at acute haemorrhage?
A. 25-30%*
B. 6-9%
C. 10-14%
D. 15-20%
E. 35-50%

36. What preparations are used for prevention of fungal infection?


A. Fluconozol, Orungol, Nisoral*
B. Rubomycin, Bleomycin, Mytomycin C
C. Cytosar, Cormyctin, Lomycitin
D. Captopril, Enalapril
E. Isoniazid, Ftibazid, Pyrazinamid

37. What developes most often after accidental intake of Hydrochloric acid?
A. Cardiac insufficiency*
B. Cushing’s syndrome
C. Kutling’s syndrome
D. Deylads’s syndrome
E. Acute pancreatitis

38. A woman, aged 40, primigravida, with infertility in the medical history, on the 42-43 week of
pregnancy. Labour activity is weak. Longitudinal presentation of the fetus, I position, anterior
position. The head of the fetus is engaged to pelvic inlet. Fetus heart rate is 140 bmp,
rhythmic, muffled. Cervix dilation is 4 cm. On amnioscopy: greenish colour of amni-otic fluid
and fetal membranes. Cranial bones are dense, cranial sutures and small fontanel are
A. Caesarean section*
B. Amniotomy, labour stimulation, fetal hypoxia treatment
C. Fetal hypoxia treatment, in the II period - forceps delivery
D. Fetal hypoxia treatment, conservative delivery
E. Medication sleep, amniotomy, labour stimulation

39. 30 y.o. woman, had mild trauma of 5th finger of the left hand 15 days ago. She has treated her
self at home. She presents to the hospital due to deterioration of the condition and
temperature rise. Objectively: hyperemia and swelling on the ventral surface of finger.
Restricted movements of the finger. X-ray of the left hand: an early stage of osteomyolitis of
the fifth finger could not be excluded. The diagnosis: panaris of 5th finger of the left hand.
What form of panaris has occurred in the patient?
A. Bony*
B. Hypodermic
C. Paronychia
D. Tendon type
E. Joints type

8
Завантажено з сайту https://тестування.укр/ - онлайн тестування КРОК

40. A 36 y.o. patient is diagnosed with right sided pneumothorax. What method of treatment is
indicated to the patient?
A. Surgical drainage of the pleural cavity*
B. Antiinflammation therapy
C. Symptomatic therapy
D. Pleural puncture
E. Thoracotomy

41. A youth, aged 15, from childhood suffers from atopic dermatitis and allergy to the shellfish. In
the last 3 months after acquiring aquarium fish, rhinitis, conjunctivitis, itching in the nose
developed. What level of immunologic index should be defined in this case?
A. IgE*
B. IgJ
C. IgM
D. IgA
E. Circulating immunocomplexes

42. The patient has developed pain in the axillary area, rise of temperature developed 10 hours
ago. On examination: shaky gait is marked, the tongue is coated by white coating. The pulse is
frequent. The painful lymphatic nodules are determined in the axillary area. The skin is
erythematous and glistering over the lymphatic nodules. What is the most probable diagnosis?
A. Bubonic plague*
B. Acute purulent lymphadenitis
C. Lymphogranulomatosis
D. Anthrax
E. Tularemia

43. A 17 y.o. patient complains of acute pain in the knee joint and t0- 380C. He was ill with angina
3 weeks ago. Objectively: deformation and swelling of the knee joints with skin hyperemia.
Small movement causes an acute pain in the joints. Which diagnose is the most correct?
A. Rheumatism, polyarthritis*
B. Systemic lupus eritematodes
C. Reactive polyarthritis
D. Infectious-allergic polyarthritis
E. Rheumarthritis

44. A 38 y.o. woman was hospitalized to the surgical unit with vomiting and acute abdominal pain
irradiating to the spine. On laparocentesis hemmorhagic fluid is obtained. What disease should
be suspected?
A. Acute pancreatitis*
B. Renal colic
C. Acute enterocolitis
D. Perforated gastric ulcer
E. Acute appendicitis

45. The girl is 12 y.o. Yesterday she was overcooled. Now she complains on pain in suprapubic

9
Завантажено з сайту https://тестування.укр/ - онлайн тестування КРОК

area, frequent painful urination by small portions, temperature is 37,80C. Pasternatsky


symptom is negative. Urinalysis: protein- 0,033 g/L, WBC-20-25 in f/vis, RbC- 1-2 in f/vis. What
diagnosis is most probable?
A. Acute cystitis*
B. Dysmetabolic nephropathy
C. Acute glomerulonephritis
D. Acute pyelonephritis
E. Urolithiasis

46. The girl of 11 y.o. She is ill for 1 month. She has "butterflytype rash on face (spots and
papules), pain and swelling of small joints on arms and legs, signs of stomatitis (small-sized
ulcers in mouth). CBC: Hb- 80 g/L, RBC- 2,9 * 1012/L, WBC- 15 * 109/L, ESR- 40 mm/hour.
Urinalysis: protein- 0,33 g/L. What is the most probable diagnosis?
A. Systemic lupus erythematosus*
B. Juvenile rheumatoid arthritis, systemic type
C. Periarteriitis nodosa
D. Acute rheumatic fever
E. Dermatomyositis

47. The 56 y.o. patient has worked at the aluminium plant more than 20 years. Within 3 last years
he has developed loosening of teeth, bone and joint pains, piercing pains in heart area,
vomiting. The preliminary diagnosis is:
A. Fluorine intoxication*
B. Mercury intoxication
C. Lead intoxication
D. Phosphorus intoxication
E. Manganese intoxication

48. A 38 y.o. woman complains of a purulent discharge from the left nostril. The body temperature
is 37,50C. The patient is ill during a week and associates her illness with common cold. Pain
on palpation of her left cheek reveals tenderness. The mucous membrane in the left nasal
cavity is red and turgescent. The purulent exudates is seen in the middle meatus in maxillary.
What is the most probable diagnosis?
A. Acute purulent maxillary sinusitis*
B. Acute purulent frontitis
C. Acute purulent ethmoiditis
D. Acute purulent sphenoiditis
E. -

49. A woman, primagravida, consults a gynecologist on 05.03.2002. A week ago she felt the fetus
movements for the first time. Last menstruation was on 10.01.2002. When should she be given
maternity leave?
A. 8 August*
B. 25 July
C. 22 August
D. 11 July
E. 5 September

10
Завантажено з сайту https://тестування.укр/ - онлайн тестування КРОК

50. An infant aged 1 year on the third day of common cold at night developed inspiratory stridor,
hoarse voice and barking cough. Physical examination revealed suprasternal and intercostal
chest retractions. There is a bluish skin discoloration moistly seen over the upper lip. The
respiratory rate is 52 per min and pulse- 122 bpm. The body temperature is 37,50C. What
disease does the infant have?
A. Acute infectious croup due to viral laryngotracheitis*
B. Acute laryngitis
C. Bronchopneumonia without complications
D. Acute bronchiolitis with respiratory distress
E. Acute epiglottitis

51. A newborn aged 3 days with hyperbilirubinemia (428 mkmol/L) developed following disorders.
From beginning there were severe jaundice with poor suckling, hypotomia and hypodynamia.
Little bit later periodical excitation, neonatal convulsions and neonatal primitive reflexes loss
are noted. Now physical examination reveals convergent squint, rotatory nystagmus and
setting sun eye sign. How to explain this condition?
A. Encephalopathy due to hyperbilirubinemia*
B. Skull injury
C. Brain tumour
D. Hydrocephalus
E. Spastic cerebral palsy

52. A 52 y.o. hard smoker patient complains of persistent cough with purulent sputum discharge
especially in mornings, dyspnea provoked even by slight physi- cal exertion, wheezing chest,
tahypnoe, general weakness. He considers himself to be ill during 12 years. The overwritten
conditions appear 3-4 times per year usually after common cold and have tendency to
progress. What disease do you think about first of all?
A. Chronic obstructive lung disease*
B. Bronchial asthma
C. Mucoviscidosis (cystic fibrosis)
D. Bronchoectatic disease
E. Aspergillosis

53. 3 weeks ago the patient was ill with tonsillitis. Clinical examination reveals edema, arterial
hypertension, hematuria, proteinuria (1,8 g/per day), granular and erythrocital casts. What is
the preliminary diagnosis?
A. Glomerulonephritis*
B. Cystitis
C. Pyelonephritis
D. Intestinal nephritis
E. Renal amyloidosis

54. 47 y.o. patient complains of intensive skin itching, jaundice, bone pain. The skin is
hyperpigmentated. There is multiple xanthelasma palpebrae. The liver is +6 cm enlarged, hard
with acute edge. The blood analysis revealed total bilirubin 160 mkmol/L, direct -110 mkmol/L,
AST (asparate aminotransferase)- 2,1 mmol/L per hour, ALT- 1,8 mmol/L, alkaline
phosphotase- 4,6 mmol/L per hour, cholesterol- 9,2 mmol/L, antimitochondrial antibodies M2

11
Завантажено з сайту https://тестування.укр/ - онлайн тестування КРОК

in a high titer. What is the probable diagnosis?


A. Primary biliary liver cirrhosis*
B. Primary liver cancer
C. Chronic viral hepatitis B
D. Acute viral hepatitis B
E. Alcoholic liver cirrhosis

55. The complications of acute cholecystitis which require surgical intervention are as follows
EXCEPT:
A. Jaundice*
B. Empyema of the gall-bladder
C. Emphysematous gall-bladder
D. Gall-bladder perforation
E. Cholangitis conditioned by the presence of stones in the bile tract

56. The 67 y.o. patient had 5 recurrent fractures of the lower extremities without considerable
cause within 5 years. O-shaped deformity of the legs in the knee joints has appeared. The
skull, pelvis and lower extremities X-Ray shows the thickening of flat bones. In the long bones
there is a hyperostosis along the bone axis.The blood test has not revealed any inflammation
activity. Serum calcium is normal. What disease do you consider in this case?
A. Paget’s disease*
B. Hyperparathyoid dystrophy
C. Chronic osteomyelitis
D. Myeloma
E. Mottled disease (marble disease)

57. The woman who has delivered twins has early postnatal hypotonic uterine bleeding reached
1,5% of her bodywei-ght. The bleeding is going on. Conservative methods to arrest the
bleeding have been found ineffective. The conditions of patient are pale skin, acrocyanosis,
oliguria. The woman is confused. The pulse is 130 bpm, BP- 75/50 mm Hg. What is the further
treatment?
A. Uterine extirpation*
B. Supravaginal uterine amputation
C. Uterine vessels ligation
D. Inner glomal artery ligation
E. Putting clamps on the uterine cervix

58. A 26 y.o. woman complains of a mild bloody discharge from the vagina and pain in the lower
abdomen. She has had the last menstruation 3,5 months ago. The pulse is 80 bpm. The blood
pressure (BP) is 110/60 mm Hg and body temperature is 36,60C. The abdomen is tender in the
lower parts. The uterus is enlarged up to 12 weeks of gestation. What is your diagnosis?
A. Inevitable abortion*
B. Incipient abortion
C. Incomplete abortion
D. Complete abortion
E. Disfunctional bleeding

12
Завантажено з сайту https://тестування.укр/ - онлайн тестування КРОК

59. 18 y.o. woman complains of pain in the lower abdomen. Some minutes before she has suddenly
appeared unconscious at home. The patient had no menses within last 3 months. On
examination: pale skin, the pulse- 110 bpm, BP- 80/60 mm Hg. The Schyotkin’s sign is positive.
Hb- 76 g/L. The vaginal examination: the uterus is a little bit enlarged, its displacement is
painful. There is also any lateral swelling of indistinct size. The posterior fornix of the vagina is
tendern and overhangs inside. What is the most probable diagnosis?
A. Impaired extrauterine pregnancy*
B. Ovarian apoplexy
C. Twist of cystoma of right uterine adnexa
D. Acute salpingoophoritis
E. Acute appendicitis

60. A 20 y.o. pregnant woman with 36 weeks of gestation was admitted to the obstetrical hospital
with complains of pain in the lower abdomen and bloody vaginal discharge. The general
condition of the patient is good. Her blood pressure is 120/80 mm Hg. The heart rate of the
fetus is 140 bpm, rhythmic. Vaginal examination: the cervix of the uterus is formed and closed.
The discharge from vagina is bloody up to 200 ml per day. The head of the fetus is located high
above the minor pelvis entry. A soft formation was defined through the anterior fornix of the
vagina. What is the probable diagnosis?
A. Placental presentation*
B. Premature placental separation
C. Uterine rupture
D. Threatened premature labor
E. Incipient abortion

61. In the gynecologic office a 28 y.o. woman complains of sterility within three years. The
menstrual function is not impaired. There were one artificial abortion and chronic salpingo-
oophoritis in her case history. Oral contraceptives were not used. Her husband’s analysis of
semen is without pathology. From what diagnostic method will you start the workup in this
case of sterility?
A. Hysterosalpingography*
B. Hormone investigation
C. Ultra sound investigation
D. Diagnostic scraping out of the uterine cavity
E. Hysteroscopia

62. A 33 y.o. patient was admitted to the reception room of the Central District Hospital. He
complains of a severely painful swelling localized on posterior neck, fever up to 38,40C and
general weakness. In anamnesis: diabetes mellitus within 5 years. On physical examination on
the posterior neck surface there is an infiltrate elevated above surrounding skin. The tissues
affected by swelling are tense and blue reddish discoloration in central area. There are also
several purulent necrotic pustules which are connected with each other and form a large skin
necrosis. A thinned necrotic skin of this swelling has holes looking like sieve, pus discharges
through out. What disease should a doctor consider first of all?
A. Carbuncle*
B. Furuncle
C. Acute skin cellulitis
D. Carbuncle associated with anthrax

13
Завантажено з сайту https://тестування.укр/ - онлайн тестування КРОК

E. Skin abscess

63. A 19 y.o. man was admitted to the reception department in 20 minutes after being wounded
with the knife to the left chest. The patient is confused. The heart rate is 96 bpm and BP- 80/60
mm Hg. There are the dilated neck veins, sharply diminished apical beat and evident heart
enlargement What kind of penetrative chest wound complications has developed in patient?
A. Pericardium tamponade*
B. Massive hemothorax
C. Open pneumothorax
D. Closed pneumothorax
E. Valve-likes pneumothorax

64. A 35 y.o. patient complains of a difficult swallowing, pain behind the breastbone. He can eat
only liquid food. While swallowing sometimes he has attacks of cough and dyspnea. Above
mentioned complaints are progressing. It is known that the patient has had a chemical burn of
esophagus one month ago. What complication does the patient have?
A. Corrosive esophagitis and stricture*
B. Esophagitis
C. Esophageal diverticula
D. Cardiac achalasia
E. Cardiac insufficiency

65. An employee of a private company was ill with acute respiratory viral infection. Consulted a
district doctor, who determined the fact of temporary loss of working ability, but refused to
issue a sick-list, arguing that the patient worked in the private and not state-owned company.
Should the sick-list be issued to the employees of private companies?
A. Issued regardless of company’s ownership*
B. Issued only to empties of state-owned companies
C. Issued only on condition of payment guarantee by the company’s proprietor
D. Issued a medical certificate of a set form
E. Issued a medical certificate of a free form

66. A worker was temporarily off work because of illness during 16 days, was under out-patient
treatment. The doctor in charge issued a sick-list first for 5 days, then prolonged it for 10 days.
Who can further prolong the sick-list to this patient?
A. The doctor in charge of the case together with the head of department*
B. Deputy head physician on the working ability expertise
C. The doctor in charge of the case with the permission of the head of department
D. Working ability expertise committee
E. The head of department

67. A 13 y.o. patient was treated in dermatological hospital for atopic dermatitis exacerbation. He
was discharged in the condition of clinical remission. What recommendations should the
doctor give to prevent exacerbations?
A. Use of neutral creams to protect skin*
B. Frequent skin washing with detergents

14
Завантажено з сайту https://тестування.укр/ - онлайн тестування КРОК

C. Systematic use of local corticosteroids


D. Systematic skin disinfection
E. Avoidance of skin insolation

68. A full-term new-born suffered ante-and intranatal hypoxia, was born in asphyxia (Apgar score
2-5 points). After birth baby’s excitation is progressing, occurs vomiting, nystagmus, spasms,
squint, spontaneous Babinski and Moro’s reflexes. What is the most probable location of the
intracranial haemorrhage in this case?
A. Subarachnoid hemorrhages*
B. Small hemorrhages in brain tissue
C. Subdural hemorrhages
D. Periventricular hemorrhages
E. Haemorrhages in ventricles of brain

69. A patient, aged 16, complains of headache, mainly in the frontal and temporal areas,
superciliary arch, appearing of vomiting at the peak of headache, pain during the eyeballs
movement, joint’s pain. On examination: excited, t0- 390C, Ps- 110/min. Tonic and clonus
cramps. Uncertain meningeal signs. What is the most likely diagnosis?
A. Influenza with cerebral edema manifestations*
B. Influenza, typical disease duration
C. Respiratory syncytial virus
D. Parainfluenza
E. Adenovirus infection

70. A 64 y.o. patient has developed of squeering substernal pain which had appeared 2 hours ago
and irradiated to the left shoulder, marked weakness. On examination: pale skin, cold sweat.
Pulse- 108 bpm, AP- 70/50 mm Hg, heart sound are deaf, vesicular breathing, soft abdomen,
painless, varicouse vein on the left shin, ECG: synus rhythm, heart rate is 100 bmp, ST-
segment is sharply elevated in II, III aVF leads. What is the most likely disorder?
A. Cardiogenic shock*
B. Cardiac asthma
C. Pulmonary artery thromboembolia
D. Disquamative aortic aneurizm
E. Cardiac tamponade

71. In treatment and prevention establishments, regardless of their organisational and proprietary
form, the rights of the patients should be observed. Which of these rights is the most
significant?
A. The right to the protection of the patient’s interests*
B. The right to the free choice
C. The right to the information
D. The right to be heard
E. The right to the protection from incompetence

72. A military unit stopped for 3-day’s rest in inhabited locality after a long march. The sanitary-
epidemiological reconnaissance found several water sources. It is necessary to choose the

15
Завантажено з сайту https://тестування.укр/ - онлайн тестування КРОК

source complying with the hygienic standards for potable water in the field
A. Artesian well water*
B. Spring water
C. River water
D. Rain water
E. Water from melted snow

73. The district pediatrician is charged with the analysis of infant mortality. What is taken for the
unit of observation in infant mortality investigation?
A. A baby dead at the age up to 12 months*
B. A baby dead at the age up to 1 months
C. A baby dead at the age over 28 days
D. A baby dead at the age up to 6 days
E. A baby dead at birth

74. Chief district pediatrician has to carry out analysis of infant mortality rate. What should he
take as a unit of the observation?
A. Child death case at the age up to 1 year*
B. Child death case at the age up to the first month
C. Child death case after 28 days of life
D. Child death case during first 7 days of life
E. Child death case on labor

75. A 43 y.o. patient complains of mass and, pain in the right breast, elevation of temperature to
37,20C during 3 last months. Condition worsens before the menstruation. On examination:
edema of the right breast, hyperemia, retracted nipple. Unclear painful infiltration is palpated
in the lower quadrants. What is the most probable diagnosis?
A. Cancer of right mammary gland*
B. Right side acute mastitis
C. Right side chronic mastitis
D. Premenstrual syndrome
E. Tuberculosis of right mammary gland

76. A patient, aged 40, has been ill during approximately 8 years, complains of pain in the lumbar
part of the spine on physical excertion, in cervical and thoracal part (especially when
coughing), pain in the hip and knee joints on the right. On examination: the body is fixed in the
forward inclination with head down, gluteal muscles atrophy. Spine roentgenography: ribs
osteoporosis, longitudinal ligament ossification. What is the most likely diagnosis?
A. Ancylosing spondyloarthritis*
B. Tuberculous spondylitis
C. Psoriatic spondyloarthropatia
D. Spondyloarthropatia on the background of Reiter’s disease
E. Spread osteochondrosis of the vertebral column

77. A worker, aged 38, working in the slate production during 15 years, complains of expiratory
exertional dyspnea, dry cough. On examination: deafening of the percutory sounds in

16
Завантажено з сайту https://тестування.укр/ - онлайн тестування КРОК

interscapular region, rough breath sounds, dry disseminated rales. On fingers’ skin - greyish
warts. Factory’s sectorial doctor suspects asbestosis. Which method is the most informative for
diagnosis verification?
A. Thorax roentgenography*
B. Bronchoscopy
C. Spirography
D. Bronchoalveolar lavage
E. Blood gases examination

78. Deputy of chief medical officer carried out a study of morbidity rate for population which had
been served at the polyclinics for the last 5 years. What statistical values can help in
calculations of diseases level dissamination?
A. Relative values*
B. Standart values
C. Average values
D. Absolute values
E. Dynamic row

79. A 37 y.o. woman is suffering from squeezing substernal pain on physical exertion. On
examination: AP- 130/80 mm Hg, heart rate=pulse rate 72 bpm, heart boarders are dilated to
the left side, aortic systolic murmur. ECG- signs of the left venticle hypertrophy. What method
of examination is the most informative in this case?
A. Echocardiography*
B. Phonocardiography
C. Coronarography
D. Sphygmography
E. X-ray

80. A child, aged 4, has being ill for 5 days, suffers from cough, skin rash, t0- 38, 20C, facial
hydropy, photosensitivity, conjunctivitis. On the face, neck, upper part of the chest there is
bright maculopapular rash with areas of merging. Hyperemic throat. Seropurulent nasal
discharge. In lungs there are dry crackles. What is the most probable preliminary diagnosis?
A. Measles*
B. Adenovirus infection
C. Scarlet fever
D. Rubella
E. Enterovirus exanthema

81. There were registered 500 cases of urolithiasis per 10000 inhabitants. What kind of statictical
indices is presented?
A. Prevalence rate*
B. Correlation coefficient
C. Index of visualization
D. Incidence rate
E. Index of compliance

17
Завантажено з сайту https://тестування.укр/ - онлайн тестування КРОК

82. At year-end hospital administration has obtained the following data: annual number of treated
patients and average annual number of patient-used beds. What index of hospital work can be
calculated based upon this data?
A. Bed turnover*
B. Bed resources of the hospital
C. Average annual bed occupacy
D. Average duration of patients presence in the hospital
E. Average bed idle time

83. A 52 y.o. patient fell from 3 m hight on the ground with the right lumbar area. He complains of
pain here. There is microhematuria in the urea. On urography kidney’s functioning is
satisfactory. What is the most probable diagnosis?
A. Kidney’s contusion*
B. Subcapsular kidney’s rupture
C. Multile kidney’s ruptures
D. Paranephral hematoma
E. Kidney’s abruption

84. A 3 y.o. child with weight deffici-ency suffers from permanent moist cough. There was a
history of some pneumonias with obstruction. On examination: distended chest, dullness on
percussion over the lower parts of lungs. On auscultation: a great number of different moist
rales. Level of sweat chloride is 80 mol/L. What is the most probable diagnosis?
A. Mucoviscidosis (cystic fibrosis)*
B. Bronchial asthma
C. Recurrent bronchitis
D. Bronchiectasis
E. Pulmonary hypoplasia

85. A 14 y.o. girl complains of profuse bloody genital discharges during 10 days after suppresion
of menses for 1,5 months. Similiar bleedings recur since 12 years on the background of
disordered menstrual cycle. On rectal examination: no pathology on the internal genitalia. In
blood: Hb- 70 g/L, RBC- 2, 3 * 1012/L, Ht-20. What is the most probable diagnosis?
A. Juvenale bleeding, posthemorrhagic anemia*
B. Werlholf’s disease
C. Polycyst ovarian syndrome
D. Hormonoproductive ovary tumor
E. Noncomplete spontaneous abortion

86. A 14 y.o. patient suddenly fell ill when high fever, acute pain in the right shin. In two weeks X-
ray showed translucent spaces (destructive focuses) with unevel countours in the middle third
of tibia di-aphysis. Along the bone edge there was a narrow line of shadow (periostitis) 1-2 mm
from the surface. What is the most likely diagnosis?
A. Right shin osteomielitis*
B. Right shin tuberculosis
C. Right shin syphilis
D. Bone cyst of tibia
E. Right shin trauma

18
Завантажено з сайту https://тестування.укр/ - онлайн тестування КРОК

87. A 40 y.o. patient was admitted to the gasteroenterology with skin itching, jaundice, discomfort
in the right subcostal area, generalized weakness. On examination: skin is jaundice, traces of
scratches, liver is +5 cm, splin is 6x8 cm. In blood: alkaline phosphatase - 2,0 mmol/(hour*L),
general bilirubin - 60 mkmol/L, cholesterol - 8,0 mmol/L. What is the leading syndrome in the
patient?
A. Cholestatic*
B. Cytolytic
C. Mesenchymal inflammatory
D. Asthenic
E. Liver-cells insufficiency

88. A 43 y.o. patient had cholecystectomy 6 years ago due to chronic calculous cholecystitis.
Lately he has suffered from pain in the right subcostal area and recurrent jaundice. During
last 2 weeks jaundice hasn’t gone. Stenoutic papillitis 0,5 cm in length has been revealed.
What is the best way of treatment?
A. To perform endocsopic papillosphincterotomy*
B. To treat conservatively: antibiotics, spasmolytics, antiinflammatory drugs
C. To perform external choledoch drain
D. To perform choledochoduodenostomy
E. -

89. A 12 y.o. child with acute glomerulonephritis presented with hypertensive syndrom during first
days of the disease. What is the role of angiotesin II in the pathogenesis?
A. Intensifies production and secretion of aldosterone*
B. Increases heart output
C. Infibits deppresive action of prostaglandins
D. Increases erythropoetin production
E. Increases renine level

90. On physiologic-sanitary examination of railway department work it was revealed that loaders
work is of III degree of difficulty. They unload vagons with sand, manually break coagulated
mass by shovel and shift it. What criteria was used to evaluate work of loaders?
A. Maximun load weigh which is shifted*
B. Value of static loading for the shift
C. Time of active activities, % to the shift duration
D. Time of passive observation, % to the shift duration
E. Intellectual efforts

91. A 51 y.o. woman complains of dull pain in the right subcostal area and epigastric area, nausea,
appetite decline during 6 months. There is a history of gastric peptic ulcer. On examination:
weight loss, pulse is 70 bpm, AP is 120/70 mm Hg. Diffuse tenderness and resistance of
muscles on palpation.There is a hard lymphatic node 1x1cm in size over the left clavicle. What
method of investigation will be the most useful?
A. Esophagogastroduodenoscopy with biopsy*
B. Ultrasound examination of abdomen
C. pH-metry
D. Ureatic test

19
Завантажено з сайту https://тестування.укр/ - онлайн тестування КРОК

E. Stomach X-ray

92. A man, aged 68, complains of ti- redness, sweating, enlargement of cervical, submaxillary and
axillary lymph nodes. Blood tests: WBC - 35 * 109/L, lymphocytes - 60%, Botkin and
Gumprecht bodies, level of haemoglobin and quantity of thrombocytes is normal. Myelogram
showed 40% of lymphocytes. What is the most probable diagnosis?
A. Chronic lympholeucosis*
B. Chronic myeloleucosis
C. Lymphogranulomatosis
D. Acute leucosis
E. Tuberculous lymphadenitis

93. Examination of placenta, which has just been born, reveals defect 2x3 cm in size. There is no
bleeding. What tactics is the most reasonable?
A. Manual uretus cavity revision*
B. Administration of uterotonic medicines
C. External uterus massage
D. Parturient supervision
E. Instrumental uterus cavity revision

94. A 27 y.o. gravida with 17 weeks of gestation was admitted to the hospital. There was a history
of 2 spontaneous miscarriages. On bimanual examination: uterus is enlarged to 17 weeks of
gestation, uterus cervix is shortened, isthmus allows to pass the finger tip. The diagnosis is
isthmico-cervical insufficiency. What is the doctor’s tactics?
A. To place suture on the uterus cervix*
B. To administer tocolytic therapy
C. To interrupt pregnancy
D. To administer hormonal treatment
E. To perform amniocentesis

95. A district pediarician has carried out infant mortality rate analysis in his area. What data has
been used?
A. Mortality of children under 1 y.o. structured by age, sex, causes*
B. Mortality of children under 1 y.o., natimortality
C. Hospital mortality of children, structured by age
D. Mortality of district adolescents
E. Mortality of district newborn

96. A 26 y.o. male patient with postoperative hypothyroidism take thyroxine 100 mg 2 times a day.
He has developed tachycardia, sweating, irritability, sleep disorder. Determine further
treatment tactics.
A. To decrease thyroxine dosage*
B. To increase thyroxine dosage
C. To administer betablockers
D. To add mercasolil to the treatment
E. To administer sedatives

20
Завантажено з сайту https://тестування.укр/ - онлайн тестування КРОК

97. Body temperature of a 12 y.o. girl increased up to 39—40°C in 4-5 hours after she had taken 2
pills of aspirin. Complains of general discomfort, dizziness, sudden appearance of red spots on
the skin with blister formation or exfoliation of the epidermis with erosive surface. Lesions on
the skin looked like burns of II degree. Nikolsky syndrome is positive. What is the most
probable diagnosis?
A. Acute epidermical necrolysis*
B. Pemphigus vulgaris
C. Polymorphic exudative erythema
D. Bullous dermatitis
E. Duhring’s disease

98. A patient complains of pathological lump, appearing in the right inguinal region on exercise.
The lump is round-shaped, 4 cm in diameter, on palpation: soft elastic consistency, positions
near the medial part of Poupart’s ligament. The lump is situated inwards from the spermatic
cord. What is the most probable preliminary diagnosis?
A. Right-sided direct inguinal hernia*
B. Right-sided oblique inguinal hernia
C. Right-sided femoral hernia
D. Varicose veins of the right hip
E. Lipoma of the right inguinal area

99. A 52 y.o. woman complains of weakness, painful itching after washing and bathing, sensation
of heaviness in the head. On examination: hyperemia of skin of face, neck, extremities. AP-
180/100 mm Hg. Speeln is 4 cm below the rib arch edge. What is the most probable diagnosis?
A. Erythremia*
B. Essential hypertension
C. Dermatomyositis
D. Allergic dermatitis
E. Systemic sclerodermia

100. A 55 y.o. patient complains of distended abdomen and rumbling, increased winds evacuation,
liguid foamy feces with sour smell following the diary products consumption. What is the
correct name of this syndrome?
A. Syndrome of fermentative dyspepsia*
B. Syndrome of decayed dyspepsia
C. Syndrome of fatty dyspepsia
D. Dyskinesia syndrome
E. Malabsorption syndrome

101. In an inhabited locality there is an increase of diphtheria during the last 3 years with separate
outbursts in families. What measure can effectively influence the epidemic process of
diphtheria and reduce the morbidity rate to single cases?
A. Immunization of the population*
B. Hospitalization of patients
C. Detection of carriers
D. Early diagnostics
E. Disinfection in disease focus

21
Завантажено з сайту https://тестування.укр/ - онлайн тестування КРОК

102. A 27 y.o. woman turns to the maternity welfare centre because of infertility. She has had
sexual life in marriage for 4 years, doesn’t use contraceptives. She didn’t get pregnant. On
examination: genital development is without pathology, uterus tubes are passable, basal
(rectal) temperature is one-phase during last 3 menstrual cycles. What is the infertility cause?
A. Anovular menstrual cycle*
B. Chronic adnexitis
C. Abnormalities in genital development
D. Immunologic infertility
E. Genital endometriosis

103. A 43 y.o. woman complains of contact hemorrhages during the last 6 months. Bimanual exam:
cervix of the uterus is enlarged, restricted in mobility. Mirrors showed the following: cervix of
the uterus is in the form of cauliflower. Chrobak and Schiller tests are positive. What is the
most probable diagnosis?
A. Cancer of cervix of the uterus*
B. Polypus of the cervis of the uterus
C. Cervical pregnancy
D. Nascent fibroid
E. Leukoplakia

104. A local doctor has to prepare a report about the health condition of the population of his
region. What medical indexes of population health condition should he use?
A. Morbidity, disabilities, demographic, physical development*
B. Social welfare, satisfaction of life quality
C. Way of life, genetic, pollution
D. Average treatment duration, complications
E. Average longevity

105. A boy, aged 9, is examined: height-127 cm (-0,36), weight - 28,2 kg (+0,96), chest
circumference- 64,9 cm (+0,66), lung vital capacity -1520 ml (-0,16). What is the integrated
assessment of the child’s physical development?
A. Harmonious*
B. Disharmonious
C. Markedly disharmonious
D. Excessive
E. Below the average

106. A 14 y.o. child suffers from of vegetovascular dystonia of pubertal period. He has developed
sympathoadrenal crisis. What medicine should be used for crisis reduction?
A. Obsidan*
B. No-shpa
C. Amisyl
D. Euphyline
E. Corglicone

107. A woman complains of high temperature to 380C, mild pain in the throat during 3 days. On

22
Завантажено з сайту https://тестування.укр/ - онлайн тестування КРОК

examinati- on: angle lymphatic nodes of the jaw are 3 cm enlarged, palatinel tonsils are
enlarged and coated with grey plaque which spreads to the uvula and frontal palatinel arches.
What is the most probable diagnosis?
A. Larynx dyphtheria*
B. Infectious mononucleosis
C. Vincent’s angina
D. Agranulocytosis
E. Oropharyngeal candidosis

108. A 36 y.o. patient was admitted to the hospital with sharp pain in substernal area following
occasional swallowing of a fish bone. On esophagoscopy the foreign body wasn’t revealed. The
pain increased and localized between scapulas. In a day temperature elevated, condition
became worse, dysphagia intensified. What complication has developed?
A. Perforation of esophagus with mediasti-nitis*
B. Esophageal hemorrhage
C. Obstruction of esophagus
D. Pulmonary atelectasis
E. Aspirative pneumonia

109. A child from the first noncomplicated pregnancy but complicated labor had cephalhematoma.
On the second day there developed jaundice. On the 3th day appeared changes of neurologic
status: nystagmus, Graefe’s sign. Urea is yellow, feces- golden-yellow. Mother’s blood group is
A(II)Rh-, child-A(II)Rh+. On the third day child’s Hb is 200 g/L, RBC- 6,1 * 1012/L, bilirubin in
blood - 58 mk mol/L due to unconjugated bilirubin, Ht- 0,57 What is the child’s jaundice
explanation?
A. Brain delivery trauma*
B. Physiologic jaundice
C. Hemolytic disease of newborn
D. Bile ducts atresia
E. Fetal hepatitis

110. A gravida with 7 weeks of gestation is referred for the artificial abortion. On operation while
dilating cervical canal with Hegar dilator N°8 a doctor suspected uterus perforation. What is
immediate doctors tactics to confirm the diagnosis?
A. Probing of uterus cavity*
B. Bimanual examination
C. Ultrasound examination
D. Laparoscopy
E. Metrosalpingography

111. A 18 y.o. male patient complains of pain in knee and ankle joints, temperature elevation to
39,50C. He had a respiratory disease 1,5 week ago. On examination: temperature- 38,50C,
swollen knee and ankle joints, pulse- 106 bpm, rhythmic, AP- 90/60 mm Hg, heart borders
without changes, sounds are weakened, soft systolic apical murmur. What indicator is
connected with possible etiology of the process?
A. Antistreptolysine-0*
B. 1-antitrypsine

23
Завантажено з сайту https://тестування.укр/ - онлайн тестування КРОК

C. Creatinkinase
D. Rheumatic factor
E. Seromucoid

112. A 19 y.o. patient was admitted to the hospital with acute destructive appendicitis. He sufferes
from hemophilia B-type. What antihemophilic medicine should be inclended in pre-and post-
operative treatment plan?
A. Fresh frosen plasma*
B. Cryoprecipitate
C. Fresh frosen blood
D. Native plasma
E. Dried plasma

113. A 24 y.o. male patient was transferred to the chest surgery department from general surgical
department with acute post-traumatic empyema of pleura. On the X-ray: wide level horizontal
of fluid on the right. What method of treatment should be prescribed?
A. Punction and drainage of pleural cavity*
B. Decortication of pleura
C. Pneumoectomy
D. Thoracoplasty
E. Lobectomy

114. A 28 y.o. homeless male was admitted to the hospital because of initial diagnosis "influenza".
Roseolo-petechiae rash has appeared on the trunk and internal surfaces of the limbs on the
fifth day. Temperature is 410C, euphoria, face and sclera’s hyperemia, tongue tremor,
tachycardia, splenomegaly, excitement. What is the most probable diagnosis?
A. Typhus*
B. Alcogolic delirium
C. Leptospirosis
D. Measles
E. Abdominal typhoid

115. A patient complains of intense pressing pain in the pharynx, mainly to the right, impossibility
to swallow even liquid food. The illness started 5 days ago. The patient’s condition is grave.
Body temperature - 38, 90C, speech is difficult, voice is constrained, difficulties in opening the
mouth. Submaxillary glands to the right are painful, enlarged. What is the most probable
diagnosis?
A. Peritonsillar abscess*
B. Diphtheria
C. Pharyngeal tumour
D. Vincent’s disease
E. Phlegmonous tonsillitis

116. In a 65 y.o. female patient a tumor 13x8 cm in size in the umbilical area and above is palpated,
mild tenderness on palpation, unmovable, pulsates. On ausculation: systolic murmur. What is
the most probable diagnosis?

24
Завантажено з сайту https://тестування.укр/ - онлайн тестування КРОК

A. Abdominal aortic aneurism*


B. Stomach tumor
C. Arterio-venous aneurism
D. Tricuspid valve insufficiency
E. Mitral insufficiency

117. A 25 y.o. woman complains of profuse foamy vaginal discharges, foul, burning and itching in
genitalia region. She has been ill for a week. Extramarital sexual life. On examination:
hyperemia of vaginal mucous, bleeding on touching, foamy leucorrhea in the urethral area.
What is the most probable diagnosis?
A. Trichomonas colpitic*
B. Gonorrhea
C. Chlamydiosis
D. Vagina candidomicosis
E. Bacterial vaginosis

118. A 18 y.o. woman consulted a gynecologist with complaints of the pain in the lower part of the
abdomen, fever up to 37,50C, considerable mucopurulent discharges from the genital tract,
colic during urinating. After mirror and vagina examination the results are the following: the
urethra is infiltrated, cervix of the uterus is hyperemic, erosive. The uterus is painful, ovaries
are painful, thickened, free. Bacterioscopy test showed diplococcus. What diagnosis is the
most probable?
A. Recent acute ascending gonorrhea*
B. Trichomoniasis
C. Candydomycosis
D. Chronic gonorrhea
E. Chlamydiosis

119. A 30 y.o. male patient complains of itching of the skin which intensifies in the evening. He has
been ill for 1,5 months. On examination: there is rash with paired papules covered with bloody
crusts on the abdomen, hips, buttocks, folds between the fingers, flexor surfaces of the hand.
There are traces of line scratches. What additional investigations are necessary to make
diagnosis?
A. Examination of rash elements scrape*
B. Determination of dermographism
C. Serologic blood examination
D. Blood glucose
E. Examination for helmints

120. A child was delivered severely premature. After the birth the child has RI symptoms, anasarca,
fine bubbling moist rales over the lower lobe of the right lung. Multiple skin extravasations,
bloody foam from the mouth have occured after the 2 day. On chest X-ray: atelectasis of the
lower lobe of the right lung. In blood: Hb-100 g/L, Ht- 0,45. What is the most probable
diagnosis?
A. Edematous-hemorrhagic syndrome*
B. Disseminated intravascular clotting syndrome
C. Pulmonary edema

25
Завантажено з сайту https://тестування.укр/ - онлайн тестування КРОК

D. Hyaline membrane disease


E. Congenital pneumonia

121. A 58 y.o. male patient is examined by a physician and suffers from general weakness, fatigue,
mild pain in the left subcostal area, sometimes frequent painful urination. Moderate
splenomegaly has been revealed. Blood test: neutrophilic leukocytosis with the progress to
myelocyte; basophil- 2%; eosinophil-5%. There is a urate crystales in urine, erythrocyte- 2-3 in
the field of vision. What is the preliminary diagnosis?
A. Chronic myeloleucosis*
B. Leukemoid reaction
C. Lymphogranulomatosis
D. Hepar cirrhosis
E. Urolithiasis

122. A 2 m.o. child was delivered at term with weight 3500 g and was on the mixed feeding.
Current weight is 4900 g. Evaluate the current weight of child.
A. Corresponding to the age*
B. 150 g less than necessary
C. Hypotrophy of I grade
D. Hypotrophy of II grade
E. Paratrophy of I grade

123. A 2 m.o. breast-fed child suffers from cheek skin hyperemia, sporadic papulous elements on
the skin of the chest and back following the apple juice introduction. The child is restless.
What is the initial pediatritian’s tactics?
A. Clarify mother’s diet and exlude obligate allergens*
B. Refer to prescribe dermathologist
C. Administer general ultraviolet irradiation
D. Treat with claritine
E. Apply ointment with corticosteroids to affected skin areas

124. A 43 y.o. woman complains of severe pain in the right abdominal side irradiating in the right
supraclavicular area, fever, dryness and bitterness in the mouth. There were multiple
vomitings without relief. Patient relates the onset of pain to the taking of fat and fried food.
Physical examination: the patient lies on the right side, pale, dry tongue, tachycardia. Right
side of abdomen is painful during palpation and somewhat tense in right hypochondrium. What
is the most likely diagnosis?
A. Perforative ulcer*
B. Acute cholecystitis
C. Acute bowel obstruction
D. Acute appendicitis
E. Right-sided renal colic

125. On the 5-th day of the respiratory disease a 24 y.o. man has developed progressive headaches
systemic dizziness, feeling of seeing double, paresis of mimic muscles on the right, choking
while swallowing. Acute viral encephalitis has been diagnosed. What is the main direction of

26
Завантажено з сайту https://тестування.укр/ - онлайн тестування КРОК

urgent therapy?
A. Zovirax*
B. Glucocorticoids
C. Ceftriaxon
D. Lasix
E. Hemodesis

126. A 65 y.o. woman complains of complicated mouth opening following foot trauma 10 days ago.
Next day she ate with difficulties, there were muscles tension of back, the back of the head
and abdomen. On the third day there was tension of all muscle groups, generalized
convulsions every 10-15 min. What is the most probable diagnosis?
A. Tetanus*
B. Tetania
C. Meningoencephalitis
D. Hemorrhagic stroke
E. Epilepsy

127. A 18 y.o. female student complains of dyspnea during the intensive exertion. The condition
became worse half a year ago. On examination: pulse rate is 88 bpm, accelerated, AP-180/20
mm Hg, pale skin, heart borders are dilated to the left and up. There is systolic-diastolic
murmur in the 2hd intercostal space, S2 at pulmonary artery is accentuated. ECG has revealed
both ventricles hypertrophy. Thoracic X-ray has revealed pulsation and protrusion of the left
ventricle, lung trunk. What doctor’s tactics should be?
A. Cardiosurgeon consultation*
B. Dispensary observation
C. Administration of therapeutic treatment
D. Continuation of investigation
E. Exemption from physical exercises

128. A 35 y.o. woman is suspected of aplastic anemia. The bone marrow puncti-on has been
administered with the diagnostic purpose. What changes in the marrow punctatum are
suggested?
A. Replacement of marrow elements with adipose tissue*
B. Replacement of marrow elements with fibrous tissue
C. Prevalence of megaloblasts
D. Presence of blast cells
E. Absolute lymphocytosis

129. A girl, aged 13, consults the school doctor on account of moderate bloody discharge from the
genital tracts, which appeared 2 days ago. Secondary sexual characters are developed. What is
the most probable cause of bloody discharge?
A. Menarche*
B. Juvenile haemorrhage
C. Haemophilia
D. Endometrium cancer
E. Werlhof’s disease

27
Завантажено з сайту https://тестування.укр/ - онлайн тестування КРОК

130. A 55 y.o. male patient complains of weakness during 2 months, pain in the right side of the
thorax, cough, blood-streaked sputum. On X-ray: intensive triangle shadow in the area of lower
lobe that is connected to mediastinum. What is the most likely disorder in the lungs?
A. Central cancer of lungs*
B. Tuberculosis of lungs
C. Bronchiectasia
D. Pulmonary infarction
E. Pleuropneumonia

131. In a forest summer camp children have variable procedures to harden their organisms. What
procedure has the most hardening power?
A. Contrast shower*
B. Morning exercises on the fresh air
C. Hygienic shower
D. Walking on the fresh air
E. Bath with hydromassage

132. There is a dynamic growth of number of congenital abnormalities such as central paralysis,
newborns blindness, idiocy among the population that lives near to pesticides production
enterprise. Compounds of which pollutant can cause the development of this pathology?
A. Mercury*
B. Strontium
C. Cadmium
D. Iron
E. Chrome

133. A 7 y.o. child had elevation of temperature tol 40° C in anamnesis. For the last 3 months he
presents fusiform swelling of fingers, ankle joints and knee joint, pain in the upper part of the
sternum and cervical part of the spinal column. What is the most probable diagnosis?
A. Juvenile rheumatic arthritis*
B. Rheumatism
C. Toxic synovitis
D. Septic arthritis
E. Osteoarthrits

134. A 5 y.o. child with stigmas of dysembryogenesis (small chin, thick lips, opened mouth,
hyperthelorismus) has systolic murmur in the second intercostal to the right of the sternum.
The murmur passes to the neck and along the sternum left edge. The pulse on the left brachial
artery is weakened. BP on the right arm is 110/60 mm Hg, on the left - 100/60 mm Hg. ECG
results: hypertrophy of the right ventricle. What defect is the most probable?
A. Aortic stenosis*
B. Defect of interventricular septum
C. Defect of interatrial septum
D. Coarctation of the aorta
E. Open aortic duct

28
Завантажено з сайту https://тестування.укр/ - онлайн тестування КРОК

135. At’s planned to construct multifield a new hospital in one of the cental city districts. What
building type is the most appropriate in this case?
A. Centralized and blocked*
B. Centralized
C. Decentralized
D. Mixed
E. Blocked

136. A 60 y.o. patient experiences acute air insufficiency following of the venoectomy due to
subcutaneous vein thrombophlebitis 3 days ago. Skin became cianotic, with grey shade.
Marked psychomotor excitement, tachypnea, substernal pain. What postoperative complication
has occured?
A. Thromboembolia of pulmonary artery*
B. Hemorrhagia
C. Hypostatic pneumonia
D. Myocardial infarction
E. Valvular pneumothorax

137. A 1,5 y.o. child fell ill acutely with high temperature 38°C, headache, fatigue. The temperature
declined on the fifth day, muscular pain in the right leg occured in the morning, there were no
movements and tendon reflexes, sensitivity was reserved. What is the initial diagnosis?
A. Polyomyelitis*
B. Viral encephilitis
C. Polyartropathy
D. Osteomyelitis
E. Hip joint arthritis

138. On observation of sanitary conditions of studying at the technical university it was necessary
to evaluate the visual regimen of students, who study from 9 a.m to 3 p.m. What index of
natural light will be the most informative?
A. Natural light coefficient*
B. Light coefficient
C. Depth of study room
D. Time of the room insolation
E. Presence of mixed (upper-lateral) light

139. A 24 y.o. woman presents with prolonged fever, nocturnal sweating. She’s lost weight for 7 kg
during the last 3 months. She had irregular intercourses. On examination: enlargement of all
lymphaden groups, hepatolienal syndrom. In blood: WBC- 2, 2 * 109/L. What is the most likely
diagnosis?
A. HIV-infection*
B. Lymphogranulomatosis
C. Tuberculosis
D. Infectious mononucleosis
E. Chroniosepsis

29
Завантажено з сайту https://тестування.укр/ - онлайн тестування КРОК

140. A female rheumatic patient experiences diastolic thoracic wall tremor (diastolic thrill),
accentuated Si at apex, there is diastolic murmur with presystolic intensification, opening
snap, S2 accent at pulmonary artery. What rind of heart disorder is observed?
A. Mitral stenosis*
B. Aortic valve insufficiency
C. Pulmonary artery stenosis
D. Mitral valve insufficiency
E. Opened arterial duct

141. A 31 y.o. woman has suffered from systemic sclerodermia for 14 years. She was treated in
hospitals many times. She complains of periodical dull pain in the cardiac area, palpitation,
dyspnea, headache, eyelids swelling, weight loss, pain and limbs deformities. Which organ’s
disorder worsens the prognosis?
A. Kidneys*
B. Heart
C. Lungs
D. Gastro-intestinal tract
E. Skin and joints

142. A 70 y.o. male patient with mild headaches complains of speech disorder, weakness in right
limbs. There was a history of miocardial infarction and arrhythmia. On nu eroligical
examination there are elements of motor aphasia, central paresis of VII and XII cranial nerves
pairs on the right side, cental type of hemiparesis and hemihyperesthisia on the same side.
What is the most probable diagnosis?
A. Ischemic stroke*
B. Hemorrhagic stroke
C. Transitory ischemic attack
D. Epidural hematoma
E. Cerebral tumor

143. A 52 y.o. male patient suffers from squeezing pain attacks in substernal area which irradiates
to the left hand and occurs occasionally and on physical exertion. He has had it for 1 year. On
exami- nation: heart boarders are dilated to the left side, sounds are muffled, pulse- 76 bmp,
rhythmic, AP- 155/80 mm Hg, ECG: the left type, the rest of signs are normal. What additional
examination is necessary to confirm the diagnosis?
A. Bicycle ergometry*
B. Echocardiography
C. Blood lipoproteins
D. General blood count
E. Transaminases of blood

144. A 35 y.o. male patient suffers from chronic glomerulohephritis and has been on hemodialysis
for the last 3 years. He has developed irregularities in the heart activity, hypotension,
progressive weakness, dyspnea. On ECG: bradycardia, 1st degree atrioventicular block, high
sharpened T-waves. Before he had severely disturbed the drinking and diet regimen. What is
the most likely cause of these changes?
A. Hyperkaliemia*

30
Завантажено з сайту https://тестування.укр/ - онлайн тестування КРОК

B. Hyperhydratation
C. Hypokaliemia
D. Hypernatremia
E. Hypocalcemia

145. A 20 daily y.o. female patient is suffering from chronic bronchitis. Recently there has been
production about 0,5 L of purulent sputum with maximum discharge in the morning. Fingers
are like "drum sticks", there are "watching glass"nails. What is the most probable diagnosis?
A. Bronchiectasia*
B. Pneumonia
C. Chronic bronchitis
D. Gangrene of lungs
E. Tuberculosis

146. A 18 y.o. woman complains of weakness, dizziness, loss of appetite, menorrhagia. There are
petechiae on the skin of the upper extremities. Blood test: Hb- 105 g/L; RBC- 3, 2 * 10i2/L;
coloured index- 0,95; thromb.- 20 * 109/L. The sedi-mantation time according to Lee White is
5'; hemorrhagia duration according to Duke is 8', "pinch and tourniquet"test is positive. What
is the most probable diagnosis?
A. Idiopathic thrombocytopenic purpura*
B. Hemophilia
C. Hemorrhagic diathesis
D. Iron deficiency anemia
E. Marchiafava-Micheli’s disease

147. A 30 y.o. primipara has intensive labor pushings with an interval of 1-2 min and of 50 sec
duration. There is a appearing of the fetus head. Perineum is of 4 cm height, has turned pale.
What should be done in this case?
A. Episiotomy*
B. Perineum protection
C. Perineotomy
D. Vacuum extraction of the fetus
E. Observation

148. A 28 y.o. male patient was admitted to the hospital because of high temperature 39° C,
headache, generalized fatigue, constipation, sleep disorder for 9 days. There are sporadic
roseolas on the abdomen, pulse- 78 bpm, liver is enlarged for 2 cm. What is the most probable
diagnosis?
A. Abdominal typhoid*
B. Typhus
C. Sepsis
D. Brucellosis
E. Leptospirosis

149. A 40 h.o. child age has hyperosthesia, CNS depression, dyspepsia. Sepsis is suspected. What
should the differential diagnosis be made with?

31
Завантажено з сайту https://тестування.укр/ - онлайн тестування КРОК

A. Hypoglycemia*
B. Hypocalcemia
C. Hyperbilirubinemia
D. Hyperkaliemia
E. Hypomagnesemia

150. A 20 y.o. patient with bronchial asthma experiences dyspnea attacks 3-4 times a week.
Nocturnal attacks are 1 time a week. FEV1- 50% of necessary figures, during the day it’s
variations is 25 %. What is the severity of bronchial asthma condition?
A. Moderate severity condition*
B. Mild condition
C. Serious condition
D. Asthmatic status
E. Intermittent flow

151. A 40 y.o. man complains of headache in occipital area. On physical examination: the skin is
pale; face and hand edema, BP-170/130 mm Hg. On EchoCG: concentric hypertrophy of the
left ventricle. Ultrasound examination of the kidneys reveals thinned cortical layer. Urine
analysis shows proteinuria of 3,5 g/day. What is the probable diagnosis?
A. Essential arterial hypertension*
B. Chronic pyelonephritis
C. Chronic glomerulonephritis
D. Polycystic disease of the kidneys
E. Cushing’s disease

152. A 28 y.o. primagravida, pregnancy is 15-16 weaks of gestation, presents to the maternity
clinics with dull pain in the lower part of the abdomen and in lumbar area. On vaginal
examination: uterus cervix is 2,5 cm, external isthmus allows to pass the finger tip. Uterus
body is enlarged according to the pregnancy term. Genital discharges are mucous, mild. What
is the diagnosis?
A. Threatened spontaneous abortion*
B. Spontaneous abortion which has begun
C. Stopped pregnancy
D. Hydatid molar pregnancy
E. Placenta presentation

153. A primapara with pelvis size 25-2831-20 cm has active labor activity. Waters poured out, clear.
Fetus weight is 4500 g, the head is engaged to the small pelvis inlet. Vasten’s sign as positive.
Cervix of uterus is fully dilated. Amniotic sac is absent. The fetus heartbeat is clear, rhythmic,
136 bpm. What is the labor tactics?
A. Caesarean section*
B. Vacuum extraction of the fetus
C. Obstetrical forseps
D. Conservative tactics of labor
E. Stimulation of the labor activity

32
Завантажено з сайту https://тестування.укр/ - онлайн тестування КРОК

154. A 41 y.o. man complains of acute pain in the right side of the thorax and sudden increase of
dyspnea following the lifting of heavy object. The patient’s condition is serious: lips and
mucous are cyanotic, breathing rate is 28 per min, pulse-122 bpm., AP- 80/40 mm Hg. There is
tympanitis on percussion and weakened breathing on auscultaion on the right. S2 is
accentuated over pulmonary artery. What is the urgent measure on the prehospital stage?
A. Air aspiration from the pleural cavity*
B. Epinephrine introduction
C. Euphilline introduction
D. Call for cardiologic team
E. Oxygen inhalation

155. A 6 y.o child complains of thirst, polyuria, increased appetite for 2 months with weight loss for
3 kg. There has been nocturnal enuresis during last week. On examination: hyperglycemia 14
mol/L. The diagnosis is diabetis mellitus I type. What is the genesis of this disease?
A. Autoimmune*
B. Viral
C. Bacterial
D. Neurogenic
E. Virus-bacterial

156. A 74 y.o. female patient complains of pain, distended abdomen, nausea. She suffers from heart
ichemia, postinfarction and diffusive cardiosclerosis. On examination: grave condition,
distended abdomen, abdominal wall fails to take active part in breathing. On laparoscopy:
some cloudy effusion, one of the bowel loops is dark-blue. What is the most probable
diagnosis?
A. Mesenterial vessels thrombosis*
B. Volvulus
C. Acute intestinal obstruction
D. Ichemic abdominal syndrome
E. Erysipelas

157. A 56 y.o. man, who has taken alcoholic drinks regularly for 20 years, complains of intensive
girdle pain in the abdomen. Profuse nonformed stool 23- times a day has appeared for the last
2 years, loss of weight for 8 kg for 2 years. On examination: abdomen is soft, painless. Blood
amylase - 12g/L. Feces examination-neutral fat 15 g per day, starch grains. What is the most
reasonable treatment at this stage?
A. Pancreatine*
B. Contrykal
C. Aminocapron acid
D. Levomicytine
E. Imodium

158. A 30 y.o. woman has second labor which lasts for 14 hours. The fetus heartbeat is mufflet,
arrhythmic, 100 bpm. On vaginal examination: complete cervix dilatation, fetus head is in the
area of small pelvis outlet. Sagital suture is in the direct size. The small fontanelle is at the
symphis. What is the further tactics of the labor?
A. Application of obstetrical forceps*

33
Завантажено з сайту https://тестування.укр/ - онлайн тестування КРОК

B. Stimulation of the labor activity with oxitocyne


C. Ceasarian section
D. Application of craniodermal forceps by Ivanov’s
E. Application of obstetrical cavity forceps

159. A 31y.o. patient has had mental disorder for a long time. He suffers from insomnia for a long
time. He has developed fears, suicidal thoughts, tried to hang himself. His mood is depressed,
he refuses from treatment. What measures are the most expedient for the prevention of
suicide?
A. Admission to the mental hospital*
B. Admission to the nu erological department
C. Out-patient treatment
D. Psychotherapeutic conversation
E. Strict home supervision

160. A 43 y.o. woman complains of shooting heart pain, dyspnea, irregularities in the heart activity,
progressive fatigue during 3 weeks. She had acute respiratory disease a month ago. On
examination: AP- 120/80 mm Hg, heart rate 98 bpm, heart boarders +1,5 cm left side, sounds
are muffled, soft systolic murmur at apex and Botkin’s area; sporadic extrasystoles. Liver isn’t
palpated, there are no edema. Blood test: wBc- 6, 7* 109/L, sedimentation rate- 21 mm/hour.
What is the most probable diagnosis?
A. Acute myocarditis*
B. Climacteric myocardiodystrophia
C. Ichemic heart disease, angina pectoris
D. Rheumatism, mitral insufficiency
E. Hypertrophic cardiomyopathy

161. A 52 y.o. male patient has become ill gradually. There is pain in the left side of the thorax
during 2 weeks, elevation of temperature till 38 - 39° C. On examination: left chest side falls
behind in breathing movement no voice tremor over the left lung. Dullness that is more
intensive in lower parts of this lung. Right heart border is deviated outside. Sharply weakened
breathing over the left lung, no rales. Heart sounds are mufflet, tachycardia. What is the most
probable diagnosis?
A. Exudative pleuritis*
B. Spotaneous pneumothorax
C. Atelectasis of lung
D. Cirrhotic tuberculosis
E. Infarction-pneumonia

162. A mother of a newborn child suffers from chronic pyelonephritis. She had acute respiratory
viral disease before the labor. Labor in time, with prolonged period without waters. A child
had erythematous eruption on the 2 day, then there were seropurulent vesicles about 1cm.
Nikolsky’s symptom is positive. Erosions have occured after vesicle rupture. The child is
flabby. The temperature is subfebrile. What is the most probable diagnosis?
A. Newborn pemphigus*
B. Vesiculopustulosis
C. Pseudofurunculosis

34
Завантажено з сайту https://тестування.укр/ - онлайн тестування КРОК

D. Sepsis
E. Ritter’s dermatitis

163. A child was born at 34 weeks of gestation in bad condition. The cardinal symptoms show
respiratoty disorders: sound prolonged expiration, additional muscles taking part in breathing,
crepitation rales on the background of the rough breath sounds. Assesment according to
Silverman’s scale was 0, in 3 hours- 6 with presence of clinical data. What diagnostic method
can determine pneumopathy’s type in the child?
A. Chest X-ray*
B. Blood test
C. Blood gases
D. Proteinogram
E. Immunologic investigation

164. During intramuscular DTP vaccination in clinic, a 3 m.o. child developed signs of
laryngospasm, paleness of skin, cyanosis of lips, "cock cry", stop of respiration, tension of the
whole body with overturned backward head. Allergologi-cal history of the child is not
complicated. What is the most probable diagnosis?
A. Spasmophilia, tonic spasms*
B. Anaphylactic shock, clonic spasms
C. Meningoencephalitic reaction, clonic and tonic spasms
D. Cerebral haemorrhage, tonic spasms
E. Meningism, clonic and tonic spasms

165. A 60 y.o. man complains of significant pain in the right eye, photophobia, lacrimation, reduced
vision of this eye, headache of the right part of the head. Pain occured 2 days ago. On
examination: Vis OD- 0,03, congested injection of the eye ball, significant cornea edema, front
chamber is deep, pupil is narrow, athrophic iris, there is optic nerve excavation on the eye
fundus, intraocular pressure- 38 mm Hg. Vis OS-0,8 unadjustable. The eye is calm, healthy.
Intraoccular pressure- 22 mm Hg. What is the most probable diagnosis?
A. Acute glaucoma attack*
B. Right eye’s uveitis
C. Right eye’s keratitis
D. Eye nerve’s neuritis
E. Maculodystrophy

166. A 41 y.o. woman has suffered from nonspecific ulcerative colitis during 5 years. On
rectoromanoscopy: marked inflammatory process of lower intestinal parts, pseudopolyposive
changes of mucous. In blood: WbC- 9,8 * 109/L, RBC- 3,0 * 1012/L, sedimentation rate-52
mm/hour. What medication provides pathogenetic treatment of this patient?
A. Sulfasalasine*
B. Motilium
C. Vikasolum
D. Linex
E. Kreon

35
Завантажено з сайту https://тестування.укр/ - онлайн тестування КРОК

167. A 49 y.o. female patient presents with acute attacks of headache associated with pulsation in
temples, increasing AP to 280/140 mm Hg. Pheochromocytoma is suspected. What is the
mechanism of hypertensive crisis in this patient?
A. Increasing of catecholamines concentration*
B. Increasing of aldosterone level in blood
C. Increasing of plasma renin activity
D. Increasing of vasopressin excretion
E. Increasing of thyroxine excretion

168. To replace the blood loss replacement 1000 ml of the same group of Rhesus-compatible
donated blood was transfused to the patient. The blood was conserved by sodium citrate. At
the end of hemotransfusion there appeared excitement, pale skin, tachycardia, muscles
cramps in the patient. What complication should be suspected?
A. Citrate intoxication*
B. Citrate shock
C. Allergic reaction
D. Anaphylactic shock
E. Pyrogenous reaction

169. A 20 y.o. patient suddely felt ill 12 hours ago. There was pain in epigactric area, nausea,
sporadic vomiting. He had taken alcohol before. In few hours the pain localized in the right
iliac area. On examination: positive rebound tenderness symptoms. WBC-12, 2 * 109/L. What is
the most probable diagnosis?
A. Acute appendicitis*
B. Acute pancreatitis
C. Perforated ulcer
D. Rightside kidney colic
E. Acute cholecystitis

170. A patient, aged 58, was fishing in the winter. On return home after some time felt some pain in
the feet. Consulted a doctor. On examination: feet skin was pale, then after rewarming became
red, warm to the touch. Edema is not significant, limited to the toes. All types of sensitivity are
preserved. No blisters. What degree of frostbite is observed?
A. I degree*
B. II degree
C. III degree
D. IV degree
E. V degree

171. A 24 y.o. emotionally-labile woman presents with irritation, depressed mood, palpitation,
shooting pain in the heart area, generalized fatigue following the divorce. On examination:
palm hyperhydrosis, pulse rate- 72-78 bpm, labile, heart without changes. ECG is normal.
What is the most probable pathology in this case?
A. Neurasthenia*
B. Ipochondric neurosis
C. Compulsive neurosis
D. Schizophrenia

36
Завантажено з сайту https://тестування.укр/ - онлайн тестування КРОК

E. Depressive neurosis

172. A 98 y.o. male patient complains of pain in the left lower limb which intensifies on walking,
feeling of cold and numbness in both feet. He has been ill for 6 years. On examination: pale dry
skin, hyperkeratosis. Hairy covering is poorly developed on the left shin. "Furrow symptom "is
positive on the left. Pulse on foot arteries and popliteal artery isn’t palpated, on the femoral
artery it’s weak. On the right limb the artery pulsation is reserved. What is the most probable
diagnosis?
A. Arteriosclerosis obliterans*
B. Obliterating endarteritis
C. Hemoral arthery thombosis
D. Raynauld’s disease
E. Buerger’s disease (thromboangiitis obliterans)

173. A patient had macrofocal myocardial infarction. He is overweight for 36%, AP is 150/90 mm
Hg, blood sugar- 5,9 mmol/L, general cholesterol- 4,9 mmol/L, uric acid-0,211 mmol/L. Which
risk factor should be urgently eradicated during the secondary prevention?
A. Obesity*
B. Arterial hypertension
C. Hyperglycemia
D. Hypercholesterolemia
E. Hyperuricemia

174. A 2 y.o. boy was admitted to the hospital with weight loss, unstable discharges, anorexia,
following the semolina’s introduction (since 5 months). The child is adymanic, flabby, pale dry
skin, subcutaneous layer is emaciated. Distended and tensed abdomen, tympanitis on
percussion of the upper part of the abdomen, splashing sounds, feces are foamy, of light color,
foul. On coprocytogram: a lot of neutral fat. What is the cause of the disease?
A. Celiakia (celiac disease)*
B. Mucoviscidosis (cystic fibrosis)
C. Intestinal dysbacteriosis
D. Chronic enteritis
E. Disaccharidase insufficiency

175. On medical observation a doctor identified girl (162 cm tall and 59 kg weight) who complained
loss of ability to see surrounding objects clearly in the evening. On examination: dry skin,
hyperkeratosis. Her daily ration includes the following vi-tamines: vitamine A- 0,5 mg, vit.Bi-
2,0 mg, vit.B2- 2,5 mg, vit.B6- 2 mg, vit.C- 70 mg. What is the hypovitaminosis type?
A. A-hypovitaminosis*
B. Bv-hypovitaminosis
C. B2-hypovitaminosis
D. B6-hypovitaminosis
E. C-hypovitaminosis

176. A woman in labor, on vaginal inspection: cervix dilation is up to 2 cm, fetal bladder is intact.
Sacral cavity is free, sacral promontory is reachable only with a bent finger, the inner surface

37
Завантажено з сайту https://тестування.укр/ - онлайн тестування КРОК

of the sacrococcygeal joint is accessible for inspection. The head of the fetus presents. Sagittal
suture occupies the transverse diameter of pelvic inlet, the small fontanel to the left, on the
side. What labor stage is this?
A. Cervix dilation stage*
B. Preliminary stage
C. Prodromal stage
D. Expulsion of fetus stage
E. Placental stage

177. A 30 y.o. patient had deep burn covering 30% of body 30 days ago. Now he presents with
continued fever, loss of appetite, night sweats. Burned surface weakly granulates. What is the
stage of burn disease?
A. Septicotoxemia*
B. Primary burn shock
C. Secondary burn shock
D. Acute burn toxemia
E. Convalescence

178. A 45 y.o. woman complains of contact bleedings during 5 months. On speculum examination:
hyperemia of uterus cervix, looks like cauliflower, bleeds on probing. On bimanual
examination: cervix is of densed consistensy, uterus body isn’t enlarged, mobile, nonpalpable
adnexa, parametrium is free, deep fornixes. What is the most likely diagnosis?
A. Cancer of cervix of uterus*
B. Cancer of body of uterus
C. Fibromatous node which is being born
D. Cervical pregnancy
E. Polypose of cervix of uterus

179. A female, aged 20, after smoking notices a peculiar inebriation with the feeling of burst of
energy, elation, irreality and changing of surroundings: the world gets full of bright colours,
the objects change their dimensions, people’s faces get cartoon features, loss of time and
space judgement. What is the most likely diagnosis?
A. Cocainism*
B. Morphinism
C. Barbiturism
D. Nicotinism
E. Cannabism

180. A 75 y.o patient can not tell the month, date and season of the year. After long deliberations
she manages to tellher name. She is in irritable and dissatisfied mood. She always carries a
bundle with belongings with her, hides a parcel with bread, shoes in her underwear in her
bosom as well as "invaluable books". What is the most probable diagnosis?
A. Senile dementia*
B. Atherosclerotic (lacunar) dementia
C. Presenile melancholia
D. Behaviour disorder
E. Dissociated personality (psychopathy)

38
Завантажено з сайту https://тестування.укр/ - онлайн тестування КРОК

181. A 29 y.o. patient was admitted to the hospital with acute girdle pain in epigastric area,
vomiting in 1 hour after the meal. On examination: pale, acrocyanosis. Breathing is frequent,
shallow. Abdominal muscles are tensed, positive Schotkin-Blumberg’s symptom. What is the
maximal term to make a diagnosis?
A. In 2 hours*
B. In 0,5 hours
C. In 1 hour
D. In 3 hours
E. In 6 hours

182. A 33 y.o. patient was admitted to the hospital with stopped recurrent peptic ulcer bleeding. On
examination he is exhausted, pale. Hb- 77 g/L, Ht- 0,25. Due to anemia there were two
attempts of blood transfution of identical blood group A(II)Rh+. Both attempts were stopped
because of anaphylactic reaction. What blood transfution environment is desirable in this
case?
A. Washed erythrocytes*
B. Freshcitrated blood
C. Erythrocyte mass (native)
D. Erythrocyte emulsion
E. Erythrocyte mass poor for leucocytes and thrombocytes

183. A 19 y.o. boy was admitted to the hospital with closed abdominal trauma. On operation
multiple ruptures of spleen and small intestine were revealed. AP is falling, it is necessary to
perform hemotransfusion. Who can determine patient’s blood group and rhesus compatibility?
A. A doctor of any speciality*
B. A laboratory physician
C. A surgeon
D. A traumotologist
E. An anaesthesilogist

184. A 27 y.o. woman suffers from pyelonephritits of the only kidney. She presents to the maternity
welfare centre because of suppresion of menses for 2,5 months. On examination pregnancy 11
weeks of gestation was revealed. In urine: albumine 3,3 g/L, leucocytes cover the field of
vision. What is doctor’s tactics in this case?
A. Immediate pregancy interruption*
B. Pregnancy interruption after urine normalization
C. Maintenance of pregnancy till 36 weeks
D. Pregnancy interruption at 24-25 weeks
E. Maintenance of pregnancy till delivery term

185. A 35 y.o. female patient was admitted to the surgical department with symptoms of ulcerative
gastric hemorrhage. It’s been the third hemorrhage for the last 2 years. After conservative
treatment vomiting with blood stopped, hemoglobin elevated from 60 till 108 g/L. General
condition became better. But profuse vomiting with blood reoccured in 2-3- hours. Hemoglobin
decreased to 93,1 g/L then to 58,1 g/L. What is the tactics of treatment?
A. Urgent surgery*
B. Deferred surgery

39
Завантажено з сайту https://тестування.укр/ - онлайн тестування КРОК

C. Conservative treatment
D. Conservative treatment with following surgery
E. Taylor’s treatment

186. A victim of a road accident, aged 44, is operated on account of intraperitoneal haemorrhage.
In which case can the patient’s blood from the abdominal cavity be used for autotransfusion?
A. Stomach rupture*
B. Bladder rupture
C. Liver rupture
D. Splenic rupture
E. Small intestines rupture

187. A man, aged 30, complains of intense pain, reddening of skin, edema in the ankle-joint area,
fever up to 390C. Sudden onset of the illness. In the past there were similar attacks lasting 5-6
days without residual changes in the joint. The skin over the joint is hyperemic without definite
borders and without infiltrative bank on the periphery. What is the most likely diagnosis?
A. Gout*
B. Infectional arthritis
C. Rheumatoid arthritis
D. Erysipelatous inflammation
E. Osteoarthritis

188. A patient, aged 25, suffering from stomach ulcer. Had a course of treatment in the
gastroenterological unit. 2 weeks later developed constant pain, increasing and resistant to
medication. The abdomen is painful in epigastric area, moderate defence in pyloroduodenal
area. Which complication development aggravated the patient’s state?
A. Malignisation*
B. Penetration
C. Perforation
D. Haemorrhage
E. Stenosis

189. A 54 y.o. male patient suffers from dyspnea during mild physical exertion, cough with sputum
which is excreted with diffculty. On examination: diffuse cyanosis. Is Barrel-chest. Weakened
vesicular breathing with prolonged expiration and dry whistling rales. Ap is 140/80 mm Hg,
pulse is 92 bpm, rhythmic. Spi-rography: vital capacity (vC)/predicted vital capacity- 65%,
FEV1/FVC- 50%. Determine the type of respiratory insufficiency (RI).
A. RI of mixed type with prevailing obstruction*
B. RI of restrictive type
C. RI of obstructive type
D. RI of mixed type with prevailing resri-ction
E. There is no RI

190. A patient aged 18 with a cranial injury was in comatose state during several hours. In post-
comatose period gets tired quickly, non-productive in dialog - in the beginning answers 2-3
questions, then gets tired and can not understand the point of the question. Which

40
Завантажено з сайту https://тестування.укр/ - онлайн тестування КРОК

psychotropic should be given to the patient to prevent psychoorganic syndrome?


A. Nootropics*
B. Neuroleptics
C. Stimulators
D. Tranquillisers
E. Antidepressants

191. A 25 y.o. patient was admitted with chest trauma. Clinical and X-ray examination have
revealed tense pneumothorax on the left. What emergency treatment should be undertaken?
A. Pleural cavity drainage*
B. Intravenous infusions
C. Oxigenotherapy
D. Intubation
E. Analgetics

192. A 38 y.o. patient complains of pain in lumbar part of spinal column with irradiation to the back
surface of the left leg following the lifting of a heavy object. Pain is increasing on change of the
body position and in vertical position. positive stretching symptoms were revealed on
examination. What is an initial diagnosis?
A. Intervertebral ligaments disorder*
B. Spinal cord tumor
C. Arachnomielitis
D. Polyneuritis
E. Myelopathy

193. A child is being discharged from the surgical department after conservative treatment of
invagination. What recommendations should doctor give to mother to prevent this disease
recurrence?
A. Strict following of feeding regimen*
B. Common cold prophilaxis
C. Feces observation
D. Gastro-intestinal disease prevention
E. Hardening of the child

194. A male patient presents with swollen ankles, face, eyelids, elevated AP-160/100 mm Hg, pulse-
54 bpm, daily loss of albumine with urine- 4g. What therapy is pathogenetic in this case?
A. Corticosteroids*
B. Diuretics
C. NSAID
D. Calcium antagonists
E. Antibiotics

195. During dynamic investigation of a patient the increase of central venous pressure is combined
with the decrease of arterial pressure. What process is proved by such combination?
A. Increase of bleeding speed*
B. Developing of cardiac insufficiency

41
Завантажено з сайту https://тестування.укр/ - онлайн тестування КРОК

C. Shunting
D. Depositing of blood in venous channel
E. Presence of hypervolemia

196. A male patient complains of heartburn which gest stronger while bending the body, substernal
pain during swallowing. There is a hiatus hernia on X-ray. What disoeder should be expected
at gastroscopy?
A. Gastroesophageal reflux*
B. Chronic gastritis
C. Gastric peptic ulcer
D. Acute erosive gastritis
E. Duodenal peptic ulcer

197. A 2,5 m.o. child presents with muscle hypotonia, sweating, alopecia of the back of the head.
The child is prescribed massage, curative gymnastics and vitamin What is the dosage and
frequency of vitamin D administration?
A. 3000 IU daily*
B. 500 IU daily
C. 1000 IU daily
D. 500 IU every other day
E. 1000 IU every other day

198. A 43 y.o. male complains of stomach pain, which relieves with defecation, and is accompanied
by abdominal winds, rumbling, the feeling of incomplete evacuation or urgent need for bowel
movement, constipation or diarrhea in alternation. These symptoms have lasted for over 3
months. No changes in laboratory tests. What is the most likely diagnosis?
A. Irritable bowel syndrome*
B. Spastic colitis
C. Colitis with hypertonic type dyskinesia
D. Chronic enterocolitis, exacerbation phase
E. Atonic colitis

199. After delivery and revision of placenta there was found the defect of placental lobe. General
condition of woman is normal, uterine is firm, there is moderate bloody discharge. Inspection
of birth canal with mirrors shows absence of lacerations. What is the following necessary
action?
A. Manual exploration of the uterine cavity*
B. External massage of uterus
C. Use of uterine contracting agents
D. Urine drainage, cold at lower abdomen
E. Use of hemostatic medications

200. A patient, aged 81, complains of constant urinary excretion in drops, feeling of fullness in the
lower abdomen. On examination: above pubis there is a spherical protrusion, over which there
is a dullness of percussion sound, positive suprapubic punch. What symptom is observed in
this patient?

42
Завантажено з сайту https://тестування.укр/ - онлайн тестування КРОК

A. Paradoxal ischuria*
B. Urinary incontinence
C. Dysuria
D. Enuresis
E. Pollakiuria

43
Завантажено з сайту https://тестування.укр/ - онлайн тестування КРОК

ТЕСТУВАННЯ.УКР Бази тестів

Буклет 2006 року

Цей тест можна пройти в режимі онлайн тестування на сайті https://тестування.укр/testkrok/studing/567

Це офіційні тести з сайту Центру тестування https://www.testcentr.org.ua/

1. A 47 y.o. woman complains of having paroxysmal headaches for the last 5 years. The pain is
one-sided, intense, localised in frontal region of head, accompanied by nausea and stomach
discomfort, begins one of a sudden. Onset is usually preceded by vision reduction. Anamnesis
gives evidence of periodical AP rise, but at the moment the woman doesn’t take any medicines.
Inbetween the onsets of headache her state is satisfactory. Objectively: high-calorie diet (body
weight index - 29), AP- 170/95 mm Hg. Neurologic state has no pecularities. What is the most
probable diagnosis?
A. Migraine*
B. Chronic subdural hematoma
C. Epilepsy
D. Benign intracranial hypertension
E. Hypertensive encephalopathy

2. A 7 y.o. boy suddenly felt pain in his right knee, it became edematic. The day before he took
part in a crosscountry race. Family anamnesis has no data about hemophilia and bleeding
sickness. Objectively: body temperature is 37,50C. The knee is painful, hot to the touch,
edematic with local tissue tension over it. Blood count: Hb- 123 g/L, leukocytes - 5,6 * 103 * * *
* * 9/L, thrombocytes -354 * 109/L, prothrombin time -12 seconds (normally 10-15 seconds),
partly activated thromboplastin time - 72 seconds (normally 35-45 seconds). Hemorrhage time
is normal, VIII:C factor is 5% of norm. What is the most probable diagnosis?
A. Hemophilia A*
B. Hemophilia B
C. Schoenlein-Henoch disease
D. Vitamin K deficiency
E. Thrombocytopenia

3. On the 3rd day after the acute anterior myocardial infarction a 55 y.o. patient complains of
dull ache behind his breast bone, that can be reduced by bendi- ng forward, and of dyspnea.
Objectively: AP- 140/180 mm Hg, heart sounds are dull. ECG results: atrial fibrillation wi- th
frequence of ventricular contractions at the rate of 110/min, pathological Q wave and S-T
segment raising in the ri- ght chest leads. The patient refused from thrombolisis. What is the
most probable diagnosis?
A. Acute pericarditis*
B. Pulmonary embolism
C. Tietze’s syndrome
D. Dissecting aortic aneurysm
E. Dressler’s syndrome

4. A 54 y.o. man was admitted to the hospital with complaints of sudden intense headache in
occipital region and vomiting. In the medical hystory: moderate arterial hypertension, the
patient was taking hydrochlorothiazide. Three days ago he consulted a therapeutist about

1
Завантажено з сайту https://тестування.укр/ - онлайн тестування КРОК

intense headache that was suppressed by an analgetic. Objectively: consciousness is confused,


left pupil is mydriatic. Evident photophobia and tension of neck muscles. Left-side hemiparesis
with increased muscle tonus and reflexes. Body temperature is low, rash is absent. AP-230/130
mm Hg, Ps- 50 bpm, BR- 12/min. What is your preliminary diagnosis?
A. Acute subdural hematoma*
B. Myasthenia
C. Disseminated sclerosis
D. Migraine
E. Acute bacterial meningitis

5. A 76 y.o. woman complains of progressing swallowing disorder, mostly she has had problems
with solid food for the last 6 weeks. Sometimes she has regurgitation of solid masses.
Swallowing is not painful. She lost 6 kg. 10 years ago she had myocardiac infarction, she takes
constantly aspirine and prolonged nitrates. She consumes alcochol in moderate proportions,
smokes. Objectively: icteric skin, neck has no pecularities, lymph nodes are not enlarged.
Thorax has no changes, cardiovascular system has no evident changes. Liver is +3 cm. What is
the preliminary diagnosis?
A. Cancer of esophagus*
B. Diaphragmatic hernia
C. Diffuse constriction of esophagus
D. Myasthenia
E. Esophageal achalasia

6. A 38 y.o. man complains of having occasional problems with swallowing of both hard and fluid
food for many months. Sometimes he feels intense pain behind his breast bone, epecially after
hot drinks. There are asphyxia onsets at night. He has not put off weight. Objectively: his
general condition is satisfactory, skin is of usual colour. Examination revealed no changes of
gastrointestinal tract. X-ray picture of thorax organs presents esophagus dilatation with level
of fluid in it. What is the preliminary diagnosis?
A. Esophagus achalasia*
B. Myastenia
C. Cancer of esophagus
D. Esophagus candidosis
E. Gastroesophageal reflux

7. A woman is admitted to the maternity hospital with stopped birth activity and mild bloody
discharges from the vagina. The condition is serious, the skin is pale, consciousness is
confused. AP- 80/40 mm Hg. The palpitation of the fetus is not determined. In medical hystory
there was a Cesarean section a year ago. Make a diagnosis:
A. Hysterorrhesis*
B. Presentation of the cord
C. Placental presentation
D. Abjointing of the mucous fuse from cervix of the uterus
E. Premature expultion of the amniotic waters

8. A 35 y.o. woman consulted a doctor about occasional pains in paraumbilical and iliac region
that reduce after defecation or passage of gases. Defecation takes place up to 6 times a day,

2
Завантажено з сайту https://тестування.укр/ - онлайн тестування КРОК

stool is not solid, with some mucus in it. Appetite is normal, she has not put off weight. First
such symptoms appeared 1,5 year ago, but colonoscopy data reveals no organic changes.
Objectively: abdomen is soft, a little bit painful in the left iliac region. Blood and urine are
normal. What is the preliminary diagnosis?
A. Irritable bowels syndrome*
B. Celiac disease
C. Crohn’s disease
D. Pseudomembranous colitis
E. Dispancreatism

9. A 60 y.o. man complains of sense of heaviness in the region of scrotum. Objectively: scrotum
edema in the left part. Testicle is of normal size, but there is a soft, scrotum limited edema
over it that can be pressed and disappears when the patient lies down. What is the preliminary
diagnosis?
A. Varicocele*
B. Inguinal lymphadenopathy
C. Ectopic testicle
D. Inguinal hernia
E. Varicosity of subcutaneous veins 10

10. A 26 y.o. woman complains of sudden pains in the bottom of abdomen irradiating to the anus,
nausea, giddiness, bloody dark discharges from sexual tracts for one week, the delay of
menses for 4 weeks. Signs of the peritoneum irritation are positive. Bimanual examination:
borders of the uterus body and its appendages are not determined because of sharp
painfullness. The diverticulum and painfullness of the back and dextral fornixes of the vagina
are evident. What is the most probable diagnosis?
A. Broken tubal pregnancy*
B. Apoplexy of the ovary
C. Acute right-side adnexitis
D. Torsion of the crus of the ovary tumour
E. Acute appendicitis

11. Name a statistical observation unit for determination of blood sugar impact on the healing of
wound’s surface in a postoperative period:
A. The patient in a postoperative period*
B. An amount of blood sugar
C. Blood analysis
D. The patient who has a wound surface
E. The patient who was discharged on an after-care

12. Choose a method of graphic representation of monthly information about the number of
registered cases of acute intestinal infection and their comparison to the average monthly
values, obtained for 5 last years:
A. The linear diagram*
B. The radial diagram
C. The sector diagram
D. The figured diagram

3
Завантажено з сайту https://тестування.укр/ - онлайн тестування КРОК

E. Curvilinear diagram

13. A patient, aged 25, complains of pain in the I finger on the right hand. On examination: the
finger is homogeneously hydropic, in bent position. On attempt to unbend the finger the pain
is getting worse. Acute pain appears during the probe in ligament projection. What decease is
the most likely?
A. Thecal whitlow (ligament panaritium)*
B. Subcutaneous panaritium
C. Articular (joint) panaritium
D. Bone panaritium
E. Paronychia

14. An employee has been invalid for 6 months as a result of a hip fracture. Who has the right to
authorize the issue of the medical sick-list for the last 2 months?
A. MSEC*
B. Head physician of the polyclinic
C. DCC
D. DCC together with the head physician of a polyclinic
E. Deputy head physician on working capacity

15. An employee had an abortion by medical indications on the 6.03.2001 and she stayed in a
hospital till 1703.2001. What term is the medical sick-list issued for?
A. For 12 days*
B. For 3 days
C. For 4 days
D. For 10 days
E. For 11 days

16. A 5 tons milk batch was sampled. The lab analysis revealed: fat content 2%, specific density
-1,04 g/cm3, acidity - 21°T, reductase probe - weak-positive. What way is the product to be
used in?
A. Sell but inform customers about milk quality*
B. Discard for animal feeding
C. Technical utilization
D. Sell without limitations
E. Do the product away

17. Patient with thyreotoxicosis is in the 2 beds hospital ward of therapeutic department. The area
of the ward is 18 m2, height 3 m, ventilation rate 2,5/hr. Air temperature - 20° C, relative
humidity -45%, air movement velocity - 0,3 m/s, light coefficient - 1/5, noise level - 30 dB. Do
hygienic evaluation of the conditions meet the standards?
A. Discomfortable microclimate*
B. Non-effective ventilation
C. Poor lighting
D. High level of noise
E. All conditions meet the requirements

4
Завантажено з сайту https://тестування.укр/ - онлайн тестування КРОК

18. A 33 y.o. patient, works as a secretary. Her diet contains 150 g of protein (including 100 g of
animal protein), 200 g of fat, 600 g of carbohydrates. What pathology can result from this diet?
A. Obesity*
B. Schizophrenia
C. Paradontosis
D. Common cold
E. Uterine fibromyoma

19. A 15 year old adolescent was taken to the hospital with complaints of poor night vision.
Objectively: increased darkness adaptation time, Bitot’s spots on conjucti-va. The patient’s skin
is dry, scales off, folliculitis signs of the face skin are present. What is the cause of this
disease?
A. Retinole deficit*
B. Thiamine deficit
C. Biotin deficit
D. Folic acid deficit
E. Napthtochynones deficit

20. What guarantees against the preconceived attitude to the physician in cases of professional
law violations do you know?
A. Sanction of public prosecutor, inquiry by preliminary investigator of prosecutor’s office,
committee of experts*
B. Draw up a statement about forensic medical examination
C. Conduct an inquiry by preliminary investigator of police department
D. Utilisation copy of medical documents
E. Conduct forensic medical examination by district forensic medicine expert

21. A 23 y.o. patient has a gunshot wound of his left arm.The bones of the arm are not damaged.
What appropriate surgical care must be provided to the patient?
A. Primary surgical processing with a flowing suction*
B. Wound saturing
C. Wound saturing and drying
D. Wound drying with towel gauzes
E. Aseptic dressing of the wound

22. A 34 y.o. patient 3 hours ago was bitten by a dog. He has got a non-bleeding wound in his left
arm caused by the dog’s bite. What surgical care would you provide to the patient?
A. Wound bathing with detergent water and antiseptic application*
B. Aseptic bandage
C. Cream bandage
D. Complete suturing of the wound
E. Incomplete suturing of the wound

23. A 37 y.o. patient complains of pain in the right arm which increases during motion, raised body
temperature up to 39°C. In the right cubital fossa there is a trace of injection, hyperemia and
thickening along the vein. Your diagnosis?

5
Завантажено з сайту https://тестування.укр/ - онлайн тестування КРОК

A. Phlebit*
B. Phlegmon
C. Abscess
D. Inflammation of lymph
E. Erysipelas

24. A 35 y.o. woman was admitted to thoracic surgery department with fever up to 40° C, onset of
pain in the side caused by deep breathing, cough with considerable quantity of purulent
sputum and blood with bad smell. What is the most likely diagnosis?
A. Abscess of the lung*
B. Complication of liver echinococcosis
C. Bronchiectatic disease
D. Actinomycosis of lungs
E. Pulmonary tuberculosis

25. A laboratory obtained a milk sample sent for analysis. Analysis gave the following data: color -
whitish, smell - has no pecularities, taste - typical for milk, density - 1,038, acidity - Turner’s
35°, fat - 3,2%. What is the quality level of this milk?
A. The milk is of poor quality*
B. The milk is of high quality
C. The milk is of reduced quality
D. The milk is falsificated
E. The milk is nominally qualified

26. For the persons who live in a hot area after an accident at a nuclear object, the greatest risk
within the first decade is represented by cancer of:
A. Thyroid gland*
B. Skin
C. Reproduction system organs
D. Breast
E. Lungs

27. A 40 y.o. patient was diagnosed: 1. Medular thyroid gland cancer. 2. Feochromocytoma. What
operation should be performed at first?
A. Operation on account of feochromocytoma*
B. Operation on thyroid gland
C. Krail’s operation
D. Subtotal resection of thyroid gland and fascicular resection of limphatic nodes
E. Vanach’s operation

28. A 28 y.o. woman consulted a doctor with the complaints of enlargening in size of an inborn
pigment nevus, it was also wetting and itching. What test should not be used for diagnostics in
this case?
A. Incision biopsy*
B. Yaks’s reaction
C. Radioisotope diagnostics

6
Завантажено з сайту https://тестування.укр/ - онлайн тестування КРОК

D. Termography
E. Glass-print

29. A 74 y.o. patient has been ill with benign prostate hyperplasy for the last 5 years. 4 days ago,
after alcochol consumption, there was an acute retention of urination. At the pre-admission
stage his urinary bladder was catheterized with metallic catheter. Examination revealed: right
epididymis is enlarged, thick and painful, there are purulent discharges from urethra. What
way of emergency care must be chosen?
A. Trocar or open epicystostomy*
B. Transuretral resection or prostatectomy
C. Introduction of permanent urethral catheter
D. Microwave thermotherapy of prostate
E. Placing of intraprostatic stent

30. A 10 y.o. boy was ill with angina 2 weeks ago, has complaints of joint pain and stiffness of his
left knee and right elbow. There was fever (38,5°) and ankle disfunction, enlargement of
cardiac dullness by 2 cm, tachycardia, weakness of the 1st sound, gallop rhythm, weak systolic
murmur near apex. What diagnosis corresponds with such symptoms?
A. Acute rheumatic fever*
B. Systemic lupus erythematosis
C. Juvenile rheumatoid arthritis
D. Reiter’s disease
E. Reactive arthritis

31. A baby boy was born in time, it was his mother’s 1st pregnancy. The jaundice was revealed on
the 2nd day of life, then it progressed. The adynamia, vomiting and hepatomegaly were
presented. The indirect bilirubin level was 275 mcmol/L, the direct bilirubin level - 5 mcmol/L,
Hb-150 g/L. Mother’s blood group - 0(I),Rh+, child’s blood group - A(II),Rh+. Make a
diagnosis.
A. Hemolytic disease of newborn (ABO incompatibility), icteric type*
B. Jaundice due to conjugation disorder
C. Hepatitis
D. Physiological jaundice
E. Hemolytic disease of newborn (Rh -incompatibility)

32. Ambulance was called to a 48 y.o. man. According to the relatives he has had three episodes of
lost consciousness and attacks during the day. Patient is unconscious, fell on the floor, tonic
and then clonic convulsions of trunk and extremities happened. The attack lasted for 4
minutes, ended by involuntary urination. What type of attack was observed?
A. Major epileptic seizure*
B. Vegetatic crisis
C. Absence
D. Episode of hysteria
E. Fainting

33. A 2 y.o. girl has been ill for 3 days. Today she has low grade fever, severe catarrhal

7
Завантажено з сайту https://тестування.укр/ - онлайн тестування КРОК

presentations, slight maculopapular rash on her buttocks and enlarged occipital lymph nodes.
What is your diagnosis?
A. Rubella*
B. Scarlet fever
C. Measles
D. Adenoviral infection
E. Pseudotuberculosis

34. A 7 y.o. girl has mild form of varicella. Headache, weakness, vertigo, tremor of her limbs,
ataxia, then mental confusion appeared on the 5th day of illness. Meningeal signs are negative.
Cerebrospinal fluid examination is normal. How can you explain these signs?
A. Encephalitis*
B. Meningitis
C. Meningoencephalitis
D. Myelitis
E. Neurotoxic syndrome

35. A mother with an infant visited the pediatrician for expertise advice. Her baby was born with
body weight 3,2 kg and body length 50 cm. He is 1 year old now. How many teeth the baby
should have?
A. 8*
B. 10
C. 12
D. 20
E. 6

36. A boy of 7 y.o. had an attack of asthma and distant whistling rales after playing with a dog. In
the medical hystory: atopic dermatitis caused by eating eggs, chicken, beef. What group of
allergens is the reason of the development of bronchial astma attacks?
A. Epidermal*
B. Dust
C. Pollen
D. Itch mite
E. Chemical

37. A 40 y.o. patient with rheumatic heart disease complains of anorexia, weakness and loss of
weigth, breathlessness and swelling of feet. The patient had tooth extraction one month ago.
On examination: t0- 390C, Ps- 100/min. Auscultation: diastolic murmur in the mitral area.
Petechial lesion are round of clavicle; spleen was palpable.
A. Subacute bacteria endocarditis*
B. Recurrence of rheumatic fever
C. Thrombocytopenia purpura
D. Mitral stenosis
E. Aortic stenosis

38. A 62 y.o. patient suffers from DM-2. Diabetes is being compensated by diet and Maninilum.

8
Завантажено з сайту https://тестування.укр/ - онлайн тестування КРОК

Patient has to undergo an operation on inguinal hernia. What tactics of hypoglycemic therapy
should be used?
A. Prescribe fast-acting insulin*
B. Give Glurenorm in place of Maninilum
C. Continue with the current therapy
D. Prescribe long-acting insulin
E. Prescribe guanyl guanidines

39. A 33 y.o. woman has been suffering from DM (diabetes mellitus) for 5 years. For the last 3
years she has been taking more than 100 units of insulin per day. Body weight has increased
up to 10 kg. Fasting blood glucose is 13 mmol/L, glucoseuria - 3%. Generalized
microangiopathy. By increasing the dose of insulin the parameters of glycemia do not change.
The diagnosis is:
A. DM 1st type, severe form, decompensation, insulin resistant*
B. DM 2nd type, severe form, decompensation
C. DM st type, severe form, subcompensation, Somoji phenomenon
D. DM 2nd type, moderate form, Zabrodi phenomenon
E. DM 1st type, severe form, decompensation, allergic reaction to insulin

40. A 32 y.o. man is divorced, has an irregular sexual life. He complains of falling out of hair in the
region of eyelashes, eyebrows, scalp. Objectively: diffuse alopecia is observed, eyebrow margin
is absent, eyelashes are stair-like (Pinkus’ si- gn). What examination should be carried out first
of all?
A. Wasserman test, IFT*
B. T.pallidum Immobilization Test (TPI)
C. Detection of the nasal mucous membrane for Micobacterium Leprae Hansen
D. Consultation of neuropathist
E. CBC

41. A triad of symptoms (“stearing spot" “terminal film" “blood dew”) have been revealed on
examination of a patient. What disease should you think about?
A. Psoriasis*
B. Lichen ruber planus
C. Vasculitis
D. Seborrhea
E. Ritter’s disease

42. A 4 y.o. child attends the kindergarten. Complains of poor appetite, fatigue. Objective
examination: skin and mucous membrane are pale, child is asthenic. In the hemogram:
hypochromatic anemia 1st, leucomoide reaction of the eosinophi-le type. What pathology must
be excluded first of all?
A. Helminthic invasion*
B. Lymphoprolipherative process
C. Hypoplastic anemia
D. Duodenal ulcer
E. Atrophic gastritis

9
Завантажено з сайту https://тестування.укр/ - онлайн тестування КРОК

43. A 36 y.o. woman is in the 12-th week of her first pregnancy. She was treated for infertility in
the past. She contacted a child who fell ill with rubella 2 days after their meeting. Woman
doesn’t know if she has ever been infected with rubella. What is the adequate tactics?
A. Monitoring of the specific IgG IgM with the ELISA*
B. Fetus wastage
C. Immunoglobulin injection
D. Cyclovin prescription
E. Interferon prescription

44. A 32 y.o. woman has got the Laiel’s syndrome after taking the biceptol. What immunotrope
medicines are to be prescribed in this situation?
A. Steroid immunosupressants*
B. Non-specific immune modulators
C. Specific immune modulators
D. Interferons
E. Non-steroid immunosupressants

45. A 28 y.o. man fell seriously ill, he feels chill, has got a fever, body temperature raised up to
38,50C, paroxysmal pain in the left iliac region, frequent defecation in form of fluid bloody and
mucous mass. Abdomen palpation reveals painfulness in its left half, sigmoid colon is spasmed.
What is the most probable diagnosis?
A. Acute dysentery*
B. Amebiasis
C. Colibacillosis
D. Nonspecific ulcerative colitis
E. Malignant tumors of large intestine

46. The disease of a 21 y.o. patient began with raise of temperature up to 39, 00C, headache, chill,
repeated vomiting. Rigidity of occipital muscles is determined. The analysis of liquor revealed:
cytosis -1237 in 1 ml, including: 84% of neutrophils, 16% of lymphocytes. On bacterioscopy:
gram-negative cocci are found in liquor. What is the most probable disease?
A. Meningococcal infection: purulent meningitis*
B. Meningococcal infection: serous meningitis
C. Secondary purulent meningitis
D. Serous meningitis
E. Infectious mononucleosis

47. The family doctor examined a patient and diagnosed an acute bleeding of an intestine. What is
professional tactics of the doctor in this situation?
A. The urgent hospitalization in surgical department*
B. To inject intravenously the ami-nocapronic acid
C. The urgent hospitalization in therapeutic department
D. Treatment at a day time hospital
E. Treatment at home

48. A 40 y.o. woman is ill with rheumatic disease with composite mitral disease with prevalence of

10
Завантажено з сайту https://тестування.укр/ - онлайн тестування КРОК

the stenosis of left venous foramen.Complains of palpitation, fatigability, progressing dyspnea,


attacks of dyspnea and hemoptysis. Now she cannot be engaged even in the easy activities.
What tactics is the most expedient?
A. Mitral comissurotomia*
B. Conduction of current bicilino-prophilaxis
C. Prescription of anticoagulants
D. Prescription of venous vasodilatators
E. -

49. A man, 42 years old, died in a road accident after the hemorrhage on the spot, because of
acute hemorrhagic anemia. What minimum percent of the whole blood volume could result in
death by acute hemorrhage?
A. 25-30%*
B. 6-9%
C. 10-14%
D. 15-20%
E. 35-50%

50. A 50 y.o. male patient was taken to the emergency department with diffuse abdominal pain
and signs of cardiovascular collapse. On arrival he had BP-95/60 mm Hg, Ps- 120/min, diuresis
- 20 ml/h, HgB- 100 g/L, RBC- 2,1 * 1012/L. The patient needs introduction of:
A. Crystalloid and colloid*
B. Crystalloid and red blood cells
C. Crystalloid and 5% dextrose
D. 5% dextrose and red blood cells
E. 5% dextrose and colloid

51. A 58 y.o. man complained of severe inspiratory dyspnea and expectoration of frothy and blood-
tinged sputum. He has been suffering from essential hypertension and ischemic heart disease.
On examination: acrocyanosis, "bubbli-ng"breathing, Ps- 30/min, BP- 230/130 mm Hg, bilateral
rales. Choose medicines for treatment.
A. Morphine, furosemide, nitroprusside sodium*
B. Theophylline, prednisolon
C. Albuterol, atropine, papaverine
D. Strophanthine, potassium chloride, plathyphylline
E. Cordiamine, isoproterenol

52. A patient has got a sudden attack of severe substernal pain at night. On examination:
confusion, pallor of the skin, acrocyanosis, cold sweat, BP- 80/50 mm Hg, Ps- 120/min,
irregular and weak pulse. What condition are these symptoms typical for?
A. Cardiogenic shock*
B. Acute left-side heart failure
C. Acute right-side heart failure
D. Radicular syndrome
E. Acute vascular insufficiency

11
Завантажено з сайту https://тестування.укр/ - онлайн тестування КРОК

53. A 61 y.o. man complained of sneezing and substernal pain on exertion. In the last 2 weeks
such pain appeared at rest, with increased frequency, and couldn’t be suppressed by 1 tablet
of nitroglycerin. What is the most likely diagnosis?
A. Unstable angina pectoris*
B. Angina pectoris of a new onset
C. Myocarditis
D. Radiculitis
E. Stable angina pectoris of the III functional class

54. A 41 y.o. woman complains of weakness, fatigue, fever up to 380C, rash on the face skin, pain
in the wrists and the elbows. On physical examination: erythematous rash on the cheeks with
"butterfly"look, the wrists and elbow joints are involved symmetrically, swollen, sensitive,
friction rub over the lungs, the heart sounds are weak, regular, HR-88/min, BP- 160/95 mm
Hg. CBC shows anemia, leucopenia, lymphopenia; on urine analysis: proteinuria, leukocyturia,
casts. What is the main mechanism of disease development?
A. Production of antibodies to doublestranded DNA*
B. Production of myocytes antibodies
C. Production of antibodies to endothelial cells
D. Production of myosin antibodies
E. Production of antimitochondrial antibodies

55. A 56 y.o. woman has an acute onset of fever up to 390C with chills, cough, and pain on
respiration in the right side of her chest. On physical examination: HR-90/min, BP- 95/60 mm
Hg, Ps- 26/min. There is dullness over the right lung. On X-ray: infiltrate in the right middle
lobe of the lung en palpation. What is the diagnosis?
A. Community-acquired lobar pneumonia with moderate severity*
B. Community-acquired bronchopneumonia
C. Acute pleurisy
D. Acute lung abscess
E. Hospital-acquired lobar pneumonia

56. A 29 y.o. woman is critically ill. The illness is presented by high fever, chills, sweating, aching
pain in lumbar area, discomfort during urination and frequent voiding. Pasternatsky’s sign is
positive in both sides. On lab examination: WBC-20 * 109/L; on urine analysis: protein -0,6g/L,
leukocyturia, bacteriuria. Your preliminary diagnosis.
A. Acute pyelonephritis*
B. Exacerbation of chronic pyelonephritis
C. Acute glomerulonephritis
D. Acute cystitis
E. Nephrolithiasis

57. A 45 y.o. man has complained of having epigastric and right subcostal aching pain, pruritus,
indigestion, dark color of the urine and acholic stool, fever and significant weight loss for 1
month. On examination: jaundice, presence of Curvuasier’s sign. US scan did not reveal stones
in the gallbladder and choledochus. What is the most likely diagnosis?
A. Cancer of the pancreas head*
B. Gallbladder stones

12
Завантажено з сайту https://тестування.укр/ - онлайн тестування КРОК

C. Chronic pancreatitis
D. Chronic cholangitis
E. Chronic hepatitis

58. A 27 y.o. man complained of aching epigastric pain right after meal, heartburn and nausea.
Stomach endoscopy revealed a large amount of mucus, hyperemia and edema of mucous
membrane in gastric fundus with areas of atrophy. Make a diagnosis.
A. Chronic gastritis of type A*
B. Chronic gastritis of type B
C. Peptic ulcer of stomach
D. Chronic gastritis of type C
E. Menetrier’s disease

59. A 25 y.o. woman complained of fatigue, hair loss and brittle nails. The examination revealed
pallor of skin, Ps- 94/min, BP- 110/70 mm Hg. On blood count: Hb- 90 g/L, RBC- 3,5 * 1012/L,
C.I.- 0,7; ESR- 20 mm/h. Serum iron level was 8,7 mcmol/L. What treatment would you
initiate?
A. Ferrous sulfate orally*
B. Iron dextrin injections
C. Vitamin B12 intramuscularly
D. Blood transfusion
E. Packed RBCs transfusion

60. A 60 y.o. woman has had increased BP up to 210/110 mm Hg for the last 7 years. On
examination: heart apex is displaced to the left. There are signs of left ventricular hypertrophy
on ECG. What is the most probable diagnosis?
A. Essential hypertension, 2nd stage*
B. Essential hypertension, 1st stage
C. Symptomatic hypertension
D. Cardiomyopathy
E. Ischemic heart disease

61. A 30 y.o. man complains of intense pain, skin reddening in the region of ankle joint,
temperature rise up to 390C. He fell ill suddenly. In the past there were such onsets that
lasted for 5-6 days and didn’t cause any residual changes of the joint. The skin over the joint is
hyperemic, without distinct outlines and infiltrative bank at the periphery. What is the most
probable diagnosis?
A. Gout*
B. Infectional arthritis
C. Rheumatoid arthritis
D. Erysipelatous inflammation
E. Osteoarthrosis

62. A 6 y.o. asthmatic child was taken to the emergency hospital because of severe coughing and
wheezing for the last 24 hours. Physical examination reveals that the child is excitable, has
intercostal and suprasternal retractions, expiratory wheezing throughout all lung fields,

13
Завантажено з сайту https://тестування.укр/ - онлайн тестування КРОК

RR-60/min. Initial treatment may include the prescription of:


A. Subcutaneous epinephrine*
B. Parenteral phenobarbital
C. Intravenous fluids in the first 2 h to compensate water deficiency
D. N-acetyl cysteine and cromolyn by inhalation
E. Parenteral gentamicyn

63. A surgical department admitted a newborn boy with foamy discharges from nose and mouth,
cyanosis attacks. X-ray examination: blind end of esophagus is at the level of the II thoracic
vertebra, gastric air bubble is under the left cupula of diaphragm. What is the most probable
diagnosis?
A. Esophagus atresia, tracheo-esophageal fistula*
B. Total esophagus atresia
C. Paraesophageal hernia
D. Esophagus atresia without a fistula
E. Bronchoesophageal fistula

64. A 30 y.o. man complains of sharp pain in the right ear, hearing loss, high temperature for
three days. Objectively: right ear whispering language - 0,5 m, external ear is intact,
otoscopically - eardrum protrusion, hyperemia and swelling, loss of landmarks. What disease is
it?
A. Acute purulent otitis media*
B. Acute mastoiditis
C. Chronic secretory otitis media
D. Chronic purulent otitis media
E. Eustachian tube disfunction

65. A youth, aged 15, from childhood suffers from atopic dermatitis and allergy to the shellfish. In
the last 3 months after acquiring aquarium fish rhinitis, conjunctivitis, itching in the nose
developed. Level of what immunologic index should be defined in this case?
A. IgE*
B. IgJ
C. IgM
D. IgA
E. Circulating immunocomplexes

66. A pregnant woman (35 weeks), aged 25, was admitted to the hospital because of bloody
discharges. In her medical history there were two artificial abortions. In a period of 28-32
weeks there was noted the onset of hemorrhage and USD showed a placental presentation.
The uterus is in normotonus, the fetus position is transversal (Ist position). The heartbeats is
clear, rhythmical, 140 bpm. What is the further tactics of the pregnant woman care?
A. To perform a delivery by means of Cesareansection*
B. To perform the hemotransfusion and to prolong the pregnancy
C. To introduct the drugs to increase the blood coagulation and continue observation
D. Stimulate the delivery by intravenous introduction of oxytocin
E. To keep the intensity of hemorrhage under observation and after the bleeding is
controlled to prolong the pregnancy

14
Завантажено з сайту https://тестування.укр/ - онлайн тестування КРОК

67. Studying of pulmonary tuberculosis incidence provided data about patients’ socioeconomic
living conditions and bad habits. What method allows to estimate the impact of these factors
on tuberculosis incidence?
A. Calculation of correlation coefficient*
B. Calculation of correspondence index
C. Calculation of regression coefficient
D. Standardized index calculation
E. Calculation of reliability coefficient

68. A 15 y.o. girl was examined. Her medi- cal history registers gradual onset of fever, malaise,
loss of weight. There was nothing typical about the kind of fever which has been present for
more than 7-10 days and changed quickly. Physical examination didn’t give evident results.
What is the only most important examination for excluding miliary tuberculosis?
A. Chest X-ray*
B. Liver or bone marrow biopsy
C. Tuberculin skin testing
D. Sputum smear and culture of m. tuberculosis
E. Bronchoscopy

69. A patient has got pain in the axillary area, rise of temperature developed 10 hours ago. On
examination: shaky gait is evident, the tongue is coated with white coating. The pulse is
frequent. The painful lymphatic nodes are revealed in the axillary area. The skin is
erythematous and glistering over the lymphatic nodes. What is the most probable diagnosis?
A. Bubonic plague*
B. Acute purulent lymphadenitis
C. Lymphogranulomatosis
D. Anthrax
E. Tularemia

70. A 17 y.o. patient complains of acute pain in the knee joint and t0- 38° C. He was ill with angina
3 weeks ago. Objectively: deformation and swelling of the knee joints with skin hyperemia.
Small movement causes an acute pain in the joints. Which diagnose is the most correct?
A. Rheumatism, polyarthritis*
B. Systemic lupus erythematodes
C. Reactive polyarthritis
D. Infectious-allergic polyarthritis
E. Rheumatoid arthritis

71. A girl is 12 y.o. Yesterday she was overcooled. Now she is complaining on pain in suprapubic
area, frequent painful urination by small portions, temperature is 37,8°C. Pasternatsky
symptom is negative. Urine analysis: protein - 0,033 g/L, WBC- 20-25 in f/vis, RBC-1-2 in f/vis.
What diagnosis is the most probable?
A. Acute cystitis*
B. Dysmetabolic nephropathy
C. Acute glomerulonephritis
D. Acute pyelonephritis
E. Urolithiasis

15
Завантажено з сайту https://тестування.укр/ - онлайн тестування КРОК

72. A 56 y.o. patient has worked at the aluminium plant over 20 years. Within 3 last years he has
got loosening of teeth, bone and joint pains, piercing pains in heart area, vomiting. The
preliminary diagnosis is:
A. Fluorine intoxication*
B. Mercury intoxication
C. Lead intoxication
D. Phosphorus intoxication
E. Manganese intoxication

73. A 38 y.o. woman complains of a purulent discharge from the left nostril. The body temperature
is 37,50C. The patient has been ill for a week and associates her illness with common cold.
There are a pain and tenderness on palpation of her left cheek. The mucous membrane in the
left nasal cavity is red and turgescent. The purulent exudate is seen in the middle meatus in
maxillary. What is the most probable diagnosis?
A. Acute purulent maxillary sinusitis*
B. Acute purulent frontitis
C. Acute purulent ethmoiditis
D. Acute purulent sphenoiditis
E. -

74. Condition of a parturient woman has been good for 2 hours after live birth: uterus is thick,
globe-shaped, its bottom is at the level of umbilicus, bleeding is absent. The clamp put on the
umbilical cord remains at the same level, when the woman takes a deep breath or she is being
pressed over the symphysis with the verge of hand, the umbilical cord drows into the vagina.
Bloody discharges from the sexual tracts are absent. What is the doctor’s further tactics?
A. To do manual removal of afterbirth*
B. To apply Abduladze method
C. To apply Crede’s method
D. To do curettage of uterine cavity
E. To introduct oxitocine intravenously

75. A 34 y.o. patient has been suffering from pulmonary tuberculosis for 7 years; he complains of
muscle weakness, weight loss, diarrhea, frequent urination. Objectively: hyperpigmentation of
skin, gums, internal surface of cheeks. AP- 90/58 mm Hg. Blood count: RBC- 3,1 * 1012/L, Hb-
95 g/L, C.I.- 0,92; leukocytes - 9,4 * 109/L, eosinophils - 7, segmentonuclear leukocytes - 45,
stab neutrophils - 1, lymphocytes - 40, monocytes - 7, Na+-115 mmol/L, K+- 7,3 mmol/L. What
is the preliminary diagnosis?
A. Primary adrenocortical insufficiency*
B. Pheochromocytoma
C. Primary hyperaldosteronism
D. Congenital adrenocortical hyperplasia
E. Diabetes insipidus

76. A 52 y.o. hard smoker complains of persistent cough with purulent sputum discharge
especially in the mornings, dyspnea provoked even by slight physical exercises, wheezing
chest, tachypnoe, general weakness. He considers himself to be ill for 12 years. The foresaid
presentations appear 3-4 times per year usually after a common cold and have tendency to

16
Завантажено з сайту https://тестування.укр/ - онлайн тестування КРОК

progress. What disease do you think about first of all?


A. Chronic obstructive lung disease*
B. Bronchial asthma
C. Mucoviscidosis (cystic fibrosis)
D. Bronchoectatic disease
E. Aspergillosis

77. 3 weeks ago a patient was ill with tonsillitis. Clinical examination reveals edema, arterial
hypertension, hematuria, proteinuria (1,8 g/per day), granular and erythrocital casts. What is
the preliminary diagnosis?
A. Glomerulonephritis*
B. Cystitis
C. Pyelonephritis
D. Intestinal nephritis
E. Renal amyloidosis

78. 47 y.o. patient complains of intensive skin itching, jaundice, bone pain. The skin is
hyperpigmentated. There are multiple xanthelasma palpebrae. The liver is +6 cm enlarged,
solid with acute edge. The blood analysis revealed total bilirubin -160 mkmol/L, direct -110
mkmol/L, AST- 2.1 mmol/L, ALT- 1,8 mmol/L, alkaline phosphotase - 4,6 mmol/L, cholesterol-
9.2 mmol/L, antimitochondrial antibodies M2 in a high titer. What is the probable diagnosis?
A. Primary biliary liver cirrhosis*
B. Primary liver cancer
C. Chronic viral hepatitis B
D. Acute viral hepatitis B
E. Alcoholic liver cirrhosis

79. In the gynecologic office a 28 y.o. woman complains of sterility within three years. The
menstrual function is not impaired. There were one artificial abortion and chronic salpingo-
oophoritis in her case history. Oral contraceptives were not used. Her husband’s analysis of
semen is without pathology. What diagnostic method will you start from the workup in this
case of sterility?
A. Hysterosalpingography*
B. Hormone investigation
C. Ultra sound investigation
D. Diagnostic scraping out of the uterine cavity
E. Hysteroscopia

80. A patient has undergone an operation on account of perforated ulcer of stomach, terminal
phase of diffuse peritonitis and endotoxic shock. In the postoperative period he is prescribed
artificial pulmonary ventilation with 60% oxygen inhalation. Blood gases: Pa02- 7078 mm Hg,
hypoxemy doesn’t ecrease, CVP (central venous pressure) - 150180 mm of water column, AP-
90/60 mm Hg (against the backgound of taking big doses of dopamine). Radiogram shows
diffuse pulmonary infiltration. What cause the refractory arterial hypoxemia?
A. Respiratory distress syndrome*
B. Bilateral pneumonia
C. Pneumothorax

17
Завантажено з сайту https://тестування.укр/ - онлайн тестування КРОК

D. Mendelson’s syndrome
E. Pulmonary edema

81. According to results of medical and pedagogical observation during the gymnastics lesson in
the 9th grade there was plotted a phisiological curve characterized by gradual increasing of
pulse rate during the opening part of lesson, 80% increase during the main part; the curve has
4 waves. How can the lesson’s structure be assessed?
A. Long interval between exercises*
B. The lesson’s structure is correct
C. Exercise stress is insufficient
D. Exercise stress is excesive
E. Exercise stress is adequate

82. A patient is 16 y.o. In the last year his behaviour has gradually changed: he secluded himself,
was not interested in communication with friends, in learning. He became indifferent towards
relatives, motivelessly rude, was speaking or laughing to himself. He answers the questions
formally correctly, laconically. Considers himself to be absolutely healthy but a little tired,
says, he’s thinking about writing a book "Projection of humanity on the plane of Universe". He
always has a copybook with which is full of a great many of the same daggers. What is the
most
A. Schizophrenia*
B. Depressive disorder
C. Schizoid personality disorder
D. Autistic personality disorder
E. Pick’s disease

83. A worker was temporarily off work because of illness for l6 days, was under out-patient
treatment. The doctor in charge issued a sick-list first for 5 days, then prolonged it for 10 days.
Who can further prolong the sick-list of this patient?
A. The doctor in charge of the case together with the head of department*
B. Working ability expertise committee
C. The doctor in charge of the case with the permission of the head of department
D. Deputy head physician on the working ability expertise
E. The head of department

84. The results of a separate diagnostic curettage of the mucous of the uterus’ cervix and body
made up in connection with bleeding in a postmenopausal period: the scrape of the mucous of
the cervical canal revealed no pathology, in endometrium - the highly differentiated
adenocarcinoma was found. Metastases are not found. What method of treatment is the most
correct?
A. Surgical treatment and hormonotherapy*
B. Surgical treatment + chemotherapy
C. Surgical treatment and radial therapy
D. Radial therapy
E. -

18
Завантажено з сайту https://тестування.укр/ - онлайн тестування КРОК

85. A 27 y.o. woman complains of having the disoders of menstrual function for 3 months,
irregular pains in abdomen. On bimanual examination: in the dextral appendage range of
uterus there is an elastic spherical formation, painless, 7 cm in diameter. USI: in the right
ovary - a fluid formation, 4 cm in diameter, unicameral, smooth. What method of treatment is
the most preferable?
A. Prescription of an estrogen-gestogen complex for 3 months with repeated examination*
B. Operative treatment
C. Dispensary observation of the patient
D. Anti-inflammatory therapy
E. Chemotherapeutic treatment

86. A 40 y.o. patient complains of yellowi- sh discharges from the vagina. Bimanual examination:
no pathological changes. The smear contains Trichomonas vaginalis and blended flora.
Colposcopy: two hazy fields on the front labium, with a negative Iodum test. Your tactics:
A. Treatment of specific colpitis and with the subsequent biopsy*
B. Diathermocoagulation of the cervix of the uterus
C. Specific treatment of Trichomonas colpitis
D. Cervix ectomy
E. Cryolysis of cervix of the uterus

87. A full-term new-born suffered from ante- and intranatal hypoxia, was born in asphyxia (Apgar
score 2-5 points). After birth baby’s excitation is progressing, occurs vomiting, nystagmus,
spasms, squint, spontaneous Babinski and Moro’s reflexes. What is the most probable location
of the intracranial hemorrhage in this case?
A. Subarachnoid hemorrhages*
B. Small hemorrhages in brain tissue
C. Subdural hemorrhages
D. Periventricular hemorrhages
E. Hemorrhages in ventricles of brain

88. A 37 y.o. primigravida woman has been having labor activity for 10 hours. Labor pains last for
20-25 seconds every 6-7 minutes. The fetus lies in longitude, presentation is cephalic, head is
pressed upon the entrance to the small pelvis. Vaginal examination results: cervix of uterus is
up to 1 cm long, lets 2 transverse fingers in. Fetal bladder is absent. What is the most probable
diagnosis?
A. Primary uterine inertia*
B. Secondary uterine inertia
C. Normal labor activity
D. Discoordinated labor activity
E. Pathological preliminary period

89. A patient at a doctor complains of temperature rise up to 38, 20C, edema in the region of his
upper lip. Objectively: upper lip is evidently edematic, in the middle of edema there is a cone-
shaped swelling. Skin and mucous membrane over it are dark-red. Diagnosis: labial furuncle. A
surgeon cut the furuncle, treated the wound with hydrogen peroxide solution and applied a
bandage with hypertensive solution. What therapeutic regimen should be recommended?
A. In-patient treatment with common regimen*

19
Завантажено з сайту https://тестування.укр/ - онлайн тестування КРОК

B. First out-patient treatment, then outpatient treatment


C. Out-patient treatment
D. In-patient treatment with bed rest
E. -

90. A 15 y.o. boy was twice attacked by bees, as a result he had severe anaphylactic shock. What
is the most effective prophylaxis method?
A. Desensibilisation by means of bee venom extract*
B. Prescription of corticosteroids for summer
C. Long-term prophylactic treatment with antihistamines
D. Limitation of outside staying during summer months
E. Protective clothing

91. A healthy 75 y.o. woman who leads a moderately active way of life went through a preventive
examination that revealed serum concentration of common cholesterol at the rate of 5,1
mmol/l and HDL (high-density lipoproteins) cholesterol at the rate of 70 mg/dl. ECG reveals no
pathology. What dietary recommendation is the most adequate?
A. Any dietary changes are necessary*
B. Decrease of cholesterol consumption
C. Decrease of saturated fats consumption
D. Decrease of carbohydrates consumption
E. Increase of cellulose consumption

92. In treatment and prevention establishments, regardless of their organisational and proprietary
form, the rights of patients should be observed. Which of these rights are the most significant?
A. The right to the protection of the patient’s interests*
B. The right to the free choice
C. The right to the information
D. The right to be heard
E. The right to the protection from incompetence

93. A patient has complained of great weakness for 6 years. He fell seriously ill, the illness is
accompanied by body temperature rise, indisposition, pain in joints and along the legs
muscles. Objectively: violet-bluish erythema around eyes and over knee joints. HR- 120/min,
heart sounds are weak. Blood count: leukocytes - 12 * 109/L, ESR- 40 mm/h. What is the
A. Dermatomyositis*
B. Systemic lupus erythematosus
C. Rheumathoid arthritis
D. Atopic dermatitis
E. Reactive polyarthritis

94. A military unit stopped for 3-day’s rest in inhabited locality after a long march. The sanitary-
epidemiological reconnaissance found several water sources. It is necessary to choose the
source complying with the hygienic standards for drinking water in the field
A. Artesian well water*
B. Spring water

20
Завантажено з сайту https://тестування.укр/ - онлайн тестування КРОК

C. River water
D. Rain water
E. Water from melted snow

95. A district doctor was commisioned with a task to work out a plan of treatment-and-prophylaxis
actions for the population of his district. What actions of secendary prophylaxis must he
include into this plan?
A. Prevention of disease complications*
B. Disease prevention
C. Elimination of disease causes
D. Improvement of population’s living conditions
E. Rehabilitation actions

96. A 43 y.o. patient complains of formation and pain in the right mammary gland, rise of
temperature up to 37, 2°C during the last 3 months. Condition worsens before the
menstruation. On examination: edema of the right breast, hyperemia, retracted nipple.
Unclear painful infiltration is palpated in the lower quadrants. What is the most probable
diagnosis?
A. Cancer of the right mammary gland*
B. Right-side acute mastitis
C. Right-side chronic mastitis
D. Premenstrual syndrome
E. Tuberculosis of the right mammary gland

97. A 20 y.o. patient was admitted to the hospital with complaints of having skin and sclera
icteritiousness, dark urine, single vomiting, appetite loss, body temperature rise up to 380C
for 2 days. Three weeks ago he went in for fishing and shared his dishes with friends.
Objectively: the patient is flabby, t0- 36,80C, skin and scleras are icteritious, liver sticks from
under the costal margin by 3 cm, it is sensi- tive; spleen isn’t palpable. Urine is dark, stool is
partly acholic. What is the most probable diagnosis?
A. Virus A hepatitis*
B. Leptospirosis
C. Infectious mononucleosis
D. Hemolytic anemia
E. Intestinal yersiniosis

98. A full-term newborn child has a diagnosis Rh-factor hemolytic disease of newborn. Bilirubin
rate is critical. The child’s blood group is B(III), his mother’s blood group - A(II). The child has
indication for hemotransfusion. What donor blood must be chosen?
A. Blood group B(III), Rh (-)*
B. Blood group A(II), Rh (-)
C. Blood group B(III), Rh (+)
D. Blood group A(II), Rh (+)
E. Blood group O(I), Rh (-)

99. At year-end hospital administration has obtained the following data: annual number of treated

21
Завантажено з сайту https://тестування.укр/ - онлайн тестування КРОК

patients and average annual number of beds used for patient’s treatment. What index of
hospital work can be calculated based upon this data?
A. Bed turnover*
B. Bed resources of the hospital
C. Average annual bed occupancy
D. Average duration of patients presence in the hospital
E. Average bed idle time

100. A 52 y.o. patient fell from 3 m height on the flat ground with the right lumbar area. He
complains of pain in this area. There is microhematuria in the urea. Excretory urography
revealed that kidney’s functioning is satisfactory. What is the most probable diagnosis?
A. Kidney’s contusion*
B. Subcapsular kidney’s rupture
C. Multiple kidney’s ruptures
D. Paranephral hematoma
E. Kidney’s abruption

101. A 3 y.o. child with weight deficiency suffers from permanent moist cough. In history there are
some pneumonias with obstruction. On examination: distended chest, dullness on percussion
over the lower parts of lungs. On auscultation: a great number of different rales. Level of
sweat chloride is 80 mmol/L. What is the most probable diagnosis?
A. Mucoviscidosis (cystic fibrosis)*
B. Bronchial asthma
C. Recurrent bronchitis
D. Bronchiectasis
E. Pulmonary hypoplasia

102. A 14 y.o. girl complains of profuse bloody discharges from genital tracts during 10 days after
suppresion of menses for 1,5 month. Similiar bleedings recur since 12 years on the
background of disordered menstrual cycle. On rectal examination: no pathology of the internal
genitalia. In blood: Hb- 70 g/L, RBC- 2, 3 * 1012/L, Ht-20. What is the most probable
diagnosis?
A. Juvenile bleeding, posthemorrhagic anemia*
B. Werlholf’s disease
C. Polycyst ovarian syndrome
D. Hormonoproductive ovary tumor
E. Noncomplete spontaneous abortion

103. A 33 y.o. patient with first time detected diabetes mellitus keeps the diet and maintains
glycemia at the level of 10,0 mmol/L after meal. He keeps himself from insulinotherapy. What
examination is the most important for differentiation of the 1st (insulin-dependent) and the
2nd (insulin-independent) types of diabetes?
A. Determination of insular cells antibodies*
B. Glucose-tolerant test
C. Glycemia examination on an empty stomach
D. Estimation of glucolized blood hemoglobin
E. Fructosamine estimation

22
Завантажено з сайту https://тестування.укр/ - онлайн тестування КРОК

104. A 43 y.o. patient had cholecystectomy 6 years ago because of chronic calculous cholecystitis.
Lately he has been suffering from pain in the right subcostal area and recurrent jaundice.
Jaundice hasn’t gone for the last 2 weeks. Stenoutic papillitis 0,5 cm long has been revealed.
What is the best way of treatment?
A. To perform endocsopic papillosphincterotomy*
B. To treat conservatively: antibiotics, spasmolytics, antiinflammatory drugs
C. To perform external choledoch drainage
D. To perform transduodenal papillosphincterotomy
E. To perform choledochoduodenostomy

105. A 7 y.o. boy has been treated in a hospital for a month. At the time of admission he had evident
edemata, proteinuria - 7,1 g/L, protein content in the daily urine - 4,2 g. Biochemical blood
analysis reveals permanent hypoproteinemia (43,2 g/L), hypercholesterolemia (9,2 mmol/L).
What variant of glomerulonephritis is the most probable?
A. Nephrotic*
B. Nephritic
C. Isolated urinary
D. Hematuric
E. Mixed

106. A man, aged 68, complains of tiredness, sweating, enlargement of cervical, submaxillary and
axillary lymph nodes. Blood test: WBC- 35 * 109/L, lymphocytes - 60%, Botkin and Gumprecht
bodies, level of haemoglobin and quantity of thrombocytes is normal. Myelogram showed 40%
of lymphocytes. What is the most probable diagnosis?
A. Chronic lympholeucosis*
B. Chronic myeloleucosis
C. Lymphogranulomatosis
D. Acute leucosis
E. Tuberculous lymphadenitis

107. Examination of a 32 y.o. patient who had apendectomy because of gangrenous appendicitis
revealed clinical picture of pelvic abscess. What is the best way to expose pelvic abscess of this
patient?
A. Through the rectum*
B. Through the postoperative wound
C. By means of the sacral approach
D. Through the anterior abdominal wall
E. Through the obturator foramen

108. Examination of a just born placenta reveals defect 2x3 cm large. Hemorrhage is absent. What
tactic is the most reasonable?
A. Manual uretus cavity revision*
B. Prescription of uterotonic medicines
C. External uterus massage
D. Parturient supervision
E. Instrumental uterus cavity revision

23
Завантажено з сайту https://тестування.укр/ - онлайн тестування КРОК

109. A 3 y.o. girl has had a temperature rise up to 380C, rhinitis, dry superficial cough, flabbiness,
appetite loss. Palpation didn’t reveal any changes over her lungs. Percussion sound has a
wooden resonance, auscultation revealed puerile breathing, no rales. In blood: leukopenia,
lymphocytosis, increased ESR. What is the most probable diagnosis?
A. Acute simple tracheitis*
B. Acute obstructive bronchitis
C. Recurrent bronchitis, acute condition
D. Acute simple bronchitis
E. Bilateral microfocal pneumonia

110. A 12 y.o. girl took 2 pills of aspirine and 4 hours later her body temperature raised up to 39 —
40oC. She complains of general indisposition, dizziness, sudden rash in form of red spots and
blisters. Objectively: skin lesions resemble of second-degree burns, here and there with
erosive surface or epidermis peeling. Nikolsky’s symptom is positive. What is the most
probable diagnosis?
A. Acute epidermal necrolisis*
B. Pemphigus vulgaris
C. Polymorphous exudative erythema
D. Bullous dermatitis
E. Duhring’s disease

111. A patient complains of pathological lump, appearing in the right inguinal region on exercise.
The lump is round-shaped, 4 cm in diameter, on palpation: soft elastic consistency, is
positioned near the medial part of Poupart’s ligament. The lump is situated inwards from the
spermatic cord. What is the most probable preliminary diagnosis?
A. Right-sided direct inguinal hernia*
B. Right-sided oblique inguinal hernia
C. Right-sided femoral hernia
D. Varicose veins of the right hip
E. Lipoma of the right inguinal area

112. A 54 y.o. patient has been sufferi- ng from osteomyelitis of femoral bone for over 20 years. In
the last month there appeared and gradually progressed edemata of lower extremities. Urine
analysis: proteinuria - 6,6 g/L. Blood analysis: disproteinemia in form of hypoalbuminemia,
increase of a2- and q-globulins, ESR- 50 mm/h. What is the most probable diagnosis?
A. Secondary renal amyloidosis*
B. Acute glomerulonephritis
C. Myelomatosis
D. Chronic glomerulonephritis
E. Systemic lupus erythematosus

113. A mineshaft is situated on the territory of homestead land, it is 20 m away from the house, 10
m- from the toilet and 15 m - from the neighbour’s house. What is the smallest distance that,
according to the sanitary code, should be established between the well and the source of
A. 30 m*
B. 25 m
C. 20 m

24
Завантажено з сайту https://тестування.укр/ - онлайн тестування КРОК

D. 15 m
E. 10 m

114. A 43 y.o. woman complains of contact hemorrhages during the last 6 months. Bimanual
examination: cervix of the uterus is enlarged, its mobility is reduced. Mirrors showed the
following: cervix of the uterus is in the form of cauliflower. Chrobak and Schiller tests are
positive. What is the most probable diagnosis?
A. Cancer of cervix of the uterus*
B. Polypus of the cervis of the uterus
C. Cervical pregnancy
D. Nascent fibroid
E. Leukoplakia

115. A boy, aged 9, was examined: height -127 cm (-0,36), weight - 28,2 kg (+0,96), chest
circumference - 64,9 cm (+0,66), lung vital capacity - 1520 ml (-0,16). What is the complex
assessment of the child’s physical development?
A. Harmonious*
B. Disharmonious
C. Apparently disharmonious
D. Excessive
E. Below the average

116. A 14 y.o. child suffers from vegetovascular dystonia of pubertal period. He has got
sympathoadrenal atack. What medicine should be used for atack reduction?
A. Obsidan*
B. No-shpa
C. Amisyl
D. Euphyline
E. Corglicone

117. A 16 y.o. teenager complains of weakness, dizziness, sense of heaviness in the left
hypochondrium. Objectively: skin and visible mucous membranes are icteric. Steeple skull.
Liver +2 cm, the lower pole of spleen is at the level of navel. Blood test: RBC- 2, 7 * 1012/L,
Hb- 88 g/L, WBC-5,6 * 109/L, ESR- 15 mm/h. What is the most probable reason of bilirubin
level change?
A. Increase of unconjugated bilirubin*
B. Increase of conjugated bilirubin
C. Increase of unconjugated and conjugated bilirubin
D. Decrease of conjugated bilirubin
E. Decrease of unconjugated bilirubin

118. A 28 y.o. woman consulted a doctor about edematic face, moderate legs edemata; occasionally
her urine has colour of "meat slops". When she was a teenager she often fell ill with angina.
Objectively: skin is pallor, body temperature is 36,80C, Ps- 68/min, rhythmic. AP- 170/110 mm
Hg. What urine changes are the most probable?
A. Proteinuria, hematuria, cylindrouria*

25
Завантажено з сайту https://тестування.укр/ - онлайн тестування КРОК

B. Increase of relative density, hematuria, bacteriuria


C. Decrease of relative density, proteinuria, some urinary sediment
D. Erythrocyturia and urinozuria
E. Decrease of relative density, proteinuria

119. A 58 y.o. woman had voluminous bleeding from the ruptured varicous node on the left crus.
What first aid should be provided?
A. Heightened position of extremity, compressive sterile bandage*
B. Distal tourniquet
C. Proximal turniquet
D. Troyanov-Trendelenburg operation
E. Z-shaped stitch on the raptured varicose node

120. It is determined that 30 of a 100 women with risk factor had preterm labor, and of a 100
women without risk factor 5 women had preterm labor. What method of statistic data
processing should the doctor use in order to estimate reliability of differences between the
compared groups?
A. Student’s criterion calculation*
B. Average computing
C. Relative numbers calculation
D. Standardization method
E. Correlation analysis

121. A patient was admitted to the hospital with complaints of occasional pains at the bottom of
abdomen that get worse during menses, weakness, indisposition, nervousness, some dark
bloody discharges from vagina on the day before and the day after menses. Bimanual
examination results: body of womb is enlarged, appendages cannot be determined, posterior
fornix has tuberous surface. Laparoscopy results: ovaries, peritoneum of rectouterine pouches
and pararectal fat are covered with "cyanotic spots". What is the most probable diagnosis?
A. Widespread form of endometriosis*
B. Polycystic ovaries
C. Chronic salpingitis
D. Genital organs tuberculosis
E. Ovarian cystoma

122. A pregnant woman in her 8th week was admitted to the hospital for artificial abortion. In
course of operation during dilatation of cervical canal of uterus by means of Hegar’s dilator N°
8 the doctor suspected uterus perforation. What is the immediate tactics for confirmation of
this diagnosis?
A. Uterine probing*
B. Bimanual examination
C. US examination
D. Laparoscopy
E. Metrosalpingography

123. A 19 y.o. patient was admitted to the hospital with acute destructive appendicitis. He suffers

26
Завантажено з сайту https://тестування.укр/ - онлайн тестування КРОК

from hemophilia B-type. What antihemophilic medicine should be included in pre- and post-
operative treatment plan?
A. Fresh frozen plasma*
B. Cryoprecipitate
C. Fresh frozen blood
D. Native plasma
E. Dried plasma

124. A 28 y.o. patient without permanent residence was admitted to the hospital with the
preliminary diagnosis influenza. On the fith day of illness he got a maculopapular petechial
rash on his body and internal surfaces of extremities. Body temperature is 410C, euphoria,
face hyperemia, sclera reddening, tongue tremor, tachycardia, splenomegaly, excitement.
What is the most probable diagnosis?
A. Epidemic typhus*
B. Delirium alcoholicum
C. Leptospirosis
D. Measles
E. Typhoid fever

125. An 18 y.o. woman consulted a gynecologist about the pain in the lower part of abdomen, fever
up to 37,5°C, considerable mucopurulent discharges from the genital tracts, painful urination.
Vaginal examination with mirrors: the urethra is infiltrated, cervix of the uterus is hyperemic,
erosive. The uterus is painful, ovaries are painful, thickened; fornixes are free. Bacterioscopy
test revealed diplococcus. What diagnosis is the most probable?
A. Recent acute ascending gonorrhea*
B. Trichomoniasis
C. Candydomycosis
D. Chronic gonorrhea
E. Chlamydiosis

126. A 28 y.o. patient complains of pain in legs during walking, chill of feet and toes. He has been ill
for a year. Objectively: leg skin is pale, cool; turgor is decreased, hypotrichosis. Femoral and
popliteal artery pulsation is weak, it is palpable on the foot arteries only after nitroglycerine
test. Rheographic index is <1. What is the most probable diagnosis?
A. Obliterating endarteritis*
B. Chronic thrombophlebitis
C. Obliterating aterosclerosis
D. Raynaud’s disease
E. Buerger’s disease

127. A 2 m.o. child was delivered in time with weight 3500 g and was on the mixed feeding. Current
weight is 4900 g. Evaluate the current weight of the child:
A. Corresponding to the age*
B. 150 g less than necessary
C. Hypotrophy of the I grade
D. Hypotrophy of the II grade
E. Paratrophy of the I grade

27
Завантажено з сайту https://тестування.укр/ - онлайн тестування КРОК

128. A woman consulted a doctor on the 14th day after labor about sudden pain, hyperemy and
induration of the left mammary gland, body temperature rise up to 390C, headache,
indisposition. Objectively: fissure of nipple, enlargement of the left mammary gland, pain on
palpation. What pathology would you think about in this case?
A. Lactational mastitis*
B. Lacteal cyst with suppuration
C. Fibrous adenoma of the left mammary gland
D. Breast cancer
E. Phlegmon of mammary gland

129. A 60 y.o. woman complains of unbearable pains in the right hypochondrium. In the medical
hystory: acute pancreatitis. Body temperature is 38, 20C. Objectively: sclera icteritiousness.
No symptoms of peritonium irritation are present. There are positive Ortner’s and Hubergrits-
Skulski’s symptoms. Urine diastase is 320 g/h. What diagnosis is the most probable?
A. Chronic pancreatitis*
B. Acute cholangitis
C. Chronic cholecystitis
D. Acute cholecystitis
E. Cancer of pancreas

130. A 48 y.o. patient has been staying in the emergency department for 2 days on account of acute
anteroseptal myocardial infarction. In the course of examination he suddenly "snored". There
was a tonic contraction of skeletal muscles; eye pupils dilatated. Pulse on a.carotis is absent.
What is the immediate tactics?
A. Electric defibrillation*
B. Saphar’s triple airway maneuver
C. ECG record
D. Intracardiac introduction of adrenalin with atropine
E. Precardiac stroke

131. A 31 y.o. woman has complained for 3 years of pain and swelling of radiocarpal and
metacarpophalangeal articulations, morning stiffness that lasts up to 1,5 hours. Two weeks
ago she felt pain, swelling and reddening of knee joints, body temperature raised up to 37,50C.
Examination of her internal organs revealed no pathologic changes. Her diagnosis was
rheumatoid arthritis. What changes in X-ray pictures of her joints are the most probable?
A. Constriction of joint space, usura*
B. Constriction of joint space, subchondral osteosclerosis
C. Cysts in subchondral bone
D. Multiple marginal osteophytes
E. Epiphysis osteolysis

132. 8 hours after a road accident an unconscious victim with closed craniocerebral trauma was
admitted to the hospital. Objectively: anisocoria, wound in the parietal region - 3,0x1,0 cm,
neck muscles rigidity, Ps- 58/min, tense. Convulsive syndrome. What is the most important
indication for the immediate surgical procedure?
A. Intracranial hemorrhage*
B. Anisocoria

28
Завантажено з сайту https://тестування.укр/ - онлайн тестування КРОК

C. Unconsciousness
D. Wounds
E. Intensification of convulsions

133. A girl, aged 13, consulted the school doctor on account of moderate bloody discharge from the
genital tracts, which appeared 2 days ago. Secondary sexual characters are developed. What is
the most probable cause of bloody discharge?
A. Menarche*
B. Juvenile hemorrhage
C. Haemophilia
D. Endometrium cancer
E. Werlhof’s disease

134. A pregnant woman was registered in a maternity welfare clinic in her 11th week of pregnancy.
She was being under observation during the whole term, the pregnancy course was normal.
What document must the doctor give the pregnant woman to authorize her hospitalization in
maternity hospital?
A. Exchange card*
B. Appointment card for hospitalization
C. Individual prenatal record
D. Medical certificate
E. Sanitary certificate

135. In course of observation of sanitary conditions of studying at the technical university it was
necessary to evaluate the visual regimen of students, who study from 9 a.m to 3 p.m. What
index of natural light will be the most informative?
A. Natural light coefficient*
B. Light coefficient
C. Depth of study room
D. Time of the room insolation
E. Presence of mixed (upper-lateral) light

136. A 70 y.o. man is ill with ischemic heart disease. His mood is evidently depressed, anxious. As a
result of conti-nious sleeplessness he has got fears, suicidal thoughts. He would sit for a long
time in the same pose, answer after a pause, in a low, monotonous voice. His face has a look of
suffering, pain, fear. What is the main psychopathologic syndrome?
A. Depressive syndrome*
B. Paranoid syndrome
C. Asthenic syndrome
D. Phobic syndrome
E. Obsessive syndrome

137. 200 patients suffering from essential hypertension were examined in order to obtain data
about patients’ arterial pressure and age. What statistic value should be applied in order to
measure relation between these characteristics?
A. Correlation coefficient*

29
Завантажено з сайту https://тестування.укр/ - онлайн тестування КРОК

B. Student’s coefficient
C. Coefficient of variation
D. Representation error
E. Sygmal deviation

138. A 24 y.o. woman consulted a doctor about continued fever, night sweating. She lost 7 kg
within the last 3 months. She had casual sexual contacts. Objectively: enlargement of all lymph
nodes, hepatolienal syndrome. Blood count: leukocytes - 2, 2 * 109/L. What disease can be
suspected?
A. HIV-infection*
B. Lymphogranulomatosis
C. Tuberculosis
D. Infectionous mononucleosis
E. Chroniosepsis

139. A 31 y.o. patient has been suffering from systemic scleroderma for 14 years. She has been
treated in hospital many times. She complains of occasional dull pain in the heart region,
palpitation, dyspnea, headache, eye-lid edemata, weight loss and deformation of extremities
joints. What organ affection worsens the disease prognosis?
A. Kidneys*
B. Heart
C. Lungs
D. Gastrointestinal tract
E. Skin and joints

140. A 2 month old child who was born with body weight 5100 g has jaundice, hoarse cry, umbilical
hernia, developmental lag. His liver is +2 cm, spleen isn’t enlarged. Stool and urine are of
normal color. In anamnesis: delayed falling-away of umbilical rest. Blood count: Hb- 120 g/L,
RBC- 4,5 * 1012/L, ESR- 3 mm/h. General level of serum bilirubin - 28 mcmol/L, unconjugated
bilirubin - 20 mcmol/L, conjugated bilirubin - 8 mcmol/L. What disease would you think about
first of all?
A. Congenital thyreoid deficiency*
B. Congenital hepatitis
C. Hemolitic anemia
D. Conjugated jaundice
E. Cytomegalovirus infection

141. The doctors in maternity hospital made a newborn boy the following diagnosis: congenital
heart disease (interventricular septal defect). At the age of 2 months the boy has got a
dyspnea. Objectively: BR- up to 60/min, tachycardia up to 170/min, liver is 3 cm below the
costal margin. What medicines must be immediately prescribed?
A. Cardiac glycosides*
B. Nonsteroidal antiinflammatory drugs
C. Potassium preparations
D. 3-adrenoceptor blockers
E. Glucocorticoids

30
Завантажено з сайту https://тестування.укр/ - онлайн тестування КРОК

142. A 50 y.o. woman who suffers from chronic pyelonephritis was prescribed a combination of
antibiotics for the period of exacerbation - gentamicin (80 mg 3 times a day) and biseptol (960
mg twice a day). What consequences may be caused by such a combination of antibiotics?
A. Acute renal insufficiency*
B. Glomerulosclerosis
C. Chronic renal insufficiency
D. Antibiotic combination is optimal and absolutely safe
E. Acute suprarenal insufficiency

143. A 60 y.o. patient complains of having passing reduction of strength in his left extremities for a
month. Some time later he has got persistent weakness of extremities in the mornings.
Objectively: AP-140/90 mm Hg, conscious, central paresis of the VII and XII pair of left-side
cranial nerves, central hemiparesis and hemi-hyperesthesia also on the left side. What
medicines should be chosen for the differentiated treatment of the patient?
A. Anticoagulants*
B. Hemostatics
C. Hypotensive
D. Diuretics
E. Corticosteroids

144. A 52 y.o. male patient suffers from squeezing pain attacks in substernal area which irradiates
to the left hand and occurs occasionally and on physical exercises. He has had it for 1 year. On
examination: heart borders are enlargement to the left side, sounds are muffled, Ps- 76 bpm,
rhythmic, AP-155/80 mm Hg, ECG: the left type, the rest signs are normal. What additional
examination is necessary to confirm the diagnosis?
A. Veloergometry*
B. Echocardiography
C. Lipoprotein test
D. General blood count
E. Transaminases of blood

145. A 70 y.o. patient complains of weakness, dizziness, short periods of loss of consciousness, pain
in the region of heart. Objectively: HR- 40/min, sounds are rhytmic, the 1st sound is dull,
occasionally very intensive. AP- 180/90 mm Hg. What is the most probable reason of
hemodynamic disorders?
A. III degree atrioventricular heart block*
B. I degree atrioventricular heart block
C. Bradysystolic form of the atrial fibrillation
D. Sinus bradycardia
E. Complete block of the left branch of His bundle

146. A 32 y.o. patient who has been staying in a hospital on account of acute abscess of his right
lung suddenly felt pain after coughing in the right half of thorax, he got heavy breathing,
cyanosis. What complication is the most probable?
A. Pyopneumothorax*
B. Infarction-pneumonia
C. Myocardial infarction

31
Завантажено з сайту https://тестування.укр/ - онлайн тестування КРОК

D. Esophagus perforation
E. Exudative pleurisy

147. Fluorography of a 45 y.o. man revealed some foci of small intensity with nondistinct outlines
on the top of his right lung. The patient doesn’t feel worse. He has been smoking for many
years. Objectively: vesicular resonance over lungs, respiration is vesicular, rales are absent.
Blood count is not changed. What is the most probable diagnosis?
A. Focal pulmonary tuberculosis*
B. Peripheral cancer of lung
C. Eosinophilic pneumonia
D. Bronchiolitis
E. Disseminated pulmonary tuberculosis

148. An 18 y.o. patient was admitted to the hematologic department with complaints of headache,
general weakness, poor appetite, body temperature rise up to 390C, neck swelling.
Objectively: skin and mucous membranes are extremely pale, lymph nodes on the both sides of
neck are up to 1 cm large, painless. Liver is enlarged +1 cm, painless, spleen +0,5 cm, t0-
380C. Blood count: Hb- 98g/L, RBC-2,9 * 1012/L, leukocytes - 32 * 109/L, stab neutrophils -
0%, segmental leukocytes - 28%, monocytes - 2%, lymphocytes -39%, blasts - 31%,
reticulocytes - 31%, thrombocytes - 120* 109/L, ESR- 36 mm/h. What form of leukosis does the
patient have?
A. Acute lymphoblastic leukosis*
B. Acute myeloblastic leukosis
C. Chronic lympholeukosis
D. Chronic myeloleukosis
E. Undifferentiated leukosis

149. A mother of a 5 y.o. girl consulted a doctor about doughter’s involuntary urination at night,
nightmares, sleep disorders, slow gaining of body weight. Objectively: malnutrition,
intellectual development is good, the girl can read and explains common situations quite
adultly. Her skin is very pale, liver is enlarged in size. Her mother suffers from holetithi-asis.
What type of diathesis is the most probable in the child’s case?
A. Gouty diathesis*
B. Urine acid diathesis
C. Exudative diathesis
D. Allergic diathesis
E. Lymphohypoplastic diathesis

150. After manual reposition and application of plaster splint a patient with fractures of forearm
bones had an edema of hand and fingers, he felt pain and lack of sensitivity. What tactics
should the doctor choose?
A. To cut the bandage that fastens the splint*
B. To prescribe analgetics and diuretics
C. To remove the plaster
D. It’s a natural phenomena, the edema will dissipate in a day
E. To repeat reposition

32
Завантажено з сайту https://тестування.укр/ - онлайн тестування КРОК

151. A 35 y.o. male patient suffers from chronic glomerulonephritis and has been treated with
hemodialysis for the last 3 years. He has got irregularities in the heart activity, hypotension,
progressive weakness, dyspnea. On ECG: bradycardia, 1st degree atrioventricular block, high
sharpened T-waves. Some time before he had seriously broken the water consumption and
dietary pattern. What is the most likely cause of these changes?
A. Hyperkaliemia*
B. Hyperhydratation
C. Hypokaliemia
D. Hypernatremia
E. Hypocalcemia

152. A 60 y.o. patient complains of pain in interphalangeal joints of hand that gets worse during
working. Objectively: distal and proximal joints of the II-IV fingers are defigured, with
Heberden’s and Bouchard’s nodes, painful, stiff. X-ray picture of joints: joint spaces are
constricted, there are marginal osteophytes, subchondral sclerosis. What is the most probable
diagnosis?
A. Osteoarthritis*
B. Reiter’s disease
C. Bechterew’s disease
D. Rheumatic arthritis
E. Psoriatic arthritis

153. A 42 y.o. man who has been ill with duodenal ulcer for 20 years complains of getting a sense of
heaviness in stomach after meal, foul-smelling eructation, vomiting, weight loss. Objectively:
his state is relatively satisfactory, tissue turgor is diminished. On palpation the belly is soft,
there are no symptoms of peritenium irritation, "splashing sounds"in epigastrium. Defecation -
once in 3 days. What complication corresponds with the patient’s state and described clinical
presentations?
A. Ulcerative pyloric stenosis*
B. Concealed ulcer perforation
C. Stomach cancer
D. Ulcer penetration
E. Chronic pancreatitis

154. An 18 y.o. girl complains of weakness, dizziness, loss of appetite, menorrhagia. There are
many-coloured petechiae on the skin of the upper extremities. Blood test: Hb- 105 g/l; RBC- 3,
2 * 1012/L; C.I.-0,95; thromb.- 20 * 109/L. The sedimentation time according to Lee White is 5';
hemorrhagia duration according to Duke is 8', "pinch and tourniquet"test is positive. What is
the most probable diagnosis?
A. Idiopathic thrombocytopenic purpura*
B. Hemophilia
C. Hemorrhagic diathesis
D. Iron deficiency anemia
E. Marchiafava-Micheli’s disease

155. A 46 y.o. patient complains of skin itch, sweating, especially at night, body temperature rise up
to 38, 6°C. Objectively: chest skin has marks of scratching, supraclavicular lymph nodes are as

33
Завантажено з сайту https://тестування.укр/ - онлайн тестування КРОК

big as a pigeon egg, they are not matted together with skin. What test is the most reasonable?
A. Punction of an enlarged lymph node*
B. Common blood count
C. Plan radiography of thorax
D. Immunogram
E. Whole protein and protein fractions

156. A 32 y.o. patient lives in an endemic echinococcous region. For the last 6 months he has been
having pain in the right hypochondrium, temperature rise. An echinococcus liver affection is
suspected. What type of examination will be the most informative in this case?
A. USI examination*
B. Plan radiography of abdominal cavity
C. Biochemical laboratory analysis
D. Angiography
E. Liver scanning

157. A 30 y.o. primigravida woman has got intensive labor pains every 1-2 minutes that last 50
seconds. The disengagement has started. The perineum with the height of 4 cm has grown
pale. What actions are necessary in this situation?
A. Episiotomy*
B. Perineum protection
C. Perineotomy
D. Vacuum extraction of fetus
E. Expectant management

158. A 50 y.o. patient was admitted to the hospital with complaints of blood in urine. There was no
pain or urination disorders, hematuria has lasted for 3 days. Objectively: kidneys are not
palpable, suprapubic region has no pecularities, external genitals have no pathology. Rectal
examination revealed: prevesical gland is not enlarged. Cytoscopy revealed no changes. What
disease would you think about first of all?
A. Cancer of kidney*
B. Tuberculosis of urinary bladder
C. Varicocele
D. Kidney dystopy
E. Necrotic papillitis

159. A patient complains of skin painfullness and reddness of the right gastrocnemius muscle.
Objectively: body temperature is 38,50C, enlarged and painful inguinal lymph nodes on the
right. Skin of extremity is edematic, hyperemic, covered with eruption in form of vesicles
containing dark fluid; its palpation is painful. There is distinct border between normal and
hyperemic skin. What is the most probable diagnosis?
A. Erysipelas, hemorrhagic form*
B. Anthrax, dermal form
C. Herpetic infection
D. Chickenpox
E. Crus phlegmon

34
Завантажено з сайту https://тестування.укр/ - онлайн тестування КРОК

160. A 1,5 y.o. child fell seriously ill: chill, body temperature rise up to 40,10C, then rapid dropping
to 36, 20C, skin is covered with voluminous hemorrhagic rash and purple cyanotic spots.
Extremities are cold, face features are sharpened. Diagnosis: meningococcosis, fulminant
form, infection-toxic shock. What antibiotic must be used at the pre-admission stage?
A. Soluble Levomycetine succinate*
B. Penicillin
C. Lincomycin
D. Gentamycin
E. Sulfamonometoxin

161. A 41 y.o. patient complains of acute pain in the right side of the thorax and sudden progress of
dyspnea following the lifting of a heavy object. The patient’s condition is grave: lips and
mucous membranes are cyanotic, BR- 28/min, Ps-122 bpm., AP- 80/40 mm Hg. There is
tympanitis on percussion and weakened breathing on auscultaion on the right. S2 sound is
accentuated above pulmonary artery. What is the main urgent measure on the prehospital
stage?
A. Air aspiration from the pleural cavity*
B. Epinephrine introduction
C. Euphilline introduction
D. Call for cardiologic team
E. Oxygen inhalation

162. A 38 y.o. woman suffers from paroxysmal AP rises up to 240/120 mm Hg accompanied by


nausea, vomiting, tachycardia, excessive sweating. During the onset blood is hyperglycemic.
After the onset there is voluminous urination. Kidneys sonography revealed accessory mass
bordering upon the upper pole of the right kidney, presumably it belongs to the adrenal gland.
What laboratory test will allow to make a more precise diagnosis?
A. Estimation of catecholamine and vani-llylmandelic acid excretion with urine*
B. Estimation of insulin and C-peptide content in blood
C. Estimation of glomerular filtration rate
D. Estimation of thyroxin and thyrotropic hormon in blood
E. Estimation of renin content in blood

163. A 48 y.o. farmer was admitted to the hospital with complaints of headache, nausea, vomiting,
cough with sputum, breath shortage, weak sight, sweating, salivation. He was cultivated the
garden with phosphoorganic pesticides. Blood count: RBC- 4,1 * 1012/L, Hb- 136 g/L, C.I.- 0,9,
leukocytes - 13,0 * 109/L, ESR- 17 mm/h. His diagnosis is acute intoxication with
phosphoorganic pesticides. What is the most important diagnostic criterion for this pathology?
A. Low level of choline esterase*
B. Reticulocytosis
C. Leukocytosis
D. Anemia
E. Thrombocytopenia

164. A 74 y.o. female patient complains of pain, abdominal distension, nausea. She suffers from
heart ichemia, post-infarction and atherosclerotic cardiosclerosis. On examination: grave
condition, distended abdomen, abdominal wall fails to take active part in breathing. On

35
Завантажено з сайту https://тестування.укр/ - онлайн тестування КРОК

laparoscopy: some cloudy effusion in abdominal cavity, one of the bowel loops is dark-blue.
What is the most probable diagnosis?
A. Mesenterial vessels thrombosis*
B. Volvulus
C. Acute intestinal obstruction
D. Ichemic abdominal syndrome
E. Erysipelas

165. A 30 y.o. woman is in her second labor that has been lasting for 14 hours. Fetal heartbeats are
muffled, arrhythmic, 100/min. Vaginal examination results: cervical dilatation is complete,
fetal head is close to the exit of small pelvis. Sagittal suture has the direct diameter, small
crown is close to the pubis. What is the further tactics of labor management?
A. Use of obstetrical forceps*
B. Labor stimulation by means of oxitocine
C. Cesarean section
D. Craniodermal forceps
E. Use of cavity forceps

166. A 31 y.o. patient has had mental disorder for a long time. He suffers from insomnia for a long
time. He has got fears, suicidal thoughts, tried to hang himself. His mood is depressed, he
refuses from treatment. What measures are the most expedient for the prevention of suicide?
A. Admission to the mental hospital*
B. Admission to the neurological department
C. Out-patient treatment
D. Psychotherapeutic conversation
E. Strict supervision at home

167. A 63 y.o. woman complains of moti- veless weakness, rapid fatigability, loss of appetite,
aversion to meat. Two days ago she had stomach bleeding. Objectively: temperature - 37,50C,
BR- 20/min, Ps-96/min, AP-110/75 mm Hg. On palpation in epigastrium - pain and muscle
tension. Blood count: Hb- 82 g/L, ESR- 35 mm/h. What examination will allow to make a
diagnosis?
A. Cytologic*
B. Radiography
C. Endoscopy
D. Stomach content examination
E. Coprology

168. A 42 y.o. woman suffers from micronodular cryptogenic cirrhosis. During the last week her
condition has been worsening: she had spasms, consciousness aberration, jaundice. What
examination may account for the worsening of the patient’s condition?
A. Estimation of serum ammonia*
B. Estimation of cholesterol esters
C. Estimation of o-fetoprotein content
D. Estimation of alanine aminotransferase and aspartate aminotransferase
E. Estimation of alkaline phosphatase level

36
Завантажено з сайту https://тестування.укр/ - онлайн тестування КРОК

169. A mother of a newborn child suffers from chronic pyelonephritis. She had acute respiratory
viral disease before the labor. Labor in time, with prolonged period without waters. A child
had erythematous eruption on the 2 day, then there were seropurulent vesicles for about 1cm
large. Nikolsky’s symptom is positive. Erosions have occured after vesicle rupture. The child is
flabby. The temperature is subfebrile. What is the most probable diagnosis?
A. Newborn pemphigus*
B. Vesiculopustulosis
C. Pseudofurunculosis
D. Sepsis
E. Ritter’s dermatitis

170. A 41 y.o. woman has suffered from nonspecific ulcerative colitis for 5 years. On
rectoromanoscopy: evident inflammatory process of lower intestinal parts, pseudopolyposive
changes of mucous membrane. In blood: WBC-9,8* 109/L, RBC- 3,0 *1012/L, sedimentation
rate - 52 mm/hour. What medication provides pathogenetic treatment of this patient?
A. Sulfasalasine*
B. Motilium
C. Vikasolum
D. Linex
E. Kreon

171. A 49 y.o. female patient was admitted to the hospital with acute attacks of headache
accompanied by pulsation in temples, increasing AP up to 280/140 mm Hg.
Pheochromocytoma is suspected. What mechanism of hypertensive atack does this patient
have?
A. Increasing of catecholamines concentration*
B. Increasing of aldosterone level in blood
C. Increasing of plasma renin activity
D. Increasing of vasopressin excretion
E. Increasing of thyroxine excretion

172. A child was born with body weight 3250 g and body length 52 cm. At the age of 1,5 month the
actual weight is sufficient (4350 g), psychophysical development corresponds with the age.
The child is breast-fed, occasionally there are regurgitations. What is the cause of
regurgitations?
A. Aerophagia*
B. Pylorostenosis
C. Pylorospasm
D. Acute gastroenteritis
E. Esophageal atresia

173. A 32 y.o. woman has been suffering for 5 months from pain in lumbar region, low grade fever,
frequent urination. Urine analysis: moderate proteinuria, leukocytes occupy the whole field of
sight, bacteriuria. Blood analysis: leukocytosis, increased ESR. What is the most probable
diagnosis?
A. Chronic pyelonephritis*
B. Acute glomerulonephritis

37
Завантажено з сайту https://тестування.укр/ - онлайн тестування КРОК

C. Chronic glomerulonephritis
D. Acute pyelonephritis
E. Urolithiasis

174. A 2 y.o. boy was admitted to the hospital with weight loss, unstable feces, anorexia, following
the semolina’s introduction (since 5 months). The child is adynamic, flabby, his skin is pale and
dry, subcutaneous fat layer is emaciated. Distended and tensed abdomen, tympanitis on
percussion of the upper part of abdomen, splashing sounds, feces are foamy, of light color,
foul. On coprocytogram: a lot of neutral fat. What is the most probable cause of the disease?
A. Celiakia (celiac disease)*
B. Mucoviscidosis (cystic fibrosis)
C. Intestinal dysbacteriosis
D. Chronic enteritis
E. Disaccharidase insufficiency

175. A child with chronic cardialtis, cardial insufficiency IIA that is being treated with digoxin has
got progressing bradycardia, nausea, vomiting. dizziness, sleep disorders. ECG results:
extrasystole, PQ-0,18. What is the most probable cause of this condition?
A. Cardiac glucosides overdose or intolerance*
B. Pulmonary edema
C. Atrioventricular heart block of the I degree
D. Acute enteric infection
E. Hypokaliemia

176. Examination of a hanged man’s corpse revealed the following: livores mortis disappear when
pressed, restore in 50 seconds, rigor mortis is moderately evident only in masseteric muscles,
neck muscles and fingers. Body temperature is 31,0oC. What is the prescription of death
coming?
A. 6-7 hours*
B. 1-2 hours
C. 16-24 hours
D. 8-10 hours
E. 10-18 hours

177. In course of herniotomy of a 12 y.o. patient doctors revealed a testicle in the hernial sac. What
type of hernia is it?
A. Congenital oblique inguinal hernia*
B. Acquired oblique inguinal hernia
C. Direct inguinal hernia
D. Femoral hernia
E. Obturator hernia

178. Vaginal inspection of a parturient woman revealed: cervix dilation is up to 2 cm, fetal bladder
is intact. Sacral cavity is free, sacral promontory is reachable only with a bent finger, the inner
surface of the sacrococcygeal joint is accessible for examination. The fetus has cephalic
presentation. Sagittal suture occupies the transverse diameter of pelvic inlet, the small

38
Завантажено з сайту https://тестування.укр/ - онлайн тестування КРОК

fontanel to the left, on the side. What labor stage is this?


A. Cervix dilatation stage*
B. Preliminary stage
C. Prodromal stage
D. Stage of fetus expulsion
E. Placental stage

179. Ambulace brought to the hospital a patient with acute respiratory viral infection. The illness
began suddenly with temperature rise up to 39,90C. He complains of headache in
frontotemporal lobes, pain in eyeballs, aching of the whole body, nose stuffiness, sore throat,
dry cough. At home he had a nasal hemorrhage twice. What type of acute respiratory viral
infection is it?
A. Influenza*
B. Adenoviral infection
C. Parainfluenza
D. RS-infection
E. Enterovirus infection

180. A 52 y.o. patient complains of dyspnea caused even by moderate physical activity, cough with
hardly secreted sputum. He has been ill for 12 years. Objectively: BR- 26/min. Lung
examination: tympanitis, diminished vesicular breathing with prolonged expiration,
disseminated dry rales. In the past he was taking only theopecym or ami-nophylline
inravenously. Prescribe him the basic treatment after the exacerbation is suppressed:
A. Atrovent*
B. Alupent
C. Inhacort
D. Tilade
E. Aminophylline

181. Plan radiography of the patient’s abdominal cavity reveals some hemispherical lucent areas
situated above distinct horizontal levels. What is the cause of such X-ray picture?
A. Intestinal obstruction*
B. Perforative ulcer
C. Meteorism
D. Cancer of large intestine
E. Price’s disease

182. A patient of a somatic hospital has got psychomotor agitation as a result of high fever: he tried
to run about the department; thought that some water was running down the walls, he
pretended to see rats and cockroaches on the floor. Claimed he were in a hostel, recognized
his "aquaintances". After introduction of sedative drugs he fell asleep. In the morni- ng he
remembered this condition. What psychopathologic syndrome is it?
A. Delirium*
B. Oneiric syndrome
C. Twilight disorder of consciousness
D. Hallucinatory paranoid syndrome
E. Maniacal syndrome

39
Завантажено з сайту https://тестування.укр/ - онлайн тестування КРОК

183. A 19 y.o. boy was admitted to the hospital with closed abdominal trauma. In course of
operation multiple ruptures of spleen and small intestine were revealed. Ap is falling rapidly, it
is necessary to perform hemotransfusion. Who can determine the patient’s blood group and
rhesus compatibility?
A. A doctor of any speciality*
B. A laboratory physician
C. A surgeon
D. A traumatologist
E. An anaesthesilogist

184. A 20 y.o. patient complains of amenorrhea. Objectively: hirsutism, obesity with fat tissue
prevailing on the face, neck, upper part of body. On the face there are acne vulgaris, on the
skin - striae cutis distense. Psychological and intellectual development is normal.
Gynecological condition: external genitals are moderately hairy, acute vaginal and uterine
hypoplasia. What diagnosis is the most probable?
A. Itsenko-Cushing syndrome*
B. Turner’s syndrome
C. Stein-Levental’s syndrome
D. Shichan’s syndrome
E. Babinski-Froehlich syndrome

185. A man, aged 30, complains of intense pain, reddening of skin, edema in the ankle-joint area,
fever up to 390C. There was acute onset of the illness. In the past there were similar attacks
lasting 5-6 days without residual changes in the joint. The skin over the joint is hyperemic
without definite borders and without infiltrative bank on the periphery. What is the most likely
diagnosis?
A. Gout*
B. Infectional arthritis
C. Rheumatoid arthritis
D. Erysipelatous inflammation
E. Osteoarthritis

186. A 5 y.o. girl was by accident closed in a dark room for several minutes. When the door was
opened, the child was standing motionless in the middle of the room staring at one point, her
face had a look of terror, she didn’t respond to any stimuli. 40 minutes after her state changed
into crying. On the next day she could remember nothing of this incident. What is the most
probable mechanism of this reaction?
A. Psychogenic*
B. Endogenic
C. Exogenously organic
D. Endogenically organic
E. Conditioned reflex

187. A 42 y.o. patient was admitted 3 hours after a trauma with evident subcutaneous emphysema
of the upper part of his body, dyspnea, tachycardia 120/min. X-ray examination revealed no
pneumothorax, significant dilatation of mediastinum to the both sides. What emergency care is
needed?

40
Завантажено з сайту https://тестування.укр/ - онлайн тестування КРОК

A. Drainage of anterior mediastinum*


B. Pleural cavity punction
C. Pleural cavity drainage
D. Toracoscopy
E. Toracotomy

188. An 8 y.o. boy was ill with B hepatitis one year ago. In the last 2 months he has complaints of
undue fatiguability, sleep disorder, appetite loss, nausea, especially in the mornings. Skin isn’t
icterious, liver and spleen are 1 cm below the costal margins, painless. Alanine
aminotransferase activity is 2,2 mcmol/L. How can this condition be estimated?
A. Development of chronic hepatitis*
B. Recurrance of viral hepatitis type B
C. Biliary dyskinesia
D. Residual effects of old viral hepatitis type B
E. Development of liver cirrhosis

189. A 2,5 m.o. child has got muscle hypotony, sweating, occipital alopecia. Along with massage
and therapeutic exercises the child was prescribed vitamin D. What dosage and frequency are
correct?
A. 3000 IU every day*
B. 500 IU every day
C. 1000 IU every day
D. 500 IU every other day
E. 1000 IU every other day

190. After delivery and revision of placenta there was found the defect of placental lobe. General
condition of woman is normal, uterus is firm, there is moderate bloody discharge. Inspection of
birth canal with mirrors shows absence of lacerations and raptures. What action is nesessary?
A. Manual exploration of the uterine cavity*
B. External massage of uterus
C. Use of uterine contracting agents
D. Urine drainage, cold on the lower abdomen
E. Use of hemostatic medications

191. A 25 y.o. patient complains of body temperature rise up to 370C, pain at the bottom of her
abdomen and vaginal discharges. Three days ago, when she was in her 11th week of
pregnancy, she had an artificial abortion. Objectibely: cervix of uterus is clean, uterus is a
little bit enlarged in size, painful. Appendages cannot be determined. Fornixes are deep,
painless. Vaginal discharges are sanguinopurulent. What is the most probable diagnosis?
A. Postabortion endometritis*
B. Hematometra
C. Pelvic peritonitis
D. Postabortion uterus perforation
E. Parametritis

192. A 67 y.o. patient complains of dyspnea, breast pain, common weakness. He has been ill for 5

41
Завантажено з сайту https://тестування.укр/ - онлайн тестування КРОК

months. Objectively: t0- 37, 30C, Ps- 96/min. Vocal tremor over the right lung cannot be
determined, percussion sound is dull, breathing cannot be auscultated. In sputum: blood
diffusively mixed with mucus. What is the most probable diagnosis?
A. Cancer of lung*
B. Macrofocal pneumonia
C. Bronchoectatic disease
D. Focal pulmonary tuberculosis
E. Exudative pleuritis

193. People who live in the radiation polluted regions are recommended to include pectins into
their dietary intake for the radioactive nuclides washout. What products are the main source of
pectins?
A. Fruit and vegetebles*
B. Bread
C. Milk
D. Meat
E. Macaroni

194. A 25 y.o. pregnant woman in her 34th week was taken to the maternity house in grave
condition. She complains of headache, visual impairment, nausea. Objectively: solid edemata,
AP- 170/130 mm Hg. Suddenly there appeared fibrillary tremor of face muscles, tonic and
clonic convulsions, breathing came to a stop. After 1,5 minute the breathing recovered, there
appeared some bloody spume from her mouth. In urine: protein - 3,5 g/L. What is the most
probable diagnosis?
A. Eclampsia*
B. Epilepsy
C. Cerebral hemorrhage
D. Cerebral edema
E. Stomach ulcer

195. The Transcarpathian region is characterized by constant high (over 80%) air moisture.
Population of this region feels an intense cold in winter when the temperature is temperately
low. What way of heat emission becomes more active?
A. Convection*
B. Irradiation
C. Evaporation
D. Conduction
E. Radiation

196. A 51 y.o. patient complains of having intensive bloody discharges from vagina for 15 days after
delay of menstruation for 2,5 months. In anamnesis: disorders of menstrual function during a
year, at the same time she felt extreme irritability and had sleep disorders. US examination
results: uterus corresponds with age norms, appendages have no pecularities, endometrium is
14 mm thick. What is the doctor’s tactics?
A. Diagnostic curettage of uterine cavity*
B. Conservative treatment of bleeding
C. Hysterectomy

42
Завантажено з сайту https://тестування.укр/ - онлайн тестування КРОК

D. Supravaginal amputation of uterus without appendages


E. TORCH-infection test

197. An 18 y.o. patient complains of painfulness and swelling of mammary glands, headaches,
irritability, edemata of lower extremities. These symptoms have been present since the begin
of menarche, appear 3-4 days before regular menstruation. Gynecological examination
revealed no pathology. What is the most probable diagnosis?
A. Premenstrual syndrome*
B. Neurasthenia
C. Renal disease
D. Mastopathy
E. Disease of cardiovascular system

198. A 15 y.o. patient has a developmental lag, occasionally he has skin yellowing. Objectively:
spleen is 16x12x10 cm, cholecystolithiasis, skin ulcer of the lower third of left crus. Blood
count: RBC- 3,0 * 1012/L, Hb- 90 g/L, C.I.-1,0; microspherocytosis, reticulocytosis. Total serum
bilirubin is 56 mcmol/L, unconjugated - 38 mcmol/L. What therapy will be the most
appropriate?
A. Splenectomy*
B. Spleen transplantation
C. Portacaval shunt
D. Omentosplenopexy
E. Omentohepatopexy

199. In a city with population 400000 people 5600 fatal cases were recorded, including 3300 cases
because of blood circulation diseases, 730 - because of tumors. What index will allow to
characterize mortality from blood circulation diseases in this city?
A. Intensive index*
B. Extensive index
C. Relative intensity index
D. Visuality index
E. Correlation index

200. During the medical examination a port crane operator complained of dizziness, nausea, sense
of pressure against tympanic membranes, tremor, dyspnoea, cough. He works aloft, the work
is connected with emotional stress. Workers are affected by vibration (general and local),
noise, ultrasound, microclimate that warms in summer and cools in winter. What factor are the
worker’s complaints connected with?
A. Infrasound*
B. Noise
C. Vibration
D. Intensity of work
E. Altitude work

43
Завантажено з сайту https://тестування.укр/ - онлайн тестування КРОК

ТЕСТУВАННЯ.УКР Бази тестів

Буклет 2007 року

Цей тест можна пройти в режимі онлайн тестування на сайті https://тестування.укр/testkrok/studing/566

Це офіційні тести з сайту Центру тестування https://www.testcentr.org.ua/

1. An 8 y.o. child presents with low-grade fever, arthritis, colicky abdominal pain, and a purpuric
rash limited to the lower extremities. laboratory studies reveal a guaiac-positive stool, a
urinalysis with red blood cell (RBC) casts and mild proteinuria, and a normal platelet count.
The most likely diagnosis is:
A. Henoch-Schonlein’s vasculitis*
B. Systemic lupus erythematosus (SLE)
C. Rocky Mountain spotted fever
D. Idiopathic thrombocytopenic purpura
E. Poststreptococcal glomerulonephritis

2. A 32 y.o. woman consulted a gynecologist about having abundant long menses within 3
months. Bimanual investigation: the body of the uterus is enlarged according to about 12
weeks of pregnancy, distorted, tuberous, of dense consistence. Appendages are not palpated.
Histological test of the uterus body mucosa: adenocystous hyperplasia of endometrium.
Optimal medical tactics:
A. Surgical treatment*
B. Hormonetherapy
C. Phytotherapy
D. Radial therapy
E. Phase by phase vitamin therapy

3. A woman was hospitalised with full-term pregnancy. In survey: the uterus is morbid, the
abdomen is tense, heart sounds of the fetus are not auscultated. What is the most probable
complication of pregnancy?
A. Premature detachment of the normally posed placenta*
B. Preterm labour
C. Back occipital presentation
D. Acute hypoxia of a fetus
E. Hydramnion

4. By the end of the 1st period of physiological labour the clear amniotic waters were given vent.
Contractions lasted 35-40 sec every 4-5 min. Palpitation of the fetus is 100 bpm. The AP is
140/90 mm Hg. Diagnosis:
A. Acute hypoxia of the fetus*
B. Labors before term
C. Premature detachment of normally posed placenta
D. Back occipital presentation
E. Hydramnion

5. A 51 y.o. women was taken to the emergency department in convulsive status epilepticus. The

1
Завантажено з сайту https://тестування.укр/ - онлайн тестування КРОК

first means of medi- cal management should be:


A. Ensuring that the airway is open and the patient is oxygenating*
B. Inserting a tongue blade
C. Administering an instravenous bolus of 50% dextrose
D. Injecting 5 mg of diazepam followed by a loading dose of phenytoin
E. Inducing pentobarbital coma

6. Which gestational age gives the most accurate estimation of weeks of pregnancy by uterine
size?
A. Less that 12 weeks*
B. Between 12 and 20 weeks
C. Between 21 and 30 weeks
D. Between 31 and 40 weeks
E. Over 40 weeks

7. A number of viable fetuses per 1000 women at the age between 15 and 44 is determined by:
A. Genital index*
B. Reproductive level
C. Birth rate
D. Perinatal rate
E. Obstetric rate

8. A 21 y.o. man complains of having morning pains in his back for the last three months. The
pain can be relieved during the day and after physical exercises. Physical examination
revealed reduced mobility in the lumbar part of his spine, increase of muscle tonus in the
lumbar area and sluch during moving. X-ray pattern of spine revealed bilateral sclerotic
changes in the sacrolumbal part. What test will be the most necessary for confirming a
diagnosis?
A. HLA-B27*
B. ESR
C. Rheumatoid factor
D. Uric acid in blood plasma
E. Antinuclear antibodies

9. A 44 y.o. man has acute disarthria, right-sided Horner’s syndrome, hiccup, right-sided ataxia,
loss of pain sensation of his face to the right and of his body to the left. The man is conscious.
Computer tomography of brain is normal. The most reasonable measure will be:
A. Introduction of direct coagulants and observation*
B. Endarterectomy of the right carotid artery
C. Endarterectomy of the left carotid artery
D. Out-patient observation
E. Surgical cerebral decompression

10. A 65 y.o. man who has problems with urination as a result of benign prostate gland adenoma
dveloped fever and chill, hypotension, sinus tachycardia. Skin is warm and dry. Clinical blood
analysis revealed absolute granulocytopenia. These hemodynamic changes are most likely to

2
Завантажено з сайту https://тестування.укр/ - онлайн тестування КРОК

be caused by:
A. Endotoxemia with activation of complement system*
B. Secondary reflex vasodilatation as a result of lowered cardiac output
C. Secondary circulation insufficiency with retained systolic function as a result of
peripheral vasoconstriction
D. Reflex vagus stimulation with lowered cardiac output
E. Secondary endothelial changes as a result of bacterial lesion

11. A woman in her 39-th week of pregnancy, the second labor, has regular birth activity. Uterine
contractions take place every 3 minutes. What criteria describe the beginning of the II labor
stage the most precisely?
A. Cervical dilatation no less than 4 cm*
B. Cervical smoothing over 90%
C. Duration of uterine contractions more than 30 seconds
D. Presenting part is in the lower region of small pelvis
E. Rupture of membranes

12. A 20 y.o. man has a stab knife wound in the left half of thorax close to nipple. AP is 90/60 mm
Hg, Ps- 130/min, BR- 32/min. During inspiration there is increase of pulse wave in the region
of jugular vein, decrease of peripheral arterial pulse and reduction of AP Respiratory murmurs
are unchanged. X-ray pattern of thorax organs has no pecularities. After introduction of 2 l of
isotonic solution the AP stayed low, CVP raised up to 32 cm of water column. The first step in
further treatment of the patient will be:
A. Echocardiogram*
B. Catheterization of left pleural cavity while the outer end of catheter is submerged in
water
C. Continued parenteral introduction of fliud in order to raise AP
D. Introduction of peripheral vasodilatators in order to reduce CVP
E. Introduction of loop diuretics in order to reduce CVP

13. A young man has painful indurations in the peripapillary regions of both mammary glands. The
most reasonable action will be:
A. To leave these indurations untouched*
B. To remove them
C. To cut and drain them
D. To take an aspirate for bacterial inoculation and cytology
E. To administer steroids locally

14. A 34 y.o. woman in her 29-th week of pregnancy, that is her 4-th labor to come, was admitted
to the obstetric department with complaints of sudden and painful bloody discharges from
vagina that appeared 2 hours ago. The discharges are profuse and contain grumes. Cardiac
funnction of the fetus is rhytmic, 150 strokes in the minute, uterus tone is normal. The most
probable provisional diagnosis will be:
A. Placental presentation*
B. Detachment of normally located placenta
C. Vasa previa
D. Bloody discharges

3
Завантажено з сайту https://тестування.укр/ - онлайн тестування КРОК

E. Disseminated intravascular coagulation syndrome

15. A patient is 65 y.o. He has been a smoker for 40 years. Hew has lost 10 kg during the last 3
months. Complains of pain in the epigastric area after taking meals, diarrhea, jaundice.
Physical examination revealed enlarged, painless gallbladder. Feces are light-coloured and
clay-like. Blood analysis revealed increased level of whole and direct bilirubin, alkaline
phosphotase and glutami-nepyruvate transferase. Clinical urine analysis showed positive
bilirubin reaction and negative urobilinogene reaction. Where is the initial process that caused
these changes?
A. In pancreas*
B. In common bile duct
C. In liver
D. In duodenum
E. In gallbladder

16. A 75 y.o. man has acute pain in the paraumbilical region accompanied by vomiting and feeling
of abdominal swelling in approximately 30 minutes after meals. He lost 10 kg during the last
months because he doesn’t eat in order to avoid pain. Abdomen examination reveals no
changes in the periods between pain attacks. Above the right femoral artery a murmur can be
auscultated, peripheral pulsation in the lower extrimities is weak. X-ray examination of
stomach and colonoscopy reealed no changes. What is the leading factor of this pathogenesis?
A. Ischemia*
B. Psychogenic changes
C. Neoplastic process
D. Inflammation
E. Transient obstruction

17. A 75 y.o. woman with coronary heart disease constantly takes warfarin. She was taken to the
acute care department with complaints of sudden sensation of weakness in the left half of her
body and eyeball deviation to the left. What examination of the patient will be a primary task?
A. Computer tomography of brain*
B. Magnetic resonance tomography of brain
C. Electroencephalogram
D. Ultrasonic examination of carotid arteries
E. Spinal punction

18. A 65 y.o. patient has acute pain, paresthesia, paleness of his left extremity. Pulse in the a.
dorsalis pedis is absent. There is skin coldness and paleness that gradually spreads upwards.
These symptoms are most likely to be the evidence of:
A. Arterial occlusion*
B. Thrombophlebitis of superficial veins
C. Hernia of lumbar disc
D. Thrombophelebitis of deep veins
E. -

19. A 58 y.o. patient developed acute myocardium infarction 4 hours ago, now he is in the acute

4
Завантажено з сайту https://тестування.укр/ - онлайн тестування КРОК

care department. ECG registers short paroxysms of ventricular tachycardia. The most
appropriate measure will be to introduct:
A. Lidocain*
B. Flecainid
C. Amyodaron
D. Propafenone
E. Veropamil

20. A 23 y.o. woman who suffers from insulin-dependent diabetes was admitted to the acute care
department with mental confusion, inadequate anxious behaviour, hyperhidrosis, excessive
salivation, tachycardia. What examination will be a primary task?
A. Blood test for sugar*
B. Clinical blood analysis
C. Plasma electrolytes test
D. Gaseous composition of arterial blood
E. Blood urea and creatinine test

21. A 40 y.o. woman has changes of mammary gland. What are the most often symtomps that
precede the malignizati-on?
A. Skin induration with inverted nipple*
B. Painful movable induration
C. Painless movable induration
D. Bloody discharges from the nipple
E. Pure discharges from the nipple

22. A patient who takes diuretics has developed arrhythmia as a result of cardiac glycoside
overdose. What is the treatment tactics in this case?
A. Increased potassium concentration in blood*
B. Increased sodium consentration in blood
C. Reduced magnesium concentration in blood
D. Increased calcium level in blood
E. -

23. Name a statistical observation unit for determination of influence amount of bloodsugar on the
healing of wound’s surface in a postoperative period:
A. The patient in a postoperative period*
B. An amount of bloodsugar
C. Blood analysis
D. The patient who has a wound surface
E. The patient who was discharged on an after-care

24. The parameter of infantile mortality for the last year was - 16,3, in present year -15,7 Name a
kind of the diagram that can be used for a graphic representation of it:
A. Stylar*
B. Linear
C. Intrastylar

5
Завантажено з сайту https://тестування.укр/ - онлайн тестування КРОК

D. Sector
E. Radial

25. A 25 y.o. patient complains of pain in the I finger on the right hand. On examination: the finger
is homogeneously hydropic, in bent position. On attempt to unbend the finger the pain is
getting worse. Acute pain appears during the probe in ligament projection. What decease is
the most likely?
A. Thecal whitlow (ligament panaritium)*
B. Subcutaneous panaritium
C. Articular (joint) panaritium
D. Bone panaritium
E. Paronychia

26. A 9 y.o. child with diagnosis "chronic tonsillitis"stands dispanserization control. Within 1 year
of observation there was one exacerbation of disease. Physical condition is satisfactory. The
general state is not infringed. Define group of health:
A. III (a)*
B. II-d
C. I-st
D. III (b)
E. III (c)

27. An engineer-chemist at the age of 47 often fells ill with an occupational skin disease. Who
makes a decision to transfer him to other job accepts?
A. DCC*
B. A head physician
C. The attending physician
D. The chief of shop
E. MSEC

28. A 5 tons milk batch was sampled. The lab analysis revealed: fat content 2%, specific density
-1,04 g/cm3, acidity - 21°T, reductase probe - weak-positive. What way is the product to be
used in?
A. Sell but inform customers about milk quality*
B. Discard for animal feeding
C. Technical utilization
D. Sell without limitations
E. Do the product away

29. An anestesiologist gives narcosis to the patient, he uses a non-reversive contour. Anesthetic is
halothane. Air temperature in the operation room is 21°C, humidity 50%, level of noise 30 dB.
What occupational hazard is the principal one under these conditions?
A. Air pollution with anesthetic*
B. Improper occupational microclimate
C. High level of noise
D. Mental overfatigue

6
Завантажено з сайту https://тестування.укр/ - онлайн тестування КРОК

E. Compelled working pose

30. What guarantees against the preconceived attitude to the physician in cases of professional
law violations do you know?
A. Sanction of public prosecutor, inquiry by preliminary investigator of prosecutor’s office,
committee of experts*
B. Draw up a statement about forensic medical examination
C. Conduct an inquiry by preliminary investigator of police department
D. Utilisation copy of medical documents
E. Conduct forensic medical examination by district forensic medicine expert

31. A 34 y.o. patient 3 hours ago was bitten by a dog. He has got a non-bleeding wound in his left
arm caused by the dog’s bite. What surgical care would you provide to the patient?
A. Wound bathing with detergent water and antiseptic application*
B. Aseptic bandage
C. Cream bandage
D. Complete suturing of the wound
E. Incomplete suturing of the wound

32. A 37 y.o. patient complains of pain in the right arm which increases during motion, raised body
temperature up to 39°C. In the right cubital fossa there is a trace of injection, hyperemia and
thickening along the vein. Your diagnosis?
A. Phlebit*
B. Phlegmon
C. Abscess
D. Inflammation of lymph
E. Erysipelas

33. A 38 y.o. woman was hospitalized to the surgical unit with acute abdominal pain irradiating to
the spine and vomiting. On laparocentesis hemmorhagic fluid is obtained. What disease is
suspected?
A. Acute pancreatitis*
B. Renal colic
C. Acute enterocolitis
D. Perforative gastric ulcer
E. Acute appendicitis

34. A 40 weeks pregnant woman in intrinsic obstetric investigation: the cervix of a uterus is
undeveloped. The oxytocin test is negative. Upon inspection at 32 weeks it is revealed: AP-
140/90 mm Hg, proteinuria 1 g/l, peripheric edemata. Reflexes are normal. Choose the most
correct tactics of guiding the pregnant:
A. Laborstimulation after preparation*
B. Strict bed regimen for 1 month
C. Complex therapy of gestosis for 2 days
D. Cesarean section immediately
E. Complex therapy of gestosis for 7 days

7
Завантажено з сайту https://тестування.укр/ - онлайн тестування КРОК

35. A 41 y.o. patient was admitted to the intensive care unit with hemorrhagic shock due to gastric
bleeding. He has a history of hepatitis B during the last 5 years. The source of bleeding are
esophageal veins. What is the most effective method for control of the bleeding?
A. Introduction of obturator nasogastric tube*
B. Intravenous administration of pituitrin
C. Hemostatic therapy
D. Operation
E. Administration of plasma

36. A woman had the rise of temperature up to 390C on the first day after labour. The rupture of
fetal membranes took place 36 hours before labour. The investigation of the bacterial flora of
cervix of the uterus revealed hemocatheretic streptococcus of group A. The uterus body is soft,
tender. Discharges are bloody, mixed with pus. Specify the most probable postnatal
complication:
A. Metroendometritis*
B. Thrombophlebitis of pelvic veins
C. Infected hematoma
D. Infection of the urinary system
E. Apostatis of junctures after the episi-otomy

37. A 24 y.o. patient 13 months after the first labour consulted a doctor about amenorrhea.
Pregnancy has concluded by a Cesarean section concerning to a premature detachment of
normally posed placenta hemorrhage has made low fidelity 2000 ml owing to breakdown of
coagulability of blood. Choose the most suitable investigation:
A. Determination of the level of Gonadotropins*
B. USI of organs of a small pelvis
C. Progesteron assay
D. Computer tomography of the head
E. Determination of the contents of Testosteron-Depotum in Serum of blood

38. For the persons who live in a hot area after an accident at a nuclear object, the greatest risk
within the first decade is represented by cancer of:
A. Thyroid gland*
B. Skin
C. Reproduction system organs
D. Breast
E. Lungs

39. A 34 y.o. woman in the 10-th week of gestation (the second pregnancy) consulted a doctor of
antenatal clinic with purpose of statement on the dyspensary record. In the previous
pregnancy there took place hydramnion, the child’s birth weight was 4086. What method of
examination is necessary for carrying out, first of all?
A. The test for tolerance to glucose*
B. Determination of the contents of fetoproteinum
C. Bacteriological investigation of discharge from the vagina
D. A cardiophonography of fetus
E. USI of the fetus

8
Завантажено з сайту https://тестування.укр/ - онлайн тестування КРОК

40. Patient 22 y.o., was admitted to trauma center with complains of pain in the left ankle joint,
which increased while movements and weight bearing. On the clinical examination it was
found, that the patient had the closed fracture of medial malleolus without displacement. In
which position the foot has to be fixed in plaster cast?
A. At right angle with varus positioning of the foot*
B. In position of planter flexion of foot
C. In position of pronation
D. In position of supination
E. In position of dorsal flexion of foot

41. The 10 y.o. boy has complains on headache, weakness, fever 400C, vomiting, expressed
dyspnea, pale skin with flush on right cheek, lag of right hemithorax respiratory movement,
dullness on percussion over low lobe of right lung, weakness of vesicular respiration in this
zone. The abdomen is painless and soft at palpati- on. Which disease lead to these symptoms
and signs?
A. Pneumonia croupousa*
B. Intestinal infection
C. Acute appendicitis
D. Acute cholecystitis
E. Flu

42. The patient with acute respiratory viral infection (3-rd day of disease) has complaints on pain
in lumbar region, nausea, dysuria, oliguria. Urinalysis -hematuria (100-200 RBC in eyeshot
spot), specific gravity - 1002. The blood creatinin level is 0,18 mmol/L, potassium level - 6,4
mmol/L. Make the diagnosis:
A. Acute interstitial nephritis*
B. Acute renal failure
C. Acute glomerylonephritis
D. Acute cystitis
E. Acute renal colic

43. The 7 m.o. infant is suffering from acute pneumonia which was complicated by cardiovascular
insufficiency and respiratory failure of II degree. The accompanied diagnosis is malnutrition of
II degree. Choose the best variant of therapy:
A. Ampiox and Amicacin*
B. Macropen and Penicillin
C. Penicillin and Ampiox
D. Gentamycin and Macropen
E. Ampiox and Polymixin

44. A 2 y.o. girl has been ill for 3 days. Today she has low grade fever, severe catarrhal
presentations, slight maculopapular rash on her buttocks and enlarged occipital lymph nodes.
What is your diagnosis?
A. Rubella*
B. Scarlet fever
C. Measles
D. Adenoviral infection

9
Завантажено з сайту https://тестування.укр/ - онлайн тестування КРОК

E. Pseudotuberculosis

45. A 7 y.o. girl has mild form of varicella. Headache, weakness, vertigo, tremor of her limbs,
ataxia, then mental confusion appeared on the 5th day of illness. Meningeal signs are negative.
Cerebrospinal fluid examination is normal. How can you explain these signs?
A. Encephalitis*
B. Meningitis
C. Meningoencephalitis
D. Myelitis
E. Neurotoxic syndrome

46. A 7 y.o. girl fell ill abruptly: fever, headache, severe sore throat, vomiting. Minute bright red
rash appear in her reddened skin in 3 hours. It is more intensive in axillae and groin. Mucous
membrane of oropharynx is hyperemic. Greyish patches is on the tonsills. Submaxillary lymph
nodes are enlarged and painful. What is your diagnosis?
A. Scarlet fever*
B. Measles
C. Rubella
D. Pseudotuberculosis
E. Enteroviral infection

47. The child has complains of the "ni-ght"and "hungry"abdominal pains. At fi-broscopy in area a
bulbus ofa duodenum the ulcerrative defect of 4 mms diameter is found, the floor is obtected
with a fibrin, (H.p +). Administer the optimum schemes of treatment:
A. Omeprasole - Trichopolum - Clari-tromicin*
B. De-nol
C. Maalox - Ranitidin
D. Vicalinum - Ranitidin
E. Trichopolum

48. 6 m.o. infant was born with body’s mass 3 kg and length 50 cm. He is given natural feeding.
How many times per day the infant should be fed?
A. 5*
B. 7
C. 6
D. 8
E. 4

49. A 40 y.o. patient with rheumatic heart disease complains of anorexia, weakness and loss of
weight, breathlessness and swelling of feet. The patient had tooth extraction one month ago.
On examination: t0- 390C, Ps- 100/min. Auscultation: diastolic murmur in the mitral area.
Petechial lesion around the clavicle; spleen was palpable.
A. Subacute bacteria endocarditis*
B. Recurrence of rheumatic fever
C. Thrombocytopenia purpura
D. Mitral stenosis

10
Завантажено з сайту https://тестування.укр/ - онлайн тестування КРОК

E. Aortic stenosis

50. A patient with nosocomial pneumonia presents signs of collapse. Which of the following
pneumonia complications is most likely to be accompanied with collapse?
A. Septic shock*
B. Exudative pleuritis
C. Bronchial obstruction
D. Toxic hepatitis
E. Emphysema

51. A 38 y.o. patient has been treated in a hospital. A fever of 390C, chest pain which is worsened
by breathing, cough, brownish sputum appeared on the 7-th day of the treatment. Chest X- ray
shows left lower lobe infiltrate. Which of the following is the treatment of choice for this
patient?
A. Cephalosporins of the III generation*
B. Penicillin
C. Erythromycin
D. Tetracycline
E. Streptomycin

52. A 62 y.o. patient suffers from DM-2. Diabetes is being compensated by diet and Maninilum.
Patient has to undergo an operation on inguinal hernia. What tactics of hypoglycemic therapy
should be used?
A. Prescribe fast-acting insulin*
B. Give Glurenorm in place of Maninilum
C. Continue with the current therapy
D. Prescribe long-acting insulin
E. Prescribe guanyl guanidines

53. A patient of 32 y.o. complains of severe weakness, tremor of extremities. Objective


examination: body weight loss, wet and warm skin. The thyroid gland is enlarged up to the 3-
rd degree, painless, elastic. Ps-108/min. BP- 160/55 mm Hg. There are no other abnormalties.
The diagnosis is:
A. Diffuse toxic goiter of the 3-rd degree, thyrotoxicosis of the average degree*
B. Diffuse euthyroid goiter of the 3-rd degree
C. Chronic autoimmune thyroiditis, hypertrophic type
D. Chronic fibrous thyroiditis
E. Toxiferous adenoma of the thyroid gland

54. A triad of symptoms ("stearing spot", "terminal film", "blood dew") have been revealed on
examination of a patient. What disease should you think about?
A. Psoriasis*
B. Lichen ruber planus
C. Vasculitis
D. Seborrhea
E. Ritter’s disease

11
Завантажено з сайту https://тестування.укр/ - онлайн тестування КРОК

55. Patient 27 y.o. was hospitalized to the psychiatric hospital for the 4-th time during 2 years.
Heard voices commenting on his actions, had delusions of persecution (was sure that the
Mafia wanted to kill him). After a course of treatment with neuroleptics was discharged from
hospital with the diagnosis of schizophrenia, state of remission. The secondary prevention of
the relapses of schizophrenia requires:
A. Supportive treatment with neuroleptics of prolonged action*
B. Long-term hospitalization
C. Psychiatric observation
D. Participation in a self-help group
E. Psychoanalytic treatment

56. A 42 y.o. woman works at the factory on the fabrication of mercury thermometers, complains
of the headache, swoons, reduction of memory, small and frequent flutter of fingers of drawn
hands, the eyelids and the tongue, bleeding gums, gingivitis. What preparation is it nessesary
to use for the elimination of mercury from the organism?
A. Unithiol*
B. Pentoxil
C. Magnesium sulphate
D. Sodium hydrate of carbon
E. Seduxen

57. The disease began acutely. The frequent watery stool developed 6 hours ago. The body’s
temperature is normal. Then the vomiting was joined. On examination: his voice is hoarse,
eyes are deeply sunken in the orbits. The pulse is frequent. Blood pressure is low. There is no
urine. What is the preliminary diagnosis?
A. Cholera*
B. Toxic food-borne infection
C. Salmonellosis
D. Dysentery
E. Typhoid fever

58. A 28 y.o. man fell seriously ill, he feels chill, has got a fever, body temperature raised up to
38,50C, paroxysmal pain in the left iliac region, frequent defecation in form of fluid bloody and
mucous mass. Abdomen palpation reveals painfulness in its left half, sigmoid colon is spasmed.
What is the most probable diagnosis?
A. Acute dysentery*
B. Amebiasis
C. Colibacillosis
D. Nonspecific ulcerative colitis
E. Malignant tumors of large intestine

59. The patient was admitted to the hospital on the 7-th day of the disease with complaints of high
temperature, headache, pain in the muscles, especially in calf muscles. The dermal
integuments and scleras are icteric. There is hemorrhagic rash on the skin. Urine is bloody.
The patient went fishing two weeks ago. What is the diagnosis?
A. Leptospirosis*
B. Yersiniosis

12
Завантажено з сайту https://тестування.укр/ - онлайн тестування КРОК

C. Salmonellosis
D. Brucellosis
E. Trichinellosis

60. A 43 y.o. patient was admitted to the hospital with complaints of high temperature of the body
and severe headache. On examination: carbuncle is revealed on the forearm. There are intense
edema around it, insignificant pain, regional lymphadenitis. The patient is a worker of cattle-
ranch. What disease is it necessary to think about first?
A. Anthrax*
B. Carcinoma of skin
C. Erysipelas
D. Erysipeloid
E. Eczema

61. A16 y.o. female presents with abdominal pain and purpuric spots on the skin. Laboratory
investigations reveals a normal platelet count, with haematuria and proteinuria.The most likely
diagnosis:
A. Schonlein-Henoch purpura*
B. Haemolytic uraemic syndrome
C. Thrombotic thrombocytopenic purpura
D. Heavy metal poisoning
E. Sub acute bacterial endocarditis

62. A 60 y.o. asthmatic man comes for a check up and complains that he is having some difficulty
in "starting to urinate". Physical examination indicates that the man has blood pressure of
160/100 mm Hg, and a slight enlarged prostate. Which of the following medications would be
useful in treating both of these conditions:
A. Doxazosin*
B. Labetalol
C. Phetolamine
D. Propranolol
E. Isoproterenol

63. A 30 y.o. man presents with a history of recurrent pneumonias and a chronic cough production
of foul smelling, pi-rulent sputum, ocassionally glood tinged, which is worse in the morning
and on lying down. On physical examination, the patient appears chronically ill with clubbing
of fingers, inspiratory rales at the base of lungs posteriorly. Most likely diagnosis:
A. Bronchoectasis*
B. Chronic bronchitis
C. Disseminated pulmonary tuberculosis
D. Pulmonary neoplasm
E. Chronic obstructive emphysema

64. The family doctor examined a patient and diagnosed an acute bleeding of an intestine. What is
the doctor’s professional tactics in this situation?
A. The urgent hospitalization in to the surgical department*

13
Завантажено з сайту https://тестування.укр/ - онлайн тестування КРОК

B. Aminocapronic acid intravenously


C. The urgent hospitalization in to tthe herapeutic department
D. Treatment at a day time hospital
E. Treatment at home

65. A 19 y.o. girl was admitted to Emergency Department: unconsionsness state, cyanosis, myotic
pupil, superficial breathing - 12/min. BP- 90/60 mm Hg, Ps-78/min. Choose the action
corresponding to this clinical situation:
A. Controlled respiration*
B. Gastric lavage
C. Oxygen inhalation
D. Caffeine injection
E. Cordiamine injection

66. A 58 y.o. man complaines of severe inspiratory dyspnea and expectoration of frothy and blood-
tinged sputum. He has been suffering from essential hypertension and ischemic heart disease.
On examination: acrocyanosis, "bubbli-ng"breathing, Ps- 30/min, BP- 230/130 mm Hg, bilateral
rales. Choose medicines for treatment.
A. Morphine, furosemide, nitroprusside sodium*
B. Theophylline, prednisolon
C. Albuterol, atropine, papaverine
D. Strophanthine, potassium chloride, plathyphylline
E. Cordiamine, isoproterenol

67. A 41 y.o. woman complains of weakness, fatigue, fever up to 380C, rash on the face skin, pain
in the wrists and the elbows. On physical examination: erythematous rash on the cheeks with
"butterfly"look, the wrists and elbow joints are involved symmetrically, swollen, sensitive,
friction rub over the lungs, the heart sounds are weak, regular, HR-88/min, BP- 160/95 mm
Hg. Hematology shows anemia, leucopenia, lymphopenia; on urinalysis: proteinuria,
leukocyturia, casts. What is the main mechanism of disease development?
A. Production of antibodies to doublestranded DNA*
B. Production of myocytes antibodies
C. Production of antibodies to endothelial cells
D. Production of myosin antibodies
E. Production of antimitochondrial antibodies

68. A 56 y.o. woman has an acute onset of fever up to 390C with chills, cough, and pain on
respiration in the right side of her chest. On physical examination: HR-90/min, BP- 95/60 mm
Hg, RR- 26/min. There is dullness over the right lung on percussion. On X-ray: infiltrate in the
right middle lobe of the lung. What is the diagnosis?
A. Community-acquired lobar pneumonia with moderate severity*
B. Community-acquired bronchopneumonia
C. Acute pleurisy
D. Acute lung abscess
E. Hospital-acquired lobar pneumonia

14
Завантажено з сайту https://тестування.укр/ - онлайн тестування КРОК

69. A 50 y.o. woman for 1 year complained of attacks of right subcostal pain after fatty meal. Last
week the attacks have repeated every day and become more painful. What diagnostic study
would you recommend?
A. Ultrasound examination of the gallbladder*
B. Liver function tests
C. X-ray examination of the gastrointestinal tract
D. Ultrasound study of the pancreas
E. Blood cell count

70. A 25 y.o. woman complained of fatigue, hair loss and brittle nails. The examination revealed
pallor of skin, Ps- 94/min, BP- 110/70 mm Hg. On blood count: Hb-90 g/L, RBC- 3,5- 1012/L,
C.I.- 0,7; ESR- 20 mm/h. Serum iron level was 8,7 mcmol/l. What treatment would you initiate?
A. Ferrous sulfate orally*
B. Iron dextrin injections
C. Vitamin B12 intramuscularly
D. Blood transfusion
E. Packed RBCs transfusion

71. A 38 y.o. woman is seriously ill. She complains of frequent paroxysms of expiratory dyspnea.
The last paroxysm lasted over 12 hours and failed to respond to theophylline. The skin is palish
gray, moist, RR of 26/min. On auscultation, breath sounds are absent over some areas. Your
preliminary diagnosis?
A. Bronchial asthma, status asthmaticus*
B. Chronic obstructive bronchitis
C. Atopic bronchial asthma, respiratory failure of the III degree
D. Bronchiectasis, respiratory failure of the II-III degree
E. Ischemic heart disease, pulmonary edema

72. A 19 y.o. girl admitted to the hospital complained of pain in the knee and fever of 38,60C. She
is ill for 2 weeks after acute tonsillitis. On exam, hyperemia and swelling of both knees,
temperature is 37,40C, HR- 94/min, BP- 120/80 mm Hg, and heart border is displaced to the
left; S1 is weak, systolic murmur is present. Total blood count shows the following: Hb- 120
g/L, WBC- 9,8 - 109/L, ESR of 30 mm/L. ECG findings: the rhythm is regular, PQ = 0,24 sec.
What is a causative agent of the disease?
A. Beta-hemolytic streptococci*
B. Viral-bacterial association
C. Autoimmune disorder
D. Staphylococci
E. Ricchetsia

73. A 42 y.o. woman complains of dyspnea, edema of the legs, and tachycardia during small
physical exertion. Heart borders are displaced to the left and S1 is accentuated, there is
diastolic murmur on apex. The liver is enlarged by 5 cm. What is the cause of heart failure?
A. Mitral stenosis*
B. Mitral regurgitation
C. Tricuspid stenosis
D. Tricuspid regurgitation

15
Завантажено з сайту https://тестування.укр/ - онлайн тестування КРОК

E. Aortic stenosis

74. A 54 y.o. woman complains of increasing fatigue and easy bruising of 3 weeks’ duration.
Physical findings included pale, scattered ecchymoses and petechiae and mild
hepatosplenomegaly. In blood: RBC-2,5 • 1012/L; Hb- 73 g/L; HCT- 20%; PLT-23 • 109/L; and
WBC-162 • 109/L with 82% blasts, that contained Auric rods; peroxidase stain was positive.
What is the most probable diagnosis?
A. Acute leukemia*
B. Chronic leukemia
C. Thrombocytopenia
D. Hemolytic anemia
E. Megaloblastic anemia

75. A 39 y.o. woman complaines of squeezed epigastric pain 1 hour after meal and heartburn. She
had been ill for 2 years. On palpation, there was moderate tenderness in pyloroduodenal area.
Antral gastritis was revealed on gastroscopy. What study can establish genesis of the disease?
A. Revealing of Helicobacter infection in gastric mucosa*
B. Detection of autoantibodies in the serum
C. Gastrin level in blood
D. Examination of stomach secretion
E. Examination of stomach motor function

76. A male, 50 y.o., has a black flat mole on the skin of the leg for 10 years. Since 4 months ago
the shapes of the mole become irregular. What diagnostic methods should be used?
A. Excision biopsy*
B. Fine needle biopsy
C. Smear for microscopic examination
D. Incision biopsy
E. Thermography

77. The highest risk of congenital anomalies probably occurs when human embryos or fetuses are
exposed to ionizing radiation. During which part of gestational period does it occur?
A. 18-45 days after conception*
B. The first 7 days
C. 10-14 days after conception
D. 90-120 days after conception
E. The third trimester

78. A 7 d.o. boy is admitted to the hospital for evaluation of vomiting and dehydration. Physical
examination is otherwise normal except for minimal hyperpigmentation of the nipples. Serum
sodium and potassium concentrations are 120 meq/L and 9 meq/L respectively. The most likely
diagnosis is:
A. Congenital adrenal hyperplasia*
B. Pyloric stenosis
C. Secondary hypothyroidism
D. Panhypopituitarism

16
Завантажено з сайту https://тестування.укр/ - онлайн тестування КРОК

E. Hyperaldosteronism

79. A 7 y.o. boy has crampy abdominal pain and a rash on the back of his legs and buttocks as well
as on the extensor surfaces of his forearms. Laboratory analysis reveals proteinuria and
microhematuria. He is most likely to be affected by:
A. Anaphylactoid purpura*
B. Systemic lupus erythematosus
C. Poststreptococcal glomerulonephritis
D. Polyarteritis nodosa
E. Dermatomyositis

80. A 27 y.o. patient has been having for almost a year fatigue, hyperhi-drosis, heaviness in the left
hypochondri-um, especially after meals. Objectively: spleen and liver enlargement. In blood:
erythrocytes - 3,2 • 1012//, Hb- 100 g/l, colour index - 0,87, leukocutes -100 • 109//, basophils -
7%, eosinophils -5%, myelocytes - 15%, juveniles - 16%, stab neutrophils - 10%,
segmentonuclear leukocytes - 45%, lymphocytes - 2%, monocytes - 0%, reticulocytes - 0,3%,
thrombocytes - 400 • 109//, eSr- 25 mm/h. What is the most probable diagnosis?
A. Chronic myeloleukosis*
B. Chronic lympholeukosis
C. Acute leukosis
D. Erythremia
E. Hepatocirrhosis

81. A 28 y.o. woman comes to the Emergency Room with a slightly reddened, painful "knot", 8 cm
above the medial malleolus. Examination in the standing position demonstrates a distended
vein above and below the mass. There are no other abnormalities on physical examination. The
most likely diagnosis is:
A. Superficial venous thrombosis*
B. Early deep vein thrombosis
C. Insect bite
D. Cellulitis
E. Subcutaneous hematoma

82. A patient with a history of coronary artery disease and atrial fibrillation has the onset of
sudden pain and weakness of the left leg. Examination reveals a cool, pale extremity with
absent pulses below the groin and normal contralateral leg. The most likely diagnosis is:
A. Arterial embolism*
B. Arterial thrombosis
C. Acute thrombophlebitis
D. Cerebrovascular accident
E. Dissecting aortic aneurysm

83. A 30 y.o. man complains of sharp pain in the right ear, hearing loss, high temperature for
three days. Objectively: right ear whispering language - 0,5 m, external ear is intact,
otoscopically - eardrum protrusion, hyperemia and swelling, loss of landmarks. What disease is
it?

17
Завантажено з сайту https://тестування.укр/ - онлайн тестування КРОК

A. Acute purulent otitis media*


B. Acute mastoiditis
C. Chronic secretory otitis media
D. Chronic purulent otitis media
E. Eustachian tube disfunction

84. A 22 y.o. man complains of acute throat pain, increasing upon swallowing during 3 days. Body
temperature 38, 30C, neck lymph nodules are slightly enlarged and painful.
Pharyngoscopically - tonsi-lar hyperemia, enlargement and edema, tonsils are covered by
round yellow fibrinous patches around crypts openings. Beta-haemolytic streptococcus in swab
analysis. What is the diagnosis?
A. Acute membranous tonsilitis*
B. Acute follicular tonsilitis
C. Pharyngeal diphtheria
D. Infectious mononucleosis
E. Pharyngeal candidosis

85. A patient is staying in the hospital with the diagnosis of abdominal typhus. During the 3-d
week from the beginning of the disease the patient stopped keeping diet and confinement to
bed. As a result the body temperature and rapid pulse decreased and melena appeared. What
ki- nd of complications should we think about first of all?
A. Intestinal haemorrhage*
B. Thrombophlebitis
C. Meningitis
D. Nephroso-nephritis
E. Hepatite

86. The patients has sustained blunt trauma to the chest. Which of the following would most likely
be the cause of acute cardiopulmonary collapse?
A. Pneumothorax*
B. Hemothorax
C. Pulmonary contusion
D. Rib fractures
E. Acute adult respiratory distress syndrome (ARDS)

87. A patient has got pain in the axillary area, rise of temperature developed 10 hours ago. On
examination: shaky gait is evident, the tongue is coated with white coating. The pulse is
frequent. The painful lymphatic nodes are revealed in the axillary area. The skin is
erythematous and glistering over the lymphatic nodes. What is the most probable diagnosis?
A. Bubonic plague*
B. Acute purulent lymphadenitis
C. Lymphogranulomatosis
D. Anthrax
E. Tularemia

88. A 56 y.o. patient has worked at the aluminium plant over 20 years. Within 3 last years he has

18
Завантажено з сайту https://тестування.укр/ - онлайн тестування КРОК

got loosening of teeth, bone and joint pains, piercing pains in heart region, vomiting. The
provisional diagnosis is:
A. Fluorine intoxication*
B. Mercury intoxication
C. Lead intoxication
D. Phosphorus intoxication
E. Manganese intoxication

89. A 34 y.o. patient has been suffering from pulmonary tuberculosis for 7 years; he complains of
muscle feebleness, weight loss, diarrheas, increased frequency of urination. Objectively:
hyperpigmentation of skin, gums, internal cheek surfaces. AP is 90/58 mm Hg. Blood count:
erythrocutes - 3,1 • 1012/L, Hb- 95 g/L, C.I.- 0,92; leukocytes - 9,4 • 109/L, eosinophils - 7,
segmentonuclear leukocytes - 45, stab neutrophils - 1, lymphocytes - 40, monocytes - 7, Na+-
115 mmole/L, K+-7,3 mmole/L. What is the preliminary di- agnosis?
A. Primary insufficiency of adrenal cortex*
B. Pheochromocytoma
C. Primary hyperaldosteronism
D. Congenital adrenal hyperplasia
E. Diabetes insipidus

90. 3 weeks ago a patient was ill with tonsillitis. Clinical examination reveals edema, arterial
hypertension, hematuria, proteinuria (1,8 g/per day), granular and erythrocital casts. What is
the preliminary diagnosis?
A. Glomerulonephritis*
B. Cystitis
C. Pyelonephritis
D. Intestinal nephritis
E. Renal amyloidosis

91. A patient complains of feeling heaviness behind his breast bone, periodical sensation of food
stoppage, dysphagy. During the X-ray examination barium contrast revealed a single saccular
outpouching of anterodextral esophagus wall with regular contours and rigidly outlined neck.
What is the most probable diagnosis?
A. Esophageal diverticulum*
B. Cancer of esophagus
C. Hiatal hernia
D. Varix dilatation of esophageal veins
E. Esophageal polyp

92. A 40 y.o. patient complains of yellowish discharges from the vagina. Bimanual examination: no
pathological changes. The smear contains Trichomonas vaginalis and blended flora.
Colposcopy: two hazy fields on the front labium, with a negative Iodum test. Your tactics:
A. Treatment of specific colpitis and with the subsequent biopsy*
B. Diathermocoagulation of the cervix of the uterus
C. Specific treatment of Trichomonas colpitis
D. Cervix ectomy
E. Cryolysis of cervix of the uterus

19
Завантажено з сайту https://тестування.укр/ - онлайн тестування КРОК

93. A 52 y.o. patient with previously functional Class II angina complains of 5 days of intensified
and prolonged retrosternal pains, decreased exercise tolerance. Angina is less responsive to
Nitroglycerinum. Which of the following diagnosis is most likely?
A. IHD. Unstable angina*
B. Cardialgia due to spine problem
C. IHD. Functional Class II angina
D. Myocarditis
E. Myocardial dystrophy

94. A full-term child survived antenatal and intranatal hypoxia, it was born in asphyxia (2-5 points
on Apgar score). After birth the child has progressing excitability, there are also vomiting,
nystagmus, spasms, strabismus, spontaneous Moro’s and Babinsky’s reflexes. What
localization of intracranial hemorrhage is the most probable?
A. Subarachnoid hemorrhage*
B. Small cerebral tissue hemorrhages
C. Subdural hemorrhage
D. Periventricular hemorrhages
E. Hemorrhages into the brain ventricles

95. The patient 25 y.o. was admitted on the 1st day of the disease with complaints of double vision
in the eyes, heavy breathing. The day before the patient ate homemade mushrooms. On
objective examination: paleness, widened pupils, disorder of swallowing, bradycardia,
constipation are marked. What is the diagnosis?
A. Botulism*
B. Yersiniosis
C. Leptospirosis
D. Salmonellosis, gastrointestinal form
E. Lambliasis

96. A healthy 75 y.o. woman who leads a moderately active way of life went through a preventive
examination that revealed serum concentration of common cholesterol at the rate of 5,1
mmol/l and HDL (high-density lipoproteins) cholesterol at the rate of 70 mg/dl. ECG reveals no
pathology. What dietary recommendation is the most adequate?
A. Any dietary changes are necessary*
B. Decrease of cholesterol consumption
C. Decrease of saturated fats consumption
D. Decrease of carbohydrates consumption
E. Increase of cellulose consumption

97. Laparotomy was performed to a 54 y.o. woman on account of big formation in pelvis that
turned out to be one-sided ovarian tumor along with considerable omental metastases. The
most appropriate intraoperative tactics involves:
A. Ablation of omentum, uterus and both ovaries with tubes*
B. Biopsy of omentum
C. Biopsy of an ovary
D. Ablation of an ovary and omental metastases
E. Ablation of omentum and both ovaries with tubes

20
Завантажено з сайту https://тестування.укр/ - онлайн тестування КРОК

98. Heart auscultation of a 16 y.o. boy without clinical symptoms revealed accent of the S II and
systolic murmur above the pulmonary artery. Heart sounds are resonant, rhythmic. What is
the most probable diagnosis?
A. Functional murmur*
B. Stenosis of pulmonary artery valve
C. Insufficiency of pulmonary artery valve
D. Nonclosure of Botallo’s duct
E. Defection of interatrial septum

99. A 74 y.o. patient has been suffering from hypertension for 20 years. He complains of frequent
headache, dizziness, he takes enalapril. Objectively: accent of the SII above aorta, Ps- 84 bpm,
rhythmic, AP- 180/120 mm Hg. What group of hypotensive medications could be additionally
prescribed under consideration of the patient’s age?
A. Thiazide diuretics*
B. Loop diuretics
C. 3-adrenoceptor blockers
D. ^-adrenoceptor blockers
E. Central sympatholytics

100. In treatment and prevention establishments, regardless of their organisational and proprietary
form, the rights of patients should be observed. Which of these rights are the most significant?
A. The right to the protection of the patient’s interests*
B. The right to the free choice
C. The right to the information
D. The right to be heard
E. The right to the protection from incompetence

101. A military unit stopped for 3-day’s rest in inhabited locality after a long march. The sanitary-
epidemiological reconnaissance found several water sources. It is necessary to choose the
source complying with the hygienic standards for drinking water in the field conditions.
A. Artesian well water*
B. Spring water
C. River water
D. Rain water
E. Water from melted snow

102. A 10 y.o. boy with hemophilia has signs of acute respiratory viral infection with fever. What of
the mentioned antifebrile medications are contraindicated to this patient?
A. Acetylsalicylic acid*
B. Analgin
C. Pipolphen
D. Paracetamol
E. Panadol extra

103. A full-term newborn child has a diagnosis newborn’s Rh-factor hemolytic disease. Bilirubin
rate is critical. The child’s blood group is B(III), his mother’s blood group - A(II). The child has

21
Завантажено з сайту https://тестування.укр/ - онлайн тестування КРОК

indication for hemotransfusion. What donor blood must be chosen?


A. Blood group B(III)Rh-*
B. Blood group A(II)Rh-
C. Blood group B(III)Rh+
D. Blood group A(II)Rh+
E. Blood group O(I)Rh-

104. At year-end hospital administration has obtained the following data: annual number of treated
patients and average annual number of beds used for patient’s treatment. What index of
hospital work can be calculated based upon this data?
A. Bed turnover*
B. Bed resources of the hospital
C. Average annual bed occupancy
D. Average duration of patients presence in the hospital
E. Average bed idle time

105. A 52 y.o. patient fell from 3 m height on the flat ground with the right lumbar area. He
complains of pain in this area. There is microhematuria in the urea. Excretory urography
revealed that kidney’s functioning is satisfactory. What is the most probable diagnosis?
A. Kidney’s contusion*
B. Subcapsular kidney’s rupture
C. Multiple kidney’s ruptures
D. Paranephral hematoma
E. Kidney’s abruption

106. A 3 y.o. child with weight deficiency suffers from permanent moist cough. In history there are
some pneumonias with obstruction. On examination: distended chest, dullness on percussion
over the lower parts of lungs. On auscultation: a great number of different rales. Level of
sweat chloride is 80 mmol/L. What is the most probable diagnosis?
A. Mucoviscidosis (cystic fibrosis)*
B. Bronchial asthma
C. Recurrent bronchitis
D. Bronchiectasis
E. Pulmonary hypoplasia

107. A 14 y.o. girl complains of profuse bloody discharges from genital tracts during 10 days after
suppresion of menses for 1,5 month. Similiar bleedings recur since 12 years on the
background of disordered menstrual cycle. On rectal examination: no pathology of the internal
genitalia. In blood: Hb- 70 g/L, RBC- 2, 3 • 1012/L, Ht-20. What is the most probable
diagnosis?
A. Juvenile bleeding, posthemorrhagic anemia*
B. Werlholf’s disease
C. Polycyst ovarian syndrome
D. Hormonoproductive ovary tumor
E. Noncomplete spontaneous abortion

22
Завантажено з сайту https://тестування.укр/ - онлайн тестування КРОК

108. A 43 y.o. patient had cholecystectomy 6 years ago because of chronic calculous cholecystitis.
Lately he has been suffering from pain in the right subcostal area and recurrent jaundice.
Jaundice hasn’t gone for the last 2 weeks. Stenoutic papillitis 0,5 cm long has been revealed.
What is the best way of treatment?
A. To perform endocsopic papillosphincterotomy*
B. To treat conservatively: antibiotics, spasmolytics, antiinflammatory drugs
C. To perform external choledoch drainage
D. To perform transduodenal papillosphincterotomy
E. To perform choledochoduodenostomy

109. Prevalence of a disease in region N amounted 1156 occurences per 1000 of inhabitants. What
of the mentioned indices characterizes the disease prevalence?
A. Intensive*
B. Extensive
C. Ratio
D. Visual index
E. Standardized

110. A patient suffers from chronic recurrent pancreatitis with evident disturbance of exocrinous
function. After intake of rich spicy food and spirits his stool becomes fatty. Reduced production
of what factor is the most probable cause of steatorrhea?
A. Lipase*
B. Tripsin
C. Acidity of gastric juice
D. Amylase
E. Alkaline phosphatase

111. A 54 y.o. woman has been ill with osteomyelitis of femoral bone for over 20 years. During the
last month there appeared and have been steadily increasing edemata of lower extremities.
Urine analysis revealed: proteinuria - 6,6 g/l. Blood analysis: disproteinemia in form of
hypoalbuminemia, raise of a2- and 7-globulines, ESR- 50 mm/h. What is the most probable
diagnosis?
A. Secondary renal amyloidosis*
B. Acute glomerulonephritis
C. Myelomatosis
D. Chronic glomerulonephritis
E. Systematic lupus erythematosus

112. A 43 y.o. woman complains of contact hemorrhages during the last 6 months. Bimanual
examination: cervix of the uterus is enlarged, its mobility is reduced. Mirrors showed the
following: cervix of the uterus is in the form of cauliflower. Chrobak and Schiller tests are
positive. What is the most probable diagnosis?
A. Cancer of cervix of the uterus*
B. Polypus of the cervis of the uterus
C. Cervical pregnancy
D. Nascent fibroid
E. Leukoplakia

23
Завантажено з сайту https://тестування.укр/ - онлайн тестування КРОК

113. After a long periode of subfebrility a patient registered increase of dyspnea, pain in the right
hypochondrium, leg edemata. Objectively: neck veins are edematic. Ps is 120 bpm, sometimes
it disappears during inspiration. Heart sounds are very weakened. ECG showed low-voltage
waves of ventricular complex. A month ago there was raise of ST V1 — V4 segment. Cardiac
silhouette is enlarged, roundish. What is the most probable diagnosis?
A. Exudative pericarditis*
B. Small-focal myocardial infarction
C. Postinfarction cardiosclerosis
D. Metabolic postinfection myocardi-opathy
E. Primary rheumatic carditis

114. A 14 y.o. child suffers from vegetovascular dystonia of pubertal period. He has got
sympathoadrenal atack. What medicine should be used for atack reduction?
A. Obsidan*
B. No-shpa
C. Amisyl
D. Euphyline
E. Corglicone

115. A patient consulted a doctor about acure respiratory viral infection. The patient was
acknowledged to be off work. The doctor issued him a medical certificate for 5 days. The
patient is not recovering. What measures should the doctor take in order to legalize the
further disability of patient?
A. To prolong the medical certificate at his own discretion but no more than for 10 days in
total*
B. To prolong the medical certificate at his own discretion but no more than for 6 days in
total
C. To prolong the medical certificate together with department superintendent
D. To send the patient to the medical consultative commission
E. To send the patient to the medical social expert comission

116. A 58 y.o. patient complains of weakness, leg edemata, dyspnea, anorexia. He has been
suffering from chronic bronchitis for many years. During the last 5 years he has been noting
intensified discharge of sputum that is often purulent. Objectively: RR- 80/min, AP-120/80 mm
Hg. Disseminated edemata, skin is dry and pale, low turgor. In urine: intense proteinuria,
cylindruria. Specify the most probable pathological process in kidneys:
A. Renal amyloidosis*
B. Chronic glomerulonephritis
C. Chronic pyelonephritis
D. Interstitial nephritis
E. Acute glomerulonephritis

117. A 4 y.o. boy was admitted to the hospital with complaints of dyspnea, rapid fatigability. His
anamnesis registers frequent respiratory diseases. On percussion: heart borders are dilatated
to the left and upwards. On auscultation: amplification of the SII above pulmonary artery, a
harsh systolodyastolic "machi-ne"murmur is auscultated between the II and the III rib to the
left of breast bone, this murmur is conducted to all other poi- nts including back. AP is 100/20

24
Завантажено з сайту https://тестування.укр/ - онлайн тестування КРОК

mm Hg. What is the most probable diagnosis?


A. Opened arterial duct*
B. Interventricular septal defect
C. Isolated stenosis of pulmonary arterial orifice
D. Interatrial septal defect
E. Valvar aortic stenosis

118. A patient was admitted to the hospital with complaints of periodical pain in the lower part of
abdomen that gets worse during menses, weakness, malaise, nervousness, dark bloody smears
from vagina directly before and after menses. Bimanual examination revealed that uterus body
is enlarged, appendages cannot be palpated, posterior fornix has tuberous surface.
Laparoscopy revealed: ovaries, peritoneum of rectouterine pouch and pararectal fat have
"cyanotic eyes". What is the most probable diagnosis?
A. Disseminated form of endometriosis*
B. Polycystic ovaries
C. Chronic salpingitis
D. Tuberculosis of genital organs
E. Ovarian cystoma

119. A 19 y.o. patient was admitted to the hospital with acute destructive appendicitis. He suffers
from hemophilia B-type. What antihemophilic medicine should be included in pre- and post-
operative treatment plan?
A. Fresh frozen plasma*
B. Cryoprecipitate
C. Fresh frozen blood
D. Native plasma
E. Dried plasma

120. A 28 y.o. patient who has no permanent residence was admitted to the hospital with
preliminary diagnosis "influenza", on the 5-th day of disease there are appeared
maculopapular and petechial rash on his body and internal surfaces of his extremities. Body
temperature is 410C, euphoria, hyperemic face, scleras reddening, tongue tremor,
tachycardia, splenomegaly, excitement. What is the most probable diagnosis?
A. Spotted fever*
B. Delirium alcoholicum
C. Leptospirosis
D. Measles
E. Typhoid fever

121. An 18 y.o. woman consulted a gynecologist about the pain in the lower part of abdomen, fever
up to 37,50C, considerable mucopurulent discharges from the genital tracts, painful urination.
Vaginal examination with mirrors: the urethra is infiltrated, cervix of the uterus is hyperemic,
erosive. The uterus is painful, ovaries are painful, thickened; fornixes are free. Bacterioscopy
test revealed diplococcus. What diagnosis is the most probable?
A. Recent acute ascending gonorrhea*
B. Trichomoniasis
C. Candydomycosis

25
Завантажено з сайту https://тестування.укр/ - онлайн тестування КРОК

D. Chronic gonorrhea
E. Chlamydiosis

122. An infant is 2 d.o. It was full-term born with signs of intrauterine infection, that’s why it was
prescribed antibiotics. Specify, why the gap between antibiotic introductions to the new-born
children is longer and dosage is smaller compared to the older children and adults?
A. The newborns have a lower level of glomerular filtration*
B. The newborns have lower concentration of protein and albumins in blood
C. The newborns have reduced activity of glucuronil transferase
D. The newborns have diminished blood pH
E. The newborns have bigger hematocrit

123. A district doctor keeps the record of reconvalescents after infectious diseases, people who are
disposed to frequent and long-lasting diseases, patients with chronic pathologies. What
category of patients should belong to the III health group?
A. People with chronic diseases*
B. People disposed to frequent and long-lasting diseases
C. People with chronic pathologies and disposed to frequent and long-lasting diseases
D. Reconvalescents after infectious diseases and patients with chronic pathologies
E. All above mentioned categories

124. A 2 m.o. child was delivered in time with weight 3500 g and was on the mixed feeding. Current
weight is 4900 g. Evaluate the current weight of the child:
A. Corresponding to the age*
B. 150 g less than necessary
C. Hypotrophy of the I grade
D. Hypotrophy of the II grade
E. Paratrophy of the I grade

125. A 46 y.o. patient complains of colicky pain in the right lumbar region that is irradiating to the
lower part of abdomen, nausea. She didn’t have such pains before. Survey radiograph of
abdominal cavity organs didn’t reveal any pathological stains. Ultrasonic sonogram revealed in
the enlarged right renal pelvis a hyperechoic mass approximately 1,5 cm large that gives rise
to an "ultrasonic track". What is the most probable diagnosis?
A. Renal calculus*
B. Benign tumor of kidney
C. Renal cyst
D. Renal tuberculosis
E. Malignant tumor of kidney

126. A woman consulted a doctor on the 14-th day after labor about sudden pain, hyperemy and
induration of the left mammary gland, body temperature rise up to 390C, headache,
indisposition. Objectively: fissure of nipple, enlargement of the left mammary gland, pain on
palpation. What pathology would you think about in this case?
A. Lactational mastitis*
B. Lacteal cyst with suppuration

26
Завантажено з сайту https://тестування.укр/ - онлайн тестування КРОК

C. Fibrous adenoma of the left mammary gland


D. Breast cancer
E. Phlegmon of mammary gland

127. A 42 y.o. patient complains of weakness, heartbeat, nasal hemorrhages, cutaneous


hemorrhages. His condition has been worsening progressively for a month. Objectively: grave
condition, the extremities and body skin has spotted and petechial hemorrhages, lymph nodes
are not palpable, Ps- 116/min, liver is +2 cm enlarged, spleen is not palpable. Blood has
evident pancytopenia. What disease should you think about first of all?
A. Hypoplastic anemia*
B. Acute leukosis
C. Werlhof’s disease
D. Hemorrhagic vasculitis
E. Acute agranulocytosis

128. A 63 y.o. patient was operated on account of big multinodular euthyroid goiter. Despite of
techical difficulties a forced subtotal resection of both parts of the thyroid gland was
performed. On the 4-th day after the operation the woman had cramps of face muscles and
upper extremities, stomach ache. Positive Chvostek’s and Trousseau’s signs. What is the most
probable cause of such condition?
A. Insufficiency of parathyroid glands*
B. Postoperative hypothyroidism
C. Thyrotoxic crisis
D. Injury of recurrent nerve
E. Tracheomalacia

129. A girl 13 y.o. consulted the school doctor on account of moderate bloody discharge from the
genital tracts, which appeared 2 days ago. Secondary sexual characters are developed. What is
the most probable cause of bloody discharge?
A. Menarche*
B. Juvenile hemorrhage
C. Haemophilia
D. Endometrium cancer
E. Werlhof’s disease

130. A 30 y.o. victim of fire has thermal burns of III-A and III-B degree that amount 20% of total
skin coverlet. AP is 110/70 mm Hg, HR- 120/min. What transfusion means shoul be used for
blind infusion correction during transportation?
A. Salines*
B. Polyglucine
C. 10% glucose solution
D. Fresh frozen plasma
E. Albumin

131. In course of observation of sanitary conditions of studying at the technical university it was
necessary to evaluate the visual regimen of students, who study from 9 a.m to 3 p.m. What

27
Завантажено з сайту https://тестування.укр/ - онлайн тестування КРОК

index of natural light will be the most informative?


A. Natural light coefficient*
B. Light coefficient
C. Depth of study room
D. Time of the room insolation
E. Presence of mixed (upper-lateral) light

132. A 25 y.o. man who has been suffering from disseminated sclerosis for 4 years complains of
increasing unsteadyness, weakness of his lower extremities, urinary retention. Objectively:
central tetraparesis. Cerebellar ataxia. Disturbed function of pelvic organs. What is the most
appropriate therapy in this case?
A. Glucocorticoids*
B. Antibiotics
C. Nootropics
D. Desensitizing medications
E. Vitamins

133. A 70 y.o. man is ill with ischemic heart disease. His mood is evidently depressed, anxious. As a
result of conti-nious sleeplessness he has got fears, suicidal thoughts. He would sit for a long
time in the same pose, answer after a pause, in a low, monotonous voice. His face has a look of
suffering, pain, fear. What is the main psychopathologic syndrome?
A. Depressive syndrome*
B. Paranoid syndrome
C. Asthenic syndrome
D. Phobic syndrome
E. Obsessive syndrome

134. A 32 y.o. patient has been suffering from systematic scleroderma for 14 years. She was
repeatedly exposed to treatment in the in-patient department. Complains of periodical dull
cardiac pain, dyspnea, headache, eyelid edemata, weight loss, pain and deformation of
extremities joints. What organ’s lesion deteriorates the prognosis for the disease?
A. Kidneys*
B. Heart
C. Lungs
D. Gastrointestinal tract
E. Skin and joints

135. A 2 m.o. child with birth weight 5100 g has jaundice, hoarse cry, umbilical hernia, physical
development lag. Liver is +2 cm enlarged, spleen is not enlarged. In anamnesis: delayed
falling-away of umbilical cord rest. In blood: Hb- 120 g/L, erythrocytes - 4,5 • 1012/L, ESR- 3
mm/h. Whole serum bilirubin is 28 mcmole/L, indirect - 20 mcmole/L, direct - 8 mcmole/L.
What is the most probable diagnosis?
A. Congenital hypothyreosis*
B. Congenital hepatitis
C. Hemolitic anemia
D. Conjugated jaundice
E. Cytomegalovirus infection

28
Завантажено з сайту https://тестування.укр/ - онлайн тестування КРОК

136. A 13 y.o. girl complains of having temperature rises up to febrile figures for a month, joint
ache, periodical skin rash. Examination revealed steady enhancing of ESR, LE-cells. What is
the most probable diagnosis?
A. Systematic lupus erythematosus*
B. Juvenile rheumatoid arthritis
C. Systematic scleroderma
D. Acute lymphoblast leukosis
E. Rheumatics

137. A 50 y.o. woman who suffers from chronic pyelonephritis was prescribed a combination of
antibiotics for the period of exacerbation - gentamicin (80 mg 3 times a day) and biseptol (960
mg twice a day). What consequences may be caused by such a combination of antibiotics?
A. Acute renal insufficiency*
B. Glomerulosclerosis
C. Chronic renal insufficiency
D. Antibiotic combination is optimal and absolutely safe
E. Acute suprarenal insufficiency

138. A patient has been suffering from morning cough accompanied by discharge of small amount
of sputum, dyspnea for 8 years. He has been smoking for 10 years. Objectively: cyanosis,
prolonged expiration, dry rales. What is the most probable diagnosis?
A. Chronic obstructive bronchitis*
B. Chronic non-obstructive bronchitis
C. Idiopatic fibrosing alveolitis
D. Multiple bronchiectasis
E. Bronchial asthma

139. A 70 y.o. patient complains of weakness, dizziness, short periods of unconsciousness, pain in
the cardiac area. Objectively: HR- 40 bpm, heart sounds are rhythmic, the S1 is dull,
periodically amplified. AP is 180/90 mm Hg. What is the most probable cause of hemodynamic
disturbances?
A. Atrioventricular block type III*
B. Atrioventricular block type I
C. Bradysystolic form of ciliary arrhythmia
D. Sinus bradycardia
E. Complete left bandle-branch block

140. A child is 1 y.o. Within the last months after the begining of supplemental feeding the child has
appetite loss, diarrhea with massive defecation, sometimes vomiting. Objectively: body
temperature is normal. Body weight is 7 kg. Evident pallor of skin, leg edemata, enlarged
abdomen. Coprogram shows a lot of fatty acids and soaps. The child was diagnosed with celiac
disease and prescribed gluten-free diet. What shoul be excluded from the dietary intake in this
case?
A. Cereals - wheat, oats*
B. Milk and dairy produce
C. Fruit
D. Animal protein

29
Завантажено з сайту https://тестування.укр/ - онлайн тестування КРОК

E. Digestible carbohydrates

141. Fluorography of a 45 y.o. man revealed some little intensive foci with indistinct outlines on the
top of his right lung for the first time. The patient doesn’t feel worse. He has been smoking for
many years. Objectively: pulmonary sound above lungs on percussion, respiration is vesicular,
no rales. Blood count is unchanged. What is the most probable diagnosis?
A. Focal pulmonary tuberculosis*
B. Peripheral cancer of lung
C. Eosinophilic pneumonia
D. Bronchopneumonia
E. Disseminated pulmonary tuberculosis

142. An 8 y.o. boy complains of constant cough along with discharge of greenish sputum, dyspnea
during physical activities. At the age of 1 year and 8 months he fell ill for the first time with
bilateral pneumonia that had protracted course. Later on there were recurrences of the
disease 5-6 times a year, during the remission periods there was constant productive cough.
What examination results will be the most important for making a final diagnosis?
A. Bronchography*
B. Roentgenography of thorax organs
C. Bacterial inoculation of sputum
D. Bronchoscopy
E. Spirography

143. A 35 y.o. patient who suffers from chronic glomerulonephritis and has been hemodialysis-
dependent for the last three years developed intermissions of heart activity, hypotension,
increasing weakness, dyspnea. ECG showed bradycardia, atrioventricular block type I, high
pointed waves T. The day before the flagrant violation of diet took place. What is the most
probable cause of these changes?
A. Hyperkaliemia*
B. Hyperhydratation
C. Hypokaliemia
D. Hypernatriemia
E. Hypocalciemia

144. A 60 y.o. patient cpmplains of weakness, dizziness, heaviness in the upper part of abdomen,
paresthesia of toes and fingers. Objectively: skin icteri-tiousness, tongue is crimson, smooth.
Hepatomegaly. In blood: Hb- 90 g/l, erythrocytes - 2, 3 ■ 1012/l, reticulocytes - 0,2%; color
index - 1,2, macrocytosis; Jolly’s bodies, Cabot’s ring bodies. What medication is the most
appropriate for treatment?
A. Vitamin B12*
B. Feroplex
C. Packed red blood cells
D. Prednisolone
E. Dyspherol

145. A 30 y.o. primigravida woman has got intensive labor pain every 1-2 minutes that lasts 50

30
Завантажено з сайту https://тестування.укр/ - онлайн тестування КРОК

seconds. The disengagement has started. The perineum with the height of 4 cm has grown
pale. What actions are necessary in this situation?
A. Episiotomy*
B. Perineum protection
C. Perineotomy
D. Vacuum extraction of fetus
E. Expectant management

146. A doctor of the general practice has registered the following death causes for the previous
year: the first place was taken by cardiovascular diseases (60%), the second - by tumors (18%),
then - traumas (8,3%) etc. What diagrams will provide the most substantial information about
the registered ocurrences?
A. Pie diagram*
B. Cartogram
C. Line diagram
D. Circle diagram
E. Column diagram

147. A patient complains of nycturia, constant boring pain in perineum and suprapubic region,
weak urine jet, frequent, obstructed and painful urination. He has been ill for several months,
pain in perineum appeared after getting problems with urination. Rectal examination revealed
that prostate is enlarged (mostly owing to the right lobe), dense, asymmetric, its central sulcus
is flattened, the right lobe is dense, painless, tuberous. What disease can it be?
A. Cancer of prostate*
B. Prostate sclerosis
C. Urolithiasis, stone of the right lobe of prostate
D. Prostate tuberculosis
E. Chronic congestive prostatitis

148. A 43 y.o. patient complains of periodical pain attacks in the right half of her face. The attack is
characterized by spasm of mimetic muscles of the right face’s half, reddening of skin on this
side. Blood has no pathologies. She was diagnosed with right-sided trifacial neuralgia. What
medication should be prescribed?
A. Finlepsin*
B. Prednisolone
C. Actovegine
D. Analgine
E. Indometacin

149. A 35 y.o. patient experienced a strong nervous stress that resulted in formation of reddened
and edematic areas on the back surface of her hands with further formation of small
inflammated nodules, vesicles and then erosions accompanied by profuse discharge of serous
liquid. The process is also accompanied by intense itching. What is the most probable
diagnosis?
A. Common eczema*
B. Allergic dermatitis
C. Microbial eczema

31
Завантажено з сайту https://тестування.укр/ - онлайн тестування КРОК

D. Common contact dermatitis


E. Toxicodermia

150. A 42 y.o. patient lifted a heavy object that resulted in acute pain in the right half of his chest,
increased dyspnea. The patient’s condition is grave: cyanosis of lips and mucous membranes,
RR is 28 pm, Ps-122 bpm. On percussion there is tympanitis above the right half of chest, on
auscultation - stongly diminished breath sounds; accent of the II heart sound above the
pulmonary artery. AP is 80/40 mm Hg. What is the main emergency action at the pre-
admission stage?
A. Air aspiration from the pleural cavity*
B. Adrenaline introduction
C. Aminophylline introduction
D. Calling a cardiological brigade
E. Oxygen inhalation

151. A 74 y.o. patient complains of abdomen pain and sweling, nausea. She suffers from ischemic
heart disease, postinfarction and atherosclerotic cardiosclerosis. Objectively: the patient is in
grave condition, abdomen is swollen, abdominal wall doesn’t take active part in respiration.
Laparoscopy revealed a small amount of muddy effusion in abdominal cavity, one of the loops
of small intestine is dark-cyan. What is the most probable diagnosis?
A. Thrombosis of mesenteric vessels*
B. Twisted bowels
C. Acute intestinal obstruction
D. Ischemic abdominal syndrome
E. Erysipelas

152. A 10 y.o. child who is at oligoanuretic stage of acute renal insufficiency has got sensations of
pricking in the mucous membrane of oral cavity and tongue, extremities numbness, reduced
reflexes, respiratory disturbance, arrhythmia. What are these symptoms caused by?
A. Hyperkaliemia*
B. Hyponatremia
C. Hyperazotemia
D. Acidosis
E. Alkalosis

153. A 30 y.o. woman has the 2-nd labour that has been lasting for 14 hours. Hearbeat of fetus is
muffled, arrhythmic, 100/min. Vaginal examination: cervix of uterus is completely opened,
fetus head is level with outlet from small pelvis. Saggital suture is in the straight diameter,
small crown is near symphysis. What is the further tactics of handling the delivery?
A. Use of obstetrical forceps*
B. Stimulation of labour activity by oxytocin
C. Cesarean section
D. Cranio-cutaneous (Ivanov’s) forceps
E. Use of cavity forceps

154. A 75 y.o. man with IHD (atherosclerotic cardiosclerosis, atrial fibrillation, cardiac insufficiensy

32
Завантажено з сайту https://тестування.укр/ - онлайн тестування КРОК

stage 2 B; chronic pyelonephritis) was prescribed di-goxin. During the first 6 days the digoxin
dose amounted 0,25 mg twice per day that let to abatement of dyspnea, edemata and cyanosis.
But on the 7-th day the patient developed nausea and bradycardia. What is the most probable
cause of digoxin intoxication?
A. Disturbed elimination of the medication by kidneys*
B. Too long intake of saturating dose
C. Exceeding daily saturating dose
D. Disturbed metabolism of digoxin in liver
E. Treatment complex doesn’t include unitiole

155. A patient who has been contacting with benzol for 6 years has a nonevident leukopenia,
moderate reti-culocytosis, gingival hemorrhage, dizzi- ness, asthenovegetative syndrome. What
a severity degree of chronic intoxication with benzol corresponds with described symptoms?
A. Minor*
B. Moderate
C. Severe
D. Disease is not connected with work conditions
E. -

156. A 30 y.o. man was always reserved by nature. He never consulted psychiatrists. He complains
of headache, sensation "as if something bursts, moves, bubbles under his skin". Objectively: no
pathology was revealed. What is the most probable psychopathologic symptom in this case?
A. Cenestopathy*
B. Paresthesia
C. Hallucination
D. Hypersthesia
E. Dysmorphopsia

157. A 45 y.o. man complains of having intensive pain in the epigastric region 1,52 hours later after
food intake. He has been suffering from ulcer for 11 years. Objectively: t0- 36,50C, RR- 16/min,
Ps-70 bpm, AP- 120/80 mm Hg. On palpation: local painfulness in the right epigastric region.
What parameters of intragastric Ph-meter in the region of stomach body are the most typical
for this patient’s disease?
A. pH = 1,0-2,0*
B. pH = 3,0-4,0
C. pH = 4,0-5,0
D. pH = 5,0-6,0
E. pH = 6,0-7,0

158. A 13 y.o. teenager who suffers from hemophilia A was taken to the hospital after a fight at
school. His diagnosis is right-sided hemarthros of knee joint, retroperitoneal hematoma. What
should be primarily prescribed?
A. Fresh frozen plasma*
B. Aminocapronic acid
C. Washed thrombocytes
D. Placental albumin
E. Dry plasma

33
Завантажено з сайту https://тестування.укр/ - онлайн тестування КРОК

159. A 3 m.o. child fell seriously ill, body temperature rised up to 37,80C, there is semicough. On
the 3-rd day the cough grew worse, dyspnea appeared. On percussion: tympanic sound above
lungs, on auscultation: a lot of fine moist and wheezing rales during expiration. What is the
most probable diagnosis?
A. Acute respiratory viral infection, bronchiolitis*
B. Acute respiratory viral infection, bronchopneumonia
C. Acute respiratory viral infection, bronchitis
D. Acute respiratory viral infection, bronchitis with asthmatic component
E. Acute respiratory viral infection, focal pneumonia

160. A worker at a porcelain factory who has been in service for 10 years complains of cough,
dyspnea, ache in his chest. What occupational disease are these complaints most typical for?
A. Silicosis*
B. Multiple bronchiectasis
C. Chronic dust bronchitis
D. Occupational bronchial asthma
E. Chronic cor pulmonale

161. Mother of a newborn child suffers from chronoc pyelonephritis. She survived acute respiratory
viral infection directly before labour. Delivery was at term, the period before discharge of
waters was prolonged. On the 2-nd day the child got erythematous rash, later on - vesicles
about 1 cm large with seropurulent content. Nikolsky’s symptom is positive. Dissection of
vesicles results in erosions. The child is inert, body temperature is subfebrile. What is the most
probable diagnosis?
A. Impetigo neonatorum*
B. Vesicular pustulosis
C. Pseudofurunculosis
D. Sepsis
E. Ritter’s dermatitis

162. Medical examination of a man revealed "geographic tongue". This microsymptom is the
evidence of the following vitamin deficiency:
A. Vitamins of B group*
B. Vitamin A
C. Vitamin C
D. Vitamin D
E. Vitamin PP

163. A 41 y.o. woman has suffered from nonspecific ulcerative colitis for 5 years. On
rectoromanoscopy: evident inflammatory process of lower intestinal parts, pseudopolyposive
changes of mucous membrane. In blood: WBC- 9,8 • 109/L, RBC- 3,0 • 1012/L, sedimentation
rate - 52 mm/hour. What medication provides pathogenetic treatment of this patient?
A. Sulfosalasine*
B. Motilium
C. Vikasolum
D. Linex
E. Kreon

34
Завантажено з сайту https://тестування.укр/ - онлайн тестування КРОК

164. A 49 y.o. female patient was admitted to the hospital with acute attacks of headache
accompanied by pulsation in temples, AP rised up to 280/140 mm Hg. Pheochromocytoma is
suspected. What mechanism of hypertensive atack does this patient have?
A. Increasing of catecholamines concentration*
B. Increasing of aldosterone level in blood
C. Increasing of plasma renin activity
D. Increasing of vasopressin excretion
E. Increasing of thyroxine excretion

165. A child was born with body weight 3250 g and body length 52 cm. At the age of 1,5 month the
actual weight is sufficient (4350 g), psychophysical development corresponds with the age.
The child is breast-fed, occasionally there are regurgitations. What is the cause of
regurgitations?
A. Aerophagia*
B. Pylorostenosis
C. Pylorospasm
D. Acute gastroenteritis
E. Esophageal atresia

166. A patient with hepatic cirrhosis drank some spirits that resulted in headache, vomiting,
aversion to food, insomnia, jaundice, fetor hepaticus, abdominal swelling. What complication
of hepatic cirrhosis is meant?
A. Hepatocellular insufficiency*
B. Hemorrhage from varicosely dilatated veins of esophagus
C. Portal hypertension
D. Acute stomach ulcer
E. Thrombosis of mesenteric vessels

167. On the 4-th day of injections a 60 y.o. patient felt pain and tissue induration in the left buttock.
Objectively: the skin in the superexternal quadrant of the left buttock is red and hot, palpation
reveals a painful infiltrate 6x6 cm large with softening in the centre. Body temperature is
37,90C. What action is necessary to di- agnose an abscess?
A. Punction*
B. Biopsy
C. Ultrasonic examination
D. X-ray investigation
E. Clinical blood analysis

168. A 39 y.o. patient complains of having dyspnea during physical activity, crus edemata,
palpitation, heart intermissions. Objectively: HR is 150 bpm, atrial fibrillation. Heart is both
ways enlarged. Heart sounds are muted. Liver is 6 cm below the costal margin.
Echocardiogram reveals dilatation of heart chambers (end diastolic volume of left ventricle is
6,8 cm) is 29% EF, valve apparatus is unchanged. What is the most probable diagnosis?
A. Dilated cardiomyopathy*
B. Exudative pericarditis
C. Restrictive cardiomyopathy
D. Hypertrophic cardiomyopathy

35
Завантажено з сайту https://тестування.укр/ - онлайн тестування КРОК

E. Thyreotoxic cardiomyopathy

169. The major repair of a hospital included renewal of colour design of hospital premises because
it is of great psychological and aesthetical importance; and so the walls of patient wards will
be painted under consideration of:
A. Windows orientation*
B. Hospital profile
C. Diseases of patients who will be staying in these wards
D. Wall re flection coefficient
E. Creation of cozy atmosphere

170. A 10 y.o. child has average indices of body length and her chest circumference exceeds
average indices, body weight index is heightened due to lipopexia. Functional characteristics
of physical development are below average. Physical development of this child can be
estimated as:
A. Disharmonic*
B. Average
C. Below average
D. Harmonic
E. Deeply disharmonic

171. Atmospheric air of an industrial centre is polluted with the following wastes of metallurgical
plants: sulphuric, nitric, metal, carbon oxides that have negative influence upon the
inhabitants’ health. The effct of these hazards can be characterized as:
A. Combined*
B. Complex
C. Associated
D. Adjacent
E. Mixed

172. X-ray pattern of thorax organs revealed a large intensive inhomogeneous opacity with
indistinct outlines on the right side at the level of the 4-th rib. In the centre of this opacity
there is a horizontal level and clearing of lung tissue above it. What disease does this X-ray
pattern correspond with?
A. Abscess of the right lung*
B. Peripheral cancer
C. Tuberculoma of the right lung
D. Right-sided pneumothorax
E. -

173. A 30 y.o. patient has got multiple body skin rash consisting of small paired elements that are
scattered on the skin diorderly and mostly focally, they are accompanied by itch. The rash
appeared a few days after attending sport centre and sauna. What is the most probable
diagnosis?
A. Scab*
B. Eczema

36
Завантажено з сайту https://тестування.укр/ - онлайн тестування КРОК

C. Contact dermatitis
D. Allergic dermatitis
E. Neurodermitis

174. A 56 y.o. patient ill with cholecystectomy suddenly had an intense hemorrhage. She needs
blood transfusion. Her blood group is AB(IV)Rh-. Hemotransfusion station doesn’t dispose of
this group. What group of donors can be involved?
A. Donors of rare blood groups*
B. Donors of active group
C. Relatives
D. Emergency donors
E. Reserve donors

175. A 33 y.o. patient was admitted to the hospital with stopped repeated ulcerative bleeding. He
was pale and exhausted. Blood count: Hb- 77 g/l, Ht- 0,25. In view of anemia there were made
two attempts of blood transfusion of the same group - A(II)Rh+. In both cases the transfusion
had to be stopped because of development of anaphylactic reaction. What transfusion medium
would be advisable in this case?
A. Washed erythrocytes*
B. Fresh citrate blood
C. Erythrocytic mass (native)
D. Erythrocytic suspension
E. Erythrocytic mass poor in leukocytes and thrombocytes

176. A 19 y.o. boy was admitted to the hospital with closed abdominal trauma. In course of
operation multiple ruptures of spleen and small intestine were revealed. Ap is falling rapidly, it
is necessary to perform hemotransfusion. Who can determine the patient’s blood group and
rhesus compatibility?
A. A doctor of any speciality*
B. A laboratory physician
C. A surgeon
D. A traumatologist
E. An anaesthesilogist

177. A 20 y.o. patient complains of amenorrhea. Objectively: hirsutism, obesity with fat tissue
prevailing on the face, neck, upper part of body. On the face there are acne vulgaris, on the
skin - striae cutis distense. Psychological and intellectual development is normal.
Gynecological condition: external genitals are moderately hairy, acute vaginal and uterine
hypoplasia. What diagnosis is the most probable?
A. Itsenko-Cushing syndrome*
B. Turner’s syndrome
C. Stein-Levental’s syndrome
D. Shichan’s syndrome
E. Babinski-Froehlich syndrome

178. A 14 y.o. girl got a bad mark at the lesson of math as well as teacher’s reprimand that made

37
Завантажено з сайту https://тестування.укр/ - онлайн тестування КРОК

her cry for a long time. At the end of the lesson she suddenly lost consciousness and fell down.
Objectively: skin is of pale pink colour, Ps- 100 bpm, satisfactory, AP- 110/70 mm Hg. Eyelids
are closed and don’t give away to the tries of their opening. There are no evident injuries.
What is the provisional diagnosis?
A. Hysteric syncope*
B. Epilepsy
C. Vagotonic syncope
D. Long QT syndrome
E. Sympathicotonic collapse

179. A newborn child has purulent discharges from the umbilical wound, skin around the umbilicus
is swollen. Objectively: the child’s skin is pale, of yellow-greyish colour, generalized
hemorrhagic rash. Body temperature is of hectic nature. What is the most probable diagnosis?
A. Sepsis*
B. Hemorrhagic disease of newborn
C. Hemolytic disease of newborn
D. Thrombocytopathy
E. Omphalitis

180. Elderly people develop tumors more frequently. One of the main causes is:
A. Decreased activity of cellular immunity*
B. Decreased intensity of antibodies formation
C. Increased disfunctioning of mitoses
D. Increased activity of cellular immunity
E. Increased intensity of antibodies formation

181. A 30 y.o. man complains of intense pain, reddening of skin, edema in the ankle-joint area, fever
up to 390C. There was acute onset of the illness. In the past there were similar attacks lasting
5-6 days without residual changes in the joint. The skin over the joint is hyperemic without
definite borders and without infiltrative bank on the periphery. What is the most likely
diagnosis?
A. Gout*
B. Infectional arthritis
C. Rheumatoid arthritis
D. Erysipelatous inflammation
E. Osteoarthritis

182. A 7 y.o. boy was admitted to the hospital. He complains of unpleasant sensations in the heart
region, pain in the epigastrium, dizziness, vomiting. Objectively: evident paleness of skin,
dyspnea, jugular pulse. Heart borders are within the normal range. Heart sounds are clear,
HR- 170/min, small pulse. AP- 90/50 mm Hg. EKG showed: paroxysm of ventricular
tachycardia. The paroxysm can be suppressed by:
A. Lidocain*
B. Morphine
C. Enalapril
D. Nifedipine
E. Strophanthine

38
Завантажено з сайту https://тестування.укр/ - онлайн тестування КРОК

183. A child is 2 m.o. Inguinofemoral folds contain acutely inflamed foci with distinct borders in
form of spots that are slightly above the surrounding areas due to skin edema. The rash has
appeared during the week. Vesiculation and wetting are absent. What is the most probable
diagnosis?
A. Napkin-area dermatitis*
B. Infantile eczema
C. Dermatomycosis
D. Psoriasis
E. Complicated course of scabies

184. A 38 y.o. patient lifted a heavy object that resulted in pain in the lumbar part of spine
irradiating to the posterior surface of his left leg. The pain increases during changing body
position and also in the upright position. Examination revealed positive symptoms of tension.
What is the preliminary diagnosis?
A. Pathology of intercostal disks*
B. Spinal cord tumor
C. Arachnomyelitis
D. Polyneuritis
E. Myelopathy

185. A 35 y.o. patient was admitted to the local hospital a week after a road accident with clinical
picture of clotted hemothorax. What is the most appropriate treatment tactic for prevention of
acute pleural empyema?
A. Surgical removal of clotted hemothorax*
B. Treatment by pleural punctions
C. Complex conservative therapy
D. Passive drainage of pleural cavity
E. Active drainage of pleural cavity

186. An aircraft factory processes materials with use of lasers. It is determined that the device
radiates in the light spectrum and that levels of laser radiation at the workplaces exceed the
alarm level. Specify, what organs will be affected in the first place?
A. Eyes*
B. Skin
C. Liver
D. Spleen
E. Kidneys

187. At a machine-building plant the casts are cleaned by means of abrasion machines that are a
source of local vibration. What are the most efficient preventive measures for preventing
harmful effect of vibration on workers’ organisms?
A. Use of gloves that reduce vibration*
B. Preliminary and periodical medical examinations
C. Hand massaging
D. Warm hand baths
E. Giving sanitary instructions to the workers

39
Завантажено з сайту https://тестування.укр/ - онлайн тестування КРОК

188. After delivery and revision of placenta there was found the defect of placental lobe. General
condition of woman is normal, uterus is firm, there is moderate bloody discharge. Inspection of
birth canal with mirrors shows absence of lacerations and raptures. What action is nesessary?
A. Manual exploration of the uterine cavity*
B. External massage of uterus
C. Use of uterine contracting agents
D. Urine drainage, cold on the lower abdomen
E. Use of hemostatic medications

189. A 67 y.o. patient complains of dyspnea, breast pain, common weakness. He has been ill for 5
months. Objectively: t0- 37, 30C, Ps- 96/min. Vocal tremor over the right lung cannot be
determined, percussion sound is dull, breathing cannot be auscultated. In sputum: blood
diffusively mixed with mucus. What is the most probable diagnosis?
A. Lung cancer*
B. Macrofocal pneumonia
C. Bronchoectatic disease
D. Focal pulmonary tuberculosis
E. Exudative pleuritis

190. Poorly refined wastes of an industrial plant are usually thrown into the river that supplies
drinking water. It causes perishing of some microorganisms, disturbs processes of water self-
purification and worsens its quality that can have negative influence upon people’s health.
How is this effect of environmental factors called?
A. Indirect*
B. Direct
C. Associated
D. Complex
E. Combined

191. A 22 y.o. patient complains of having boring pain in the right iliac region for one week,
morning sickness, taste change. Delay of menstruation is 3 weeks. Objectively: AP- 110/70 mm
Hg, Ps- 78/min, t0-37,00C. Bimanual examination revealed that uterus is a little enlarged, soft,
movable, painless. Appendages palpation: a painful formation 3x4 cm large on the right, it is
dense and elastic, moderately movable. What is the most probable diagnosis?
A. Progressing tubal pregnancy*
B. Interrupted tubal pregnancy
C. Cyst of the right ovary
D. Uterine pregnancy
E. Acute appendicitis

192. A 15 y.o. patient has developmental lag, periodical skin yellowing. Objectively: spleen is
16x12x10 cm large, holecistoli-thiasis, skin ulcer on the lower third of his left crus. Blood
count: erythrocytes - 3,0 • 1012/L, Hb- 90 g/L, C.I.- 1,0, microspherocytosis, reticulocytosis.
Blood bilirubin - 56 mmole/L, indirect bilirubin - 38 mmole/L. Choose the way of treatment:
A. Splenectomy*
B. Spleen transplantation
C. Portocaval anastomosis

40
Завантажено з сайту https://тестування.укр/ - онлайн тестування КРОК

D. Omentosplenopexy
E. Omentohepatopexy

193. A 30 y.o. parturient woman was taken to the maternity house with complaints of having acute,
regular labour pains that last 25-30 seconds every 1,5-2 minutes. Labour activity began 6
hours ago. Uterus is in higher tonus, head of the fetus is above the opening into the small
pelvis. Fetal heartbeat is 136/min. PV: cervical dilatation is 4 cm, uterine fauces is spasming at
a height of parodynia. Head is level with opening into the small pelvis, it is being pushed off.
What is the most probable diagnosis?
A. Discoordinated labour activity*
B. Secondary powerless labour activity
C. Pathological preliminary period
D. Primary powerless labour activity
E. Normal labour activity

194. A primigravida woman appealed to the antenatal clinic on the 22.03.03 with complaints of
boring pain in the lower part of abdomen. Anamnesis registered that her last menstruation
was on the 4.01.03. Bimanual examination revealed that uterine servix is intact, external
fauces is closed, uterus is enlarged up to the 9-th week of pregnancy, movable, painless. What
complication can be suspected?
A. Risk of abortion in the 9-th week of pregnancy*
B. Abortion that started in the 9-th week of pregnancy
C. Hysteromyoma
D. Vesicular mole
E. -

195. On the 15-th day after a minor trauma of the right foot a patient felt malaise, fatigability,
irritability, headache, high body temperature, feeling of compression, tension and muscular
twitching of his right crus. What disease can it be?
A. Tetanus*
B. Anaerobic gas gangrene
C. Erysipelas
D. Acute thrombophlebitis
E. Thromboembolism of popliteal artery

196. 10 years ago a patient had a fracture in the middle one-third of his left femoral bone, and
during the last 7 years he has been having acute inflammation in the area of old fracture
accompanied by formation of a fistula through which some pus with small fragments of bone
tissue is discharged. After a time the fistula closes. What complication of the fracture is it?
A. Chronic osteomyelitis*
B. Bone tuberculosis
C. Soft tissue phlegmon
D. False joint
E. Trophic ulcer

197. Medical examination of a 43 y.o. man revealed objectively pailness of skin and mucous

41
Завантажено з сайту https://тестування.укр/ - онлайн тестування КРОК

membranes, smoothness of lingual papillas, transverse striation of nails, fissures in the mouth
corners, tachycardia. Hemoglobin content amounts 90 g/l; there are anisocytosis,
poikilocytosis. The most probable causative agent of this condition is deficiency of the
following microelement:
A. Iron*
B. Copper
C. Zinc
D. Magnesium
E. Selenium

198. The total area of ground intended for building of a regional hospital is 2,0 hectare. What is the
highest possible capacity of the in-patient hospital that can be built upon this ground?
A. 100 beds*
B. 200 beds
C. 400 beds
D. 800 beds
E. Over 1000 beds

199. A 33 y.o. woman survived two operations on account of extrauterine pregnancy, both uterine
tubes were removed. She consulted a doctor with a question about possibility of having a chi-
ld. What can be advised in this case?
A. Extracorporal fertilization*
B. Insemination with her husband’s semen
C. Substitutional maternity
D. Artifical fertilization with donor’s semen
E. Induction of ovulation

200. A worker diagnosed with "acute dysentery"was sent to the infectious department by a doctor
of aid post. What document should be used for registration of this disease?
A. Urgent report on infectious disease*
B. Statistic coupon for registration of final diagnoses
C. Outpatient’s card
D. Inpatient’s card
E. Statistic card of the patient who left in-patient hospital

42
Завантажено з сайту https://тестування.укр/ - онлайн тестування КРОК

ТЕСТУВАННЯ.УКР Бази тестів

Буклет 2008 року

Цей тест можна пройти в режимі онлайн тестування на сайті https://тестування.укр/testkrok/studing/565

Це офіційні тести з сайту Центру тестування https://www.testcentr.org.ua/

1. An 8 year old child has low-grade fever, arthritis, colicky abdominal pain and a purpuric rash
llocalized on the lower extremities. laboratory studies reveal a guaiac-positive stool, urinalysis
with red blood cell (RBC) casts and mild proteinuria, and a normal platelet count. The most
likely diagnosis is:
A. Henoch-Schonlein’s vasculitis*
B. Systemic lupus erythematosus (SLE)
C. Rocky Mountain spotted fever
D. Idiopathic thrombocytopenic purpura
E. Poststreptococcal glomerulonephritis

2. A woman complains of having slight dark bloody discharges and mild pains in the lower part of
abdomen for several days. Last menses were 7 weeks ago. The pregnancy test is positive.
Bimanual investigation: the body of the uterus indicates for about 5-6 weeks of pregnancy, it is
soft, painless. In the left appendage there is a retort-like formation, 7x5 cm large, mobile,
painless. What examination is necessary for detection of fetus localization?
A. Ultrasound*
B. Hysteroscopy
C. Hromohydrotubation
D. Colposcopy
E. Cystoscopy

3. A pregnant woman in her 40th week of pregnancy undergoes obstetric examination: the cervix
of uterus is undeveloped. The oxytocin test is negative. Examination at 32 weeks revealed: AP
140/90 mm Hg, proteinuria 1 g/l, peripheral edemata. Reflexes are normal. Choose the most
correct tactics:
A. Labour stimulation after preparation*
B. Absolute bed rest for 1 month
C. Complex therapy of gestosis for 2 days
D. Caesarian section immediately
E. Complex therapy of gestosis for 7 days

4. A 26 year old woman had the second labour within the last 2 years with oxytocin application.
The child’s weight is 4080 gr. After the placent birth there were massive bleeding, signs of
hemorrhagic shock. Despite the introduction of contractive agents, good contraction of the
uterus and absence of any cervical and vaginal disorders, the bleeding proceeds. Choose the
most probable cause of bleeding:
A. Atony of the uterus*
B. Injury of cervix of the uterus
C. Hysterorrhexis
D. Delay of the part of placenta
E. Hypotonia of the uterus

1
Завантажено з сайту https://тестування.укр/ - онлайн тестування КРОК

5. On the first day after labour a woman had the rise of temperature up to 39°C. Rupture of fetal
membranes took place 36 hours before labour. Examination of the bacterial flora of cervix of
the uterus revealed hemocatheretic streptococcus of A group. The uterus body is soft, tender.
Discharges are bloody, with admixtures of pus. Specify the most probable postnatal
complication:
A. Metroendometritis*
B. Thrombophlebitis of veins of the pelvis
C. Infectious hematoma
D. Infective contamination of the urinary system
E. Apostasis of sutures after the episi-otomy

6. A woman of a high-risk group (chronic pyelonephritis in anamnesis) had vaginal delivery. The
day after labour she complained of fever and loin pains, frequent urodynia. Specify the most
probable complication:
A. Infectious contamination of the urinary system*
B. Thrombophlebitis of veins of the pelvis
C. Infectious hematoma
D. Endometritis
E. Apostasis of sutures after episiotomy

7. A woman in her 39th week of pregnancy, the second labour, has regular birth activity. Uterine
contractions take place every 3 minutes. What criteria describe the beginning of the II labor
stage the most precisely?
A. Cervical dilatation by no less than 4 cm*
B. Cervical smoothing over 90%
C. Duration of uterine contractions over 30 seconds
D. Presenting part is in the lower region of small pelvis
E. Rupture of fetal bladder

8. A 24 years old primipara was hospitalised with complaints about discharge of the amniotic
waters. The uterus is tonic on palpation. The position of the fetus is longitudinal, it is pressed
with the head to pelvic outlet. Palpitation of the fetus is rhythmical, 140 bpm, auscultated on
the left below the navel. Internal examination: cervix of the uterus is 2,5 cm long, dense, the
external os is closed, light amniotic waters out of it. Point a correct component of the
diagnosis:
A. Antenatal discharge of the amniotic waters*
B. Early discharge of the amniotic waters
C. The beginning of the 1st stage of labour
D. The end of the 1st stage of labour
E. Pathological preterm labour

9. A 29 year old patient underwent surgical treatment because of the benign serous epithelial
tumour of an ovary. The postoperative period has elapsed without complications. What is it
necessary to prescribe for the rehabilitational period:
A. Hormonotherapy and proteolytic enzymes*
B. Antibacterial therapy and adaptogens
C. Lasertherapy and enzymotherapy

2
Завантажено з сайту https://тестування.укр/ - онлайн тестування КРОК

D. Magnitotherapy and vitamin therapy


E. The patient does not require further care

10. A patient was delivered to a surgical department after a road accident with a closed trauma of
chest and right-sided rib fracture. The patient was diagnosed with right-sided pneumothorax,
it is indicated to perform drainage of pleural cavity. Pleural puncture should be made in:
A. In the 2nd intercostal space along the middle clavicular line*
B. In the 6th intercostal space along the posterior axillary line
C. In the 7th intercostal space along the scapular line
D. In the projection of pleural sinus
E. In the point of the greatest dullness on percussion

11. A primagravida with pregnancy of 3738 weeks complains of headache, nausea, pain in
epigastrium. Objective: the skin is acyanotic. Face is hydropic, there is short fibrillar twitching
of blepharons, muscles of the face and the inferior extremities. The look is fixed. AP- 200/110
mm Hg; sphygmus of 92 bpm, intense. Respiration rate is 32/min. Heart activity is rhythmical.
Appreciable edemata of the inferior extremities are present. Urine is cloudy. What medication
should be administered?
A. Droperidolum of 0,25% - 2,0 ml*
B. Dibazolum of 1% - 6,0 ml
C. Papaverine hydrochloride of 2% - 4,0 ml
D. Hexenalum of 1% - 2,0 ml
E. Pentaminum of 5% - 4,0 ml

12. A 40 year old woman has changes of mammary gland. What are the most often symtomps that
precede the malignizati-on?
A. Skin induration with inverted nipple*
B. Painful movable induration
C. Painless movable induration
D. Bloody discharges from the nipple
E. Pure discharges from the nipple

13. A patient who takes diuretics has developed arrhythmia as a result of cardiac glycoside
overdose. What is the treatment tactics in this case?
A. Increased potassium concentration in blood*
B. Increased sodium consentration in blood
C. Reduced magnesium concentration in blood
D. Increased calcium concentration in blood
E. -

14. An onset of severe preeclampsia at 16 weeks gestation might be caused by:


A. Hydatidiform mole*
B. Anencephaly
C. Twin gestation
D. Maternal renal disease
E. Interventricular defect of the fetus

3
Завантажено з сайту https://тестування.укр/ - онлайн тестування КРОК

15. A 9 year old girl with a history of intermittent wheezing for several years is brought to the
pediatrician. The child has been taking no medications for some time. Physical examination
reveals agitation and perioral cyanosis. Intercostal and suprasternal retractions are present.
The breath sounds are quiet, and wheezing is audible bilaterally. The child is admitted to the
hospital. Appropriate interventions might include all of the following EXCEPT:
A. Prescribe nebulized cromolyn sodium*
B. Prescribe intravenous aminophylline
C. Administer supplemental oxygen
D. Prescribe intravenous corticosteroids
E. Prescribe nebulized metaproterenol

16. Routine examination of an otherwise healthy child with a history of bronchial asthma reveals
AP of 140/90 mm Hg. The most likely cause of the hypertension is:
A. Renal disease*
B. Theophylline toxicity
C. Chronic lung disease
D. Coarctation of the aorta
E. Obesity

17. The average body lenth of newborn boys is 50,9 cm at a sigma 1,66; and average mass - 3432
at a sigma 5,00. What criterion is necessary to compare degree of variability of these signs?
A. Coefficient of variation*
B. Sigma
C. Limit
D. Amplitude
E. Coefficient of association

18. What method is applied to establish rate of correlation between age of men and their mortality
due to myocardial infarction?
A. Method of grade correlation (Spirman)*
B. Correlation ratio
C. The quadrate method (Pirson)
D. The Indirect method (Student)
E. Method of graduated correlation (Armler)

19. Adenosine triphosphate may be expected to convert which of the following arrhythmias to
sinus rhythm?
A. Paroxysmal supraventricular tachycardia*
B. Paroxysmal ventricular tachycardia
C. Atrial fibrillation
D. Atrial flutter
E. Ventricular fibrillation

20. A 48 year old man complains of fati-que and shortness of breath. His Ht is 32%, and Hb - 103
g/l. Peripheral blood smear reveals macrocytosis. Serum vitamin Bi2 level is 90 pg/ml (normal
is 170 to 940); serum folate level is 6 ng/ml (normal is 2 to 14). Possible causes to consider

4
Завантажено з сайту https://тестування.укр/ - онлайн тестування КРОК

include all of the following EXCEPT:


A. Colonic diverticulitis*
B. Vegetarianism
C. Regional enteritis
D. Pancreatitis
E. Fish tapeworm infection

21. A 56 year old man complains of fatigue, dyspnea on exertion and palpitations. He has had a
murmur since chi- ldhood. Examination reveals a lift at the left sternal border, split Si, and
fixed splitting of S2. There is a grade 3/6 midsystolic pulmonic murmur and a 1/6 middiastolic
tricuspid murmur at the lower left sternal border. Chest x-ray shows right ventricular
enlargement and prominent pulmonary arteries. ECG demonstrates atrial fibrillation with a
right bundle branch block. The most likely diagnosis is:
A. Arterial septal defect*
B. Coarctation of the aorta
C. Patent ductus arteriosus
D. Tetralogy of Fallot
E. Ventricular septal defect

22. Generalized low voltage on an ECG (QRS deflection < 5 mm in limb leads and <10 mm in
precordial leads) may be a marker for all of the following disorders EXCEPT:
A. Hyperthyroidism*
B. Pericardial effusion
C. Cardiac transplant rejection
D. Amyloidosis
E. Coronary artery disease

23. Five days after a total hip replacement a 72 year old woman becomes acutely short of breath,
diaphoretic and hypotensive. Both lung fields are clear to auscultation and percussion, but
examination of the neck reveals mild jugular venous distension with prominent A waves. Heart
sounds are normal. ECG shows sinus tachycardia with a new right bundle branch block and
minor nonspecific ST — T wave changes. The most likely diagnosis is:
A. Pulmonary thromboembolism*
B. Acute myocardial infarction
C. Aortic dissection
D. Pericarditis
E. Aspiration

24. A child is 6 years old. Within one year of observation he had URI that lasted 8 days. Physical
state is satisfactory. Specify hi health group:
A. I*
B. II
C. III (a)
D. III (b)
E. III (c)

5
Завантажено з сайту https://тестування.укр/ - онлайн тестування КРОК

25. Which of the following IS NOT TYPICAL for Addison’s disease?


A. High serum Na+*
B. High serum K+
C. Elevated BUN
D. Dilute urine
E. Elevated hematocrit

26. A 38 year old man, previously in good health, suddenly develops severe abdominal pain
radiating from the left loin to groin and accompanied by nausea, perspiration and the need for
frequent urination. He is restless, tossing in bed but has no abnormal findings. The most likely
diagnosis is:
A. Leftsided renal colic*
B. Herpes zoster
C. Sigmoid diverticulitis
D. Torsion of the left testicle
E. Retroperitoneal haemorrhage

27. A 40 year old woman has a selfdetected hard breast mass. The procedure of choice for
confirming the diagnosis is:
A. Excision biopsy*
B. Mammography
C. Thermography
D. Ultrasonography
E. Aspiration biopsy with cytology

28. A 38 year old man was admitted to a hospital from his working place on July 19 because of hip
fracture. He was invalid till November 19. Requires prolongation of treatment. Who decides on
the issue of further temporary invalidity?
A. Specialized (traumatologic) MSEC*
B. DCC
C. The head physician of a polyclinic
D. Interregional general MSEC
E. Regional MSEC

29. During investigation for chronic, severe, epigastric pain, a 40 year old alcoholic man is found
to have multiple areas of narrowing alternating with dilatation ("chain of lakes"appearance) of
the main pancreatic duct. The operation of choice is:
A. Lateral pancreaticojejunostomy*
B. Distal pancreaticojejunostomy
C. Sphincterotomy
D. Distal pancreatectomy
E. Total pancreatectomy

30. The treatment of choice for duodenal obstruction caused by secondary duodenal hematoma
that developed a few days after blunt abdominal injury is:
A. Nasogastric decompression and parenteral alimentation*

6
Завантажено з сайту https://тестування.укр/ - онлайн тестування КРОК

B. Retrocolic gastrojejunostomy
C. Duodenojejunostomy
D. Immediate exploration
E. Tube duodenostomy

31. A 25 year old man has a sudden onset of chest pain on the right and dyspnea. His trachea is
deviated to the left. All of the following would be anticipated EXCEPT:
A. Pleural friction rub on the left*
B. Absence of rhonchi
C. Absence of rales
D. Hyperresonance over the right chest
E. Distant breath sounds on the right

32. A 5 tons milk batch was sampled. The lab analysis revealed: fat content 2%, specific density
-1,04 g/cm3, acidity - 210T, reductase probe - weak-positive. What way is the product to be
used in?
A. Sell but inform customers about milk quality*
B. Discard for animal feeding
C. Technical utilization
D. Sell without limitations
E. Do the product away

33. What guarantees against the preconceived attitude to the physician in cases of professional
law violations do you know?
A. Sanction of public prosecutor, inquiry by preliminary investigator of prosecutor’s office,
committee of experts*
B. Draw up a statement about forensic medical examination
C. Conduct an inquiry by preliminary investigator of police department
D. Utilisation copy of medical documents
E. Conduct forensic medical examination by district forensic medicine expert

34. If a child has adherent fingers on his right hand, then what will be your diagnosis?
A. Syndactyly*
B. Polydactyly
C. Macrodactyly
D. Ectrodactyly
E. Ectromelia

35. A 16 year old patient with complaints of frequent pain in the abdomen was diagnosed with
melanoma, examination revealed also pigmentation of the mucosa and skin, polyp in the
stomach and large intestine. It is know that the patient’s mother has an analogous
pigmentation and has been often treated for anemia. What disease is suspected?
A. Peytz-Egers’s polyposis*
B. Chron’s disease
C. Tuberculosis of the intestine
D. Adolescent polyposis

7
Завантажено з сайту https://тестування.укр/ - онлайн тестування КРОК

E. Hirschprung’s disease

36. A 41 year old patient was admitted to the intensive care unit with haemorrhagic shock due to
gastric bleeding. He has a history of hepatitis B during the last 5 years. The source of bleeding
are esophageal veins. What is the most effective method for control of the bleeding?
A. Introduction of obturator nasogastric tube*
B. Intravenous administration of pituitrin
C. Hemostatic therapy
D. Operation
E. Administration of plasma

37. It is suspected that a 34 year old patient has an abscess of Douglas pouches. What diagnostic
method is to be chosen?
A. Digital examination of rectum*
B. Rectoromanoscopy
C. Laparoscopy
D. Percussion and auscultation of stomach
E. R-scopy of abdominal cavity

38. A 52 year old man has recurrent transient ischemic attacks. Auscultation of the carotid
arteries detected murmur. What diagnostic method is to be applied in the first place?
A. Ultrasound dopplerography*
B. CT of the brain
C. MRI of the brain
D. Cerebral angiography
E. Electroencephalography

39. A 34 year old woman in the 10th week of gestation (the second pregnancy) consulted a doctor
of antenatal clinic in order to be registered there. In the previous pregnancy hydramnion was
observed, the child’s birth weight was 4086 g. What examination method should be applied in
the first place?
A. The test for tolerance to glucose*
B. Determination of the contents of fetoproteinum
C. Bacteriological examination of discharges from vagina
D. A cardiophonography of fetus
E. US of fetus

40. A patient with acute purulent otitis media complicated by mastoiditis was admitted to a
hospital. Roentgenogram of mastoid processes showed the shadiowi-ng of the cellular system
on the lesion, absence of bone septa was present. What are the necessary therapeutic +actions
at the second stage of mastoiditis?
A. Mastoidotomy*
B. Paracentesis of the drum
C. Radical operation on the middle ear
D. Tympanoplasty
E. Cateterization of the Eustachian tube

8
Завантажено з сайту https://тестування.укр/ - онлайн тестування КРОК

41. A 22 year old patient was admitted to trauma center with complaints of pain in the left ankle
joint that was getting worse during moving and weight bearing. On the clinical examination it
was found that the patient had the closed fracture of medial malleolus without displacement.
In which position should the foot be fixed in plaster cast?
A. At right angle with varus positioning of the foot*
B. In position of planter flexion of foot
C. In position of pronation
D. In position of supination
E. In position of dorsal flexion of foot

42. A patient with acute respiratory viral infection (3rd day of disease) complains of pain in lumbar
region, nausea, dysuria, oliguria. Urinalysis - hematuria (100-200 RBC in eyeshot spot),
specific gravity -1002. The blood creatinin level is 0,18 mi-llimole/l, potassium level - 6,4
millimole/l. Make the diagnosis:
A. Acute interstitial nephritis*
B. Acute renal failure
C. Acute glomerylonephritis
D. Acute cystitis
E. Acute renal colic

43. A 3 month old infant suffering from acute segmental pneumonia has dyspnea (respiration rate
- 80 per minute), paradoxical breathing, tachycardia, total cyanosis. Respiration / pulse ratio is
1:2. The heart dullness under normal size. Such signs characterise:
A. Respiratory failure of III degree*
B. Respiratory failure of I degree
C. Respiratory failure of II degree
D. Myocarditis
E. Congenital heart malformation

44. A 3 year old child has been suffering from fever, cough, coryza, conjunctivitis for 4 days. He
has been taking sulfadimethoxine. Today it has fever up to 39oC and maculopapular rash on its
face. Except of rash the child’s skin has no changes. What is your diagnosis?
A. Measles*
B. Allergic rash
C. Rubella
D. Scarlet fever
E. Pseudotuberculosis

45. A 2 year old girl has been ill for 3 days. Today she has low grade fever, severe catarrhal
presentations, slight maculopapular rash on her buttocks and enlarged occipital lymph nodes.
What is your diagnosis?
A. Rubella*
B. Scarlet fever
C. Measles
D. Adenoviral infection
E. Pseudotuberculosis

9
Завантажено з сайту https://тестування.укр/ - онлайн тестування КРОК

46. A 3 year old boy fell ill abruptly: fever up to 39oC, weakness, vomi-tng. Haemorrhagic rash of
various size appeared on his lower limbs in 5 hours. Meningococcemia with infective - toxic
shock of the 1 degree was diagnosed. What medications should be administered?
A. Chloramphenicol succinate and prednisone*
B. Penicillin and prednisone
C. Penicillin and immunoglobulin
D. Chloramphenicol succinate and interferon
E. Ampicillin and immunoglobulin

47. A 7 year old girl has mild form of varicella. Headache, weakness, vertigo, tremor of her limbs,
ataxia, then mental confusion appeared on the 5th day of illness. Meningeal signs are negative.
Cerebrospinal fluid examination is normal. How can you explain these signs?
A. Encephalitis*
B. Meningitis
C. Meningoencephalitis
D. Myelitis
E. Neurotoxic syndrome

48. A 14 year old patient complains of chest pain, temperature up to 38,5oC, breathlessness. He
had acute tonsillitis 2 weeks ago. He is in grave condition. The skin is pale. Heart borders are
dilated, heart sounds are quiet. Above total heart area you can hear pericardium friction
sound. Electrocardiogram: the descent of QRS voltage, the inversion T. The liver is enlarged
by 3 cm. ESR - 4 mm/h, ASL - 0 - 1260, C-reactive protein +++. Your diagnosis:
A. Rheumatic pancarditis*
B. Rheumatic pericarditis
C. Rheumatic myocarditis
D. Rheumatic endocarditis
E. Septic endocarditis

49. A woman born a child. It was her fith pregnancy but the first delivery. Mother’s blood group is
A(II)Rh-, newborn’s -A(II)Rh+. The level of indirect bilirubin in umbilical blood was 58
micromole/l, hemoglobin -140 g/l, RBC- 3,8 • 1012/l. In 2 hours the level of indirect bilirubin
turned 82 micromole/l. The hemolytic disease of newborn (icteric-anemic type, Rh-
incompatibility) was diagnosed. Choose the therapeutic tactics:
A. Replacement blood transfusion (conservative therapy)*
B. Conservative therapy
C. Blood transfusion (conservative therapy)
D. Symptomatic therapy
E. Antibiotics

50. A mother consulted a pediatrician about her son. Her son was born with body mass of 3 kg and
length of 48 cm. He’s 1 year old now. What is the required normal mass?
A. 10,5 kg*
B. 9,0 kg
C. 11,0 kg
D. 12,0 kg
E. 15,0 kg

10
Завантажено з сайту https://тестування.укр/ - онлайн тестування КРОК

51. A 2 month old healthy infant with good appetite is given artificial feeding since he turned 1
month old. When is it recommended to start the corrective feeding (fruit juice)?
A. 4,0 months*
B. 1,5 months
C. 2,0 months
D. 3,0 months
E. 1,0 months

52. A nine year old child is at a hospital with acute glomerulonephritis. Clinical and laboratory
examinations show acute condition. What nutrients must not be limited during the acute
period of glomerulonephritis?
A. Carbohydrates*
B. Salt
C. Liquid
D. Proteins
E. Fats

53. Examination of a 3-month-old child revealed scrotum growth on the right. This formation has
elastic consistency, its size decreases during sleep and increases when the child is crying.
What examination will be helpful for making a correct diagnosis?
A. Palpation of the thickened cord crossing the pubical tubercule (sign of the silk glove)*
B. Diaphanoscopy
C. Palpation of the external inguinal ring
D. Puncture of the scrotum
E. Examination of the formation in Trendelenburg’s position

54. A rounded well-defined shadow was found in the costo-vertebral angle on the chest
roentgenogram of an otherwise healthy 9 year old girl. Make a preliminary diagnosis:
A. Ganglioneuroma*
B. Sympatoblastoma
C. Ganglioneuroblastoma
D. Sympatogonioma
E. Sarcoma of the vertebra

55. A 52 year old patient with history of functional Class II angina complains of having intense and
prolonged retrosternal pains, decreased exercise tolerance for 5 days. Angina is less
responsive to nitroglycerine. What is the most probable diagnosis?
A. IHD. Unstable angina*
B. Cardialgia due to spine problem
C. IHD. Functional Class II angina
D. Myocarditis
E. Myocardial dystrophy

56. A 52 year old patient has hypervolaemic type of essential hypertension. Which of the following
medications is to be prescribed either as monotherapy or in complex with other
antihypertensive drugs?

11
Завантажено з сайту https://тестування.укр/ - онлайн тестування КРОК

A. Hypothiazid*
B. Dibazol
C. Clonidine
D. Kapoten
E. Nifedipin

57. A 62 year old patient complains of rest dyspnea, heart pains. 3 years ago he had myocardial
infarction. Physical examination: orthopnea, acrocyanosis, swollen cervical veins. Ps - 92, total
heart enlargement, the liver is enlarged by 7 cm, shin edema. What is the stage of chronic
heart failure (CHF)?
A. CHF- 2 B*
B. CHF- 1
C. CHF- 2 A
D. CHF- 0
E. CHF- 3

58. A patient, aged 49, complains of fever of 37,5oC, heart pain, dyspnea. Si is clapping; S2 is
accentuated in the aortic area; opening snap, presystolic murmur can be auscultated. What is
the most efficient examination for valvular disorder assessment?
A. Echocardiography+Doppler-Echocardiography*
B. Phonocardiography
C. Ballistocardiogram
D. Chest X-ray
E. ECG

59. A patient with nosocomial pneumonia presents signs of collapse. Which of the following
pneumonia complications is most likely to be accompanied by collapse?
A. Septic shock*
B. Exudative pleuritis
C. Bronchial obstruction
D. Toxic hepatitis
E. Emphysema

60. A patient, aged 48, complains of heaviness in the right hypochondrium, itching of the skin.
Repeatedly he had been treated in infectious diseases hospital because of icterus and itch.
Objectively: meteorism, ascitis, dilation of abdominal wall veins, protruding navel, spleen
enlargement. Diagnosis is:
A. Liver cirrhosis*
B. Cancer of the liver
C. Cancer of the head of pancreas
D. Gallstones
E. Viral hepatitis B

61. A 27 year old man complains of pains in epigastrium which are relieved by food intake. EGDFS
shows antral erosive gastritis, biopsy of antral mucous presents Helicobacter Pylori. Diagnosis
is:

12
Завантажено з сайту https://тестування.укр/ - онлайн тестування КРОК

A. Gastritis of type B*
B. Gastritis of type A
C. Reflux-gastritis
D. Menetrier’s gastritis
E. Rigid antral gastritis

62. A 62 year old patient suffers from DM-2. Diabetes is being compensated by diet and
Maninilum. The patient has to undergo an operation on inguinal hernia. What tactics of
hypoglycemic therapy should be chosen?
A. Prescribe fast-acting insulin*
B. Give Glurenorm instead of Maninilum
C. Continue with the current therapy
D. Prescribe long-acting insulin
E. Prescribe guanyl guanidines

63. Prophylactic photoroentgenography examination of a 25 year old man revealed focal


shadowings of small and medium intensity with irregular contours in the 1st and 2nd segments
of the right lung. Which clinical form can be suspected?
A. Focal*
B. Disseminated
C. Miliary
D. Fibro-cavernous
E. Tuberculoma

64. A triad of symptoms ("stearing spot", "terminal film", "blood dew") has been revealed on
examination of a patient. What disease should you think about?
A. Psoriasis*
B. Lichen ruber planus
C. Vasculitis
D. Seborrhea
E. Ritter’s disease

65. A female 28 years old patient became depressed, her mood is melancholic; this state is
accompanied by hypobulia, hypokinesia, slow speed of thinking. Her attitude towards her past,
present and future is pessimistic. The pathogenetic mechanism of this state is supposed to
involve dysfunction in the:
A. Hypothalamus*
B. Frontal lobes
C. Pituitary
D. Hippocampus
E. Corpus callosum

66. The observed patient’s movements are retarded, she answers no questions. Sometimes she
spontaneously stays in strange postures. It is possible to set her body and limbs into different
positions artificially. If the psychiatrist lifts her arm or leg, so that she remains standing on the
other leg, the patient can stay in such a position for quite a long time. Name the probable

13
Завантажено з сайту https://тестування.укр/ - онлайн тестування КРОК

disorder:
A. Catatonic stupor, schizophrenia*
B. Depressive stupor, bipolar disorder
C. Apathetic stupor, schizophrenia
D. Psychogenic stupor, stress disorder
E. Dissociative stupor, dissociative psychosis

67. A 50 year old patient has been admitted to the clinics with atrophic gastritis. Blood count:
erythrocytes -3,8 • 1012/l, Hb - 68 g/l, c.i. - 1, macroanisocytosis, poikilocytosis. There is
megaloblastic type of haemopoesis. A number of leukocytes, reticulocytes and thrombocytes is
lreduced. Which pathology is suspected?
A. B12-deficiency anemia*
B. Irondeficiency anemia
C. Hemolytic anemia
D. Post-hemorrhagic anemia
E. Thalassaemia

68. A 28 year old patient was admitted to the clinic with complaints of the temperature rise up to
39,0oC, headache, weakness, constipation on the 9th day of the disease. On examination:
single roseolas on the skin of the abdomen are present. The pulse rate is 78 bpm. The liver is
enlarged by 2 cm. What is the most probable diagnosis?
A. Typhoid fever*
B. Leptospirosis
C. Brucellosis
D. Sepsis
E. Malaria

69. A patient has been in a hospital. The beginning of the disease was gradual: nausea, vomiting,
dark urine, acholic stools, yellowness of the skin and scleras. The liver is protruded by 3 cm.
Jaundice progressed on the 14th day of the disease. The liver diminished in size. What
complication of viral hepatitis caused deterioration of the patient’s condition?
A. Hepatic encephlopathy*
B. Meningitis
C. Relapse of viral hepatitis
D. Cholangitis
E. Infectious-toxic shock

70. An 18 year old patient was admitted to a hospital with complaints of headache, weakness, high
temperature, sore throat. Objectively: enlargement of all groups of lymph nodes was revealed.
The liver is enlarged by 3 cm, spleen - by 1 cm. In blood: leukocytosis, atypical lymphocytes -
15%. What is the most probable diagnosis?
A. Infectious mononucleosis*
B. Acute lymphoid leukosis
C. Diphtheria
D. Angina
E. Adenoviral infection

14
Завантажено з сайту https://тестування.укр/ - онлайн тестування КРОК

71. A 75 year old man who has been suffering from diabetes for the last six months was found to
be jaundiced. He was asymptomatic except for weight loss at the rate of 10 pounds in 6
months. Physical examination revealed a hard, globular, right upper quadrant mass that moves
during respiration. A CT scan shows enlargement of the head of the pancreas, with no filling
defects in the liver. The most likely diagnosis is:
A. Carcinoma of the head of the pancreas*
B. Infectious hepatitis
C. Haemolytic jaundice
D. Malignant biliary stricture
E. Metastatic disease of liver

72. A 33 year old male patient was brought to Emergency Department with the signs of
cardiovascular collapse: BP - 60/30 mm Hg, Ps - 140 bpm, the skin is pale and moist, diuresis
20 ml/h, Hb - 80 g/l, red blood cell count - 2,5 • 1012/l. The reduction of blood volume
averages:
A. 30-40%*
B. 10-15%
C. 15-20%
D. 20-25%
E. 25-30%

73. A 19 year old girl was admitted to emergency department: unconsciousness, cyanosis, myotic
pupils are present, superficial breathing is 12/min. BP is 90/60 mm Hg, Ps- 78/min. Choose the
action necessary in this clinical situation:
A. Controlled respiration*
B. Gastric lavage
C. Oxygen inhalation
D. Caffeine injection
E. Cordiamine injection

74. A pregnant woman may be diagnosed with hepatitis if it is confirmed by the presence of
elevated:
A. SGOT (ALT)*
B. Sedimentation rates
C. WBCs
D. Alkaline phosphatase
E. BUN

75. An endometrial adenocarcinoma that has extended to the uterine serosa would be classified as
stage:
A. IIIA*
B. IC
C. IIA
D. IIB
E. IVAB

15
Завантажено з сайту https://тестування.укр/ - онлайн тестування КРОК

76. A 56 year old woman has an acute onset of fever up to 39°C with chills, cough, and pain on
respiration in the right side of her chest. On physical examination: HR - 90/min, BP- 95/60 mm
Hg, RR-26/min. There is dullness over the right lung on percussion. On X-ray: infiltrate in the
right middle lobe of the lung. What is the diagnosis?
A. Community-acquired lobar pneumonia of moderate severity*
B. Community-acquired bronchopneumonia
C. Acute pleuritis
D. Acute lung abscess
E. Nosocomial lobar pneumonia

77. A 26 year old man was admitted to the hospital because of stabbing back pain on inspiration
and dyspnea. Examination results: BT of 37°C, Ps of 24/min, HR of 92/min, vesicular breath
sounds. There is a dry, grating, low-pitched sound heard on both expiration and inspiration in
the left inferior lateral part of the chest. What is the most likely diagnosis?
A. Acute fibrinous pleuritis*
B. Myocarditis
C. Pneumonia
D. Acute bronchitis
E. Pneumothorax

78. A 34 year old woman fell ill 3 months ago after cold exposure. She complained of pain in her
hand and knee joints, morning stiffness and fever up to 38oC. Interphalangeal,
metacarpophalangeal and knee joints are swollen, hot, with reduced ranges of motions; ESR of
45 mm/h, CRP (+++), Vaaler-Rouse test of 1:128. What group of medicines would you
recommend the patient?
A. Nonsteroidal anti-inflammatory drugs*
B. Cephalosporines
C. Tetracyclines
D. Sulfonamides
E. Fluorchinolones

79. A 50 year old woman complained of attacks of right subcostal pain after fatty meal she has
been suffering from for a year. Last week the attacks repeated every day and became more
painful. What diagnostic study would you recommend?
A. Ultrasound examination of the gallbladder*
B. Liver function tests
C. X-ray examination of the gastrointestinal tract
D. Ultrasound study of the pancreas
E. Blood cell count

80. A 25 year old woman complained of edema on her face and legs, rise of blood pressure up to
160/100 mm Hg and weakness. She fell ill 3 weeks after recovering from angina. Urinalysis
data: protein of 0,5 g/l, erythrocytes of 17-20/field, leukocytes of 2-3/field, erythrocyte casts.
What treatment should be initiated after specifying the diagnosis?
A. Penicillin OS*
B. Heparin
C. Ceftriaxone

16
Завантажено з сайту https://тестування.укр/ - онлайн тестування КРОК

D. Dipyridamole
E. Ciprofloxacine

81. A fitter of a metallurgic factory with occupational exposure to high concentrations of mercury
fumes for 16 years presents instability of pulse and blood pressure, general hyperhydrosis,
asymmetric innervations of facial muscles and tongue, positive subcortical reflexes, hand
tremor on physical examination. A dentist revealed paradontosis and chronic stomatitis. What
is the most probable diagnosis?
A. Chronic mercury intoxication*
B. Neuroinfection
C. Parkinson syndrome
D. Acute mercury intoxication
E. Mercury encephalopathy

82. A 6 week old child is admitted because of tachypnea. Birth had been uneventful, although
conjunctivitis developed on the third day of life and lasted for about 2 weeks. Physical
examination reveals tachypnea, bilateral inspiratory crackles and single expiratory wheezing.
Bilateral pneumonia is evident on chest X-ray. The child is afebrile and has no history of fever.
White blood cell count is 15 • 109/l, with 28% of eosinophils. The most likely cause of this
child’s symptoms is:
A. Clamydia trachomanis*
B. Pneumocystis carinii
C. Mycoplasma pneumoniae
D. Visceral larva migrans
E. Varicella

83. A 42 year old woman complains of dyspnea, edema of the legs and tachycardia during minor
physical exertion. Heart borders are displaced to the left and S1 is accentuated, there is
diastolic murmur on apex. The liver is enlarged by 5 cm. What is the cause of heart failure?
A. Mitral stenosis*
B. Mitral regurgitation
C. Tricuspid stenosis
D. Tricuspid regurgitation
E. Aortic stenosis

84. A 60 year old man complains of fever, significant weight loss, bone and joint pain, bleeding
gums. Examination revealed paleness, lymphadenopathy, hepato- and splenomegaly. CBC:
WBC - 270 • 109/l with 13% lymphocytes, 1% monocytes, 21% basophiles, 29% neutrophils,
9% blasts, 12% promyelocytes, 12% myelocytes, 2% metamyelocytes, 1% eosinophils. ESR -22
mm/h. Name the drug for treatment:
A. Myelosan*
B. Prednisolone
C. Cytosar
D. Vinblastine
E. Blood transfusion

17
Завантажено з сайту https://тестування.укр/ - онлайн тестування КРОК

85. A 54 year old woman complains of increasing fatigue and easy bruising of 3 weeks’ duration.
Physical findings included pale, scattered ecchymoses and petechiae and mild
hepatosplenomegaly. Blood count: RBC- 2,5 • 1012//; Hb - 73 g/l; Ht - 20%; PLT- 23 • 109//;
and WBC-162 • 109// with 82% blasts, that contained Auric rods; peroxidase stain was
positive. What is the most probable diagnosis?
A. Acute leukemia*
B. Chronic leukemia
C. Thrombocytopenia
D. Hemolytic anemia
E. Megaloblastic anemia

86. The highest risk of congenital anomalies probably occurs when human embryos or fetuses are
exposed to ionizing radiation. At what stage of gestational period does it occur?
A. 18-45 days after conception*
B. The first 7 days
C. 10-14 days after conception
D. 90-120 days after conception
E. The third trimester

87. A 28 year old woman was admitted to the emergency room with a slightly reddened, painful
"knot"8 cm above the medial malleolus. Examination in the standing position demonstrates a
distended vein above and below the mass. There are no other abnormalities on physical
examination. The most likely diagnosis is:
A. Superficial venous thrombosis*
B. Early deep vein thrombosis
C. Insect bite
D. Cellulitis
E. Subcutaneous hematoma

88. A 25 year old patient complains of pain in the I finger on his right hand. On examination: the
finger is homogeneously hydropic, in bent position. On attempt to unbend the finger the pain
is getting worse. Acute pain appears during the probe in ligament projection. What decease is
the most likely?
A. Thecal whitlow (ligament panaritium)*
B. Subcutaneous panaritium
C. Articular (joint) panaritium
D. Bone panaritium
E. Paronychia

89. A 30 year old man complains of acute pain in his right ear, hearing loss, high temperature for
three days. Objectively: right ear whispering language - 0,5 mm, external ear is intact,
otoscopically - eardrum protrusion, hyperemia and swelling, loss of landmarks. What disease is
it?
A. Acute purulent otitis media*
B. Acute mastoiditis
C. Chronic secretory otitis media
D. Chronic purulent otitis media

18
Завантажено з сайту https://тестування.укр/ - онлайн тестування КРОК

E. Eustachian tube disfunction

90. Among the following causes of acute renal failure, the one that would be classified as
"postrenal"is:
A. Calculi*
B. Cardiac failure
C. Septicemia
D. Rhabdomyolysis
E. Acute glomerulonephritis

91. A patient complains about strong dyspnea that is getting worse during physical activity.
Presentations appeared suddenly 2 hours ago at work: acute chest pain on the left, cough. The
pain was abating, but dyspnea, dizziness, pallor, cold sweat and cyanosis were progressing.
Vesicular respiration is absent, X-ray picture shows a shadow on the left. What pathology
might be suspected?
A. Spontaneous left-sided pneumothorax*
B. Pulmonary infarction
C. Pleuritis
D. Left-sided pneumonia
E. Pulmonary abscess

92. During an operation for presumed appendicitis the appendix was found to be normal; however,
the terminal ileum is evidently thickened and feels rubbery, its serosa is covered with grayish-
white exudate, and several loops of apparently normal small intestine are adherent to it. The
most likely diagnosis is:
A. Crohn’s disease of the terminal ileum*
B. Perforated Meckel’s diverticulum
C. Ulcerative colitis
D. Ileocecal tuberculosis
E. Acute ileitis

93. A 50 year old woman with a 2-year history of mild, diffuse, tender thyroid enlargement
complains of 10 pound weight gain and fatigue. What is the most probable diagnosis?
A. Hashimoto’s thyroiditis*
B. Riedel’s thyroiditis
C. Subacute thyroiditis
D. Suppurative thyroiditis
E. Papillary thyroid carcinoma

94. A severely traumatized patient who has been receiving prolonged parenteral alimentation
develops diarrhea, mental depression, alopecia and perioral and periorbital dermatitis.
Administration of which of the following trace elements is most likely to reverse these
complications?
A. Zinc*
B. Iodine
C. Selenium

19
Завантажено з сайту https://тестування.укр/ - онлайн тестування КРОК

D. Silicon
E. Copper

95. A patient has got pain in the axillary area, rise of temperature developed 10 hours ago. On
examination: shaky gait is evident, the tongue is coated with white deposit. The pulse is
frequent. The painful lymphatic nodes are revealed in the axillary area. The skin over the
lymph nodes is erythematous and glistering. What is the most probable diagnosis?
A. Bubonic plague*
B. Acute purulent lymphadenitis
C. Lymphogranulomatosis
D. Anthrax
E. Tularemia

96. A 56 year old patient has worked at the aluminium plant over 20 years. Within 3 last years he
has been experiencing loosening of teeth, bone and joint pains, piercing pains in heart region,
vomiting. The provisional diagnosis is:
A. Fluorine intoxication*
B. Mercury intoxication
C. Lead intoxication
D. Phosphorus intoxication
E. Manganese intoxication

97. An excited patient constantly tries to look into the next room because he is sure of his friends’
presence. He claims to hear his friends and other people talking although no-one is there. He
tries to persuade his doctor that people in the next room are disputing about "his punishment",
repeats aloud phrases he has allegedly heard from the next room. What state is it?
A. Acute hallucinosis*
B. Verbal illusions
C. Delusion
D. Confabulations
E. Obsessional ideas

98. Survey radiograph of chest of a 62 year old smoker who often suffers from
"pneumonias"showed a triangle shadow in the right lung, its vertex is pointing towards the
lung root. It also showed devi- ation of heart and mediastinum shadows towards the lesion.
What is the most probable diagnosis?
A. Cenral cancer of lung*
B. Lung abscess
C. Lung cyst
D. Peripheral cancer of lung
E. Atelectasis

99. A 68 year old patient has been suffering from chronic pancreatitis for 35 years. During the last
5 years he has been observing abatement of pain syndrome, abdominal swelling, frequent
defecations up to 3-4 times a day (feces are greyish, glossy, with admixtures of undigested
food), progressing weight loss. Change of symptom set is caused by joining of:

20
Завантажено з сайту https://тестування.укр/ - онлайн тестування КРОК

A. Exocrine pancreatic insufficiency*


B. Endocrine pancreatic insufficiency
C. Syndrome of lactase deficiency
D. Irritable bowels syndrome
E. Chronic enterocolitis

100. A 60 year old patient has been suffering from arterial hypertension for 15 years. After
recurrent stroke she started complaining about amotivati-onal bad mood, problems with
attention concentration; she forgets to close the entrance door, cannot recall events of the last
day. Computer tomography shows areas of postinfarction changes in the cortical postfrontal
regions. What is the most probable diagnosis?
A. Vascular dementia*
B. Alzheimer’s disease
C. Huntington’s chorea
D. Pick’s disease
E. Dissociative amnesia

101. A 40 year old patient complains of yellowish discharges from the vagina. Bimanual
examination revealed no pathological changes. The smear contains Trichomonas vaginalis and
blended flora. Colposcopy revealed two hazy fields on the frontal labium, with a negative
Iodine test. Your tactics:
A. Treatment of specific colpitis and subsequent biopsy*
B. Diathermocoagulation of the cervix of the uterus
C. Specific treatment of Trichomonas colpitis
D. Cervix ectomy
E. Cryolysis of cervix of the uterus

102. A patient complained about problems with pain and tactile sensitivity, pain in the nail bones at
the end of the working day. He works at a plant with mechanical devices. What pathology can
be suspected?
A. Vibration disease*
B. Caisson disease
C. Noise disease
D. Overwork symptoms
E. Hypovitaminosis of B1

103. A 25 year old patient complains about weakness, dizziness, appearance of haemorrhagic skin
rash. She has been suffering from this for a month. Blood count: erythrocytes: 1,0 • 1012/l,
Hb- 37 g/l, colour index - 0,9, leukocytes - 1,2 • 109/l, thrombocytes - 42 • 109/l. What
diagnostic method will be the most effective?
A. Sternal puncture*
B. Spleen biopsy
C. Liver biopsy
D. Coagulogram
E. Abdominal ultrasound

21
Завантажено з сайту https://тестування.укр/ - онлайн тестування КРОК

104. A 28 year old woman had the second labour and born a girl with manifestations of anemia and
progressing jaundice. The child’s weight was 3 400 g, the length was 52 cm. The woman’s
blood group is B (III) Rh-, the father’s blood group is A (III) Rh+, the child’s blood group is B
(III) Rh+. What is the cause of anemia?
A. Rhesus incompatibility*
B. Antigen A incompatibility
C. Antigen B incompatibility
D. Antigen AB incompatibility
E. Intrauterine infection

105. A 25 year old patient was admitted on the 1st day of the disease with complaints of double
vision in the eyes, heavy breathing. The day before the patient ate home-made mushrooms. On
objective examination: paleness, mydriatic pupils, difficult diglutition, bradycardia,
constipation. What is the diagnosis?
A. Botulism*
B. Yersiniosis
C. Leptospirosis
D. Salmonellosis, gastrointestinal form
E. Lambliasis

106. Maximum permissible concentration of carbon dioxide in the air is considered to be a sanitary
index of air purity in a classroom. What concentration of carbon dioxide in the air is accepted
as maximum permissible?
A. 0,1%*
B. 0,05%
C. 0,15%
D. 0,2%
E. 0,3%

107. Study of actual diet of an adult revealed the following: proteins make up 16% of energy value
of daily ration, fats - 25%, carbohydrates - 59%. Evaluate compliance of protein, fat and
carbohydrate share in the energy value of daily ration with the recommended shares of these
nutrients?
A. Carbohydrate share is insufficient, there is excess of proteins*
B. Fat share is insufficient
C. Carbohydrate share is insufficicent
D. Carbohydrate share is excessive
E. Nutrient content complies with the recommended shares of energy value

108. A healthy 75 year old woman who leads a moderately active way of life went through a
preventive examination that revealed serum concentration of common cholesterol at the rate
of 5,1 millimol/l and HDL (high-density lipoproteins) cholesterol at the rate of 70 mg/dl. ECG
reveals no pathology. What dietary recommendation is the most adequate?
A. Any dietary changes are necessary*
B. Decrease of cholesterol consumption
C. Decrease of saturated fats consumption
D. Decrease of carbohydrates consumption

22
Завантажено з сайту https://тестування.укр/ - онлайн тестування КРОК

E. Increase of cellulose consumption

109. Periodical survey of a worker of a chemicals plant revealed a malignant neoplasm on the
urinary bladder. This occupational disease was the most probably caused by contact with the
following industrial poison:
A. Benzidine*
B. Vinyl chloride
C. Nickel carbonyl
D. Asbestos
E. Arsenic

110. A 52 year old patient was admitted to a hospital because of high hemorrhagic diathesis of
mucous membranes, massive skin haemorrhages in form of ecchymoses and spots, nasal and
stomachal haemorrhages. After clinical examinations her illness was diagnosed
A. Generation of antithrombocytic antibodies*
B. Disturbed hemostasis
C. Deficit of the VIII factor of blood coagulation
D. Inherited insufficiency of plasm factors of blood coagulation
E. Iron deficit in blood serum, bone marrow and depot

111. A child with tetralogy of Fallot is most likely to exhibit:


A. Increased pressure in the right ventricle*
B. Increased pulmonary blood flow
C. Increased pulse pressure
D. Normal pressure gradient across the pulmonary valve
E. Normal oxygen tension (PaO2) in the left ventricle

112. In treatment and prevention establishments, regardless of their organisational and proprietary
form, the rights of patients should be observed. Which of these rights is the most significant?
A. The right to the protection of the patient’s interests*
B. The right to the free choice
C. The right to the information
D. The right to be heard
E. The right to the protection from incompetence

113. A military unit stopped for 3-day’s rest in an inhabited locality after a long march. The
sanitary-epidemiological reconnaissance found several water sources. It is necessary to choose
the source complying with the hygienic standards for drinking water in the field conditions:
A. Artesian well water*
B. Spring water
C. River water
D. Rain water
E. Melt snow water

114. A man in grave condition was delivered to the admission ward of a hospital on the 2nd day of

23
Завантажено з сайту https://тестування.укр/ - онлайн тестування КРОК

illness. Examination revealed body temperature of 36,1oC, sharpened features of face, dry skin
that makes a fold, aphonia, convulsive twitching of some muscle groups. Acrocyanosis is
present. Heart sounds are muffled, Ps is 102 bpm, AP is 50/20 mm Hg. Abdomen is soft,
drawn-in, painless. Anuria is presnt. Stool is liquid in form of rice water. What is the most
probable diagnosis?
A. Cholera*
B. Acute dysentery
C. Salmonellosis
D. Escherichiosis
E. Intestinal amebiasis

115. At year-end hospital administration has obtained the following data: annual number of treated
patients and average annual number of beds used for patients’ treatment. What index of
hospital work can be calculated on the base of this data?
A. Bed turnover*
B. Bed resources of the hospital
C. Average annual bed occupancy
D. Average duration of patients’ presence in the hospital
E. Average bed idle time

116. A 3 year old child with weight deficiency suffers from permanent moist cough. In history there
are some pneumonias with obstruction. On examination: distended chest, dullness on
percussion over the lower parts of lungs. On auscultation: a great number of different rales.
Level of sweat chloride is 80 millimol/l. What is the most probable diagnosis?
A. Mucoviscidosis (cystic fibrosis)*
B. Bronchial asthma
C. Recurrent bronchitis
D. Bronchiectasis
E. Pulmonary hypoplasia

117. A 14 year old girl complains of profuse bloody discharges from genital tracts during 10 days
after suppresion of menses for 1,5 month. Similiar bleedings recur since 12 years on the
background of disordered menstrual cycle. On rectal examination: no pathology of the internal
genitalia. In blood: Hb - 70 g/l, RBC-2, 3 • 1012/l, Ht - 20. What is the most probable
diagnosis?
A. Juvenile bleeding, posthemorrhagic anemia*
B. Werlholf’s disease
C. Polycyst ovarian syndrome
D. Hormonoproductive ovary tumor
E. Incomplete spontaneous abortion

118. A 43 year old patient had cholecystectomy 6 years ago because of chronic calculous
cholecystitis. Lately he has been suffering from pain in the right subcostal area and recurrent
jaundice. Jaundice hasn’t gone for the last 2 weeks. Stenosing papillitis 0,5 cm long has been
revealed. What is the best way of treatment?
A. To perform endocsopic papillosphincterotomy*
B. To treat conservatively: antibiotics, spasmolytics, antiinflammatory drugs

24
Завантажено з сайту https://тестування.укр/ - онлайн тестування КРОК

C. To perform external choledoch drainage


D. To perform transduodenal papillosphincterotomy
E. To perform choledochoduodenostomy

119. A full-term infant is 3 days old. On the different parts of skin there are erythemas, erosive
spots, cracks, areas of epidermis peeling. The infant has scalded skin syndrome. Nikolsky’s
symptom is positive. General condition of the infant is grave. Anxiety, hyperesthesia, febrile
temperature are evident. What is the most probable diagnosis?
A. Exfoliative dermatitis*
B. Phlegmon of newborn
C. Finger’s pseudofurunculosis
D. Impetigo neonatorum
E. Mycotic erythema

120. An outpatient hospital made record of 11600 diseases within one year. Among them influenza
and ARD make up 5800, circulatory system diseases - 3480, digestion diseases - 1300, other
diseases - 1020. What relative index can be calculated according this data?
A. Extensive*
B. Intensive
C. Visualization
D. Correlation
E. -

121. A 35 year old woman consulted a doctor about affection of arm skin and lower third of forearm
in form of a large edema, hyperemia, vesiculation and maceration. The disease developed after
using a laundry detergent "Lotos". The patient has been using it for a month. She hasn’t
suffered from dermatological diseases before. What is the most probable diagnosis?
A. Allergic dermatitis*
B. Dermatitis simplex
C. Toxicoallergic dermatitis
D. Microbial eczema
E. Localized neurodermatitis

122. A 32 year old patient complains about heartburn and dull pain in the epigastrium that appear
2-3 hours after meal. Exacerbations happen in spring and in autumn. The patient has food
intolerance of eggs and fish. Objectively: stomach palpation reveals painfulness in the
gastroduodenal area. Electrophasoduodenoscopy revealed a 5 mm ulcer on the anterior wall of
duodenum. Urease test is positive. What is the most probable leading mechanism of disease
development?
A. Chelicobacterial infection*
B. Dietary allergy
C. Autoantibody production
D. Reduced prostaglandin synthesis
E. Disorder of gastric motor activity

123. A 48 year old woman complains about weakness, weight loss, appetite loss, headache. The

25
Завантажено з сайту https://тестування.укр/ - онлайн тестування КРОК

patient had acute glomerulonephritis when she was young. She has been suffering from
arterial hypertension since she was 25. She didn’t undergo systematic treatment, consulted a
doctor rarely. Examination revealed signs of chronic renal insufficiency of the I stage
(creatinine - 0,43 millimole/l). What dietary recommendations are the most reasonable?
A. Restriction of protein consumption*
B. Restriction of fat consumption
C. Restriction of carbohydrate consumption
D. Diet with high content of "alkali-ne"dishes
E. Consumption of higher amounts of liquid

124. A 50 year old woman complains about dull cardiac pain, asphyxia, body temperature rise up to
38oC. She had influenza a week ago. Objectively: Ps -100 bpm, dropped-beat pulse during
inspiration. AP - 100/70 mm Hg, heart sounds are muffled. ECG: reduced voltage, ST segment
is above the isoline in all leads. X-ray picture shows extensively enlarged cardiac silhouette.
Palmus is of small amplitude. What is the most probable diagnosis?
A. Exudative pericarditis*
B. Myocardium infarction
C. Dilatation cardiomyopathy
D. Myocarditis
E. Stenocardia

125. Examination of a 4 month old child revealed some lemon-yellow squamae with fatty crusts on
the scalp. What is the most probable diagnosis?
A. Gneiss*
B. Milk crust
C. Strophulus
D. Pseudofurunculosis
E. Infantile eczema

126. A 39 year old patient complained about morning headache, appetite loss, nausea, morning
vomiting, periodic nasal haemorrhages. The patient had acute glomerulonephritis at the age of
15. Examination revealed rise of arterial pressure up to 220/130 mm Hg, skin haemorrhages
on his arms and legs, pallor of skin and mucous membranes. What chemestry test has the
greatest diagnostic importance in this case?
A. Blood creatinine*
B. Blood bilirubin
C. Blood sodium
D. Uric acid
E. Fibrinogen

127. A 14 year old child suffers from vegetovascular dystonia of pubertal period. He has got
sympathoadrenal atack. What medicine should be used for attack reduction?
A. Obsidan*
B. No-shpa
C. Amysyl
D. Aminophylline
E. Corglicone

26
Завантажено з сайту https://тестування.укр/ - онлайн тестування КРОК

128. A 23 year old man complains about face edemata, headache, dizziness, reduced urination,
change of urine colour (dark-red). These presentations appeared after pharyngitis. Objectively:
face edemata, pale skin, temperature -37,4oC; heart rate - 86/min, AP - 170/110 mm Hg. Heart
sounds are muffled, the II sound is accentuated above aorta. What etiological factor is
probable in this case?
A. (3-hemolytic streptococcus*
B. Staphylococcus aureus
C. Alpha-hemolytic streptococcus
D. Pyogenic streptococcus
E. Saprophytic staphylococcus

129. A 12 year old girl complains about abrupt weakness, nausea, dizziness, vision impairment. The
day before she ate home-made stockfish, beef. Examination revealed skin pallor, a scratch on
the left knee, dryness of mucous membranes of oral pharynx, bilateral ptosis, mydriatic pupils.
The girl is unable to read a simple text (mist over the eyes). What therapy
A. Parenteral introduction of polyvalent antibotulinic serum*
B. Parenteral disintoxication
C. Parenteral introduction of antibiotics
D. Gastric lavage
E. Parenteral introduction of antitetanus serum

130. A 19 year old patient was admitted to a hospital with acute destructive appendicitis. He suffers
from hemophilia of B type. What antihemophilic medications should be included in pre- and
postoperative treatment plan?
A. Fresh frozen plasma*
B. Cryoprecipitate
C. Fresh frozen blood
D. Native plasma
E. Dried plasma

131. An 18 year old woman consulted a gynecologist about the pain in the lower part of abdomen,
fever up to 37,5oC, considerable mucopurulent discharges from the genital tracts, painful
urination. Vaginal and speculum examination results: the urethra is infiltrated, cervix of the
uterus is hyperemic, erosive. The uterus is painful, ovaries are painful, thickened; fornixes are
free. Bacterioscopy test revealed diplococcus. What diagnosis is the most probable?
A. Recent acute ascending gonorrhea*
B. Trichomoniasis
C. Candydomycosis
D. Chronic gonorrhea
E. Chlamydiosis

132. A 26 year old woman who delivered a child 7 months ago has been suffering from nausea,
morning vomiting, sleepiness for the last 2 weeks. She suckles the child, menstruation is
absent. She hasn’t applied any contraceptives. What method should be applied in order to
specify her diagnosis?
A. Ultrasonic examination*
B. Roentgenography of small pelvis organs

27
Завантажено з сайту https://тестування.укр/ - онлайн тестування КРОК

C. Palpation of mammary glands and pressing-out of colostrum


D. Bimanual vaginal examination
E. Speculum examination

133. A 2 month old full-term child was born with weight 3500 g and was on the mixed feeding.
Current weight is 4900 g. Evaluate the current weight of the child:
A. Corresponding to the age*
B. 150 g less than necessary
C. Hypotrophy of the I grade
D. Hypotrophy of the II grade
E. Paratrophy of the I grade

134. A woman consulted a doctor on the 14th day after labour about sudden pain, hyperemy and
induration of the left mammary gland, body temperature rise up to 39oC, headache,
indisposition. Objectively: fissure of nipple, enlargement of the left mammary gland, pain on
palpation. What pathology would you think about in this case?
A. Lactational mastitis*
B. Lacteal cyst with suppuration
C. Fibrous adenoma of the left mammary gland
D. Breast cancer
E. Phlegmon of mammary gland

135. A 32 year old patient complai- ns about pain in small joints of her hands, paresthesia at the
tips of fingers, weakness, difficult diglutition. She has been suffering from this for 13 years.
Objectively: face amimia, shortening of nail bones, skin indurations in the area of shoulder
girdle are present. Roentgenological examination of lungs revealed basal pneumosclerosis.
Fibrogastroscopy revealed esophagus constriction in its cardial part. Blood count: leukocytes -
9,8 • 109/l, ESR - 22 mm/h, y-globulin - 22%. What is the most probable diagnosis?
A. Systemic scleroderma*
B. Systemic lupus erythematosus
C. Rheumatoid arthritis
D. Dermatomyositis
E. Myxedema

136. A 30 year old woman ill with influenza felt palpitation and dull cardiac pain during moderate
physical exercise. Objectively: Ps - 96 bpm, AP - 100/60 mm Hg. The first sound is quiet above
the apex, soft systolic murmur is present. What complication is indicated by these clinical
presentations?
A. Acute viral myocarditis*
B. Acute allergic infectious myocarditis
C. Idiopathic myocarditis
D. Myocardiopathy
E. Neurocirculatory dystonia

137. A 5 month old boy was born prematurely, he didn’t suffer from any disease at the infant age
and later on. Examination at an outpatient’s hospital revealed paleness of skin, sleepiness.

28
Завантажено з сайту https://тестування.укр/ - онлайн тестування КРОК

Blood count: Hb - 95 g/l, erythrocytes - 3,5 • 1012/l, reticulocytes - 9%o, colour index - 0,7,
osmotic stability of erythrocytes - 0,44-0,33%, serum iron - 4,9 micromole/l. What is the most
probable cause of anemia?
A. Iron deficit*
B. Hemogenesis immaturity
C. Infectious process
D. Erythrocyte hemolysis
E. B12 deficit

138. A 13 year old girl consulted the school doctor on account of moderate bloody discharge from
the genital tracts, which appeared 2 days ago. Secondary sexual characters are developed.
What is the most probable cause of bloody discharge?
A. Menarche*
B. Juvenile hemorrhage
C. Haemophilia
D. Endometrium cancer
E. Werlhof’s disease

139. An 8 year old girl complains about joint pain, temperature rise up to 38oC, dyspnea.
Objectively: the left cardiac border is deviated by 2,5 cm to the left, tachycardia, systolic
murmur on the apex and in the V point are present. Blood count: leukocytes - 20,0 • 109/l, ESR
- 18 mm/h. What sign gives the most substantial proof for rheumatism diagnosis?
A. Carditis*
B. Arthralgia
C. Leukocytosis
D. Fever
E. Accelerated ESR

140. During inspection of sanitary conditions of studying at a technical university it was necessary
to evaluate the visual regimen of students, who study from 9 a.m to 3 p.m. What index of
natural light will be the most informative?
A. Natural light coefficient*
B. Light coefficient
C. Depth of study room
D. Time of the room insolation
E. Presence of mixed (superolateral) light

141. A 70 year old man is suffering from coronary heart disease. His mood is evidently depressed,
anxious. As a result of continious sleeplessness he has got fears, suicidal thoughts. He would
sit for a long time in the same pose, answer after a pause, in a low, monotonous voice. His face
has a look of suffering, pain, fear. What is the main psychopathologic syndrome?
A. Depressive syndrome*
B. Paranoid syndrome
C. Asthenic syndrome
D. Phobic syndrome
E. Obsessive syndrome

29
Завантажено з сайту https://тестування.укр/ - онлайн тестування КРОК

142. A 37 year old farmer complains about general weakness, spastic pain in the lower parts of his
abdomen, mainly in the left iliac area, frequent defecations up to 18 times a day, feces contain
admixtures of mucus and blood. The illness began abruptly 3 days ago with chill, fever,
headache. General condition is moderately severe, body temperature is 37,8oC. Sigmoid colon
is spasmed and painful. What is the most probable diagnosis?
A. Dysentery*
B. Amebiasis
C. Nonspecific ulcerative colitis
D. Yersiniosis
E. Salmonellosis

143. A 3 year old child fell acutely ill, body temperature rose up to 39,5oC, the child became inert,
there appeared recurrent vomiting, headache. Examination revealed positive meningeal
symptoms, after this lumbal puncture was performed. Spinal fluid is turbid, runs out under
pressure, protein concentration is 1,8 g/l; Pandy reaction is +++, sugar concentration is 2,2
millimole/l, chloride concentration - 123 millimole/l, cytosis is 2, 35 • 109 (80% of neutrophils,
20% of lymphocytes). What is the most probable diagnosis?
A. Purulent meningitis*
B. Serous viral meningitis
C. Serous tuberculous meningitis
D. Subarachnoid haemorrhage
E. Brain tumour

144. A 39 year old patient suffering from hypertension felt suddenly intense headache in the region
of occiput; there appeared recurrent vomiting. These presentations has been lasting for 5
hours. Objectively: Ps - 88 bpm, AP - 205/100 mm Hg, painfulness of occipital points, rigidity of
occipital muscles are present. Kernig’s symptom is bilaterally positive. Subarachnoid
haemorrhage is also suspected. What diagnostic method will be of the greatest importance for
confirmation of provisional diagnosis?
A. Lumbar puncture*
B. Examination of eye fundus
C. Ultrasonic dopplerography
D. EEG
E. Computer tomography

145. A 10 year old girl complains about abdominal pain that is arising and getting worse after
eating rough or spicy food. She complains also about sour eructation, heartburn, frequent
constipations, headache, irritability. She has been suffering from this for 12 months.
Objectively: the girl’s diet is adequate. Tongue is moist with white deposit at the root.
Abdomen is soft, painful in its epigastric part. What study method will help to make a
diagnosis?
A. Esophagogastroduodenoscopy*
B. Intragastral pH-metry
C. Fractional examination of gastric juice
D. Contrast roentgenoscopy
E. Biochemical blood analysis

30
Завантажено з сайту https://тестування.укр/ - онлайн тестування КРОК

146. A 35 year old patient who suffers from chronic glomerulonephritis and has been hemodialysis-
dependent for the last three years developed intermissions of heart activity, hypotension,
progressing weakness, dyspnea. ECG showed bradycardia, atrioventricular block type I, high
pointed waves T. The day before the flagrant violation of diet took place. What is the most
probable cause of these changes?
A. Hyperkaliemia*
B. Hyperhydratation
C. Hypokaliemia
D. Hypernatriemia
E. Hypocalciemia

147. A 34 year old coal miner who has been in service for 10 years was discharged from an
occupational disease clinic after examination and treatment. He was diagnosed with the I
stage of anthracosilicosis, peribronchitis, 0 stage of respiratory failure. What expert decision
should be taken about his working capacity?
A. He should be referred to the local medical expert comission for determination of his
disability group for the period of re-training*
B. He can continue working as coal miner provided that sanitary and hygienic conditions
are observed
C. He should be given medical certificate for the further out-patient treatment
D. He should stay on the sick-list to consolidate treatment results
E. He should be referred to the local medical expert comission for determination of
percentage of working capacity loss

148. A 52 year old woman complains about face distortion. It appeared 2 days ago after
supercooling. Objectively: body temperature is 38, 2oC. Face asymmetry is present. Frontal
folds are flattened. Left eye is wider than right one and doesn’t close. Left nasolabial fold is
flattened, mouth corner is lowered. Examination revealed no other pathology. Blood count:
leukocytes - 10 • 109/l, ESR - 20 mm/h. What is the most probable diagnosis?
A. Facial neuritis*
B. Trigeminus neuralgia
C. Hemicrania (migraine)
D. Ischemic stroke
E. Brain tumour

149. A patient complained about general weakness, fever, painful rash on his trunk skin . He has
been suffering from this for 3 days. Objectively: lateral surface of trunk on the left is
hyperemic and edematic, there are some groups of vesicles with serous and haemorrhagic
contents. What is the most probable diagnosis?
A. Herpes zoster*
B. Contact dermatitis simplex
C. Contact allergic dermatitis
D. Microbial eczema
E. Herpetiform Duhring’s dermatosis

150. Internal obstetric examination of a parturient woman revealed that the sacrum hollow was
totally occupied with fetus head, ischiadic spines couldn’t be detected. Sagittal suture is in the

31
Завантажено з сайту https://тестування.укр/ - онлайн тестування КРОК

straight diameter, occipital fontanel is directed towards symphysis. In what plane of small
pelvis is the presenting part of the fetus?
A. Plane of pelvic outlet*
B. Wide pelvic plane
C. Narrow pelvic plane
D. Plane of pelvic inlet
E. Over the pelvic inlet

151. A 24 year old patient complains about general weakness, dizziness, body temperature rise up
to 37,5oC, sore throat, neck edema, enlargement of submaxillary lymph nodes. Objectively:
mucous membrane of oropharynx is edematic and cyanotic, tonsils are enlarged and covered
with films that spread beyond the tonsils and cannot be easily removed. What is the leading
mechanism of this illness’ development?
A. Action of bacterial exotoxin*
B. Action of bacterial endotoxin
C. Allergic
D. Accumulation of suboxidated products
E. Bacteriemia

152. A 38 year old man complains about mild pain and muscle weakness of shoulder and pelvic
girdles and back that has been progressing for the last 3 weeks. He has also significant
problems with getting up, going up and down the stairs and shaving. It is suspected that the
patient is suffering from dermatomyosi-tis. Blood count: Hb - 114 g/l, leukocytes - 10,8 • 109/l,
eosin - 9%, ESR - 22 mm/h, C-reactive protein (++). Change of the following laboratory factor
will be of the greatest diagnostic importance:
A. Creatine phosphokinase*
B. Ceruloplasmin
C. Sialic acids
D. Antibodies to the native DNA
E. y-globulins

153. A 9 year old boy has been suffering from diabetes mellitus for a year. He gets insulin injections
(humulin R, NPH), the dose makes up 0,4 units per 1 kg of body weight a day. Insulin is
untroduced subcutaneously (into the shoulder) by means of a syringe. What measures should
be taken in order to prevent lipodystrophy?
A. To change point of introduction*
B. To limit fats in the boy’s diet
C. To reduce insulin dose
D. To apply periodically other types of insulin
E. To administer antioxidants

154. A 25 year old pediatrician fell ill a week ago: body temperature rose up to 37,6oC, there
appeared a slight swelling on his neck. His illness was diagnosed as ARD, cervical
lymphadenitis. Treatment course included erythromycin, hot compress on the neck. In course
of treatment body tempearture rose up to 39oC, there appeared headache, repeated vomiting,
meningeal syndrome. What studies are necessary for the final diagnosis?
A. Cerebrospinal puncture*

32
Завантажено з сайту https://тестування.укр/ - онлайн тестування КРОК

B. Puncture of cervical lymph node


C. Complete blood count
D. Sputum test for secondary flora
E. Roentgenological examination of lungs

155. A 41 year old woman has suffered from nonspecific ulcerative colitis for 5 years. On
rectoromanoscopy: evident inflammatory process of lower intestinal parts, pseudopolyposive
changes of mucous membrane. In blood: WBC- 9,8 • 109/l, RBC- 3,0 • 1012/l, ESR - 52
mm/hour. What medication provides pathogenetic treatment of this patient?
A. Sulfosalasine*
B. Motilium
C. Vikasolum
D. Linex
E. Kreon

156. A 49 year old female patient was admitted to a hospital with acute attacks of headache
accompanied by pulsation in temples, AP rose up to 280/140 mm Hg. Pheochromocytoma is
suspected. What mechanism of hypertensive atack does this patient have?
A. Increase of catecholamine concentration*
B. Increase of aldosterone level in blood
C. Increase of plasma renin activity
D. Increase of vasopressin secretion
E. Increase of thyroxine secretion

157. A child was born with body weight 3250 g and body length 52 cm. At the age of 1,5 month the
actual weight is sufficient (4350 g), psychophysical development corresponds with the age.
The child is breast-fed, occasionally there are regurgitations. What is the cause of
regurgitations?
A. Aerophagia*
B. Pylorostenosis
C. Pylorospasm
D. Acute gastroenteritis
E. Esophageal atresia

158. Three weeks after acute angina the patient is still weak, inert, subfebrile, his retromaxillary
lymph nodes are enlarged. Tonsils are flabby, stick together with arches, there are purulent
plugs in lacunae. What is the most probable diagnosis?
A. Chronic tonsillitis*
B. Chronic pharyngitis
C. Acute lacunar tonsillitis
D. Paratonsillitis
E. Tonsillar tumour

159. A 43 year old patient had rightsided deep vein thrombosis of iliofemoral segment 3 years ago.
Now he is suffering from the sense of heaviness, edema of the lower right extremity.
Objectively: moderate edema of shin, brown induration of skin in the lower third of shin, varix

33
Завантажено з сайту https://тестування.укр/ - онлайн тестування КРОК

dilatation of superficial shin veins are present. What is the most probable diagnosis?
A. Postthrombophlebitic syndrome, varicose form*
B. Acute thrombosis of right veins
C. Lymphedema of lower right extremity
D. Parkes-Weber syndrome
E. Acute thrombophlebitis of superficial veins

160. A 23 year old patient fell ill 3 weeks ago when she noticed a very painful induration in her
axillary crease. 4-5 days later it burst and discharged a lot of pus. After that some new
infiltrations appeared around the affected area. The patient has never suffered from skin
diseases before. What is the most probable diagnosis?
A. Hydradenitis*
B. Furuncle
C. Mycosis
D. Herpes zoster
E. Streptococcal impetigo

161. A child is 1 day old. During delivery there had been problems with extraction of shoulders.
Body weight is 4300,0. Right arm hangs down along the body, hand is pronated, movement in
the arm is absent. "Scarf"symptom is positive. What is the most probable diagnosis?
A. Total right-sided obstetric paralysis*
B. Proximal right-sided obstetric paralysis
C. Distal right-sided obstetric paralysis
D. Hemiparesis
E. Tetraparesis

162. An outbreak of food poisoning was recorded in an urban settlement. The illness was diagnosed
as botulism on the grounds of clinical presentations. What foodstuffs should be chosen for
analysis in the first place in order to confirm the diagnosis?
A. Tinned food*
B. Potatoes
C. Pasteurized milk
D. Boiled meat
E. Cabbage

163. A 42 year old metalworker has been working at the turning machine for production of heavy
large-size parts for 5 years. His work requires using of hand and pedal levers that involves
considerable physical force. What means for osteoarthrosis prevention should be
recommended?
A. To limit physical work*
B. To administer protein-and-carbohydrate diet
C. To administer protein-and-vitamin diet
D. To improve health at the Black sea coast
E. To go in for weightlifting

164. A 30 year old patient undergoes treatment because of Werlhof’s disease. Objectively: the

34
Завантажено з сайту https://тестування.укр/ - онлайн тестування КРОК

patient is pale, there are petechial haemorrhages on the extension surfaces of forearms. Ps is
92 bpm, AP is 100/60 mm Hg. The lower edge of spleen is at a level with umbilicus. Blood
count: erythrocytes: 2,8 • 1012/l, Hb - 90 g/l, Ht -0,38, thrombocytes - 30 • 109/l. The patient
is being prepared for splenectomy. What transfusion medium should be chosen in the first
place for the preoperational preparation?
A. Thrombocytic mass*
B. Stored blood
C. Native erythrocytic mass
D. Erythrocytic suspension
E. Washed erythrocytes

165. A 19 year old boy was admitted to a hospital with closed abdominal trauma. In course of
operation multiple ruptures of spleen and small intestine were revealed. AP is falling rapidly, it
is necessary to perform hemotransfusion. Who can specify the patient’s blood group and
rhesus compatibility?
A. A doctor of any speciality*
B. A laboratory physician
C. A surgeon
D. A traumatologist
E. An anaesthesilogist

166. Clinical and statistical study was devoted to the effect of a new pharmacological medication
upon the patients with coronary heart disease. What parametric criterion (coefficient) can be
used for estimation of results validity?
A. Student’s coefficient (t)*
B. Sign criterion
C. Conformity coefficient
D. Wilcoxon’s t-criterion
E. Kolmogorov-Smirnov’s criterion

167. It is planned to make complete isolation boxes in the infectious department in order to prevent
nosocomial airborne infections. The boxes consist of a tambour, a ward and a lock chamber.
What structure should be also included in a complete isolation box?
A. Bathroom unit*
B. Manipulation room
C. Doctor’s consulting room
D. Patient’s examination room
E. Nursing room

168. A 30 year old man complains of intense pain, reddening of skin, edema in the ankle-joint area,
fever up to 39oC. There was an acute onset of the illness. In the past there were similar
attacks lasting 5-6 days without residual changes in the joint. The skin over the joint is
hyperemic and ill-defined, without infiltrative bank on the periphery. What is the most likely
diagnosis?
A. Gout*
B. Infectious arthritis
C. Rheumatoid arthritis

35
Завантажено з сайту https://тестування.укр/ - онлайн тестування КРОК

D. Erysipelatous inflammation
E. Osteoarthritis

169. Preventive examination of a 55 year old patient revealed diabetes mellitus. The patient hasn’t
received treatment for it. Objectively: height is 170 cm, weight is 106 kg, skin humidity is
normal. Ps is 76 bpm, rhythmic, left border of relative heart dullness is deviated by 1 cm to the
left from the middle clavicular line, heart sounds are quiet, AP is 140/80 mm Hg. Glycemia on
an empty stomach is 6,9 millimole/l. Glucose rate in the daily urine is 0,5%, diuresis makes up
2,5 l. What treatment tactics should be chosen?
A. To administer dietotherapy*
B. To administer metformin
C. To administer glibenclamid
D. To administer repaglinide
E. To administer insulin

170. The results of 5 year monitoring allowed to estimate the level of environmental influence upon
health indices of popultaion. What statistic method should be chosen?
A. Calculation of correlation coefficient*
B. Calculation of conformity coefficient
C. Calculation of coefficient of difference validity
D. Calculation of regression coefficient
E. Calculation of dynamic indices

171. In order to improve organism tolerance of boarding-school pupils a doctor developed a


program. The program is based upon the following principles: graduality, consistency,
individuality, coomplexity. What of the main principles of organism tempering wasn’t taken
into account?
A. Systematicness*
B. Autodefense increase
C. Increase of influence force
D. Increase of influence intensity
E. Increase of resistance

172. A patient complains about pyrosis and permanent pain behind his breastbone. When he bends
forward after eating there appears regurgitation. Roentgenological examination revealed
extrasaccular cardiofunctional hernia of esophageal opening of diaphragm. Esophagoscopy
revealed signs of reflux-esophagitis. What is the necessary tretment tactics?
A. Operation in a surgical department*
B. Conservative treatment in an outpatients’ clinic
C. Conservative treatment in the therapeutic hospital
D. Conservative treatment in a surgical department
E. Treatment at a health resort

173. 15 minutes after the second vaccination with diphteria and tetanus toxoids and pertussis
vaccine a 4 month old boy manifested symptoms of Quincke’s edema. What medication should
be applied for emergency aid?

36
Завантажено з сайту https://тестування.укр/ - онлайн тестування КРОК

A. Prednisolone*
B. Heparin
C. Adrenalin
D. Furosemide
E. Seduxen

174. During examination a patient is unconscious, his skin is dry and hot, face hyperemia is
present. The patient has Kussmaul’s respiration, there is also smell of acetone in the air.
Symptoms of peritoneum irritation are positive. Blood sugar is at the rate of 33 millimole/l.
What emergency actions should be taken?
A. Intravenous infusion of short-acting insulin*
B. Intravenous infusion of glucose along with insulin
C. Introduction of long-acting insulin
D. Intravenous infusion of neohaemodesum along with glutamic acid
E. Intravenous infusion of sodium chloride saline

175. In order to study impact of microclimate upon the human organism it is necessary to make
systematic observation of air temperature over 3 days. Choose a device that will allow to make
the most precise temperature records:
A. Thermograph*
B. Alcohol thermometer
C. Mercury thermometer
D. August’s psychrometer
E. Assmann psychrometer

176. A 3 month old child has occiput alopecia, anxious sleep, excessive sweating. What disease
might be suspected?
A. Rachitis*
B. Spasmophilia
C. Anemia
D. Phosphate diabetes
E. Chondrodystrophy

177. After delivery and revision of placenta there was found the defect of placental lobule. General
condition of woman is normal, uterus is firm, there is moderate bloody discharge. Speculum
inspection of birth canal shows absence of lacerations and raptures. What action is nesessary?
A. Manual exploration of the uterine cavity*
B. External massage of uterus
C. Introduction of uterine contracting agents
D. Urine drainage, cold on the lower abdomen
E. Introduction of hemostatic medications

178. A 7 year old boy has periods of loss of attention that last 10-15 seconds and occur several
times a day. During these periods he stands still in one position, doesn’t answer the questions
or react when he is called by his name. Neurological examination revealed no changes. What
diagnostic method should be applied to specify his diagnosis?

37
Завантажено з сайту https://тестування.укр/ - онлайн тестування КРОК

A. Electroencephalography*
B. Skull roentgenogram
C. Computer tomogram
D. Echoencephalography
E. Examination of evoked potentials

179. A 52 year old patient complains about pain in the right part of her chest, dyspnea, cough with
a lot of foul-smelling albuminoid sputum in form of "meat slops". Objectively: the patient’s
condition is grave, cyanosis is present, breathing rate is 31/min, percussion sound above the
right lung is shortened, auscultation revealed different rales. What is the most probable
diagnosis?
A. Lung gangrene*
B. Lung abscess
C. Pleura empyema
D. Multiple bronchiectasis
E. Chronic pneumonia

180. A 15 year old patient suffers from headache, nasal haemorrhages, sense of lower extremity
coldness. Objectively: muscles of shoulder girdle are developed, lower extremities are
hypotrophied. Pulsation on the pedal and femoral arteries is sharply dampened. AP is 150/90
mm Hg, 90/60 on the legs. Systolic murmur can be auscultated above carotid arteries. What is
the most probable diagnosis?
A. Aorta coarctation*
B. Aorta aneurism
C. Aortal stenosis
D. Aortal insufficiency
E. Coarctation of pulmonary artery

181. A patient with frostbite of both feet was delivered to the admission ward. What actions should
be taken?
A. To apply a bandage, to introduce vasodilating medications*
B. To administer cardiac medications
C. To put feet into hot water
D. To rub feet with snow
E. To apply an alcohol compress

182. A patient has a stab wound on his right foot. On the fourth day after injury the patient’s body
temperature rose up to 38oC, inguinal lymph nodes became enlarged and painful, skin over
them reddened. What complication might be suspected?
A. Lymphadenitis*
B. Lymphangitis
C. Phlegmon
D. Tetanus
E. Erysipelas

183. A 52 year old patient complains about headache, weakness of his upper left extremity.

38
Завантажено з сайту https://тестування.укр/ - онлайн тестування КРОК

Neurological symptoms become more intense during physical stress of the left extremity.
Pulsation on the arteries of the left extremity is sharply dampened but it remains unchanged
on the carotid arteries. What is the most probable diagnosis?
A. Occlusion of the left subclavicular artery, steal syndrome*
B. Thoracal outlet syndrome
C. Raynaud’s syndrome
D. Takayasu’s disease
E. Occlusion of brachiocephalic trunk

184. A 57 year old patient abruptly lost the sight of one eye. Examination revealed that his sight
made up 0,02 excentri-cally, eye fundus has haemorrhages of different forms and sizes
("squashed tomato"symptom). Disc of optic nerve is hyperemic. In anamnesis general vascular
pathology is recorded. Direct-acting anticoagulants were administered. What is the most
probable diagnosis?
A. Thrombosis of central vein of retina*
B. Hypertensive angiopathy
C. Hypertensive angioneuropathy
D. Embolism of central artery of retina
E. Diabetic retinopathy

185. A 28 year old patient complained about prolongation of intermenstrual periods up to 2 months,
hirsutism. Gynaecological examination revealed that the ovaries were enlarged, painless,
compact, uterus had no peculari-ties. Pelvic ultrasound revealed that the ovaries were 4-5 cm
in diameter and had multiple enlarged follicles on periphery. Roentgenography of skull base
showed that sellar region was dilated. What is the most probable diagnosis?
A. Stein-Leventhal syndrome*
B. Algodismenorrhea
C. Sheehan’s syndrome
D. Premenstrual syndrome
E. Morgagni-Stewart syndrome

186. A woman consulted a therapeutist about fatigability, significant weight loss, weakness, loss of
appetite. She has been having amenorrhea for 8 months. A year ago she born a full-term child.
Haemorrhage during labour made up 2 l. She got blood and blood substitute transfusions.
What is the most probable diagnosis?
A. Sheehan’s syndrome*
B. Stein-Leventhal syndrome
C. Shereshevsky-Turner’s syndrome
D. Homological blood syndrome
E. Vegetovascular dystonia

187. A 68 year old patient complains about acute pain in his right foot, toe edema and darkening of
skin of the IV toe. He has been suffering from diabetes melli-tus for 15 years, doesn’t receive
regular treatment. What complication of diabetes mellitus is it?
A. Gangrene of the IV toe on the right foot*
B. Panaritium
C. Haematoma

39
Завантажено з сайту https://тестування.укр/ - онлайн тестування КРОК

D. Erysipelas
E. Fracture of the IV toe on the right foot

188. Preventive examination of an 11 year old boy helped to determine his habitus type. It was
established that the child’s shoulders were deviated and brought forward, with forward flexion
of head, the thorax was flattened, abdomen was convex. The child’s backbone had signs of
deepened cervical and lumbar curvatures. What habitus is it?
A. Kyphosis*
B. Lordosis
C. Round-shouldered
D. Corrected
E. Normal

189. During hygienic examination of a hospital it was established that the area for each bed in a
double ward was: in the infectious department for children - 7 m2, in the infectious
department for adults -8 m2, in the burns department - 9 m2, in the radiological department
-10 m2, in the critical care department - 13 m2. In which wards the area for each bed doesn’t
correspond with hygienic requirements?
A. In burns wards*
B. In infectious wards for children
C. In infectious wards for adults
D. In radiological wards
E. In critical care wards

190. A 2 year old child has been ill with acute respiratory viral infection of upper thrice a year - in
February, in April and in December. How should these occurences be recorded?
A. It is necessary to fill in 3 statistic talons signed (+)*
B. It is necessary to fill in 3 statistic talons signed (-)
C. It is necessary to fill in 1 statistic talon signed (+)
D. It is necessary to fill in 1 statistic talon signed (+) and 2 statistic talons signed (-)
E. It is necessary to fill in 1 statistic talon signed (-)

191. Researchers studied disease incidence of influenza and acute respiratory viral infection within
the last 5 years. What kind of graphic presentation should be used for the best visualization of
this data?
A. Linear diagram*
B. Pie diagram
C. Bar diagram
D. Histogram
E. Radial diagram

192. A 55 year old patient felt suddenly sick in a hospital corridor, he was immediately examined by
a doctor. Examination revealed that the patient’s skin was pale, autonomous respiration was
absent, pulse on carotid arteries couldn’t be felt, pupils were mydriatic. What action should be
taken at the beginning of cardiac resuscitation?
A. Precordial thump*

40
Завантажено з сайту https://тестування.укр/ - онлайн тестування КРОК

B. Mouth-to-mouth ventilation
C. Closed-chest cardiac massage
D. Restoration of airway patency
E. Defibrillation

193. A 24 year old patient complained about putting on weight, limosis. Objectively: the patient’s
constitution is of hypersthenic type, body weight index is 33,2 kg/m2, waist circumference is
100 cm. Correlation of waist circumference to the thigh circumference is 0,95. What is the
most probable diagnosis?
A. Alimentary constitutional obesity of the I stage, abdominal type*
B. Hypothalamic Itsenko-Cushing obesity of the II stage, gynoid type
C. Alimentary constitutional obesity of the III stage, gynoid type
D. Alimentary constitutional obesity of the II stage, abdominal type
E. Hypothalamic Itsenko-Cushing obesity of the I stage, abdominal type

194. A parturient woman is 27 year old, it was her second labour, delivery was at term, normal
course. On the 3rd day of postpartum period body temperature is 36,8oC, Ps - 72/min, AP -
120/80 mm Hg. Mammary glands are moderately swollen, nipples are clean. Abdomen is soft
and painless. Fundus of uterus is 3 fingers below the umbilicus. Lochia are bloody, moderate.
What is the most probable diagnosis?
A. Physiological course of postpartum period*
B. Subinvolution of uterus
C. Postpartum metroendometritis
D. Remnants of placental tissue after labour
E. Lactostasis

195. Esophagus wall of a 72 year old patient with severe concomitant pathology was injured during
urgent fi-broesophagogastroscopy. This resulted in progressing of acute respiratory failure
and collapse of the left lung. What aid should be rendered?
A. Drainage of pleural cavity by Bullaux method, mediastinum drainage, antibacterial
therapy*
B. Buelau’s drainage of pleural cavity, antibacterial therapy
C. Left-sided thoracotomy, closure of esophagus and mediastinum wound
D. Left-sided thoracotomy, closure of esophagus wound
E. Endoscopic closure of esophagus wound, drainage

196. A 46 year old woman who has been suffering from hypertension for 5 years was diagnosed
with hypertensive crisis. She complains about palpitation, sense of head pulsation; heart rate
is 100/min, AP is 190/100 mm Hg (haemodynamics is of hyperkinetic type). What medication
should be the medication of choice?
A. 3-adrenoceptor blocker*
B. Adenosine pyrophosphate inhibitor
C. Diuretic
D. ^-adrenoceptor blocker
E. Dihydropyridine calcium antagonist

41
Завантажено з сайту https://тестування.укр/ - онлайн тестування КРОК

197. A 30 year old patient complains about inability to become pregnant over 3 years of married
life. The patient is of supernutrition type, she has hair along the median abdominal line, on the
internal thigh surface and in the peripapillary area. Menses started at the age of 16, they are
infrequent and non-profuse. US revealed that the uterus was of normal size, ovaries were
4x5x5 cm large and had a lot of cystic inclusions. What is the most probable diagnosis?
A. Polycystic ovaries*
B. Ovarian cystoma
C. Chronic oophoritis
D. Menstrual irregularity
E. Bilateral ovarian tumours

198. A 4 month old child was admitted to a surgical department 8 hours after the first attack of
anxiety. The attacks happen every 10 minutes and last for 23 minutes, there was also one-time
vomiting. Objectively: the child’s condition is grave. Abdomen is soft, palpation reveals a
tumour-like formation in the right iliac area. After rectal examination the doctor’s finger was
stained with blood. What is the most probable diagnosis?
A. Ileocecal invagination*
B. Gastrointestinal haemorrhage
C. Wilm’s tumour
D. Helminthic invasion
E. Pylorostenosis

199. During preventive examination a 16 year old patient presented no problems. Objectively: the
patient has signs of malnutrition, he is asthenic, AP is 110/70 mm Hg, Ps is 80 bpm, cardiac
border is normal, auscultation above the cardiac apex reveals three sounds, cardiac murmur is
absent. ECG shows no pathological changes, phonocardiogram shows that the third sound
comes 0,15 s after the second one above the apex. How are these changes called?
A. III physiological sound*
B. Fout-ta-ta-rou (reduplication of the 2nd sound)
C. Protodiastolic gallop rhythm
D. Atrial gallop rhythm
E. IV physiological sound

200. A 52 year old patient with disseminated vertebral osteochondrosis lifted a significant load that
resulted in lumbar pain and pain along the sciatic nerve. Objectively: positive Lasegue’s sign
on the left, reduced Achilles reflex. What drug would be the most effective from the
pathogenetic point of view?
A. Diclofenac*
B. Aspirin
C. Analgin
D. Novocaine
E. Spasmalgon

42
Завантажено з сайту https://тестування.укр/ - онлайн тестування КРОК

ТЕСТУВАННЯ.УКР Бази тестів

Буклет 2009 року

Цей тест можна пройти в режимі онлайн тестування на сайті https://тестування.укр/testkrok/studing/564

Це офіційні тести з сайту Центру тестування https://www.testcentr.org.ua/

1. A woman complains of having slight dark bloody discharges and mild pains in the lower part of
abdomen for several days. Last menses were 7 weeks ago. The pregnancy test is positive.
Bimanual investigation: the body of the uterus indicates for about 5-6 weeks of pregnancy, it is
soft, painless. In the left appendage there is a retort-like formation, 7x5 cm large, mobile,
painless. What examination is necessary for detection of fetus localization?
A. Ultrasound*
B. Hysteroscopy
C. Hromohydrotubation
D. Colposcopy
E. Cystoscopy

2. A pregnant woman in her 40th week of pregnancy undergoes obstetric examination: the cervix
of uterus is undeveloped. The oxytocin test is negative. Examination at 32 weeks revealed: AP
140/90 mm Hg, proteinuria 1 g/l, peripheral edemata. Reflexes are normal. Choose the most
correct tactics:
A. Labour stimulation after preparation*
B. Absolute bed rest for 1 month
C. Complex therapy of gestosis for 2 days
D. Caesarian section immediately
E. Complex therapy of gestosis for 7 days

3. A 26 year old woman had the second labour within the last 2 years with oxytocin application.
The child’s weight is 4080 g. After the placent birth there were massive bleeding, signs of
hemorrhagic shock. Despite the injection of contractive agents, good contraction of the uterus
and absence of any cervical and vaginal disorders, the bleeding proceeds. Choose the most
probable cause of bleeding:
A. Atony of the uterus*
B. Injury of cervix of the uterus
C. Hysterorrhexis
D. Delay of the part of placenta
E. Hypotonia of the uterus

4. A woman of a high-risk group (chronic pyelonephritis in anamnesis) had vaginal delivery. The
day after labour she complained of fever and loin pains, frequent urodynia. Specify the most
probable complication:
A. Infectious contamination of the urinary system*
B. Thrombophlebitis of veins of the pelvis
C. Infectious hematoma
D. Endometritis
E. Apostasis of sutures after episiotomy

1
Завантажено з сайту https://тестування.укр/ - онлайн тестування КРОК

5. In the woman of 24 years about earlier normal menstrual function, cycles became irregular,
according to tests of function diagnostics - anovulatory. The contents of prolactin in blood is
boosted. Choose the most suitable investigation:
A. Computer tomography of the head*
B. Determination of the level of gonadotropins
C. USI of organs of small pelvis
D. Progesterone assay
E. Determination of the contents of testosteron-depotum in blood serum

6. A patient was delivered to a surgical department after a road accident with a closed trauma of
chest and right-sided rib fracture. The patient was diagnosed with right-sided pneumothorax,
it is indicated to perform drainage of pleural cavity. Pleural puncture should be made in:
A. In the 2nd intercostal space along the middle clavicular line*
B. In the 6th intercostal space along the posterior axillary line
C. In the 7th intercostal space along the scapular line
D. In the projection of pleural sinus
E. In the point of the greatest dullness on percussion

7. A patient who takes diuretics has developed arrhythmia as a result of cardiac glycoside
overdose. What is the treatment tactics in this case?
A. Increased potassium concentration in blood*
B. Increased sodium consentration in blood
C. Reduced magnesium concentration in blood
D. Increased calcium concentration in blood
E. -

8. A 20 year old patient was delivered to the hospital in summer from the street with haemorrage
from the brachial artery. First medical aid involved aplication of a tourniquet for provisional
arrest of bleeding. What is the maximal exposure of the tourniquet?
A. 120 minutes*
B. 15 minutes
C. 30 minutes
D. 60 minutes
E. 180 minutes

9. The average body lenth of newborn boys is 50,9 cm at a sigma 1,66; and average mass - 3432
at a sigma 5,00. What criterion is necessary in order to compare degree of variability of these
signs?
A. Coefficient of variation*
B. Sigma
C. Limit
D. Amplitude
E. Coefficient of association

10. Indicate the registration medical document for the patient, who 21.02. was addressed to the
doctor with diagnosis ARVD for the first time in this year:

2
Завантажено з сайту https://тестування.укр/ - онлайн тестування КРОК

A. The statistical coupon is to be filled in and it is necessary to deliver on a sign (+)*


B. The statistical coupon for registration of final diagnosis is not necessary
C. The statistical coupon is to be filled in, but a sign (+) is not necessary to be put in
D. It is necessary to fill in the emergency notice on a case of a contagion
E. The necessary registration form is not indicated

11. Five days after a total hip joint replacement a 72 year old woman becomes acutely short of
breath, diaphoretic and hypotensive. Both lung fields are clear to auscultation and percussion,
but examination of the neck reveals mild jugular venous distension with prominent A waves.
Heart sounds are normal. ECG shows sinus tachycardia with a new right bundle branch block
and minor nonspecific ST - T wave changes. The most likely diagnosis is:
A. Pulmonary thromboembolism*
B. Acute myocardial infarction
C. Aortic dissection
D. Pericarditis
E. Aspiration

12. A 38 year old man, previously in good health, suddenly develops severe abdominal pain
radiating from the left loin to groin and accompanied by nausea, perspiration and the need for
frequent urination. He is restless, tossing in bed but has no abnormal findings. The most likely
diagnosis is:
A. Leftsided renal colic*
B. Herpes zoster
C. Sigmoid diverticulitis
D. Torsion of the left testicle
E. Retroperitoneal haemorrhage

13. A 40 year old woman has a selfdetected hard breast mass. The procedure of choice for
confirming the diagnosis is:
A. Excision biopsy*
B. Mammography
C. Thermography
D. Ultrasonography
E. Aspiration biopsy with cytology

14. What is the maximum duration of medical certificate in case of tuberculosis?


A. 2 months*
B. Week
C. 2 weeks
D. Month
E. 10 months

15. The student has the following devices: Geiger counter, Ebert counter, Krotov’s apparatus,
Mischuk device, Ebert device. What device can he use to assess air germ pollution?
A. Krotov’s apparatus*
B. Ebert’s counter

3
Завантажено з сайту https://тестування.укр/ - онлайн тестування КРОК

C. Geiger’s counter
D. Mischuk’s device
E. Ebert’s device

16. Student B. lives in the canalized house in the flat with complete set of sanitary equipment
(WC, bath, shower, local water heater). How much water consumption has he got?
A. 160-200 l*
B. 10-151
C. 50-1001
D. 300-4001
E. 500 l

17. What guarantees against the preconceived attitude to the physician in cases of professional
law violations do you know?
A. Sanction of public prosecutor, inquiry by preliminary investigator of prosecutor’s office,
committee of experts*
B. Draw up a statement about forensic medical examination
C. Conduct an inquiry by preliminary investigator of police department
D. Utilisation copy of medical documents
E. Conduct forensic medical examination by district forensic medicine expert

18. A 63 year old patient was diagnosed with purulent mediastinitis. What of the below listed
diseases are NOT the cause of purulent mediastinitis?
A. Cervical lymphadenitis*
B. Deep neck phlegmon
C. Perforation of the cervical part of the oesophagus
D. Perforation of the thoracic part of the oesophagus
E. Iatrogenic injury of the trachea

19. A 36 year old patient was diagnosed with right-sided pneumothorax. What method of
treatment is indicated to the patient?
A. Surgical treatment: drainage of the pleural cavity*
B. Antiinflammation therapy
C. Symptomatic therapy
D. Pleural puncture
E. Thoracotomy

20. It is suspected that a 34 year old patient has an abscess of Douglas pouches. What diagnostic
method is to be chosen?
A. Digital examination of rectum*
B. Rectoromanoscopy
C. Laparoscopy
D. Percussion and auscultation of stomach
E. R-scopy of abdominal cavity

4
Завантажено з сайту https://тестування.укр/ - онлайн тестування КРОК

21. A patient has restrained umbilateral hernia complicated by phlegmon hernia, it is necessary to
take following actions:
A. Herniotomy by Mayo-Sapezhko*
B. Herniotomy by Mayo
C. Herniotomy by Sapezhko
D. Herniotomy by Lekser
E. Herniotomy by Grenov

22. A 10 year old boy complains about pain in his left eye and strong photophobia after he had
injured his left eye with a pencil at school. Left eye examination: blepharospasm, ciliary and
conjunctival congestion, cornea is transparent, other parts of eyeball have no changes. Visus
0,9. Right eye is healthy, Visus 1,0. What additional method would you choose first of all?
A. Staining test with 1 % fluorescein*
B. X-ray examination of orbit
C. Tonometria
D. Gonioscopia
E. Cornea sensation-test

23. A patient with acute purulent otitis media complicated by mastoiditis was admitted to a
hospital. Roentgenogram of mastoid processes showed the shadi-owing of the cellular system
on the lesion, absence of bone septa was present. What are the necessary therapeutic actions
at the second stage of mastoiditis?
A. Mastoidotomy*
B. Paracentesis of the drum
C. Radical operation on the middle ear
D. Tympanoplasty
E. Cateterization of the Eustachian tube

24. The most available and informative diagnostic method for closed trauma of the urinary bladder
is:
A. Retrograde cystography*
B. Pelvic arteriography
C. Cystography
D. Sonography of the urinary bladder
E. Palpation and percussion of abdomen

25. Female 45 year old patient was admitted to the traumatological ward with the closed fracture
of the medial malleolus with its displacement up to 3 mm. The foot is to be fixed with a plaster
cast in the following position:
A. At right angle with varus positioning of the foot*
B. In position of planter flexion of foot
C. In position of pronation
D. In position of supination
E. In position of dorsal flexion of foot

26. A 3 month old infant suffering from acute segmental pneumonia has dyspnea (respiration rate

5
Завантажено з сайту https://тестування.укр/ - онлайн тестування КРОК

- 80 per minute), paradoxical breathing, tachycardia, total cyanosis. Respiration and pulse -
ratio is 1:2. The heart dullness under normal size. Such signs characterise:
A. Respiratory failure of III degree*
B. Respiratory failure of I degree
C. Respiratory failure of II degree
D. Myocarditis
E. Congenital heart malformation

27. A 3 year old child has been suffering from fever, cough, coryza, conjunctivitis for 4 days. He
has been taking sulfadimethoxine. Today it has fever up to 39oC and maculopapular rash on its
face. Except of rash the child’s skin has no changes. What is your diagnosis?
A. Measles*
B. Allergic rash
C. Rubella
D. Scarlet fever
E. Pseudotuberculosis

28. A 2 year old girl has been ill for 3 days. Today she has low grade fever, severe catarrhal
presentations, slight maculopapular rash on her buttocks and enlarged occipital lymph nodes.
What is your diagnosis?
A. Rubella*
B. Scarlet fever
C. Measles
D. Adenoviral infection
E. Pseudotuberculosis

29. A 3 year old boy fell ill abruptly: fever up to 39oC, weakness, vomi-tng. Haemorrhagic rash of
various size appeared on his lower limbs within 5 hours. Meningococcemia with infective -
toxic shock of the 1 degree was diagnosed. What medications should be administered?
A. Chloramphenicol succinate and prednisone*
B. Penicillin and prednisone
C. Penicillin and immunoglobulin
D. Chloramphenicol succinate and interferon
E. Ampicillin and immunoglobulin

30. A woman delivered a child. It was her fifth pregnancy but the first delivery. Mother’s blood
group is A(II)Rh-, newborn’s - A(II)Rh+. The level of indirect bilirubin in umbilical blood was
58 micromole/l, haemoglobin -140 g/l, RBC-3,8 • 1012/l. In 2 hours the level of indirect
bilirubin turned 82 micromole/l. The hemolytic disease of newborn (icteric-anemic type, Rh-
incompatibility) was diagnosed. Choose the therapeutic tactics:
A. Replacement blood transfusion (conservative therapy)*
B. Conservative therapy
C. Blood transfusion (conservative therapy)
D. Symptomatic therapy
E. Antibiotics

6
Завантажено з сайту https://тестування.укр/ - онлайн тестування КРОК

31. Infant is 6,5 months now and is given natural feeding since birth. Body mass was 3,5 kg, with
length 52 cm at birth. How many times per day the supplement (up feeding) should be given?
A. 2*
B. 3
C. 1
D. 0
E. 4

32. A 12 year old child has the ulcer disease of stomach. What is the etiology of this disease?
A. Intestinal bacillus*
B. Helicobacter pylory
C. Salmonella
D. Lambliosis
E. Influenza

33. A nine year old child is at a hospital with acute glomerulonephritis. Clinical and laboratory
examinations show acute condition. What nutrients must not be limited during the acute
period of glomerulonephritis?
A. Carbohydrates*
B. Salt
C. Liquid
D. Proteins
E. Fats

34. Examination of a 3-month-old child revealed scrotum growth on the right. This formation has
elastic consistency, its size decreases during sleep and increases when the child is crying.
What examination will be helpful for making a correct diagnosis?
A. Palpation of the thickened cord crossing the pubical tubercule (sign of the silk glove)*
B. Diaphanoscopy
C. Palpation of the external inguinal ring
D. Punction of the scrotum
E. Examination of the formation in Trendelenburg's position

35. A 52 year old patient with history of functional Class II angina complains of having intense and
prolonged retrosternal pains, decreased exercise tolerance for 5 days. Angina is less
responsive to nitroglycerine. What is the most probable diagnosis?
A. IHD. Unstable angina*
B. Cardialgia due to spine problem
C. IHD. Functional Class II angina
D. Myocarditis
E. Myocardial dystrophy

36. A 52 year old patient has hypervolaemic type of essential hypertension. Which of the following
medications is to be prescribed either as monotherapy or in complex with other
antihypertensive drugs?
A. Hypothiazid*

7
Завантажено з сайту https://тестування.укр/ - онлайн тестування КРОК

B. Dibazol
C. Clonidine
D. Kapoten
E. Nifedipin

37. A 62 year old patient complains of rest dyspnea, heart pains. 3 years ago he had myocardial
infarction. Physical examination: orthopnea, acrocyanosis, swollen cervical veins. Ps - 92, total
heart enlargement, the liver is enlarged by 7 cm, shin edema. What is the stage of chronic
heart failure (CHF)?
A. CHF- 2 B*
B. CHF-1
C. CHF- 2 A
D. CHF- 0
E. CHF- 3

38. A 27 year old man complains of pains in epigastrium which are relieved by food intake. EGDFS
shows antral erosive gastritis, biopsy of antral mucous presents Helicobacter Pylori. Diagnosis
is:
A. Gastritis of type B*
B. Gastritis of type A
C. Reflux-gastritis
D. Menetrier’s gastritis
E. Rigid antral gastritis

39. Prophylactic photoroentgenography examination of a 25 year old man revealed focal


shadowings of small and medium intensity with irregular contours in the 1st and 2nd segments
of the right lung. Which clinical form can be suspected?
A. Focal*
B. Disseminated
C. Miliary
D. Fibro-cavernous
E. Tuberculoma

40. A woman 26 years old has abused alcohol for 7 years. She has psychological dependence on
alcohol, but no withdrawal syndrome. Drinks almost every day approximately 50-100 g of wine.
She is in her 4-th week of pregnancy. Primary prevention of fetal alcohol syndrome requires:
A. Treatment of alcoholism and full absti-nance from alcohol during all the period of
pregnancy*
B. Medical abortion
C. Decrease of alcohol use
D. Participation in the A-ANON group
E. Gyneacological observation

41. The observed patient’s movements are retarded, she answers no questions. Sometimes she
spontaneously stiffens in strange postures. It is possible to set her body and limbs into
different positions artificially. If the psychiatrist lifts her arm or leg, so that she remains

8
Завантажено з сайту https://тестування.укр/ - онлайн тестування КРОК

standing on the other leg, the patient can stay in such a position for quite a long time. Name
the probable disorder:
A. Catatonic stupor, schizophrenia*
B. Depressive stupor, bipolar disorder
C. Apathetic stupor, schizophrenia
D. Psychogenic stupor, stress disorder
E. Dissociative stupor, dissociative psychosis

42. The man, aged 42, applied to the therapeutist with complaints of pricking pains in scapulas
area, dyspnea on physical exertion, cough with discharge of small amount of sputum. During
10 years he works in coal mining. On percussion-box-note sound in the lower parts, on
auscultation- a harsh breathing. There were no changes in the heart. Possible diagnosis?
A. Silicosis*
B. Tuberculosis of lungs
C. Silicatosis
D. Bronchiectatic disease
E. Chronic bronchitis

43. A man, aged 37, working on the collective farm on sowing, was admitted to the infectious
hospital with the clinical symptoms: miosis, labored breathing, sweating.What kind of
poisoning is it and what is the first aid?
A. Poisoning by POC. Treatment: atropine*
B. Poisoning by lead. Treatment: tetacine Calcii
C. Poisoning by the methylic alcohol. Treatment: ethylic alcohol
D. Poisoning by vapours of mercury. Treatment: unithiol
E. -

44. The 25 year old patient was admitted on the 1st day of the disease with complaints of double
vision in the eyes, difficult respiration. The day before the patient ate home-made mushrooms.
On objective examination: paleness, widened pupils, disorder of swallowing, bradycardia,
constipation are marked. What is the diagnosis?
A. Botulism*
B. Yersiniosis
C. Leptospirosis
D. Salmonellosis, gastrointestinal form
E. Lambliasis

45. A 28 year old patient was admitted to the clinic with complaints of the temperature rise up to
39,0oC, headache, weakness, constipation on the 9th day of the disease. On examination:
single roseolas on the skin of the abdomen are present. The pulse rate is 78 bpm. The liver is
enlarged by 2 cm. What is the most probable diagnosis?
A. Typhoid fever*
B. Leptospirosis
C. Brucellosis
D. Sepsis
E. Malaria

9
Завантажено з сайту https://тестування.укр/ - онлайн тестування КРОК

46. A patient has been in a hospital. The beginning of the disease was gradual: nausea, vomiting,
dark urine, acholic stools, yellowness of the skin and scleras. The liver is protruded by 3 cm.
Jaundice progressed on the 14th day of the disease. The liver diminished in size. What
complication of viral hepatitis caused deterioration of the patient’s condition?
A. Hepatic encephlopathy*
B. Meningitis
C. Relapse of viral hepatitis
D. Cholangitis
E. Infectious-toxic shock

47. An 18 year old patient was admitted to a hospital with complaints of headache, weakness, high
temperature, sore throat. Objectively: enlargement of all groups of lymph nodes was revealed.
The liver is enlarged by 3 cm, spleen - by 1 cm. In blood: leukocytosis, atypical lymphocytes -
15%. What is the most probable diagnosis?
A. Infectious mononucleosis*
B. Acute lymphoid leukosis
C. Diphtheria
D. Angina
E. Adenoviral infection

48. A 4 year old girl was playing with her toys and suddenly she got an attack of cough, dyspnea.
Objectively: respiration rate - 45/min, heart rate - 130/min. Percussion revealed dullness of
percutory sound on the right in the lower parts. Auscultation revealed diminished breath
sounds with bronchial resonance on the right. X-ray pictue showed shadowing of the lower
part of lungs on the right. Blood analysis revealed no signs of inflammation. The child was
diagnosed with foreign body in the right bronchus. What complication caused such clinical
presentations?
A. Atelectasis*
B. Emphysema
C. Pneumothorax
D. Bronchitis
E. Pneumonia

49. A 75 year old man who has been suffering from diabetes for the last six months was found to
be jaundiced. He was asymptomatic except for weight loss at the rate of 10 pounds in 6
months. Physical examination revealed a hard, globular, right upper quadrant mass that moves
during respiration. A CT scan shows enlargement of the head of the pancreas, with no filling
defects in the liver. The most likely diagnosis is:
A. Carcinoma of the head of the pancreas*
B. Infectious hepatitis
C. Haemolytic jaundice
D. Malignant biliary stricture
E. Metastatic disease of liver

50. A 60 year old man with unstable angina pectoris fails to respond to heparin, nitroglycerin, beta
adrenegic blockers and calcium channel antagonist. The best management includes:
A. Coronary artery bypass grafting*

10
Завантажено з сайту https://тестування.укр/ - онлайн тестування КРОК

B. Intravenous streptokinase
C. Excercise testing
D. Oral aspirin
E. Antihypertensive therapy

51. A 22 year old woman complained of right subcostal aching pain, nausea, and decreased
appetite. She fell ill 2 months after appendectomy when jaundice appeared. She was treated in
an infectious hospital. 1 year later above mentioned symptoms developed. On exam: the
subicteric sclerae, enlarged firm liver. Your preliminary diagnosis:
A. Chronic viral hepatitis*
B. Calculous cholecystitis
C. Gilbert’s disease
D. Acute viral hepatitis
E. Chronic cholangitis

52. A 25 year old woman complained of edema on her face and legs, rise of blood pressure up to
160/100 mm Hg and weakness. She fell ill 3 weeks after recovering from angina. Urinalysis
data: protein of 0,5 g/l, erythrocytes of 17-20/field, leukocytes of 2-3/field, erythrocyte casts.
What treatment should be initiated after specifying the diagnosis?
A. Penicillin OS*
B. Heparin
C. Ceftriaxone
D. Dipyridamole
E. Ciprofloxacine

53. A fitter of a metallurgic factory with occupational exposure to high concentrations of mercury
fumes for 16 years presents instability of pulse and blood pressure, general hyperhydrosis,
asymmetric innervations of facial muscles and tongue, positive subcortical reflexes, hand
tremor on physical examination. A dentist revealed paradontosis and chronic stomatitis. What
is the most probable diagnosis?
A. Chronic mercury intoxication*
B. Neuroinfection
C. Parkinson syndrome
D. Acute mercury intoxication
E. Mercury encephalopathy

54. A 42 year old woman complains of dyspnea, edema of the legs and tachycardia during minor
physical exertion. Heart borders are displaced to the left and S1 is accentuated, there is
diastolic murmur on apex. The liver is enlarged by 5 cm. What is the cause of heart failure?
A. Mitral stenosis*
B. Mitral regurgitation
C. Tricuspid stenosis
D. Tricuspid regurgitation
E. Aortic stenosis

55. A 32 year old welder complains of weakness and fever. His illness started as tonsillitis a month

11
Завантажено з сайту https://тестування.укр/ - онлайн тестування КРОК

before. On exam, BT of 38,90C, RR of 24/min, HR of 100/min, BP of 100/70 mm Hg,


hemorrhages on the legs, enlargement of the lymph nodes. CBC shows Hb of 70 g/l, rBc of 2, 2
• 1012/l, WBC of 3,0 • 109/l with 32% of blasts, 1% of eosinophiles, 3% of bands, 36% of
segments, 20% of lymphocytes, and 8% of monocytes, ESR of 47 mm/h. What is the cause of
anemia?
A. Acute leukemia*
B. Chronic lympholeukemia
C. Aplastic anema
D. Vitamin B12 deficiency anemia
E. Chronic hemolytic anemia

56. A male patient, 60 years old, tobacco smoker for 30 years, alcoholic, has dysphagia and weight
loss since 4 months. Suggested diagnosis?
A. Cancer of the esophagus*
B. Esophageal achalasia
C. Hanter’s disease
D. Esophagitis
E. Esophageal diverticulum

57. Which of the following symptoms would occur only if a total-body acute radiation exposure
exceeded 5.000 rad (50 Gy)
A. Hallucinations and impairment of vision*
B. Nausea and vomiting
C. Diarrhea
D. Bleeding gums
E. Epilation (hair loss)

58. In the 43rd week of gestation a long, thin infant was delivered. He is apneic, limp, pale, and
covered with "pea soup"amniotic fluid. The first step in the resuscitation of this infant at
delivery should be:
A. Suction of the trachea under direct vision*
B. Artificial ventilation with bag and mask
C. Artificial ventilation with endotracheal tube
D. Administration of 100% oxygen by mask
E. Catheterization of the umbilical vein

59. 25 children at the age of 2-3 years who don’t attend any child welfare institutions should be
observed by a district pediatrician within the current year. How many initial visits of this
group of children should be planned?
A. 50*
B. 20
C. 40
D. 100
E. 200

60. A 25 year old patient complains of pain in the I finger on his right hand. On examination: the

12
Завантажено з сайту https://тестування.укр/ - онлайн тестування КРОК

finger is homogeneously hydropic, in bent position. On attempt to unbend the finger the pain
is getting worse. Acute pain appears during the probe in ligament projection. What decease is
the most likely?
A. Thecal whitlow (ligament panaritium)*
B. Subcutaneous panaritium
C. Articular (joint) panaritium
D. Bone panaritium
E. Paronychia

61. A 26 year old manual worker complained of 3 weeks history of fevers and fatigue, weight loss
with no other symptoms. Physical findings: Temperature 37,6oC, Ps- 88 bpm, blood pressure
115/70 mm Hg, superficial lymph nodes (occipital, submental, cervical, axillary) are enlarged,
neither tender nor painful. Rubella-like rash on the trunk and extremities. Herpes simplex
lesions on the lips. Candidosis of oral cavity. What infectious disease would you suspect?
A. HIV infection*
B. Influenza
C. Rubella
D. Infectious mononucleosis
E. Tuberculosis

62. A patient complains about strong dyspnea that is getting worse during physical activity.
Presentations appeared suddenly 2 hours ago at work: acute chest pain on the left, cough. The
pain was abating, but dyspnea, dizziness, pallor, cold sweat and cyanosis were progressing.
Vesicular respiration is absent, X-ray picture shows a shadow on the left. What pathology
might be suspected?
A. Spontaneous left-sided pneumothorax*
B. Pulmonary infarction
C. Pleuritis
D. Left-sided pneumonia
E. Pulmonary abscess

63. Which of the methods of examination is the most informative in the diagnostics of a tube
infertility?
A. Laparoscopy with chromosalpingoscopy*
B. Pertubation
C. Hysterosalpingography
D. Transvaginal echography
E. Bicontrast pelviography

64. A 38 year old man worked at roofing and drain pipes production for 15 years. He seeks
medical help for expiratory breathlessness on exertion, and dry cough. On exam, wheezes
above both lungs, grayish warts on fingers are seen. Factory physician has diagnosed
asbestosis. What method is the most important for this diagnosis?
A. Chest X-ray*
B. Bronchoscopy
C. Blood gas analysis
D. Spirography

13
Завантажено з сайту https://тестування.укр/ - онлайн тестування КРОК

E. Electrocardiography

65. A patient has got pain in the axillary area, rise of temperature developed 10 hours ago. On
examination: shaky gait is evident, the tongue is coated with white deposit. The pulse is
frequent. The painful lymphatic nodes are revealed in the axillary area. The skin over the
lymph nodes is erythematous and glistering. What is the most probable diagnosis?
A. Bubonic plague*
B. Acute purulent lymphadenitis
C. Lymphogranulomatosis
D. Anthrax
E. Tularemia

66. Examination of a 9 month old girl revealed skin pallor, cyanosis during excitement. Percussion
revealed transverse dilatation of cardiac borders. Auscultation revealed continuous systolic
murmur on the left from the breastbone in the 3-4 intercostal space. This murmur is conducted
above the whole cardiac region to the back. What congenital cardiac pathology can be
suspected?
A. Defect of interventricular septum*
B. Defect of interatrial septum
C. Coarctation of aorta
D. Fallot’s tetrad
E. Pulmonary artery stenosis

67. A 32 year old patient complains about cardiac irregularities, dizziness, dyspnea at physical
stress. He has never suffered from this before. Objectively: Ps- 74 bpm, rhythmic. AP-130/80
mm Hg. Auscultation revealed systolic murmur above aorta, the first heart sound was normal.
ECG showed hypertrophy of the left ventricle, signs of repolarization disturbance in the I, V5
and V6 leads. Echocardiogram revealed that interventricular septum was 2 cm. What is the
most probable diagnosis?
A. Hypertrophic cardiomyopathy*
B. Aortic stenosis
C. Essential hypertension
D. Myocardium infarction
E. Coarctation of aorta

68. An 8 year old boy suffering from haemophilia was undergoing transfusion of packed red cells.
Suddenly he got pain behind the breastbone and in the lumbar area, dyspnea, cold sweat.
Objectively: pale skin, heart rate - 100/min, AP- 60/40 Hg; oliguria, brown urine. For treatment
of this complication the following drug should be administered:
A. Prednisolone*
B. Lasix
C. Adrenaline
D. Aminophylline
E. Analgine

69. A maternity hospital registered 616 live births, 1 stillbirth, 1 death on the 5th day of life over a

14
Завантажено з сайту https://тестування.укр/ - онлайн тестування КРОК

1 year period. What index allows the most precise estimation of this situation?
A. Perinatal mortality*
B. Crude mortality rate
C. Natality
D. Neonatal mortality
E. Natural increase

70. After objective clinical examination a 12 year old child was diagnosed with mitral valve
prolapse. What complementary instrumental method of examination should be applied for the
diagnosis confirmation?
A. Echocardiography*
B. Roentgenography of chest
C. Phonocardiography
D. ECG
E. Veloergometry

71. A 28 year old parturient complai- ns about headache, vision impairment, psychic inhibition.
Objectively: AP- 200/110 mm Hg, evident edemata of legs and anterior abdominal wall. Fetus
head is in the area of small pelvis. Fetal heartbeats is clear, rhythmic, 190/min. Internal
investigation revealed complete cervical dilatation, fetus head was in the area of small pelvis.
What tactics of labor management should be chosen?
A. Forceps operation*
B. Cesarean
C. Embryotomy
D. Conservative labor management with episiotomy
E. Stimulation of labor activity

72. A patient complained about problems with pain and tactile sensitivity, pain in the nail bones at
the end of the working day. He works at a plant with mechanical devices. What pathology can
be suspected?
A. Vibration disease*
B. Caisson disease
C. Noise disease
D. Overwork symptoms
E. Hypovitaminosis of B1

73. A 25 year old patient complains about weakness, dizziness, haemorrhagic skin rash. She has
been suffering from this for a month. Blood count: erythrocytes: 1,0 • 1012/l, Hb- 37 g/l, colour
index - 0,9, leukocytes - 1,2 • 109/l, thrombocytes -42 • 109/l. What diagnostic method will be
the most effective?
A. Sternal puncture*
B. Spleen biopsy
C. Liver biopsy
D. Coagulogram
E. Abdominal ultrasound

15
Завантажено з сайту https://тестування.укр/ - онлайн тестування КРОК

74. A 68 year old female patient complains about temperature rise up to 38,3oC, haematuria. ESR-
55 mm/h. Antibacterial therapy turned out to be ineffective. What diagnosis might be
suspected?
A. Renal cancer*
B. Polycystic renal disease
C. Renal amyloidosis
D. Urolithiasis
E. Chronic glomerulonephritis

75. A 48 year old female patient complains about contact haemorrhage. Speculum examination
revealed hypertrophy of uterus cervix. It resembles of cauliflower, it is dense and can be easily
injured. Bimanual examination revealed that forni-ces were shortened, uterine body was
nonmobile. What is the most probable diagnosis?
A. Cervical carcinoma*
B. Metrofibroma
C. Endometriosis
D. Cervical pregnancy
E. Cervical papillomatosis

76. A 34 year old female patient has been suffering from anxious depression accompanied by panic
attacks for 2 years. She has been a patient of a psychotherapist. Treatment resulted in
incomplete remission. The patient had to break off psychotherapy because of moving to a new
place of residence. Soon after this her condition grew significantly worse, she was almost
permanently anxious, panic attacks turned up 5-6 times a day and were accompanied by
palpitation, dyspnea, cold sweat, thanatophobia. What drug group is the most appropriate for
medicamental therapy?
A. Antidepressants*
B. Sedative neuroleptics
C. Antipsychotic neuroleptics
D. Lithium drugs
E. Cardiotonics, respiratory analeptics

77. 1,5 hour after start of gullet bougienage a 48 year old patient suffering from corrosive
stricture felt acute abdominal pain. Previously he had been suffering from duodenal ulcer.
Examination revealed that abdomen was very tense and painful; Ps- 110 bpm, painful
sialophagia, skin pallor. What is the most probable diagnosis?
A. Perforation of abdominal part of esophagus*
B. Acute myocardium infarction
C. Strangulation of diaphragmal hernia
D. Perforation of duodenal ulcer
E. Thrombosis of mesenteric vessels

78. A healthy 75 year old woman who leads a moderately active way of life went through a
preventive examination that revealed serum concentration of common cholesterol at the rate
of 5,1 millimole/l and HDL (high-density lipoproteins) cholesterol at the rate of 70 mg/dl. ECG
reveals no pathology. What dietary recommendation is the most adequate?
A. Any dietary changes are necessary*

16
Завантажено з сайту https://тестування.укр/ - онлайн тестування КРОК

B. Decrease of cholesterol consumption


C. Decrease of saturated fats consumption
D. Decrease of carbohydrates consumption
E. Increase of cellulose consumption

79. A parturient complains about pain in the mammary gland. Palpation revealed a 3x4 cm large
infiltration, soft in the centre. Body temperature is 38,50C. What is the most probable
diagnosis?
A. Acute purulent mastitis*
B. Pneumonia
C. Pleuritis
D. Retention of milk
E. Birth trauma

80. A 52 year old patient was admitted to a hospital because of high hemorrhagic diathesis of
mucous membranes, massive skin haemorrhages in form of ecchymoses and spots, nasal and
stomachal haemorrhages. After clinical examinations her illness was diagnosed as
thrombocytopenic purpura. What is the most probable cause of this disease?
A. Generation of antithrombocytic antibodies*
B. Disturbed hemostasis
C. Deficit of the VIII factor of blood coagulation
D. Inherited insufficiency of plasm factors of blood coagulation
E. Iron deficit in blood serum, bone marrow and depot

81. A 37 year old patient applied to a local therapeutist. As a result of exacerbation of chronic
obstructive bronchitis the patient had been temporarily disabled for 117 days within 1 year.
What tactics will be legally correct?
A. The patient should be referred to the medicosocial expertise*
B. The therapeutist should extend a medical certificate
C. The patient should be referred to the medical consultation comission for extension of
medical certificate
D. The therapeutist should issue a new medical certificate
E. The patient shoul be referred to the sanatorium-and-spa treatment

82. A 54 year old male patient complains about permanent dull pain in the mesogastral region,
weight loss, dark blood admixtures in the feces, constipations. He put off 10 kg within a year.
In blood: erythrocytes: 3,5• 1012/l, Hb- 87 g/l, leukocytes - 12,6 • 109/l, stab neutrophil shift,
ESR- 43 mm/h. What is the most probable diagnosis?
A. Cancer of transverse colon*
B. Gastric ulcer
C. Chronic colitis
D. Chronic pancreatitis
E. Stomach cancer

83. A 10 month old boy has been ill for 5 days after consumption of unboiled milk. Body
temperature is 38 - 39oC, there is vomiting, liquid stool. The child is pale and inert. His tongue

17
Завантажено з сайту https://тестування.укр/ - онлайн тестування КРОК

is covered with white deposition. Heart sounds are muffled. Abdomen is swollen, there is
borborygmus in the region of ubbilicus, liver is enlarged by 3 cm. Stool is liquid, dark-green,
with admixtures of mucus, 5 times a day. What is the most probable diagnosis?
A. Salmonellosis*
B. Staphylococcal enteric infection
C. Escherichiosis
D. Acute shigellosis
E. Rotaviral infection

84. Examination of a 22 year old man suffering from polyarthralgia and high fever revealed right-
sided exudative pleuritis. X-ray picture showed a homogenous shadow below the IV rib on the
right. In the II segment there were single dense focal shadows. Mantoux test with 2 TU
resulted in formation of a papula 16 mm large. Pleural liquid has increased protein
concentration, Ri-valta’s reaction is positive, there was also increased number of leukocytes
with prevailing lymphocytes. What is the most probable etiology of pleuritis?
A. Tuberculous*
B. Cancerous
C. Staphylococcal
D. Viral
E. Autoimmune

85. At year-end hospital administration has obtained the following data: annual number of treated
patients and average annual number of beds used for patients’ treatment. What index of
hospital work can be calculated on the base of this data?
A. Bed turnover*
B. Bed resources of the hospital
C. Average annual bed occupancy
D. Average duration of patients’ presence in the hospital
E. Average bed idle time

86. A female patient has been suffering from pain in the right subcostal area, bitter taste in the
mouth, periodical bile vomiting for a month. The patient put off 12 kg. Body temperature in the
evening is 37,6oC. Sonography revealed that bile bladder was 5,5x2,7 cm large, its wall -0,4
cm, choledochus - 0,8 cm in diameter. Anterior liver segment contains a roundish hypoechoic
formation up to 5 cm in diameter and another two up to 1,5 cm each, walls of these formations
are up to 0,3 cm thick. What is the most probable diagnosis?
A. Alveolar echinococcus of liver*
B. Liver cancer
C. Liver abscess
D. Cystous liver cancer
E. Paravesical liver abscesses

87. A 3 year old child with weight deficiency suffers from permanent moist cough. In history there
are some pneumonias with obstruction. On examination: distended chest, dullness on
percussion over the lower parts of lungs. On auscultation: a great number of different rales.
Level of sweat chloride is 80 millimol/l. What is the most probable diagnosis?
A. Mucoviscidosis (cystic fibrosis)*

18
Завантажено з сайту https://тестування.укр/ - онлайн тестування КРОК

B. Bronchial asthma
C. Recurrent bronchitis
D. Bronchiectasis
E. Pulmonary hypoplasia

88. A 14 year old girl complains of profuse bloody discharges from genital tracts during 10 days
after suppresion of menses for 1,5 month. Similiar bleedings recur since 12 years on the
background of disordered menstrual cycle. On rectal examination: no pathology of the internal
genitalia. In blood: Hb - 70 g/l, RBC-2,3 • 1012/l, Ht - 20. What is the most probable diagnosis?
A. Juvenile bleeding, posthemorrhagic anemia*
B. Werlholf’s disease
C. Polycyst ovarian syndrome
D. Hormonoproductive ovary tumor
E. Incomplete spontaneous abortion

89. A 46 year old patient is to be prepared to the operation on account of stomach cancer.
Preoperative preparation involves infusion therapy. It was injected up to 3 l of solutions into
his right lunar vein. On the next day he got tensive pain in the region of his right shoulder.
Examination of interior brachial surface revealed an oblong area of hyperemia, skin edema
and painful cord. What complication is it?
A. Acute thrombophlebitis*
B. Vein puncture and edema of paravenous cellular tissue
C. Necrosis of paravenous cellular tissue
D. Acute lymphangitis
E. Phlegmon of paravenous cellular tissue

90. A 58 year old female patient complains about periodical headache, dizziness and ear noise.
She has been suffering from diabetes mellitus for 15 years. Objectively: heart sounds are
rhythmic, heart rate is 76/min, there is diastolic shock above aorta, AP is 180/110 mm Hg. In
urine: OD- 1,014. Daily loss of protein with urine is 1,5 g. What drug should be chosen for
treatment of arterial hypertension?
A. Ihibitor of angiotensin converting enzyme*
B. 3-blocker
C. Calcium channel antagonist
D. Thiazide diuretic
E. o-blocker

91. A 25 year old patient had pharyngitis 2 weeks ago. Now he complains about body temperature
rise up to 380C', general weakness, dyspnea during walking, swelling and shifting pain in the
articulations. Objectively: cyanosis of lips, rhythmic pulse of poor volume -100 bpm. Left
cardiac border deviates outwards from the mediaclavicular line by 1 cm. The first heart sound
is weakened on the apex, auscultation revealed systolic souffle. What is the most probable aeti-
ological factor that caused this pathological process?
A. 3-haemolytic streptococcus*
B. Staphylococcus
C. Pneumococcus
D. Virus

19
Завантажено з сайту https://тестування.укр/ - онлайн тестування КРОК

E. Fungi

92. A 50 year old locksmith was diagnosed with typhoid fever. The patient lives in a separate
apartment with all facilities. Apart of him there are also 2 adults in his family. What actions
should be taken about persons communicating with the patient?
A. Bacteriological study*
B. Antibiotic prophylaxis
C. Isolation
D. Dispensary observation
E. Vaccination

93. A 39 year old patient complained about morning headache, appetite loss, nausea, morning
vomiting, periodic nasal haemorrhages. The patient had acute glomerulonephritis at the age of
15. Examination revealed rise of arterial pressure up to 220/130 mm Hg, skin haemorrhages
on his arms and legs, pallor of skin and mucous membranes. What biochemical index has the
greatest diagnostic importance in this case?
A. Blood creatinine*
B. Blood bilirubin
C. Blood sodium
D. Uric acid
E. Fibrinogen

94. A 43 year old patient was admitted to the infectious diseases hospital with high body
temperature and intense headache. The iIlness has lasted for 2 days. Examination revealed a
carbuncle on his forearm. The area around it was apparently edematic and slightly painful.
Regional lymphadenitis and hepatoli-enal syndrome were also present. It is known from the
anamnesis that the patient works at a cattle-breeding farm. What disease should be suspected
in the first place?
A. Anthrax*
B. Erysipelas
C. Erysipeloid
D. Skin cancer
E. Eczema

95. Study of morbidity with temporary disability among workers of a machine building plant
revealed that average duration of a case was 20 days. What diseases influenced upon the index
value?
A. Chronic*
B. Acute
C. Subacute
D. Preexisting diseases
E. Hard to determine

96. A 14 year old child suffers from vegetovascular dystonia of pubertal period. He has got
sympathoadrenal atack. What medicine should be used for attack reduction?
A. Obsidan*

20
Завантажено з сайту https://тестування.укр/ - онлайн тестування КРОК

B. No-shpa
C. Amysyl
D. Aminophylline
E. Corglicone

97. A 52 year old male patient complains about attacks of asphyxia, pain in his left side during
respiration. These manifestations turned up all of a sudden. It is known from his anamnesis
that he had been treated for thrombophlebitis of the right leg for the last month. In the
admission ward the patient suddenly lost consciousness, there was a sudden attack of asphyxia
and pain in his left side. Objectively: heart rate - 102/min, respiratory rate - 28/min, AP- 90/70
mm Hg. Auscultation revealed diastolic shock above the pulmonary artery, gallop rhythm,
small bubbling rales above the lungs under the scapula on the right, pleural friction rub. What
examination method will be the most informative for a diagnosis?
A. Angiography of pulmonary vessels*
B. Echocardioscopy
C. Study of external respiration function
D. ECG
E. Coagulogram

98. A 4 month old child fell seriously ill: body temperature rose up to 38,5oC, the child became
inert and had a single vomiting. 10 hours later there appeared rash over the buttocks and
lower limbs in form of petechiae, spots and papules. Some haemorrhagic elements have
necrosis in the centre. What is the most probable disease?
A. Meningococcemia*
B. Rubella
C. Influenza
D. Haemorrhagic vasculitis
E. Scarlet fever

99. A 13 year old girl was admitted to the cardiological department because of pain in the muscles
and joints. Examination of her face revealed an edematic erythema in form of butterfly in the
region of nose bridge and cheeks. What is the most probable diagnosis?
A. Systemic lupus erythematosus*
B. Rheumatism
C. Dermatomyositis
D. Rheumatoid arthritis
E. Periarteritis nodosa

100. A 15 year old girl suddenly got arthralgia, headache, nausea, vomiting; pain and muscle
tension in the lumbar area; body temperature rose up to 38 -39oC. Pasternatsky’s symptom
was distinctly positive on the right. In the urine: bacteriuria, pyuria. What is the most probable
diagnosis?
A. Acute pyelonephritis*
B. Renal colic
C. Acute glomerulonephritis
D. Pararenal abscess
E. Cystitis

21
Завантажено з сайту https://тестування.укр/ - онлайн тестування КРОК

101. A 19 year old patient was admitted to a hospital with acute destructive appendicitis. He suffers
from hemophilia of B type. What antihemophilic medications should be included in pre- and
postoperative treatment plan?
A. Fresh frozen plasma*
B. Cryoprecipitate
C. Fresh frozen blood
D. Native plasma
E. Dried plasma

102. A 59 year old female patient applied to a maternity welfare clinic and complained about bloody
discharges from the genital tracts. Postmenopause is 12 years. Vaginal examination revealed
that external genital organs had signs of age involution, uterus cervix was not erosive, small
amount of bloody discharges came from the cervical canal. Uterus was of normal size, uterine
appendages were unpalpable. Fornices were deep and painless. What method should be
applied for the diagnosis specification?
A. Separated diagnosic curretage*
B. Laparoscopy
C. Puncture of abdominal cavity through posterior vaginal fornix
D. Extensive colposcopy
E. Culdoscopy

103. A 26 year old woman who delivered a child 7 months ago has been suffering from nausea,
morning vomiting, sleepiness for the last 2 weeks. She suckles the child, menstruation is
absent. She hasn’t applied any contraceptives. What method should be applied in order to
specify her diagnosis?
A. Ultrasonic examination*
B. Roentgenography of small pelvis organs
C. Palpation of mammary glands and pressing-out of colostrum
D. Bimanual vaginal examination
E. Speculum examination

104. A boy is 1 year old. Previously he had purulent otitis. After that he started complaining about
pain in the upper third of his left thigh, body temperature rose up to 39oC. Objectively: the
thigh is swollen in its upper third, inguinal fold is smoothed. Extremity is in half-bent position.
Active and passive movements are impossible because of acute pain. What is the most
probable diagnosis?
A. Acute haematogenic osteomyelitis*
B. Acute coxitis
C. Intermuscular phlegmon
D. Osteosarcoma
E. Brodie’s abscess

105. A 2 month old full-term child was born with weight 3500 g and was on the mixed feeding.
Current weight is 4900 g. Evaluate the current weight of the child:
A. Corresponding to the age*
B. 150 g less than necessary
C. Hypotrophy of the I grade

22
Завантажено з сайту https://тестування.укр/ - онлайн тестування КРОК

D. Hypotrophy of the II grade


E. Paratrophy of the I grade

106. A woman consulted a doctor on the 14th day after labour about sudden pain, hyperemy and
induration of the left mammary gland, body temperature rise up to 39oC, headache,
indisposition. Objectively: fissure of nipple, enlargement of the left mammary gland, pain on
palpation. What pathology would you think about in this case?
A. Lactational mastitis*
B. Lacteal cyst with suppuration
C. Fibrous adenoma of the left mammary gland
D. Breast cancer
E. Phlegmon of mammary gland

107. A 32 year old patient suffering from chronic viral hepatitis complains about dull pain in the
right subcostal area, nausea, dry mouth. Objectively: liver dimensions are 13-21-11 cm
(according to Kurlov), spleen is by 2 cm enlarged, aspartate aminotransferase is 3,2
micromole/l-h, alanine aminotransferase - 4,8 millimole/l-h. Serological study revealed HBeAg,
high concentration of DNA HBV. What drug should be chosen for treatment of this patient?
A. a-interferon*
B. Acyclovir
C. Remantadinum
D. Arabinoside monophosphate
E. Essentiale-forte

108. A 5 month old boy was born prematurely, he didn’t suffer from any disease at the infant age
and later on. Examination at an outpatient’s hospital revealed paleness of skin, sleepiness.
Blood count: Hb - 95 g/l, erythrocytes - 3,5 - 1012/l, reticulocytes - 90/oo, colour index - 0,7,
osmotic stability of erythrocytes - 0,44-0,33%, serum iron - 4,9 micromole/l. What is the most
probable cause of anemia?
A. Iron deficit*
B. Hemogenesis immaturity
C. Infectious process
D. Erythrocyte hemolysis
E. B12 deficit

109. A 45 year old woman complains about unbearable pain attacks in the left part of face lasting
1-2 minutes. Such attacks are provoked by mastication. These symptoms appeared 2 months
ago after exposure to cold. Objectively:the pain is localized in the ending points of trigeminus
on the left. A touch near nose wing provokes another pain attack and tonic spasm of face
muscles. What is the most probable diagnosis?
A. Neuralgia of trigeminus*
B. Neuralgia of glossopharyngeal nerve
C. Arthritis of mandibular joint
D. Facial migraine
E. Maxillary sinusitis

23
Завантажено з сайту https://тестування.укр/ - онлайн тестування КРОК

110. A 38 year old man was delivered to the hospital in unconscious state. The symptoms of illness
turned up a day before: headache, nausea, vomiting, to-38,5oC, dizziness, delusion. For the
last 4 days he had been complaining of pain and hearing loss in the left ear. Objectively: sopor,
rigidity of occipital muscles, bilateral Kernig’s symptom, general hyperesthesia, purulent
discharges from the left ear. What is the most probable diagnosis?
A. Secondary purulent meningitis*
B. Primary purulent meningitis
C. Tuberculous meningitis
D. Subarachnoidal haemorrhage
E. Parenchymatous subarachnoidal haemorrhage

111. A 70 year old man is suffering from coronary heart disease. His mood is evidently depressed,
anxious. As a result of continious sleeplessness he has got fears, suicidal thoughts. He would
sit for a long time in the same pose, answer after a pause, in a low, monotonous voice. His face
has a look of suffering, pain, fear. What is the main psychopathologic syndrome?
A. Depressive syndrome*
B. Paranoid syndrome
C. Asthenic syndrome
D. Phobic syndrome
E. Obsessive syndrome

112. A 23 year old female patient complains about periodical chill and body temperature rise up to
40oC, sense of heat taking turns with profuse sweating. The patient has had already 3 attacks
that came once in two days and lasted 12 hours. She has lived in Africa for the last 2 months.
Liver and spleen are enlarged. In blood: erythrocytes - 2,5 • 1012/l. What is the most probable
diagnosis?
A. Malaria*
B. Spotted fever
C. Sepsis
D. Haemolytic anaemia
E. Leptospirosis

113. A 39 year old patient suffering from hypertension suddenly gotb intense headache in the
region of occiput; there appeared recurrent vomiting. These presentations has been lasting for
5 hours. Objectively: Ps - 88 bpm, Ap -205/100 mm Hg, painfulness of occipital points, rigidity
of occipital muscles are present. Kernig’s symptom is bilaterally positive. Subarachnoid
haemorrhage is also suspected. What diagnostic method will be of the greatest importance for
confirmation of provisional diagnosis?
A. Lumbar puncture*
B. Examination of eye fundus
C. Ultrasonic dopplerography
D. EEG
E. Computer tomography

114. Immediately after delivery a woman had haemorrhage, blood loss exceeded postpartum
haemorrhage rate and was progressing. There were no symptoms of placenta detachment.
What tactics should be chosen?

24
Завантажено з сайту https://тестування.укр/ - онлайн тестування КРОК

A. Manual removal of placenta and afterbirth*


B. Uterus tamponade
C. Instrumental revision of uterine cavity walls
D. Removal of afterbirth by Crede’s method
E. Intravenous injection of methylergometrine with glucose

115. A 33 year old patient was delivered to the infectious diseases department on the 7-th day of
disease. He complained about great weakness, high temperature, pain in the lumbar area and
leg muscles, icteritiousness, dark colour of urine, headache. The acute disease started with
chill, body temperature rise up to 40oC, headache, pain in the lumbar area and sural muscles.
Icterus turned up on the 4th day, nasal and scleral haemorrhages came on the 5th day. Fever
has lasted for 6 days. Diuresis - 200 ml. What is the most probable diagnosis?
A. Leptospirosis*
B. Typhoid fever
C. Virus A hepatitis
D. Sepsis
E. Yersiniosis

116. A 72 year old male patient complains about itch in his left shin, especially around a trophic
ulcer. Skin is reddened and edematic, there are some oozing lesions, single yellowish crusts.
The focus of affection is well-defined. What is the most probable diagnosis?
A. Microbial eczema*
B. Allergic dermatitis
C. Seborrheic eczema
D. Cutaneous tuberculosis
E. Streptococcal impetigo

117. A 29 year old female patient complains about periodical right-sided headache that is usually
provoked by strong smells or excitement. The patient’s mother suffers from the same disease.
Objectively: examination of internal organs revealed no pathology. During the attack there are
general hyperesthesia and nausea, at the end of the attack polyuria is observed. Palpation of
the right temporal artery during the attack revealed tension and painfulness of the artery.
Complete blood count and urine analysis reveale dno pecularities. What is the most probable
diagnosis?
A. Migraine*
B. Neuritis of the facial nerve
C. Neuralgia of trigeminus
D. Meniere’s syndrome
E. Epilepsy

118. A 52 year old woman complains about face distortion. It turned up 2 days ago after
supercooling. Objectively: body temperature is 38,2oC. Face asymmetry is present. Frontal
folds are flattened. Left eye is wider than right one and doesn’t close. Left nasolabial fold is
flattened, mouth corner is lowered. Examination revealed no other pathology. Blood count:
leukocytes - 10 • 109/l, ESR - 20 mm/h. What is the most probable diagnosis?
A. Facial neuritis*
B. Trigeminus neuralgia

25
Завантажено з сайту https://тестування.укр/ - онлайн тестування КРОК

C. Hemicrania (migraine)
D. Ischemic stroke
E. Brain tumour

119. A 10 year old boy suffers from chronic viral hepatitis type B with maximal activity. What
laboratory test can give the most precise characteristic of cytolysis degree?
A. Transaminase test*
B. Weltman’s coagulation test
C. Takata-Ara test
D. Prothrombin test
E. Test for whole protein

120. Examination of a 12 year old chi- ld revealed diffuse thyroid enlargement of the II degree.
Heart auscultation revealed dullness of heart sounds, heart rate was 64/min. The child has
frequent constipations, anemia. Concentration of thyreoglobulin antibodies is increased. What
disease might have caused such symptoms?
A. Autoimmune thyroiditis*
B. Diffuse toxic goiter
C. Thyroid carcinoma
D. Thyroid hyperplasia
E. Endemic goiter

121. A 13 year old patient is suffering from an acute disease with the following symptoms: thirst,
polyuria, weakness. Objectively: his general condition is satisfactory, there is no smell of
acetone. Glucose concentration in blood on an empty stomach is 32 micromole/l, in urine - 6%,
acetone +. What treatment should be administered?
A. Short-acting insulin*
B. Long-acting insulin
C. Biguanides
D. Sulfonylurea
E. Diet

122. Surgical department admitted a 37 year old patient with a big crushed wound of his left thigh
4 hours after he got this trauma. What is the main provision for successful prevention of
gaseous gangrene?
A. Removal of necrotic tissues and timely surgical processing of the wound*
B. Injection of specific serum 3 000 U
C. Injection of specific serum 30 000 U
D. Infiltration of soft tissues around the wound with antibiotic solution
E. Wound lavage with 6% solution of hydrogen peroxide

123. A 40 year old patient was bitten by a stray dog for about an hour ago. The bite can be seen on
the patient’s left shin in form of a wound 4x2x0,5 cm large. What kind of aid would be
recommended in this case?
A. Wound lavage with soapsuds, retension sutures*
B. Aseptic bandage

26
Завантажено з сайту https://тестування.укр/ - онлайн тестування КРОК

C. Salve bandage
D. Blind suture
E. Retension sutures

124. A patient has got acute macrofocal myocardial infarction complicated by cardiogenic shock.
The latter is progresing under conditions of weak general peripheric resistance and lowered
cardiac output. What antihypotensive drug should be injected to the patient in the first place?
A. Dopamine*
B. Noradrenaline
C. Adrenaline
D. Mesatonum
E. Prednisolone

125. A patient applied to the traumatology cenre and complained about a trauma of the lower third
of the volar forearm surface caused by cut on a piece of galss. Objectively: flexion of the IV and
V fingers is impaired, sensitivity of the interior dorsal and palmar surface of hand as well as of
the IV finger is reduced. What nerve is damaged?
A. Ulnar*
B. Radial
C. Median
D. Musculoskeletal
E. Axillary

126. It was noticed that a 7 year old pupil had been innattentive several times during the lessons.
The teacher also noticed that the child had been smacking his lips and had vacant look. There
were no falls and convulsions. During such short periods of absence the child didn’t react to
calling his name. His mother notic such phenomena before but didn’t pay much attention to
them thinking that the child was deep in thought. What type of epileptic attack (according to
the standard classification) is it?
A. Absentia*
B. Generalized tonoclonic epilepsy
C. Simple partial epilepsy
D. Complex partial epilepsy
E. Jacksonian partial epilepsy

127. A 41 year old woman has suffered from nonspecific ulcerative colitis for 5 years. On
rectoromanoscopy: evident inflammatory process of lower intestinal parts, pseudopolyposive
changes of mucous membrane. In blood: WBC-9,8 • 109/l, RBC- 3,0 • 1012/l, ESR -52
mm/hour. What medication provides pathogenetic treatment of this patient?
A. Sulfosalasine*
B. Motilium
C. Vikasolum
D. Linex
E. Kreon

128. A 38 year old female patient complains about body stiffness in the morning, especially in the

27
Завантажено з сайту https://тестування.укр/ - онлайн тестування КРОК

articulations of her upper and lower limbs, that disappears 30-60 minutes later after active
movements. She has also arthritis of metacarpophalangeal and proximal phalangeal
articulations, subfebrile temperature. ESR- 45 mm/h. Roentgenography revealed osteoporosis
and erosion of articular surface of small hand and foot articulations. What is the most probable
diagnosis?
A. Rheumatoid arthritis*
B. Psoriatic arthropathy
C. Osteoarthrosis deformans
D. Systemic lupus erythematosus
E. Reactive polyarthritis

129. A child was born with body weight 3250 g and body length 52 cm. At the age of 1,5 month the
actual weight is sufficient (4350 g), psychophysical development corresponds with the age.
The child is breast-fed, occasionally there are regurgitations. What is the cause of
regurgitations?
A. Aerophagia*
B. Pylorostenosis
C. Pylorospasm
D. Acute gastroenteritis
E. Esophageal atresia

130. Three weeks after acute angina the patient is still weak, inert, subfebrile, his retromaxillary
lymph nodes are enlarged. Tonsils are flabby, stick together with arches, there are purulent
plugs in lacunae. What is the most probable diagnosis?
A. Chronic tonsillitis*
B. Chronic pharyngitis
C. Acute lacunar tonsillitis
D. Paratonsillitis
E. Tonsillar tumour

131. A 23 year old patient fell ill 3 weeks ago when she noticed a very painful induration in her
axillary crease. 4-5 days later it burst and discharged a lot of pus. After that some new
infiltrations appeared around the affected area. The patient has never suffered from skin di-
seases before. What is the most probable diagnosis?
A. Hydradenitis*
B. Furuncle
C. Mycosis
D. Herpes zoster
E. Streptococcal impetigo

132. A 28 year old woman has bursting pain in the lower abdomen during menstruation; chocolate-
like discharges from vagina. It is known from the anamnesis that the patient suffers from
chronic adnexitis. Bimanual examination revealed a tumour-like formation of heterogenous
consistency 7x7 cm large to the left from the uterus. The formation is restrictedly movable,
painful when moved. What is the most probable diagnosis?
A. Endometrioid cyst of the left ovary*
B. Follicular cyst of the left ovary

28
Завантажено з сайту https://тестування.укр/ - онлайн тестування КРОК

C. Fibromatous node
D. Exacerbation of chronic adnexitis
E. Tumour of sigmoid colon

133. As a result of prophylactic medical examination a 35 year old woman was diagnosed with
alimentary and constitutive obesity of the III degree. It is known from her anamnesis that the
patient doesn’t observe rules of rational nutrition: she often overeats, the last food intake is
usually 10-15 minutes before going to bed, prefers fattening and rich in carbohydrates food.
What is the main alimentary risk factor of obesity development?
A. Energetic unprofitableness of nutrition*
B. Excess of carbohydrates
C. Excess of fats
D. Lack of cellulose
E. Violation of dietary pattern

134. A 40 year old female patient has been observing excessive menstruation accompanied by
spasmodic pain in the lower abdomen for a year. Bimanual examination performed during
menstruation revealed a dense formation up to 5 cm in diameter in the cervical canal. Uterus
is enlarged up to 5-6 weeks of pregnancy, movable, painful, of normal consistency.
Appendages are not palpable. Bloody discharges are profuse. What is the most probable
diagnosis?
A. Nascent submucous fibromatous node*
B. Abortion in progress
C. Cervical carcinoma
D. Cervical myoma
E. Algodismenorrhea

135. A child is 1 day old. During delivery there had been problems with extraction of shoulders.
Body weight is 4300,0. Right arm hangs down along the body, hand is pronated, movement in
the arm is absent. "Scarf"symptom is positive. What is the most probable diagnosis?
A. Total right-sided obstetric paralysis*
B. Proximal right-sided obstetric paralysis
C. Distal right-sided obstetric paralysis
D. Hemiparesis
E. Tetraparesis

136. A 42 year old man applied to a hospital 10 minutes after he got stung by a bee and complained
about face edema and difficult respiration. Objectively: Ps- 98 bpm, AP- 130/80 mm Hg. A
doctor on duty injected him 1 ml of 1% dimedrol solution intramuscularly and recommended to
apply to his local therapeutist on the next day. What tactics of treatment should be chosen for
this patient?
A. Intravenous introduction of prednisolone and hospitalization*
B. Intravenous introduction of calcium chloride
C. The patient needs no further medical aid
D. Hospitalization for observation
E. Intravenous introduction of calcium chloride and hospitalization

29
Завантажено з сайту https://тестування.укр/ - онлайн тестування КРОК

137. A 44 year old man has been working in coke industry for 16 years. Dust concentration at his
workplace is 5-10 times more than maximum permissible concentration. Roentgenography of
lungs revealed changes that are typical for pneumoconiosis. What is the most probable type of
pneumoconiosis in this case?
A. Anthracosis*
B. Anthracosilicosis
C. Silicatosis
D. Asbestosis
E. Siderosis

138. Study of morbidity rate in a city N revealed that population of different administrative districts
differed in age structure. What statistic method allows to eliminate influence of this factor
upon morbidity indices?
A. Standardization*
B. Wilcoxon’s t-criterion
C. Correlative regressive analysis
D. Analysis of dynamic series
E. Calculation of average values

139. An outbreak of food poisoning was recorded in an urban settlement. The illness was diagnosed
as botulism on the grounds of clinical presentations. What foodstuffs should be chosen for
analysis in the first place in order to confirm the diagnosis?
A. Tinned food*
B. Potatoes
C. Pasteurized milk
D. Boiled meat
E. Cabbage

140. A 72 year old female patient has been treated for urolithiasis in the urological department.
After atropine injection she got acute pain in her left eye and abrupt vision impairment.
Objectively: visual acuity of the left eye is 0,01, the eye is dense but painful on palpation,
cornea is opaque, there is cyanotic induration of eyeball vessels. What is the most probable
diagnosis?
A. Acute attack of primary glaucoma of the left eye*
B. Acute iridocyclitis of the left eye
C. Secondary glaucoma of the left eye
D. Acute keratitis of the left eye
E. Degeneration of the left eye cornea

141. Administration of a plant producing red lead paint intends to form a group of medical
specialists for periodical medical examinations. What specialist must be obligatory included
into this group?
A. Neuropathologist*
B. Gynaecologist
C. Psychiatrist
D. Dermatologist
E. Otolaryngologyst

30
Завантажено з сайту https://тестування.укр/ - онлайн тестування КРОК

142. A 42 year old man works in a boiler room. He complains about girdle headache and recurring
vomiting. There was also short-term consciousness loss. Objectively: increase of tendon
reflexes, spontaneous myofibrillations. AP is 150/80 mm Hg, Ps- 104 bpm. Visible mucous
membranes and cutaneous surfaces have crimson colouring. What is the most probable
diagnosis?
A. Poisoning with carbon monooxide*
B. Poisoning with hydrocyanic acid
C. Poisoning with anilin colouring agents
D. Poisoning with methane
E. Poisoning with benzine

143. The amount of ultraviolet radiation dose was measured in minutes. What device was applied
for measurement of the biodose?
A. Gorbachev’s biodosimeter*
B. UV-meter
C. Actinometer
D. Radiometer
E. Catathermometer

144. A 63 year old patient complained about pain in the lumbar area. He underwent a course of
physiological treatment on account of radiculitis but this led to no improvement of his
condition. R-graphy of spinal column and pelvic bones revealed osteoporosis and serious bone
defects. Blood analysis revealed moderate normochromic anaemia, urine analysis revealed
proteinuria. Whole blood protein made up 10,7 g/l. What disease should be suspected?
A. Myelomatosis*
B. Urolithiasis
C. Acute radiculitis
D. Metastases in bones
E. Systemic osteoporosis

145. A sergeant was injured by a shell splinter in the left subcostal area. He was bandaged with a
first-aid pack on a battlefield. The patient was delivered to the regiment medical aid station.
He complains about dizziness, weakness, thirst, abdominal pain. General condition is grave,
the patient is pale. Ps is 120 bpm. Abdomen is soft, painful on palpation. The bandage is well
fixed but a little bit soaked with blood. The patient should be evacuated to the medical
battalion with the following transport and in the following turn:
A. With medical vehicle in the first turn*
B. With a passing car in the first turn
C. With medical vehicle in the second turn
D. With a passing car in the second turn
E. With a passing car in the third turn

146. On the 5th day after labor body temperature of a parturient suddenly rose up to 38,7oC. She
complains about weakness, headache, abdominal pain, irritability. Objectively: AP- 120/70 mm
Hg, Ps- 92 bpm, to- 38,7oC. Bimanual examination revealed that the uterus was enlarged up to
12 weeks of pregnancy, it was dense, slightly painful on palpation. Cervical canal lets in 2
transverse fingers, discharges are moderate, turbid, with foul smell. In blood: skeocytosis,

31
Завантажено з сайту https://тестування.укр/ - онлайн тестування КРОК

lymphopenia, ESR- 30 mm/h. What is the most probable diagnosis?


A. Endometritis*
B. Parametritis
C. Pelviperitonitis
D. Metrophlebitis
E. Lochiometra

147. A 19 year old boy was admitted to a hospital with closed abdominal trauma. In course of
operation multiple ruptures of spleen and small intestine were revealed. AP is falling rapidly, it
is necessary to perform hemotransfusion. Who can specify the patient’s blood group and
rhesus compatibility?
A. A doctor of any speciality*
B. A laboratory physician
C. A surgeon
D. A traumatologist
E. An anaesthesilogist

148. A 36 year old man was delivered to the surgical department an hour after a road accident. His
condition is getting worse: respiratory insufficiency is progressing, there are cardiac
abnormalities. Clinical and roentgenological investigations revealed mediastinal displacement.
What process has caused this complication?
A. Valvular pneumothorax*
B. Open pneumothorax
C. Closed pneumothorax
D. Subcutaneous emphysema
E. Mediastinitis

149. An 18 year old primigravi-da in her 27-28 week of pregnancy underwent an operation on
account of acute phlegmonous appendicitis. In the postoperative period it is necessary to take
measures for prevention of the following pegnancy complication:
A. Noncarrying of pregnancy*
B. Intestinal obstruction
C. Fetus hypotrophy
D. Premature placenta detachment
E. Late gestosis

150. It is planned to make complete isolation boxes in the infectious department in order to prevent
nosocomial airborne infections. The boxes consist of a tambour, a ward and a lock chamber.
What structure should be also included in a complete isolation box?
A. Bathroom unit*
B. Manipulation room
C. Doctor’s consulting room
D. Patient’s examination room
E. Nursing room

151. A 3 year old boy has petechial eruption. Examination revealed no other pathological changes.

32
Завантажено з сайту https://тестування.укр/ - онлайн тестування КРОК

Thrombocyte number is 20 • 109g/l; haemoglobin and leukocyte concentration is normal. What


is the most probable diagnosis?
A. Immune thrombocytopenic purpura*
B. Schonlein-Henoch disease
C. Disseminated intravascular coagulopathy
D. Acute lymphoblastic leukemia
E. Systemic lupus erythematosus

152. Indices that characterize population health include demographic indices. What environment is
used for calculation of these indices?
A. Population number*
B. Employment number
C. Number of hospitalized people
D. Number of patients
E. Number of population being liable to preventive examination

153. An infant is full-term. Delivery was pathological, with breech presentation. Examination of the
infant revealed limited abduction of the right leg to 50o, positive "clicking"symptom on the
right, asymmetric inguinal folds. What is the most probable diagnosis?
A. Inborn dislocation of the right hip*
B. Inborn dislocation of both hips
C. Varus deformity of both femoral necks
D. Fracture of both femoral necks
E. Right hip dysplasia

154. A 30 year old man complains of intense pain, reddening of skin, edema in the ankle-joint area,
fever up to 39oC. There was an acute onset of the illness. In the past there were similar
attacks lasting 5-6 days without residual changes in the joint. The skin over the joint is
hyperemic and ill-defined, without infiltrative bank on the periphery. What is the most likely
diagnosis?
A. Gout*
B. Infectious arthritis
C. Rheumatoid arthritis
D. Erysipelatous inflammation
E. Osteoarthritis

155. The results of 5 year monitoring allowed to estimate the level of environmental influence upon
health indices of popultaion. What statistic method should be chosen?
A. Calculation of correlation coefficient*
B. Calculation of conformity coefficient
C. Calculation of coefficient of difference validity
D. Calculation of regression coefficient
E. Calculation of dynamic indices

156. A 37 year old miner has lifted significant loads and afterwards felt pain in the lumbar area
irradiating to his left leg. He walks slowly and carefully. Lumbar lordosis is flattened. There is

33
Завантажено з сайту https://тестування.укр/ - онлайн тестування КРОК

also left-sided scoliosis and tension of paravertebral muscles. Neri’s and Dejeri-ne’s symptoms
are positive, there is Lasegue’s sign on the left from the angle of 350. What method will help to
specify the diagnosis?
A. CT of lumbosacral part of vertebral column*
B. Lumbal puncture
C. Renal sonography
D. Descending myelography
E. Electromyography

157. A 47 year old male patient got a flame burn of trunk and upper extremities and was delivered
to the hospital. The patient is in grave condition, confused mental state, with fever. AP- 80/50
mm Hg, Ps-118 bpm. It was locally stated that the patient got III B degree burns with total
area of 20%. What medical actions should be taken?
A. Injection of narcotic analgetics and powdered blood substitutes*
B. Primary surgical pocessing
C. Administration of detoxicating blood substitutes
D. Necrotomy of burn surface, haemotransfusion
E. Antibacterial and detoxicating therapy

158. A 54 year old female patient was admitted to the hospital with evident acrocyanosis, swollen
cervical veins, enlarged liver, ascites. Cardiac borders are dilated. Heart sounds cannot be
auscultated, apical beat is undetectable. AP is 100/50 mm Hg. X-ray picture of chest shows
enlarged heart shadow in form of a trapezium. What pathology might have caused these
symptoms?
A. Cardiac tamponade*
B. Exudative pleuritis
C. Complex heart defect
D. Acute cardiac insufficiency
E. Hiatal hernia

159. 15 minutes after the second vaccination with diphteria and tetanus toxoids and pertussis
vaccine a 4 month old boy manifested symptoms of Quincke’s edema. What medication should
be applied for emergency aid?
A. Prednisolone*
B. Heparin
C. Adrenalin
D. Furosemide
E. Seduxen

160. A 16 year old female patient underwent an operation on account of diffuse toxic goiter of the
III-IV degree 12 years ago. Now she has recurrence of thyrotoxicosis. The patient was offered
operative intervention, but it is necessary first to localize the functioning gland tissue. What
method should be applied for this purpose?
A. Gland scanning*
B. USI
C. Puncture aspiration biopsy
D. Roentgenography of neck

34
Завантажено з сайту https://тестування.укр/ - онлайн тестування КРОК

E. Roentgenography of esophagus

161. Examination of a 26 year old female patient revealed a node in the right lobe of thyroid gland.
The node appeared no earlier than 3 months ago. The patient associates this node with stress.
She doesn’t complain either about pain or enlargement of the node. Ultrasonic scanning
revealed a 2x2,5 cm large node in the inferior part of the right lobe of thyroid gland. What
treatment should be administered?
A. Surgical intervention*
B. Conservative therapy
C. Dynamic observation
D. No need for treatment
E. -

162. During examination a patient is unconscious, his skin is dry and hot, face hyperemia is
present. The patient has Kussmaul’s respiration, there is also smell of acetone in the air.
Symptoms of peritoneum irritation are positive. Blood sugar is at the rate of 33 millimole/l.
What emergency actions should be taken?
A. Intravenous infusion of short-acting insulin*
B. Intravenous infusion of glucose along with insulin
C. Introduction of long-acting insulin
D. Intravenous infusion of neohaemodesum along with glutamic acid
E. Intravenous infusion of sodium chloride saline

163. A patient complains about evaginati-ons in the region of anus that appear during defecation
and need to be replaced. Examination with anoscope revealed 1x1 cm large evaginations of
mucosa above the pectineal line. What is the most probable diagnosis?
A. Internal hemorrhoids*
B. Acute paraproctitis
C. External hemorrhoids
D. Anal fissure
E. -

164. A 34 year old patient was delivered to the hospital because of follicular tonsillitis charactirized
by high temperature. The patient has been abusing alcohol for 12 years. In the evening on the
day of hospitalization he became anxious, couldn’t stay in bed, left his ward several times and
applied to the staff on duty with different complaints. He reported about seeing alot of spiders
and flies in his ward as well as abou hearing threats from the corridor. He was exasperated by
the fact that other patients didn’t hear them. He lost also spatial orientation. What
psychopathological syndrome is it?
A. Delirious*
B. Oneiric
C. Amentive
D. Twilight state
E. Asthenic confusion

165. On the third day of life an infant’s skin got icteric colouring. The child was born with body

35
Завантажено з сайту https://тестування.укр/ - онлайн тестування КРОК

weight of 3,200 kg, body length of 52 cm. The child is active. There is puerile respiration
above the lungs. Respiratory rate is 36/min, heart sounds are rhythmic, heart rate is 130/min.
Abdomen is soft, liver comes out from the edge of costal arch by 2 cm, spleen is not palpable.
Feces are in form of meconium. What is the most probable diagnosis?
A. Physiologic jaundice*
B. Hemolytic disease of newborn
C. Neonatal sepsis
D. Minkowsky-Shauffard disease
E. Biliary tracts atresia

166. A pediatrician talked to a mother of a 7 month old breast-fed boy and found out that the child
was fed 7 times a day. How many times should the child of such age be fed?
A. 5 times*
B. 3 times
C. 4 times
D. 6 times
E. 7 times

167. In order to study impact of microclimate upon the human organism it is necessary to make
systematic observation of air temperature over 3 days. Choose a device that will allow to make
the most precise temperature records:
A. Thermograph*
B. Alcohol thermometer
C. Mercury thermometer
D. August’s psychrometer
E. Assmann psychrometer

168. A prematurely born girl is now 8 months old. She has dyspnea, tachycardia,
hepatosplenomegaly, physical developmental lag, limb cyanosis. There is also parasternal
cardiac hump, auscultation revealed systolodiastolic murmur in the II intercostal space on the
left. AP is 90/0 mm Hg. What disease should be suspected?
A. Patent ductus arteriosus*
B. Coarctation of aorta
C. Stenosis of aortal valve
D. Stenosis of pulmonary artery
E. Nonclosure of interventricular septum

169. A 72 year old patient complains about pain and bleeding during defecation. Digital rectal
investigation revealed a tumour of anal canal. After verification of the diagnosis the patient
was diagnosed with squamous cell carcinoma. The secondary (metastatic) tumour will be most
probably found in:
A. Lungs*
B. Liver
C. Pelvic bones
D. Mediastinum
E. Brain

36
Завантажено з сайту https://тестування.укр/ - онлайн тестування КРОК

170. A 52 year old patient complains about headache, weakness of his upper left extremity.
Neurological symptoms become more intense during physical stress of the left extremity.
Pulsation on the arteries of the left extremity is sharply dampened but it remains unchanged
on the carotid arteries. What is the most probable diagnosis?
A. Occlusion of the left subclavicular artery, steal syndrome*
B. Thoracal outlet syndrome
C. Raynaud’s syndrome
D. Takayasu’s disease
E. Occlusion of brachiocephalic trunk

171. A 25 year old woman applied to a maternity welfare clinic and complained about being unable
to conceive within 3 years of regular sexual life. Examination revealed weight gain, male
pattern of hair distribution on the pubis, excessive pilosis of thighs. Ovaries were dense and
enlarged, basal temperature was monophase. What is the most probable diagnosis?
A. Sclerocystosis of ovaries*
B. Tubo-ovaritis
C. Adrenogenital syndrome
D. Premenstrual syndrome
E. Gonadal dysgenesis

172. A woman consulted a therapeutist about fatigability, significant weight loss, weakness, loss of
appetite. She has had amenorrhea for 8 months. A year ago she born a full-term child.
Haemorrhage during labour made up 2 l. She got blood and blood substitute transfusions.
What is the most probable diagnosis?
A. Sheehan’s syndrome*
B. Stein-Leventhal syndrome
C. Shereshevsky-Turner’s syndrome
D. Homological blood syndrome
E. Vegetovascular dystonia

173. A 35 year old female patient suffering from cholelithiasis has broken her diet, and this caused
an acute pain attack in the right subcostal are. The pain eased off on the third day, but the
patient got progressing jaundice. What non-invasive diagnostic method should be applied?
A. Endoscopic retrograde cholangiopancreatography*
B. Infusive cholecystocholangiography
C. Test for bilirubin
D. Duodenal probing
E. Survey radiography of abdominal organs

174. A 68 year old patient complains about acute pain in his right foot, toe edema and darkening of
skin of the IV toe. He has been suffering from diabetes mellitus for 15 years, doesn’t receive
regular treatment. What complication of diabetes mellitus is it?
A. Gangrene of the IV toe on the right foot*
B. Panaritium
C. Haematoma
D. Erysipelas
E. Fracture of the IV toe on the right foot

37
Завантажено з сайту https://тестування.укр/ - онлайн тестування КРОК

175. A plot of land with total area of 2,0 hectare was intended for building of a hospital. The
maximal capacity of the hospital will be:
A. 100 beds*
B. 200 beds
C. 400 beds
D. 800 beds
E. Over 1000 beds

176. A 26 year old woman complains about edemata, swelling and painfulness of mammary glands,
headache, tearfulness, irritability. These signs turn up 5 days before menstruation and
disappear after its start. What clinical syndrome is it?
A. Premenstrual syndrome*
B. Postcastration syndrome
C. Adrenogenital syndrome
D. Climacteric syndrome
E. Stein-Leventhal syndrome

177. A 27 year old patient suffers from haemophilia. He was admitted to the hospital with melena
and skin pallor. Objectively: Ps-110 bpm, AP-100/60 mm Hg. In blood: Hb- 80 g/l, erythrocytes
-2,8 • 1012/l. What medication should be administered in the first place?
A. Cryoprecipitate*
B. Stored blood
C. Packed red blood cells
D. Dicinone
E. Epsilon-aminocapronic acid

178. A 38 year old patient complains about inertness, subfebrile temperature, enlargement of
lymph nodes, nasal haemorrhages, bone pain. Objectively: the patient’s skin and mucous
membranes are pale, palpation revealed enlarged painless lymph nodes; sternalgia; liver was
enlarged by 2 cm, spleen - by 5 cm, painless. In blood: erythrocytes - 2,7 • 1012/l, Hb- 84 g/l,
leukocytes - 58 • 109/l, eosinophils -1%, stab neutrophils - 2%, segmented neutrophils - 12%,
lymphocytes - 83%, lymphoblasts - 2%, smudge cells; ESR-57 mm/h. What is the most probable
diagnosis?
A. Chronic lymphatic leukemia*
B. Chronic myeloleukemia
C. Acute lymphatic leukemia
D. Acute myeloleukemia
E. Lymphogranulomatosis

179. A 36 year old female patient complains about general weakness, edemata of her face and
hands, rapid fatigability during walking, difficult di-glutition, cardiac irregularities. These
symptoms turned up 11 days after holiday at the seaside. Objectively: face erythema, edema of
shin muscles. Heart sounds are muffled, AP is 100/70 mm Hg. In blood: ASAT activity is 0,95
millimole/h-l, ALAT - 1,3 micromole/h-l, aldolase - 9,2 IU/l, creatine phosphoki-nase - 2,5
micromole P/g-l. What method of study would be the most specific?
A. Muscle biopsy*
B. ECG

38
Завантажено з сайту https://тестування.укр/ - онлайн тестування КРОК

C. Echocardiogram
D. Electromyography
E. Determination of cortisol concentration in blood and urine

180. A 33 year old patient has acute blood loss (erythrocytes - 2,2• 1012/l, Hb- 55 g/l), blood group
is A(II)Rh+. Accidentally the patient got transfusion of donor packed red blood cells of
AB(IV)Rh+ group. An hour later the patient became anxious, got abdominal and lumbar pain.
Ps- 134 bpm, AP- 100/65 mm Hg, body temperature - 38,60C. After catheterization of urinary
bladder 12 ml/h of dark-brown urine were obtained. What complication is it?
A. Acute renal insufficiency*
B. Cardial shock
C. Allergic reaction to the donor red blood cells
D. Citrate intoxication
E. Toxic infectious shock

181. A parturient woman is 27 year old, it was her second labour, delivery was at term, normal
course. On the 3rd day of postpartum period body temperature is 36,8oC, Ps - 72/min, AP -
120/80 mm Hg. Mammary glands are moderately swollen, nipples are clean. Abdomen is soft
and painless. Fundus of uterus is 3 fingers below the umbilicus. Lochia are bloody, moderate.
What is the most probable diagnosis?
A. Physiological course of postpartum period*
B. Subinvolution of uterus
C. Postpartum metroendometritis
D. Remnants of placental tissue after labour
E. Lactostasis

182. A patient suffering from gastroesophageal reflux has taken from time to time a certain drug
that "reduces acidity"for 5 years. This drug was recommended by a pharmaceutist. The
following side effects are observed: osteoporosis, muscle weakness, indisposition. What drug
has such following effects?
A. Aluminium-bearing antacid*
B. Inhibitor of proton pump
C. H2-blocker
D. Metoclopramide
E. Gastrozepin

183. A 34 year old patient complains of profuse sweating at night, skin itching, weight loss (9 kg
within the last 3 months). Examination revealed malnutrition, skin pallor. Palpation of neck
and inguinal areas revealed dense elastic lymph nodes for about 1 cm in diameter, nonmobile,
non-adhering to skin. What is the most probable diagnosis?
A. Lymphogranulomatosis*
B. Chronic lymphadenitis
C. Lymphosarcoma
D. Burkitt’s lymphoma
E. Cancer metastases

39
Завантажено з сайту https://тестування.укр/ - онлайн тестування КРОК

184. A department chief of an in-patient hospital is going to examine resident doctors as to


observation of medical-technological standards of patient service. What documentation should
be checked for this purpose?
A. Health cards of in-patients*
B. Statistic cards of discharged patients
C. Treatment sheets
D. Registry of operative interventions
E. Annual report of a patient care institution

185. A woman is 34 years old, it is her tenth labor at full term. It is known from the anamnesis that
the labor started 11 hours ago, labor was active, painful contractions started after discharge of
waters and became continuous. Suddenly the parturient got knife-like pain in the lower
abdomen and labor activity stopped. Examination revealed positive symptoms of peritoneum
irritation, ill-defined uterus outlines. Fetus was easily palpable, movable. Fetal heartbeats
wasn’t auscultable. What is the most probable diagnosis?
A. Rupture of uterus*
B. Uterine inertia
C. Discoordinated labor activity
D. Risk of uterus rupture
E. II labor period

186. A 22 year old female patient complains about frequent and painful urination, urge to urinate at
night, enuresis, pain in the suprapubic and lumbar area. Her urine often has beer colouring.
She got married a month ago. Objectively: general state is satisfactory. Lung examination
revealed vesicular respiration. Heart sounds are rhythmic, heart rate is 78/min, AP- 128/68
mm Hg. Abdomen is soft, painful in the suprapubic area. Urine contains 12-18 erythrocytes
and 12-15 bacteria within eyeshot. What is the most probable diagnosis?
A. Infection of inferior urinary tracts -cystitis*
B. Urolithiasis
C. Infection of superior urinary tracts -pyelonephritis
D. Gonorrhoea
E. Primary syphilis

187. Examination of placenta revealed a defect. An obstetrician performed manual investigation of


uterine cavity, uterine massage. Prophylaxis of endometritis in the postpartum period should
involve following actions:
A. Antibacterial therapy*
B. Instrumental revision of uterine cavity
C. Haemostatic therapy
D. Contracting agents
E. Intrauterine instillation of dioxine

188. A 9 year old boy had acute respiratory viral infection. After it there appeared polydipsia,
polyuria, weakness, nausea. Examination revealed the following symptoms: mental confusion,
dry skin, soft eyeballs, Kussmaul’s respiration, acetone smell from the mouth, muffled heart
sounds, soft and painless abdomen. Blood sugar was 19 millimole/l. What acute condition is it?
A. Ketoacidotic coma*

40
Завантажено з сайту https://тестування.укр/ - онлайн тестування КРОК

B. Hyperosmolar coma
C. Cerebral coma
D. Hepatic coma
E. Acute renal insufficiency

189. A patient consulted a venereologist about painful urination, reddening of the external opening
of urethra, profuse purulent discharges from the urethra. He considers himself to be ill for 3
days. He also associates the disease with a casual sexual contact that took place for about a
week ago. If provisional diagnosis "acute gonorrheal urethritis"will be confirmed, then
bacteriological study of urethral discharges will reveal:
A. Gram-negative diplococci*
B. Gram-positive diplococci
C. Spirochaete
D. Proteus vulgaris
E. Mycoplasma

190. A 30 year old patient complains about inability to become pregnant over 3 years of married
life. The patient is of supernutrition type, she has hair along the median abdominal line, on the
internal thigh surface and in the peripapillary area. Menses started at the age of 16, they are
infrequent and non-profuse. US revealed that the uterus was of normal size, ovaries were
4x5x5 cm large and had a lot of cystic inclusions. What is the most probable diagnosis?
A. Polycystic ovaries*
B. Ovarian cystoma
C. Chronic oophoritis
D. Menstrual irregularity
E. Bilateral ovarian tumours

191. On the second day after preventive vaccination a 2 year old boy got abdominal pain without
clear localization, body temperature rose up to 380C'. On the third day the child got red
papular haemorrhagic eruption on the extensor surfaces of limbs and around the joints. Knee
joints were edematic and slightly painful. Examination of other organs and systems revealed
no pathological changes. What is the most probable diagnosis?
A. Haemorrhagic vesiculitis*
B. Thrombocytopenic purpura
C. Meningococcemia
D. Urticaria
E. DIC syndrome

192. On the 6th day of life a child got multiple vesicles filled with seropurulent fluid in the region of
occiput, neck and buttocks. General condition of the child is normal. What disease should be
suspected?
A. Vesiculopustulosis*
B. Impetigo neonatorum
C. Miliaria
D. Impetigo
E. Epidermolysis bullosa

41
Завантажено з сайту https://тестування.укр/ - онлайн тестування КРОК

193. A 4 month old child was admitted to a surgical department 8 hours after the first attack of
anxiety. The attacks happen every 10 minutes and last for 2-3 minutes, there was also one-time
vomi- ting. Objectively: the child’s condition is grave. Abdomen is soft, palpation reveals a
tumour-like formation in the right iliac area. After rectal examination the doctor’s finger was
stained with blood. What is the most probable diagnosis?
A. Ileocecal invagination*
B. Gastrointestinal haemorrhage
C. Wilm’s tumour
D. Helminthic invasion
E. Pylorostenosis

194. Estimation of physical development of a child involved dynamometry and estimation of body
weight and length, annual gain in body length, chest circumference, number of permanent
teeth, secondary sexual characters, lung vital capacity. Which of the mentioned indices relates
to the physiometric ones?
A. Lung vital capacity, dynamometry*
B. Body length and weight, chest circumference
C. Secondary sexual characters
D. Number of permanent teeth
E. Annual gain in body length

195. During preventive examination a 16 year old patient presented no problems. Objectively: the
patient has signs of malnutrition, he is asthenic, AP is 110/70 mm Hg, Ps is 80 bpm, cardiac
border is normal, auscultation above the cardiac apex reveals three sounds, cardiac murmur is
absent. ECG shows no pathological changes, phonocardiogram shows that the third sound
comes 0,15 s after the second one above the apex. How are these changes called?
A. III physiological sound*
B. Fout-ta-ta-rou (reduplication of the 2nd sound)
C. Protodiastolic gallop rhythm
D. Atrial gallop rhythm
E. IV physiological sound

196. Estimation of community health level involved analysis of a report on diseases registered
among the population of district under charge (reporting form 12). What index is calculated on
the grounds of this report?
A. Common sickness rate*
B. Index of pathological affection
C. Index of morbidity with temporary disability
D. Index of hospitalized morbidity
E. Index of basic non-epidemic morbidity

197. A 37 year old male patient was admitted to the resuscitation department because of attacks of
tonoclonic spasms repeating every half an hour. Between the attacks the patient remains
unconscious. AP is 120/90 mm Hg, Ps- 100 bpm. A day before the patient was at wedding and
consumed alcohol. 5 years ago he had a closed craniocerebral trauma and brain contusion that
later caused single convulsive attacks accompanied by loss of consciousness, but the patient
didn’t undergo antiepileptic treatment. What drug should be injected for emergency aid?

42
Завантажено з сайту https://тестування.укр/ - онлайн тестування КРОК

A. Diazepam*
B. Magnesium sulfate
C. Sodium oxybutyrate
D. Aminazine
E. Sodium thiopental

198. A 43 year old female patient was delivered to the hospital in grave condition. She suffers from
Addison’s disease. The patient had been regularly taking prednisolone but a week before she
stopped taking this drug. Objectively: sopor, skin and visible mucous membranes are
pigmented, skin and muscle turgor is lowered. Heart sounds are muffled, rapid. AP- 60/40 mm
Hg, heart rate - 96/min. In blood: Na-120 millimole/l, K- 5,8 micromole/l. Development of this
complication is primarily caused by the deficit of the following hormone:
A. Cortisol*
B. Corticotropin (ACTH)
C. Adrenaline
D. Noradrenaline
E. Adrostendion

199. A 67 year old female patient complains about edemata of face and legs, pain in the lumbar
area that is getting worse at moving; great weakness, sometimes nasal haemorrhages, rise of
body temperature up to 38,4oC. Objectively: painfulness of vertebral column and ribs on
palpation. Laboratorial study revealed daily proteinuria of 4,2 g, ESR-52 mm/h. What changes
of laboratory indices are to be expected?
A. Whole protein of blood serum -101 g/l*
B. Leukocytes - 15,3 g/l
C. Haemoglobin - 165 g/l
D. Albumins - 65 %
E. 7-globulins -14%

200. A female patient consulted a doctor about gain in weight, chill, edemata, dry skin, sleepiness,
problems with concentration. Objectively: the patient’s height is 165 cm, weight is 90 kg,
gynoid body proportions, to- 35,8oC, ESR-58/min, AP- 105/60 mm Hg. Heart sounds are
weakened, bradycardia is present. Other internal organs have no changes. Thyroid gland is not
palpable. Mammary glands ooze milk droplets. Hormonal study revealed rise of TSH and
prolactin concentration, reduction of T4. What factor caused obesity?
A. Primary hypothyroidism*
B. Secondary hypothyroidism
C. Prolactinoma
D. Hypopituitarism
E. Adiposogenital dystrophy

43
Krok 2 Medicine 2010 1

1. A 40-year-old patient complains A. To continue the worker in office with


of colic pains in the lower abdomen a warning of dismissal in case of repeated
and profuse bloody discharges from the violation of labor discipline
genital tracts. Over the last 2 years she B. To discharge the worker, i.e. to satisfy
has been having menses for 15-16 days, demands of the collective
profuse, with clots, painful. In anamnesis C. To issue the sick list
there are 2 medical abortions. On bi- D. To shift the solution of this problem on
manual investigation: in the canal of the other officials or public organizations
uterine cervix some fibromatous nodes E. -
are palpable, they are 3 cm in diameter,
on the thin crus. Discharges are bloody, 5. 25 unorganized children in the age 2-3
moderate. Choose the correct treatment year will be observed on a pediatric distri-
tactics: ct it in the current year. What scheduled
number of initial visitations will make to
A. Operation: untwisting of the nodes this group of children?
B. Hormonal hemostasis
C. Step-by-step vitamin therapy A. 50
D. Supravaginal ablation of the uterus B. 20
without appendages C. 40
E. Hysterectomy without appendages D. 100
E. 200
2. 13 months after the first labor a 24-year-
old patient complained of amenorrhea. 6. A patient with high temperature came
Pregnancy ended in Caesarian secti- to a first-aid post in the evening. The fact
on because of premature detachment of temporary disability was established.
of normally positioned placenta which Indicate the order of examination in this
resulted in blood loss at the rate of 2000 case:
ml owing to disturbance of blood clotting. A. The night duty doctor should issue
Choose the most suitable investigation: a medical certificate, which will be
A. Estimation of gonadotropin rate subsequently used for issuing a sick list
B. USI of small pelvis organs from the date of the previous day
C. Progesteron assay B. The sick list for 1 day should be issued
D. Computer tomography of head C. The sick list for up to 3 days should be
E. Estimation of testosteron rate in blood issued
serum D. The sick list for 3 days should be issued
E. Any document shouldn’t be issued
3. A 20-year-old patient was delivered to
the hospital in summer from the street wi- 7. An 8-year-old boy fell ill acutely: he
th haemorrage from the brachial artery. presents with fever, weakness, headache,
First medical aid involved application of a abdominal pain, recurrent vomiting, then
tourniquet for provisional arrest of bleedi- diarrhea and tenesmus. Stools occur 12
ng. What is the maximal exposure of the times daily, are scanty, contain a lot of
tourniquet? mucus, pus, streaks of blood. His sigmoid
gut is tender and hardened. What is your
A. 120 minutes diagnosis?
B. 15 minutes
C. 30 minutes A. Dysentery
D. 60 minutes B. Salmonellosis
E. 180 minutes C. Cholera
D. Staphylococcal gastroenteritis
4. Head of a department and a trade- E. Escherichiosis
union group have appealed to the head
of a hospital about dismissal of the seni- 8. An infant was born with body mass 3 kg
or nurse who has 17 year record of servi- and body length 50 cm. Now he is 3 years
ce. The facts of charge were confirmed old. His brother is 7 years old, suffers from
and recognized by the nurse herself. This rheumatic fever. Mother asked the doctor
nurse lives with a daughter (who is di- for a cardiac check up of the 3-year-old
son. Where is the left relative heart border
vorced and unemployed) and a 9-month- located?
old grandson. Make an administrative
decision:
Krok 2 Medicine 2010 2

A. 1 cm left from the left medioclavicular A. Erythrocyte hemolysis


line B. Intravascular hemolysis
B. 1 cm right from the left medioclavicular C. Disturbance of the conjugative function
line of liver
C. Along the left medioclavicular line D. Bile condensing
D. 1 cm left from he left parasternal line E. Mechanical obstruction of the bile
E. 1 cm right from the left parasternal line outflow
9. A 14-year-old boy has rheumati- 13. A 4-month-old girl with blond hair
sm. Over the last 2 years he has had and blue eyes has "mousy"odor of
3 rheumatic attacks. What course of sweat and urine, delayed psychomotoric
rheumatism does the patient have? development. The most typical laboratory
data for this disorder is:
A. Prolonged
B. Acute A. Positive urine ferric chloride test
C. Subacute B. High level of oxyproline in urine
D. Latent C. High level of glycosaminoglycanes in
E. Persistent-reccurent urine
D. High concentration of chlorides in
10. The patient with aquired heart failure sweat
has diastolic pressure of 0 mm Hg. What E. Low level of thyroid gland hormones in
heart failure does the child have? blood
A. Aortal insufficiency 14. A man, aged 25, presents with faci-
B. Mitral stenosis al edema, moderate back pains, body
C. Aortal stenosis temperature of 37, 5o C, BP- 180/100 mm
D. Mitral insufficiency Hg, hematuria (up to 100 in v/f), protei-
E. Rheumatism nuria (2,0 g/l), hyaline casts - 10 in v/f,
11. An 18-month-old child was taken specific gravity - 1020. The onset of the
to a hospital on the 4-th day of the disease is probably connected with acute
disease. The disease began acutely wi- tonsillitis 2 weeks ago. The most likely di-
th temperature 39, weakness, cough, agnosis is:
breathlessness. He is pale, cyanotic, A. Acute glomerulonephritis
has had febrile temperature for over 3 B. Acute pyelonephritis
days. There are crepitative fine bubbli- C. Cancer of the kidney
ng rales on auscultation. Percussi- D. Urolithiasis
on sound is shortened in the right E. Chronic glomerulonephritis
infrascapular region. X-ray picture shows
non-homogeneous segment infiltration 8- 15. A 62-year-old patient has DM-2. Di-
10 mm on the right, the intensification of abetes is being compensated by diet and
lung pattern. Your diagnosis: Maninilum. Pаtient has to undergo an
operation for inguinal hernia. What tacti-
A. Segmentary pneumonia cs of hypoglycemic therapy should be
B. Grippe chosen?
C. Bronchitis
D. Bronchiolitis A. Prescribe the short-acting insulin
E. Interstitial pneumonia B. Give Glurenorm in place of Maninilum
C. To continue with the current therapy
12. A baby was born at 36 weeks D. Prescribe thelong-acting insulin
of gestation. Delivery was normal, by E. Prescribe guanylguanidine
natural way. The baby has a large
cephalohematoma. The results of blood 16. A neonate is 5 days old. What vacci-
count are: Hb- 120g/l, Er- 3, 5 · 1012 /l, total nation dose of BCG vaccine (in мg) is
serum bilirubin - 123 mmol/l, direct bili- necessary for vaccination of this child?
rubin - 11 mmol/l, indirect - 112 mmol/l.
What are causes of hyperbilirubinemia in A. 0,05 мg
this case? B. 0,025 мg
C. 0,075 мg
D. 0,1 мg
E. 0,2 мg
17. Patient, male, 16 years old was behind
Krok 2 Medicine 2010 3

other children in development since early 22. A 20-year-old woman has a 3-4 month
childhood and still has moderate mental history of bloody diarrhoea; stool exami-
retardation. He is short, has dismorphic nation proved negative for ova and parasi-
body, his face is round, flattened, his eyes tes; stool cultures negative for clostridi-
are narrow and slanted, and there are epi- um, campylobacter and yersinia; normal
cantial folds in the corners of his eyes. small bowel series; edema, hyperemia
There is only one transversal flexor line and ulceration of the rectum and sigmoid
on his palms. What is the probable eti- colon seen on sigmoidoscopic examinati-
ology of this state? on. Select the most likely diagnosis:
A. Chromosome abnormality A. Ulcerative colitis
B. Gene abnormality B. Gastroenteritis
C. Maternal alcohol abuse during C. Carcinoid syndrome
pregnancy D. Zollinger-Ellison syndrome
D. Pathological delivery E. Granulomatous colitis
E. Infection in mother during pregnancy
23. A 36-year-old alcoholic patient has ci-
18. A patient who works as a nightman rrhosis and pancreatic insufficiency due
was diagnosed with of chronic arseni- to recurrent pancreatitis. He complains of
ous intoxication. What form of anemia night blindness, decreased ability to taste
is characteristic for this disease? food, and dry skin with hyperpigmentati-
on. These complaints suggest deficiency
A. Haemolytic anemia of:
B. Aplastic anemia
C. Iron deficiency anemia A. Zinc
D. Hyper sideric anemia B. Copper
E. Normochromic anemia C. Selenium
D. Chromium
19. A patient, aged 52, works as a E. Manganese
street cleaner. He has been suffering
from pneumoconiosis for 2 years. What 24. A 60-year-old woman, mother of 6
treatment should be recommended? children, developed a sudden onset of
upper abdominal pain radiating to the
A. Alkaline inhalations back, accompanied by nausea, vomiting,
B. Broncholitics fever and chills. Subsequently, she noti-
C. Oil inhalations ced yellow discoloration of her sclera and
D. Sulfonilamides skin. On physical examination the pati-
E. Antibiotics ent was found to be febrile with temp. of
38, 9oC, along with right upper quadrant
20. A 28-year-old patient was hospitalized tenderness. The most likely diagnosis is:
with preliminary diagnosis "influenza".
Roseolous-petechial rash appeared on A. Choledocholithiasis
the 5th day of disease on the trunk. The B. Benign biliary stricture
temperature is 41oC. Hyperemia of face, C. Malignant biliary stricture
reddening of scleras, tremor of tongue, D. Carcinoma of the head of the pancreas
tachycardia, splenomegaly are present. E. Choledochal cyst
What is the most likely diagnosis?
25. A 30-year-old patient with complai-
A. Epidemic typhus nts of occipital headache, disturbed sleep
B. Measles with nightmares came to a policlinic. BP
C. Alcohol delirium was 150/95 mm Hg. He was diagnosed wi-
D. Leptospirosis th hypertensic crisis. The patient should
E. Typhoid fever be registered in the following dispensary
group for arterial hypertension survei-
21. The diagnostics of the AIDS epidemic llance:
initially was made in the USA by means
of: A. In the second
B. In the first
A. The epidemiological method C. In the fourth
B. The bacteriological method D. In the third
C. The virological method E. In the fifth
D. The viroscopic method
E. The serological method 26. A young patient who came to a poli-
Krok 2 Medicine 2010 4

clinic was diagnosed with the 1 stage rheumatic fever complains of fever up
of hypertension. How often should he to 38 − 39o C, abdominal pain, dyspnea,
undergo the medical check-up? tachycardia. Heart borders are displaced
to the left by 2 cm, systolic and di-
A. Twice a year astolic murmurs above aorta, BP of
B. Once a year 160/30 mm Hg. Petechial rash occurs after
C. 3 times a year measurement of blood pressure. Liver is
D. 4 times a year enlarged by 3 cm, spleen is palpable. Uri-
E. 5 times a year ne is brown-yellow. What is the most likely
diagnosis?
27. A 45-year-old male patient was admi-
tted to the intensive care unit because of A. Infectious endocarditis
myocardial infarction. An hour later the B. Rheumatic fever
ventricular facilitation occurred. Which of C. Acute hepatitis
the following should be administered? D. Acute nephritis
E. Aortic regurgitation
A. Defibrillation
B. External chest compression 32. A 30-year-old patient complains of
C. Lidocaine injection breathlessness, pain in the right rib arc
D. Adrenalin injection region, dry cough and the edema of legs.
E. Cardiac pacing He is ill for 2 months. He had been treated
for rheumatic fever without any effect.
28. A 47-year-old obese man complained On exam: cyanosis, edema of legs, BT of
of periodic attacks of acute arthritis in 36, 6oC, RR of 28/min, HR of 90/min, BP
the st left tarsophalangeal joint. Lab exam of 110/80 mm Hg, crackles above low parts
revealed increased serum rate of uric acid. of both lungs, heart borders are displaced
What is the diagnosis? to the left and to the right, weak sounds,
A. Gout arthritis systolic murmur above the apex. What is
B. Reiter’s disease the preliminary diagnosis?
C. Rheumatoid arthritis A. Dilated cardiomyopathy
D. Rheumatic arthritis B. Infectious endocarditis
E. Osteoarthritis C. Acute myocarditis
29. A 40-year-old woman who has worked D. Rheumatic fever, mitral stenosis
in weaving branch for 10 years complai- E. Acute pericarditis
ns of frequent headache, sleeplessness, 33. A full term infant was born after
irritability, fatigue, tiredness. Physical a normal pregnancy, delivery, however,
examination revealed instability of blood was complicated by marginal placental
pressure, internal organs are without detachment. At 12 hours of age the chi-
changes. What is the most likely di- ld, although appearing to be in good
agnosis? health, passes a bloody meconium stool.
A. Noise-induced disease For determining the cause of the bleedi-
B. Hypertension ng, which of the following diagnostic
C. Atopic bronchial asthma procedures should be performed first?
D. Asthenovegetative syndrome A. Barium enema
E. Encephalopathy B. An Apt test
30. A 45-year-old driver was admitted to C. Gastric lavage with normal saline
the hospital with 5 hour substernal pain. D. An upper gastrointestinal series
Nitroglycerin is not effective. He is pale, E. Platelet count, prothrombin time, and
heart sounds are regular but weak. HR - partial thromboplastin time
96 per minute, BP of 100/60 mm Hg. What 34. A newborn infant has mild cyanosis,
is the most likely diagnosis? diaphoresis, poor peripheral pule,
A. Acute myocardial infarction hepatomegaly and cardiomegaly. Respi-
B. Stable angina ratory rate is 60 breaths per minute, and
C. Pulmonary embolism heart rate is 230 beats per minute. The chi-
D. Acute myocarditis ld most likely has congestive heart failure
E. Acute left ventricular failure caused by:

31. A 33-year-old man with a history of


Krok 2 Medicine 2010 5

A. Paroxysmal atrial tachycardia test data - mild leukocytosis and eosi-


B. A ventricular septal defect and nophilia. What is the diagnosis?
transposition of the great vessels
C. Atrial flutter and partial atrioventricular A. Allergic rhinitis
block B. Foreign body in the nose
D. Hypoplastic left heart syndrome C. Infective rhinitis
E. A large atrial septal defect and valvular D. Atrophic rhinitis
pulmonary stenosis E. Acute sinusitis
35. A 6-year-old boy was brought to the 40. A 27-year old patient with malaria
emergency room with a 3-hour history of caused by P. falciparum was treated wi-
fever up to 39, 5o C and sore throat. The th Chloroquine (600 mg base followed by
child looks alert, anxious and has a mild 300 mg base in 6 hours, then 300 mg base a
inspiratory stridor. You should immedi- day for 2 days) without clinical and parasi-
ately: tologic responses to the treatment. What
is the most likely reason for the failure to
A. Prepare to establish an airway respond to the therapy?
B. Obtain an arterial blood gas and start
an IV line A. Chloroquine resistant strain of P. falci-
C. Order a chest x-ray and lateral view of parum
the neck B. Glucose-6-phosphate dehydrogenase
D. Examine the throat and obtain a culture deficiency in patient
E. Admit the child and place him in a mist C. Late recognition of infection due to P.
tent falciparum
D. Inappropriate route of administration
36. For a patient with a stab injury to the E. Hypersensitivity of the patient to
femoral artery, the treatment of choice is: Chloroquine
A. End- to-end anastomosis 41. Ten hours before initial observation
B. Interposition of autogenous vein graft patient had frequent feces and vomiti-
C. Ligation of both transected ends ng. Fecal and vomiting masses looked
D. Interposition of homologous arterial like rice-water. Nausea and abdomi-
graft nal pain weren’t observed. Hiccup and
E. Interposition of Dacron graft convulsions of lower limbs, temperature
37. Which of the following best summari- 35, 4oC, hoarse voice, greyish wry face,
acrocyanosis were observed. Respiratory
zes indications for operation on an
abdominal aortic aneurysm? rate 40/min, threadlike pulse 120/min,
blood pressure 40/0 mm/Hg, abdomen is
A. Any aneurysm greater than 5 cm in drawn. What treatment measures should
diameter be taken first of all?
B. Any abdominal aortic aneurysm
C. Only symptomatic aneurysm A. Intravenous rehydration
B. Antibiotic therapy
D. Only symptomatic aneurysm greater
C. Cardiac glycosides
than 5 cm in diameter
E. Only ruptured aneurysm D. Disintoxicational therapy
E. Antibotulinic serum injection
38. Which of the following symptoms
42. A nurse of the kindergarten was taken
is the most typical for the adenoid
hypertrophy? to the hospital with complaints of accute
pain in parumbilical region, convulsions
A. Nasal obstruction of lower limbs, multiple bile vomiting,
B. Deglutition disturbance frequent watery foul feces of green colour
C. Olfaction disturbance in huge amounts. At the same time all
D. Giddiness the staff in the kindergarden got ill. Two
E. Headache days ago all of them ate cottage cheese wi-
th sour cream. General condition of pati-
39. A 30-year-old woman complains of ents is of moderate severity. Temperature
nasal blockage, sneezing, watery nasal di- 38, 2oC. Heart tones: rhythmic and muted.
scharges. The body temperature is normal. Heart rate 95/min, arterial pressure: 160
Objectively - edema of nasal mucous mm/Hg. Abdomen is slightly swollen, pai-
membrane on both sides, especially of the nful. Liver +2 cm. What is the most likely
lower turbinate (with cyanosis). Blood diagnosis?
Krok 2 Medicine 2010 6

A. Acute pancreatitis
A. Salmonellosis B. Acute cholecystitis
B. Dysentery C. Acute appendicitis
C. Cholera D. Acute diverticulitis
D. Food toxic infection E. Mesenteric adenitis
E. Enterovirus infection
46. In which of the following di-
43. A 45-year-old woman, mother of four sorders does the pathophysiology of
children, comes to the emergency room portal hypertension involve presinusoidal
complaining of a sudden onset of the epi- intrahepatic obstruction?
gastric and right upper quadrant pain,
radiating to the back, accompanied by A. Congenital hepatic fibrosis
vomiting. On examination, tenderness is B. Alcoholic cirrhosis
elicited in the right upper quadrant, bowel C. Hemochromatosis
sounds are decreased, and laboratory data D. Budd-Chiari syndrome
shows leukocytosis, normal serum levels E. Cavernomatous transformation of the
of amylase, lipase, and bilirubin. The most portal vein
likely diagnosis is:
47. A 25-year-old man was admitted to a
A. Acute cholecystitis hospital with a 2 month history of cough
B. Perforated peptic ulcer disease and fever. A chest x-ray showed extensive
C. Myocardial infarction left upper lobe disease with a 2 cm cavi-
D. Sigmoid diverticulitis ty. All three points were strongly positive
E. Acute pancreatitis on direct smear and grew M. Tuberculosis,
fully sensitive to all first-line drugs. Pati-
44. A 50-year-old man comes to the ent must be treated with:
emergency room with a history of vomiti-
ng of 3 days’ duration. His past history A. Isoniazid + rifampicin + pyrazinamide
examination reveals that for about 20 B. Streptomycin + isoniazid
years he has been suffering from epigasric C. Isoniazid + ethambutol
pain lasting for 2 to 3 weeks, during early D. Kanamycin + ethambutol + pyrazinami-
spring and autumn. He remembers getti- de
ng relief from pain by taking milk and E. P-aminosalicylic acid + streptomycin
antacids. Physical examination showed a
fullness in the epigastric area with visi- 48. A 5-year-old boy was progressively
ble peristalsis, absence of tenderness, and getting worse compared to the previous
normal active bowel sounds. The most li- 2 months. A chest x-ray has shown ri-
kely diagnosis is: ght middle lobe collapse. A tuberculin
skin test was strongly positive. What is
A. Gastric outlet obstruction the most characteristic finding in primary
B. Small bowel obstruction tuberculosis?
C. Volvulus of the colon
D. Incarcerated umbilical hernia A. Hilar or paratracheal lymph node
E. Cholecystitis enlargement
B. Atelectasis with obstructive pneumonia
45. A 24-year-old law student is brought C. Cavity formation
to the emergency room complaining of D. Miliary tuberculosis
severe abdominal pain of 6-8 hours durati- E. Hematogenous dissemination leading
on. He had been to a party the night to extrapulmonary tuberculosis
before. The pain is in the epigastrium
radiating to the back and is accompani- 49. A 50-year-old man was examined in
ed by nausea. The patient had vomited the clinic for persistent cavitation and
twice prior to coming to the emergency sputum. He was treated with rifampi-
room. Clinical examination revealed that cin, isoniazid, ethambuthol. The most
the young man was anxious, with acute common toxic effect of ethambutol is:
condition, with a regular pulse rate of A. Optic neuritis
100/min, blood pressure of 100/68 mm B. Eighth cranial nerve damage
Hg, and body temperature of 38, 1oC. The C. Hepatic enzyme elevation
most likely diagnosis is: D. Peripheral neuropathy
E. Mental symptoms
50. A 3-year-old child has been admi-
Krok 2 Medicine 2010 7

tted to a hospital because of ostealgia on the left from the breastbone in the
and body temperature rise up to 39oC. 3-4 intercostal space. This murmur is
Objectively: the patient is in grave condi- conducted above the whole cardiac regi-
tion, unable to stand for ostealgia, there on to the back. What congenital cardiac
is apparent intoxication, lymph nodesare pathology can be suspected?
enlarged up to 1,5 cm. Liver can be
palpated 3 cm below the costal margin, A. Defect of interventricular septum
spleen - 2 cm below the costal margin. In B. Defect of interatrial septum
blood: RBCs - 3, 0·1012 /l, Hb- 87 g/l, colour C. Coarctation of aorta
index - 0,9, thrombocytes - 190 · 109 /l, D. Fallot’s tetrad
E. Pulmonary artery stenosis
WBCs - 3, 2 · 109 /l, eosinophils - 1, stab
neutrophils - 1, segmented neutrophils - 54. A 27-year-old patient with a history
0, lymphocytes - 87, monocytes - 2, ESR of ronchial asthma was stung by a bee.
- 36 mm/h. What examination should He had a sensation of chest compression,
be conducted in order to specify the di- breath shortage, difficult expiration, sense
agnosis? of heat in the upper half of body, dizziness,
apparent itch, convulsions. Objectively:
A. Sternal puncture noisy wheezing breath, AP - 90/60 mm
B. Ultrasound Hg, Ps- 110 bpm. Auscultation revealed
C. Lymph node puncture weak rhythmic heart sounds, rough respi-
D. Lymph node biopsy ration above lungs, sibilant rales. What
E. Computer tomography drug group should be administered in the
51. A 22-year-old girl has been complai- first place?
ning of having itching rash on her face A. Glucocorticoids
for 2 days. She associates this disease wi- B. Methylxanthines
th application of cosmetic face cream. C. Cardiac glycosides
Objectively: apparent reddening and D. Anticonvulsive
edema of skin in the region of cheeks, E. Analgetics
chin and forehead; fine papulovesicular
rash. What is the most likely diagnosis? 55. A 32-year-old patient complains of
cardiac irregularities, dizziness, dyspnea
A. Allergic dermatitis at physical stress. He has never suffered
B. Dermatitis simplex from this before. Objectively: Ps- 74 bpm,
C. Eczema rhythmic. AP- 130/80 mm Hg. Auscultati-
D. Erysipelas on revealed systolic murmur above aorta,
E. Neurodermatitis the first heart sound was normal. ECG
52. Apgar test done on a newborn gi- showed hypertrophy of the left ventri-
rl at 1st and 5th minute after birth gave cle, signs of repolarization disturbance in
the result of 7-8 scores. During the deli- the I, V5 and V6 leads. Echocardiogram
very there was a short-term difficulty wi- revealed that interventricular septum was
th extraction of shoulder girdle. After bi- 2 cm. What is the most likely diagnosis?
rth the child had the proximal extremity A. Hypertrophic cardiomyopathy
dysfunction and the arm couldn’t be raised B. Aortic stenosis
from the side. The shoulder was turned C. Essential hypertension
inwards, the elbow was flexed, there was D. Myocardium infarction
also forearm pronation, obstetric palsy of E. Coarctation of aorta
brachial plexus. What is the clinical di-
agnosis? 56. On the 21 day after appearance of vesi-
culous chickenpox rash a 7-year-old chi-
A. Duchenne-Erb palsy ld developed ataxia, nystagmus, intenti-
B. Trauma of thoracic spine on tremor, muscle hypotonia. Liquor
C. Right hand osteomyelitis analysis shows insignificant lymphocytic
D. Intracranial haemorrhage pleocytosis, slightly increased protein rate.
E. Trauma of right hand soft tissues What complication is it?
53. Examination of a 9-month-old girl
revealed skin pallor, cyanosis during exci-
tement. Percussion revealed transverse
dilatation of cardiac borders. Auscultati-
on revealed continuous systolic murmur
Krok 2 Medicine 2010 8

A. Encephalitis 60. A painter working at a motorcar


B. Purulent meningitis plant was diagnosed with acute intoxi-
C. Pneumonitis cation with amide compounds of benzene,
D. Acute nephritis moderate severity grade. The in-patient
E. Postherpetic neuralgia treatment resulted in considerable health
improvement. What expert decision
57. Clinic of a research instutute for should be made in this case?
occupational diseases examined a worker
who works at a concentration plant and A. The patient should be issued a sick list
diagnosed him with chronic dust bronchi- for out-patient treatment
tis. The case is investigated by a commi- B. The patient may get back to work
ssion including the representatives of: the providing he will keep to hygiene and
plant, clinic, territorial SES, department sanitary regulations
of Social Insurance Fund, trade union. C. The patient should be referred to the
According to the "regulation on investi- Medical and Social Expert Commission
gation of. . . ", the commission should for assigning the disability group because
be headed by the representative of the of an occupational disease
following authority: D. The patient should be referred to the
Medical and Social Expert Commission
A. Territorial SES for determination of percentage of work
B. Plant capicty loss
C. Social Insurance Fund E. -
D. Trade union
E. Clinic 61. A maternity hospital registered 616 li-
ve births, 1 stillbirth, 1 death on the 5th
58. Basing upon the data of laboratory day of life over a 1 year period. What
assessment of sanitary state of soil in index allows the most precise estimation
a certain territory, the soil was found of this situation?
to be low-contaminated according to
the sanitary indicative value; contami- A. Perinatal mortality
nated according to the coli titer; low- B. Crude mortality rate
contaminated according to the anaerobe C. Natality
titer (Cl. Perfringens). This is indicative of: D. Neonatal mortality
E. Natural increase
A. Fresh fecal contamination
B. Insufficient intensity of soil humification 62. A 44-year-old patient complai-
C. Old fecal contamination ns about difficult urination, sensati-
D. Constant entry of organic protein on of incomplete urinary bladder
contaminations emptying. Sonographic examination of
E. Insufficient insolation and aeration of the urinary bladder near the urethra
soil entrance revealed an oval well-defined
hyperechogenic formation 2x3 cm large
59. A 28-year-old patient underwent that was changing its position during the
endometrectomy as a result of incomplete examination. What conclusion can be
abortion. Blood loss was at the rate made?
of 900 ml. It was necessary to start
hemotransfusion. After transfusion of A. Concrement
60 ml of erythrocytic mass the patient B. Malignant tumour of the urinary
presented with lumbar pain and fever whi- bladder
ch resulted in hemotransfusion stoppage. C. Urinary bladder polyp
20 minutes later the patient’s conditi- D. Prostate adenoma
on got worse: she developed adynamia, E. Primary ureter tumour
apparent skin pallor, acrocyanosis,
profuse perspiration. to - 38, 5oC, Ps- 63. An emergency team has delivered to
110/min, AP- 70/40 mm Hg. What is the a hospital an unconscious patient found
most likely diagnosis? lying in the street in winter. Objectively:
the patient is pale, with superficial respi-
A. Hemotransfusion shock ration; bradycardia with heartrate 54/min,
B. Hemorrhagic shock to - 35, 0oC. AP- 100/60 mm Hg. Palpati-
C. Septic shock on of chest and abdomen revealed no
D. Anaphylactic shock peritoneal symptoms. There is a smell of
E. DIC syndrome alcohol from the patient’s mouth. What is
the most likely diagnosis?
Krok 2 Medicine 2010 9

A. Up to 4500
A. Hypothermia B. Up to 2500
B. Acute cardiovascular insufficiency C. Up to 3500
C. Apparent death D. Up to 5500
D. Frostbite of trunk and extremities E. Up to 7500
E. -
68. A patient who has been consumi-
64. A 28-year-old parturient complai- ng refined foodstuffs for a long time
ns about headache, vision impairment, complains about headache, fatiguability,
psychic inhibition. Objectively: AP- depression, insomnia, irritability. Objecti-
200/110 mm Hg, evident edemata of vely: muscle asthenia, pain and cramps in
legs and anterior abdominal wall. Fetus the gastrocnemius muscles, during walki-
head is in the area of small pelvis. Fetal ng the patient lands onto his heel first,
heartbeats is clear, rhythmic, 190/min. then on the external edge of foot. Cardi-
Internal examination revealed complete ovascular system exhibits tachycardia,
cervical dilatation, fetus head was in the hypoxia, dystrophic changes of myocardi-
area of small pelvis. What tactics of labor um. There are also gastrointestinal di-
management should be chosen? sorders. What is the most likely diagnosis?
A. Forceps operation A. Hypovitaminosis B1
B. Cesarean B. Hypovitaminosis B2
C. Embryotomy C. Hypovitaminosis B12
D. Conservative labor management with D. Hypovitaminosis B6
episiotomy E. Hypovitaminosis B15
E. Stimulation of labor activity
69. A 9-year-old boy has been sufferi-
65. A 35-year-old patient complains about ng from bronchoectasis since he was 3.
pain and morning stiffness of hand joi- Exacerbations occur quite often, 3-4 ti-
nts and temporomandibular joints that mes a year. Conservative therapy results
lasts over 30 minutes. She has had these in short periods of remission. The di-
symptoms for 2 years. Objectively: edema sease is progressing, the child has physi-
of proximal interphalangeal digital joi- cal retardation. The child’s skin is pale,
nts and limited motions of joints. What acrocyanotic, he has "watch glass"nail
examination should be administered? deformation. Bronchography revealed
saccular bronchiectases of the lower lobe
A. Roentgenography of hands of his right lung. What is the further
B. Complete blood count treatment tactics?
C. Rose-Waaler reaction
D. Immunogram A. Surgical treatment
E. Proteinogram B. Further conservative therapy
C. Physiotherapeutic treatment
66. A 68-year-old female patient complai- D. Sanatorium-and-spa treatment
ns about temperature rise up to 38, 3oC, E. Tempering of the child’s organism
haematuria. ESR- 55 mm/h. Antibacterial
therapy turned out to be ineffective. What 70. A 46-year-old patient once took
diagnosis might be suspected? part in elimination of breakdown at an
atomic power plant. Currently he is bei-
A. Renal cancer ng treated at an in-patient hospital. He
B. Polycystic renal disease was diagnosed with progressing vegetati-
C. Renal amyloidosis ve insufficiency. This disease relates to
D. Urolithiasis the following group of ionizing radiation
E. Chronic glomerulonephritis effects:
67. Bacterial analysis of air in a livi- A. Somato-stochastic
ng space in winter period by means of B. Somatic
Krotov’s apparatus revealed that total C. Genetic
number of microorganisms in 1m3 of air D. Hormesis
was 7200. What is the allowed number E. Heterosis
of microorganisms for the air to be
characterized as "pure"? 71. A child is 4 years old, has been
ill for 5 days. There are complaints of
cough, skin rash, to - 38, 2oC, face puffiness,
photophobia, conjunctivitis. Objectively:
Krok 2 Medicine 2010 10

there is bright, maculo-papulous, in some A. Physical


areas confluent rash on the face, neck, B. Chemical
upper chest. The pharynx is hyperemic. C. Biological
There are seropurulent discharges from D. Combined
the nose. Auscultation revealed dry rales E. Physicochemical
in lungs. What is the most likely di-
agnosis? 75. In autumn a 25-year-old patient
developed stomach ache that arose 1,5-2
A. Measles hours after having meals and at night. He
B. Adenoviral infection complains about pyrosis and constipation.
C. Scarlet fever The pain is getting worse after consumi-
D. Rubella ng spicy, salty and sour food, it can be
E. Enterovirus exanthema relieved by means of soda and hot-water
bag. The patient has been suffering from
72. A female patient has been sufferi- this disease for a year. Objectively: furred
ng from pain in the right subcostal area, moist tongue. Abdomen palpation reveals
bitter taste in the mouth, periodical bile epigastrial pain on the right, resistance
vomiting for a month. The patient put off of abdominal muscles in the same region.
12 kg. Body temperature in the evening What is the most likely diagnosis?
is 37, 6o C. Sonography revealed that bi-
le bladder was 5,5х2,7 cm large, its wall - A. Duodenal ulcer
0,4 cm, choledochus - 0,8 cm in diameter. B. Chronic cholecystitis
Anterior liver segment contains a roundi- C. Diaphragmatic hernia
sh hypoechoic formation up to 5 cm in D. Stomach ulcer
diameter and another two up to 1,5 cm E. Chronic pancreatitis
each, walls of these formations are up to
0,3 cm thick. What is the most likely di- 76. On the 4th day after suturing the
agnosis? perforative stomach ulcer a patient wi-
th pulmonary emphysema developed
A. Alveolar echinococcus of liver spontaneous pneumothorax. What is the
B. Liver cancer best place for pleural drainage?
C. Liver abscess
D. Cystous liver cancer A. The second intercostal space along the
E. Paravesical liver abscesses medioclavicular line
B. The eighth intercostal space along the
73. A 46-year-old patient is to be prepared posterior axillary furrow
to the operation on account of stomach C. The seventh intercostal space along the
cancer. Preoperative preparation involves anterior axillary furrow
infusion therapy. It was injected up to 3,0 D. The sixth intercostal space along the
l of solutions into his right lunar vein. On anterior axillary furrow
the next day he got tensive pain in the E. The fifth intercostal space along the
region of his right shoulder. Examination medioclavicular line
of interior brachial surface revealed a stri-
pe of hyperemia, skin edema and a painful 77. A 32-year-old male patient has been
cord. What complication is it? suffering from pain in the sacrum and
coxofemoral joints, painfulness and sti-
A. Acute thrombophlebitis ffness in the lumbar spine for a year. ESR
B. Vein puncture and edema of paravenous - 56 mm/h. Roentgenography revealed
cellular tissue symptoms of bilateral sacroileitis. The
C. Necrosis of paravenous cellular tissue patient is the carrier of HLA B27 anti-
D. Acute lymphangitis gen. What is the most likely diagnosis?
E. Phlegmon of paravenous cellular tissue
A. Ankylosing spondylitis
74. A children’s health camp received a B. Coxarthrosis
party of tinned food. External examinati- C. Rheumatoid arthritis
on of the tins revealed that they had deep D. Reiter’s disease
dents; formed a concavity when pressed E. Spondylosis
and didn’t immediately return to the ini-
tial state; rust was absent; the tins were 78. A 58-year-old female patient complai-
greased with inedible fat. Specify the bloat ns about periodical headache, dizziness
type: and ear noise. She has been suffering from
diabetes mellitus for 15 years. Objecti-
vely: heart sounds are rhythmic, heart rate
Krok 2 Medicine 2010 11

is 76/min, there is diastolic shock above A. Acute glomerulonephritis with nephri-


aorta, AP is 180/110 mm Hg. In urine: tic syndrome
OD- 1,014. Daily loss of protein with uri- B. Acute glomerulonephritis with
ne is 1,5 g. What drug should be chosen nephrotic syndrome
for treatment of arterial hypertension? C. Acute glomerulonephritis with
nephrotic syndrome, hematuria and
A. Ihibitor of angiotensin converting hypertension
enzyme D. Acute glomerulonephritis with isolated
B. β-blocker urinary syndrome
C. Calcium channel antagonist E. Nephrolithiasis
D. Thiazide diuretic
E. α-blocker 82. A 60-year-old female patient was
admitted to a hospital for acute
79. A 50-year-old locksmith was di- transmural infarction. An hour ago
agnosed with typhoid fever. The patient the patient’s contition got worse. She
lives in a separate apartment with all faci- developed progressing dyspnea, dry
lities. Apart of him there are also 2 adults cough. Respiratory rate - 30/min, heart
in his family. What actions should be taken rate - 130/min, AP- 90/60 mm Hg. Heart
about persons communicating with the sounds are muffled, diastolic shock on the
patient? pulmonary artery. There are medium moi-
st rales in the lower parts of lungs on the
A. Bacteriological study right and on the left. Body temperature -
B. Antibiotic prophylaxis 36, 4oC. What drug should be given in the
C. Isolation first place?
D. Dispensary surveillance
E. Vaccination A. Promedol
B. Aminophylline
80. A child was taken to a hospital wi- C. Dopamine
th focal changes in the skin folds. The D. Heparin
child was anxious during examination, E. Digoxin
examination revealed dry skin with soli-
tary papulous elements and ill-defined 83. A 52-year-old male patient complains
lichenification zones. Skin eruption was about attacks of asphyxia, pain in his ri-
accompanied by strong itch. The child ght side during respiration. These mani-
usually feels better in summer, his condi- festations turned up all of a sudden. It is
tion is getting worse in winter. The child known from his anamnesis that he had
has been artificially fed since he was 2 been treated for thrombophlebitis of the
months old. He has a history of exudative right leg for the last month. In the admissi-
diathesis. Grandmother by his mother’s si- on ward the patient suddenly lost consci-
de has bronchial asthma. What is the most ousness, there was a sudden attack of
likely diagnosis? asphyxia and pain in his side. Objecti-
vely: heart rate - 102/min, respiratory rate
A. Atopic dermatitis - 28/min, AP- 90/70 mm Hg. Auscultati-
B. Contact dermatitis on revealed diastolic shock above the
C. Seborrheal eczema pulmonary artery, gallop rhythm, small
D. Strophulus bubbling rales above the lungs under the
E. Urticaria scapula on the right, pleural friction rub.
What examination method will be the
81. 2 weeks after recovering from angi- most informative for a diagnosis?
na an 8-year-old boy developed edemata
of face and lower limbs. Objectively: the A. Angiography of pulmonary vessels
patient is in grave condition, AP- 120/80 B. Echocardioscopy
mm Hg. Urine is of dark brown colour. C. Study of external respiration function
Oliguria is present. On urine analysis: D. ECG
relative density - 1,015, protein - 1,2 g/l, E. Coagulogram
RBCs are leached and cover the whole
vision field, granular casts - 1-2 in the vi- 84. A primagravida in her 20th week
sion field, salts are represented by urates of gestation complains about pain in
(big number). What is the most likely di- her lower abdomen, blood smears from
agnosis? the genital tracts. The uterus has an
increased tonus, the patient feels the
fetus movements. Bimanual examination
Krok 2 Medicine 2010 12

revealed that the uterus size corresponded was regarded as a pubertal manifestati-
the term of gestation, the uterine cervix on, the girl didn’t undergo any treatment.
was contracted down to 0,5 cm, the The girl’s irritability gradually gave place
external orifice was open by 2 cm. The to a complete apathy. The girl got puffy
discharges were bloody and smeary. What face, soft tissues pastosity, bradycardia,
is the most likely diagnosis? constipations. Skin pallor and gland densi-
ty progressed, the skin got a waxen hue.
A. Incipient abortion What disease may be assumed?
B. Risk of abortion
C. Abortion in progress A. Autoimmune thyroiditis
D. Incomplete abortion B. Diffuse toxic goiter
E. Missed miscarriage C. Thyroid carcinoma
D. Subacute thyroiditis
85. On the 4th day after recovering from E. Juvenile basophilism
a cold a patient was hospitalized with
complaints of solitary spittings of mucoid 88. A newborn’s head is of doli-
sputum. On the 2nd day there was a si- chocephalic shape, that is front-to-back
ngle discharge of about 250 ml of purulent elongated. Examination of the occipi-
blood-streaked sputum. Objectively: the tal region of head revealed a labour
patient’s condition is moderately severe. tumour located in the middle between the
Respiratory rate - 28-30/min, Ps- 96 bpm, prefontanel and posterior fontanel. The
AP- 110/70 mm Hg. Respiration above the delivery tok place with the following type
left lung is vesicular, weak above the right of fetus head presentation:
lung. There are moist rales of different
types above the lower lobe and amphoric A. Posterior vertex presentation
breath near the angle of scapula. What is B. Anterior vertex presentation
the most likely diagnosis? C. Presentation of the bregma
D. Brow presentation
A. Acute pulmonary abscess E. Face presentation
B. Exudative pleuritis
C. Acute focal pneumonia 89. A 56-year-old patient with diffuse
D. Pleural empyema toxic goiter has ciliary arrhythmia with
E. Pyopneumothorax pulse rate 110 bpm, arterial hypertensi-
on, AP- 165/90 mm Hg. What preparati-
86. 350 workers of a mettalurgical plant on should be administered along with
had to undergo a yearly preventive exami- mercazolil?
nation. A territorial polyclinic carried out
preventive examination of 325 workers. A. Propranolol
As a result of it, 1 worker was recogni- B. Radioactive iodine
zed as temporarily disabled, 15 workers C. Procaine hydrochloride
underwent further rehabilitation at an D. Verapamil
after-work sanatorium, 10 workers were E. Corinfar
provided with diet meal. What index
characterizing the preventive work of the 90. Over a current year among workers of
polyclinic should be applied in this case? an institution 10% haven’t been ill a single
time, 30% have been ill once, 15% - twice,
A. Coverage of preventive medical exami- 5% - 4 times, the rest - 5 and more times.
nations What is the percentage of workers relati-
B. Frequency of case detection during ng to the I health group?
examinations
C. Percentage of people who underwent A. 55%
rehabilitation at an after-work sanatorium B. 10%
D. Percentage of people who were provi- C. 40%
ded with diet meal D. 60%
E. Percentage of temporarily disabled E. 22%
people 91. A 16-year-old boy was admitted to the
87. A 14-year-old girl has been presenti- hospital for the reason of intractable nasal
ng with irritability and tearfulness for haemorrhage and intolerable pain in the
about a year. A year ago she was also right cubital articulation. Objectively: the
found to have diffuse enlargement of the affected articulation is enlarged and exhi-
thyroid gland (II grade). This condition bits defiguration and skin hyperaemia.
There are manifestations of arthropathy
Krok 2 Medicine 2010 13

in the other articulations. Ps- 90 bpm; A. Insulinemia on an empty stomach


colour index - 1,0, WBC - 5, 6 · 109 /l, B. Daily glycemia variability
thrombocytes- 220 · 109 /l, ESR - 6 mm/h. C. Daily glucosuria variability
Lee-White coagulation time: start - 24’, fi- D. Glycemia test an hour after taking
nish - 27’10”. What drug will be the most meals
effective for this patient treatment? E. Glucose tolerance test

A. Cryoprecipitate 95. A 52-year-old male patient has an 18


B. Calcium chloride year history of diabetes mellitus. One year
C. Erythromass ago he had cystitis. The patient takes 0,005
D. Aminocapronic acid g of maninil thrice a day. Objectively: hei-
E. Vicasol ght - 176 cm, weight - 82 kg. Glycemia
variability on an empy stomach is at the
92. A 42-year-old woman complains about rate of 10,3-12,4 millimole/l. Analyses
bruises on her both legs and prolonged revealed proteinuria at the rate of 0,033
menstruation; general weakness, tinni- g/l. The most efficient way to prevent di-
tus cerebri. Objectively: multiple macular abetic nephropathy progress will be:
haemorrhages on the legs and body.
The patient presents with tachypnoe, A. To replace maninil with insulin
tachycardia, systolic murmur in all B. To increase maninil dosage
auscultatory points. AP- 75/50 mm Hg. C. To decrease daily caloric content
Blood count: RBC - 1, 9 · 1012 /l, Нb- 60 D. To supplement the present therapy with
g/l, colour index - 0,9, WBC - 6, 5 · 109 /l, insulin
thrombocytes - 20 · 109 /l, ESR- 12 mm/h. E. To administer antibacterial therapy
Duke bleeding time - 12 minutes. Bone 96. A 34-year-old female patient complai-
marrow analysis revealed plenty of juveni- ns about weakness, 12 kg weight loss
le immature forms of megacaryocytes within 6 months, sweating, palpitation,
without signs of thrombocyte pinch-off. irritability. Objectively: III grade thyroid
What is the most likely diagnosis? gland is elastic, diffuse enlargement is
A. True thrombocytopenic purpura present, there is also a node in the ri-
B. Type A haemophilia ght lobe. Cervical lymph nodes are not
C. Willebrand’s disease enlarged. What treatment tactics would
D. Acute megacaryoblastic leukemia be the most rational?
E. Tupe B haemophilia A. Operation after antithyroid therapy
93. A 43-year-old man who often contacts B. Radioactive iodine administration
with ethyl gasoline was admitted to C. Immediate surgical intervention
a hospital with complaints of general D. Conservative antithyroid therapy
weakness, dizziness, memory impairment, E. Immediate gamma-ray teletherapy
sleepiness at daytime and insomnia at ni- 97. On the next day after being taken by
ght, sense of hair in the mouth, colicky influenza a 46-year-old woman presented
pains in the right subcostal region. What with intensified headache, dizziness,
is the most likely diagnosis? nausea. Objectively: the patient is consci-
A. Chronic tetraethyl lead intoxication ous, psychomotor excitement is present;
B. Alcoholic delirium there is general hyperesthesia, moderate
C. Chronic mercury intoxication meningeal syndrome, nystagmus. Tendon
D. Chronic manganese intoxication teflexes are higher on the right, right
E. Chronic lead intoxication extremities display muscle weakness,
right-sided pathological Babinski’s sign is
94. A 30-year old woman taken by present. Liquor is transparent, pressure is
influenza has empty stomach glycemia at 220 mm of water column; cytosis is 46/3
the rate of 11,3 millimole/l, glucosuria at with prevailing lymphocytes. What is the
the rate of 25 g/l. The patient is 168 cm tall most likely diagnosis?
and weighs 67 kg. What test would be the
most informative for the diagnosis specifi- A. Influenzal meningoencephalitis
cation? B. Bacterial meningoencephalitis
C. Subarachnoidal haemorrhage
D. Parenchymatous subarachnoidal
haemorrhage
E. Ischemic stroke
Krok 2 Medicine 2010 14

98. After examination a 46-year-old pati- Over the last week the attacks occurred
ent was diagnosed with left breast cancer daily and became more painful. On the
T2N2M0, cl. gr. II-a. What will be the 3rd day of hospitalization the patient
treatment plan for this patient? presented with icteritiousness of skin and
scleras, light-colored feces and dark uri-
A. Radiation therapy + operation + ne. In blood: neutrophilic leukocytosis -
chemotherapy 13, 1 · 109 /l, ESR - 28 mm/h. What is the
B. Operation only most likely diagnosis?
C. Operation + radiation therapy
D. Radiation therapy only A. Chronic calculous cholecystitis
E. Chemotherapy only B. Chronic recurrent pancreatitis
C. Fatty degeneration of liver
99. A 34-year-old male patient was deli- D. Chronic cholangitis, exacerbation stage
vered to the neurological department wi- E. Hypertensive dyskinesia of gallbladder
th complaints of intense headache, double
vision, light and noise intolerance. The 103. A 30-year-old gravida consulted a
attack came suddenly while load lifting. gynecologist about bright red bloody di-
Objectively: torpor, moderate divergent scharges from the vagina in the 32 week of
strabismus, diplonia. Bilateral Kernig’s gestation. She was hospitalized with suspi-
symptoms. No paresises. Bloody liquor. cion of placental presentation. Under
What medication should be administered what conditions is it rational to conduct
in the first line? the internal examination for the diagnosis
specification?
A. Epsilon-aminocapronic acid
B. Acetylsalicilic acid A. In the operating room prepared for the
C. Heparin operation
D. Nicotinic acid B. In the examination room of antenatal
E. Glutamic acid clinic
C. In the admission ward of maternity
100. A patient suffering from acute hospital
posttraumatic pain received an injection D. In the delivery room keeping to all the
of morphine that brought him a significant aseptics regulations
relief. Which of the following mechani- E. The examination is not to be conducted
sms of action provided antishock effect of because of risk of profuse haemorrhage
morphine in this patient?
104. A 72-year-old male patient complai-
A. Stimulation of opiate receptors ns about itch in his left shin, especially
B. Block of central cholinergic receptors around a trophic ulcer. Skin is reddened
C. Stimulation of benzodiazepine receptors and edematic, there are some oozing lesi-
D. Inhibition of dopamine mediation ons, single yellowish crusts. The focus of
E. Intensification of GABA-ergic reactions affection is well-defined. What is the most
101. After supercooling a 38-year- likely diagnosis?
old woman developed muscle pain, A. Microbial eczema
body temperature rise up to 39oC, B. Allergic dermatitis
headache, dysuria, positive Pasternatsky’s C. Seborrheic eczema
symptome. In the urine: leukocyturia, D. Cutaneous tuberculosis
bacteriuria. In blood: Decrease in Hb rate E. Streptococcal impetigo
down to 103 g/l, left shift leukocytosis,
ESR acceleration up to 32 mm/h. Blood 105. A 30-year-old patient presented with
urea - 6,0 millimole/l. What is the most li- body temperature rise up to 38, 5o C, pain
kely diagnosis? in the small articulations of hands; face
edemata and erythema. In blood: RBCs -
A. Acute pyelonephritis 2, 6 · 1012 /l; Hb- 98 г/л; WBCs - 2 · 109 /l;
B. Renal tuberclosis ESR - 58 mm/h. In the urine: protein - 3,1
C. Acute glomerulonephritis g/l; RBCs - 10-15 in the vision field. What
D. Urolithiasis disease can be suspected in this case?
E. Acute cystitis
102. A 50-year-old patient complains
about having pain attacks in the right
subcostal area for about a year. He pain
arises mainly after taking fattening food.
Krok 2 Medicine 2010 15

A. Systemic lupus erythematosus A. Neuroleptic therapy


B. Sepsis B. Vitamin therapy
C. Systemic scleroderma C. Physiotherapy
D. Periarteritis nodosa D. Psychotherapy
E. Acute glomerulonephritis E. Reflex therapy
106. A 5-grade pupil complains about 110. A patient has got acute macrofocal
extensive skin rash accompanied by myocardial infarction complicated by
intensive itch, especially at night. Objecti- cardiogenic shock. The latter is progressi-
vely: there are small red papules set ng under conditions of weak general peri-
mostly in pairs in the region of interdigital pheric resistance and decreased cardiac
folds on both hands, on the flexor surface output. What antihypotensive drug should
of radicarpal articulations, abdomen and be injected to the patient in the first place?
buttock skin as well as internal surface
of thighs. In the centre of some papules A. Dopamine
vesicles or serohaemorrhagic crusts can B. Noradrenaline
be seen. There are multiple excoriations. C. Adrenaline
What is the most likely diagnosis? D. Mesatonum
E. Prednisolone
A. Scabies
B. Dermatitis 111. A patient came to the traumatology
C. Ringworm of body centre and complained about a trauma
D. Toxicoderma of the lower third of the volar forearm
E. Eczema surface caused by cut on a piece of glass.
Objectively: flexion of the IV and V fi-
107. A 43-year-old female patiet complai- ngers is impaired, sensitivity of the interi-
ns of eruption on her right leg skin, pain, or dorsal and palmar surface of hand as
weakness, body temperature rise up to well as of the IV finger is reduced. What
38o C. The disease is acute. Objectively: nerve is damaged?
there is an edema on the right leg skin in
the region of foot, a well-defined bright A. Ulnar
red spot in form of flame tips which feels B. Radial
hot. There are isolated vesicles in focus. C. Median
What is your provisional diagnosis? D. Musculoskeletal
E. Axillary
A. Erysipelas
B. Microbial eczema 112. A patient was delivered to the
C. Contact dermatitis admission ward 40 minutes after a
D. Toxicoderma minor trauma with complaints of
E. Haemorrhagic vasculitis pain in the thoracic spine. He has
a history of pulmonary tuberculosis.
108. Surgical department admitted a 37- Roentgenography results: suspected
year-old patient with a big crushed wound compression fracture of the VIII and
of his left thigh 4 hours after he got IX thoracic vertebrae. What pathologi-
this trauma. What is the main provisi- cal process in spine might have stimulated
on for successful prevention of gaseous pathological fracture of vertebral bodies?
gangrene?
A. Tuberculous spondylitis
A. Removal of necrotic tissues and timely B. Syphilis
surgical processing of the wound C. Malignant tumour metastasis
B. Injection of specific serum 3 000 U D. Osteomyelitis
C. Injection of specific serum 30 000 U E. Traumatic fracture
D. Infiltration of soft tissues around the
wound with antibiotic solution 113. On the next day after esophagoscopy
E. Wound lavage with 6% solution of and biopsy a 44-year-old female patient
hydrogen peroxide developed emphysema of both supraclavi-
cular regions, deglutition pain and pain
109. A 35-year-old patient was admitted to behind the breastbone, cyanosis. Body
the in-patient psychiatric facility for the fi- temperature - 39o C. In blood: WBCs -
rst time. He was diagnosed with paranoid 16 · 109 /l, left shift in the neutrophils. X-
schizophrenia. What is the most rational ray picture shows a limited shadow of
treatment method? the posterior mediastinum at a level wi-
th trachea bifurcation. The patient was
Krok 2 Medicine 2010 16

diagnosed with esophagus rupture. What breast. Objectively: in the upper internal
treatment tactics would be the most rati- quadrant of the left breast there is a
onal? neoplasm up to 2,5 cm in diameter, dense,
uneven, painless on palpation. Regional
A. Surgical: mediastinum drainage, gastric lymph nodes are not enlarged. What is the
fistula establishment most likely diagnosis?
B. Surveillance
C. Conservative antibacterial treatment A. Cancer
D. Radical surgical treatment with B. Cyst
esophagus suturing C. Fibroadenoma
E. Surgical: gastric fistula establishment D. Mastopathy
E. Lipoma
114. A 30-year-old woman with a long hi-
story of chronic pyelonephritis complains 118. A 40-year-old female patient
about considerable weakness, sleepiness, has been observing profuse menses
decrease in diuresis down to 100 ml per accompanied by spasmodic pain in
day. AP- 200/120 mm Hg. In blood: creati- the lower abdomen for a year. Bi-
nine - 0,62 millimole/l, hypoproteinemia, manual examination performed during
albumines - 32 g/l, potassium - 6,8 milli- menstruation revealed a dense formati-
mole/l, hypochromic anemia, increased on up to 5 cm in diameter in the cervical
ESR. What is the first step in the pati- canal. Uterus is enlarged up to 5-6 weeks
ent treatment tactics? of pregnancy, movable, painful, of normal
consistency. Appendages are not palpable.
A. Haemodialysis Bloody discharges are profuse. What is the
B. Antibacterial therapy most likely diagnosis?
C. Enterosorption
D. Haemosorption A. Nascent submucous fibromatous node
E. Blood transfusion B. Abortion in progress
C. Cervical carcinoma
115. A 10-year-old girl consulted a doctor D. Cervical myoma
about thirst, frequent urination, wei- E. Algodismenorrhea
ght loss. She has been observing these
symptoms for about a month. Objecti- 119. A 65-year-old patient complai-
vely: no pathology of internal organs was ns about pain in the lumbar spine,
revealed. What laboratory analysis should moderate disuria. He has been sufferi-
be carried out in the first place? ng from these presentations for about
half a year. Prostate volume is 45
A. Blood glucose analysis on an empty cm3 (there are hypoechogenic nodes in
stomach both lobes, capsule invasion). Prostate-
B. Glucose in urine test on the base of specific antigen is 60 ng/l. Prostate biopsy
daily diuresis revealed an adenocarcinoma. Which of
C. Acetone in urine test the supplemental examination methods
D. Glucose tolerance test will allow to determine the stage of
E. Glucosuric profile neoplastic process in this patient?
116. A 19-year-old patient complains A. Computer tomography of pelvis
about skin rash that appeared 2 days ago B. Roentgenography of lumbar spine
after eating smoked fish. The rash di- C. Excretory urography
sappears after 4-6 hours but then turns up D. Bone scintigraphy
again. It is accompanied by itch. Objecti- E. Roentgenography of chest
vely: trunk and upper limbs are covered
with multiple pink blisters as big as a pea 120. A 44-year-old man has been worki-
or a bean. What is the most likely di- ng in coke industry for 16 years. Dust
agnosis? concentration at his workplace is 5-10
times higher than maximum permissi-
A. Acute urticaria ble concentration. Roentgenography of
B. Allergic dermatitis lungs revealed changes that are typical
C. Quincke’s edema for pneumoconiosis. What is the most
D. Toxicodermia probable type of pneumoconiosis in this
E. Purigo case?
117. A 68-year-old patient consulted
a doctor about a tumour in her left
Krok 2 Medicine 2010 17

A. Anthracosis A. 4
B. Anthracosilicosis B. 1
C. Silicatosis C. 2
D. Asbestosis D. 3
E. Siderosis E. 5
121. Study of morbidity rate in a city 125. 2 days ago a patient presented with
N revealed that population of different acute pain in the left half of chest, gneral
administrative districts differed in age weakness, fever and headache. Objecti-
structure. What statistic method allows vely: between the 4 and 5 rib on the left
to eliminate influence of this factor upon the skin is erythematous, there are multi-
morbidity indices? ple groups of vesicles 2-4 mm in diameter
filled with transparent liquid. What diease
A. Standardization are these symptoms typical for?
B. Wilcoxon’s t-criterion
C. Correlative regressive analysis A. Herpes zoster
D. Analysis of dynamic series B. Pemphigus
E. Calculation of average values C. Herpes simplex
D. Streptococcal impetigo
122. A driver had been fixing a car in E. Herpetiform Duhring’s dermatosis
a closed garage and afterwards complai-
ned about headache, dizziness, nausea, 126. A 27-year-old military servant was
muscle asthenia, sleepiness. Objectively: delivered to the regimental aid post 1,5
pulse and respiratory rate elevation, hour after getting an abdominal injury wi-
excitement, hypertension, delirium of th an aerial bomb splinter. Objectively:
persecution. What is the most likely di- the patient is in grave condition, pale, Ps-
agnosis? 132/min, AP- 80/60 mm Hg. Subcostal
area has a bandage on from the first-
A. Intoxication with carbon oxide aid pack, the bandage is soaked with
B. Intoxication with ethyl gasoline blood. The abdomen is tense. Percussion
C. Posttraumatic encephalopathy revealed dullness in flat parts that changes
D. Hypertensive crisis its location with the change of body posi-
E. Asthenovegetative syndrome tion. There are symptoms of peritoneal
irritation. What department of the regi-
123. A worker who undergoes regular mental aid post should provide medical
medical check-up for duodenal ulcer care?
received a subsidized 24-day sanatori-
um voucher from his plant. The term of A. Dressing ward
annual leave of a worker is 24 calender B. Admission-and-sorting tent
days, it will take 4 days more to get to the C. Isolation ward
sanatorium and back home. What is the D. Evacuation
procedure of obtaining a 4-day sick-leave? E. In the manner of self-help
A. Medical Expert Commission issues a 127. A sergeant got injured with a shell
4-day sick list splinter in the left subcostal region. He
B. Medical Expert Commission issues a was bandaged by his fellow soldiers wi-
28-day sick list th an individual field dressing. Afterwards
C. The doctor in charge issues a 4-day sick he was delivered to the regimental medi-
list cal station. The patient complains of di-
D. The doctor in charge issues a health zziness, weakness, thirst, abdominal pain.
certificate and sanatorium patient’s file for His general condition is grave, he is pale,
28 days Ps- 120 bpm. The abdomen is painful, soft
E. Medical Expert Commission issues a on palpation. The dressing is well-fixed
4-day health certificate but slightly soaked with blood. The vi-
ctim should be evacuated to the collecti-
124. The total area of a ward at the ng battalion with the following type of
therapeutical department is 28 m2 . What transport and in the following turn:
is the maximum number of beds that can
be exploited in this ward?
Krok 2 Medicine 2010 18

A. With ambulance vehicle in the 1st turn


B. With a passing vehicle in the 1st turn A. Hypertonic solution
C. With ambulance vehicle in the 2nd turn B. Vishnevsky ointment
D. With a passing vehicle in the 2nd turn C. Ichthyol ointment
E. With a passing vehicle in the 3rd turn D. Chloramine
E. Dimexide
128. On the 5th day after labor body
temperature of a 24-year-old parturient 132. A 37-year-old patient was delivered
suddenly rose up to 38, 7oC. She complai- to a hospital in unconscious state. He
ns about weakness, headache, abdominal got a closed abdominal injury. He is
pain, irritability. Objectively: AP- 120/70 suspected to have internal haemorrhage,
mm Hg, Ps- 92 bpm, to - 38, 7oC. Bimanual intraabdominal injury. He is being
examination revealed that the uterus was prepared an emergency operation. By
enlarged up to 12 weeks of pregnancy, it means of standard sera a physician
was dense, slightly painful on palpation. determined his blood group and got the
Cervical canal lets in 2 transverse fingers, following results: 0(I) - no agglutinati-
discharges are moderate, turbid, with foul on, А(II) - no agglutination, В(III) - no
smell. In blood: skeocytosis, lymphopenia, agglutination. What is the patient’s blood
ESR - 30 mm/h. What is the most likely di- group?
agnosis?
A. 0(I)
A. Endometritis B. А(II)
B. Parametritis C. В(III)
C. Pelviperitonitis D. АВ(IV)
D. Metrophlebitis E. -
E. Lochiometra
133. A 36-year-old man was delivered
129. An 11-yearold girl was taken by an to the surgical department an hour
acute disease: she got pain in the lumbar after a road accident. His condition
region, nausea, vomiting, frequent uri- is getting worse: respiratory insuffici-
nation, body temperature 39o C. Objecti- ency is progressing, there are cardiac
vely: the abdomen is soft, painful on abnormalities. Clinical and roentgenologi-
palpation in the lumbar region. Common cal investigations revealed mediastinal di-
urine analysis revealed considerable splacement. What process has caused this
leukocyturia, bacteriuria. The urine complication?
contained colibacilli. What is the most li-
kely diagnosis? A. Valvular pneumothorax
B. Open pneumothorax
A. Acute pyelonephritis C. Closed pneumothorax
B. Acute appendicitis D. Subcutaneous emphysema
C. Chronic glomerulonephritis E. Mediastinitis
D. Acute vulvovaginitis
E. Acute glomerulonephritis 134. An 18-year-old primigravida in
her 27-28 week of gestation underwent
130. A patient is active, lively, fussy. He an operation on account of acute
opposes the examination. His speech phlegmonous appendicitis. In the
is fast and loud, his statements are postoperative period it is necessary to
spontaneous and inconsequent. Specify take measures for prevention of the
the psychopathological state of this pati- following pegnancy complication:
ent:
A. Noncarrying of pregnancy
A. Psychomotor agitation B. Intestinal obstruction
B. Catatonic agitation C. Fetus hypotrophy
C. Delirium D. Premature placenta detachment
D. Behavioral disorder E. Late gestosis
E. Paranoid syndrome
135. A 30-year-old patient presents with
131. A surgeon examined a 42-year-old sudden loss of consciousness, tonoclonic
patient and diagnosed him with right spasms, AP- 150/100 mm Hg, heart rate
forearm furuncle, purulo-necrotic stage. - 100/min. Before the attack he felt di-
The furuncle was lanced. At the hydration zziness, sense of irreality, inexplicable
stage the wound dressing should enclose fear. After the attack the patient failed
the following medication:
Krok 2 Medicine 2010 19

to remember it. What is the most likely A. Legionella


diagnosis? B. Mycoplasm
C. Streptococcus
A. Epilepsy D. Staphylococcus
B. Morgagni-Adams-Stokes syndrome E. Pneumococcus
C. Vestibular syncope
D. Transitory ischemic attack 140. A 47-year-old patient complains
E. Hyperventilation ischemic attack about cough with purulent sputum, pain
in the lower part of the left chest, peri-
136. A 26-year-old male patient complai- odical body temperature rise. She has
ns of piercing pain during breathing, been suffering from these presentations
cough, dyspnea. Objectively: to - 37, 3oC, for about 10 years. Objectively: "drumsti-
respiration rate - 19/min, heart rate=Ps ck"distal phalanges. What examination
- 92/min; AP- 120/80 mm Hg. Vesicular would be the most informative for making
respiration. In the inferolateral parts of a diagnosis?
chest auscultation in both inspiration and
expiration phase revealed noise that was A. Bronchography
getting stronger at phonendoscope pressi- B. Bronchoscopy
ng and can be still heard after cough. ECG C. Survey radiograph of lungs
showed no pathological changes. What is D. Pleural puncture
the most likely giagnosis? E. Bacteriological analysis of sputum
A. Acute pleuritis 141. A heat station working on solid fuel
B. Intercostal neuralgia is located in a residential district. On
C. Subcutaneous emphysema cloudy foggy days in december there was
D. Spontaneous pneumothorax an increase in diseases with upper airway
E. Pericarditis sicca affection and signs of general intoxicati-
on. There were also mortal cases among
137. HIV displays the highest tropism the elderly people. What is the most likely
towards the following blood cells: factor that provoked toxic effect?
A. T-helpers A. Suspended materials
B. T-suppressors B. High air humidity
C. T-killers C. Calm
D. Thrombocytes D. Low air temperature
E. Erythrocytes E. Temperature gradient
138. In terms of megacalorie (1000 kcal = 142. A 32-year-old female patient
4184 kJ) the ration of an adult includes consulted a doctor about a skin affecti-
30 g of proteins, 37 g of fats, 137 g of on of her body, face and extremities which
carbohydrates, 15 mg of vitamin C, 0,6 turned up after peroral intake of biseptol.
mg of thiamine (vitamin B1 ). The ration is The patient was diagnosed with drug toxi-
UNBALANCED as to the contents of: coderma. What treatment methods would
be the most effective for this patient?
A. Vitamin C
B. Proteins A. Extracorporal therapy
C. Fats B. Anti-inflammatory therapy
D. Carbohydrates C. Antihistamine therapy
E. Thiamine D. Disintoxication therapy
E. Immunotherapy
139. A 22-year-old patient is a clerk. His
working day runs in a conditioned room. 143. A 42-year-old woman has had
In summer he was taken by an acute di- hyperpolymenorrhea and progressing
sease with the following symptoms: fever, algodismenorrhea for the last 10 years.
dyspnea, dry cough, pleural pain, myalgia, Gynecological examination revealed no
arthralgia. Objectively: moist rales on the changes of uterine cervix; discharges are
right, pleural friction rub. X-ray picture moderate, of chocolate colour, uterus is
showed infiltration of the inferior lobe. In slightly enlarged and painful, appendages
blood: WBC - 11 · 109 /l, stab neutrophi- are not palpable, the fornices are deep
ls - 6%, segmented neutrophils - 70%, and painless. What is the most likely di-
lymphocytes - 8%, ESR - 42 mm/h. What agnosis?
is the ethiological factor pneumonia?
Krok 2 Medicine 2010 20

A. Uterine endometriosis diffuse toxic goiter of the III-IV degree


B. Uterine carcinoma 12 years ago. Now she has recurrence of
C. Subserous uterine fibromyoma thyrotoxicosis. The patient was offered
D. Endomyometritis operative intervention, but it is necessary
E. Adnexal endmetriosis first to localize the functioning gland ti-
ssue. What method should be applied for
144. On the tenth day after discharge this purpose?
from the maternity house a 2-year-old
patient consulted a doctor about body A. Gland scanning
temperature rise up to 39o C, pain in the B. USI
right breast. Objectively: the mammary C. Puncture aspiration biopsy
gland is enlarged, there is a hyperemized D. Roentgenography of neck
area in the upper external quadrant, in the E. Roentgenography of esophagus
same place there is an ill-defined indurati-
on, lactostasis, fluctuation is absent. 148. A 26-year-old woman complains of
Lymph nodes of the right axillary regi- having bloody discharges from the geni-
on are enlarged and painful. What is the tals for the last 14 days, abdominal pain,
most likely diagnosis? general fatiguability, weakness, weight
loss, body temperature rise, chest pain,
A. Lactational mastitis obstructed respiration. 5 weeks ago she
B. Abscess underwent induced abortion in the 6-7
C. Erysipelas week of gestation. Objectively: the patient
D. Dermatitis is pale and inert. Bimanual examination
E. Tumour revealed that the uterus was enlarges up
to 8-9 weeks of gestation. In blood: Hb- 72
145. A 47-year-old male patient got a g/l. Urine test for chorionic gonadotropin
flame burn of trunk and upper extremi- gave the positive result. What is the most
ties and was delivered to the hospital. The likely diagnosis?
patient is in grave condition, confused
mental state, with fever. AP- 80/50 mm A. Chorioepithelioma
Hg, Ps- 118 bpm. It was locally stated that B. Metroendometritis
the patient got III B degree burns with C. Uterus perforation
total area of 20%. What medical actions D. Uterine fibromyoma
should be taken? E. Uterine carcinoma
A. Injection of narcotic analgetics and 149. A 1,5-year-old child was taken by an
powdered blood substitutes acute disease: body temperature up to
B. Initial surgical d-bridement 39oC, frequent vomiting up to 5 times.
C. Administration of detoxicating blood Nervous system tests revealed positive
substitutes Kernig’s and Brudzinski’s signs. The gi-
D. Necrotomy of burn surface, ven symptoms relate to:
haemotransfusion
E. Antibacterial and detoxicating therapy A. Meningeal signs
B. Discoordination syndrome
146. A 54-year-old female patient was C. Motor disorder syndrome
admitted to the hospital with evident D. Encephalic syndrome
acrocyanosis, swollen cervical veins, E. Infectious toxicosis signs
enlarged liver, ascites. Cardiac borders
are dilated. Heart sounds cannot be 150. A pediatrician had a conversation wi-
auscultated, apical beat is undetectable. th a mother of a 7-month-old breast-fed
AP is 100/50 mm Hg. X-ray picture of boy and found out that the child was fed
chest shows enlarged heart shadow in 7 times a day. How many times should the
form of a trapezium. What pathology mi- child of such age be fed?
ght have caused these symptoms?
A. 5 times
A. Cardiac tamponade B. 3 times
B. Exudative pleuritis C. 4 times
C. Complex heart defect D. 6 times
D. Acute cardiac insufficiency E. 7 times
E. Hiatal hernia
151. 1 day ago a 19-year old male patient
147. A 16-year-old female patient got a trauma after a fall from 1,5 m hei-
underwent an operation on account of ght when he "bestrode"a wooden beam.
Krok 2 Medicine 2010 21

Urination is extremely difficult, there is A. Subungual panaritium


macrohematuria, urethrorrhagia. There is B. Erysipelas
a hematoma in the perineum and on the C. Paronychia
scrotum. Urethrography revealed flowing D. Erysipeloid
of contrast medium into the membranous E. Abscess
part of urethra. What is the treatment
tactics of choice? 155. Statistic of patients of common medi-
cal practice displays constant increase in
A. Epicystostomy elderly and old people number. What ki-
B. Urine bladder catheterization for urine nd of pathology is expected to prevail in
diversion the morbidity structure of population in
C. Urethra bougienage question?
D. Cold to the perineum, hemostatic
therapy A. Chronic pathology
E. Primary suturing of urethra B. Occupational pathology
C. Non-epidemic pathology
152. A 40-year-old patient underwent an D. Acute pathology
operation for a lumbar phlegmon. Body E. Infectious pathology
temparature rose again up to 38o C, he
got intoxication symptoms, there was an 156. A 30-year-old patient consulted a
increase of leukocyte number in blood. doctor about menstruation absence for
The wound that was nearly free from 2 years after labour, loss of hair, body
necrotic tissues and full of granulations weight loss. The labour was compli-
started to discharge pus, the granulations cated by a haemorrhage caused by uterus
turned pale. What complication dveloped hypotonia. Objectively: the patient is
in this patient? asthenic, external genitals are hypoplastic,
the uterus body is small and painless. The
A. Sepsis appendages are not palpaple. What is the
B. Putrid phlegmon most likely diagnosis?
C. Erysipelas
D. Allergic reaction A. Sheehan’s syndrome
E. Erysipeloid B. Ovarian amenorrhea
C. Turner’s syndrome
153. A 40-year-old female patient has a hi- D. Exhausted overy syndrome
story of rheumatism. She complains about E. Galactorrhea-amenorrhea syndrome
acute pain in her left eye, especially at
night, vision impairment, photophobia, 157. A 25-year-old patient complains
lacrimation. The patient cannot suggest of burning and pain during urination,
any reasons for the disease. Objectively: purulent discharges from the urethra that
weak pericorneal injection, flattening of turned up 2 days ago. Objectively: the
iris relief, iris discoloration. What is the urethral labia are edematic, hyperemic,
most likely diagnosis? there are profuse purulent discharges
from the urethra. Provisional diagnosis:
A. Iridocyclitis recent acute gonorrheal urethritis. What
B. Iritis supplemental data of clinical examinati-
C. Keratitis ons will alow to specify the diagnosis?
D. Choroiditis
E. Acute attack of glaucoma A. Microscopy of the urethral discharges
B. Serological blood analysis
154. Three days ago a boy underwent C. Common urine analysis
removal of a foreign body from under a D. Coccal cells detection during discharge
nail plate. 2 days later he felt acute pulsati- microscopy
ng pain at the end of the nail bone which E. Yeast cells detection during discharge
was getting worse at pressing. Nail fold microscopy
became hyperemic, body temperature
rose up to 37, 5oC, there was a change in 158. A 28-year-old patient has been admi-
nail plate colour. What is the most likely tted to the gynecological department
diagnosis? three days after a casual coitus. She
complains about pain in her lower
abdomen and during urination, profuse
purulent discharges from the vagina,
body temperature rise up to 37, 8oC. The
patient was diagnosed with acute bi-
Krok 2 Medicine 2010 22

lateral adnexitis. Supplemental examinati- ns about episodes of unconsciousness,


on revealed: the 4th degree of purity of spontaneous syncopes that are quickly
the vaginal secretion, leukocytes within over after a change of body position. A
the whole visual field, diplococcal bacteria syncope can be accompanied byquickly
located both intra- and extracellularly. elapsing bradycardia. There are no other
What is the etiology of acute adnexitis in complications of gestation. What is the
this patient? most likely reason for such condition?
A. Gonorrheal A. Postcava compresseion by the gravid
B. Colibacterial uterus
C. Chlamydial B. Pressure rise in the veins of extremities
D. Trichomonadal C. Pressure fall in the veins of extremities
E. Staphylococcal D. Vegetative-vascular dystonia (cardial
type)
159. A boy is 8 year old. His physi- E. Psychosomatic disorders
cal development is compliant with his
age. The child has had cardiac murmur 163. A 49-year-old woman complains
since birth. Objectively: skin and visi- about headache, head and neck going hot,
ble mucous membranes are of normal increased perspiration, palpitation, arteri-
colour. AP- 100/70 mm Hg. Auscultation al pressure rise up to 170/100 mm Hg,
revealed systolo-diastolic murmur and di- irritability, insomnia, tearfulness, memory
astolic shock above the pulmonary artery. impairment, rare and scarce menses, body
ECG shows overload of the left heart. weight increase by 5 kg over the last half
Roentgenoscopy shows coarsening of the a year. What is the most likely diagnosis?
lung pattern, heart shadow of normal
form. What is the most likely diagnosis? A. Climacteric syndrome
B. Premenstrual syndrome
A. Atrioseptal defect C. Vegetative-vascular dystonia
B. Pulmonary artery stenosis D. Arterial hypertension
C. Aorta coarctation E. Postcastration syndrome
D. Fallot’s tetrad
E. Patent ductus arteriosus 164. The correlation between the servi-
ce record and eosinophil concentration in
160. A 45-year-old patient with uroli- blood was studied in workers at dyeing
thiasis had an attack of renal colic. shops of textile factories. What index will
What is the mechanism of the attack be the most informative for the analysis of
development? this data?
A. Disturbed urine outflow from the A. Correlation factor
kidney B. Student’s criterion
B. Increase in relative density of urine C. Standardized index
C. Ureteric twists D. Fitting criterion
D. Destruction of glomerules E. Sign index
E. Renal artery spasm
165. A 38-year-old patient complains
161. A 25-year-old woman came to a about inertness, subfebrile temperature,
maternity welfare clinic and complained enlargement of lymph nodes, nasal
about being unable to conceive within 3 haemorrhages, ostealgia. Objectively: the
years of regular sexual life. Examinati- patient’s skin and mucous membranes
on revealed weight gain, male pattern of are pale, palpation revealed enlarged
hair distribution on the pubis, excessi- painless lymph nodes; sternalgia; liver
ve pilosis of thighs. Ovaries were dense was enlarged by 2 cm, spleen - by 5
and enlarged, basal temperature was cm, painless. In blood: erythrocytes -
monophase. What is the most likely di- 2, 7 · 1012 /l, Hb- 84 g/l, leukocytes - 58 ·
agnosis? 109 /l, eosinophils - 1%, stab neutrophi-
A. Sclerocystosis of ovaries ls - 2%, segmented neutrophils - 12%,
B. Tubo-ovaritis lymphocytes - 83%, lymphoblasts - 2%,
C. Adrenogenital syndrome smudge cells; ESR- 57 mm/h. What is the
D. Premenstrual syndrome most likely diagnosis?
E. Gonadal dysgenesis
162. A 32-year-old gravida complai-
Krok 2 Medicine 2010 23

A. Chronic lymphatic leukemia A. Acute renal insufficiency


B. Chronic myeloleukemia B. Cardial shock
C. Acute lymphatic leukemia C. Allergic reaction to the donor
D. Acute myeloleukemia erythrocyte mass
E. Lymphogranulomatosis D. Citrate intoxication
E. Toxic infectious shock
166. A 58-year-old patient complains
about sensation of numbness, sudden 169. A 45-year-old man has been exhi-
paleness of II-IV fingers, muscle ri- biting high activity for the last 2 weeks,
gidness, intermittent pulse. The pati- he became talkative, euphoric, had li-
ent presents also with polyarthralgia, ttle sleep, claimed being able "to save
dysphagia, constipations. The patient’s the humanity"and solve the problem
face is masklike, solid edema of hands is of cancer and AIDS, gave money the
present. The heart is enlarged; auscultati- starangers. What is the most likely di-
on revealed dry rales in lungs. In blood: agnosis?
ESR - 20 mm/h, crude protein - 85/l, γ-
globulines - 25%. What is the most likely A. Maniacal onset
diagnosis? B. Panic disorder
C. Agitated depression
A. Systemic scleroderma D. Schizo-affective disorder
B. Dermatomyositis E. Catatonic excitation
C. Rheumatoid arthritis
D. Systemic lupus erythematosus 170. A 67-year-old female patient sufferi-
E. Raynaud’s disease ng from the essential hypertension
suddenly at night developed headache,
167. A 36-year-old female patient dyspnea that quickly progressed to
complains of general weakness, edemata asphyxia. Objectively: the patient is pale,
of her face and hands, rapid fatiguability with sweaty forehead, AP- 210/140 mm
during walking, difficult diglutition, cardi- Hg, heart rate - 120/min, auscultation
ac irregularities. These symptoms turned revealed solitary dry rales and moist rales
up 11 days after a holiday at the seasi- in the lower parts. The shins are pastose.
de. Objectively: face erythema, edema of What kind of emergency aid would be the
shin muscles. Heart sounds are muffled, most efficient in this case?
AP is 100/70 mm Hg. In blood: ASAT
activity is 0,95 millimole/h·l, ALAT - 1,3 A. Nitroglycerin and furosemide
millimole/h·l, aldolase - 9,2 IU/l, creatine intravenously
phosphokinase - 2,5 millimole Р/g·l. What B. Enalapril and furosemide intravenously
method of study would be the most speci- C. Digoxin and nitroglycerin intravenously
fic? D. Labetalol and furosemide intravenously
E. Nitroglycerin intravenously and capoten
A. Muscle biopsy internally
B. ECG
C. Echocardiogram 171. A patient suffering from gastroesophageal
D. Electromyography reflux has taken from time to time a
E. Determination of cortisol concentration certain drug that "reduces acidity"over
in blood and urine 5 years. This drug was recommended
by a pharmaceutist. The following side
168. A 33-year-old patient has acute blood effects are observed: osteoporosis, muscle
loss: erythrocytes - 2, 2 · 1012 /l, Hb- 55 g/l, asthenia, indisposition. What drug has
blood group is A(II)Rh+ . Accidentally such following effects?
the patient got a transfusion of donor
A. Aluminium-bearing antacid
erythrocyte mass of AB(IV )Rh+ group.
An hour later the patient became anxi- B. Inhibitor of proton pump
C. H2 -blocker
ous, got abdominal and lumbar pain.
Ps- 134 bpm, AP- 100/65 mm Hg, body D. Metoclopramide
E. Gastrozepin
temperature - 38, 6oC. After catheterizati-
on of urinary bladder 12 ml/h of dark- 172. A 26-year-old patient complains
brown urine were obtained. What compli- about considerable muscle weakness, di-
cation is it? zziness, extended abdominal pain, nausea
and vomiting giving no relief. The disease
has been gradually developing within 6
months. There was progress of general
Krok 2 Medicine 2010 24

weakness, skin darkening. The patient fell A. Osteogenic sarcoma


into grave condition after an ARD: there B. Hyperparathyroid dystrophy
appeared abdominal pain and frequent C. Chronic osteomyelitis
vomiting. Objectively: the skin is dry with D. Myelomatosis
diffuse pigmentation. Heart sounds are si- E. Marble-bone disease
gnificantly weakened, heart rate - 60/min,
AP- 80/40 mm Hg. The abdomen is sli- 176. A 10-year-old girl was admitted to a
ghtly painful in the epigastrial region. In hospital with carditis presentations. It is
blood: WBCs - 8, 1 · 109 /l, glucose - 3,0 known from the anamnesis that two weeks
millimole/l. What is the most likely di- ago she had exacerbation of chronic tonsi-
agnosis? llitis. What is the most likely etiological
factor in this case?
A. Chronic adrenal insufficiency. Addi-
sonian crisis A. Streptococcus
B. Acute pancreatitis B. Staphylococcus
C. Toxic infectious shock C. Pneumococcus
D. Acute food poisoning D. Klebsiella
E. Acute cholecystitis E. Proteus

173. A puerpera is 25 years old, it is her 177. A department chief of an in-patient


second day of postpartum period. It was hospital is going to inspect resident
her first labour, it took place at full term. doctors as to observation of medical-
The lochia should be: technological standards of patient service.
What documentation should be checked
A. Bloody for this purpose?
B. Sanguino-serous
C. Mucous A. Health cards of in-patients
D. Purulent B. Statistic cards of discharged patients
E. Serous C. Treatment sheets
D. Registry of operative interventions
174. A 32-year-old patient consulted a E. Annual report of a patient care instituti-
doctor about being inable to get pregnant on
for 5-6 years. 5 ago the primipregnancy
ended in artificial abortion. After the vagi- 178. Workers of a laboratory produci-
nal examination and USI the patient was ng measuring devices (manometers,
diagnosed with endometrioid cyst of the thermometers etc) complain about a
right ovary. What is the optimal treatment mettalic taste in mouth, stomatitis,
method? dyspepsia, sleep disturbance, unsteady
walk, abrupt decrease in cardiac activi-
A. Surgical laparoscopy ty. These presentations must have been
B. Anti-inflammatory therapy caused by the intoxication with the
C. Conservative therapy with estrogen- following substance:
gestagenic drugs
D. Hormonal therapy with androgenic A. Mercury
hormones B. Lead
E. Sanatorium-and-spa treatment C. Manganese
D. Toluol
175. A 20-year-old man complains about E. Tetraethyl lead
pain arising in the lower third of femoral
bone under stress and at rest. He deni- 179. A full-term infant has respiratory
es having a trauma. Objectively: the rate of 26/min, heart rate of 90/min, blue
skin is of normal colour, deep palpati- skin, muscle hypotonia. During catheter
on reveals pastosity and pain, knee joi- suction of mucus and amniotic fluid from
nt motions are limited. X-ray picture the nose and mouth the child reacted wi-
of the meta-epyphisis of distal femur th a grimace. Low reflexes. Auscultati-
shows a destruction zone and spicules. In on revealed weakened vesicular respirati-
blood: immature cell forms, no signs of on above lungs. Heart sounds are loud.
inflammation. What is the most likely di- After 5 minutes the respiration became
agnosis? rhythmic, at the rate of 38/min, heart rate
of 120/min. What is the most likely di-
agnosis?
Krok 2 Medicine 2010 25

A. Asphyxia
B. Inborn pneumonia A. Gram-negative diplococci
C. Birth trauma B. Gram-positive diplococci
D. Bronchopulmonary dysplasia C. Spirochaete
E. Respiratory distress syndrome D. Proteus vulgaris
E. Mycoplasma
180. Examination of placenta revealed a
defect. An obstetrician performed manual 184. As a result of load lifting a 68-year-
investigation of uterine cavity, uterine old female felt acute pain in the lumbar
massage. Prophylaxis of endometritis in region, in a buttock, posterolateral surface
the postpartum period should involve of her right thigh, external surface of
following actions: the right shin and dorsal surface of foot.
Objectively: weakness of the anterior ti-
A. Antibacterial therapy bial muscle, long extensor muscle of the
B. Instrumental revision of uterine cavity right toes, short extensor muscle of the ri-
C. Haemostatic therapy ght toes. Low Achilles reflex on the right.
D. Contracting agents Positive Lasegue’s sign. What examinati-
E. Intrauterine instillation of dioxine on method would be the most effective for
specification of the diagnosis of discogenic
181. A 35-year-old man complains about compression of L5 root?
intense lumbar pain irradiating to the
inguinal area, external genitalia, thigh; A. Magnetic resonance scan
frequent urination, chill, nausea, vomi- B. Spinal column X-ray
ting. Objectively: positive Pasternatsky’s C. Electromyography
symptom. Urine analysis revealed that D. Angiography
RBCs and WBCs covered the total fi- E. Lumbar puncture
eld of microscope; the urine exhibited hi-
gh protein concentration. These clinical 185. A hospital admitted a patient wi-
presentations were most likely caused by th coarse breathing (obstructed inspirati-
the following pathology: on), skin cyanosis, tachycardia and arteri-
al hypertension. He has a histrory of
A. Urolithiasis, renal colic bronchial asthma. An hour ago he was
B. Cholelithiasis, biliary colic having salbutamol inhalation and forgot
C. Renal infarct to remove a cap that was aspired whi-
D. Intestinal obstruction le taking a deep breath. What measures
E. Osteochondrosis, acute radicular should the doctor take?
syndrome
A. To perform the Heimlich manoever
182. Head circumference of a 1-month- B. To perform conicotomy immediately
old boy with signs of excitement is 37 cm, C. To send for an anesthesiologist and wait
prefontanel is 2x2 cm large. After feedi- for him
ng the child regurgitates small portions D. To use an inhalation of β2 -adrenoceptor
of milk; stool is normal in its volume and agonist
composition. Muscle tone is within norm. E. To make a subcutaneous injection of
What is the most likely diagnosis? dexamethasone
A. Pylorospasm 186. A 28-year-old patient was delivered
B. Meningitis to the admission ward in the unconscious
C. Pylorostenosis state with generalized epileptic attacks
D. Microcephaly taking place every 15-20 minutes. Duri-
E. Craniostenosis ng transportation the patient was given
183. A patient consulted a venereologi- two injections of diazepam, magnesia
sulphate, but they failed to bring the pati-
st about painful urination, reddening of
the external opening of urethra, profuse ent to consciousness. What department
purulent discharges from the urethra. He should render emergency aid?
considers himself to be ill for 3 days. A. Resuscitation department
He also associates the disease with a B. Neurological department
casual sexual contact that took place for C. Surgcal department
about a week ago. If provisional diagnosis D. Therapeutic department
"acute gonorrheal urethritis"is confirmed, E. Psychiatric department
then bacteriological study of urethral di-
scharges will reveal: 187. On the second day after preventi-
Krok 2 Medicine 2010 26

ve vaccination a 2-year-old boy presented A. Posttraumatic osteomyelitis


with abdominal pain without clear locali- B. Hematogenic osteomyelitis
zation, body temperature rose up to 38oC. C. Wound abscess
On the third day the child got red papular D. Posttraumatic phlegmon
haemorrhagic eruption on the extensor E. Suture sinus
surfaces of limbs and around the joints.
Knee joints were edematic and slightly 191. Estimation of community health level
painful. Examination of other organs and involved analysis of a report on diseases
systems revealed no pathological changes. registered among the population of distri-
What is the most likely diagnosis? ct under charge (reporting form 12). What
index is calculated on the grounds of this
A. Haemorrhagic vasculitis report?
B. Thrombocytopenic purpura
C. Meningococcemia A. Common morbidity rate
D. Urticaria B. Index of pathological affection
E. DIC syndrome C. Index of morbidity with temporary
disability
188. On the 6th day of life a child got D. Index of hospitalized morbidity
multiple vesicles filled with seropurulent E. Index of basic non-epidemic morbidity
fluid in the region of occiput, neck and
buttocks. General condition of the chi- 192. A 49-year-old patient undergoes
ld is normal. What disease should be regular medical check-up for uterine fi-
suspected? bromyoma. Within the last year the uterus
has enlarged up to 20 weeks of gestation.
A. Vesiculopustulosis What is the rational way of treatment?
B. Impetigo neonatorum
C. Miliaria A. Surgical treatment
D. Impetigo B. Hormonal therapy
E. Epidermolysis bullosa C. Further surveillance
D. Embolization of uterine arteries
189. A 60-year-old patient complains E. Treatment with prostaglandin inhibitors
about asphyxia, palpitation, rapid fati-
guability. He has 8 year history of essential 193. A patient is being prepared for the
hypertension. Objectively: the left cardiac operation on account of varix dilatation
border is 2 cm deviated to the left from of lower extremities veins. Examination
the medioclavicular line, heart sounds of the patient’s soles revealed flour-like
are rhythmic and weak; there is diastolic desquamation along the skin folds. All
shock above aorta. AP- 170/100 mm Hg. the toenails are greyish-yellow, thickened
Liver - +2 cm; shin pastosity is present. and partially decayed. What dermatosis
ECG shows deviation of cardiac axis to should be suspected?
the left, left ventricle hypertrophy. Ejecti-
on fraction - 63%. What type of cardiac A. Rubromycosis
insufficiency is observed? B. Pityriasis versicolor
C. Candidosis
A. Diastolic D. Microsporia
B. Systolic E. Microbial eczema
C. Combined
D. It’s a norm 194. A 46-year-old patient was issued a
E. Unspecified 10-day sick list because of exacerbation of
chronic cholecystitis. The patient’s general
190. A 3-year-old male patient consulted condition got better, but the clinical mani-
a family doctor 2 months after he had festations of the disease are still present.
been operated for an open fracture of What authority is entitled to extend the
brachial bone. Objectively: the patient’s sick list?
condition is satisfactory, in the region of
the operative wound there is a fistula A. Medical Expert Commission
with some purulent discharge, redness, B. Family doctor
fluctuation. X-ray picture shows brachi- C. Deputy head doctor for terapeutic
al bone destruction with sequestra. What management
complication arose in the postoperative D. Deputy head doctor for medical-labour
period? expertise
E. Head doctor
Krok 2 Medicine 2010 27

195. A 43-year-old female patient was 198. A 52-year-old patient works as a


delivered to the hospital in grave condi- secretary and has 30 year record of servi-
tion. She suffers from Addison’s di- ce. She complains of spasms in her ri-
sease. The patient had been regularly ght hand during working and inability
taking prednisolone but a week before to type and write. Up to 80% of her
she stopped taking this drug. Objecti- work involves hand load. The patient has
vely: sopor, skin and visible mucous been presenting with these symptoms for
membranes are pigmented, skin and 2 years. Objectively: the right hand is
muscle turgor is decreased. Heart sounds tense, there is an increase in muscle tone,
are muffled, rapid. AP- 60/40 mm Hg, attempts to write cause spasms. Exami-
heart rate - 96/min. In blood: Na - 120 mi- nation revealed no pathological changes
llimole/l, K - 5,8 millimole/l. Development of CNS. What is the most likely diagnosis?
of this complication is primarily caused by
the deficit of the following hormone: A. Spastic form of coordination neurosis
B. Neuralgic form of coordination neurosis
A. Cortisol C. Paretic form of coordination neurosis
B. Corticotropin (ACTH) D. Hysteric neurosis
C. Adrenaline E. Chronic manganese intoxication
D. Noradrenaline
E. Adrostendion 199. During examination at a military
commissariat a 15-year-old teenager was
196. Forensic medical expertise of corpse found to have interval sysolic murmur on
of a newborn revealed: body weight 3500 the cardiac apex, diastolic shock above
g, body length 50 cm, the umbilical cord the pulmonary artery, tachycardia. Which
was smooth, moist, glossy, without any si- of the suuplemental examination methods
gns of drying. Hydrostatic tests were posi- will be the most informative for the di-
tive. The test results are the evidence of: agnosis specification?
A. Live birth A. Echocardiography
B. Stillbirth B. Electrocardigraphy
C. Primary atelectasis C. Roengenography
D. Secondary atelectasis D. Phonocardiography
E. Hyaline membrane disease E. Rheography
197. A maternity house has admitted a 200. A 67-year-old female patient
primagravida complaining of irregular, complains about edemata of face and legs,
intense labour pains that have been lasti- pain in the lumbar area that is getting
ng for 36 hours. The woman is tired, failed worse at moving; great weakness, someti-
to fall asleep at night. The fetus is in longi- mes nasal haemorrhages, rise of body
tudinal lie, with cephalic presentation. temperature up to 38, 4oC. Objectively:
The fetus heartbeat is clear and rhythmic, painfulness of vertebral column and ribs
145/min. Vaginal examination revealed on palpation. Laboratorial study revealed
that the uterine cervix was up to 3 cm daily proteinuria of 4,2 g, ESR- 52 mm/h.
long, dense, with retroflexion; the external What changes of laboratory indices are to
orifice was closed; the discharges were of be expected?
mucous nature. What is the most likely di-
agnosis? A. Whole protein of blood serum - 101 g/l
B. Leukocytes - 15,3 g/l
A. Pathological preliminary period C. Haemoglobin - 165 g/l
B. Uterine cervix dystocia D. Albumins - 65%
C. Primary uterine inertia E. γ-globulins - 14%
D. Physiological preliminary period
E. Secondary uterine inertia
Krok 2 Medicine 2011 1

1. A patient suddenly felt an acute 5. A schizophrenic patient considers hi-


chest pain irradiating to the left arm. mself to be "an outstanding scientist,
Objectively: the patient is excited, with a brilliant composer and an unrivalled
pale skin. Breathing rate - 38/min, AP artist". He complains that "family and
- 180/110 mm Hg. Later the patient lost friends are always jealous of him and
consciousness and fell down. Pulse on the want to poison him". Determine the
great vessels was absent, the pupils were psychopathological syndrome:
equally dilated. What is the most likely di-
agnosis? A. Paranoiac
B. Paranoid
A. Clinical death C. Manic
B. Agonal state D. Paratrophic
C. Coma E. Hebephrenic
D. Heart attack
E. Disorder of the cerebral circulation 6. Examination of a 9-month-old girl
revealed skin pallor, cyanosis during exci-
2. A 62-year-old patient complaining tement. Percussion revealed transverse
of enlargement of cervical, supraclavi- dilatation of cardiac borders. Auscultati-
cular and axillary lymph nodes, subfebri- on revealed continuous systolic murmur
le temperature for the last 3 months has to the left of the breastbone in the
been admitted to a hospital. In blood: 3-4 intercostal space. This murmur is
WBCs - 64 · 109 /l, lymphocytes - 72%. conducted above the whole cardiac regi-
What method of study should be used to on to the back. What congenital cardiac
specify the diagnosis? pathology can be suspected?
A. Myelogram A. Defect of interventricular septum
B. Lymphography B. Defect of interatrial septum
C. Lymphoscintigraphy C. Coarctation of aorta
D. X-rays D. Fallot’s tetrad
E. Thermography E. Pulmonary artery stenosis
3. A child is 2 years old. The child 7. A 42-year-old patient complains of back
complains of hoarse voice, dyspnea with pain, darkened urine, general weakness,
obstructed inspiration. The disease started dizziness that occurred after treating a
3 days ago from dry cough and nose stuffi- cold with aspirin and ampicillin. Objecti-
ness. Objectively: general condition is vely: the patient is pale, with subicteric
unbalanced, stridor is present. The child’s sclerae. HR - 98 bpm. Liver - +2 cm,
skin is pale. Body temperature is 37, 7oC. spleen - +3 cm. In blood: RBCs - 2, 6 ·
The palatine arches are hyperemic. There 1012 /l, Hb - 60 g/l, CI - 0,9, WBCs -
is no deposit. Heart sounds are rhythmic. 9, 4 · 109 /l, basophils - 0,5%, eosinophils
Auscultation of lungs reveals rough - 3%, stab neutrophils - 6% segmented
breathing sounds, crepitation is absent. neutrophils - 58%, lymphocytes - 25%,
Parainfluenza virus has been detected in monocytes - 7%, ESR - 38 mm/hour, reti-
nasopharynx lavage. What is the most li- culocytes - 24%. Total bilirubin - 38 milli-
kely diagnosis? mole/l. What complication occurred in the
patient?
A. Acute laryngotracheitis
B. Epiglottitis A. Acquired hemolytic anemia
C. Foreign body B. Toxic hepatitis
D. Diphtheria C. Cholelithiasis
E. Laryngospasm D. Agranulocytosis
E. Paroxysmal nocturnal hemoglobinuria
4. A teacher of a secondary school was
diagnosed with pulmonary tuberculosis. 8. A hospital has admitted a 52-year-
What is the maximum duration of his old patient with disseminated pulmonary
medical certificate? tuberculosis, complaints of acute pain in
the right half of chest, that developed
A. Ten months after going upstairs to the 3rd floor; cough,
B. Five months dyspnea at rest, marked cyanosis. What ki-
C. Four months nd of complication should suspected first
D. Two months of all?
E. A month
Krok 2 Medicine 2011 2

A. Spontaneous pneumothorax A. Prednisolone


B. Cardiac failure B. Lasix
C. Pulmonary failure C. Adrenaline
D. Pleuritis D. Aminophylline
E. Acute myocardial infarction E. Analgine
9. A 44-year-old patient has been admi- 13. A 16-year-old patient who has a hi-
tted to a hospital with complaints of dull, story of intense bleedings from minor
aching pain in the left lumbar region, the cuts and sores needs to have the roots
admixture of pus in the urine. Examinati- of teeth extracted. Examination reveals
on revealed a grade II staghorn calculus an increase in volume of the right knee
on the left. What method of treatment is joint, limitation of its mobility. There are
indicated for this patient? no other changes. Blood analysis shows
an inclination to anaemia (Hb- 120 g/l).
A. Surgery Before the dental intervention it is requi-
B. Contact lithotripsy red to prevent the bleeding by means of:
C. Distance lithotripsy
D. Conservative therapy A. Cryoprecipitate
E. Ascending litholysis B. Epsilon-aminocapronic acid
C. Fibrinogen
10. On the 21 day after appearance of vesi- D. Dried blood plasma
culous chickenpox rash a 7-year-old chi- E. Calcium chloride
ld developed ataxia, nystagmus, intenti-
on tremor, muscle hypotonia. Liquor 14. A 3-year-old child has been di-
analysis shows a low-grade lymphocytic agnosed with type I diabetes mellitus,
pleocytosis, slightly increased protein rate. hyperosmolar coma. The laboratory confi-
What complication is it? rmed the diagnosis. Which laboratory fi-
ndings are characteristic for such conditi-
A. Encephalitis on?
B. Purulent meningitis
C. Pneumonitis A. High hyperglycemia without ketonemia
D. Acute nephritis B. Hyperglycemia and ketonemia
E. Postherpetic neuralgia C. Hyperglycemia and glucosuria
D. Hyperglycemia and ketonuria
11. A 38-year-old male patient has been E. Hyperglycemia and high indicators of
taking alcohol for 3 years. 3 days after acid-base balance
a regular drinking period he felt anxi-
ety and fear. It appeared to him that he 15. A 58-year-old female patient came
was surrounded by spiders and worms, to the antenatal clinic with complai-
pursued by some "condemnatory voi- nts of bloody light-red discharges from
ces". His behaviour became aggressive. the genital tracts. Menopause is 12
The patient demonstrated correct self- years. Gynaecological examination found
awareness but impairment of temporal externalia and vagina to have age involuti-
and spatial orientation. What is the most on; uterine cervix was unchanged, there
likely diagnosis? were scant bloody discharges from uteri-
ne cervix, uterus was of normal size;
A. Delirium alcoholicum uterine appendages were not palpable;
B. Alcoholic paranoia parametria were free. What is the most
C. Alcoholic hallucinosis likely diagnosis?
D. Alcoholic encephalopathy
E. Pathologic intoxication A. Uterine carcinoma
B. Atrophic colpitis
12. An 8-year-old boy suffering from C. Abnormalities of menstrual cycle with
haemophilia was undergoing transfusion climacteric character
of packed red cells. Suddenly he felt pain D. Cervical carcinoma
behind the breastbone and in the lumbar E. Granulosa cell tumor of ovary
area, dyspnea, cold sweat. Objectively:
pale skin, heart rate - 100/min, AP - 60/40 16. A 3-year-old child was playing in a
mm Hg; oliguria, brown urine. For the playpen when he suddenly developed
treatment of this complication the followi- paroxysmal cough and shortness of
ng drug should be administered: breath. Objectively: dry cough, mixed
dyspnea. Lung auscultation revealed
some wheezes. Breathing sounds on the
Krok 2 Medicine 2011 3

right are diminished. The child doesn’t A. Renal cancer


mix with other children. Immunization B. Polycystic renal disease
is age-appropriate. What pathological C. Renal amyloidosis
condition can be suspected? D. Urolithiasis
E. Chronic glomerulonephritis
A. Foreign body in the respiratory tracts
B. Pneumonia 20. A 47-year-old patient complains of
C. Acute respiratory viral infection insomnia, heaviness over his entire body,
D. Pertussis constantly depressed mood. He considers
E. Bronchial asthma himself good-for-nothing, inadequate.
Believes that he is a burden to his fami-
17. A 44-year-old patient complai- ly, wants to die. The patient is depressed,
ns about difficult urination, sensati- inactive, has a hypomimic face with
on of incomplete urinary bladder sorrowful expression. He speaks qui-
emptying. Sonographic examination of etly and monotonely,gives short answers.
the urinary bladder near the urethra What is the most likely diagnosis?
entrance revealed an oval well-defined
hyperechogenic formation 2x3 cm large A. Major depressive disorder
that was changing its position during the B. Atherosclerotic depression
examination. What conclusion can be C. Initial stage of Alzheimer’s disease
made? D. Late-onset schizophrenia
E. Neurotic depression
A. Concrement
B. Malignant tumour of the urinary 21. Bacterial analysis of air in a livi-
bladder ng space in winter period by means of
C. Urinary bladder polyp Krotov’s apparatus revealed that total
D. Prostate adenoma number of microorganisms in 1m3 of
E. Primary ureter tumour air was 7200. What is the permissible
number of microorganisms for the air to
18. A 10-year-old child has been folowed- be characterized as "pure"?
up for the dilated cardiomyopathy. The
child presents with dyspnea, cardialgia. A. Up to 4500
There are dense, nonmobile edemata B. Up to 2500
on the lower extremities and sacrum. C. Up to 3500
Ps- 120/min. The cardiac borders are D. Up to 5500
extended transversely. Heart sounds are E. Up to 7500
muffled, there is blowing systolic murmur
at the apex and over the xiphoid process. 22. A 48-year-old patient got a job-related
Liver is 3 cm enlarged, urine output is injury of a hypodermic varicose vein on
reduced. The blood total protein - 58.6 g/l. his shin that was accompanied by the
In urine: protein - 0,025 g/l, WBCs - 2-4 in intensive phleborrhagia. Choose the opti-
the field of vision, RBCs - 2-3 in the field mal variant of first aid:
of vision. What is the main mechanism of
edema syndrome development: A. Pressure bandage and limb strapping
B. Application of Esmarch’s tourniquet
A. Venous congestion of greater circulati- above the injury
on C. Application of Esmarch’s tourniquet
B. Venous congestion of lesser circulation beneath the injury
C. Peripheral circulation disorder D. Occlusion of femoral artery in a typical
D. Secondary nephropathy development place
E. Hypoproteinemia E. Maximal limb flexion in knee joint

19. A 69-year-old female patient complai- 23. A patient who had eaten mushrooms
ns of temperature rise up to 38, 3oC, in the morning was delivered to the
haematuria. ESR - 55 mm/h. Antibacterial infectious diseases hospital at night. The
therapy turned out to be ineffective. What disease development was rapid. The pati-
diagnosis might be suspected? ent presented with stomach pain, frequent
diarrhea, intractable vomiting, burning
thirst, headache and dizziness. He died on
the third day. What mushrooms are most
likely to have caused mycetismus?
Krok 2 Medicine 2011 4

A. Deadly amanita A. Physical


B. Morels B. Chemical
C. Fly agarics C. Biological
D. Sulfur-tufts D. Combined
E. Russules E. Physicochemical
24. A 28-year-old patient complains of 28. 2 weeks after recovering from angina a
periodic compressing heart pain. His 29-year-old patient noticed face edemata,
brother died at the age of 34 from a weakness, decreased work performance.
cardiac disease with similar symptoms. There was gradual progress of dyspnea,
Objectively: the patients skin is pale. edemata of the lower extremities, lumbar
Heart borders display no significant devi- spine. Objectively: pale skin, weakening
ations. Heart sounds are loud, there is a of the heart sounds, anasarca. AP- 160/100
systolic murmur above all the points with mm Hg. In urine: the relative density -
a peak above the aorta. Echocardioscopy 1021, protein - 5 g/l, erythrocytes - 20-30
reveals thickening of the interventricular in the field of vision, hyaline cylinders -
septum in the basal parts, reduction of left 4-6 in the field of vision. What is the most
ventricular cavity. What drug should be likely diagnosis?
administered in order to prevent the di-
sease progression? A. Acute glomerulonephritis
B. Essential hypertension
A. Metoprolol C. Acute pyelonephritis
B. Digoxin D. Infectious allergic myocarditis
C. Nitroglycerin E. Myxedema
D. Captopril
E. Furosemide 29. A 56-year-old scientist experiences
constricting retrosternal pain several ti-
25. A 10-year-old boy underwent mes a day while walking for 100-150 m.
treatment in cardiological department for The pain lasts for up to 10 minutes and can
rheumatism, I acute attack of rheumatic be relieved by nitroglycerine. Objectively:
fever, active phase, II degree. The pati- the patient is overweight, heart borders
ent was discharged in satisfactory condi- exhibit no abnormalities, heart sounds are
tion. Which drug should be chosen for rhythmic, Ps- 78 bpm, AP- 130/80 mm Hg.
prevention of rheumatism recurrence? ECG contains low amplitude of T wave in
V4−5 . What disease might be suspected?
A. Bicillinum-5
B. Bicillinum-1 A. Stable FC III stenocardia
C. Erythromycin B. Instable stenocardia
D. Ampicillin C. Stable FC I stenocardia
E. Oxacillin D. Stable FC II stenocardia
E. Stable FC IV stenocardia
26. Examination of a 13-year-old boy
reveals that his body length is 147 сm 30. In autumn a 25-year-old patient
(+2), body weight - 38 kg (+1,5), ci- developed stomach ache arising 1,5-2
rcumference of chest - 72 cm (+0,2). Esti- hours after having meals and at night.
mate the harmonicity of the child’s physi- He complains of pyrosis and constipation.
cal development: The pain is getting worse after consumi-
ng spicy, salty and sour food, it can be
A. Disharmonious relieved by means of soda and hot-water
B. Harmonious bag. The patient has been suffering from
C. Above the average this disease for a year. Objectively: furred
D. Sharply disharmonious moist tongue. Abdomen palpation reveals
E. Supernormal epigastrial pain on the right, resistance
of abdominal muscles in the same region.
27. A children’s health camp received a What is the most likely diagnosis?
party of tinned food. External examinati-
on of the tins revealed that they had deep A. Duodenal ulcer
dents, could be easily concaved when B. Chronic cholecystitis
pressed and wouldn’t immediately return C. Diaphragmatic hernia
to the initial state; rust was absent; the tins D. Stomach ulcer
were greased with inedible fat. Specify the E. Chronic pancreatitis
bloat type:
31. A full-term infant is 3 days old.
Krok 2 Medicine 2011 5

On the different parts of skin there are suffering from pain in the sacrum and
erythemas, erosive spots, cracks, areas of coxofemoral joints, painfulness and sti-
epidermis peeling. The infant has scalded ffness in the lumbar spine for a year. ESR
skin syndrome. Nikolsky’s symptom is - 56 mm/h. Roentgenography revealed
positive. General condition of the infant symptoms of bilateral sacroileitis. The
is grave. Anxiety, hyperesthesia, febrile patient is the carrier of HLA B27 anti-
temperature are evident. What is the most gen. What is the most likely diagnosis?
probable diagnosis?
A. Ankylosing spondylitis
A. Exfoliative dermatitis B. Coxarthrosis
B. Phlegmon of newborn C. Rheumatoid arthritis
C. Finger’s pseudofurunculosis D. Reiter’s disease
D. Impetigo neonatorum E. Spondylosis
E. Mycotic erythema
36. A 58-year-old female patient complai-
32. A 7-year-old boy has been managed ns about periodical headache, dizziness
for a month. Immediately after hospi- and ear noise. She has been suffering from
talization there were apparent edemata, diabetes mellitus for 15 years. Objecti-
proteinuria - 7,1 g/l, daily urine protein vely: heart sounds are rhythmic, heart rate
- 4,2 g. Biochemical blood test shows is 76/min, there is diastolic shock above
persistent hypoproteinemia (43,2 g/l), aorta, AP is 180/110 mm Hg. In urine:
hypercholesterolemia (9,2 millimole/l). OD- 1,014. Daily loss of protein with uri-
The patient is most likely have the followi- ne is 1,5 g. What drug should be chosen
ng type of glomerulonephritis: for treatment of arterial hypertension?
A. Nephrotic A. Ihibitor of angiotensin converting
B. Nephritic enzyme
C. Isolated urinary B. β-blocker
D. Hematuric C. Calcium channel antagonist
E. Combined D. Thiazide diuretic
E. α-blocker
33. A secundipara has regular birth activi-
ty. Three years ago she had cesarean secti- 37. A 62-year-old patient has been deli-
on for the reason of acute intrauterine vered to the surgical department with
hypoxia. During parodynia she complai- complaints of sudden pain in the umbilical
ns of extended pain in the area of region irradiating to the back and groin,
postsurgical scar. Objectively: fetus pulse which was accompanied by a syncope.
is rhythmic - 140 bpm. Vaginal examinati- Objectively: the patient is in grave condi-
on shows 5 cm cervical dilatation. Fetal tion, umbilical region is tender on palpati-
bladder is intact. What is the tactics of on, bowel sounds are diminished. AP
choice? drop is present. USI reveals: free fluid in
the abdomen, thickening of the wall of
A. Cesarean section the abdominal aorta. The most likely di-
B. Augmentation of labour agnosis is:
C. Obstetrical forceps
D. Waiting tactics of labor management A. Rupture of abdominal aortic aneurism
E. Vaginal delivery B. Stomach ulcer perforation
C. Acute pancreatitis
34. A 54-year-old female patient D. Peritonitis
consulted a doctor about bloody di- E. Acute appendicitis
scharges from the genital tracts after
2 years of amenorrhea. USI and bi- 38. A therapeutist needs to analyze adult
manual examination revealed no genital health in the area of service. Which groups
pathology. What is the tactics of choice? of indicators will be included into this
analysis?
A. Fractional biopsy of lining of uterus and
uterine mucous membranes
B. Styptic drugs
C. Contracting drugs
D. Estrogenic haemostasia
E. Hysterectomy
35. A 32-year-old male patient has been
Krok 2 Medicine 2011 6

A. Demographic, sickness rates, disability A. 3,0 g/kg


B. Demographic, sickness rates, physical B. 2,0 g/kg
development C. 2,5 g/kg
C. Sickness rates, disability, death rates D. 3,5 g/kg
D. Birth rates, sickness rates, disability E. 4,0 g/kg
E. Sickness rates, death rates, physical
development 43. A 60-year-old female patient was
admitted to a hospital for acute
39. A 54-year-old patient has an over transmural infarction. An hour later
20-year history of femoral osteomyelitis. the patient’s contition got worse. She
Over the last month she has developed developed progressing dyspnea, dry
progressing edemata of the lower extremi- cough. Respiratory rate - 30/min, heart
ties. Urine test reveals: proteinuria at the rate - 130/min, AP- 90/60 mm Hg. Heart
rate of 6,6 g/l; in blood: dysproteinemia in sounds were muffled, there was also di-
form of hypoalbuminemia, increase in α2 - astolic shock on the pulmonary artery.
and γ-globulin rate, ESR - 50 mm/h. What The patient presented with medium moi-
is the most likely diagnosis? st rales in the lower parts of lungs on the
right and on the left. Body temperature -
A. Secondary renal amyloidosis 36, 4oC. What drug should be given in the
B. Acute glomerulonephritis first place?
C. Myelomatosis
D. Chronic glomerulonephritis A. Promedol
E. Systemic lupus erythematosus B. Aminophylline
C. Dopamine
40. An emergency team deliverd a 83- D. Heparin
year-old patient complaining of inability E. Digoxin
of her right leg to support the body after
falling on her right side. Objectively: the 44. A 62-year-old male has been hospi-
patient lies on a gurney, her right leg is talized in the intensive care unit with
rotated outwards, the outside edge of foot a continuous attack of retrosternal pain
touches the bed. There is positive straight that cannot be relieved by nitroglycerin.
leg raising sign. What is your provisional Objectively: AP- 80/60 mm Hg, heart rate
diagnosis? - 106/min, breathing rate - 22/min. Heart
sounds are muffled, a gallop rhythm is
A. Femoral neck fracture present. How would you explain the AP
B. Femoral diaphysis fracture drop?
C. Hip dislocation
D. Hip joint contusion A. Reduction in cardiac output
E. Cotyloid cavity fracture B. Reduction in peripheral resistance
C. Blood depositing in the abdominal
41. 3 days ago a 29-year-old patient cavity
presented with pulsating pain in the regi- D. Adrenergic receptor block
on of rectum, fever, general weakness. E. Internal haemorrhage
Objectively: local tenderness in the anal
region in the 6 o’clock position. Digital 45. A patient is on the sick leave for 4
investigation of rectum revealed a pai- months continuously from the date of
nful infiltration reaching the pectinate li- injury. The treatment is going to last for
ne. What is the most likely diagnosis? 1-2 months. Who has the right to extend
the duration of medical certificate for this
A. Acute periproctitis patient?
B. Acute anal fissure
C. Acute haemorrhoids A. Medical advisory commission after
D. Rectum tumour medico-social expert commission exami-
E. Acute prostatitis nation
B. Medical superintendent
42. A child is 7 months old. Birth wei- C. Medical advisory commission after
ght was 3450, the child is breastfed. inpatient treatment
Supplemental feeding was introduced on D. District doctor by agreement with a
time. Determine the daily protein requi- department chief
rements for the child: E. Medico-social expert commission
46. A child is 9 months old. The patient’s
body temperature is 36, 7oC, the skin is
Krok 2 Medicine 2011 7

pale, humid, there is pain in leg muscles. had to undergo a yearly preventive exami-
There is no extremities mobility, sensitivi- nation. A territorial polyclinic carried out
ty is present. The child has been diagnosed preventive examination of 325 workers.
with poliomyelitis. The causative agent of As a result of it, 1 worker was recogni-
this disease relates to the following family: zed as temporarily disabled, 15 workers
underwent further rehabilitation at an
A. Picornavirus after-work sanatorium, 10 workers were
B. Paramyxovirus provided with diet meal. What index
C. Tohovirus characterizing the preventive work of the
D. Adenovirus polyclinic should be applied in this case?
E. Rotavirus
A. Coverage of preventive medical exami-
47. Working conditions of a buildi- nations
ng company worker are characterized B. Frequency of case detection during
by cooling microclimate effect, silica- examinations
containing dust, caustic alkali (quicklime) C. Percentage of people who underwent
and noise. What medical expert should be rehabilitation at an after-work sanatorium
the chief of the commission that periodi- D. Percentage of people who were provi-
cally examines the workers of the menti- ded with diet meal
oned category? E. Percentage of temporarily disabled
people
A. Therapeutist
B. Ophthalmologist 51. A 14-year-old girl has been presenti-
C. Neurologist ng with irritability and tearfulness for
D. Dermatologist about a year. A year ago she was also
E. Otolaryngologist found to have diffuse enlargement of the
thyroid gland (II grade). This condition
48. A full-term baby (the 1st uncompli- was regarded as a pubertal manifestati-
cated pregnancy, difficult labour) had on, the girl didn’t undergo any treatment.
a cephalogematoma. On the 2nd day The girl’s irritability gradually gave place
there was jaundice, on the third the to a complete apathy. The girl got puffy
following changes in neurological status face, soft tissues pastosity, bradycardia,
appeared: nystagmus, Graefe syndrome. constipations. Skin pallor and gland densi-
Urine was yellow, feces were of golden- ty progressed, the skin became of a waxen
yellow colour. Mother’s blood group is A hue. What disease may be suspected?
(II) Rh− , the baby’s one - A (II) Rh+ . On
the third day the child’s Hb was 200g/l, A. Autoimmune thyroiditis
RBCs - 6, 1 · 1012 /l, blood bilirubin - 58 B. Diffuse toxic goiter
micromole/l at the expense of unbound C. Thyroid carcinoma
fraction. What caused the jaundice in the D. Subacute thyroiditis
child? E. Juvenile basophilism
A. Craniocerebral birth trauma 52. 4 hours after having meals a patient
B. Physiological jaundice with signs of malnutrition and steatorrhea
C. Neonatal anaemia experiences stomach pain, especially
D. Biliary atresia above navel and to the left of it. Diarrheas
E. Fetal hepatitis take turns with constipation lasting up to
3-5 days. Palpation reveals moderate pai-
49. After birth a child was pale and had nfulness in the choledochopancreatic regi-
arrhythmical breathing. Oxygen therapy on. The amylase rate in blood is stable.
didn’t have any effect. Pulse was weak X-ray reveals some calcifications located
and rapid. It was difficult to measure above navel. What is the most likely di-
arterial pressure accurately. There were no agnosis?
edemata. What is the most likely reason
for these symptoms? A. Chronic pancreatitis
B. Chronic gastroduodenitis
A. Asphyxia C. Duodenal ulcer
B. Congestive heart failure D. Zollinger-Ellison syndrome
C. Intracranial haematoma E. Chronic calculous cholecystitis
D. Intrauterine sepsis
E. Congenital pneumonia 53. A 56-year-old patient with diffuse
toxic goiter has ciliary arrhythmia with
50. 350 workers of a metalurgical plant
Krok 2 Medicine 2011 8

pulse rate of 110 bpm, arterial hypertensi- afternoon. Pain in the lumbar region,
on, AP- 165/90 mm Hg. What preparati- said about 10 years old. Objectively: pale
on should be administered along with skin, to - 37, 2oC, AP- 180/100 mm Hg,
mercazolil? minor costovertebral angle tenderness
(Pasternatsky symptom). In blood: RBCs
A. Propranolol - 3, 5 · 1012 /l, WBCs - 6, 5 · 109 /l, ESR -
B. Radioactive iodine 22 mm/h. In urine: the relative density -
C. Procaine hydrochloride 1010, leukocytes - 12-15 in the field of visi-
D. Verapamil on, erythrocytes - 2-3 in the field of vision.
E. Corinfar Urine bacterial count - 100000 in 1 ml.
54. On the 3rd day of life a baby What is the most likely diagnosis?
presented with haemorrhagic rash, A. Chronic pyelonephritis
bloody vomit, black stool. Examination B. Nephrolithiasis
revealed anaemia, extended coagulati- C. Polycystic renal disease
on time, hypoprothrombinemia, normal D. Chronic glomerulonephritis
thrombocyte rate. What is the optimal E. Amyloidosis
therapeutic tactics?
58. A 49-year-old male patient complains
A. Vitamin K of dyspnea of combined nature, cough,
B. Sodium ethamsylate shin edemata, abdomen enlargement due
C. Epsilon-aminocapronic acid to ascites. He has a 20-year history of
D. Fibrinogen chronic bronchitis. For the last 3 years
E. Calcium gluconate
he has been disabled (group II) because
55. A woman consulted a doctor on of cardiac changes. Objectively: mixed
the 14th day after labour about sudden cyanosis, edemata. Ps - 92/min, rhythmic,
pain, hyperemy and induration of the left AP - 120/70 mm Hg, respiration rate
mammary gland, body temperature ri- - 24/min. There is accentuation of the
se up to 39o C, headache, indisposition. second sound above the pulmonary artery.
Objectively: fissure of nipple, enlargement Auscultation reveals the box resonance
of the left mammary gland, pain on above the lungs. There are also dry
palpation. What pathology would you thi- rales over the entire surface of lungs.
nk about in this case? What is the mechanism of heart changes
development in this patient?
A. Lactational mastitis
B. Lacteal cyst with suppuration A. Euler-Liljestrand reflex
C. Fibrous adenoma of the left mammary B. Kitaev’s reflex
gland C. Bainbridge reflex
D. Breast cancer D. Cardiovascular reflex
E. Phlegmon of mammary gland E. Respiratory reflex

56. On the 5th day of a respiratory di- 59. A 43-year-old female patient
sease accompanied by fever a 24-year-old complains of unstable defecation with
man developed progressing headaches, frequent constipations, abdominal swelli-
systemic dizziness, double vision, faci- ng, headache, sleep disturbance. Body
al muscles paresis on the right, choking weight is unchanged. What disease are
from swallowing. He was diagnosed with these clinical presentations typical for?
acute viral encephalitis. Identify the main
tendency of the emergency treatment: A. Irritable colon syndrome
B. Chronic enteritis
A. Zovirax C. Chronic pancreatitis
B. Glucocorticoids D. Chronic atrophic gastritis
C. Ceftriaxone E. Colorectal cancer
D. Lasix
60. A 43-year-old man who often contacts
E. Haemodezum
with ethyl gasoline was admitted to
57. A 54-year-old male patient complai- a hospital with complaints of general
ns of aching pain in the lumbar regi- weakness, dizziness, memory impairment,
on, that is getting worse after standi- sleepiness at daytime and insomnia at ni-
ng in an upright position, physical ght, sensation of a hair in the mouth, coli-
exercise, supercooling. The patient also cky pains in the right subcostal region.
reports of experiencing weakness in the What is the most likely diagnosis?
Krok 2 Medicine 2011 9

A. Methotrexate
A. Chronic tetraethyl lead intoxication B. Chloroquine
B. Alcoholic delirium C. Prednisolone
C. Chronic mercury intoxication D. Diclofenac sodium
D. Chronic manganese intoxication E. Aspirin
E. Chronic lead intoxication
65. A 3-year-old child has been deli-
61. A 35-year-old patient has been in the vered to a hospital in soporose state wi-
intensive care unit for acute renal failure th considerable amyotonia, inhibition of
due to crush for 4 days. Objectively: the tendon and periosteal reflexes. Miosis and
patient is inadequate. Breathing rate - asthenocoria are also present. Corneal
32/min. Over the last 3 hours individual reflexes are preserved. Pulse is rapid and
moist rales can be auscultated in lungs. weak. AP- 80/50 mm Hg. The parents
ECG shows high T waves, right ventri- suspect the child of accidental taking
cular extrasystoles. CVP - 159 mm Hg. some tablets. Such clinical presentati-
In blood: the residual nitrogen - 62 mi- ons are typical for intoxication with the
llimole/l, K + - 7,1 millimole/l, Cl− - 78 mi- following tableted drugs:
llimole/l, Na+ - 120 millimole/l, Ht - 0,32,
Hb - 100 g/l, blood creatinine - 0,9 milli- A. Tranquilizers
mole/l. The most appropriate method of B. Antropine drugs
treatment would be: C. Antihypertensive drugs
D. Barbiturates
A. Hemodialysis E. Beta-2-adrenoceptor agonists
B. Plasma sorption
C. Hemosorption 66. A 23-year-old patient complains of a
D. Plasma filtration dull ache, sensation of heaviness and di-
E. Ultrafiltration stention in the epigastrium immediately
after meals, foul-smelling eructation; dry
62. A 47-year-old woman underwent a mouth, empty stomach nausea, diarrhea.
thyroid gland resection on ccount of Objectively: the skin is pale, the patient is
nodular euthyroid goiter. What preparati- of thin build. Abdomen is soft on palpati-
ons are most likely to prevent the disease on, there is epigastric pain. The liver does
recurrence? not extend beyond the costal arch. In
blood: Hb - 110 g/l, RBCs - 3, 4 · 1012 /l,
A. Thyroid hormones WBC count is normal. ESR - 16 mm/h.
B. Mercazolil What is the most informative study that
C. Thyrotropin will allow make a diagnosis?
D. Antistruminum (potassium iodide)
E. Radioactive iodine A. Esophageal gastroduodenoscopy
B. X-ray of digestion organs
63. A 30-year-old man was injured in a fire C. Study of gastric juice
and got thermic burns of III-A and III-B D. pH-metry
degree that affected 20% of the total skin. E. Duodenal probing
AP - 110/70 mm Hg, heart rate -120/min.
What transfusion means should be used 67. A 49-year-old patient complains of
for blind infusion before transportation? deglutition problems, especially with solid
food, hiccups, voice hoarseness, nausea,
A. Saline solutions regurgitation, significant weight loss (15
B. Polyglycine kg within 2,5 months). Objectively: body
C. 10% glucose solution weight is reduced. Skin is pale and
D. Fresh frozen plasma dry. In lungs: vesicular breathing, heart
E. Albumin sounds are loud enough, heart activity is
64. A patient has an over a year-old rhythmic. The abdomen is soft, painless
history of fast progressive rheumatoid on palpation. Liver is not enlarged. What
arthritis. X-raying confirms presence of study is required to make a diagnosis?
marginal erosions. What basic drug would A. Esophageal duodenoscopy along with
be the most appropriate in this case? biopsy
B. Clinical blood test
C. X-ray of digestive tract organs
D. X-ray in Trendelenburg’s position
E. Study of gastric secretion
Krok 2 Medicine 2011 10

68. A 60-year-old patient has been story of chronic glomerulonephritis with


admitted to a hospital with complai- latent course. Over the past six months
nts of dyspnea, tightness in the right she has developed general weakness,
subcostal area, abdomen enlargement. loss of appetite, low work performance,
These presentations have been progressi- nausea. The patient complains of
ng for a year. Heart auscultation reveals headache, pain in the joints. On exami-
presystolic gallop rhythm. Objectively: nation: anemia, blood urea - 34,5 mi-
swelling of the neck veins, ascites, llimole/l, blood creatinine - 0,766 milli-
palpable liver and spleen. What disease mole/l, hyperkalemia. What complication
requires differential diagnostics? has developed?
A. Constrictive pericarditis A. Chronic renal insufficiency
B. Hepatocirrhosis B. Acute renal insufficiency
C. Lung cancer with invasion to the pleura C. Nephrotic syndrome
D. Chronic pulmonary heart D. Renal amyloidosis
E. Pulmonary embolism E. Pyelonephritis
69. A 40-year-old patient, the forester, 72. A child is 1 year old. Ater the recent
complains of severe headache, body introduction of complementary feedi-
temperature rise up to 39, 5oC, trembling ng the child has presented with loss of
limbs. From the patient’s history we know appetite, diarrhea with large amounts
that he had seriously cut his hand duri- of feces and occasional vomiting, body
ng the dissection of a killed fox. Objecti- temperature is normal. Objectively: body
vely: depressed mood. The patient asks weight is 7 kg, the child is very pale, there
not to turn on the light or open the door. are edemata of both legs, abdomen is si-
Any noise causes apparent motor exci- gnificantly enlarged. Coprogram shows
tation. When he saw a carafe of water, many fatty acids and soaps. The child has
he developed convulsive throat spasms. been diagnosed with celiac disease and
What tactics should an emergency doctor administered the gluten-free diet. What is
choose? to be excluded from the ration?
A. Deliver the patient to the infectious A. Cereals - wheat and oats
disease hospital B. Milk and dairy products
B. Deliver the patient to the resuscitation C. Fruit
department D. Animal protein
C. Deliver the patient to the neurological E. High digestible carbohydrates
department
D. Deliver the patient to the psychiatric 73. Development of chronic venous
hospital insufficiency of lower extremities depends
E. Let him stay at home and consult a on the functional condition of so-called
psychiatrist musculovenous pump. This term refers to
the following group of muscles:
70. A 5-year-old child developed an acute
disease starting from body temperature A. Shin muscles
rise up to 38, 5o C, running nose, cough B. Abdominal wall muscles
and conjunctivitis. On the 4th day the chi- C. Buttock region muscles
ld presented with maculo-papular rash D. Thigh muscles
on face. Body temparature rose again E. Foot muscles
up to 39, 2oC. Over the next few days
the rash spread over the whole body and 74. After treating a field with pestici-
extremities. Mucous membrane of palate des a machine operator presents wi-
was hyperemic, there was whitish deposi- th great weakness, headache, nausea,
tion on cheek mucous membrane next vomiting, diarrhea, visual impairment,
to molars. What is your provisional di- watery eyes. Objectively: the patient is
agnosis? excited, hypersalivation, hyperhidrosis,
muscle fibrillation of tongue and eyelids
A. Measles are oberved. Pupils are narrowed, there
B. Acute viral respiratory infection is tachycardia, lung auscultation reveals
C. Yersinia moist small and medium bubbling rales.
D. Enterovirus diseases In blood: changed level of cholinesterase
E. Rubella activity. What is the most likely diagnosis?
71. A 28-year-old woman has a 12-year hi-
Krok 2 Medicine 2011 11

A. Intoxication with organophosphorous A. Reiter’s syndrome


pesticides B. Rheumatic arthritis
B. Intoxication with organochlorine pesti- C. Gout
cides D. Bechterew’s disease
C. Intoxication with organomercurial E. Rheumatoid arthritis
pesticides
D. Intoxication with arsenic-containing 78. Topographic percussion of lungs in
pesticides a patient who got a serious job-related
E. Intoxication with carbamic acid deri- barotrauma revealed that the lower lungs
vatives borders were located one rib below
normal, there was a significant increase
75. A 7-year-old child was brought to in both lungs height and Kronig’s isthmus.
a doctor for a check. The child has What disease should be suspected in the
a 4-year history of bronchial asthma, first place?
asthma attacks occur mainly in spri-
ng and summer. Allergy tests revealed A. Pulmonary emphysema
hypersensitivity to poplar seed tufts, fi- B. Exudative pleuritis
eld herbs. What recommendation should C. Chronic bronchitis
be given? D. Bronchial asthma
E. Pneumothorax
A. Specific hyposensitization
B. Physiotherapy 79. A 32-year-old patient lives in an area
C. Treatment at a health resort endemic for echinococcosis. In the last
D. Phytotherapy 6 months he reports of pain in the ri-
E. Needle reflexotherapy ght subcostal area, fever. He is suspected
to have liver echinococcosis. What study
76. A farmer hurt his right foot duri- would be the most informative in this
ng working in a field and came to the case?
emergency station. He doesn’t remember
when he got last vaccination and he has A. USI
never served in the army. Examination B. Survey radiography of abdominal cavity
of his right foot revealed a contaminated C. Biochemical laboratory examination
wound up to 5-6 cm long with uneven D. Angiography
edges. The further treatment tactics will E. Liver scanning
be:
80. A 30-year-old gravida consulted a
A. To make an injection of tetanus gynecologist about bright red bloody di-
anatoxin and antitetanus serum scharges from the vagina in the 32 week
B. To make an injection of tetanus of gestation. She was hospitalized wi-
anatoxin th a suspicion of placental presentation.
C. To make an injection of antitetanus Under what conditions is it rational to
serum conduct the internal examination in order
D. Surgical d-bridement only to make a diagnosis?
E. To administer an antibiotic
A. In the operating room prepared for the
77. A 35-year-old patient has been admi- operation
tted to a hospital for pain in the left B. In the examination room of antenatal
sternoclavicular and knee joints, lumbar clinic
area. The disease has an acute character C. In the admission ward of maternity
and is accompanied by fever up to 38oC. hospital
Objectively: the left sternoclavicular and D. In the delivery room keeping to all the
knee joints are swollen and painful. In aseptics regulations
blood: WBCs - 9, 5 · 109 /l, ESR - 40 mm/h, E. The examination is not to be conducted
CRP - 1,5 millimole/l, fibrinogen - 4,8 because of risk of profuse haemorrhage
g/l, uric acid - 0,28 millimole/l. Exami- 81. After a serious nervous stress a 35-
nation of the urethra scrapings reveals year-old patient has developed on the
chlamydia. What is the most likely di- dorsal surface of hands redness and
agnosis? swelling that were later replaced by
small inflammatory nodules, vesicles and
following erosion with a significant serous
discharge. The process is accompanied by
severe itching. What is the most likely di-
Krok 2 Medicine 2011 12

agnosis? 85. A 32-year-old patient has a 3-year hi-


story of asthma attacks, that can be hardly
A. True eczema stopped with berotec. Over a few last
B. Allergic dermatitis months he has experienced pain in the
C. Microbal eczema joints and sensitivity disorder of legs and
D. Simple contact dermatitis feet skin. Ps - 80/min, AP - 210/100 mm
E. Toxicoderma Hg. In blood: eosinophilia at the rate of
15%. What disease can be suspected in
82. A 36-year-old patient complains of this case?
skin rash that appeared a week ago and
doesn’t cause any subjective problems. A. Periarteritis nodosa
Objectively: palm and sole skin is covered B. Systemic lupus erythematosus
with multiple lenticular disseminated C. Systemic scleroderma
papules not raised above the skin level. D. Dermatomyositis
The papules are reddish, dense on palpati- E. Wegener’s disease
on and covered with keratinous squamae.
What is the provisional diagnosis? 86. A 46-year-old patient complains of
sudden palpitation, which is accompani-
A. Secondary syphilis ed by pulsation in the neck and head, fear,
B. Verrucosis nausea. The palpitation lasts for 15-20 mi-
C. Palmoplanar psoriasis nutes and is over after straining when
D. Palmoplanar rubrophytosis holding her breath. What kind of cardi-
E. Palm and sole callosity ac disorder may be suspected?
83. A 30-year-old patient complains of A. An attack of supraventricular
paroxysmal abdominal pain, frequent paroxysmal tachycardia
liquid stools up to 10 times a day. B. An attack of ventricular paroxysmal
Throughout the first 3 days he had tachycardia
a fever, since the 2nd day of disease C. An attack of atrial flutter
there were scant liquid stools mixed wi- D. An attack of ciliary arrhythmia
th mucus. On palpation: tenderness of E. An attack of extrasystolic arrhythmia
all colon segments. Sigmoid colon was
found spastic. What is your provisional 87. The patient complains of a painful
diagnosis? swelling in the chin region, malaise,
headache. Examination reveals an acutely
A. Acute dysentery inflamed cone-shaped dense node. The
B. Intestinal amebiasis skin over it is tense, red. In the center of
C. Salmonellosis the node there is an ulcer with overhangi-
D. Cholera ng edges and a necrotic core of a dirty-
E. Balantidiasis green colour. Submandibular lymph nodes
on the right are enlarged and painful.
84. A 38-year-old woman experiences epi- What is the most likely diagnosis?
sodic increases in arterial pressure up to
240/120 mm Hg, which is accompanied by A. Furuncle
nausea, vomiting, tachycardia, increased B. Tuberculosis
sweating, hyperglycemia. The attack is C. Carbuncle
usually followed by the excessive urinati- D. Tertiary syphilis (gummatous form)
on. Renal sonography reveals an additi- E. Parasitic sycosis
onal formation adjacent to the upper pole
of the right kidney and possibly belonging 88. A welder at work got the first-degree
to the adrenal gland. What laboratory test burns of the middle third of his right
will allow to clarify the diagnosis? shin. 5 days later the skin around the
burn became edematic and itchy. Objecti-
A. Determination of urinary excretion of vely: on a background of a well-defined
catecholamines and vanillylmandelic acid erythema there is polymorphic rash in
B. Blood test for insulin and C-peptide form of papules, vesicles, pustules, erosi-
C. Estimation of glomerular filtration ons with serous discharge. What is the
rate by measuring endogenous creatinine most likely diagnosis?
clearance
D. Blood test for thyroxine and
thyrotrophic hormone
E. Blood test for renin level
Krok 2 Medicine 2011 13

A. Microbal eczema some painless nodular elements tending


B. True eczema to peripheral growth and fusion. He has a
C. Toxicoderma 2-year history of this disease. Aggravation
D. Occupational eczema takes place mainly in spring. In anamnesis:
E. Streptococcal impetigo the patient’s father had similar skin lesi-
ons. Objectively: pathological elements
89. A 58-year-old patient has a 3-year hi- looke like guttate and nummular nodules,
story diabetes mellitus type II. He has plaques covered with white scales. What
been keeping to a diet and regularly taki- is your provisional diagnosis?
ng glyburide. He has been delivered to
a hospital on an emergency basis for A. Psoriasis
acute abdomen. Objectively: the patient B. Lichen ruber planus
is of supernutrition type. The skin is dry. C. Neurodermitis
In the lungs vesicular breathing can be D. Pityriasis rosea
auscultated. Heart sounds are regular, E. Seborrheic eczema
90/min. AP- 130/70 mm Hg. The symptom
of "wooden belly"is visible. Blood sugar 93. A 47-year-old patient came to see a
- 9,8 millimole/l. The patients has indi- doctor on the 7th day of disease. The di-
cation for laparotomy. What is the most sease developed very fast: after the chill
appropriate way of further treatment of body temperature rose to 40oC and lasted
diabetes? up to 7 hours, then dropped abruptly,
which caused profuse sweat. There were
A. To administer short insulin three such attacks occuringonce in two
B. To continue taking glyburide days. Two days ago the patient arrived
C. To administer Semilong to be taken in from Africa. Objectively: pale skin, subi-
the morning and insulin - in the evening cteric sclera, significantly enlarged liver
D. To administer 1 tablet of Glurenorm and spleen. What is the cause of fever
three times a day attacks in this disease?
E. To administer 1 tablet of Maninil three
times a day A. Erythrocytic schizogony
B. Tissue schizogony
90. An 8-year-old girl has been admitted C. Exotoxin of a causative agent
to the cardiology department. Objecti- D. Endotoxin of a causative agent
vely: there is a skin lesion over the E. Gametocytes
extensor surfaces of joints with atrophic
cicatrices, depigmentation, symmetrical 94. On the 2 nd day of illness a 27-year-
affection of skeletal muscles (weakness, old patient complains of the unbearable
edema, hypotrophy). What disease are headache, repeated vomiting. Objecti-
these changes most typical for? vely: the patient is in a grave condition.
He is conscious but adynamic. Lies in a
A. Dermatomyositis forced position with his head thrown back.
B. Systemic scleroderma There is no skin rash. Occipital muscles
C. Nodular periarteritis are evidently rigid, there are Kernig’s
D. Systemic lupus erythematosus and Brudzinski’s signs. to - 39, 5o C, Ps-
E. Reiter’s disease 120/min., AP- 130/80 mm Hg. What is the
reason for the leading syndrome of this
91. A 43-year-old female patiet complai- disease?
ns of eruption on her right leg skin, pain,
weakness, body temperature rise up to A. Liquor hypertension
38o C. The disease is acute. Objectively: B. Liquor hypotension
there is an edema on the right leg skin in C. Affection of the cranial nerve nuclei
the region of foot, a well-defined bright D. Haemorrhages into the adrenal glands
red spot in form of flame tips which feels E. Hyperthermy
hot. There are isolated vesicles in focus.
What is your provisional diagnosis? 95. On the 1st day of life a full-term girl
(2nd labour) weighing 3500g, with Apgar
A. Erysipelas score of 8 points, presented with jaundice.
B. Microbial eczema Indirect bilirubin of blood - was 80 mi-
C. Contact dermatitis cromole/l, 6 hours later - 160 micromole/l.
D. Toxicoderma What is the optimal method of treatment?
E. Haemorrhagic vasculitis
92. A 45-year-old patient complains of
Krok 2 Medicine 2011 14

A. Exchange blood transfusion A. X-ray of chest


B. Phototherapy B. Clinical blood test
C. Infusion therapy C. Determination of blood gas compositi-
D. Phenobarbital treatment on
E. Enterosorbents D. Proteinogram
E. Immunoassay
96. A 42-year-old female patient suffers
from micronodular cryptogenic cirrhosis. 100. A children’s surgical unit admi-
Over the last week her condition has tted a 1-month-old boy who had been
deteriorated: there appeared convulsi- prenatally diagnosed with the left-sided
ons, mental confusion, increased jaundice. pyelectasis. Such studies as drip infusion
What study may explain such aggravati- urography, cystography and USI allowed
on? to reveal initial hydronephrosis. There is
no information confirming the secondary
A. Determination of serum ammonia pyelonephritis. What tactics of this patient
B. Determination of cholesterol ethers management is most advisable?
C. Determination of alpha-phetoprotein
rate A. 6-month surveillance
D. Determination of ALAT and ASAT B. Urgent nephrostomy
E. Determination of alkaline phosphatase C. Anderson-Hynes operation
rate D. There is no need in further surveillance
and treatment
97. A 30-year-old patient was hospitalized E. Antibacterial therapy
with a diagnosis: intestinal obstruction.
During the surgery it was revealed that 101. A 24-years-old patient has chronic
the obstruction of the small intestine had glomerulonephritis.Urine test reveals the
been caused by a mass of helminths. What following: the relative density is 1010,
helminths are these? protein - 1,65 g/l, RBCs - 5-7 in the field
of vision, WBCs - 2-3 in the field of visi-
A. Ascarids on. Blood creatinine - 0,350 millimole/l.
B. Guinea worms Serum sodium - 148 millimole/l. What is
C. Filarial worms the main reason for hyperazotemia in this
D. Cysticerci patient?
E. Pinworms
A. Reduction of glomerular filtration rate
98. On the 5th day after a surgery for B. Reduction of tubular reabsorption rate
colon injury a patient complains of bursti- C. Increased proteinuria
ng pain in the postoperative wound, D. Reduction of renal blood flow
weakness, drowsiness, headache, fever up E. Sodium retention in the organism
to 40o C. Objectively: the skin around the
wound is swollen, there is gas crepitati- 102. After an accident a patient complains
on. The wound discharges are scarce foul- of pain in the hip joint. Objectively: the
smelling, of dark-gray colorl. What is the leg is in the position of flexion, adducti-
most likely diagnosis? on and internal rotation, significantly
contracted. There is elastic resistance to
A. Anaerobic clostridial wound infection passive adduction or abduction of the
B. Abscess extremity. Major trochanter is located hi-
C. Postoperative wound infection gh above the Roser-Nelaton line. A si-
D. Erysipelas gnificant lordosis is present. What is your
E. Phlegmon provisional diagnosis?
99. A child was born at a gestational A. Iliac dislocation of hip
age of 34 weeks in grave condition. The B. Femoral neck fracture with a di-
leading symptoms were respiratory di- splacement
stress symptoms, namely sonorous and C. Cotyloid cavity fracture with a central
prolonged expiration, involving additional dislocation of hip
muscles into respiratory process. The Si- D. Inferoposterior dislocation of hip
lverman score at birth was 0 points, in 3 E. Pertrochanteric fracture of hip
hours it was 3 points with clinical findings.
Which diagnostic study will allow to di- 103. As a result of prolonged exposure
agnose the form of pneumopathy? to the sun a 20-year-old patient has
developed low-grade fever, pain and
swelling in the knee and ankle joints,
Krok 2 Medicine 2011 15

erythema on her face and nose bridge, are hypoechogenic nodes in both lobes,
leukopenia and accelerated ESR. She capsule invasion). The rate of prostate-
has been provisionally diagnosed wi- specific antigen is 60 ng/l. Prostate biopsy
th systemic lupus erythematosus. What revealed an adenocarcinoma. Which of
pathognomonic laboratory data may the supplemental examination methods
confirm this diagnosis? will allow to determine the stage of
neoplastic process in this patient?
A. Antinuclear factor
B. Accelerated ESR A. Computer tomography of pelvis
C. C-reactive protein B. Roentgenography of lumbar spine
D. Anaemia C. Excretory urography
E. Lymphocytosis D. Bone scintigraphy
E. Roentgenography of chest
104. Examination of the corpse of a man
who died from hanging reveals: death 108. While staying in a stuffy room a 19-
spots disappear when pressed upon and year-old emotionally labile girl developed
restore after 50 seconds, rigor mortis is severe weakness, dizziness, blackout,
moderately expressed only in the masti- nausea and loss of consciousness without
catory muscles as well as neck and finger convulsions. Objectively: the patient is
muscles, body temperature is of 31o C. The unconscious, the skin is pale, extremities
time of death: are cold. AP - 90/60 mm Hg, Ps- 96/min,
deficient, breathing is shallow. Pupillary
A. 6-7 hours ago and tendon reflexes are present. There are
B. 1-2 hours ago no pathological signs. What is the most li-
C. 16-24 hours ago kely diagnosis?
D. 8-10 hours ago
E. 10-18 hours ago A. Syncope
B. Vegetovascular paroxysm
105. After contusion of the right eye a C. Epileptic attack
patient complains of sudden loss of vision D. Hysterical neurosis
with remaining light perception. Objecti- E. Transient ischemic attack
vely: the eye is not irritated. The cornea
is transparent. Pupil reacts to light. The 109. The air of a foundry worker’s working
pupil area is black. The fundus reflex is zone contains condensation aerosol with
absent. What is the most likely cause of dust particles sized 2 nm (90%), 2-5 nm
vision loss? (2%), over 5 nm(6%), below 2 nm (about
2%). Characterize the dust dispersivity:
A. Hemophthalmia
B. Retinal detachment A. Fine-dispersed
C. Traumatic cataract B. Median-dispersed
D. Acute occlusion of retinal vessels C. Coarsely dispersed
E. Optic nerve avulsion D. Ultrafine-dispersed
E. Mist
106. A 68-year-old patient consulted
a doctor about a tumour in her left 110. A 40-year-old female patient
breast. Objectively: in the upper internal complain of headache, dizziness, muscle
quadrant of the left breast there is a weakness, sometimes - cramps in the
neoplasm up to 2,5 cm in diameter, dense, extremities. She has been taking anti-
uneven, painless on palpation. Regional hypertensive medications for 10 years.
lymph nodes are not enlarged. What is the AP- 180/100 mm Hg. Blood potassium -
most likely diagnosis? 1,8 millimole/l, sodium - 4,8 millimole/l. In
urine: alkaline reaction, the relative densi-
A. Cancer ty - 1012, protein and sugar are not found,
B. Cyst WBCs - 3-4 in the field of vision, RBCs - 1-
C. Fibroadenoma 2 in the field of vision. Conn’s syndrome is
D. Mastopathy suspected. Which drug should be chosen
E. Lipoma for the treatment of arterial hypertensi-
107. A 65-year-old patient complains on?
of pain in the lumbar spine, moderate
disuria. He has been suffering from
these presentations for about half a
year. Prostate volume is 45 cm3 (there
Krok 2 Medicine 2011 16

A. Spironolactone A. Warming procedures


B. Propanolol B. Dehydrating drugs
C. Enalapril C. Analgetics
D. Hydrochlorothiazide D. Vitamins of B group
E. Clonidine E. Intravenous injection of aminophylline
111. A patient is 30 years old, works 115. The institutions which take part in
as a carpenter. Six months ago there carrying out medical examinations can be
appeared some behavioural changes: he prevention and treatment facilities, medi-
got interested in philosophy, began writi- cal board of Ministry of Defense, medi-
ng a treatise on the purpose of his human cal board of Ministry of Home Affai-
existence, quitted his job, stopped cari- rs, medico-social expert commissions,
ng about his children, went out carelessly forensic medical boards etc. What insti-
dressed, heard "voices in his head"that tutions are responsible for temporary di-
guided his behaviour. The patient claimed sability examination?
sure that he was an Ambassador of God
on Earth and was constantly feeling His A. Prevention and treatment facilities
influence. He is not critical about his di- B. Sanitary-and-prophylactic institutions
sease. What diagnosis can be assumed? C. Medico-social expert commissions
D. Medical boards of Ministry of Defense
A. Schizophrenia E. Medical boards of Ministry of Home
B. Alcocholic psychosis Affairs
C. Reactive psychosis
D. Somatogenic psychosis 116. 10 minutes after delivery a woman
E. Organic psychosis discharged placenta with a tissue defect
5х6 cm large. Discharges from the genital
112. A 27-year-old patient complains of tracts were profuse and bloody. Uterus
nasal haemorrhages, multiple bruises on tonus was low, fundus of uterus was
the anterior surface of the trunk and located below the navel. Examination
extremities, sudden weakness. In blood: of genital tracts revealed that the uteri-
Hb- 74 g/l, reticulocytes - 16%, RBCs - ne cervix, vaginal walls, perineum were
2, 5 · 1012 /l, platelets - 30 · 109 /l, ESR- 25 intact. There was uterine bleeding with
mm/h. What is the most effective measure following blood coagulation. Your actions
for the treatment of thrombocytopenia? to stop the bleeding:
A. Splenectomy A. To make manual examination of uterine
B. Iron preparations cavity
C. Hemotransfusion B. To apply hemostatic forceps upon the
D. Cytostatics uterine cervix
E. Vitamin B12 C. To introduce an ether-soaked tampon
into the posterior fornix
113. 2 days ago a patient presented with D. To put an ice pack on the lower
acute pain in the left half of chest, general abdomen
weakness, fever and headache. Objecti- E. To administer uterotonics
vely: between the 4 and 5 rib on the left
the skin is erythematous, there are multi- 117. While lifting a heavy load a 39-
ple groups of vesicles 2-4 mm in diameter year-old patient suddenly felt a severe
filled with transparent liquid. What diease headache, pain in the interscapular regi-
are these symptoms typical for? on,and started vomiting. Objectively: the
pulse is rhythmic, 60/min., AP - 180/100
A. Herpes zoster mm Hg. The patient is agitated. He
B. Pemphigus presents with photophobia, hyperacusis.
C. Herpes simplex There are positive Kernig’s and Brudzi-
D. Streptococcal impetigo nski’s signs on both sides. In blood: WBCs
E. Herpetiform Duhring’s dermatosis - 10 · 109 /l. CSF is bloody, cytosis is 240/3.
What is the most likely diagnosis?
114. After lifting a load a patient felt
undurable pain in the loin. He was di- A. Subarachnoid haemorrhage
agnosed with acute lumbosacral radiculi- B. Sympathoadrenal crisis
tis. Which of the following is contraindi- C. Acute hypertonic encephalopathy
cated for this patient? D. Meningococcal meningitis
E. Ischemic stroke
Krok 2 Medicine 2011 17

118. A surgeon examined a 42-year- A. Pneumonic plaque


old patient and diagnosed him with ri- B. Miliary tuberculosis
ght forearm furuncle at the purulo- C. Influenza
necrotic stage. The furuncle was lanced. D. Ornithosis
At the hydration stage the wound dressing E. Sepsis
should enclose the following medication:
122. HIV displays the highest tropism
A. Hypertonic solution towards the following blood cells:
B. Vishnevsky ointment
C. Ichthyol ointment A. T-helpers
D. Chloramine B. T-suppressors
E. Dimexide C. T-killers
D. Thrombocytes
119. After a 10-year-old child had been E. Erythrocytes
bitten by a bee, he was delivered to
a hospital. There were lip, face and 123. Educational rooms are illuminated
neck edemata. The patient felt hot and with various lighting fittings. What type of
short of breath. Objectively: breathi- lighting fittings is the most appropriate in
ng was laboured and noisy. There were respect of hygienic norms?
foamy discharges from the mouth, cough. A. Indirect light fittings
The skin was pale and cold. There B. Direct light fittings
was bradypnoea. Heart sounds were C. Semi-reflected light fittings
muffled and arrhythmic. Thready pulse D. Ambient light fittings
was present. What diagnosis was made by E. Combined light fittings
the expert in resuscitation?
124. A 24-year-old female patient
A. Anaphylactic shock complains of acute pain in the lower
B. Quincke’s edema abdomen that turned up after a physical
C. Bronchial asthma stress. She presents with nausea, vomiting,
D. Acute cardiovascular collapse dry mouth and body temperature 36, 6o C.
E. Cerebral coma She has a right ovarian cyst in history. Bi-
120. Half an hour after a 30-year-old manual examination reveals that uterus
woman had had some custard cake, is dense, painless, of normal size. The left
she experienced lancinating abdominal fornix is deep, uterine appendages aren’t
pain, nausea, vomiting. Objectively: body palpable, the right fornix is contracted.
temperature - 36, 0oC, pale skin, breathi- There is a painful formation on the right
ng rate - 20/min, Ps- 100/min. AP- 95/65 of uterus. It’s round, elastic and mobile.
mm Hg, loud cardiac sounds. Dry tongue. It is 7х8 cm large. In blood: leukocytosis
Abdomen was painful in its epigastrial with the left shit. What is the most likely
part, there were no signs of peritoneum diagnosis?
irritation. What is the first measure to be A. Ovarian cyst with pedicle torsion
taken? B. Right-sided pyosalpinx
A. Gastric lavage C. Subserous fibromyoma of uterus
B. Administration of an enterosorbent D. Acute metritis
C. Injection of Cerucal E. Extrauterine pregnancy
D. Intravenous rehydratation 125. A 39-year-old patient complains of
E. Antibiotic therapy a tumour on the anterior surface of her
121. A 45-year-old patient, a sailor, was neck. The tumour has been observed for
hospitalized on the 2nd day of the di- 2 years. It is nonmobile and has enlarged
sease. A week ago he returned from recently. The patient has a changed tone
India. Complains of body temperature of of voice, a sense of pressure. Objectively:
41o C, severe headache, dyspnea, cough in the left lobe of the thyroid gland a
with frothy rusty sputum. Objectively: 3 cm node is palpable; it is very dense,
the patient is pale, mucous membranes tuberous, painless. Cervical lymph nodes
are cyanotic, breathing rate - 24/min, are enlarged. Functional status of the
tachycardia is present. In lungs: dimini- thyroid gland is unchanged. What is the
shed breath sounds, moist rales over both most likely diagnosis?
lungs, crepitation. What is the most likely
diagnosis?
Krok 2 Medicine 2011 18

A. Thyroid gland cancer A. Bronchography


B. Nodular euthyroid goiter B. Bronchoscopy
C. Nodular hyperthyroid goiter C. Survey radiograph of lungs
D. Chronic lymphomatous Hashimoto’s D. Pleural puncture
thyroiditis E. Bacteriological analysis of sputum
E. Chronic fibrous Riedel’s thyroiditis
129. A 49-year-old patient complai-
126. A 22-year-old patient is a clerk. His ns of dyspnea, cough. There are no
working day runs in a conditioned room. sputum discharges. He has repeatedly
In summer he was taken by an acute di- used salbutamol and intal but with no
sease with the following symptoms: fever, effect. Objectively: he is only able to sit
dyspnea, dry cough, pleural pain, myalgia, while leaning on the table. Cyanosis of
arthralgia. Objectively: moist rales on the face, acrocyanosis are present. Breathi-
right, pleural friction rub. X-ray picture ng is shallow, laboured, in some parts it
showed infiltration of the inferior lobe. In cannot be auscultated; there are diffuse
blood: WBC - 11 · 109 /l, stab neutrophi- rales, expiration is significantly prolonged.
ls - 6%, segmented neutrophils - 70%, Heart sounds are muffled, tachycardia is
lymphocytes - 8%, ESR - 42 mm/h. What present. Ps - 112/min., AP - 110/70 mm
is the ethiological factor of pneumonia? Hg. Liver is located near the costal arch.
There are no peripheral edemata. What is
A. Legionella your provisional diagnosis?
B. Mycoplasm
C. Streptococcus A. Status asthmaticus
D. Staphylococcus B. Chronic obstructive bronchitis
E. Pneumococcus C. Bronchiale asthma, moderate gravity
D. Foreign object aspiration
127. A 53-year-old woman complained of E. Cardiac asthma
cardiac pain and rhythm intermissions.
She had experienced these presentations 130. A 3-year-old girl presents with
since childhood. The patient’s father had pertussis-like cough with thick sputum.
a history of cardiac arrhythmias. Objecti- There have been persistent changes in
vely: the patient was in grave condition, Ps lungs since the age of 6 months when she
- 220 bpm, AP - 80/60 mm Hg. ECG: heart was first diagnosed with acute pneumonia.
rate - 215/min, widening and deformati- Chloride concentration in the perspirati-
on of QRS complex accompanied by atri- on is 112 mEq/l. The child has been di-
oventricular dissociation; positive P wave. agnosed with mucoviscidosis. What is the
Some time later heart rate reduced down basis for autosomal recessive disease -
to 45/min, there was a complete dissociati- mucoviscidosis?
on of P wave and QRST complex. Which
of the following will be the most effective A. Inadequate transport of sodium and
treatment? chloride ions
B. α1 -antitrypsin deficiency
A. Implantation of the artificial pacemaker C. Deposition of calcium triphosphates
B. β-adrenoreceptor blocking agents and carbotates in the alveoles
C. Cholinolytics D. Pulmonary cysts
D. Calcium antagonists E. Pulmonary artery hypoplasia
E. Cardiac glycosides
131. A parturient woman is 23 years
128. A 47-year-old patient complains old. Vaginal obstetric examination reveals
about cough with purulent sputum, pain full cervical dilatation. There is no fetal
in the lower part of the left chest, peri- bladder. Fetal head is in the plane of
odical body temperature rise. She has pelvic outlet. Sagittal suture is in mesati-
been suffering from these presentations pellic pelvis, anterior fontanel is closer to
for about 10 years. Objectively: "drumsti- pubes. The fetal head diameter in such
ck"distal phalanges. What examination presentation will be:
would be the most informative for making
a diagnosis? A. Suboccipito-bregmaticus
B. Fronto-occipitalis recta
C. Biparietal
D. Suboccipitio-frontalis
E. Mento-occipitalis
132. A 42-year-old woman has had
Krok 2 Medicine 2011 19

hyperpolymenorrhea and progressing A. Determination of blood calcium and


algodismenorrhea for the last 10 years. phosphor
Gynaecological examination revealed no B. Determination of thyrotropic hormone
changes of uterine cervix; discharges are C. Determination of potassim
moderate, of chocolate colour, uterus is D. Determination of sodium
slightly enlarged and painful, appendages E. Determination of thyroid hormones
are not palpable, the fornices are deep
and painless. What is the most likely di- 136. A patient with bilateral hydrothorax
agnosis? has repeatedly undergone pleural
puncture on both sides. After a regular
A. Uterine endometriosis puncture the patient’s condition has
B. Uterine carcinoma become worse: he presents with fever,
C. Subserous uterine fibromyoma chest pain. The next day, the attending
D. Endomyometritis physician performing pleural puncture
E. Adnexal endmetriosis revealed some pus on the right. What
is the mechanism of acute right-sided
133. Examination of a 38-year-old pati- empyema development?
ent who had been hit with a blunt object
on the left side of chest revealed a A. Contact-and-aspiration
fracture of the X rib with fragments di- B. Lymphogenous
splacement, parietal pneumothorax. The C. Hematogenous
patient complains of pain in the left D. Implantation
subcostal area. Objectively: the patient E. Aerial
is pale, AP- 80/40 mm Hg, Ps- 138/min,
of poor volume. USI reveals fluid in the 137. A student analyzes noise level of
left abdomen. Splenic rupture is present. cold-pressing process. What device should
What treatment tactics should be chosen? be applied for this hygienic study?

A. Drainage of the left pleural cavity and A. Noise and vibration analyzer
laparotomy B. Noise analyzer
B. Immediate upper middle laparotomy C. Sound tester
and following drainage of the left pleural D. Actinometer
cavity E. Pyranometer
C. Immediate laparotomy and alcohol- 138. Environmental pollution is prevented
novocaine block of the X rib by mechanical separation of nontoxic
D. Anti-schock actions and laparotomy solid domestic waste. Specify the method
following the arterial pressure rise which can be used for mechanical utili-
E. Left-sided thoracotomy and immedi- zation of these wastes:
ately following laparotomy
A. Compressing of wastes into building
134. 15 minutes after the second vacci- blocks
nation with DTP vaccine a 4-month-old B. Hydrolysis
boy exhibited the symptoms of Quincke’s C. Burning as power-plant fuel
edema. What medication should be given D. Burial of wastes
for emergency aid? E. Waste neutralization in biothermal
A. Prednisolone boxes
B. Heparin 139. A 26-year-old woman complains of
C. Adrenalin having bloody discharges from the geni-
D. Furosemide tals for the last 14 days, abdominal pain,
E. Seduxen general fatiguability, weakness, weight
135. On the 2nd day after a surgery for loss, fever, chest pain, obstructed respirati-
toxic mixed goiter IV a 35-year-old pati- on. 5 weeks ago she underwent an induced
ent complains of heart pain. ECG shows abortion in the 6-7 week of gestation.
prolonged QT intervals. Chvostek’s and Objectively: the patient is pale and inert.
Trousseau symptoms cannot be clearly Bimanual examination revealed that the
defined. The patient is provisionally di- uterus was enlarged up to 8-9 weeks of
agnosed with latent tetany. What study gestation. In blood: Hb - 72 g/l. Urine
will allow to confirm the diagnosis? test for chorionic gonadotropin gave the
apparently positive result. What is the
most likely diagnosis?
Krok 2 Medicine 2011 20

A. Chorioepithelioma A. Poliomyelitis
B. Metroendometritis B. Diphtheria and tetanus
C. Uterus perforation C. Measles, rubella, parotitis
D. Uterine fibromyoma D. Pertussis
E. Uterine carcinoma E. Type B hepatitis
140. A 28-years-old woman complains of 144. A 40-year-old patient underwent an
nausea and vomiting about 10 times per operation for a lumbar phlegmon. Body
day. She has been found to have body temparature rose again up to 38o C, he
weight loss and xerodermia. The pulse is got intoxication symptoms, there was an
100 bpm. Body temperature is 37, 2oC. increase of leukocyte number in blood.
Diuresis is low. USI shows 5-6 weeks of The wound that was nearly free from
pregnancy. What is the most likely di- necrotic tissues and full of granulations
agnosis? started to discharge pus, the granulations
turned pale. What complication dveloped
A. Moderate vomiting of pregnancy in this patient?
B. Mild vomiting of pregnancy
C. I degree preeclampsia A. Sepsis
D. Premature abortion B. Putrid phlegmon
E. Food poisoning C. Erysipelas
D. Allergic reaction
141. A full-term baby was born with body E. Erysipeloid
weight of 3200 g, body length of 50 cm,
Apgar score - 8-10 points. What is the 145. After a car accident a 37-year-old
optimum time for the first breast-feeding? patient has an acute pain and bleeding in
the middle third of his right thigh. Objecti-
A. First 30 minutes vely: there is a wound on the anterior
B. First 6 hours surface of the right thigh with massive
C. First 24 hours bleeding, abnormal mobility at the level
D. First 48 hours of the middle third of the thigh. The first
E. After 48 hours aid is to be started with:
142. A 26-year-old patient with left lower A. Digital occlusion of the femoral artery
lobe pneumonia experiences an acute B. Injection of pain-killers
chest pain on the left during coughing. C. Tourniquet application
Objectively: diffuse cyanosis, widening of D. Immobilization of the extremity with a
the left half of chest. Percussion reveals transportation splint
high tympanitis. Auscultation reveals no E. Venipuncture and intravenous infusion
respiratory murmurs above the left half of polyglycine
of chest. There is a deviation of the right
cardiac border towards the midclavicular 146. A 9-year-old boy fell from a tree
line. What examination will be the most and hit the occipital region, there was a
informative? momentary loss of consciousness. Objecti-
vely: the child’s condition is satisfactory,
A. X-Ray he complains of the headache and dizzi-
B. Bronchoscopy ness. The X-ray of skull reveals a commi-
C. Bronchography nuted depressed fracture of occipital bone
D. Pneumotachometry in the region of inion. What treatment is
E. Spirography indicated for the patient?
143. A 3-year-old child has been taken A. Surgical intervention
to a pediatrician. He has no recent hi- B. Anti-inflammatory therapy
story of any diseases. Objective exami- C. Hemostatic therapy
nation revealed no pathology of the D. Therapeutic lumbar punctures
internal organs. The child needs the routi- E. Complex conservative treatment
ne immunization against the following di-
sease: 147. A 40-year-old female patient has a hi-
story of rheumatism. She complains about
acute pain in her left eye, especially at
night, vision impairment, photophobia,
lacrimation. The patient cannot suggest
any reasons for the disease. Objectively:
weak pericorneal injection, flattening of
Krok 2 Medicine 2011 21

iris relief, iris discoloration. What is the ent felt sharp abdominal pain irradiating
most likely diagnosis? to the right scapula, there was a single
vomiting. She has a history of rheumatoid
A. Iridocyclitis arthritis. Objectively: pale skin, AP-
B. Iritis 100/60 mm Hg, Ps- 60/min. Abdomen is
C. Keratitis significantly painful and tense in the epi-
D. Choroiditis gastrium and right subcostal areat, there
E. Acute attack of glaucoma are positive symptoms of parietal peri-
toneum irritation over the right costal
148. 14 days ago a 49-year-old patient arch, that is tympanitis. What is the tactics
was operated on for perforated appendi- of an emergency doctor?
citis and disseminated fibrinopurulent
peritonitis. The postoperative period was A. To take the patient to the surgical
uneventful. On the 9th day the patient hospital
presented with low-grade fever, abdomi- B. To inject pain-killers and watch the
nal pain, frequent liquid stools. USI of the dynamics
left mesogastrium reveals a fluid formati- C. To perform gastric lavage
on 9x10 cm large. In blood: leukocytosis D. To inject spasmolytics
with the left shift. What is your provisi- E. To take the patient to the rheumatologi-
onal diagnosis? cal department
A. Interloop abscess 152. A 72-year-old patient complains of
B. Abdominal cavity tumour pain and bleeding during defecation. Digi-
C. Liver abscess tal rectal investigation revealed a tumour
D. Left kidney cyst of the anal canal. After verification of the
E. Spleen abscess diagnosis the patient was diagnosed with
squamous cell carcinoma. The secondary
149. The Carpathian region is characteri- (metastatic) tumour will be most probably
zed by constant high humidity of
found in:
atmospheric air (over 80%). Inhabi-
tants of this region feel severe cold in A. Lungs
corresponding season at a medium low B. Liver
temperature. It’s caused by heat emission C. Pelvic bones
by: D. Mediastinum
E. Brain
A. Convection
B. Radiation 153. A patient has a stab wound on his ri-
C. Vaporization ght foot. On the fourth day after injury
D. Conduction the patient’s body temperature rose up
E. - to 38o C, inguinal lymph nodes became
enlarged and painful, skin over them
150. A 22-year-old female patient reddened. What complication might be
complains of dull pain in her right iliac suspected?
area that she has been experiencing for
a week, morning sickness and gustatory A. Lymphadenitis
change. She has a histrory of menstruati- B. Lymphangitis
on delay for 3 weeks. Objectively: AP- C. Phlegmon
80/50 mm Hg, pulse is 78 bpm, body D. Tetanus
temperature is 37o C. Bimanual examinati- E. Erysipelas
on reveals that uterus is enlarged, soft,
mobile and painless. Uterine appendages 154. A 25-year-old female patient
are palpable on the right, there is a dense, complains about having amenorrhea
elastic and moderately painful formation for 3 years. She associates it with di-
3x4 cm large. What is the most likely di- fficult labour complicated by massive
agnosis? hemorrhage. She also complains of loss
of weight, hair fragility and loss, lack of
A. Progressing fallopian pregnancy appetite and depression. Objective exami-
B. Interrupted fallopian pregnancy nation reveals no pathological changes of
C. Right ovarian cyst uterus and its appendages. What is the
D. Uterogestation desease pathogenesis?
E. Acute appendicitis
151. 3 hours ago a 65-year-old female pati-
Krok 2 Medicine 2011 22

A. Hypoproduction of gonadotropin A. Liver abscess


B. Hyperproduction of estrogens B. Pylephlebitis
C. Hyperproduction of androgens C. Hepatic cyst
D. Hypoproduction of progesterone D. Liver necrosis
E. Hyperproduction of prolactin E. Budd-Chiari syndrome
155. During a surgery on a 30-year-old 159. A 32-year-old gravida complains of
patient a dark ileo-ileal conglomerate was episodes of unconsciousness, spontaneous
discovered, the intussusceptum intesti- syncopes that are quickly over after a
ne was considered to be unviable. The change of body position. A syncope
intussuscipiens intestine was dilated to 7-8 can be accompanied by quickly elapsing
cm, swollen, full of intestinal contents and bradycardia. There are no other compli-
gases. What pathology led to the surgery? cations of gestation. What is the most li-
kely reason for such condition?
A. Invagination (combined) obstruction
B. Strangulation obstruction A. Postcava compresseion by the gravid
C. Obturation obstruction uterus
D. Paralytic obstruction B. Pressure rise in the veins of extremities
E. Spastic obstruction C. Pressure fall in the veins of extremities
D. Vegetative-vascular dystonia (cardiac
156. In the current year general practi- type)
tioners of the municipal polyclinic have E. Psychosomatic disorders
referred 11 patients with coronary artery
disease to the in-patient hospital. In 3 160. A 54-year-old female patient
cases the diagnosis wasn’t confirmed. consulted a gynaecologist about bloody
What managerial decision shoud be made discharges from the vagina for 1 month.
in such case? Last menstruation was 5 years ago.
Gynaecological examination revealed no
A. Analysis of each case of diagnostic pathological changes. What is the tactics
divergence of choice?
B. Analysis of medical check-up quality
C. Analysis of diagnostic examination A. Diagnostic fractional curettage of uteri-
quality ne cavity
D. Analysis of doctors’ skill level B. Colposcopy
E. Analysis of material and technical C. USI
basisof the polyclinic D. Cytosmear
E. Symptomatic therapy
157. It is required to analyze the level
of daylight illumination in a ward of 161. A factory worker has ARD compli-
therapeutics unit. What device should be cated by acute bronchitis. He receives
applied to estimate the level of daylight treatment in the outpatient setting. The
illumination? attending doctor has issued him a medical
certificate for 5 days and then extended its
A. Illuminometer duration by 5 more days. Patient can’t get
B. Anemometer down to work because of his health status.
C. Katathermometer Who should extend the duration of medi-
D. Actinometer cal certificate for this patient?
E. Psychrometer
A. A department chief
158. A 43-year-old patient had been admi- B. Medical superintendent
tted to a hospital with clinical presentati- C. Deputy medical superintendent in
ons of ischiorectal periproctitis. On the charge of temporary disability examinati-
12th day of treatment the patient’s condi- on
tion deteriorated: there was an increase D. Deputy medical superintendent in
in the rate of intoxication and hepatic charge of medical treatment
failure, the body temperature became E. Medical advisory commission
hectic, AP was 100/60 mm Hg. USI of
liver revealed a hydrophilic formation. 162. An ambulance delivered a 21-
In blood: WBCs - 19, 6 · 109 /l, RBCs.- year-old woman to the gynaecological
3, 0 · 1012 /l, Hb- 98 g/l. What complicati- department with complaints of colicky
on was developed? abdominal pain and bloody discharges
from the genital tracts. Bimanual exami-
nation revealed that uterus was soft,
Krok 2 Medicine 2011 23

enlarged to the size of 6 weeks of gestati- A. Correlation factor


on, a gestational sac was palpated in B. Student’s criterion
the cervical canal. Uterine appendages C. Standardized index
weren’t palpable. Fornices are free, deep D. Fitting criterion
and painless. Discharges from the genital E. Sign index
tracts are bloody and profuse. What is the
most likely diagnosis? 166. A 30-year-old female patient has
been delivered to the gynaecological
A. Abortion in progress department with complaints of acute
B. Cervical pregnancy pain in the lower abdomen and body
C. Threat of abortion temperature 38, 8o C. In history: sexual life
D. Incipient abortion out of wedlock and two artificial aborti-
E. Interrupted fallopian pregnancy ons. Gynaecological examination reveals
no changes of uterine. The appendages
163. According to the report of water are enlarged and painful on both si-
quality control, drinking city water has des. Vaginal discharges are purulent and
the following characteristics: turbidity profuse. What study is required to confirm
- 1,5 mg/m3 , odour - 3 points, metallic a diagnosis?
taste - 2 points, pale yellow colour, colour
index - 20o , temperature - 12o . Which of A. Bacteriological and bacterioscopic
these factors doesn’t comply with hygienic analysis
requirements? B. Hysteroscopy
C. Curettage of uterine cavity
A. Odour D. Colposcopy
B. Turbidity E. Laparoscopy
C. Colour index
D. Temperature 167. A 38-year-old patient complains
E. Taste of inertness, subfebrile temperature,
enlargement of lymph nodes, nasal
164. On the fifth day after a casual haemorrhages, bone pain. Objectively:
sexual contact a 25-year-old female pati- the patient’s skin and mucous membranes
ent consulted a doctor about purulent di- are pale, palpation revealed enlarged
scharges from the genital tracts and itch. painless lymph nodes; sternalgia; liver
Vaginal examination showed that vagi- was enlarged by 2 cm, spleen - by 5
nal part of uterine cervix was hyperemic cm, painless. In blood: erythrocytes -
and edematic. There was an erosive area 2, 7 · 1012 /l, Hb- 84 g/l, leukocytes - 58 ·
around the external orifice of uterus. 109 /l, eosinophils - 1%, stab neutrophi-
There were mucopurulent profuse di- ls - 2%, segmented neutrophils - 12%,
scharges from the cervical canal, uteri- lymphocytes - 83%, lymphoblasts - 2%,
ne body and appendages exhibited no smudge cells; ESR- 57 mm/h. What is the
changes. Bacterioscopic examination most likely diagnosis?
revealed bean-shaped diplococci that
became red after Gram’s staining. What A. Chronic lymphatic leukemia
is the most likely diagnosis? B. Chronic myeloleukemia
C. Acute lymphatic leukemia
A. Acute gonorrheal endocervicitis D. Acute myeloleukemia
B. Trichomonal colpitis E. Lymphogranulomatosis
C. Candidal vulvovaginitis
D. Clamydial endocervicitis 168. A 25-year-old victim of a road accient
E. Bacterial vaginism complains of chest pain, dyspnea. Objecti-
vely: the patient is in a grave condition,
165. The correlation between the servi- Ps- 120/min, AP- 90/70 mm Hg.There is
ce record and eosinophil concentration in pathological mobility of fragments of III-
blood has been studied in female workers V ribs on the right. Percussion reveals a
of dyeing shops of textile factories. What box sound over the right lung, breathi-
index will be the most informative for the ng sounds cannot be auscultated on the
analysis of this data? right. What examination should be admi-
nistered in the first place?
Krok 2 Medicine 2011 24

A. X-ray of chest organs


B. Bronchoscopy A. Bloody
C. Pleural puncture B. Sanguino-serous
D. USI of chest organs C. Mucous
E. Thoracoscopy D. Purulent
E. Serous
169. A 24-year-old patient complains
about putting on weight, limosis. Objecti- 173. A 32-year-old patient consulted a
vely: the patient’s constitution is of doctor about being inable to get pregnant
hypersthenic type, body weight index is for 5-6 years. 5 ago the primipregnancy
33,2 kg/m2 , waist circumference is 100 cm. ended in artificial abortion. After the vagi-
Correlation of waist circumference to the nal examination and USI the patient was
thigh circumference is 0,95. What is the diagnosed with endometrioid cyst of the
most likely diagnosis? right ovary. What is the optimal treatment
method?
A. Alimentary constitutional obesity of
the I stage, abdominal type A. Surgical laparoscopy
B. Hypothalamic Itsenko-Cushing obesity B. Anti-inflammatory therapy
of the II stage, gynoid type C. Conservative therapy with estrogen-
C. Alimentary constitutional obesity of the gestagenic drugs
III stage, gynoid type D. Hormonal therapy with androgenic
D. Alimentary constitutional obesity of the hormones
II stage, abdominal type E. Sanatorium-and-spa treatment
E. Hypothalamic Itsenko-Cushing obesity
of the I stage, abdominal type 174. A 6-year-old girl drank some
coloured fizzy drink which gave her
170. A 58-year-old patient complains a feeling of pressure in the throat.
about sensation of numbness, sudden 30 minutes later the child’s lips got
paleness of II-IV fingers, muscle ri- swollen, then edema gradually spread
gidness, intermittent pulse. The pati- over the whole face, laryngeal breathi-
ent presents also with polyarthralgia, ng became difficult. The child is exci-
dysphagia, constipations. The patient’s ted. Ps- 120/min, breathing rate - 28/min,
face is masklike, solid edema of hands is breathing is noisy, indrawing of intercostal
present. The heart is enlarged; auscultati- spaces is observed. What basic aid is
on revealed dry rales in lungs. In blood: most appropriate for the restoration of
ESR - 20 mm/h, crude protein - 85/l, γ- laryngeal breathing?
globulines - 25%. What is the most likely
diagnosis? A. Corticosteroids
B. Sedative drugs
A. Systemic scleroderma C. Tracheostomy
B. Dermatomyositis D. Antibacterial drugs
C. Rheumatoid arthritis E. Conicotomy
D. Systemic lupus erythematosus
E. Raynaud’s disease 175. A 60-year-old patient had eaten too
much fatty food, which resulted in sudden
171. A 45-year-old man has been exhibi- pain in the right subcostal area, nausea,
ting high activity for the last 2 weeks, he bilious vomiting, strong sensation of bi-
is talkative, euphoric, has little sleep, clai- tterness in the mouth. Two days later the
ms being able "to save the humanity and patient presented with jaundice, dark uri-
solve the problem of cancer and AIDS", ne. Objectively: sclera and skin are icteric,
gives money to starangers. What is the abdomen is swollen, liver is increased by 3
most likely diagnosis? cm, soft, painful on palpation, Ortner’s,
Kehr’s, Murphy’s, Zakharyin’s, Mayo-
A. Maniacal onset Robson’s symptoms are positive. Which
B. Panic disorder method should be applied for diagnosis in
C. Agitated depression the first place?
D. Schizo-affective disorder
E. Catatonic excitation
172. A parturient woman is 25 years old,
it is her second day of postpartum period.
It was her first full-term uncomplicated
labour. The lochia should be:
Krok 2 Medicine 2011 25

A. USI of gallbladder and biliary duct A. III


B. Fibrogastroduodenoscopy B. II
C. X-ray of abdominal organs C. I
D. Radionuclide scanning of liver and D. IV
gallbladder E. V
E. Diagnostic laparotomy
180. A 17-year-old patient complains of
176. A 20 year-old patient complains pain in the area of the left knee joint.
of nosebleeds, numbness of the lower Soft tissues of thigh in the affected regi-
extremities. Objectively: hyperaemia of on are infiltrated, joint function is limi-
face, on the upper extremities AP is160/90 ted. X-ray picture of the distal metaepi-
mm Hg, and 80/50 mm Hg on the lower physis of the left femur shows a destructi-
ones. Pulse on the popliteal and pedal on focus with periosteum detachment and
arteries is of poor volume, there is systolic Codman’s triangle found at the defect
murmur over the carotid arteries. What is border in the bone cortex. X-ray of chest
the most likely diagnosis? reveals multiple small focal metastases.
What treatment is indicated?
A. Aorta coarctation
B. Dissecting aortic aneurysm A. Palliative chemotherapy
C. Aortopulmonary window B. Radioiodine therapy
D. Ventricular septal defect C. Distance gamma-ray therapy
E. Atrial septal defect D. Disarticulation of the lower extremity
E. Amputation of the lower extremity
177. An 8-month-old baby has had
problems with nasal breathing and muco- 181. Head circumference of a 1-month-
purulent discharge from the nose for a old boy with signs of excitement is 37 cm,
week. Examination reveals a rhinedema, prefontanel is 2x2 cm large. After feeding
muco-purulent discharges from the mi- the child regurgitates small portions of mi-
ddle nasal meatus as well as on the lk; stool is normal in respect of its volume
back of pharynx. What disease are these and composition. Muscle tonus is within
symptoms most typical for? norm. What is the most likely diagnosis?
A. Ethmoiditis A. Pylorospasm
B. Sphenoiditis B. Meningitis
C. Maxillary sinusitis C. Pylorostenosis
D. Frontitis D. Microcephaly
E. Hemisinusitis E. Craniostenosis
178. A department chief of an in-patient 182. As a result of lifting a load a 62-year-
hospital is going to inspect resident old female felt acute pain in the lumbar
doctors as to observation of medical- region, in a buttock, posterolateral surface
technological standards of patient service. of her right thigh, external surface of
What documentation should be checked the right shin and dorsal surface of foot.
for this purpose? Objectively: weakness of the anterior ti-
bial muscle, long extensor muscle of the
A. Health cards of in-patients right toes, short extensor muscle of the ri-
B. Statistic cards of discharged patients ght toes. Low Achilles reflex on the right.
C. Treatment sheets Positive Lasegue’s sign. What examinati-
D. Registry of operative interventions on method would be the most effective for
E. Annual report of a patient care instituti- specification of the diagnosis of discogenic
on compression of L5 root?
179. While asessing the health status A. Magnetic resonance scan
of graduates of a secondary school, the B. Spinal column X-ray
doctor found one of them to have grade 3 C. Electromyography
tonsillar hypertrophy, chronic rhinitis and D. Angiography
vegetative-vascular dystonia. The organi- E. Lumbar puncture
sm functionality is reduced. This student
belongs to the following health group: 183. At first appointment with an
obstetrician-gynaecologist a pregnant
woman is referred to other medical speci-
alists. She must be obligatory examined
by the following specialists:
Krok 2 Medicine 2011 26

the gynecological unit with complaints of


A. Therapeutist and dentist pain in the lower abdomen and insigni-
B. Therapeutist and endocrinologist ficant bloody discharges from the genital
C. Dentist and phthisiatrician tracts for 3 hours. Last menstruation was 3
D. ENT and ophthalmologist months ago. Vaginal examination showed
E. Dentist and cardiologist that body of womb was in the 10th week
of gestation, a fingertip could be inserted
184. A hospital admitted a patient wi- into the external orifice of uterus, bloody
th coarse breathing (obstructed inspirati- discharges were insignificant. USI showed
on), skin cyanosis, tachycardia and arteri- small vesicles in the uterine cavity. What
al hypertension. He has a histrory of is the most likely diagnosis?
bronchial asthma. An hour ago he was
having salbutamol inhalation and forgot A. Grape mole
to remove a cap that was aspired whi- B. Abortion in progress
le taking a deep breath. What measures C. Incipient abortion
should the doctor take? D. Threat of spontaneous abortion
E. Incomplete abortion
A. Perform the Heimlich manoever
B. Perform conicotomy immediately 188. A 30-year-old male patient consulted
C. Send for an anesthesiologist and wait a family doctor 2 months after he had
for him been operated for an open fracture of
D. Use an inhalation of β2 -adrenoceptor brachial bone. Objectively: the patient’s
agonist condition is satisfactory, in the region of
E. Make a subcutaneous injection of the operative wound there is a fistula
dexamethasone with some purulent discharge, redness,
fluctuation. X-ray picture shows brachi-
185. A 19-year-old student has been hospi- al bone destruction with sequestra. What
talized on an emergency basis because complication arose in the postoperative
of severe dyspnea, pain in the left si- period?
de of chest. The patient got sick 3 days
ago. Objectively: body temperature - A. Posttraumatic osteomyelitis
38, 8oC. BH - 42/min., hypopnoe. There B. Hematogenic osteomyelitis
is dullness of percussion sound on the C. Wound abscess
right of the scapula middle, breathing D. Posttraumatic phlegmon
sounds cannot be auscultated. The left E. Suture sinus
border of heart is displaced outwards
by 3 cm. Embryocardia is preent, HR - 189. A 50-year-old male suburbanite
110/min. The right hypochondrium is pai- underwent treatment in rural outpati-
nful on palpation. What are the immedi- ent clinic for pneumonia. The treatment
ate treatment measures in this situation? didn’t have effect and the disease got
complicated by exudative pleuritis. What
A. Urgent puncture of pleural cavity prevention and treatment facility should
B. Administartion of antibiotics of penici- the patient be referred to for further aid?
llin group
C. Injection of lasix A. Central district hospital
D. Injection of cardiac glycosides B. Regional hospital
E. Transfer of the patient to the thoracic C. Phthisio-pulmonological dispensary
surgery department D. Municipal hospital
E. Tuberculosis dispensary
186. On the 6th day of life a child got
multiple vesicles filled with seropurulent 190. A patient is being prepared for the
fluid in the region of occiput, neck and operation on account of varix dilatation
buttocks. General condition of the chi- of lower extremities veins. Examination
ld is normal. What disease should be of the patient’s soles revealed flour-like
suspected? desquamation along the skin folds. All
the toenails are greyish-yellow, thickened
A. Vesiculopustulosis and partially decayed. What dermatosis
B. Impetigo neonatorum should be suspected?
C. Miliaria
D. Impetigo
E. Epidermolysis bullosa
187. A pregnant woman was delivered to
Krok 2 Medicine 2011 27

A. Rubromycosis bodies to Rh weren’t found at 32 weeks of


B. Pityriasis versicolor pregnancy. Redetermination of antibodies
C. Candidosis to Rh didn’t reveal them at 35 weeks of
D. Microsporia pregnancy as well. How often should the
E. Microbial eczema antibodies be determined hereafter?
191. A 58-year-old patient complains of a A. Once a week
headache in the occipital region, nausea, B. Once in two weeks
choking, opplotentes. The presentati- C. Once in three weeks
ons appeared after a physical exerti- D. Montly
on. Objectively: the patient is excited. E. There is no need in further checks
Face is hyperemic. Skin is pale. Heart
sounds are regular, the 2nd aortic sound 195. A patient is 50 years old, works as a
is accentuated. AP- 240/120 mm Hg, HR- builder with 20 years of service record.
92/min. Auscultation reveals some fine He was admitted to the hospital for chest
moisr rales in the lower parts of the lungs. pain, dry cough, minor dyspnea. Objecti-
Liver is not enlarged. ECG shows signs of vely: sallow skin, acrocyanosis, asbestos
hypertrophy and left ventricular overload. warts on the hands. In lungs - rough respi-
What is the most likely diagnosis? ration, diffuse dry rales. The x-ray pi-
cture shows intensification of pulmonary
A. Complicated hypertensic crisis pattern, signs of pulmonary emphysema.
B. Acute myocardial infarction, pulmonary What is the most likely diagnosis?
edema
C. Bronchial asthma exacerbation A. Asbestosis
D. Uncomplicated hypertensic crisis B. Lung cancer
E. Community-acquired pneumonia C. Pneumonia
D. Chronic obstructive bronchitis
192. A 43-year-old female patient was E. Tuberculosis
delivered to the hospital in grave condi-
tion. She has a history of Addison’s di- 196. A 14-year-old girl complains of pain
sease. The patient had been regularly in vaginal area and lower abdomen that
taking prednisolone but a week before last for 3-4 days and have been observed
she stopped taking this drug. Objecti- for 3 months about the same time. Each
vely: sopor, skin and visible mucous time pain is getting worse. Objectively:
membranes are pigmented, skin and mammary glands are developed, hairi-
muscle turgor is decreased. Heart sounds ness corresponds to the age. The virgi-
are muffled, rapid. AP- 60/40 mm Hg, nal membrane is intact, cyanotic and
heart rate - 96/min. In blood: Na - 120 mi- protruded. She has never had menstruati-
llimole/l, K - 5,8 millimole/l. Development on. She has been diagnosed with pri-
of this complication is primarily caused by mary amenorrhea. What is the reason of
the deficit of the following hormone: amenorrhea?
A. Cortisol A. Hymen atresia
B. Corticotropin (ACTH) B. Turner’s syndrome
C. Adrenaline C. Babinski-Frohlich syndrome
D. Noradrenaline D. Pregnancy
E. Adrostendion E. Sexual development delay
193. Forensic medical expertise of corpse 197. A 22-year-old vegetarian patient
of a newborn revealed: body weight 3500 with signs of malnutrition consulted a
g, body length 50 cm, the umbilical cord doctor about smell and taste distortion,
was smooth, moist, glossy, without any si- angular stomatitis. Objectively: marked
gns of drying. Hydrostatic tests were posi- blue sclerae. The patient was diagnosed
tive. The test results are the evidence of: with iron deficiency anemia. What is the
dominating clinical syndrome?
A. Live birth
B. Stillbirth A. Sideropenic
C. Primary atelectasis B. Anaemic
D. Secondary atelectasis C. Haemologic
E. Hyaline membrane disease D. Haemolytic
E. Myelodysplastic
194. A primigravida is 22 years old. She
has Rh(-), her husband has Rh(+). Anti- 198. Condition of a patient with purulent
Krok 2 Medicine 2011 28

otitis has abruptly deteriorated: he A. Postoperative lethality


presents with headache, vomiting, febri- B. Total lethality
le temperature, general hyperesthesia. C. Index of late hospitalization since a
There are meningeal signs, papilledemas. disease incursion
Focal symptoms are absent. Cerebrospi- D. Standardized lethality
nal fluid is turbid, pressure is high, there E. -
is albuminocytologic dissociation with
neutrophil predominance. What disease 200. A 10-year-old boy periodically
can be suspected? has short states (up to 10-15 seconds)
that can be characterized as a "sudden
A. Secondary purulent meningitis blackout"and are accompanied by gaze
B. Meningoencephalitis fixation in the upright position, absent-
C. Serous meningitis minded and vacant face expression, lack
D. Primary purulent meningitis of movements and following amnesia.
E. Subarachnoid haemorrhage Specify this state:
199. A municipal hospital reported on A. Absence
the number of operated patients includi- B. Obnubilation
ng fatal outcomes following the operati- C. Trance
ons. Which index of hospital work can be D. Fugue
calculated on the ground of this data? E. Thought block
Krok 2 Medicine 2012 1

1. Routine examination of a child with A. Developing of cardiac insufficiency


a history of bronchial asthma reveals AP B. Depositing of blood in venous channel
of 140/90 mm Hg. The most likely cause of C. Shunting
the hypertension is: D. Presence of hypervolemia
E. Increase of bleeding speed
A. Renal disease
B. Theophylline overdose 6. A neonate was born from the 1st
C. Chronic lung disease gestation on term. The jaundice was
D. Coarctation of the aorta revealed on the 2nd day of life, then it
E. Obesity became more acute. The adynamia, vomi-
ting and hepatomegaly were observed.
2. Head of a department and a trade- Indirect bilirubin level was 275µmol/L,
union group have appealed to the head direct bilirubin level - 5µmol/L, Hb- 150
of a hospital about dismissal of the seni- g/l. Mother’s blood group - 0(I), Rh+ , chi-
or nurse who has 17 year record of servi- ld’s blood group - A(II), Rh+ . What is the
ce. The facts of charge were confirmed most probable diagnosis?
and recognized by the nurse herself. This
nurse lives with a daughter (who is di- A. Hemolytic disease of the neonate (АВ0
vorced and unemployed) and a 9-month- incompatibility), icteric type
old grandson. Make an administrative B. Jaundice due to conjugation disorder
decision: C. Hepatitis
D. Physiological jaundice
A. To continue the worker in office with E. Hemolytic disease of the neonate (Rh -
a warning of dismissal in case of repeated incompatibility)
violation of labor discipline
B. To discharge the worker, i.e. to satisfy 7. An infant was born with body mass 3 kg
demands of the collective and body length 50 cm. Now he is 3 years
C. To issue the sick list old. His brother is 7 years old, suffers from
D. To shift the solution of this problem on rheumatic fever. Mother asked the doctor
other officials or public organizations for a cardiac check up of the 3-year-old
E. - son. Where is the left relative heart border
located?
3. Purulent mediastinitis is diagnosed at a
63-year-old patient. What diseases from A. 1 cm left from the left medioclavicular
the stated below CANNOT cause the line
purulent mediastinitis? B. 1 cm right from the left medioclavicular
line
A. Cervical lymphadenitis C. Along the left medioclavicular line
B. Deep neck phlegmon D. 1 cm left from he left parasternal line
C. Perforation of the cervical part of the E. 1 cm right from the left parasternal line
easophagus
D. Perforation of the thoracic part of the 8. A neonate is 5 days old. What vacci-
easophagus nation dose of BCG vaccine (in mg) is
E. Iatrogenic injury of the trachea necessary for vaccination of this child?
4. For the persons who live in a hot area A. 0,05 mg
after an accident at a nuclear object, the B. 0,025 mg
greatest risk within the first decade is C. 0,075 mg
represented by cancer of: D. 0,1 mg
E. 0,2 mg
A. Thyroid gland
B. Skin 9. A 60-year-old woman, mother of 6
C. Reproduction system organs children, developed a sudden onset of
D. Breast upper abdominal pain radiating to the
E. Lungs back, accompanied by nausea, vomiting,
fever and chills. Subsequently, she noti-
5. During dynamic investigation of a pati- ced yellow discoloration of her sclera and
ent the increase of central venous pressure skin. On physical examination the pati-
is combined with the decrease of arterial ent was found to be febrile with temp of
pressure. What process is proved by such 38, 9oC, along with right upper quadrant
combination? tenderness. The most likely diagnosis is:
Krok 2 Medicine 2012 2

A. Choledocholithiasis g/L, WBC- 20-25 in f/vis, RBC- 1-2 in f/vis.


B. Benign biliary stricture What diagnosis is the most probable?
C. Malignant biliary stricture
D. Carcinoma of the head of the pancreas A. Acute cystitis
E. Choledochal cyst B. Dysmetabolic nephropathy
C. Acute glomerulonephritis
10. 4 days ago a 32-year-old patient D. Acute pyelonephritis
caught a cold: he presented with sore E. Urolithiasis
throat, fatigue. The next morning he
felt worse, developed dry cough, body 14. A woman, primagravida, consults
temperature rose up to 38, 2o C, there a gynecologist on 05.03.2012. A week
appeared muco-purulent expectoration. ago she felt the fetus movements for
Percussion revealed vesicular resonance the first time. Last menstruation was on
over lungs, vesicular breathing weakened 10.01.2012. When should she be given
below the angle of the right scapula, fi- maternity leave?
ne sonorous and sibilant wheezes. What is
the most likely diagnosis? A. 8 August
B. 25 July
A. Focal right-sided pneumonia C. 22 August
B. Bronchial asthma D. 11 July
C. Acute bronchitis E. 5 September
D. Pulmonary carcinoma
E. Pulmonary gangrene 15. A 40-year-old female patient has been
hospitalized for attacks of asphyxia, cough
11. A 45-year-old woman, mother of four with phlegm. She has a 4-year history of
children, comes to the emergency room the disease. The first attack of asphyxia
complaining of a sudden onset of the epi- occurred during her stay in the countrysi-
gastric and right upper quadrant pain, de. Further attacks occurred while cleani-
radiating to the back, accompanied by ng the room. After 3 days of inpatient
vomiting. On examination, tenderness is treatment the patient’s condition has si-
elicited in the right upper quadrant, bowel gnificantly improved. What is the most
sounds are decreased, and laboratory data likely etiological factor?
shows leukocytosis, normal serum levels
of amylase, lipase, and bilirubin. The most A. Household allergens
likely diagnosis is: B. Pollen
C. Infectious
A. Acute cholecystitis D. Chemicals
B. Perforated peptic ulcer disease E. Psychogenic
C. Myocardial infarction
D. Sigmoid diverticulitis 16. A 3-year-old child has been admi-
E. Acute pancreatitis tted to a hospital because of ostealgia
and body temperature rise up to 39o C.
12. During an operation for presumed Objectively: the patient is in grave condi-
appendicitis the appendix was found to tion, unable to stand for ostealgia, there
be normal; however, the terminal ileum is apparent intoxication, lymph nodesare
is evidently thickened and feels rubbery, enlarged up to 1,5 cm. Liver can be
its serosa is covered with grayish-white palpated 3 cm below the costal margin,
exudate, and several loops of apparently spleen - 2 cm below the costal margin. In
normal small intestine are adherent to it. blood: RBCs - 3, 0·1012 /l, Hb- 87 g/l, colour
The most likely diagnosis is: index - 0,9, thrombocytes - 190 · 109 /l,
WBCs - 3, 2 · 109 /l, eosinophils - 1, stab
A. Crohn’s disease of the terminal ileum neutrophils - 1, segmented neutrophils -
B. Perforated Meckel’s diverticulum 0, lymphocytes - 87, monocytes - 2, ESR
C. Ulcerative colitis - 36 mm/h. What examination should
D. Ileocecal tuberculosis be conducted in order to specify the di-
E. Acute ileitis agnosis?
13. A girl is 12-year-old. Yesterday she was A. Sternal puncture
overcooled. Now she is complaining on B. Ultrasound
pain in suprapubic area, frequent painful C. Lymph node puncture
urination by small portions, temperature D. Lymph node biopsy
is 37, 8oC. Pasternatsky symptom is E. Computer tomography
negative. Urine analysis: protein - 0,033
Krok 2 Medicine 2012 3

17. A 22-year-old girl has been complai- A. Hypothermia


ning of having itching rash on her face B. Acute cardiovascular insufficiency
for 2 days. She associates this disease wi- C. Apparent death
th application of cosmetic face cream. D. Frostbite of trunk and extremities
Objectively: apparent reddening and E. -
edema of skin in the region of cheeks,
chin and forehead; fine papulovesicular 21. A 28-year-old parturient complai-
rash. What is the most likely diagnosis? ns about headache, vision impairment,
psychic inhibition. Objectively: AP-
A. Allergic dermatitis 200/110 mm Hg, evident edemata of
B. Dermatitis simplex legs and anterior abdominal wall. Fetus
C. Eczema head is in the area of small pelvis. Fetal
D. Erysipelas heartbeats is clear, rhythmic, 190/min.
E. Neurodermatitis Internal examination revealed complete
cervical dilatation, fetus head was in the
18. A 16-year-old patient who has a hi- area of small pelvis. What tactics of labor
story of intense bleedings from minor management should be chosen?
cuts and sores needs to have the roots
of teeth extracted. Examination reveals A. Forceps operation
an increase in volume of the right knee B. Cesarean
joint, limitation of its mobility. There are C. Embryotomy
no other changes. Blood analysis shows D. Conservative labor management with
an inclination to anaemia (Hb- 120 g/l). episiotomy
Before the dental intervention it is requi- E. Stimulation of labor activity
red to prevent the bleeding by means of:
22. A 35-year-old patient complains about
A. Cryoprecipitate pain and morning stiffness of hand joi-
B. Epsilon-aminocapronic acid nts and temporomandibular joints that
C. Fibrinogen lasts over 30 minutes. She has had these
D. Dried blood plasma symptoms for 2 years. Objectively: edema
E. Calcium chloride of proximal interphalangeal digital joi-
nts and limited motions of joints. What
19. A 44-year-old patient complai- examination should be administered?
ns about difficult urination, sensati-
on of incomplete urinary bladder A. Roentgenography of hands
emptying. Sonographic examination of B. Complete blood count
the urinary bladder near the urethra C. Rose-Waaler reaction
entrance revealed an oval well-defined D. Immunogram
hyperechogenic formation 2x3 cm large E. Proteinogram
that was changing its position during the
examination. What conclusion can be 23. A patient, aged 16, complains of
made? headache, mainly in the frontal and
temporal areas, superciliary arch, appeari-
A. Concrement ng of vomiting at the peak of headache,
B. Malignant tumour of the urinary pain during the eyeballs movement, joint’s
bladder pain. On examination: excited, to - 39o C,
C. Urinary bladder polyp Ps- 110/min. Tonic and clonus cramps.
D. Prostate adenoma Uncertain meningeal signs. What is the
E. Primary ureter tumour most likely diagnosis?

20. An emergency team has delivered to A. Influenza with cerebral edema mani-
a hospital an unconscious patient found festations
lying in the street in winter. Objectively: B. Influenza, typical disease duration
the patient is pale, with superficial respi- C. Respiratory syncytial virus
ration; bradycardia with heartrate 54/min, D. Parainfluenza
to - 35, 0oC. AP- 100/60 mm Hg. Palpati- E. Adenovirus infection
on of chest and abdomen revealed no
peritoneal symptoms. There is a smell of 24. A 29-year-old female patient complai-
alcohol from the patient’s mouth. What is ns of dyspnea, heaviness and chest pain
the most likely diagnosis? on the right, body temperature rise up to
37, 2oC. The disease is associated with a
chest trauma received 4 days ago. Objecti-
vely: skin is pale and moist. Ps- 90 bpm,
Krok 2 Medicine 2012 4

regular. Palpation reveals a dull sound on rates?


the right, auscultation reveals significantly
weakened vesicular breathing. In blood: A. Relative values
RBCs - 2, 8 · 1012 /l, colour index - 0,9, Hb- B. Standard values
100 g/l, WBCs - 8, 0 · 109/l, ESR - 17 mm/h. C. Average values
What results of diagnostic puncture of the D. Absolute values
pleural cavity can be expected? E. Dynamic series

A. Haemorrhagic punctate 29. A female patient has been sufferi-


B. Chylous liquid ng from pain in the right subcostal area,
C. Exudate bitter taste in the mouth, periodical bile
D. Transudate vomiting for a month. The patient put off
E. Purulent punctate 12 kg. Body temperature in the evening
is 37, 6oC. Sonography revealed that bi-
25. A 54-year-old drowned man was le bladder was 5,5х2,7 cm large, its wall -
rescued from the water and delivered 0,4 cm, choledochus - 0,8 cm in diameter.
to the shore. Objectively: the man is Anterior liver segment contains a roundi-
unconscious, pale, breathing cannot be sh hypoechoic formation up to 5 cm in
auscultated, pulse is thready. Resuscitati- diameter and another two up to 1,5 cm
on measures allowed to save the patient. each, walls of these formations are up to
What complications may develop in the 0,3 cm thick. What is the most likely di-
near future? agnosis?
A. Pulmonary edema A. Alveolar echinococcus of liver
B. Respiratory arrest B. Liver cancer
C. Encephalopathy C. Liver abscess
D. Cardiac arrest D. Cystous liver cancer
E. Bronchospasm E. Paravesical liver abscesses
26. A 46-year-old patient once took 30. A 55-year-old male had been treated
part in elimination of breakdown at an at the surgical department for acute lower-
atomic power plant. Currently he is bei- extremity thrombophlebitis. On the 7th
ng treated at an in-patient hospital. He day of treatment he suddenly developed
was diagnosed with progressing vegetati- pain in the left part of chest, dyspnea
ve insufficiency. This disease relates to and cough. Body temperature was 36, 1o C,
the following group of ionizing radiation respiratory rate - 36/min. The patient was
effects: also found to have diminished breath
sounds without wheezing. Ps- 140/min,
A. Somato-stochastic thready. AP- 70/50 mm Hg. The ECG
B. Somatic shows Q ÁÁ-S1 syndrome. What is the most
C. Genetic likely diagnosis?
D. Hormesis
E. Heterosis A. Pulmonary embolism
B. Myocardial infarction
27. A military unit stopped for 3-day’s rest C. Cardiac asthma
in an inhabited locality after a long march. D. Bronchial asthma
The sanitary-epidemiological reconnai- E. Pneumothorax
ssance found several water sources. It is
necessary to choose the source complying 31. A 16-year-old adolescent was vacci-
with the hygienic standards for drinking nated with DTP. In eight days there was
water in the field conditions: stiffness and pain in the joints, subfebri-
le temperature, urticarial skin eruption,
A. Artesian well water enlargement of inguinal, cervical lymph
B. Spring water nodes and spleen. What kind of allergic
C. River water reaction is observed?
D. Rain water
E. Melt snow water A. Immunocomplex
B. Hypersensitivity of immediate type
28. Deputy of chief medical officer carried C. Cytoxic
out a study of morbidity rate for populati- D. Hypersensitivity of delayed type
on which had been served at the polyclini- E. -
cs within the last 5 years. What statistical
values can help in calculation of morbidity 32. A 56-year-old scientist experiences
Krok 2 Medicine 2012 5

constricting retrosternal pain several ti- A. Ankylosing spondylitis


mes a day while walking for 100-150 m. B. Coxarthrosis
The pain lasts for up to 10 minutes and can C. Rheumatoid arthritis
be relieved by nitroglycerine. Objectively: D. Reiter’s disease
the patient is overweight, heart borders E. Spondylosis
exhibit no abnormalities, heart sounds are
rhythmic, Ps- 78 bpm, AP- 130/80 mm Hg. 36. A 58-year-old female patient complai-
ECG contains low amplitude of T wave in ns about periodical headache, dizziness
V4−5 . What disease might be suspected? and ear noise. She has been suffering from
diabetes mellitus for 15 years. Objecti-
A. Stable FC III stenocardia vely: heart sounds are rhythmic, heart rate
B. Instable stenocardia is 76/min, there is diastolic shock above
C. Stable FC I stenocardia aorta, AP is 180/110 mm Hg. In urine:
D. Stable FC II stenocardia OD- 1,014. Daily loss of protein with uri-
E. Stable FC IV stenocardia ne is 1,5 g. What drug should be chosen
for treatment of arterial hypertension?
33. In autumn a 25-year-old patient
developed stomach ache arising 1,5-2 A. Ihibitor of angiotensin converting
hours after having meals and at night. enzyme
He complains of pyrosis and constipation. B. β-blocker
The pain is getting worse after consumi- C. Calcium channel antagonist
ng spicy, salty and sour food, it can be D. Thiazide diuretic
relieved by means of soda and hot-water E. α-blocker
bag. The patient has been suffering from
this disease for a year. Objectively: furred 37. At the radiological unit of a hospi-
moist tongue. Abdomen palpation reveals tal gamma-devices of radiotherapy of
epigastrial pain on the right, resistance "Agat"type and other closed sources of
of abdominal muscles in the same region. ionizing radiation are used for treatment
What is the most likely diagnosis? of malignant neoplasms. What measures
are to be taken to protect personnel duri-
A. Duodenal ulcer ng working with radioactive sources of
B. Chronic cholecystitis such type?
C. Diaphragmatic hernia
D. Stomach ulcer A. Reduction of working time and screeni-
E. Chronic pancreatitis ng of the source
B. Capsulation of devices and organization
34. A secundipara has regular birth activi- of room ventilation
ty. Three years ago she had cesarean secti- C. Screening of the source and the use of
on for the reason of acute intrauterine means of individual protection of respirati-
hypoxia. During parodynia she complai- on organs
ns of extended pain in the area of D. The increase of distance to the source
postsurgical scar. Objectively: fetus pulse and individual hygiene compliance
is rhythmic - 140 bpm. Vaginal examinati- E. Systematical cleansing of surfaces
on shows 5 cm cervical dilatation. Fetal from the radioactive contamination and
bladder is intact. What is the tactics of shortening of working time
choice?
38. A 20-year-old adolescent lives in
A. Cesarean section the nidus of tuberculous infection. The
B. Augmentation of labour tuberculine Mantoux test with 2 TU
C. Obstetrical forceps was determined as hyperergic. What si-
D. Waiting tactics of labor management gns determine the hyperergic test of this
E. Vaginal delivery adolescent?
35. A 32-year-old male patient has been A. 6 mm papula, necrosis
suffering from pain in the sacrum and B. 20 mm papula
coxofemoral joints, painfulness and sti- C. 24 mm hyperemia
ffness in the lumbar spine for a year. ESR- D. 4 mm papula
56 mm/h. Roentgenography revealed E. 12 mm hyperemia
symptoms of bilateral sacroileitis. The
patient is the carrier of HLA B27 anti- 39. A survey radiograph of a miner
gen. What is the most likely diagnosis? (24 years of service record, the dust
concentration in the workplace is at the
rate of 260-280 mg/m3 with 15% of free
Krok 2 Medicine 2012 6

silica) shows lung changes that are typi- 43. To study physical development of chi-
cal for pneumoconiosis. What type of ldren and adolescents, anthropometric
pneumoconiosis is it? studies are widely used. Choose a physi-
ometric method of study from the below
A. Anthracosilicosis given.
B. Carboconiosis
C. Silicatosis A. Determination of vital capacity of lungs
D. Anthracosilicatosis B. Measurement of growth
E. Silicosis C. Determination of thorax form
D. Determination of vertebra form
40. A 5-year-old child had an attack E. Determination of body weight
of palpitation with nausea, dizziness,
generalized fatigue. On ECG: tachycardia 44. An electro-gas welding operator
with heartbeat rate of 220/min. Ventricle working at a machine workshop performs
complexes are deformed and widened. P welding and cutting of metal, which is
wave is absent. What medication is to be accompanied by intense UV-radiation.
prescribed to provide first aid? His welding station is equipped with
effective mechanical ventilation. What
A. Lydocain occupational disease is most likely
B. Isoptin to develop in an electro-gas welding
C. Seduxen operator?
D. Novocainamides
E. Strophantin A. Photoelectric ophthalmia
B. Heatstroke
41. A 57-year-old man complains of C. Vegetative-vascular dystonia
shortness of breath, swelling on shanks, D. Chronic overheating
irregularity in cardiac work, pain in the E. Pneumoconiosis
left chest half with irradiation to the
left scapula.Treatment is uineffective. On 45. A 60-year-old female patient had
physical exam: heart’s sounds are dimini- been admitted to a hospital for acute
shed, soft systolic murmur on the apex. transmural infarction. An hour later
Ps - 100/min, arrhythmical, BP - 115/75 the patient’s contition got worse. She
mm Hg. The liver is +2 cm, painful. developed progressing dyspnea, dry
Roentgenoscopy: enlargement of heart cough. Respiratory rate - 30/min, heart
shadow to all sides, pulsation is weak. rate - 130/min, AP- 90/60 mm Hg. Heart
Electrocardiogram (ECG): leftventricled sounds were muffled, there was also di-
extrasystolia, decreased voltage. What astolic shock on the pulmonary artery.
method of investigation is necessary to The patient presented with medium moi-
do to determine the diagnosis? st rales in the lower parts of lungs on the
right and on the left. Body temperature -
A. Echocardiography 36, 4oC. What drug should be given in the
B. Veloergometria first place?
C. X-ray kymography
D. ECG in the dynamics A. Promedol
E. Coronarography B. Aminophylline
C. Dopamine
42. A 27-year-old woman presents at D. Heparin
the maternity welfare centre because of E. Digoxin
infertility. She has had sexual life in marri-
age for 4 years, doesn’t use contraceptives. 46. A 65-year-old patient complains of
She hasn’t get pregnant. On examination: gradual worsening of the left eye vision
genital development is without pathology, during 10 months. On physical examinati-
uterine tubes are passable, basal (rectal) on: acuty of vision of the left eye is 0,01,
temperature is one-phase during last 3 not correctable. The eye is quiet, pupil
menstrual cycles. What is the infertility of the eye is grayish, reflex from the eye-
cause? ground is absent. Intraocular pressure is
18 mm/Hg. What is the most probable
A. Anovular menstrual cycle preliminary diagnosis?
B. Chronic adnexitis
C. Abnormalities in genital development
D. Immunologic infertility
E. Genital endometriosis
Krok 2 Medicine 2012 7

A. Senile cataract A. Acute pulmonary abscess


B. Open-angle glaucoma B. Exudative pleuritis
C. Disorder of blood circulation in retina C. Acute focal pneumonia
vessels D. Pleural empyema
D. Leukoma of the cornea E. Pyopneumothorax
E. Exfoliation of the retina
50. A 25-year-old woman complains of
47. A patient is on the sick leave for 4 profuse foamy vaginal discharges, foul,
months continuously from the date of burning and itching in genitalia region.
injury. The treatment is going to last for She has been ill for a week. Extramarital
1-2 months. Who has the right to extend sexual life. On examination: hyperemia
the duration of medical certificate for this of vaginal mucous, bleeding on touchi-
patient? ng, foamy leucorrhea in the urethral area.
What is the most probable diagnosis?
A. Medical advisory commission after
medico-social expert commission exami- A. Trichomonas colpitic
nation B. Gonorrhea
B. Medical superintendent C. Chlamydiosis
C. Medical advisory commission after D. Vagina candidomicosis
inpatient treatment E. Bacterial vaginosis
D. District doctor by agreement with a
department chief 51. A 50-year-old locksmith has a long-
E. Medico-social expert commission term record of work under the effect
of mercury vapors with concentration
48. A child from the first non-complicated exceeding MPC by 5-10 times. Clini-
pregnancy but complicated labor had cal examination revealed the lability of
cephalhematoma. On the second day vasomotors of skin, pulse and arterial
there developed jaundice. On the 3th day pressure; total hyperhydrosis; asymmetric
appeared changes of neurologic status: innervation of facial and lingual muscles,
nystagmus, Graefe’s sign. Urea is yellow, positive subcortical reflexes, intenti-
feces- golden-yellow. Mother’s blood on tremor. Against the background of
group is ü(II)Rh− , child- ü(II)Rh+ . increased emotional excitability the pati-
On the third day child’s Hb- 200 g/L, ent presents with lack of self-confidence,
RBC- 6, 1 · 1012 /L, bilirubin in blood - shyness. A dentist found him to have
58 mk mol/L due to unconjugated bili- parodontosis, chronic stomatitis. What di-
rubin, Ht- 0,57. What is the child’s jaundi- sease can be suspected?
ce explanation?
A. Chronic mercury intoxication
A. Brain delivery trauma B. Residual effects of neuroinfection
B. Physiologic jaundice C. Parkinson’s syndrome
C. Hemolytic disease of newborn D. Acute mercury intoxication
D. Bile ducts atresia E. Vascular encephalopathy
E. Fetal hepatitis
52. 4 hours after having meals a patient
49. On the 4th day after recovering from with signs of malnutrition and steatorrhea
a cold a patient was hospitalized with experiences stomach pain, especially
complaints of solitary spittings of mucoid above navel and to the left of it. Diarrheas
sputum. On the 2nd day there was a si- take turns with constipation lasting up to
ngle discharge of about 250 ml of purulent 3-5 days. Palpation reveals moderate pai-
blood-streaked sputum. Objectively: the nfulness in the choledochopancreatic regi-
patient’s condition is moderately severe. on. The amylase rate in blood is stable.
Respiratory rate - 28-30/min, Ps- 96 bpm, X-ray reveals some calcifications located
AP- 110/70 mm Hg. Respiration above the above navel. What is the most likely di-
left lung is vesicular, weak above the right agnosis?
lung. There are moist rales of different
types above the lower lobe and amphoric A. Chronic pancreatitis
breath near the angle of scapula. What is B. Chronic gastroduodenitis
the most likely diagnosis? C. Duodenal ulcer
D. Zollinger-Ellison syndrome
E. Chronic calculous cholecystitis
53. A 54-year-old patient complains
of weakness, weight loss despite the
Krok 2 Medicine 2012 8

unchanged appetite, frequent urinati- emergent therapy.


on, skin itch for six months. Some time
ago the patient underwent treatment for A. Zovirax
furunculosis. She hasn’t been examined B. Glucocorticoids
recently. Objectively: malnutrition, dry C. Cephtriaxon
skin with signs of scratching. Small lymph D. Lasix
nodes can be palpated in the axillary regi- E. Hemodesis
ons. Changes in the internal organs are
absenr. What testing must be administered 57. A 30-year-old patient was delivered
in the first place? to the admission ward of the infecti-
ous disease department. The disease had
A. Blood sugar test on an empty stomach started acutely on the background of
B. Complete blood count normal temperature with the appearance
C. Endoscopy of stomach of frequent, liquid, profuse stool without
D. Lymph node biopsy pathological impurities. Diarrhea was not
E. Blood sterility testing accompanied by abdominal pain. 12 hours
later there appeared recurrent profuse
54. A woman consulted a doctor on vomiting. The patient rapidly developed
the 14th day after labour about sudden dehydration. What is the most likely di-
pain, hyperemy and induration of the left agnosis?
mammary gland, body temperature ri-
se up to 39o C, headache, indisposition. A. Cholera
Objectively: fissure of nipple, enlargement B. Shigellosis
of the left mammary gland, pain on C. Staphylococcal food toxicoinfection
palpation. What pathology would you thi- D. Salmonellosis
nk about in this case? E. Campylobacteriosis

A. Lactational mastitis 58. A 67-year-old male complains


B. Lacteal cyst with suppuration of dyspnea on exertion, attacks of
C. Fibrous adenoma of the left mammary retrosternal pain, dizziness. He has no
gland history of rheumatism. Objectively: pale
D. Breast cancer skin, acrocyanosis. There are rales in the
E. Phlegmon of mammary gland lower parts of lungs. There is systolic thri-
ll in the II intercostal space on the right,
55. Several hours before, a 28-year- coarse systolic murmur conducted to the
old patient suddenly developed acute vessels of neck. AP- 130/90 mm Hg, heart
headache and repeated vomiting, then rate - 90/min, regular rhythm. The liver
lost consciousness. Objectively: focal extends 5 cm under the edge of costal
neurological symptoms were not found. arch, shin edemata are present. Specify
Pronounced meningeal symptoms were the assumed valvular defect:
revealed. AP - 120/80 mm Hg. Accordi-
ng to clinical and liquorological findi- A. Aortic stenosis
ngs the patient was diagnosed with B. Pulmonary artery stenosis
subarachnoid haemorrhage. After admi- C. Mitral insufficiency
nistration of dehydrants the patient’s D. Ventricular septal defect
condition somewhat improved. What is E. Tricuspid regurgitation
the main component of further emergency
care? 59. A 24-year-old female teacher complai-
ns of dizziness and heart pain irradiati-
A. Coagulants ng to the left nipple. Pain is not associ-
B. Anticoagulants ated with physical activity and cannot
C. Antiaggregants be relieved by nitroglycerin, it abates
D. Fibrinolytics after taking Valocordin and lasts an hour
E. Corticosteroids or more. The patient has a nearly 2-
year history of this disease. Objectively:
56. A 24-year-old man on the 5th day Ps- 76 bpm. AP- 110/70 mm Hg. Heart
of acute respiratory disease with high borders are norma, heart sounds are clear.
grade temperature started having strong The ECG shows respiratory arrhythmia.
headaches, systemic dizziness, sensati- Radiograph of the cervicothoracic spi-
on of double vision, paresis of mimic ne shows no pathology. Lungs, abdomen
muscles to the right, tickling by swallowi- are unremarkable. What changes in blood
ng. Diagnosis: Acute viral encephalitis. formula can be expected?
Determine the basic direction of the
Krok 2 Medicine 2012 9

A. No changes
B. Leukocytosis A. Consultation of an expert in narcology
C. Thrombocytopenia B. Prescription of medications the patient
D. Leukemic hiatus asks for
E. Increased ESR C. Additional consultation of surgeon
D. Treatment with antibiotics
60. A 51-year-old female patient complai- E. Consultation of infectious diseases
ns of frequent defecation and liquid doctor
blood-streaked stools with mucus admi-
xtures, diffuse pain in the inferolateral 63. A 1,5-year-old child fell ill acutely
abdomen, 6 kg weight loss over the with high temperature 38o C, headache,
previous month. Objectively: body fatigue. The temperature declined on the
temperature - 37, 4oC, malnutrition, skin fifth day, muscular pain in the right leg
is pale and dry. Abdomen is soft, sigmoid occured in the morning, there were no
is painful and spasmodic, makes a rumbli- movements and tendon reflexes, sensiti-
ng sound. Liver is dense, painful, extends vity was reserved. What is the initial di-
3 cm below the costal margin. What is the agnosis?
most likely diagnosis?
A. Polyomyelitis
A. Non-specific ulcerative colitis B. Viral encephilitis
B. Bacillary dysentery C. Polyartropathy
C. Sprue D. Osteomyelitis
D. Intestinal enzymopathy E. Hip joint arthritis
E. Helminthic invasion
64. Development of chronic venous
61. A 32-year-old female complains of di- insufficiency of lower extremities depends
zziness, headache, palpitation, tremor. For on the functional condition of so-called
the last several months she has been under musculovenous pump. This term refers to
outpatient monitoring for the increased the following group of muscles:
arterial pressure. Since recently such
attacks have become more frequent and A. Shin muscles
severe. Objectively: skin is covered with B. Abdominal wall muscles
clammy sweat, tremor of the extremities C. Buttock region muscles
is present. HR- 110/min, AP- 220/140 mm D. Thigh muscles
Hg. Heart sounds are weakened. In blood: E. Foot muscles
WBCs - 9, 8 · 109 /l, ESR - 22 mm/h. Blood 65. A 7-year-old child was brought to
glucose - 9,8 millimole/l. What disease is a doctor for a check. The child has
the most likely cause of this crisis? a 4-year history of bronchial asthma,
A. Pheochromocytoma asthma attacks occur mainly in spri-
B. Essential hypertension ng and summer. Allergy tests revealed
C. Preeclampsia hypersensitivity to poplar seed tufts, fi-
D. Primary hyperaldosteronism eld herbs. What recommendation should
E. Diabetic glomerulosclerosis be given?

62. A 19-year-old woman complains of A. Specific hyposensitization


pain in the abdomen and joints, asks B. Physiotherapy
for more analgetics and somnifacient C. Treatment at a health resort
injections. The patient was examined. D. Phytotherapy
Gynecological and urological pathologi- E. Needle reflexotherapy
es are absent. There are signs of previous 66. A farmer hurt his right foot duri-
punctures along superficial veins of the ng working in a field and came to the
extremities. The patient does not explain emergency station. He doesn’t remember
the origin of punctures. Tendon reflexes of when he got last vaccination and he has
upper and lower extremities are the same, never served in the army. Examination
quick. Photoreaction of the pupil of the of his right foot revealed a contaminated
eye is weak. The tongue is grey coated. wound up to 5-6 cm long with uneven
During communication the patient in edges. The further treatment tactics will
affectively not even-tempered. There is be:
diarrhea without pathologic inclusions.
What tactics is necessary to improve the
condition of this patient?
Krok 2 Medicine 2012 10

A. To make an injection of tetanus vely: there are small red papules set
anatoxin and antitetanus serum mostly in pairs in the region of interdigital
B. To make an injection of tetanus folds on both hands, on the flexor surface
anatoxin of radicarpal articulations, abdomen and
C. To make an injection of antitetanus buttock skin as well as internal surface
serum of thighs. In the centre of some papules
D. Surgical d-bridement only vesicles or serohaemorrhagic crusts can
E. To administer an antibiotic be seen. There are multiple excoriations.
What is the most likely diagnosis?
67. A 50-year-old patient complains about
having pain attacks in the right subcostal A. Scabies
area for about a year. He pain arises mai- B. Dermatitis
nly after taking fattening food. Over the C. Ringworm of body
last week the attacks occurred daily and D. Toxicoderma
became more painful. On the 3rd day of E. Eczema
hospitalization the patient presented with
icteritiousness of skin and scleras, light- 71. A 43-year-old female patiet complai-
colored feces and dark urine. In blood: ns of eruption on her right leg skin, pain,
neutrophilic leukocytosis - 13, 1 · 109 /l, weakness, body temperature rise up to
ESR- 28 mm/h. What is the most likely 38oC. The disease is acute. Objectively:
diagnosis? there is an edema on the right leg skin in
the region of foot, a well-defined bright
A. Chronic calculous cholecystitis red spot in form of flame tips which feels
B. Chronic recurrent pancreatitis hot. There are isolated vesicles in focus.
C. Fatty degeneration of liver What is your provisional diagnosis?
D. Chronic cholangitis, exacerbation stage
E. Hypertensive dyskinesia of gallbladder A. Erysipelas
B. Microbial eczema
68. In morgue there are dead bodi- C. Contact dermatitis
es with the following causes of death: D. Toxicoderma
electrotrauma; rupture of the spleen wi- E. Haemorrhagic vasculitis
th acute anemia. There is one unknown
person; one ethyl alcohol poisoned person 72. A 47-year-old patient came to see a
and one drowned man. What dead body doctor on the 7th day of disease. The di-
should the blood group be determined sease developed very fast: after the chill
for? body temperature rose to 40oC and lasted
up to 7 hours, then dropped abruptly,
A. All dead bodies of the unknown persons which caused profuse sweat. There were
B. Body of poisoned person three such attacks occuringonce in two
C. Body of drowned man days. Two days ago the patient arrived
D. Body of person with internal from Africa. Objectively: pale skin, subi-
hemorrhage cteric sclera, significantly enlarged liver
E. Body of person with a sudden death and spleen. What is the cause of fever
attacks in this disease?
69. A 36-year-old patient complains of
skin rash that appeared a week ago and A. Erythrocytic schizogony
doesn’t cause any subjective problems. B. Tissue schizogony
Objectively: palm and sole skin is covered C. Exotoxin of a causative agent
with multiple lenticular disseminated D. Endotoxin of a causative agent
papules not raised above the skin level. E. Gametocytes
The papules are reddish, dense on palpati-
on and covered with keratinous squamae. 73. On the 2 nd day of illness a 27-year-
What is the provisional diagnosis? old patient complains of the unbearable
headache, repeated vomiting. Objecti-
A. Secondary syphilis vely: the patient is in a grave condition.
B. Verrucosis He is conscious but adynamic. Lies in a
C. Palmoplanar psoriasis forced position with his head thrown back.
D. Palmoplanar rubrophytosis There is no skin rash. Occipital muscles
E. Palm and sole callosity are evidently rigid, there are Kernig’s
and Brudzinski’s signs. to - 39, 5o C, Ps-
70. A 5-grade pupil complains about 120/min., AP- 130/80 mm Hg. What is the
extensive skin rash accompanied by reason for the leading syndrome of this
intensive itch, especially at night. Objecti- disease?
Krok 2 Medicine 2012 11

ng the divorce. On examination: palm


A. Liquor hypertension hyperhydrosis, pulse rate- 72-78 bpm, labi-
B. Liquor hypotension le, heart without changes. ECG is normal.
C. Affection of the cranial nerve nuclei What is the most probable pathology in
D. Haemorrhages into the adrenal glands this case?
E. Hyperthermy
A. Neurasthenia
74. A patient has chronic heart fai- B. Ipochondric neurosis
lure of the II stage. He takes furosemi- C. Compulsive neurosis
de regularly three times a week. D. Schizophrenia
He had developed bronchopneumonia E. Depressive neurosis
and had been administered combined
pharmacotherapy. On the fifth day of 78. A 30-year-old woman with a long hi-
therapy the patient complained of hearing story of chronic pyelonephritis complains
impairment. What drug coadministered about considerable weakness, sleepiness,
with furosemide might have caused the decrease in diuresis down to 100 ml per
hearing loss? day. AP- 200/120 mm Hg. In blood: creati-
nine - 0,62 millimole/l, hypoproteinemia,
A. Gentamicin albumines - 32 g/l, potassium - 6,8 milli-
B. Linex mole/l, hypochromic anemia, increased
C. Nystatin ESR. What is the first step in the pati-
D. Tavegil ent treatment tactics?
E. Mucaltin
A. Haemodialysis
75. A 54-year-old female patient has been B. Antibacterial therapy
admitted to a hospital 12 days after the C. Enterosorption
beginning of acute pancreatitis. Objecti- D. Haemosorption
vely: the patient is in grave condition. The E. Blood transfusion
body temperature is hectic. Ps - 112 bpm.
The abdomen is swollen. Epigastrium 79. A 10-year-old girl consulted a doctor
palpation reveals a very painful infiltrati- about thirst, frequent urination, wei-
on falling in the localization of pancreas. ght loss. She has been observing these
Abdominal cavity contains fluid. There symptoms for about a month. Objecti-
is an edema of the left lumbar region. In vely: no pathology of internal organs was
blood: WBCs - 18·109 /l. What is the requi- revealed. What laboratory analysis should
red tactics of the patient’s treatment? be carried out in the first place?
A. Surgical treatment A. Blood glucose analysis on an empty
B. Further conservative treatment stomach
C. Peritoneal dialysis B. Glucose in urine test on the base of
D. Increase in antienzymes daily diuresis
E. Massive antibacterial therapy C. Acetone in urine test
D. Glucose tolerance test
76. To replace the blood loss replacement E. Glucosuric profile
1000 ml of the same group of Rhesus-
compatible donated blood was transfused 80. A 36-year-old female patient complai-
to the patient. The blood was conserved ns of bruises on the body, gingival
by sodium citrate. At the end of haemorrhage, general weakness. A month
hemotransfusion there appeared exci- ago she had a severe domestic poisoni-
tement, pale skin, tachycardia, muscles ng with some pesticide (the patient can
cramps in the patient. What complicati- not remember the name). She has a 7-
on should be suspected? year record of working in contact with
petroleum products, particularly benzene.
A. Citrate intoxication In blood: RBCs - 3, 2 · 1012 /l, WBCs -
B. Citrate shock 2, 7 · 109 /l, thrombocytes - 70 · 109 /l. What
C. Allergic reaction is the most likely pathology?
D. Anaphylactic shock
E. Pyrogenous reaction
77. A 24-year-old emotionally-labile
woman presents with irritation, depressed
mood, palpitation, shooting pain in the
heart area, generalized fatigue followi-
Krok 2 Medicine 2012 12

A. Benzene intoxication convulsions. Objectively: the patient is


B. Organophosphorus pesticide intoxicati- unconscious, the skin is pale, extremities
on are cold. AP - 90/60 mm Hg, Ps- 96/min,
C. Organochlorine pesticide Intoxication deficient, breathing is shallow. Pupillary
D. Mercury-containing pesticide intoxi- and tendon reflexes are present. There are
cation no pathological signs. What is the most li-
E. Chronic fatigue Syndrome kely diagnosis?
81. A 6-year-old child complains of A. Syncope
frequent liquid stool and vomiting. On the B. Vegetovascular paroxysm
2nd day of desease the child presented C. Epileptic attack
with inertness, temperature rise up to D. Hysterical neurosis
38, 2oC, Ps- 150 bpm, scaphoid abdomen, E. Transient ischemic attack
palpatory painful sigmoid colon, defecati-
on 10 times a day with liquid, scarce stool 85. A patient complains of impaired far vi-
with mucus and streaks of green. What is sion. Previously his eyes often turned red
a provisional diagnosis? and hurt. Objectively: the eyes are not irri-
tated, the cornea is transparent, anterior
A. Shigellosis chambers are median deep, their moisture
B. Salmonellosis is transparent. The iris of the right eye
C. Escherichiosis has not changed in colour, its pattern is
D. Intestinal amebiasis unchanged. The pupil is of irregular shape,
E. Yersiniosis scalloped. Biomicroscopy of the crystalli-
ne lens reveals the areas of opacity and
82. A 68-year-old patient consulted vacuoles. Make a diagnosis:
a doctor about a tumour in her left
mammary gland. Objectively: in the upper A. Complicated cataract of the right eye
internal quadrant of the left mammary B. Senile cataract of the right eye
gland there is a neoplasm up to 2,5 cm C. Diabetic cataract of the right eye
in diameter, dense, uneven, painless on D. Tetanic cataract of the right eye
palpation. Regional lymph nodes are not E. Radiation cataract of the right eye
enlarged. What is the most likely di-
agnosis? 86. A 26-year-old patient complains of
experiencing pain in the right hand for 4
A. Cancer days. The pain arose at the site of corns
B. Cyst on the palmar surface at the base of
C. Fibroadenoma the II and III fingers. Some time later
D. Mastopathy the dorsum of hand became edematic.
E. Lipoma I and III fingers are half-bent in the
interphalangeal joints, the hand looks like
83. A 65-year-old patient complains of "rake". What is the most likely diagnosis?
pain in the lumbar spine, moderate di-
suria. He has been suffering from these A. Phlegmon of the second interdigital
presentations for about half a year. space of the right hand
Prostate volume is 45 cm3 (there are B. Adenophlegmon of the right hand
hypoechogenic nodes in both lobes, C. Corn abscess of the right hand
capsule invasion). The rate of prostate- D. U-shaped phlegmon of the right hand
specific antigen is 60 ng/l. Prostate biopsy E. Tendovaginitis
revealed an adenocarcinoma. Which of
the supplemental examination methods 87. A 40-year-old female patient complain
will allow to determine the stage of of headache, dizziness, muscle weakness,
neoplastic process in this patient? sometimes - cramps in the extremities. She
has been taking antihypertensive medi-
A. Computer tomography of pelvis cations for 10 years. AP- 180/100 mm Hg.
B. Roentgenography of lumbar spine Blood potassium - 1,8 millimole/l, sodium
C. Excretory urography - 4,8 millimole/l. In urine: alkaline reacti-
D. Bone scintigraphy on, the relative density - 1012, protein and
E. Roentgenography of chest sugar are not found, WBCs - 3-4 in the fi-
eld of vision, RBCs - 1-2 in the field of visi-
84. While staying in a stuffy room a 19- on. Conn’s syndrome is suspected. Which
year-old emotionally labile girl developed drug should be chosen for the treatment
severe weakness, dizziness, blackout, of arterial hypertension?
nausea and loss of consciousness without
Krok 2 Medicine 2012 13

A. Spironolactone A. Splenectomy
B. Propanolol B. Iron preparations
C. Enalapril C. Hemotransfusion
D. Hydrochlorothiazide D. Cytostatics
E. Clonidine E. Vitamin B12
88. An 18-year-old patient presents no 92. A woman while working in vegetable
problems. Percussion reveals that heart garden developed severe pain in the loin.
borders are displaced to the right and Lasague’s and Nery tension signs are
left by 1 cm, there is a coarse systolic obviously marked on the right. Lumbar
murmur with its epicenter within the 4th lordosis is smoothed, movements are
intercostal space on the left. What is the harshly restrained in lumbar part of the
most informative examination to confirm spine. Right ankle (Achilles) reflex is
the clinical diagnosis? absent. What kind of disease can it be?
A. Ventriculography A. Lumbar-sacral radiculitis
B. ECG B. Lumbalgia
C. PCG C. Hepatic colic
D. Echocardiography D. Renal colic
E. Polycardiography E. Neuritis of femoral nerve
89. A 56-year-old patient complains of 93. After lifting a load a patient felt
having persistent chest pain on the ri- undurable pain in the loin. He was di-
ght for the last 2 months. The pain agnosed with acute lumbosacral radiculi-
is not associated with respiration. He tis. Which of the following is contraindi-
also complains of cough with blood- cated for this patient?
streaked sputum, weakness, decreased
performance, fatigue. Chest radiograph A. Warming procedures
shows a globular shade of 4x6 cm B. Dehydrating drugs
connected to the root of the lung in the C. Analgetics
lower part of the right lung. What is the D. Vitamins of B group
most likely diagnosis? E. Intravenous injection of aminophylline
A. Peripheral lung cancer 94. An unconscious 35-year-old patient
B. Metastasis has been delivered by an ambulance to
C. Lung abscess the intensive care unit. Objectively: the
D. Pneumonia patient is in semicoma. Moderate mydri-
E. Tuberculoma asis is present. The reaction of pupils to
light is reduced. The reaction to verbal
90. A 30-year-old patient had deep burn instructions is missing. AP is150/100 mm
covering 30% of body 30 days ago. Now Hg, there is tachycardia. Blood contains
he presents with continued fever, loss of methanol. What antidote should be admi-
appetite, night sweats. Burned surface nistered?
weakly granulates. What is the stage of
burn disease? A. Ethanol
B. Unithiol
A. Septicotoxemia C. Thiamine chloride
B. Primary burn shock D. Tavegil
C. Secondary burn shock E. Naloxone
D. Acute burn toxemia
E. Convalescence 95. A 36-year-old man was delivered
to the surgical department an hour
91. A 27-year-old patient complains of after a road accident. His condition
nasal haemorrhages, multiple bruises on is getting worse: respiratory insuffici-
the anterior surface of the trunk and ency is progressing, there are cardiac
extremities, sudden weakness. In blood: abnormalities. Clinical and roentgenologi-
Hb- 74 g/l, reticulocytes - 16%, RBCs - cal investigations revealed mediastinal di-
2, 5 · 1012 /l, platelets - 30 · 109 /l, ESR- 25 splacement. What process has caused this
mm/h. What is the most effective measure complication?
for the treatment of thrombocytopenia?
Krok 2 Medicine 2012 14

A. Valvular pneumothorax lymphocytes - 8%, ESR - 42 mm/h. What


B. Open pneumothorax is the ethiological factor of pneumonia?
C. Closed pneumothorax
D. Subcutaneous emphysema A. Legionella
E. Mediastinitis B. Mycoplasm
C. Streptococcus
96. A 26-year-old male patient complains D. Staphylococcus
of piercing pain during breathing, cough, E. Pneumococcus
dyspnea. Objectively: to - 37, 3oC, respi-
ration rate - 19/min, heart rate = Ps- 100. A 53-year-old woman complained
92/min; AP- 120/80 mm Hg. Vesicular of cardiac pain and rhythm intermissions.
respiration. In the inferolateral parts of She had experienced these presentations
chest auscultation in both inspiration and since childhood. The patient’s father had
expiration phase revealed noise that was a history of cardiac arrhythmias. Objecti-
getting stronger at phonendoscope pressi- vely: the patient was in grave conditi-
ng and can be still heard after cough. ECG on, Ps- 220 bpm, AP- 80/60 mm Hg.
showed no pathological changes. What is ECG: heart rate - 215/min, widening and
the most likely giagnosis? deformation of QRS complex accompani-
ed by atrioventricular dissociation; posi-
A. Acute pleuritis tive P wave. Some time later heart rate
B. Intercostal neuralgia reduced down to 45/min, there was a
C. Subcutaneous emphysema complete dissociation of P wave and
D. Spontaneous pneumothorax QRST complex. Which of the following
E. Pericarditis sicca will be the most effective treatment?
97. Educational rooms are illuminated wi- A. Implantation of the artificial pacemaker
th various lighting fittings. What type of B. β-adrenoreceptor blocking agents
lighting fittings is the most appropriate in C. Cholinolytics
respect of hygienic norms? D. Calcium antagonists
E. Cardiac glycosides
A. Indirect light fittings
B. Direct light fittings 101. A 49-year-old patient complai-
C. Semi-reflected light fittings ns of dyspnea, cough. There are no
D. Ambient light fittings sputum discharges. He has repeatedly
E. Combined light fittings used salbutamol and intal but with no
effect. Objectively: he is only able to sit
98. A 25-year-old patient complains of while leaning on the table. Cyanosis of
general weakness, dry cough, sweating, face, acrocyanosis are present. Breathi-
subfebrile temperature. Objectively: lung ng is shallow, laboured, in some parts it
auscultation reveals vesicular resiration cannot be auscultated; there are diffuse
with no wheezing. Fluorogram shows rales, expiration is significantly prolonged.
focal shadows of high intensity in the 1- Heart sounds are muffled, tachycardia is
2 segments of the right lung. Mantoux test present. Ps - 112/min., AP- 110/70 mm Hg.
gave a reaction of 16 mm of induration. Liver is located near the costal arch. There
What clinical form of tuberculosis is most are no peripheral edemata. What is your
likely? provisional diagnosis?
A. Focal A. Status asthmaticus
B. Infiltrative B. Chronic obstructive bronchitis
C. Disseminated C. Bronchiale asthma, moderate gravity
D. Tuberculoma D. Foreign object aspiration
E. Miliary E. Cardiac asthma
99. A 22-year-old patient is a clerk. His 102. A 42-year-old patient with acute
working day runs in a conditioned room. haemorrhage and class III blood loss
In summer he was taken by an acute di- underwent blood transfusion and got 1,8
sease with the following symptoms: fever, l of preserved blood and erythromass
dyspnea, dry cough, pleural pain, myalgia, of the same group and Rh. After the
arthralgia. Objectively: moist rales on the transfusion the patient complained of
right, pleural friction rub. X-ray picture unpleasant retrosternal sensations, his
showed infiltration of the inferior lobe. In arterial pressure dropped to 100/60 mm
blood: WBC - 11 · 109 /l, stab neutrophi- Hg, there appeared convulsions. Blood
ls - 6%, segmented neutrophils - 70%, serum calcium was at the rate of 1,7 milli-
Krok 2 Medicine 2012 15

mole/liter. What is the mechanism of this from the maternity house a 2-year-old
complication development? patient consulted a doctor about body
temperature rise up to 39o C, pain in the
A. Citrate binds calcium ions, right breast. Objectively: the mammary
hypocalcemia impairs myocardial function gland is enlarged, there is a hyperemized
B. Citrate is cardiotoxic and nephrotoxic area in the upper external quadrant, in the
C. Citrate causes the development of same place there is an ill-defined indurati-
metabolic acidosis on, lactostasis, fluctuation is absent.
D. The increased citrate rate causes Lymph nodes of the right axillary regi-
convulsions on are enlarged and painful. What is the
E. Citrate binds potassium causing severe most likely diagnosis?
hypokalemia
A. Lactational mastitis
103. A 27-year-old patient with a knife B. Abscess
stomach wound has been delivered to C. Erysipelas
a hospital 4 hours after injury. Objecti- D. Dermatitis
vely: the patient is in grave condition. E. Tumour
Ps- 120 bpm, weak. AP- 70/40 mm Hg.
Laparotomy revealed a lot of liquid blood 107. During the dynamic observation over
in the abdominal cavity. The patient has a parturient woman in the second stage of
been found to have bleeding from the labor it was registered that the fetal heart
mesenteric vessels of the small intestine. rate fell down to 90-100/min and didn’t
Damage to hollow organs has not been come to normal after contractions. Vagi-
revealed. What is the best way to restore nal examination revealed the complete
the blood loss? cervical dilatation, the fetal head filling
the entire posterior surface of the pubic
A. Autoblood reinfusiont symphysis and sacral hollow; the sagi-
B. Erythromass transfusion ttal suture lied in the anteroposterior di-
C. Transfusion of washed erythrocytes ameter of the pelvic outlet, the posteri-
D. Transfusion of fresh frozen plasma or fontanelle was in front under the
E. Rheopolyglucinum transfusion pubic arch. What plan for further labour
management should be recommended?
104. A 42-year-old woman has had
hyperpolymenorrhea and progressing A. Application of forceps minor
algodismenorrhea for the last 10 years. B. Caesarean section
Gynaecological examination revealed no C. Episiotomy
changes of uterine cervix; discharges are D. Application of cavity forceps
moderate, of chocolate colour, uterus is E. Stimulation of labour activity by
slightly enlarged and painful, appendages intravenous injection of oxytocin
are not palpable, the fornices are deep
and painless. What is the most likely di- 108. A female patient consulted a
agnosis? dermatologist about the rash on the trunk
and extremities. Objectively: interdigital
A. Uterine endometriosis folds, flexor surfaces of wrists and navel
B. Uterine carcinoma region are affected with pairs of nodulo-
C. Subserous uterine fibromyoma cystic eruptions and crusts. The rash is
D. Endomyometritis accompanied by skin itch that is getting
E. Adnexal endmetriosis stronger at night. What external treatment
should be administered?
105. The results of 5 year monitoring
allowed to estimate the level of envi- A. 20% benzyl benzoate emulsion
ronmental influence upon health indi- B. 5% sulfuric ointment
ces of popultaion. What statistic method C. 2% sulfuric paste
should be chosen? D. 5% naphthalan ointment
E. 5% tetracycline ointment
A. Calculation of correlation coefficient
B. Calculation of conformity coefficient 109. An ambulance had been called
C. Calculation of coefficient of difference to a 48-year-old man. According to
validity his relatives, the patient had had three
D. Calculation of regression coefficient attacks of unconsciousness accompanied
E. Calculation of dynamic indices by convulsions within 24 hours. On exami-
nation the doctor witnessed the followi-
106. On the tenth day after discharge ng attack: the patient lost consciousness
Krok 2 Medicine 2012 16

and fell to the floor, developed tonic, then pulse is present. Ps- 185 bpm, of poor
clonic convulsions of trunk and extremiti- volume. AP- 75/40 mm Hg. ECG taken
es. The attack lasted 1 minute and ended during an attack shows ectopic P waves,
with involuntary urination. Specify the ki- QRS wave is not deformed. At the end
nd of attack: of an attack a compensatory pause is
observed. The most likely cause of the
A. Epileptic seizure attack is:
B. Syncope
C. Vegetative crisis A. Paroxysmal atrial tachycardia
D. Coma B. Sinus tachycardia
E. Attack of hysteria C. Paroxysmal ventricular tachycardia
D. Complete AV-block
110. A patient is 60-year-old, retired, E. Atrial fibrillation
worked as deputy director of a research
institute. Behavioural changes appeared 2 113. A 1-month-old child became restless
years ago after the death of her husband: and presented with an increase in head
she stopped looking after herself and sweating. It’s known from the history that
leaving the house; then she stopped to the child has been fed with cow’s milk
clean the apartment and cook. Mental since birth (September 5). Examination
status: temporal disorientation. The pati- revealed craniotabes. A doctor admini-
ent does not understand many of the stered a course of UV radiation. Decide,
questions, is confused; does not know if the child needs ergocalciferol:
how to cook soup or fasten a button.
Her speech is characterized by stumbli- A. 2-2,5 months after the UVR withdrowal
ng and logoclonia. She does not recognize B. Does not need
doctors, fellow patients. She cries a lot but C. In combination with UVR
can not explain the reason for tears. What D. Immediately after the UVR withdrowal
is the mechanism of this pathology? E. A month after the UVR withdrowal
A. Atrophy of the cerebral cortex 114. Two hours ago a 38-year-old pati-
B. Atherosclerotic changes in cerebral ent got pain in his right shin. He was
vessels diagnosed with popliteal artery emboli-
C. Serotonin deficiency sm, acute arterial insufficiency of grade I.
D. Impaired conversion of dopamine to What is the most appropriate therapeutic
noradrenaline tactics?
E. Disorder of melatonin metabolism
A. Embolectomy
111. A 26-year-old patient has abused B. Destruction of the embolus by the
alcohol since the age of 16, needs a catheter
morning-after drink to cure hangover. He C. Resection of the popliteal artery
takes alcohol nearly every day, "a little at D. Bypass grafting
a time". Twice a week he gets severely E. Amputation at the middle of shin
drunk. The patient works as a motor
mechanic, over the last 2 years work 115. A child is 2 days old. He was born wi-
conflicts have become more frequent. th a weight of 2900 kg, body length of 50
What medical and tactical actions should cm. On examination the skin is intensely
be taken in this case? red, elastic, with preserved turgor. Pueri-
le respiration is present. Respiration rate
A. Voluntary consultation and treatment - 40/min, cardiac sounds are rhythmic,
at an addiction clinic sonorous. HR- 138/min. The abdomen is
B. Compulsory treatment soft. The liver extends 2 cm below the
C. Referral to treatment at an activity costal margin. Diuresis is sufficient. Stool
therapy centre is in form of meconium. What is the most
D. Referral to medical-social expert likely diagnosis?
commission for assessment of his working
ability A. Physiological erythema of the newborn
E. Consultation with a psychologist B. Toxic erythema of the newborn
C. Neonatal phlegmon
112. An 8-year-old girl periodically has D. Erysipelas
sudden short-term heart pain, sensation E. Exfoliative Ritter’s dermatitis
of chest compression, epigastric pain, di-
zziness, vomiting. Objectively: the patient 116. A 47-year-old female patient
is pale, respiratory rate - 40/min, jugular complains of leg heaviness, fatigue when
Krok 2 Medicine 2012 17

standing and walking. This feeling di- recommendations should doctor gi-
sappears when she takes a horizontal posi- ve to mother to prevent this disease
tion. Objectively: dilatation of the superfi- recurrence?
cial veins of the left shin and thigh with
pigmentation and trophic skin disorders. A. Strict following of feeding regimen
What functional test should the exami- B. Common cold prophilaxis
nation be started with? C. Feces observation
D. Gastro-intestinal disease prevention
A. Trendelenburg’s test E. Hardening of the child
B. Pratt test 2
C. Pratt test 1 121. A 40-year-old patient underwent an
D. Sheinis test operation for a lumbar phlegmon. Body
E. Perthes’ test temparature rose again up to 38o C, he
got intoxication symptoms, there was an
117. A 28-year-old patient complains of di- increase of leukocyte number in blood.
scomfort, acute pain in the lower third of The wound that was nearly free from
the left labia majora. The disease began necrotic tissues and full of granulations
suddenly after menstruation. Objectively: started to discharge pus, the granulations
body temperature is 38o C. The left labia turned pale. What complication dveloped
majora has a formation to 3 cm diameter, in this patient?
with hyperemic surface, extremely painful
to the touch, with symptoms of fluctuati- A. Sepsis
on. What is the most likely diagnosis? B. Putrid phlegmon
C. Erysipelas
A. Acute bartholinitis D. Allergic reaction
B. Vulvar cancer E. Erysipeloid
C. Vulvar fibroid
D. Bartholin gland cyst 122. An emergency situation at a chemical
E. Hypertrophy of the labia plant caused acute occupational intoxi-
cation. A doctor who revealed the case
118. A 40 week pregnant secundipara is of "acute occupational disease (intoxicati-
28 years old. Contractions are very active. on)"must notify the following authority:
Retraction ring is at the level of navel, the
uterus is hypertonic, in form of hourglass. A. Sanitary and epidemiological station
On auscultation the fetal heart sounds B. Plant administration
are dull, heart rate is 100/min. AP of the C. Trade union committee of the plant
parturient woman is 130/80 mm Hg. What D. Medical unit of the plant
is the most likely diagnosis? E. Ministry of Health of Ukraine
A. RIisk of hysterorrhexis 123. A 42-year-old builder consulted a
B. Mazolysis doctor about a foot injury with a nail
C. Disturbed labour that he got in the morning of the same
D. Complete hysterorrhexis day. The wound was washed with water.
E. Attack of eclampsia Three years ago he was vaccinated agai-
nst tetanus. Examination established sati-
119. A neonatologist examining a full- sfactory condition of the patient. The
term mature baby revealed the shorteni- left foot was slightly edematic, there was
ng and external rotation of the newborn’s a stab wound on the sole. In order to
lower extremity. Clinical examination prevent tetanus it is primarily required
revealed positive Ortolani sign, symptom to:
of non-vanishing pulse, additional skin
folds on the internal surface of thigh. A. Give an intravenous injection of 0,5 ml
What is the most likely diagnosis? of tetanus anatoxin
B. Give an intravenous injection of 1 ml of
A. Congenital hip dislocation tetanus anatoxin, 3000 IU of antitetanus
B. Dysplasia of the hip joint serum
C. Varus deformity of the femoral neck C. Give an intravenous injection of 3000
D. Femoral neck fracture IU of antitetanus serum
E. Fracture of the femoral shaft D. Treat the wound with suds
E. Administer a course of antibiotic
120. A child is being discharged from therapy
the surgical department after conservati-
ve treatment of invagination. What 124. A 6-year-old child has duodenal ulcer.
Krok 2 Medicine 2012 18

What antibacterial drug should be co- A. Hypoproduction of gonadotropin


administered together with metronidazole B. Hyperproduction of estrogens
and De-Nol in order to eradicate Heli- C. Hyperproduction of androgens
cobacter pylori infection? D. Hypoproduction of progesterone
E. Hyperproduction of prolactin
A. Amoxicillin
B. Tetracycline 128. Hygienic expertise of a sample taken
C. Oleandomycin from the batch of grain revealed 2% of
D. Biseptol grains infected with microscopic Fusari-
E. Sulfadimethoxinum um fungi. On the ground of laboratory
analyses this batch of grain should be:
125. Against the background of angina
a patient has developed pain in tubular A. Sold without restrictions
bones. Examination revealed generalized B. Tested for toxicity
enlargement of lymph nodes, hepatoli- C. Used for forage production
enal syndrome, sternalgia. In blood: RBCs D. Used for ethanol production
- 3, 6 · 1012 /l, Hb- 87 g/l, thrombocytes E. Destroyed
- 45 · 109 /l, WBCs - 13 · 109 /l, blasts - 129. Among the inhabitants of a
87%, stab neutrophils - 1%, segmented workmen’s settlement located near an
neutrophils - 7%, lymphocytes - 5%, ESR industrial plant the cases of nervous and
- 55 mm/h. What is the most likely di- endocrine system diseases as well as renal
agnosis? diseases became more frequent. Blood
A. Acute leukemia analyses revealed a decrease in sulfhydryl
B. Erythremia groups. These pathologies might have
C. Chronic lymphocytic leukemia been caused by the following substance
D. Chronic myeloid leukemia released into the environment:
E. Multiple myeloma A. Mercury
126. A 51-year-old patient complains B. Cadmium
of having intensive bloody discharges C. Boron
from vagina for 15 days after delay D. Chromium
of menstruation for 2,5 months. In E. Lead
anamnesis: disorders of menstrual functi- 130. It is required to analyze the level
on during a year, at the same time she of daylight illumination in a ward of
felt extreme irritability and had sleep di- therapeutics unit. What device should be
sorders. US examination results: uterus applied to estimate the level of daylight
corresponds with age norms, appendages illumination?
have no pecularities, endometrium is 14
mm thick. What is the doctor’s tactics? A. Illuminometer
B. Anemometer
A. Diagnostic curettage of uterine cavity C. Katathermometer
B. Conservative treatment of bleeding D. Actinometer
C. Hysterectomy E. Psychrometer
D. Supravaginal amputation of uterus
without appendages 131. During the medical examination
E. TORCH-infection test at school the schoolchildren had to
undergo plantography. After the analysis
127. A 25-year-old female patient of footprints platypodia was found in 30%
complains about having amenorrhea of pupils. What is the percentage of the
for 3 years. She associates it with di- flatfoot isthmus?
fficult labour complicated by massive
hemorrhage. She also complains of loss A. 65%
of weight, hair fragility and loss, lack of B. 30%
appetite and depression. Objective exami- C. 50%
nation reveals no pathological changes of D. 55%
uterus and its appendages. What is the E. 45%
desease pathogenesis?
132. A factory’s sectorial doctor selects
a group of persons who often fall ill for
thorough monitoring. At the same time
he takes into consideration the number of
Krok 2 Medicine 2012 19

etiologically related cases with temporary A. The whole carcass should be technically
disability in each of the employees over disposed
the last year. An employee falls into this B. Meat can be sold without any restricti-
group if the number of sickness cases is: ons
C. The carcass should be used for the
A. 4 or more production of canned meat
B. 1 or more D. Meat should be disinfected by boiling
C. 2 or more E. Meat should be disinfected by freezing
D. 3 or more
E. 6 or more 137. A 28-year-old female patient
complains of having haemorrhage from
133. Half an hour ago a 67-year-old patient the genital tracts for 1 month. 6 months
with a hernia picked up a heavy object, ago she had natural delivery and gave bi-
which caused acute pain in the region rth to a girl weighing 3100 g. Objectively:
of hernia protrusion, the hernia couldn’t the uterus is enlarged to 9-10 weeks, mobi-
be reduced. Objectively: the hernia in le, painless, of heterogenous consistency.
the right inguinal region is roundish, ti- Examination reveals vaginal cyanosis,
ght, moderately painful; during palpati- anaemia and body temperature rise up
on it was reduced back to the cavity, the to 37, 8oC. There is a significant increase
pain was gone. Specify the further medical in hCG concentration in the urine. What
tactics: is your provisional diagnosis?
A. Inpatient surveillance A. Uterine chorionepithelioma
B. Immediate hernioplasty B. Pregnancy
C. Immediate laparotomy C. Hydatidiform mole
D. Planned hernioplasty a month later D. Endometritis
E. Planned hernioplasty a year later E. Uterine fibromyoma
134. Production areas of a greenhouse 138. The correlation between the servi-
complex have the following microcli- ce record and eosinophil concentration in
mate parameters: air temperature - 42oC, blood was studied in workers at dyeing
humidity - 98%, air velocity - 0,05 shops of textile factories. What index will
mps, temperature of enclosing surfaces be the most informative for the analysis of
- 15o C. Characterize the microclimate of this data?
production areas:
A. Correlation factor
A. Overheated B. Student’s criterion
B. Comfortable C. Standardized index
C. Cooling D. Fitting criterion
D. Uncomfortable E. Sign index
E. Satisfactory
139. A 43-year-old male patient undergoi-
135. A 54-year-old female patient ng treatment for peptic ulcer complai-
consulted a gynaecologist about bloody ns of weakness, dizziness, coffee-ground
discharges from the vagina for 1 month. vomiting, melena. After administration of
Last menstruation was 5 years ago. haemostatics the patient’s condition has
Gynaecological examination revealed no not improved, fresh blood has shown up
pathological changes. What is the tactics in the vomit, skin bruises of different sizes
of choice? have appeared. In blood: thrombocytes -
50 · 109 /l, Lee-White clotting time - 35 mi-
A. Diagnostic fractional curettage of uteri- nutes, APTT - 80 seconds. In this case it is
ne cavity most rational to administer the following
B. Colposcopy preparation:
C. USI
D. Cytosmear A. Fresh frozen plasma
E. Symptomatic therapy B. Heparin
C. Fibrinogen
136. Sanitary-veterinary examination of a D. Rheopolyglucinum
cow carcass revealed measle contaminati- E. Vikasol
on (2-3 measles per 10 cm2 ). What tactics
should be chosen in respect of this meat 140. A girl of 3 months presents wi-
consumption? th rhinitis, dyspnea, dry cough. These
manifestations has been observed for
Krok 2 Medicine 2012 20

two days. Objectively: the child has oesophageal stricture a patient developed
pale skin, acrocyanosis, shallow respi- acute retrosternal pain getting worse
ration at the rate of 80/min. Percussi- when throwing the head back and
on reveals handbox resonance over the swallowing. Objectively: dilatation of
whole surface of lungs, there are a lot of fi- the neck veins, dropped beat pulse, si-
ne rales. What is the most likely diagnosis? gns of purulent intoxication, oliguria,
emphysema of the upper portion of chest.
A. Acute bronchiolitis What disease can be suspected?
B. Pneumonia
C. Mucoviscidosis A. Suppurative mediastinitis
D. Foreign body of the airway B. Thrombosis of the superior vena cava
E. Acute bronchitis C. Pleural empyema
D. Acute myocardial infarction
141. A 30-year-old patient has been admi- E. Spontaneous pneumothorax
tted to the intensive care unit for multiple
bee stings. The skin is covered with cold 145. A 32-year-old patient consulted a
sweat. The pulse is felt just on the carotid doctor about being inable to get pregnant
arteries, 110 bpm, respiration is 24/min, for 5-6 years. 5 ago the primipregnancy
rhythmic, weakened. Which drug should ended in artificial abortion. After the vagi-
be given in the first place? nal examination and USI the patient was
diagnosed with endometrioid cyst of the
A. Adrenalin hydrochloride intravenously right ovary. What is the optimal treatment
B. Prednisolone intravenously method?
C. Adrenalin hydrochloride
intramuscularly A. Surgical laparoscopy
D. Dopamine intravenously B. Anti-inflammatory therapy
E. Tavegil intravenously C. Conservative therapy with estrogen-
gestagenic drugs
142. A 58-year-old patient complains D. Hormonal therapy with androgenic
about sensation of numbness, sudden hormones
paleness of II-IV fingers, muscle ri- E. Sanatorium-and-spa treatment
gidness, intermittent pulse. The pati-
ent presents also with polyarthralgia, 146. The objective of a statistical study
dysphagia, constipations. The patient’s was to find out the extent of seeking
face is masklike, solid edema of hands is medical care by the population. For this
present. The heart is enlarged; auscultati- purpose 300 residents of the area were
on revealed dry rales in lungs. In blood: interviewed. Information was collected by
ESR- 20 mm/h, crude protein - 85/l, γ- means of a special questionnaire. What
globulines - 25%. What is the most likely method of collecting information was
diagnosis? used by researchers?
A. Systemic scleroderma A. Anamnestic
B. Dermatomyositis B. Immediate registration
C. Rheumatoid arthritis C. Immediate examination
D. Systemic lupus erythematosus D. Doing extracts
E. Raynaud’s disease E. -
143. A 45-year-old man has been exhi- 147. An 8-month-old baby has had
biting high activity for the last 2 weeks, problems with nasal breathing and muco-
he became talkative, euphoric, had li- purulent discharge from the nose for a
ttle sleep, claimed being able "to save week. Examination reveals a rhinedema,
the humanity"and solve the problem muco-purulent discharges from the mi-
of cancer and AIDS, gave money the ddle nasal meatus as well as on the
starangers. What is the most likely di- back of pharynx. What disease are these
agnosis? symptoms most typical for?
A. Maniacal onset A. Ethmoiditis
B. Panic disorder B. Sphenoiditis
C. Agitated depression C. Maxillary sinusitis
D. Schizo-affective disorder D. Frontitis
E. Catatonic excitation E. Hemisinusitis
144. After the pneumatic dilatation of 148. A city’s population is 400000 inhabi-
Krok 2 Medicine 2012 21

tants in 2005 there were registered 5600 A. Individual records of pregnant and
deaths, including 3300 cases caused by postpartum women
cardiovascular diseases, 730 - by tumours. B. Prenatal records
Which of the following indicators allows C. Medical records of outpatients
to characterize the share of the circulatory D. Labour and delivery record
system diseases as the cause of death in E. Neonatal record
the city?
153. Examination of placenta revealed a
A. Extensive index defect. An obstetrician performed manual
B. Intensive index investigation of uterine cavity, uterine
C. Index of relative intensity massage. Prophylaxis of endometritis in
D. Demonstrativeness index the postpartum period should involve
E. Ratio following actions:
149. It is planned to organize a rural A. Antibacterial therapy
outpatient clinic. The patients will be able B. Instrumental revision of uterine cavity
to visit the doctors of the following speci- C. Haemostatic therapy
alities: D. Contracting agents
E. Intrauterine instillation of dioxine
A. Therapeutist, dentist, pediatrician,
obstetrician-gynecologist 154. A 35-year-old patient complains
B. Therapeutist, pediatrician, neurologist of heartburn, sour eructation, burning,
C. Pediatrician, obstetrician-gynaecologist, compressing retrosternal pain and pain
ophthalmologist along the esophagus rising during forward
D. Pediatrician, therapeutist, bending of body. The patient hasn’t been
ophthalmologist examined, takes Almagel on his own initi-
E. Obstetrician-gynaecologist, therapeutist ative, claims to feel better after its taking.
Make a provisional diagnosis:
150. A 10-year-old girl was admitted to a
hospital with carditis presentations. It is A. Gastroesophageal reflux disease
known from the anamnesis that two weeks B. Functional dyspepsia
ago she had exacerbation of chronic tonsi- C. Cardiospasm
llitis. What is the most likely etiological D. Gastric ulcer
factor in this case? E. Duodenal ulcer
A. Streptococcus 155. All the joints on the left elbow of
B. Staphylococcus a newborn are extended, the whole arm
C. Pneumococcus hangs vertically along the trunk with the
D. Klebsiella forearm pronated. Active movements in
E. Proteus the elbow joint are absent but present in
the shoulder joint. The hand is flattened,
151. In a rural health district a child di- atrophied, cold to the touch, hangs
ed in the first month of life. In order to passively. Grasp reflex and hand-mouth
analyze this situation it was necessary reflex on the affected side are missing.
to complete expert evaluation of medi- Haemogram values are normal. What is
cal records. Which medical document was the most likely diagnosis?
analyzed in the first place?
A. Inferior distal obstetrical paralysis
A. Child developmental history record B. Osteomyelitis
B. Neonatal record C. Proximal obstetrical paralysis
C. Record of vaccinations D. Complete obstetrical paralysis
D. Medical record of an outpatient E. Hypoxic-ischemic encephalopathy
E. Child’s medical record
156. Head circumference of a 1-month-
152. The head of prenatal care department old boy with signs of excitement is 37 cm,
intends to complete expert evaluation of prefontanel is 2x2 cm large. After feeding
compliance with medical and technologi- the child regurgitates small portions of mi-
cal standards of pregnancy follow-up. lk; stool is normal in respect of its volume
What documents must be verified for this and composition. Muscle tonus is within
purpose? norm. What is the most likely diagnosis?
Krok 2 Medicine 2012 22

A. Pylorospasm A. To perform the Heimlich manoever


B. Meningitis B. To perform conicotomy immediately
C. Pylorostenosis C. To send for an anesthesiologist and wait
D. Microcephaly for him
E. Craniostenosis D. To use an inhalation of β2 -adrenoceptor
agonist
157. A 35-year-old victim of a road E. To make a subcutaneous injection of
accident has got an injury of the ri- dexamethasone
ght side of his chest. Objectively: respi-
ration rate - 28-30/min, respiration is 160. A patient undergoing treatment
shallow, restricted respiratory excursi- for the left-sided destructive pneumonia
on and acrocyanosis are present. Ps- presents with deterioration of his general
110 bpm, AP- 90/60 mm Hg. Respi- condition, progressing dyspnea, cyanosis.
ratory sounds over the right lung cannot Objectively: the left side of chest is not
be auscultated. Chest radiograph shows involved in respiration, breathing sounds
fractures of the VI-VII ribs on the right, cannot be auscultated. Radiograph shows
the right pleural cavity contains both air a shadow reaching the 5 rib with a hori-
and fluid, with the fluid at about the level zontal fluid level and a radiolucency above
of the V rib, the shadow of the mediasti- it, the mediastinum is displaced to the ri-
num is displaced to the left. What first aid ght. What is the medical tactics?
should be provided to the victim?
A. Thoracostomy
A. Puncture of the pleural cavity B. Open thoracotomy
B. Antibiotic administration C. Endotracheal intubation
C. Vagosympathetic blockade D. Infusion and antibacterial therapy
D. Artificial ventilation of lungs E. Emergency bronchoscopy
E. Urgent thoracotomy
161. A 10 week pregnant woman was
158. A 20-year-old patient complains of admitted to a hospital for recurrent pain
pain in the left lumbar region, arterial in the lower abdomen, bloody discharges
pressure rise up to 160/110 mm Hg. USI from the genital tracts. The problems
revealed that the structure and size of the turned up after ARVI. The woman was
right kidney were within age norms, there registered for antenatal care. Speculum
were signs of 3 degree hydronephrotic examination revealed cyanosis of vagi-
transformation of the left kidney. Doppler nal mucosa, clean cervix, open cervical
examination revealed an additional artery canal discharging blood and blood clots;
running to the lower pole of the kidney. the lower pole of the gestational sac was
Excretory urogram shows a narrowing visible. What tactics should be chosen?
in the region of ureteropelvic junction.
Specify the therapeutic tactics: A. Curettage of the uterus
B. Pregnancy maintenance therapy
A. Surgical intervention C. Expectant management, surveillance
B. Administration of spasmolytics D. Hysterectomy
C. Administration of ACE inhibitors E. Antiviral therapy
D. Kidney catheterization
E. Administration of β-blockers 162. A 29-year-old patient complains of
absent menstruation for a year, milk di-
159. A hospital admitted a patient wi- scharge from the nipples when pressed,
th coarse breathing (obstructed inspirati- loss of lateral visual fields. X-ray shows an
on), skin cyanosis, tachycardia and arteri- expansion of the sella turcica. What is the
al hypertension. He has a histrory of most likely cause of this condition?
bronchial asthma. An hour ago he was
having salbutamol inhalation and forgot A. Pituitary tumour
to remove a cap that was aspired whi- B. Mammary tumour
le taking a deep breath. What measures C. Functional disorder of the
should the doctor take? hypothalamic-pituitary-ovarian system
D. Ovarian tumor
E. Pregnancy
163. On the second day after preventi-
ve vaccination a 2-year-old boy presented
with abdominal pain without clear locali-
zation, body temperature rose up to 38o C.
Krok 2 Medicine 2012 23

On the third day the child got red papular A. Posttraumatic osteomyelitis
haemorrhagic eruption on the extensor B. Hematogenic osteomyelitis
surfaces of limbs and around the joints. C. Wound abscess
Knee joints were edematic and slightly D. Posttraumatic phlegmon
painful. Examination of other organs and E. Suture sinus
systems revealed no pathological changes.
What is the most likely diagnosis? 167. A 40-year-old female patient
complains of having a bulge on the anteri-
A. Haemorrhagic vasculitis or surface of neck for 5 years. Objecti-
B. Thrombocytopenic purpura vely: Ps- 72 bpm, arterial pressure - 110/70
C. Meningococcemia mm Hg, in the right lobe of thyroid gland
D. Urticaria palpation reveals a mobile 4x2 cm node,
E. DIC syndrome the left lobe is not palpable, the basal
metabolic rate is 6%. What is the most
164. A patient with uterine fibromyoma likely diagnosis?
sized up to 8-9 weeks of pregnancy
consulted a gynaecologist about acute A. Nodular euthyroid goiter
pain in the lower abdomen. Examination B. Nodular hyperthyroid goiter
revealed pronounced positive symptoms C. Riedel’s thyroiditis
of peritoneal irritation, high leukocytosis. D. Mixed euthyroid goiter
Vaginal examination revealed that the E. The median cervical cyst
uterus was enlarged up to 9 weeks of
pregnancy due to the fibromatous nodes, 168. A 14-year-old patient with signs of
one of which was mobile and extremely internal haemorrhage has been taken
painful. Appendages were not palpable. to a hospital after a fight. He has had
Discharges were mucous, coming in haemophilia A since childhood. He has
moderate amounts. What is the treatment been diagnosed with retroperitoneal
tactics? hematoma. What should be administered
in the first place?
A. Urgent surgery (laparotomy)
B. Surveillance and spasmolytic therapy A. Cryoprecipitate
C. Fractional diagnostic curettage of the B. Aminocapronic acid
uterine cavity C. Dried plasma
D. Surgical laparoscopy D. Platelet concentrate
E. Surveillance and antibacterial therapy E. Fresh blood

165. On the 6th day of life a child got 169. A patient is 14 years old. Cytochemi-
multiple vesicles filled with seropurulent cal study of punctate revealed 40% of
fluid in the region of occiput, neck and blasts, there was negative reaction to
buttocks. General condition of the chi- peroxidase and with Sudan black, positive
ld is normal. What disease should be reaction to glycogen. Specify the form of
suspected? acute leukemia:

A. Vesiculopustulosis A. Lymphoblastic
B. Impetigo neonatorum B. Myeloblastic
C. Miliaria C. Monoblastic
D. Impetigo D. Promyelocytic
E. Epidermolysis bullosa E. Undifferentiated

166. A 3-year-old male patient consulted 170. A 58-year-old patient complains of a


a family doctor 2 months after he had headache in the occipital region, nausea,
been operated for an open fracture of choking, opplotentes. The presentati-
brachial bone. Objectively: the patient’s ons appeared after a physical exerti-
condition is satisfactory, in the region of on. Objectively: the patient is excited.
the operative wound there is a fistula Face is hyperemic. Skin is pale. Heart
with some purulent discharge, redness, sounds are regular, the 2nd aortic sound
fluctuation. X-ray picture shows brachi- is accentuated. AP- 240/120 mm Hg, HR-
al bone destruction with sequestra. What 92/min. Auscultation reveals some fine
complication arose in the postoperative moisr rales in the lower parts of the lungs.
period? Liver is not enlarged. ECG shows signs of
hypertrophy and left ventricular overload.
What is the most likely diagnosis?
Krok 2 Medicine 2012 24

A. Complicated hypertensic crisis A. Echocardiography


B. Acute myocardial infarction, pulmonary B. ECG
edema C. Diuresis monitoring
C. Bronchial asthma exacerbation D. Monitoring of K+ , Na+ concentration
D. Uncomplicated hypertensic crisis in blood
E. Community-acquired pneumonia E. 24-hour monitoring of heart rhythm
171. A 59-year-old male patient with 174. Forensic medical expertise of corpse
essential hypertension of stage II is regi- of a newborn revealed: body weight 3500
stered with the dispensary department of g, body length 50 cm, the umbilical cord
a polyclinic. The patient regularly takes was smooth, moist, glossy, without any si-
ACE inhibitors and calcium antagonists. gns of drying. Hydrostatic tests were posi-
How often should a therapeutist examine tive. The test results are the evidence of:
this patient (except for exacerbation peri-
ods)? A. Live birth
B. Stillbirth
A. Every 3 months C. Primary atelectasis
B. Every 6 months D. Secondary atelectasis
C. Every 4 months E. Hyaline membrane disease
D. Once a year
E. Every 9 months 175. A 60-year-old patient complains of
nearly permanent sensation of heavi-
172. A 37-year-old patient complains of ness and fullness in the epigastrium,
pain in the lumbar spine that is getting that increases after eating, foul-smelling
stronger during walking; restricted mobi- eructation, occasional vomiting with food
lity, edema of the right side of abdomen. consumed 1-2 days ago, weight loss. 12
He has a history of focal tuberculosis. X- years ago he was found to have an ulcer
ray picture shows the destruction of the of pyloric channel. The patient has taken
adjacent surfaces of the 1-2 vertebral bodi- ranitidine for periodic hunger pain. The
es of the lumbar spine, vertebral body hei- patient’s condition has been deteriorati-
ght is decreased, intervertebral foramen ng over the last 3 months. Objectively:
is undetectable. Abdominal ultrasound splashing sound in the epigastrium is
reveals a 15x20 cm formation in the present. What kind of complication is it?
retroperitoneal space, there are echo si-
gnals of fluid presence. What is the most A. Pyloric stenosis
likely diagnosis? B. Penetration of gastric ulcer
C. Functional pyloric spasm
A. Tuberculous spondylitis of the lumbar D. Foreign body in the stomach (bezoar)
spine E. Malignization of gastric ulcer
B. Fracture of the 1-2 vertebral bodies of
the lumbar spine 176. A hospital admitted an 11-year-
C. Spinal metastases old boy diagnosed with medium-severe
D. Spondylolisthesis of the lumbar spine asthma, exacerbation period. In order
E. Osteochondrosis to arrest the attacks the boy was admi-
nistered broncholytic nebulizer therapy.
173. The condition of a 3-year-old child During the day the child’s condition
with acute non-rheumatic myocarditis has stabilized. What is the most appropriate
suddenly deteriorated: he presents with method for further monitoring of respi-
anxiety, acrocyanosis, peripheral edemata, ratory function in this patient?
dyspnea. Auscultation of lungs reveals
fine moist rales on both sides mainly A. Peak flowmetry
in the lower parts. AP- 65/40 mm Hg. B. Spirometry
HR- 150/min, heart sounds are muffled, C. Pneumotachometry
arrhythmic (extrasystole). Liver is +4 cm. D. Bronchodilatation tests
Oliguria is present. The child has been di- E. Veloergometry
agnosed with acute heart failure. Which
method of examination is most informati- 177. A full-term newborn was born wi-
ve for assessing the child’s status dynami- th body weight of 4000 g, body length of
cs? 57 cm. Reaction to the postnatal check
was absent. There was diffuse cyanosis,
heart rate of 80/min. What resuscitation
measures should be taken?
Krok 2 Medicine 2012 25

A. Start ALV with a mask a large portion of the esophagus. What is


B. Give 100% oxygen the most likely diagnosis?
C. Intubate the child and start ALV
D. Start tactile stimulation A. Esophageal carcinoma
E. Give an injection of naloxone B. Benign tumour
C. Esophageal achalasia
178. A 52-year-old patient works as a D. Peptic ulcer
secretary and has 30 year record of servi- E. Sideropenic dysphagia
ce. She complains of spasms in her ri-
ght hand during working and inability 182. A 27-year-old patient complai-
to type and write. Up to 80% of her ns of irritability, tearfulness, depressi-
work involves hand load. The patient has on, and sometimes aggressiveness,
been presenting with these symptoms for headache, nausea, vomiting, swelling
2 years. Objectively: the right hand is of the mammary glands. The menti-
tense, there is an increase in muscle tone, oned problems arise 5-6 days before
attempts to write cause spasms. Exami- menstruation and gradually progress until
nation revealed no pathological changes menstruation, 3 days after it the problems
of CNS. What is the most likely diagnosis? disappear. What is the most likely di-
agnosis?
A. Spastic form of coordination neurosis
B. Neuralgic form of coordination neurosis A. Premenstrual syndrome
C. Paretic form of coordination neurosis B. Premature pathological climacterium
D. Hysteric neurosis C. Secondary psychogenic amenorrhea
E. Chronic manganese intoxication D. Preclimacterium syndrome
E. Algomenorrhea
179. A multigravida with Rh-
isosensitization was found to have a 183. A 2-year-old child in a satisfactory
decrease in anti-Rh titer from 1:32 condition periodically presents with
to 1:8 at 33-34 weeks of gestation. moderate proteinuria, microhematuria.
Ultrasound revealed double contour USI results: the left kidney is
of head, ebnlargement of fetal liver, undetectable, the right one is enlarged,
placental thickness of 50 mm. The pati- there are signs of double pyelocaliceal
ent has indication for: system. What study is required to speci-
fy the diagnosis?
A. Premature delivery
B. Course of desensitizing therapy A. Excretory urography
C. Plasmapheresis B. Micturating cystography
D. Repeated (after 2 weeks) USI C. Retrograde urography
E. Administration of anti-Rh gamma D. Doppler study of renal vessels
globulin E. Radioisotope renal scan
180. A 22-year-old vegetarian patient 184. An emergency doctor has diagnosed
with signs of malnutrition consulted a a 32-year-old woman with generalized
doctor about smell and taste distortion, convulsive status epilepticus. The deteri-
angular stomatitis. Objectively: marked oration in the patient’s condition is caused
blue sclerae. The patient was diagnosed by a sudden gap in the epilepsy treatment.
with iron deficiency anemia. What is the Specify the doctor’s further tactics:
dominating clinical syndrome?
A. Hospitalization in the intensive care
A. Sideropenic unit
B. Anaemic B. Hospitalization in the department of
C. Haemologic neurology
D. Haemolytic C. Hospitalization in the department of
E. Myelodysplastic neurosurgery
D. Outpatient monitoring by a
181. A patient complains of retrosternal neuropathologist
pain, difficult swallowing, over 10 E. Outpatient monitoring by a
kg weight loss within three months, neurosurgeon
general weakness. In blood: hypochromic
anaemia, neutrophilic leukocytosis. In 185. A 75-year-old male patient complains
feces: weakly positive Gregersen’s reacti- of slight pain in the right iliac region. The
on. On esophagram a filling defect with abdominal pain arose 6 days ago and was
ill-defined serrated edges shows up along accompanied by nausea. Surgical exami-
Krok 2 Medicine 2012 26

nation revealed moist tongue, Ps- 76 bpm. The serum prolactin level is increased.
AP- 130/80 mm Hg. Abdomen was soft, What is the most likely diagnosis?
slightly painful in the right iliac region
on deep palpation, the symptoms of the A. Hyperprolactinemia
peritoneum irritation were doubtful. In B. Hypothyroidism
blood: RBCs - 4, 0 · 1012 /l, Hb- 135 g/l, C. Polycystic ovary syndrome
WBCs - 9, 5 · 109 /l, stab neutrophils - 5%, D. Pituitary adenoma
segmentonuclear - 52%, lymphocytes - E. Sheehan syndrome
38%, monocytes - 5%, ESR - 20 mm/h. 189. During self-examination a 22-year-
Specify the doctor’s further tactics: old patient revealed a mammary tumour.
A. Emergency operation for acute Palpation revealed a firm, painless, freely
appendicitis mobile formation up to 2 cm, peripheral
B. Hospitalization, dynamic surveillance lymph nodes were not changed. USI
C. Send the patient home results: in the superior external quadrant
D. Refer the patient to a district therapist of the right mammary gland there was a
E. Administration of additional examinati- big formation of increased echogenicity,
on: abdominal ultrasound, x-ray contrast sized 18x17 mm. The patient was provi-
study of the gastrointestinal tract sionally diagnosed with fibroadenoma.
What is a doctor’s further tactics?
186. A 56-year-old patient was undergoing
a surgery for suture repair of perforated A. Surgical removal of the tumour prior to
ulcer. During the operation the cardi- pregnancy
omonitor registered ventricular fibrillati- B. Dynamic follow-up
on. The first-priority measure should be: C. Surgical treatment after pregnancy
D. Radical mastectomy
A. Electrical defibrillation E. Nonsteroid anti-inflammatory drugs,
B. Injection of adrenalin oral contraceptives
C. Injection of lidocaine
D. Injection of atropine 190. A 42-year-old female lives in the
E. Injection of calcium chloride basement, is unemployed, undernouri-
shed. She complains of having general
187. On the 10th day postpartum a weakness, hair loss, brittle nails for six
puerperant woman complains of pain and months, likes to eat chalk. Objectively:
heaviness in the left mammary gland. the patient is emaciated, pale, has dry
Body temperature is 38, 8o C, Ps- 94 bpm. skin. Peripheral lymph nodes are not
The left mammary gland is edematic, enlarged. Liver is +1,5 cm. In blood: RBCs
the supero-external quadrant of skin - 1, 8 · 1012/l, Hb- 62 g/l, colour index - 0,78,
is hyperemic. Fluctuation symptom is reticulocytes - 0,5o /oo , ESR- 18 mm/h.
absent. The nipples discharge drops of Leukogram exhibits no pathology. What
milk when pressed. What is a doctor’s is a provisional diagnosis?
further tactics?
A. Nutritional iron deficiency anaemia
A. Antibiotic therapy, immobilization and B. Chronic hepatitis
expression of breast milk C. B12 -deficiency anaemia
B. Compress to both mammary glands D. Acquired haemolytic anaemia
C. Inhibition of lactation E. Congenital haemolytic anaemia
D. Physiotherapy
E. Opening of the abscess and drainage of 191. A 78-year-old patient complains of
the mammary gland severe pain in the lower abdomen, peri-
neum and rectum; intense urge to urinate
188. A 30-year-old female patient and inability to urinate without assistance.
complains of milk discharge from the Abdomen palpation reveals a moderately
mammary glands, 5-month absence of painful globular formation above the
menstruation. She had one physiologi- pubis, there is percussion dullness over
cal labour four years ago. Objectively: the formation. What is the most likely di-
mammary glands are normally developed. agnosis?
Bimanual examination reveals that the
uterus is decreased in size, the ovaries
are of normal size. MRI-scan shows no
cerebral pathologies. Concentration of
thyroid-stimulating hormone is normal.
Krok 2 Medicine 2012 27

A. Acute urinary retention ne medical examination he presents no


B. Chronic urinary retention problems. In blood: RBCs - 4, 5·1012/l, Hb-
C. Chronic incomplete urinary retention 80 g/l, WBCs - 2, 8 · 109 /l, thrombocytes -
D. Paradoxical ischuria 30 · 109 /l. Decide if this person can work
E. Cystitis with sources of ionizing radiation:
192. A 3-year-old child with ARVI had A. Working with radioactive substances
been administered biseptol, paracetamol, and other sources of ionizing radiation is
nazoferon. On the third day of treatment contraindicated
the baby’s condition deteriorated: B. The patient is allowed to work with
he developed sore throat, stomatitis, radioactive substances
conjunctivitis, hypersalivation, painful C. The patient can only work with radi-
dark red spots on the neck, face, chest oactive substances of low activity
and legs, then the spots were replaced wi- D. The patient can be allowed to work
th vesicles. Examination revealed lesions after an extended medical examination
of mucous membranes around the mouth E. The patient is allowed to work with
and anus. What is your provisional di- radioactive substances for the limited
agnosis? period of time
A. Stevens-Johnson syndrome 196. A patient who undergoes treatment
B. Atopic dermatitis at a tuberculosis clinic has complained
C. Chickenpox of having progressing headache for the
D. Serum sickness last 3 weeks. Neurological examination
E. Bullous dermatitis reveals rigidity of occipital muscles, no
193. Preventive examination of a 50-year- focal symptoms. What is your provisional
old woman revealed a dense tumour of diagnosis?
the right mammary gland up to 5 cm in di- A. Tuberculous meningitis
ameter without distinct outlines. The skin B. Chorea minor
over the tumour looked like lemon peel. C. Brain tumour
Palpation revealed a lymph node in the D. Myelitis
axillary region. What is the most likely di- E. Convexital arachnoiditis
agnosis?
197. A patient with chronic suppurati-
A. Breast cancer ve otitis has developed severe headache,
B. Lactocele vomiting, body temperature rise. The
C. Diffuse mastopathy meningeal symptoms are present. There
D. Mastitis are no focal neurological symptoms. The
E. Breast lipoma further tactics of a doctor should be:
194. A 20-year-old patient complains of A. Urgent hospitalization and diagnostic
breath shortness, continuous dull heart lumbar puncture
pain, irritability. Objectively: general B. Skull radiography
condition is satisfactory, the pulse is labi- C. Regular medical check-up
le, AP- 130/60 mm Hg. ECG shows D. Administration of anti-inflammatory
repolarization disorder. The patient has drugs
been diagnosed with cardiac-type neuroci- E. Referral for a consultation with
rculatory dystonia. The patient should otolaryngologist
receive treatment under the following
conditions: 198. A 12-year-old child had three attacks
of acute rheumatic fever accompani-
A. Outpatient treatment ed by carditis. Examination revealed
B. Inpatient treatment at the therapeutic the symptoms of chronic tonsillitis, mi-
department tral insufficiency, carious teeth. What
C. Inpatient treatment at the cardiology is the optimal method of secondary
department prophylaxis?
D. Inpatient treatment at the cardiac
surgery department
E. Inpatient treatment at the psychiatric
department
195. A 38-year-old male works within the
range of ionizing radiation. At a routi-
Krok 2 Medicine 2012 28

A. Year-round bicillin prophylaxis till the A. Physiological III sound


age of 25 B. The sound of the mitral valve opening
B. Course of cardiotrophic drugs twice a C. Protodiastolic gallop rhythm
year D. Pericardial diastolic sound
C. Year-round bicillin prophylaxis for 3 E. The sound of the tricuspid valve opening
years
D. Tonsillectomy 200. A patient’s condition is getting worse
E. Oral cavity sanitation towards evening: she becomes excited,
complains of "internal anxiety", "a wei-
199. Routine examination of a 16-year- ght on her heart", foreboding of evil -
old boy revealed the presence of three "something evil will happen to me or my
heart sounds on auscultation. The third family". The patient is sad, melancholic,
sound is low and occurs in early diastole, has poor appetite and sleep disorders.
there is no additional murmur. In history: Specify the kind of mental disorder:
pneumonia six months ago. The pati-
ent presents no problems. Examination A. Anxious depression
revealed hyposthenia, underdevelopment B. Somatized depression
of muscles. Laboratory and instrumental C. Endogenous depression
studies reveald no peculiarities. What is D. Hypochondriac depression
the origin of the additional heart sound? E. Agitated depression
Krok 2 Medicine 2013 1

1. A 28-year-old patient complains of


infertility. The patient has been marri- A. Right-sided tubal pregnancy
ed for 4 years, has regular sexual life B. Right ovary apoplexy
and does not use contraceptives but has C. Acute right-sided salpingoophoritis
never got pregnant. Examination revealed D. Pelvioperitonitis
normal state of the genitals, tubal patency. E. Incipient abortion
Basal body temperature recorded over the
course of 3 consecutive menstrual cycles 5. 5 weeks after hypothermia a 22-year-old
appeared to have a single phase. What is patient developed fever, weakness, muscle
the most likely cause of infertility? pain, inability to move independently.
Objectively: tenderness, induration of
A. Anovulatory menstrual cycle shoulder and shin muscles, restricted acti-
B. Immunological infertility ve movements, erythema on the anteri-
C. Genital endometriosis or surface of chest. There is a periorbi-
D. Chronic salpingoophoritis tal edema with heliotropic erythema.
E. Ovulatory menstrual cycle Gottron’s sign is present. What study is
required to verify the diagnosis?
2. A 40-year-old female patient has been
hospitalized for attacks of asphyxia, cough A. Muscle biopsy
with phlegm. She has a 4-year history of B. Aminotransferase activity
the disease. The first attack of asphyxia C. Pneumoarthrography
occurred during her stay in the countrysi- D. ASO titre
de. Further attacks occurred while cleani- E. Rheumatoid factor
ng the room. After 3 days of inpatient
treatment the patient’s condition has si- 6. A 24-year-old patient consulted a
gnificantly improved. What is the most doctor about enlarged submandibular
likely etiological factor? lymph nodes. Objectively: submandi-
bular, axillary and inguinal lymph nodes
A. Household allergens are enlarged. Chest radiograph shows
B. Pollen enlarged mediastinal lymph nodes. In
C. Infectious blood: RBCs - 3, 4 · 1012 /l, Hb- 100 g/l,
D. Chemicals colour index - 0,88, thrombocytes - 190 ·
E. Psychogenic 109 /l, WBCs - 7, 5 · 109/l, eosinophils -
8%, stab neutrophiles - 2%, segmented
3. A 13-year-old girl complains of peri- neutrophiles - 67%, lymphocytes - 23%,
odic prickly pain in the heart regi- ESR - 22 mm/h. What study is required to
on. Percussion revealed no changes of verify the cause of lymphadenopathy?
cardiac borders. Auscultation revealed
arrhythmic enhanced heart sounds, A. Open biopsy of lymph nodes
extrasystole at the 20-25 cardiac impulse. B. Ultrasound examination of the
ECG showed the sinus rhythm, impai- abdomen
red repolarization, single supraventricular C. Mediastinal tomography
extrasystoles at rest. What is the most li- D. Puncture biopsy of lymph nodes
kely diagnosis? E. Sternal puncture
A. Vegetative-vascular dysfunction 7. A patient operated for acute paraprocti-
B. Rheumatism tis undergoes antibacterial and detoxi-
C. Nonrheumatic carditis fication therapy, the local course of the
D. Myocardial degeneration disease has the positive dynamics. Since
E. Intoxication syndrome the operation the patient has had chi-
lls, pyrexia, tachycardia, euphoria for five
4. A 28-year-old patient has been taken days. The doctor suspected sepsis. What
to a hospital for acute pain in the lower study will confirm the diagnosis?
abdomen. There was a brief syncope.
The delay ofmenstruation is 2 months. A. Blood culture for a pathogen
Objectively: the patient has pale skin, B. X-ray of lungs
AP- 90/50 mm Hg, Ps- 110/min. Lower C. Liver ultrasound
abdomen is extremely painful. Vaginal D. Determining the rate of microbial
examination reveals uterus enlargement. contamination of wound
There is positive Promtov’s sign. Right E. Determining the rate of average-weight
appendages are enlarged and very pai- molecules
nful. Posterior vault hangs over. What is
the most likely diagnosis? 8. A painter working at a motorcar
Krok 2 Medicine 2013 2

plant has been diagnosed with A. Tuberculous meningitis


moderately severe intoxication with ami- B. Secondary purulent meningitis
de compounds of benzene. The in-patient C. Epidemic cerebrospinal meningitis
treatment resulted in a considerable D. Serous meningitis
health improvement. What expert deci- E. Pneumococcal meningitis
sion should be made in this case?
11. A 13-year-old girl has a 5-year histrory
A. The patient should be issued a sick list of pain in the right hypochondrium irradi-
for out-patient treatment ating to the right shoulder blade. The pain
B. The patient may get back to work attacks are usually associated with diet vi-
providing he will keep to hygiene and olations, they are short and can be easily
sanitary regulations relieved by antispasmodic drugs. During a
C. The patient should be referred to pain attack, palpation of the abdomen is
the medio-social expert commission for painful, the pain is most intensive in the
evaluation of percentage of work capicty projection of the gallbladder. What is the
loss most likely diagnosis?
D. The patient should be referred to
the medio-social expert commission for A. Biliary dyskinesia
attributing the disability group because of B. Chronic cholecystitis
an occupational disease C. Chronic gastroduodenitis
E. - D. Chronic pancreatitis
E. Duodenal ulcer
9. A 58-year-old female patient came
to the antenatal clinic complaining of 12. A 24-year-old patient got a puncture
bloody light-red discharges from the injury below the Poupart’s ligament
genital tracts. Menopause is 12 years. accompanied by intense arterial bleedi-
Gynaecological examination revealed ng. The best method to temporarily stop
age involution of externalia and vagi- the bleeding in the patient would be:
na; uterine cervix was unchanged, there A. Compression band
were scant bloody discharges from uteri- B. Esmarch’s tourniquet
ne cervix, uterus was of normal size; C. Maximum limb bending
uterine appendages were not palpable; D. Compressing a blood vessel with a
parametria were free. What is the most clamp
likely diagnosis? E. Wound suturing
A. Uterine carcinoma 13. A 64-year-old patient complains of
B. Atrophic colpitis severe pain in the right side of chest,
C. Abnormalities of menstrual cycle of dyspnea, dry cough which appeared
climacteric nature suddenly on exertion. Objectively: the ri-
D. Cervical carcinoma ght side of the chest lags behind in the act
E. Granulosa cell tumor of ovary of breathing. Percussion reveals tympanic
10. A 7-year-old boy had complai- sound. Auscultation reveals pronouncedly
ned of headache, nausea, fatigue for 3 diminished breath sounds on the right. Ps-
weeks. His condition gradually deteri- 100/min, weak, arrhythmic. AP- 100/50
orated, headache and general weakness mm Hg. Cardiac sounds are decreased.
progressed. The boy had bronchitis at What disease can be suspected in this pati-
the age of 3. His father has a history ent?
of pulmonary tuberculosis. Objectively: A. Right-sided pneumothorax
body temperature 37, 5oC, conscious, li- B. Right-sided hydrothorax
es supine, with the hip and knee flexed C. Right-sided dry pleurisy
to 90 degrees, nuchal rigidity +6 cm, D. Right-sided pleuropneumonia
partial ptosis of the right eyelid, the di- E. PATE
lated right pupil. General hyperalgesia
is present. Liquor: transparent, pressure 14. An 18-year-old patient since childhood
- 400 mm of water column, protein - suffers from bleeding disorder after mi-
1,5%, cytosis - 610/3 with predominant nor injuries. His younger brother also
lymphocytes, sugar - 1,22 mmol/l, chlori- has bleeding disorders with occasional
des - 500 mmol/l. What is the most likely haemarthrosis. Which laboratory test will
diagnosis? be informative for diagnosis verification?
Krok 2 Medicine 2013 3

A. Clotting time A. II degree flame burn of hands with an


B. Fibrinogen rate area of 4/
C. Blood clot retraction B. II degree flame burn of hands with an
D. Thrombocyte count area of 2/
E. Determination of prothrombin time C. IIIa degree flame burn of hands with an
area of 4/
15. A patient complains of fatigue, lack D. III degree flame burn of hands with an
of appetite, pain and burning sensation area of 4/
in the tongue, numbness of the distal li- E. IIb degree flame burn of hands with an
mbs, diarrhea. Objectively: pale skin with area of 2/
lemon-yellow tint, face puffiness, brown
pigmentation in the form of a "butterfly", 18. During the periodic medical exami-
bright red areas on the tongue. The li- nation an assembly fitter (works on
ver is 3 cm below the costal margin, soldering details) didn’t report any health
soft. Blood count: RBCs - 1, 5 · 1012 /l, problems. Closer examination revealed
colour index - 1,2, WBCs - 3, 8 · 109 /l, signs of asthenic-vegetative syndrome.
thrombocytes - 180 · 109 /l, eosinophils - Blood included red blood cells with
0%, stab neutrophils - 1%, segmented basophilic aggregations and a somewhat
neutrophils - 58%, lymphocytes - 38% higher number of reticulocytes, uri-
monocytes - 3%, RBC macrocytosis. ESR ne had a high concentration of delta-
- 28 mm/h. What diagnosis are these aminolevulinic acid. The complex of
presentation typical for? symptoms indicates the initial stage of
chronic intoxication with:
A. B12 -deficiency anemia
B. Iron deficiency anemia A. Lead
C. Aplastic anemia B. Manganese
D. Acute erythromyelosis C. Mercury
E. Chronic adrenal failure D. Tin
E. Ethanol
16. Medical examination of a 19-
year-old worker revealed generalized 19. A patient complains about sudden
lymphadenopathy mainly affecting the onsets of paroxysmal pain in the right
posterior cervical, axillary and ulnar lumbar region. 2 hours after the onset the
lymph nodes. There are multiple injecti- patient had hematuria. Plain radiograph
on marks on the elbow bend skin. The of the lumbar region shows no pathologi-
man denies taking drugs, the presence cal shadows. USI reveals pyelocaliectasis
of injection marks ascribes to influenza on the right, the left kidney is normal.
treatment. Blood count: RBCs- 3, 2·1012/l, What is the most likely diagnosis?
Hb- 100 g/l, WBCs- 3, 1 · 109 /l, moderate
lymphopenia. What study is required in A. Renal colic
the first place? B. Acute appendicitis
C. Bowel volvulus
A. ELISA for HIV D. Torsion of the right ovary cyst
B. Immunogram E. Right renal pelvis tumour
C. Sternal puncture
D. X-ray of lungs 20. Examination of a group of persons
E. Lymph node biopsy living on the same territory revealed
the following common symptoms: dark-
17. A patient got flame burns of both yellow pigmentation of the tooth enamel,
hands. On the dorsal and palmar surface diffuse osteoporosis of bone apparatus,
of hands the blisters are filled with serous ossification of ligaments and joints, functi-
fluid. The wrist joint region is hyperemic. onal disorders of the central nervous
The forearms were not injured. What is system. This condition may be caused
the provisional diagnosis? by the excessive concentration of the
following microelement in food or drinki-
ng water:
A. Fluorine
B. Copper
C. Nickel
D. Iodine
E. Cesium
Krok 2 Medicine 2013 4

21. A 50-year-old patient complains of A. Panzinorm forte


bursting pain in the left lower limb that B. Insulin
is getting worse on exertion, swelling in C. Gastrozepin
the region of shin and foot. Objectively: D. Contrycal
left shin and foot are doughy, skin of the E. No-spa
lower shin is indurated and has a bronze
tint, subcutaneous veins are dilated, there 25. A patient is on the sick leave for 4
is an ulcer with necrotic masses. What is months continuously from the date of
the most likely diagnosis? injury. The treatment is going to last for
1-2 months more. Who has the right to
A. Postthrombophlebitic syndrome extend the duration of medical certificate
B. Chronic arterial insufficiency for this patient?
C. Acute arterial thrombosis
D. Deep vein thrombosis of the lower A. Medical advisory commission after
limbs medico-social expert commission exami-
E. Gangrene of the lower extremity nation
B. Medical superintendent
22. A 12-year-old boy has a 6-year hi- C. Medical advisory commission after
story of insulin-dependent diabetes. The inpatient treatment
disease is labile. Since recently there D. District doctor by agreement with a
have been periodical rises in blood department chief
pressure. Microalbuminuria test gave E. Medico-social expert commission
positive results. The patient’s condition
corresponds with the following stage of 26. Full-term pregnancy. Body weight
diabetic nephropathy: of the pregnant woman is 62 kg. The
fetus has the longitudinal position, the
A. Stage III - early-stage nephropathy fetal head is pressed against the pelvic
B. Stage I - renal hypertrophy and inlet. Abdominal circumference is 100
hyperfunction cm. Fundal height is 35 cm. What is the
C. Stage II - histological changes in the approximate weight of the fetus?
kidneys
D. Stage IV - advanced clinical A. 3 kg 500 g
nephropathy B. 4 kg
E. V stage - chronic renal failure C. 2 kg 500 g
D. 3 kg
23. A 48-year-old patient was found to E. 4 kg 500 g
have diffuse enlargement of the thyroid
gland, exophthalmia, weight loss of 4 kg 27. A rural hospital serves 6200 people.
in 2 months, sweating. Objectively: HR- Preventive examinations were planned
105/min, AP- 140/70 mm Hg. Defecati- for 560 farm workers with different risk
on act is normal. What kind of therapy is factors. 400 workers underwent preventi-
recommended in this case? ve examination. 120 individuals were
found to have cardiovascular problems,
A. Mercazolil 90 of them were registered with a di-
B. Radioiodine spensary department for health care mai-
C. Propranolol ntenance. Which indicator should be used
D. Lugol’s solution to assess the organization of health care
E. Thyroxine maintenance at the hospital?
24. A 48-year-old male patient complains A. Share of workers who actually
of constant pain in the upper abdomen, underwent preventive examinations
mostly on the left, that is getting worse B. Share of newly diagnosed patients
after taking meals; diarrhea, weight loss. C. Share of persons subject to preventive
The patient is an alcohol abuser. 2 years examinations
ago he had acute pancreatitis. Blood D. Share of diseased persons
amylase is 4 g/h·l. Coprogram shows E. Incidence of cardiovascular diseases
steatorrhea, creatorrhea. Blood glucose
is 6,0 mmol/l. What treatment is indicated 28. A 20-year-old patient complai-
for this patient? ns of severe headache, double vision,
weakness, fever, irritability. Objectively:
body temperature is at the rate of 38, 1o C,
the patient is reluctant to contact, sensi-
tive to stimuli. There is ptosis of the
Krok 2 Medicine 2013 5

left eyelid, exotropia, anisocoria S>D, A. Posterior vertex presentation


pronounced meningeal syndrome. On B. Anterior vertex presentation
lumbar puncture the cerebrospinal fluid C. Presentation of the bregma
flowed out under a pressure of 300 mm D. Brow presentation
Hg, the fluid is clear, slightly opalescent. E. Face presentation
24 hours later there appeared the fibri-
nous film. Protein - 1,4 g/l, lymphocytes - 32. During medical monitoring of labor
600/3 per mm3 , sugar - 0,3 mmol/l. What and professional training of 6th grade
is the provisional diagnosis? schoolchildren the school doctor found
that lessons in carpentry workshop are
A. Tuberculous meningitis held on Friday as the fourth and fifth
B. Meningococcal meningitis lesson of a total of 90 minutes, the motor
C. Lymphocytic Armstrong’s meningitis density at the first lesson is 69%, and 65%
D. Syphilitic meningitis at the second. Does the lesson organizati-
E. Mumps meningitis on meet the hygienic requirements?

29. A 32-year-old patient has developed A. Meets the requirements


an acute condition after hypothermia: B. Does not meet the requirements in
temperature - 40oC, cough with 200 terms of training duration
ml of sputum per day. The sputum is C. Does not meet the requirements in
purulent, foul-smelling. To the right of terms of motor density at the 1st lesson
the lower lobe the mixed moist rales can D. Does not meet the requirements in
be auscultated. Blood tst results: WBCs - terms of motor density at the 2nd lesson
18, 0 · 109 /l, ESR - 45 mm/h. Radiographi- E. Does not meet the requirements as
cally: in the lower lobe of the right lung there are two consecutive lessons
there is a thick-walled cavity up to 6 cm
in diameter with a high horizontal level. 33. During coal extraction in a mine the
What is the most likely diagnosis? concentration of coal dust in the working
area is 450 mg/m3 (MPC is 10 mg/m3 ).
A. Lung abscess What occupational respiratory disease
B. Fibro-cavernous pulmonary may develop in miners?
tuberculosis
C. Lung cyst A. Anthracosis
D. Decomposing lung carcinoma B. Allergic nasopharyngitis
E. Infiltrative pulmonary tuberculosis C. Byssinosis
D. Siderosis
30. A family consists of 5 persons. E. Talcosis
The husband is a stope miner. His
spouse is a housewife. Their 20-year-old 34. A patient with suspected pheochromocytoma
daughter works as a kindergarten teacher. has normal blood pressure in the peri-
Their 18-year-old son is a student. The ods between the atacks and a tendency
grandmother is a pensioner, she has di- to tachycardia. Urine test revealed no
abetes. Which member of this family can pathology. It was decided to use a
be primarily classed among the group of provocative test with histamine. What
persons with a high risk of tuberculosis in medication should be prepared to provide
the planning of preventive examinations emergency care in case of a positive test
for tuberculosis? result?

A. Grandmother A. Phentolamine
B. Husband B. Pipolphen
C. Daughter C. Nifedipine
D. Son D. Mesatonum
E. Wife E. Prednisolone

31. A newborn’s head is of doli- 35. A 54-year-old patient complains of


chocephalic shape, that is front-to-back frequent painful urination, chills, fever
elongated. Examination of the occipital up to 38o C. Urine test results: protein -
region revealed a labour tumour located 0,33 g/L, WBCs - up to 50-60 in the field
in the middle between the prefontanel of vision, RBCs - 5-8 in the field of vi-
and posterior fontanel. Specify the type sion, gram-negative bacilli. Which of the
of fetal presentation: listed antibiotics should be preferred in
this case?
Krok 2 Medicine 2013 6

A. Ciprofloxacin A. Chronic type A gastritis


B. Oxacillin B. Chronic type B gastritis
C. Erythromycin C. Chronic type C gastritis
D. Tetracycline D. Menetrier disease
E. Tseporin E. Stomach cancer
36. The share of circulatory diseases was 40. A 45-year-old female patient complai-
15% in the total of registered diseases ns of frequent liquid stools with a lot of
among city population. What statistic indi- mucus, pus and blood; pain across the
cator is it? abdomen, loss of 7 kg within 6 months.
She has a 1-year history of non-specific
A. Extensive ulcerative colitis. What group of drugs
B. Intensive should be preferred for this patient?
C. Demonstrative
D. Ratio A. Corticosteroids
E. Average B. Antibacterial
C. Sulfonamides
37. During a regular medical examination D. Nitrofurans
at a metallurgical plant 20% of workers E. Polyenzymes
were found overweight (body weight was
5-14% higher than normal), and had early 41. A 24-year-old female teacher complai-
signs of obesity (grade I-II) with Quetelet ns of dizziness and heart pain irradiati-
index from 26 to 30. What products share ng to the left nipple. Pain is not associ-
must be reduced in the diet of this group ated with physical activity and cannot be
of people in the first place in order to relieved by nitroglycerin, it abates after
normalize their body weight? taking Valocordin and lasts an hour or
more. The patient has a nearly 2-year hi-
A. Bakery products story of this disease. Objectively: Ps- 76
B. Vegetables bpm. AP- 110/70 mm Hg. Heart borders
C. Fruit are normal, heart sounds are clear.
D. Milk and dairy products The ECG shows respiratory arrhythmia.
E. Meat and fish products Radiograph of the cervicothoracic spi-
ne shows no pathology. Lungs, abdomen
38. Several hours before, a 28-year- are unremarkable. What changes in blood
old patient suddenly developed acute formula can be expected?
headache and repeated vomiting, then
lost consciousness. Objectively: focal A. No changes
neurological symptoms were not found. B. Leukocytosis
Pronounced meningeal symptoms were C. Thrombocytopenia
revealed. AP - 120/80 mm Hg. Accordi- D. Leukemic hiatus
ng to clinical and liquorological findi- E. Increased ESR
ngs the patient was diagnosed with
subarachnoid haemorrhage. After admi- 42. A 60-year-old male patient, who works
nistration of dehydrants the patient’s as a construction worker, complains of
condition somewhat improved. What is pain in the right hip and knee joints,
the main component of further emergency that is getting worse on exertion. These
care? presentations have been observed for
the last 5 years. Objectively: the pati-
A. Coagulants ent is overnourished. Right knee joi-
B. Anticoagulants nt is moderately deformed. Examinati-
C. Antiaggregants on of other organs and systems revealed
D. Fibrinolytics no pathology. Blood tet results: WBCs -
E. Corticosteroids 8, 2 · 109 /l, ESR - 15 mm/h. Uric acid - 0,35
mmol/l. What is the most likely diagnosis?
39. Gastric juice analysis of a 42-year-
old male patient revealed absence of A. Deforming osteoarthritis
free hydrochloric acid at all stages. B. Reactive arthritis
Endoscopy revealed pallor, thinning of C. Gout
gastric mucosa, smoothed folds. Mi- D. Rheumatoid arthritis
croscopically the atrophy of glands with E. Reiter’s disease
intestinal metaplasia was found. What di-
sease is this situation typical for? 43. Chief physician of a polyclinic
encharged a district doctor with a task to
Krok 2 Medicine 2013 7

determine the pathological prevalence of appeared 5 days ago after a contact with
disease N in his district. What document a person having ARVI. Objectively: the
allows to estimate the disease prevalence child is in grave condition. Temperature
in the population of a medical district? of 38o C, cyanosis of nasolabial triangle is
present. RR- 54/min, nasal flaring while
A. Prophylactic examinations register breathing. There was percussion dullness
B. Statistic coupons (+) on the right below the scapula angle, and
C. Statistic coupons (-) tympanic sound over the rest of lungs.
D. Statistic coupons (+) and (-) Auscultation revealed bilateral fine moist
E. Vouchers for medical appointments rales predominating on the right. What is
the most likely diagnosis?
44. A 32-year-old female complains of di-
zziness, headache, palpitation, tremor. For A. Acute pneumonia
the last several months she has been under B. ARVI
outpatient observation for the increased C. Acute laryngotracheitis
arterial pressure. Since recently such D. Acute bronchitis
attacks have become more frequent and E. Acute bronchiolitis
severe. Objectively: skin is covered with
clammy sweat, tremor of the extremities 48. A 40-year-old patient complains of
is present. HR- 110/min, AP- 220/140 mm fever up to 39o C, cough with sputum and
Hg. Heart sounds are muffled. Blood test blood admixtures, dyspnea, weakness,
results: WBCs - 9, 8 · 109/l, ESR - 22 mm/h. herpetic rash on the lips. Objectively:
Blood glucose - 9,8 millimole/l. What di- respiration rate - 32/min. Under the
sease is the most likely cause of this crisis? shoulder blade on the right the increased
vocal fremitus and dullness of percussi-
A. Pheochromocytoma on sound were revealed. Auscultation
B. Essential hypertension revealed bronchial respiration. Blood
C. Preeclampsia count: WBCs - 14 · 109 /l, ESR - 35 mm/h.
D. Primary hyperaldosteronism What is the provisional diagnosis?
E. Diabetic glomerulosclerosis
A. Right-sided croupous pneumonia
45. A 55-year-old male has a 1,5-year B. Focal right-sided pneumonia
history of viral cirrhosis with symptoms C. Cavernous tuberculosis of the right lung
of portal hypertension. Over the last D. Lung cancer
month the weakness has progrssed, E. Exudative pleuritis
there appeared coffee ground vomit. Fi-
brogastroduodenoscopy revealed vari- 49. A patient underwent stomach resecti-
ceal esophageal haemorrhage. What drug on. During the operation, the left upper
should be used to reduce the pressure in limb of the patient was abducted and fi-
the portal vein? xed to the operating table for anesthetic
management. Postoperatively, the pati-
A. Vasopressin ent developed dysfunction of the upper
B. Reserpine extremity in form of "drop hand."This
C. Calcium gluconate symptom results from the damage of the
D. Dicynone following nerve:
E. Furosemide
A. Radial nerve
46. In an urban settlement situated on the B. Axillary nerve
riverbank an outbreak of hepatitis A was C. Ulnar nerve
registered. The disease might have water D. Median nerve
origin. This assumption can be confirmed E. Musculocutaneous nerve
by growth of the following indicators of
water quality: 50. In the morning a patient had nausea,
abdominal discomfort, single vomiti-
A. Number of coli-phages ng, dry mouth. In the evening, the
B. Escherichia coli index patient presented with the increasing
C. Oxidability general weakness, double vision, diffi-
D. Presence of benign leptospirosis cult swallowing of solid food. Objectively:
pathogen ptosis, mydriasis, anisocoria, absence of
E. Index of fecal coli-forms gag and pharyngeal reflex, dry mucous
membranes. The previous evening the
47. A 9-month-old child presents with patient had dinner with canned food
fever, cough, dyspnea. The symptoms
Krok 2 Medicine 2013 8

and alcohol. What is the presumptive di- tive airway pressure). Ro-gram of lungs
agnosis? shows reticular and nodular pattern, air
bronhogram. What is the most likely cause
A. Botulism of respiratory distress syndrome?
B. Food toxicoinfection
C. Intoxication with unknown poison A. Hyaline membrane disease
D. Acute ischemic stroke B. Segmental atelectasis
E. Poliomyelitis C. Bronchopulmonary dysplasia
D. Congenital pulmonary emphysema
51. An 8-year-old child complains of fever E. Edematous hemorrhagic syndrome
up to 38, 8o C, throat pain when swallowi-
ng, skin rash. Objectively: lacunar tonsi- 55. An 8-year-old child with a 3-year
llitis, circumscribed hyperaemia and history of diabetes was hospitalized in
enanthema of soft palate, pinoint-sized hyperglycemic coma. Specify the initial
skin rash, mostly in the folds and on dose of insulin to be administered:
the flexor surfaces of the extremities,
pale nasolabial triangle. Which antibiotic A. 0,1-0,2 U/kg of body weight per hour
should be administered in the first place? B. 0,05 U/kg of body weight per hour
C. 0,2-0,3 U/kg of body weight per hour
A. Penicillin D. 0,3-0,4U/kg of body weight per hour
B. Gentamicin E. 0,4-0,5 U/kg of body weight per hour
C. Ampicillin
D. Lincomycin 56. A 12-year-old girl undergoes
E. Tetracycline regular gastroenterological check-ups for
duodenal ulcer, biliary dyskinesia. What
52. A 28-year-old female patient with a is the recommended frequency of anti-
six-year history of Raynaud’s syndrome relapse treatment?
has recently developed pain in the small
joints of hands, difficult movement of food A. Twice a year
down the esophagus. What kind of disease B. Every 2 months
can you think of in this case? C. Every 3 months
D. Once a year
A. Systemic scleroderma E. Three times a year
B. Periarteritis nodosa
C. Rheumatoid arthritis 57. A 47-year-old patient came to see a
D. Systemic lupus erythematosus doctor on the 7th day of disease. The di-
E. Pseudotrichiniasis sease developed very fast: after the chill
body temperature rose up to 40o C and
53. A 16-year-old girl had had polyuria, lasted up to 7 hours, then it dropped
polydipsia for 2 months. She had lost abruptly, which caused profuse sweat.
8 kg with a good appetite. The patient There were three such attacks occuri-
was urgently hospitalized for abdominal ng once in two days. Two days ago the
pain and nausea. Examination revealed patient arrived from Africa. Objectively:
glycemia at the rate of 18 mmol/l, pale skin, subicteric sclera, significantly
glycosuria at the rate of 24 g/l. Insulin enlarged liver and spleen. What is the
and infusion of isotonic solutions of sodi- cause of fever attacks in this disease?
um chloride and glucose eliminated these
problems, including thirst. What is the A. Erythrocytic schizogony
most likely diagnosis? B. Tissue schizogony
C. Exotoxin of a causative agent
A. Diabetes mellitus type 1 D. Endotoxin of a causative agent
B. Diabetes mellitus type 2 E. Gametocytes
C. Renal glycosuria
D. Diabetes insipidus 58. On the 2nd day of illness a 27-
E. Secondary (symptomatic) diabetes year-old patient complains of unbearable
headache, repeated vomiting. Objecti-
54. A boy was born at 32 weeks gestati- vely: the patient is in a grave condition.
on. 2 hours after birth he developed He is conscious but adynamic. Lies in a
respiratory distress. The RD severity forced position with his head thrown back.
assessed by Silverman score was 5. The There is no skin rash. Nuchal muscles
respiratory disorders progressed, respi- are evidently rigid, there are Kernig’s and
ratory failure couldn’t be eliminated by Brudzinski’s signs. to - 39, 5oC, Ps- 120/min,
Martin-Bouyer CPAP (continuous posi- AP- 130/80 mm Hg. The leading syndrome
Krok 2 Medicine 2013 9

of this disease is caused by: Heart sounds are muffled, rhythmic.


ECG: Rv1+Sv5=15 mm. X-ray picture
A. Liquor hypertension shows the bulging of pulmonary artery
B. Liquor hypotension cone, right ventricle enlargement. What is
C. Affection of the cranial nerve nuclei the most likely cause of this condition?
D. Haemorrhages into the adrenal glands
E. Hyperthermy A. Pulmonary heart
B. Atherosclerotic cardiosclerosis
59. A 42-year-old female patient suffers C. Dilatation cardiomyopathy
from micronodular cryptogenic cirrhosis. D. Mitral stenosis
Over the last week her condition has E. Primary pulmonary hypertension
deteriorated: she developed convulsions,
mental confusion, progressing jaundice. 63. A 40-year-old woman with a history
What study may give reasons for such of combined mitral valve disease wi-
aggravation? th predominant stenosis complains of
dyspnea, asthma attacks at night, heart
A. Determination of serum ammonia problems. At present, she is unable to do
B. Determination of cholesterol ethers easy housework. What is the optimal tacti-
C. Determination of alpha-phetoprotein cs of the patient treatment?
D. Determination of ALAT and ASAT
E. Determination of alkaline phosphatase A. Mitral commissurotomy
B. Implantation of an artificial valve
60. A 60-year-old patient complai- C. Antiarrhythmia therapy
ns of recurrent pain in the proximal D. Treatment of heart failure
interphalangeal and wrist joints, their E. Antirheumatic therapy
periodic swelling and reddening that
have been observed for 4 years. X-ray 64. A 25-year-old patient complains of
picture represents changes in form of having dull heart pain for the last 10
osteoporosis, joint space narrowing and days, dyspnea on mild exertion, palpi-
single usuras. What is the most likely di- tations. The diasease developed 2 weeks
agnosis? ago after a respiratory infection. Objecti-
vely: acrocyanosis, AP- 90/75 mm Hg, Ps-
A. Rheumatoid arthritis 96/min. Cardiac borders appear to be shi-
B. Osteoarthritis fted to the left and right. Heart sounds
C. Gout are weak and have triple rhythm, there is
D. Pseudogout systolic murmur at the apex. ECG showed
E. Multiple myeloma sinus rhythm, complete left bundle branch
block. What is the most likely diagnosis?
61. A 28-year-old patient has been hospi-
talized for the pain in the epigastric regi- A. Infectious-allergic myocarditis
on. He has a 10-year history of duodenal B. Exudative pericarditis
ulcer (DU). Recently, the pain character C. Infective endocarditis
has changed: it became permanent, persi- D. Myocarditic cardiosclerosis
stent, irradiating to the back. There are E. Vegetative-vascular dystonia
general weakness, dizziness, fatigue. The
patient has put off weight. Objectively: 65. A child undergoes in-patient
HR- 68/min, AP- 120/80 mm Hg. What is treatment for acute staphylococcal
most likely cause of deterioration? destruction of the right lung. Unexpectedly
he develped acute chest pain on the ri-
A. Penetration ght, dyspnea, cyanosis. The right side of
B. Haemorrhage chest lags behind in the respiratory act.
C. Perforation of duodenal wall Percussion reveals dullness in the lower
D. Exacerbation of duodenal ulcer parts on the right, bandbox resonance in
E. Stenosis development the upper parts. Borders of the relati-
ve cardiac dullness are shifted to the
62. A 57-year-old male patient complai- left. What complication has most likely
ns of dyspnea on exertion, heaviness developed?
in the right hypochondrium and shin
edemata towards evening. Objectively:
temperature - 38, 1oC, HR- 20/min,
HR=Ps=92/min, AP- 140/90 mm Hg.
There is apparent kyphoscoliosis. In the
lungs single dry rales can be auscultated.
Krok 2 Medicine 2013 10

A. Right-sided pyopneumothorax ameter. Excretory urograms show normal


B. Pleural empyema condition on the left, and the deformati-
C. Spontaneous pneumothorax on of the superior renal calyces with sati-
D. Exudative pleuritis sfactory function on the right. What kind
E. Right lung abscess of disease can you think of?
66. Examination of a 13-year-old gi- A. Simple cyst of the right kidney
rl revealed acute glomerulonephritis, B. Multicystic kidney disease
nephritic syndrome at the initial stage wi- C. Multiple cysts of the right kidney
thout renal dysfunction. What is the main D. Tumour of the right kidney
drug of choice for the basic therapy of this E. Right hydronephrosis
patient?
70. While staying in a stuffy room a 19-
A. Antibiotic year-old emotionally labile girl developed
B. Prednisolone severe weakness, dizziness, blackout,
C. Curantyl nausea and loss of consciousness without
D. Heparin convulsions. Objectively: the patient is
E. Saluretic unconscious, the skin is pale, extremiti-
es are cold. AP- 90/60 mm Hg, Ps- 96/min,
67. A 28-year-old patient consulted deficient, breathing is shallow. Pupillary
a surgeon about pain, edema and and tendon reflexes are present. There are
hyperemia of the left side of his face, no pathological signs. What is the most li-
weakness, fever up to 39o C. These mani- kely diagnosis?
festations has been present for three days.
Objectively: there is an inflammatory infi- A. Syncope
ltrate of the left nasolabial fold 4x4 cm B. Vegetovascular paroxysm
large with a necrotic core in the center, the C. Epileptic attack
pronounced edema of the left side of face, D. Hysterical neurosis
moderate nuchal rigidity. What treatment E. Transient ischemic attack
is needed?
71. A 13-year-old boy complains of pain
A. Hospitalization, antibiotics, anti- in the upper third of his left thigh, body
coagulants, rest temperature rise up to 39oC. There is a
B. Antibiotics, surgery under general swelling in the upper third of thigh and
anesthesia inguinal fold smoothness. The extremity is
C. Wet-to-dry dressings with antiseptics, in a half-bent position. Active and passi-
aspirin ve movements are not possible because of
D. Physiotherapy, immunomodulators the sharp pain. What is the most likely di-
E. Antibiotics, angioprotectors, daily agnosis?
dressings
A. Acute hematogenous osteomyelitis
68. A 6-year-old child complains of B. Acute coxitis
frequent liquid stool and vomiting. On the C. Intermuscular phlegmon
2nd day of desease the child presented D. Osteosarcoma
with inertness, temperature rise up to E. Brodie’s disease
38, 2oC, Ps- 150 bpm, scaphoid abdomen,
palpatory painful sigmoid colon, defecati- 72. A 12-year-old cild has been hit on
on 10 times a day with liquid, scarce stool the stomach. The patient is in moderately
with mucus and streaks of green. What is grave condition, has a forced position in
a provisional diagnosis? bed. The skin is pale, Ps- 122/min. The
stress on the left costal arch causes a
A. Shigellosis slight pain. There are positive Weinert,
B. Salmonellosis Kulenkampff symptoms. Macroscopically
C. Escherichiosis the urine is unchanged. What is the most
D. Intestinal amebiasis likely diagnosis?
E. Yersiniosis
A. Spleen rupture, abdominal bleeding
69. A 48-year-old patient complains of B. Left kidney rupture, retroperitoneal
having dull pain in the right lumbar regi- hematoma
on for over three years. USI shows that C. Rupture of the pancreas
kidneys are of normal size, at the upper D. Liver rupture, abdominal bleeding
pole of the right kidney there is a fluid- E. Rupture of a hollow organ, peritonitis
containing formation up to 12 cm in di-
Krok 2 Medicine 2013 11

73. A 30-year-old multigravida has been vacuoles. Make a diagnosis:


in labour for 18 hours. 2 hours ago the
pushing stage began. Fetal heart rate is A. Complicated cataract of the right eye
clear, rhythmic, 136/min. Vaginal exami- B. Senile cataract of the right eye
nation reveals the completecervical di- C. Diabetic cataract of the right eye
latation, the fetal head in the pelvic outlet D. Tetanic cataract of the right eye
plane. Sagittal suture in line with obstetric E. Radiation cataract of the right eye
conjugate, the occipital fontanel is near
the pubis. The patient has been diagnosed 77. During her first visit to the prenatal
with primary uterine inertia. What is the clinic a pregnant woman was referred to
further tactics of labour management? other doctors for mandatory consultation.
The patient was refered to:
A. Outlet forceps
B. Labour stimulation A. Internist, dentist
C. Cesarean section B. Internist, surgeon
D. Skin-head Ivanov’s forceps C. Dentist, surgeon
E. Vacuum extraction of the fetus D. Surgeon, oculist
E. Therapist, oculist
74. A patient complains of frequent, bulky,
frothy stools with greenish mucus, crampi- 78. A 59-year-old male complains of heart
ng pain in the umbilical region, abdomi- pain, cough, fever up to 38o C. Three
nal murmur, body temperature at the weeks ago he suffered a heart attack.
rate of 39o C. The patient associates the Objectively: Ps- 86/min, rhythmic, blood
disease with consumption of soft-boiled pressure - 110/70 mm Hg. Auscultation
eggs. What is the most likely pathogen? reveals pericardial rub, rales beneath the
shoulder blade. Radiography reveals no
A. Salmonella pathology. Blood count: WBCs - 10 · 109 /l,
B. Yersinia ESR - 35 mm/h. ECG shows no dynamics.
C. Shigella It would be most reasonable to administer
D. Enteropathogenic E.Coli the drugs of the following pharmaceutical
E. Vibrio cholerae El Tor group:
75. X-ray picture of chest shows a density A. Glucocorticoids
and an abrupt decrease in the upper lobe B. Antibiotics
of the right lung. The middle and lower C. Direct anticoagulants
lobe of the right lung exhibit significant D. Nitrates and nitrites
pneumatization. The right pulmonary E. Fibrinolytics
hilum comes up to the dense lobe. In
the upper and middle parts of the left 79. A 40-year-old female patient complai-
pulmonary field there are multiple focal ns of headache, dizziness, muscle
shadows. In the basal region of the left weakness, occasional cramps in the
pulmonary field there are clear outlines of extremities. She has been taking anti-
two annular shadows with quite thick and hypertensive medications for 10 years.
irregular walls. What disease is this X-ray AP- 180/100 mm Hg. Blood potassium -
pattern typical for? 1,8 millimole/l, sodium - 4,8 millimole/l. In
urine: alkaline reaction, the relative densi-
A. Fibro-cavernous pulmonary ty - 1012, protein and sugar are not found,
tuberculosis WBCs - 3-4 in the field of vision, RBCs - 1-
B. Atelectasis of the right upper lobe 2 in the field of vision. Conn’s syndrome is
C. Abscessing pneumonia suspected. Which drug should be chosen
D. Peripheral cancer for the treatment of arterial hypertensi-
E. Pancoast tumour on?
76. A patient complains of impaired far vi- A. Spironolactone
sion. Previously his eyes often turned red B. Propanolol
and hurt. Objectively: the eyes are not C. Enalapril
irritated, the cornea is transparent, anteri- D. Hydrochlorothiazide
or chambers are median deep, their liquid E. Clonidine
is transparent. The iris of the right eye
has not changed in colour, its pattern is 80. A 27-year-old patient complains of
unchanged. The pupil is of irregular shape, nasal haemorrhages, multiple bruises on
scalloped. Biomicroscopy of the crystalli- the anterior surface of the trunk and
ne lens reveals the areas of opacity and extremities, sudden weakness. In blood:
Krok 2 Medicine 2013 12

Hb- 74 g/l, reticulocytes - 16%, RBCs - the patient is pale, mucous membranes
2, 5 · 1012 /l, platelets - 30 · 109 /l, ESR- 25 are cyanotic, breathing rate is 24/min,
mm/h. What is the most effective measure tachycardia is present. In lungs: dimini-
for the treatment of thrombocytopenia? shed breath sounds, moist rales over both
lungs, crepitation. What is the most likely
A. Splenectomy diagnosis?
B. Iron preparations
C. Hemotransfusion A. Pneumonic plaque
D. Cytostatics B. Miliary tuberculosis
E. Vitamin B12 C. Influenza
D. Ornithosis
81. After lifting a load a patient felt E. Sepsis
undurable pain in the loin. He was di-
agnosed with acute lumbosacral radiculi- 85. An 8-month-old baby has decreased
tis. Which of the following is contraindi- appetite, pale skin, enlarged right side
cated for this patient? of abdomen. Palpation the right side of
abdomen reveals a dense elastic tumour-
A. Warming procedures like formation 10x7 cm large. There is
B. Dehydrating drugs a positive ballotement sign. What is the
C. Analgetics most likely diagnosis?
D. Vitamins of B group
E. Intravenous injection of aminophylline A. Nephroblastoma (Wilms’ tumour)
B. Liver tumour
82. The institutions which take part in C. Congenital hydronephrosis
medical examinations can be prevention D. Colon pathology
and treatment facilities, medical board E. Intestinal tumour
of Ministry of Defense, medical board of
Ministry of Home Affairs, medico-social 86. An infant is 3 weeks old. Since bi-
expert commissions, forensic medical rth there has been observed periodical
boards etc. What institutions are responsi- vomiting within a few minutes after feedi-
ble for temporary disability examination? ng. The amount of vomitive masses does
not exceed that of previous feeding. The
A. Prevention and treatment facilities infant has age-appropriate body weight.
B. Sanitary-and-prophylactic institutions What is the most likely cause of this
C. Medico-social expert commissions symptom?
D. Medical boards of Ministry of Defense
E. Medical boards of Ministry of Home A. Pylorospasm
Affairs B. Esophageal chalasia
C. Adrenogenital syndrome
83. While lifting a heavy load a 39-year-old D. Pyloristenosis
patient suddenly felt a severe headache, E. Esophageal achalasia
pain in the interscapular region, and
started vomiting. Objectively: the pulse 87. A city somatic hospital with 300
is rhythmic, 60/min, AP- 180/100 mm Hg. beds consists of the main building whi-
The patient is agitated. He presents wi- ch houses the therapeutic and surgical
th photophobia, hyperacusis. There are departments. Several separate buildings
positive Kernig’s and Brudzinski’s signs house the maternity, pediatric and radi-
on both sides. In blood: WBCs - 10 · 109 /l. ologic departments that are connected
CSF is bloody, cytosis is 240/3. What is the to the main building by underground
most likely diagnosis? walkways and above-ground covered
skybridges. Specify the building system
A. Subarachnoid haemorrhage of the hospital:
B. Sympathoadrenal crisis
C. Acute hypertonic encephalopathy A. Central-unit
D. Meningococcal meningitis B. Centralized
E. Ischemic stroke C. Decentralized
D. Free
84. A 45-year-old patient, a sailor, was E. Combined
hospitalized on the 2nd day of the di-
sease. A week ago he returned from 88. A 13-year-old girl complains of fever
India. Complains of body temperature of up to 37, 4o C during the last 2 months
41o C, severe headache, dyspnea, cough after recovering from ARVI. Objectively:
with frothy rusty sputum. Objectively: malnutrition, diffuse grade II enlargement
Krok 2 Medicine 2013 13

of the thyroid gland feeling dense on provisional diagnosis?


palpation, exophthalmos, tachycardia.
What kind of pathological syndrome is A. Status asthmaticus
it? B. Chronic obstructive bronchitis
C. Bronchiale asthma, moderate gravity
A. Thyrotoxicosis D. Foreign object aspiration
B. Hypothyroidism E. Cardiac asthma
C. Hypoparathyroidism
D. Hyperparathyroidism 92. Blood typing resulted in positi-
E. Thymomegaly ve isohemagglutination reaction with
standard sera of А(II) and В(III) groups
89. A 53-year-old female patient complai- and negative reaction with sera of 0(I)
ns of cardiac pain and rhythm intermissi- and АВ(IV) groups. What is this result
ons. She has experienced these presentati- indicative of?
ons since childhood. The patient’s father
had a history of cardiac arrhythmias. A. Faulty standard sera
Objectively: the patient is in grave condi- B. The first blood group
tion, Ps- 220 bpm, AP- 80/60 mm Hg. C. The second blood group
ECG results: heart rate - 215/min, extensi- D. The third blood group
on and deformation of QRS complex E. The fourth blood group
accompanied by atrioventricular dissoci-
ation; positive P wave. Some time later 93. A patient complains of jerking,
heart rate reduced down to 45/min, there throbbing pain in the III finger on the
was a complete dissociation of P wave and right hand. The patient associates these
QRST complex. Which of the following pain onsets with an injury by a nail. The
will be the most effective treatment? finger skin is hyperemic and tense, palpati-
on with a bulbous-end probe reveals the
A. Implantation of the artificial pacemaker most painful area. What is the provisional
B. β-adrenoreceptor blocking agents diagnosis?
C. Cholinolytics
D. Calcium antagonists A. Subcutaneous felon
E. Cardiac glycosides B. Pandactylitis
C. Bone felon
90. On the 2nd day of life a full-term D. Paronychia
boy developed mild jaundice of skin and E. Articular felon
mucous membranes, the general condition
of the child is normal. Blood test results: 94. Sanitary examination of the burns unit
indirect hyperbilirubinemia - 120 mmol/l. for adults revealed that 4-bed wards had
The child’s blood group is A(II) Rh(+), an area of 28 m2 . What is the minimally
his mother’s blood group - B(III) Rh(+). required ward area for this department?
What is the doctor’s tactics of choice?
A. 40 m2
A. No drug therapy B. 24 m2
B. Treatment with enterosorbents C. 28 m2
C. Treatment with prednisolone D. 30 m2
D. Treatment with cholekinetics E. 52 m2
E. Blood transfusion
95. A patient presented to a hospital
91. A 49-year-old patient complains of with a carbuncle of the upper lip. The
dyspnea, cough. There are no sputum body temperature is 39o C. There is a
discharges. He has repeatedly used pronounced edema of the upper lip and
salbutamol and intal but with no effect. eyelids. What is the surgeon’s tactics of
Objectively: he is only able to sit whi- choice?
le leaning on the table. Cyanosis of
face, acrocyanosis are present. Breathi- A. Hospitalize in the surgical unit
ng is shallow, laboured, in some parts it B. Administer out-patient course of antibi-
cannot be auscultated; there are diffuse otics
rales, expiration is significantly prolonged. C. Disclose the carbuncle and administer
Heart sounds are muffled, tachycardia is out-patient treatment
present. Ps - 112/min, AP- 110/70 mm Hg. D. Administer physiotherapy
Liver is located near the costal arch. There E. Disclose the carbuncle and administer
are no peripheral edemata. What is your antibiotics
Krok 2 Medicine 2013 14

96. From urine of a 14-year-old boy with test results: ESR- 47 mm/h, total bili-
the exacerbation of secondary obstructive rubin - 86,1 mmol/l, direct bilirubin - 42,3
pyelonephritis Pseudomonas aeruginosa mmol/l. Total protein - 62 g/l, albumins
was isolated with a titer of 1000000 mi- - 40%, globulins - 60%, gamma globuli-
crobes per 1 ml. Which antibiotic is most ns - 38%. Viral hepatitis markers were
advisable to be administered in this case? not detected. The antibodies to smooth
muscle cells are present. On ultrasound
A. Ciprofloxacin the portal vein diameter was of 1 cm.
B. Ampicillin What is the most likely diagnosis?
C. Cefazolinum
D. Azithromycin A. Autoimmune hepatitis
E. Chloramphenicol B. Primary biliary cirrhosis
C. Gilbert’s syndrome
97. A 62-year-old male patient complai- D. Cholangiogenic hepatitis
ns of intense pain in the left leg that E. Hemachromatosis
suddenly arose three hours before, leg
numbness and coldness. During the year 100. A patient is 60 years old, retired,
there has been pain in the leg while walki- worked as deputy director of a research
ng, hypersensitivity to cooling. Objecti- institute. Behavioural changes appeared 2
vely: the left foot and shin have marbled years ago after the death of her husband:
skin, subcutaneous veins are collapsed. she stopped looking after herself and
The foot is cold, active movements of leaving the house; then she refused to
the foot and toes are preserved. Pulse is clean the apartment and cook. Mental
present only on the femoral artery. There status: temporal disorientation. The pati-
is rough systolic murmur above the artery. ent does not understand many of the
Make a provisional diagnosis: questions, is confused; does not know
how to cook soup or fasten a button.
A. Acute occlusion of the left femoral Her speech is characterized by stumbli-
artery ng and logoclonia. She does not recognize
B. Occlusive disease doctors, fellow patients. She cries a lot but
C. Stenosis of the left popliteal artery can not explain the reason for tears. What
D. Acute thrombophlebitis is the mechanism of this pathology?
E. Acute arterial thrombosis ileofemoralny
A. Atrophy of the cerebral cortex
98. A 28-year-old patient complai- B. Atherosclerotic changes in cerebral
ns of profuse, painful and prolonged vessels
menstruation. Before and after the C. Serotonin deficiency
menstrual period there is spotting lasting D. Impaired conversion of dopamine to
for 4-6 days. Vaginal examination reveals noradrenaline
that the uterus is enlarged correspondi- E. Disorder of melatonin metabolism
ng to 5-6 weeks of pregnancy, has limited
mobility, is painful. Appendages are not 101. A 70-year-old patient with a
palpable. On the 15th day of the menstrual strangulated inguinal hernia called a
cycle, the uterus was of normal size, pai- therapist in. The strangulation took place
nless. On account of stated problems and 10 hours ago. There are signs of intestinal
objective examination the patient has obstruction. The skin over the herniation
been diagnosed with internal endometri- is hyperemic. What is the tactics of choi-
osis. Which drug should be used for the ce?
efective treatment of this patient?
A. Emergency hospitalization to a surgical
A. Duphaston hospital
B. Synoestrolum B. Referral to a surgeon
C. Parlodel C. Cold to the hernia, analgesics, antibioti-
D. Ovidon cs
E. - D. Reduction of hernia after a narcotic
injection
99. A 24-year-old female patient complai- E. Reduction of hernia
ns of pain in the right hypochondrium
that is getting worse after taking meals; 102. An employee was on a business trip
nausea, fever up to 37, 7oC, icteric skin, to another city, where he fell ill and was
pain in the large joints. These presentati- hospitalized. The sick leave certificate can
ons have been observed for 8 months. be issued:
Objectively: hepatosplenomegaly. Blood
Krok 2 Medicine 2013 15

A. With the permission of the head doctor 106. Examination of a 38-year-old pati-
of the city hospital ent who had been hit with a blunt object
B. With the permission of the deputy head on the left side of chest revealed a
doctor after the disability examination fracture of the X rib with fragments di-
C. With the permission of the deputy chief splacement, parietal pneumothorax. The
doctor in charge of medical work patient complains of pain in the left
D. By doctor in charge and chief of subcostal area. Objectively: the patient
department is pale, AP- 80/40 mm Hg, Ps- 138/min,
E. By doctor in charge of poor volume. USI reveals fluid in the
left abdomen. Splenic rupture is present.
103. A 14-year-old boy with a history What treatment tactics should be chosen?
of chronic tonsillitis and sinusitis has
developed a feeling of heart irregulari- A. Drainage of the left pleural cavity
ties and additional pulse. HR- 83/min. followed by laparotomy
ECG results: regular impulses with no vi- B. Immediate upper median laparotomy
sible P wave that occur every two sinus followed by drainage of the left pleural
contractions, QRS complex is dramati- cavity
cally deformed and prolonged to over C. Immediate laparotomy and alcohol-
0,11 s, T wave is discordant followed by novocaine block of the X rib
a complete compensatory pause. Specify D. Anti-schock actions followed by
the arrhythmia type: laparotomy after the arterial pressure rise
E. Left-sided thoracotomy immediately
A. Trigeminal extrasystole followed by laparotomy
B. Bigeminal extrasystole
C. Partial AV-blockade 107. A baby is 3 months old. The mother
D. Complete AV-block consulted a pediatrician about lack of
E. Left bundle branch block breast milk. After several test weighings
it was found that the child had to receive
104. An 8-year-old girl periodically has supplementary feeding. What is the opti-
sudden short-term heart pain, sensation mal milk formula for this child?
of chest compression, epigastric pain, di-
zziness, vomiting. Objectively: the patient A. Malysh
is pale, respiratory rate - 40/min, jugular B. Milk formula № 2
pulse is present. Ps- 185 bpm, of poor C. Milk formula № 3
volume. AP- 75/40 mm Hg. ECG taken D. Whole cow’s milk
during an attack shows ectopic P waves, E. Malutka
QRS wave is not deformed. At the end
of an attack a compensatory pause is 108. Examination of a newborn revealed
observed. The most likely cause of the skin redness that appeared immediately
attack is: after birth and reached the maximum
intensity on the second day of life. What
A. Paroxysmal atrial tachycardia is your provisional diagnosis?
B. Sinus tachycardia
C. Paroxysmal ventricular tachycardia A. Simple erythema
D. Complete AV-block B. Toxic erythema
E. Atrial fibrillation C. Transient erythema
D. Erythema nodosum
105. A 45-year-old patient with acute E. Annular erythema
abscess of the left lung has suddenly
developed acute chest pain and dyspnea 109. A 27-year-old sexually active female
while coughing, tachycardia has increased. complains of numerous vesicles on the ri-
The control Ro-gram shows collapsed left ght sex lip, itch and burning. Eruptions
lung, the air in the left pleural cavity regularly turn up before menstruation and
and a horizontal fluid level. What is the disappear 8-10 days later. What is the most
mechanism of this complication? likely diagnosis?
A. Abscess burst into the pleural cavity A. Herpes simplex virus
B. Bullae rupture of the left lung B. Bartholinitis
C. Inflammation spread to the visceral C. Primary syphilis
pleura D. Cytomegalovirus infection
D. Atelectasis of the left lung E. Genital condylomata
E. Acute cardiovascular insufficiency
110. A 38-year-old patient has suddenly
Krok 2 Medicine 2013 16

developed pain in the left side of his chest, score was 4. Which of the following is a
suffocation. Objectively: moderately risk factor for a spinal cord injury?
grave condition, Ps- 100/min, AP- 90/60
mm Hg, breath sounds on the left cannot A. Difficult delivery of the head and
be auscultated. Chest radiography shows shoulders
the collapse of the left lung up to 1/2. B. Young age of the mother
What kind of treatment should be admi- C. Pernicious habits
nistered? D. Uterine inertia
E. Chronic hypoxia
A. Passive thoracostomy
B. Rest, resolution therapy 115. A 10-year-old child has been admi-
C. Pleural puncture tted to a hospital with a closed crani-
D. Operative therapy ocerebral injury with suspected cerebral
E. Active thoracostomy edema. The patient is in grave condition,
unconscious. The dyspnea, tachycardia,
111. A 40 week pregnant secundipara is hypertension are present. Muscle tone is
28 years old. Contractions are very active. increased, there is nystagmus, pupillary
Retraction ring is at the level of navel, the and oculomotor reactions are disturbed.
uterus is hypertonic, in form of hourglass. The mandatory component of intensi-
On auscultation the fetal heart sounds ve care is dehydration. What diuretic is
are dull, heart rate is 100/min. AP of the adequate in this case?
parturient woman is 130/80 mm Hg. What
is the most likely diagnosis? A. Furosemide
B. Mannitol
A. RIisk of hysterorrhexis C. Hydrochlorthiazide
B. Mazolysis D. Spironolactone
C. Disturbed labour E. Moduretic
D. Complete hysterorrhexis
E. Attack of eclampsia 116. A 57-year-old patient taken to the
surgical department by ambulance has
112. A 10-year-old patient has a hi- been provisionally diagnosed with acute
story of mild bronchial asthma. During intestinal obstruction. Acute pancreatitis
a regular check-up the patient should be is suspected. What is the most informative
recommended: method of study to verify the diagnosis?
A. To avoid allergenic food A. Plan radiography of stomach
B. To avoid body tempering procedures B. Biochemical blood analysis
C. Not to go in for sports C. Complete blood count and clinical
D. To avoid spa treatment urinalysis
E. Not to go to the seaside D. Fibrogastroduodenoscopy
E. Ultrasound
113. A 26-year-old patient with left lower
lobe pneumonia experiences an acute 117. After myocardial infarction, a 50-
chest pain on the left during coughing. year-old patient had an attack of asthma.
Objectively: diffuse cyanosis, extension of Objectively: bubbling breathing with
the left side of chest. Percussion reveals frequency of 32/min, cough with a lot
high tympanitis. Auscultation reveals no of pink frothy sputum, acrocyanosis,
respiratory murmurs above the left side swelling of the neck veins. Ps- 108/min,
of chest. There is a deviation of the right AP- 150/100 mm Hg. Heart sounds
cardiac border towards the midclavicular are muffled. Mixed moist rales can be
line. What examination will be the most auscultated above the entire lung surface.
informative? What drug would be most effective in this
situation?
A. X-Ray
B. Bronchoscopy A. Nitroglycerin intravenously
C. Bronchography B. Pentamin intravenously
D. Pneumotachometry C. Strophanthin intravenously
E. Spirography D. Dopamine intravenously
E. Aminophylline intravenously
114. A baby was born by a young smoker.
The labour was complicated by uterine 118. A baby born after fast labour has
inertia, difficult delivery of the baby’s palsy of hand muscles. Grasp reflex is
head and shoulders. The baby’s Apgar absent, as well as hand-to-mouth reflex.
Krok 2 Medicine 2013 17

Hand sensitivity is absent. What is the A. Emergency notification on infectious


most likely diagnosis? disease (form № 058/o)
B. Sick child care certificate
A. Dejerine-Klumpke palsy C. Infant’s record (report form № 112/o)
B. Duchenne-Erb’s palsy D. House call record (form № 031/o)
C. Total lesion of the brachial plexus E. Child care sick leave
D. Muscle paresis
E. Bernard-Horner syndrome 123. Hygienic expertise of a sample taken
from the batch of grain revealed that
119. Against the background of angina 2% of grains were infected with mi-
a patient has developed pain in tubular croscopic Fusarium fungi. On the ground
bones. Examination revealed generalized of laboratory analyses this batch of grain
enlargement of lymph nodes, hepatoli- should be:
enal syndrome, sternalgia. In blood: RBCs
- 3, 6 · 1012 /l, Hb- 87 g/l, thrombocytes A. Sold without restrictions
- 45 · 109 /l, WBCs - 13 · 109 /l, blasts - B. Tested for toxicity
87%, stab neutrophils - 1%, segmented C. Used for forage production
neutrophils - 7%, lymphocytes - 5%, ESR D. Used for ethanol production
- 55 mm/h. What is the most likely di- E. Destroyed
agnosis?
124. Examination of a 43-year-old man
A. Acute leukemia objectively revealed pallor of skin and
B. Erythremia mucous membranes, loss of tongue papi-
C. Chronic lymphocytic leukemia llae, transverse striation of fingernails,
D. Chronic myeloid leukemia cracks in the mouth corners, tachycardia.
E. Multiple myeloma Blood test results: Hb- 90 g/l, anisocytosis,
poikilocytosis. The most likely causative
120. In order to reduce weed growth on agent of this state is inadequate intake of:
agricultural land, some herbicides have
been used for a long time. In terms of A. Iron
environmental stability these herbicides B. Copper
are rated as stable. Specify the most likely C. Zinc
route of their entry into the human body: D. Magnesium
E. Selene
A. Soil-plants-humans
B. Soil-microorganisms-humans 125. Factory’s sectorial doctor chooses a
C. Soil-animals-humans group of chronically ill people. He takes
D. Soil-protozoa-humans into account the duration of etiologically
E. Soil-insects-humans related cases with a temporary disability
over the last year in each of the workers.
121. A selective population research study The employees will fall into this group if
was aimed at exploring the effect of air the duration is:
emissions from a metallurgical plant on
the obstructive bronchitis morbidity in a A. 40 days or more
city. The calculated correlation coefficient B. 60 days or more
was +0,79. Evaluate the strength and di- C. 20 days or more
rection of the relationship: D. 30 days or more
E. 10 days or more
A. Direct, strong
B. Inverse, strong 126. 6 people live in a modern flat wi-
C. Direct, average th the total area of 60 m2 . There are
D. Inverse, average TV-video equipment, radios, microwave
E. - ovens, computer. The residents of the
flat complain of bad health, occasional
122. A general practitioner visited a 2- headaches, arrhythmia, conjunctivitis.
year-old child and diagnosed him with What is the most likely cause of this condi-
measles. The child attends a nursery, has tion?
a 5-year-old sister. What document must
be filled in for the effective antiepidemic
measures in the given health locality?
Krok 2 Medicine 2013 18

A. Electromagnetic fields
B. Anthropotoxins A. Status epilepticus
C. Carbon dioxide B. Frequent generalized seizures
D. Tetrachlor plumbum C. Frequent jacksonian seizures
E. Formaldehyde D. Hysterical attacks
E. Frequent complex partial seizures
127. A man abused alcohol, drank away
the property and wages thus getting hi- 131. A 45-year-old patient complains
mself, his wife and two underage children of fever up to 40o C, general weakness,
into deep financial problems. He was regi- headache and spasmodic contraction of
stered in a local drug abuse clinic. His wife muscles in the region of a shinwound. The
asked a family doctor, what kind of petiti- patient got injured five days ago when ti-
on she could file in court: lling soil and didn’t seek medical attenti-
on. What kind of wound infection can be
A. On restriction of her husband’s civil suspected?
capacity
B. On recognition of her husband’s A. Tetanus
incapacity B. Anthrax
C. On recognition of her husband’s partial C. Erysipelas
incapacity D. Gram-positive
D. On recognition of her husband’s E. Gram-negative
incompetence
E. On recognition of her husband’s disabi- 132. A 60 year-old female has been sufferi-
lity ng weakness, dizziness, fatigue over the
last year. Recently she has also developed
128. A child is 12 years old. He complai- dyspnea, paresthesia. Objectively: skin
ns of a dull aching pain in the epigastri- and mucous membranes are pale and sli-
um and right hypochondrium, that is ghtly icteric. The tongue is smooth due
getting worse after taking fatty or fried to the loss of lingual papillae. Liver and
food, headache, weakness, nausea, low- spleen are located at the costal margin.
grade fever. Abdominal palpation reveals Blood count: Hb- 70 g/l, RBCs - 1, 7·1012/l,
a marked resistance of muscles in the right colour index - 1,2, macrocytes. Admini-
hypochondrium, positive Kerr’s, Ortner’s, ster the patient a pathogenetically justifi-
Murphy’s symptoms. What is the most li- ed drug:
kely diagnosis?
A. Vitamin B12
A. Chronic cholecystitis B. Vitamin B6
B. Acute appendicitis C. Ascorbic acid
C. Viral hepatitis D. Iron preparations
D. Acute gastritis E. Vitamin B1
E. Acute pancreatitis
133. Analysis of organization of medi-
129. A 3-month-old girl presents wi- cal care in a regional centre has shown
th rhinitis, dyspnea, dry cough. These that every year about 12% of patients
manifestations has been observed for receive inpatient care for diseases that
two days. Objectively: the child has don’t require round-the-clock monitori-
pale skin, acrocyanosis, shallow respi- ng and intensive care. What are the most
ration at the rate of 80/min. Percussi- appropriate organizational changes requi-
on reveals handbox resonance over the red to address this problem?
whole surface of lungs, massive fine rales.
What is the most likely diagnosis? A. Development of medical care forms
replacing the in-patient care
A. Acute bronchiolitis B. Changes to the statute of outpatient
B. Pneumonia clinics
C. Mucoviscidosis C. Development of primary care
D. Foreign body of the airway D. Restructuring of specialized care
E. Acute bronchitis E. Upgrading of hospital facilities
130. A patient had 4 generalized convulsi- 134. A newborn (mother’s I pregnancy)
ve seizures within a day. Between the sei- weighing 3500 g presents with jaundice,
zures the patient did not maintain clear lethargy, reduced reflexes. Objectively:
consciousness (was in a coma or stupor). second grade jaundice of skin with saffron
Specify his state: tint, liver - +2cm, spleen - +1 cm. Urine
Krok 2 Medicine 2013 19

and feces are yellow. Blood count: Hb- 100 kely diagnosis?
g/l, RBCs - 3, 2 · 1012 /l, WBCs - 18, 7 · 109 /l,
mother’s blood type - 0(I) Rh(+), baby’s A. Acute bartholinitis
blood type - А(II) Rh(-), bilirubin - 170 B. Labial furuncle
C. Acute vulvovaginitis
mmol/l, indirect fraction. ALT, AST rates
are normal. What disease is the child most D. Bartholin gland cyst
E. Carcinoma of vulva
likely to have?
138. A 28-year-old female patient has
A. Hemolytic disease of newborn, AB0-
been admitted to a hospital. She states
conflict
B. Perinatal hepatitis to be ill for 12 years. On examination she
has been diagnosed with bronchiectasis
C. Hemolytic disease of newborn, Rh-
conflict with affection of the left lower lobe of
lung. What is the optimal treatment tacti-
D. Biliary atresia
cs for this patient?
E. Physiologic jaundice
A. Left lower lobectomy
135. Doctors of a polyclinic conduct
a statistical research of the disease B. Left-sided pneumoectomy
C. Antibiotic therpy
outcomes in two groups of patients (those
registered with dispensary departments D. Active drainage of the left pleural cavity
E. Bronchopulmonary lavage
and unregistered ones), depending on
age and level of hygiene. What type of 139. 4 weeks after myocardial infarction a
statistical tables would be most suitable 56-year-old patient developed acute heart
for profound analysis of the interrelation pain, pronounced dyspnea. Objectively:
between the above-mentioned variables? the patient’s condition is extremely grave,
there is marked cyanosis of face, swelling
A. Cross tabulation and throbbing of neck veins, peripheral
B. Analytical pulse is absent, the carotid artery pulse is
C. Group rhythmic, 130 bpm, AP is 60/20 mm Hg.
D. Simple Auscultation of heart reveals extremely
E. Developing muffled sounds, percussion reveals heart
136. A 57-year-old female complains border extension in both directions. What
of having a sensation of esophageal is the optimal treatment tactis for this
compresion, palpitation, difficult breathi- patient?
ng during eating solid food, occasional A. Pericardiocentesis and immediate
vomiting with a full mouth, "wet pi- thoracotomy
llow"sign at night for the last 6 months. B. Oxygen inhalation
Objectively: body tempearture - 39oC, C. Puncture of the pleural cavity on the left
height - 168 cm, weight - 72 kg, Ps- 76/min, D. Conservative treatment, infusion of
АP- 120/80 mm Hg. X-ray revealed a adrenomimetics
considerable dilation of esophagus and E. Pleural cavity drainage
its constriction in the cardial part. What
pathology is most likely to have caused 140. A 15-year-old boy feels pain in the
dysphagia in this patient? region of the left knee joint. Objecti-
vely: the soft tissues in the affected region
A. Achalasia cardiae are infiltrated, the joint function is limi-
B. Primary esophagism ted. Radiography reveals a focus of bone
C. Hiatal hernia destruction in the distal metaepiphysial
D. Esophageal carcinoma segment of the left femur. The destruction
E. Reflux esophagitis is accompanied by periosteal detachment
137. A 37-year-old patient complains of and a defect formed within cortex of
acute pain in the region of genitals, Codman triangle bone. X-ray of chest
swelling of the labia, pain when walki- shows multiple microfocal metastases.
ng. Objectively: body temperature is What is the most likely pathology?
38, 7oC, Ps- 98/min. In the interior of A. Osteogenic sarcoma
the right labia there is a dense, painful B. Fibrosarcoma
tumour-like formation 5,0x4,5 cm large, C. Chondrosarcoma
the skin and mucous membrane of geni- D. Ewing’s sarcoma
tals is hyperemic, there are profuse foul- E. Juxtacortical sarcoma
smelling discharges. What is the most li-
Krok 2 Medicine 2013 20

141. On the first day after a surgery for di- tation: she makes stereotypic grimaces,
ffuse toxic goiter a patient developed di- exposed, masturbating in front of a loud
fficulty breathing, cold sweats, weakness. laugh, repeating the stereotypical abusive
Objectively: pale skin, body temperature shouts. The patient should be assigned:
- 38, 5o C, RR - 25/min, Ps- 110/min, AP-
90/60 mm Hg. What early postoperative A. Neuroleptics
complication occurred in the patient? B. Antidepressants
C. Tranquilizers
A. Thyrotoxic crisis D. Nootropics
B. Hypothyroid crisis E. Mood stabilizers
C. Postoperative tetany
D. Acute thyroiditis 145. A young woman with seborrhea adi-
E. Compression of the trachea by the posa has numerous non-itchy light brown
hematoma and white spots with clear outlines and
defurfuration on the torso and shoulder
142. A 49-year-old patient consulted a skin. What is the provisional diagnosis?
doctor about difficult swallowing, voice
hoarseness, weight loss. These symptoms A. Pityriasis versicolor (scaly skin disease)
have been gradually progressing for the B. Tinea corporis
last 3 months. Objectively: the patient is C. Seborrheic dermatitis
exhausted, there are enlarged supraclavi- D. Pityriasis rosea
cular lymph nodes. Esophagoscopy E. Vitiligo
revealed no oesophageal pathology. Whi-
ch of the following studies is most 146. 10 days after birth a newborn
appropriate in this case? developed a sudden fever up to 38, 1o C.
Objectively: the skin in the regi-
A. Computed tomography of chest, medi- on of navel, abdomen and chest is
astinum erythematous; there are multiple pea-
B. X-ray of lungs sized blisters with no infiltration at the
C. Multiplanar imaging of esophagus base; single bright red moist erosions wi-
D. Radioisotope investigation of chest th epidermal fragments on the periphery.
E. Ultrasound investigation of mediasti- What is your provisional diagnosis?
num
A. Epidemic pemphigus of newborn
143. A 45-year-old female patient B. Syphilitic pemphigus
complaining of general weakness, nausea C. Streptococcal impetigo
and vomiting hass been delivered to a D. Vulgar impetigo
hospital by the ambulance. Recently there E. Atopic dermatitis
has been a lack of appetite, weight loss.
Objectively: hyperpigmentation of skin, 147. An emergency physician arrived to
blood pressure at the rate of 70/45 mm Hg, provide medical care for a hangman taken
bradycardia. Additional studies revealed out of the loop by his relatives. The doctor
the reduced concentration of aldosterone revealed no pulse in the carotid arteri-
and cortisol in blood, decreased excretion es, absence of consciousness, spontaneous
of 17-ketosteroids and 17-oxyketosteroids breathing and corneal reflexes; cadaver
in the urine, hyponatremia, chloropenia, spots on the back and posterior parts of
hypokalemia. What therapeutic measures extremities. A person can be declared
are required? dead if the following sign is present:

A. To administer glucocorticoids, mi- A. Cadaver spots


neralocorticoids, and a diet with a high B. Absence of spontaneous breathing
content of cooking salt C. Absence of corneal reflexes
B. To prescribe a diet with a high content D. Pulselessness
of cooking salt E. Unconsciousness
C. To administer prednisolone 148. A 40-year-old patient is registered
D. To administer aldosterone in a narcological dispensary. Somatically:
E. To administer insulin skin is dramatically hyperemic, sclera are
144. A 23-year-old female patient has a injected, hyperhidrosis is present. AP-
mental disease since the age of 18, the 140/100 mm Hg, heart rate - 100/min.
course of disease has no remission peri- Mental state: autopsychic orientation is
ods. At a hospital the patient mostly intact, allopsychic orientation is distorted.
presents with non-purposeful foolish exci- The patient presents with motor anxiety.
Krok 2 Medicine 2013 21

There is a look of fear on his face. He Prostate-specific antigen rate is of 5 ng/ml.


refuses to talk about his problems and What is the most likely disease that caused
asks to release him immediately, because acute urinary retention?
he "may be killed."This state developed a
day after a regular drinking bout. What is A. Prostatic hyperplasia
your provisional diagnosis? B. Prostate carcinoma
C. Sclerosis of the prostate
A. Delirium tremens D. Tuberculosis of the prostate
B. Organic delirium E. Acute prostatitis
C. Paranoia
D. Alcoholic hallucinosis 152. A 10 week pregnant woman was
E. Alcoholic paranoid admitted to a hospital for recurrent pain
in the lower abdomen, bloody discharges
149. A 35-year-old female patient has gai- from the genital tracts. The problems
ned 20 kg weight within a year with the turned up after ARVI. The woman was
normal diet. She complains of chill, sleepi- registered for antenatal care. Speculum
ness, shortness of breath. The patient’s examination revealed cyanosis of vagi-
mother and sister are corpulent. Objecti- nal mucosa, clean cervix, open cervical
vely: height - 160 cm, weight - 92 kg, canal discharging blood and blood clots;
BMI - 35,9. Obesity is uniform, there are the lower pole of the gestational sac was
no striae. The face is amimic. The skin visible. What tactics should be chosen?
is dry. The tongue is thickened. Heart
sounds are muffled. HR- 56/min, AP- A. Curettage of the uterus
140/100 mm Hg. The patient has consti- B. Pregnancy maintenance therapy
pations, amenorrhea for 5 months. TSH- C. Expectant management, surveillance
28 mkME/l (normal rate - 0,32-5). Crani- D. Hysterectomy
ogram shows no pathology. What is the E. Antiviral therapy
etiology of obesity?
153. A patient with fibromyoma of uterus
A. Hypothyroid sized up to 8-9 weeks of pregnancy
B. Hypo-ovarian consulted a gynaecologist about acute
C. Hypothalamic-pituitary pain in the lower abdomen. Examination
D. Alimentary and constitutive revealed pronounced positive symptoms
E. Hypercorticoid of peritoneal irritation, high leukocytosis.
Vaginal examination revealed that the
150. A 26-year-old patient consulted a uterus was enlarged corresponding to
doctor abut sore throat, fever up to 9 weeks of pregnancy due to the fi-
38, 2oC. A week ago, the patient had angi- bromatous nodes, one of which was mobi-
na, didn’t follow medical recommendati- le and extremely painful. Appendages
ons. On examination, the patient had were not palpable. There were moderate
forced position of his head, trismus of mucous discharges. What is the optimal
chewing muscles. Left peritonsillar regi- treatment tactics?
on is markedly hyperemic, swollen. What
is the provisional diagnosis? A. Urgent surgery (laparotomy)
B. Surveillance and spasmolytic therapy
A. Left-sided peritonsillar abscess C. Fractional diagnostic curettage of the
B. Meningitis uterine cavity
C. Phlegmonous angina D. Surgical laparoscopy
D. Diphtheria of the pharynx E. Surveillance and antibacterial therapy
E. Tonsil tumour
154. A 40-year-old female patient
151. A 77-year-old patient complains of complains of having a bulge on the anteri-
inability to urinate, bursting pain above or surface of neck for 5 years. Objecti-
the pubis. The patient developed acute vely: Ps- 72 bpm, arterial pressure - 110/70
condition 12 hours ago. Objectively: mm Hg, in the right lobe of thyroid gland
full urinary bladder is palpable above palpation reveals a mobile 4x2 cm node,
the pubis. Rectal prostate is enlarged, the left lobe is not palpable, the basal
dense and elastic, well-defined, with no metabolic rate is 6%. What is the most
nodes. Interlobular sulcus is distinct. likely diagnosis?
Ultrasonography results: prostate volume
is 120 cm3 , it projects into the bladder
cavity, has homogeneous parenchyma.
Krok 2 Medicine 2013 22

A. Nodular euthyroid goiter headache in the occipital region, nausea,


B. Nodular hyperthyroid goiter choking, opplotentes. The presentati-
C. Riedel’s thyroiditis ons appeared after a physical exerti-
D. Mixed euthyroid goiter on. Objectively: the patient is excited.
E. The median cervical cyst Face is hyperemic. Skin is pale. Heart
sounds are regular, the 2nd aortic sound
155. A multigravida at 39 weeks of gestati- is accentuated. AP- 240/120 mm Hg, HR-
on has been delivered to a hospital having 92/min. Auscultation reveals some fine
a regular labour activity for 8 hours, the moist rales in the lower parts of the lungs.
waters burst an hour ago. She complains Liver is not enlarged. ECG shows signs of
of headache, seeing spots. AP is of 180/100 hypertrophy and left ventricular overload.
mm Hg. Urine test results: protein - 3,3 What is the most likely diagnosis?
g/l, hyaline cylinders. Fetal heart rate is
140/min, rhythmical. Vaginal examination A. Complicated hypertensic crisis,
reveals complete crevical dilatation, the pulmonary edema
fetal head is on the pelvic floor, sagittal B. Acute myocardial infarction, pulmonary
suture is in line with obstetric conjugate, edema
the occipital fontanel is under the pubis. C. Bronchial asthma exacerbation
What is the optimal tactics of labour D. Uncomplicated hypertensic crisis
management? E. Community-acquired pneumonia
A. Outlet forceps 159. A patient complains of being unable
B. Cavity forceps to get pregnant for 5 years. A complete
C. Cesarean section clinical examination gave the following
D. Vacuum extraction of the fetus results: hormonal function is not impai-
E. Conservative labour management red, urogenital infection hasn’t been
found, on hysterosalpingography both
156. After a contact with chemicals a tubes were filled with the contrast medi-
plant worker has suddenly developed um up to the isthmic segment, abdominal
stridor, voice hoarseness, barking cough, contrast was not visualized. The patient’s
progressing dyspnea. Objective exami- husband is healthy. What tactics will be
nation reveals acrocyanosis. What is your most effective?
provisional diagnosis?
A. In-vitro fertilization
A. Laryngeal edema B. Insemination with husband’s sperm
B. Laryngeal carcinoma C. ICSI within in-vitro fertilization
C. PATE program
D. Pulmonary atelectasis D. Hydrotubation
E. Pneumothorax E. Laparoscopic tubal plasty
157. A 58-year-old patient complains of 160. Six months ago, a 5-year-old child
general weakness, loss of 10 kg of wei- was operated for CHD. For the last 3
ght within 1,5 months, progressive pain weeks he has complained of fever, heart
in the lumbar region, increased blood pain, aching muscles and bones. Exami-
pressure up to 220/160 mm Hg, subfebri- nation results: "white-coffee"skin colour,
le temperature. Objectively: in the ri- auscultation revealed systolic murmur in
ght hypochondrium palpation reveals a the region of heart along with a noise in
formation with uneven surface and low the III-IV intercostal space. Examinati-
mobility; veins of the spermatic cord and on of fingertips revealed Janeway lesions.
scrotum are dilated. Blood test results: What is your provisional diagnosis?
Hb- 86 g/l, ESR- 44 mm/h. Urine test
results: specific gravity - 1020, protein - A. Infectious endocarditis
0,99 g/l, RBCs - cover the whole field of B. Sepsis
vision, WBCs - 4-6 in the field of vision. C. Nonrheumatic carditis
What is the provisional diagnosis? D. Acute rheumatic fever
E. Typhoid fever
A. Renal tumour
B. Urolithiasis 161. A 45-year-old female patient has
C. Acute pyelonephritis worked as a painter for 14 years. Contacts
D. Acute glomerulonephritis with synthetic paint result in face skin
E. Nephroptosis redness, swelling, intense itching, oozi-
ng lesions. The symptoms disappear after
158. A 58-year-old patient complains of a the exposure to chemical agents, but recur
Krok 2 Medicine 2013 23

even at the smell of paint. The symptom by pain syndrome. Objectively: body
intensity progresses with relapses. Make a temperature is at the rate of 36, 8oC, Ps-
provisional diagnosis: 78/min, abdomen is soft and painless,
the symptoms of peritoneal irritation are
A. Occupational eczema present, palpation reveals a dramatically
B. Simple contact dermatitis enlarged, tense gallbladder. What disease
C. Contact-type allergy are these symptoms typical for?
D. Urticaria
E. Toksikodermiya A. Cancer of the pancreatic head
B. Duodenal ulcer
162. In a cold weather, the emergency C. Acute cholecystitis
room admitted a patient pulled out of D. Chronic cholecystitis
the open water. There was no respiratory E. Lamblia-induced cholecystitis
contact with the water. The patient is exci-
ted, pale, complains of pain, numbness of 166. A 22-year-old vegetarian patient wi-
hands and feet, cold shiver. Breathing rate th signs of malnutrition consulted a doctor
is 22/min, AP- 120/90 mm Hg, Ps- 110/min, about smell and taste distortion, angular
rectal temperature is 34, 5oC. What kind stomatitis. Objectively: expressively blue
of warming is indicated for this patient? sclerae. The patient was diagnosed with
iron deficiency anemia. What is the domi-
A. Passive warming nating clinical syndrome?
B. Infusion of 37o C solutions
C. Hot compresses A. Sideropenic
D. Warm bath B. Anaemic
E. Hemodialysis with blood warming C. Haemologic
D. Haemolytic
163. Survey radiograph of a 52-year-old E. Myelodysplastic
worker of an agglomeration plant (28
years of experience, the concentration of 167. A 13-year-old girl was admitted to
metal dust is 22-37 mg/m3 ) shows mildly the gynecological department with heavy
pronounced interstitial fibrosis with di- bleeding, which appeared after a long
ffused contrast well-defined small nodular delay of menstruation. Shortly before,
shadows. The patient has no complaints. the girl suffered a serious psychotrauma.
Pulmonary function is not compromised. Her menarche occurred at the age of
What is the provisional diagnosis? 11, she has a 30-day cycle with 5 to
6 days of moderate, painless bleeding.
A. Siderosis The patient is somatically healthy, of
B. Silicosis normosthenic constitution with height
C. Anthraco-silicatosis of 160 cm, weight of 42 kg. The pati-
D. Silicatosis ent is pale. Rectoabdominal examination
E. Anthracosis revealed that the uterus was of normal si-
ze and consistency, anteflexio-versio, the
164. A week ago a 65-year-old patient appendages were not changed. What is
suffered an acute myocardial infarction, the most likely diagnosis?
his general condition deteriorated: he
complains of dyspnea at rest, pronounced A. Juvenile bleeding
weakness. Objectively: edema of the B. Ovarian cyst
lower extremities, ascites is present. Heart C. Hysteromyoma
borders are extended, paradoxical pulse is D. Girl is healthy
2 cm displaced from the apex beat to the E. Amenorrhea
left. What is the most likely diagnosis?
168. It is planned to build a multi-
A. Acute cardiac aneurysm disciplinary hospital with 500 beds in a
B. Recurrent myocardial infarction town. Specify the location of a polyclinic
C. Acute pericarditis within the medical centre:
D. Cardiosclerotic aneurysm
E. Pulmonary embolism
165. A 64-year-old patient has been
referred to planned hospitalization
for general weakness, poor appetite,
progressive jaundice which appeared over
3 weeks ago and wasn’t accompanied
Krok 2 Medicine 2013 24

A. At the main entrance A. Post-castration syndrome


B. In the garden and park area B. Premenstrual syndrome
C. In the centre of the territory near C. Early pathological menopause
medical buildings D. Secondary psychogenic amenorrhea
D. It is not allowed to place the polyclinic E. Physiological premenopause
within the centre territory
E. In the service zone 172. A 2-year-old child in a satisfactory
condition periodically presents with
169. A 48-year-old patient complains of moderate proteinuria, microhematuria.
weakness, subfebrile temperature, achi- USI results: the left kidney is
ng pain in the kidney region. These undetectable, the right one is enlarged,
presentations turned up three months there are signs of double pyelocaliceal
ago after hypothermia. Objectively: ki- system. What study is required to speci-
dneys are painful on palpation, there is bi- fy the diagnosis?
laterally positive Pasternatsky’s symptom.
Urine test res: acid reaction, pronounced A. Excretory urography
leukocyturia, microhematuria, minor B. Micturating cystography
proteinuria - 0,165-0,33 g/l. After the urine C. Retrograde urography
sample had been inoculated on conventi- D. Doppler study of renal vessels
onal media, bacteriuria were not found. E. Radioisotope renal scan
What research is most required in this
case? 173. Explosion of a tank with benzene at a
chemical plant has killed and wounded
A. Urine test for Mycobacterium a large number of people. There are
tuberculosis over 50 victims with burns, mechani-
B. Daily proteinuria cal traumas and intoxication. Specify
C. Nechiporenko urine test the main elements of medical care and
D. Zimnitsky urine test evacuation of population in this situation:
E. Isotope renography
A. Sorting, medical assistance, evacuation
170. A 22-year-old patient complains B. Sorting, evacuation, treatment
of amenorrhea for 8 months. Menarche C. Medical assistance, evacuation, isolation
occured at the age of 12,5. Since the age D. Isolation, rescue activity, recovery
of 18 the patient has a history of irregular E. Sorting, recovery, rescue activity
menstruation. The patient is nulligravi-
da. The mammary glands are developed 174. A 26-year-old patient with affective
properly, nipples discharge drops of mi- bipolar disorder has developed a condi-
lk when pressed. Gynecological study tion manifested by mood improvement,
results: prolactin level is 2 times higher behavioural and sexual hyperactivity,
than normal. CT reveals a bulky formati- verbosity, active body language, reduced
on with a diameter of 4 mm in the region need for sleep. Which of the following
of sella. What is the most likely diagnosis? drugs are most effective in this case?

A. Pituitary tumour A. Neuroleptics with a sedative effect


B. Lactational amenorrhea B. Antidepressants with an activating
C. Stein-Leventhal syndrome effect
D. Sheehan’s syndrome C. Neuroleptics with an activating effect
E. Pituitary basophilia D. Tranquilizers
E. Antidepressants with a sedative effect
171. A 38-year-old female patient
complains about hot flashes and feeli- 175. An emergency doctor has diagnosed
ng of intense heat arising up to 5 times a 32-year-old woman with generalized
a day, headaches in the occipital region convulsive status epilepticus. The deteri-
along with high blood pressure, palpitati- oration in the patient’s condition is caused
ons, dizziness, fatigue, irritability, memory by a sudden gap in the epilepsy treatment.
impairment. 6 months ago the patient Specify the doctor’s further tactics:
underwent extirpation of the uterus wi-
th its appendages. What is the most likely
diagnosis?
Krok 2 Medicine 2013 25

A. Hospitalization in the intensive care accompanied by nausea. Surgical exami-


unit nation revealed moist tongue, Ps- 76 bpm.
B. Hospitalization in the department of AP- 130/80 mm Hg. Abdomen was soft,
neurology slightly painful in the right iliac region
C. Hospitalization in the department of on deep palpation, the symptoms of the
neurosurgery peritoneum irritation were doubtful. In
D. Outpatient monitoring by a blood: RBCs - 4, 0 · 1012 /l, Hb- 135 g/l,
neuropathologist WBCs - 9, 5 · 109 /l, stab neutrophils - 5%,
E. Outpatient monitoring by a segmentonuclear - 52%, lymphocytes -
neurosurgeon 38%, monocytes - 5%, ESR - 20 mm/h.
176. A 19-year-old patient complains Specify the doctor’s further tactics:
of dyspnea on exertion. He often has A. Emergency operation for acute
bronchitis and pneumonia. Since chi- appendicitis
ldhood, the patient presents with cardi- B. Hospitalization, dynamic surveillance
ac murmur. Auscultation revealed splitti- C. Send the patient home
ng of the II sound above the pulmonary D. Refer the patient to a district therapist
artery, systolic murmur in 3 intercostal E. Administration of additional examinati-
space at the left sternal border. ECG on: abdominal ultrasound, x-ray contrast
showed right bundle branch block. What study of the gastrointestinal tract
is the provisional diagnosis?
180. A 55-year-old patient whose
A. Atrial septal defect menstruation stopped 5 years ago
B. Open ductus arteriosus complains of vaginal dryness, frequent
C. Aortarctia and painful urination. Gynecologist
D. Aortic stenosis revealed signs of atrophic colpitis. Urine
E. Mitral insufficiency analysis revealed no peculiarities. Whi-
177. 20 minutes after a normal delivery ch locally acting product will provide the
at 39 weeks a puerpera had a single proper therapeutic effect?
temperature rise up to 38o C. Objecti- A. Vaginal suppositories "Ovestin"
vely: the uterus is dense, located between B. Vaginal tablets "Tergynan"
the navel and the pubis, painless. Lochia C. Vaginal cream "Meratin Combi"
are bloody, of small amount. Breasts are D. Vaginal gel "Metronidazole"
moderately soft and painless. What is the E. Vaginal cream "Dalacin"
optimal tactics?
181. A 63-year-old patient with persistent
A. Further follow-up atrial fibrillation complains of moderate
B. Antibiotic therapy dyspnea. Objectively: peripheral edemata
C. Appointment antipyretic are absent, vesicular respiration is present,
D. Manual examination of the uterine heart rate - 72/min, AP- 140/90 mm Hg.
cavity What combination of drugs will be most
E. Expression of breast useful in the secondary prevention of
178. A 30-year-old patient got in a car heart failure?
accident. He is unconscious, pale, has A. Beta-blockers, ACE inhibitors
thready pulse. In the middle third of the B. Beta-blockers, cardiac glycosides
right thigh there is an extensive lacerati- C. Cardiac glycosides, diuretics
on with ongoing profuse external arteri- D. Cardiac glycosides, ACE inhibitors
al bleeding. What urgent actions must be E. Diuretics, beta-blockers
taken to save the life of the patient?
182. A 57-year-old patient had an attack
A. Tourniquet above the wound of the of retrosternal pain that lasted more than
right thigh 1,5 hours. Objectively: the patient is inert,
B. Tourniquet below the wound of the adynamic, has pale skin, cold extremities,
right thigh poor volume pulse, heart rate - 120/min,
C. Artificial lung ventilation AP- 70/40 mm Hg. ECG shows ST elevati-
D. Precordial thump on in leads II, III, aVF. What condition are
E. Plaster bar these changes typical for?
179. A 75-year-old male patient complains
of slight pain in the right iliac region. The
abdominal pain arose 6 days ago and was
Krok 2 Medicine 2013 26

A. Cardiogenic shock nodes were not changed. USI results: in


B. Arrhythmogenic shock the superior external quadrant of the right
C. Perforated gastric ulcer mammary gland there was a big formation
D. Acute pericarditis of increased echogenicity, sized 18x17 mm.
E. Acute pancreatitis The patient was provisionally diagnosed
with fibroadenoma. What is a doctor’s
183. An 8-year-old boy has a 2-year hi- further tactics?
story of blotchy itchy rash appearing
after eating citrus fruit. The first eruption A. Surgical removal of the tumour prior to
occurred at the age of 6 months after the pregnancy
introduction of juices to the baby’s diet. B. Dynamic follow-up
Father has a history of bronchial asthma, C. Surgical treatment after pregnancy
mother - that of allergic rhinitis. What is D. Radical mastectomy
the most likely diagnosis? E. Nonsteroid anti-inflammatory drugs,
oral contraceptives
A. Atopic dermatitis
B. Psoriasis 187. A patient is 31 years old. Double-
C. Pityriasis Rosea contrast barium swallow revealed a filli-
D. Urticaria ng defect on the posterior wall in the mi-
E. Quincke’s edema ddle segment of esophagus. The defect
looked like a well-defined oval 1,8x1,3 cm
184. On the 10th day postpartum a large. Mucosal folds adjacent to the defect
puerperant woman complains of pain and were intact, peristalsis and elasticity of
heaviness in the left mammary gland. the walls remained unchanged. Digestive
Body temperature is 38, 8o C, Ps- 94 bpm. tract problems were absent. What is the
The left mammary gland is edematic, provisional diagnosis?
the supero-external quadrant of skin
is hyperemic. Fluctuation symptom is A. Esophageal tumour
absent. The nipples discharge drops of B. Achalasia cardia
milk when pressed. What is a doctor’s C. Esophageal burn
further tactics? D. Diverticulum
E. Barrett’s esophagus
A. Antibiotic therapy, immobilization and
expression of breast milk 188. A 50-year-old patient has worked
B. Compress to both mammary glands at a chemical plant for 15 years. His
C. Inhibition of lactation work involved using xylene solvent. The
D. Physiotherapy patient was hospitalized with suspected
E. Opening of the abscess and drainage of chronic intoxication. He was found to
the mammary gland have anemic syndrome. What is the first-
priority measure of secondary anemia
185. A 25-year-old patient has been admi- prevention?
tted to the hospital with the followi-
ng problems: weakness, sweating, itchi- A. Job change
ng, weight loss, enlarged submandibular, B. Administration of glucocorticosteroids
cervical, axillary, inguinal lymph nodes. C. Administration of iron supplements
Objectively: hepatomegaly. Lymph node D. Including seafood into the diet
biopsy revealed giant Berezovsky-Reed- E. Including meat into the diet
Sternberg- cells, polymorphocellular
granuloma made by lymphocytes, reti- 189. A 49-year-old patient complains of
cular cells, neutrophils, eosinophils, fi- itching, burning in the external genitals,
brous tissue, plasma cells. What is the frequent urination. The symptoms has
most likely diagnosis? been present for the last 7 months. The
patient has irregular menstruation, once
A. Lymphogranulomatosis every 3-4 months. Over the last 2 years
B. Lymph node tuberculosis she presents with hot flashes, sweating,
C. Lymphoreticulosarcoma sleep disturbance. Examination revealed
D. Cancer metastases to lymph nodes no pathological changes of the internal
E. Macofollicular reticulosis reproductive organs. Complete blood
count and urinalysis showed no pathologi-
186. During self-examination a 22-year- cal changes. Vaginal smear contained 20-
old patient revealed a mammary tumour. 25 leukocytes in the field of vision, mixed
Palpation revealed a firm, painless, mobi- flora. What is the most likely diagnosis?
le formation up to 2 cm, peripheral lymph
Krok 2 Medicine 2013 27

A. Menopausal syndrome
B. Cystitis A. Amiodarone
C. Trichomonas colpitis B. Flecainide
D. Vulvitis C. Encainide
E. Bacterial vaginosis D. Moracizine
E. Digoxin
190. A 56-year-old female patient
complains of recurrent attacks of intensi- 194. A 38-year-old male works within the
ve pain irradiating along the ureters. Uri- range of ionizing radiation. At a routi-
ne test results: protein - 0,37 g/l, RBCs- ne medical examination he presents no
20-25 in the field of vision, WBCs - 12- problems. In blood: RBCs - 4, 5·1012/l, Hb-
14 in the field of vision. What method 80 g/l, WBCs - 2, 8 · 109 /l, thrombocytes -
of instrumental diagnostics is the most 30 · 109 /l. Decide if this person can work
informative for the diagnosis? with sources of ionizing radiation:
A. Intravenous urography A. Working with radioactive substances
B. USI of kidneys and other sources of ionizing radiation is
C. Computer tomography contraindicated
D. Radioisotope renography B. The patient is allowed to work with
E. Cystoscopy radioactive substances
C. The patient can only work with radi-
191. 2 weeks after labour a parturi- oactive substances of low activity
ent woman developed breast pain bei- D. The patient can be allowed to work
ng observed for 3 days. Examination after an extended medical examination
revealed body temperature at the rate E. The patient is allowed to work with
of 39o C, chills, weakness, hyperaemia, radioactive substances for the limited
enlargement, pain and deformity of the period of time
mammary gland. On palpation the infi-
ltrate was found to have an area of softeni- 195. A puerpera breastfeeding for 1,5
ng and fluctuation. What is the most likely weeks consulted a doctor about uniform
diagnosis? breast engorgement. Breasts are painful.
The body temperature is of 36, 6o C. Milk
A. Infiltrative-purulent mastitis expressing is difficult. What is the most li-
B. Phlegmonous mastitis kely diagnosis?
C. Lactostasis
D. Serous mastitis A. Lactostasis
E. Mastopathy B. Infiltrative mastitis
C. Purulent mastitis
192. A 22-day-old infant had developed D. Fibrocystic mastopathy
red subcutaneous nodules from 1,0 to 1,5 E. Gangrenous mastitis
cm large on the scalp. Later the nodules
suppurated, body temperature rose up 196. A 28-year-old male patient complains
to 37, 7o C, there appeared symptoms of of regurgitation, cough and heartburn that
intoxication, the regional lymph nodes occurs every day after a meal, when bendi-
grew bigger. Blood test results: anemia, ng forward or lying down. These problems
leukocytosis, neutrophilia, accelerated have been observed for 4 years. Objective
ESR. What is the most likely diagnosis? status and laboratory values are normal.
FEGDS revealed endoesophagitis. What
A. Pseudofurunculosis is the leading factor in the development of
B. Pemphigus this disease?
C. Vesiculopustulosis
D. Scalp phlegmon A. Failure of the inferior esophageal
E. - sphincter
B. Hypersecretion of hydrochloric acid
193. A 70-year-old patient consulted a C. Duodeno-gastric reflux
doctor about arrhythmic cardiac activity, D. Hypergastrinemia
dyspnea. Objectively: AP- 150/90 mm Hg, E. Helicobacter pylori infection
extrasystole arrhythmia (10-12 beats per
minute), left ventricular systolic dysfuncti- 197. A 24-year-old patient had been deli-
on (ejection fraction at the rate of 42%). vered to the thoracic department with
Which of antiarrhythmic drugs should a chest injury, a fracture of the IV, V,
be administered as initial therapy in this VI ribs on the right. Plan radiography
case? shows the fluid level in the pleural cavity
Krok 2 Medicine 2013 28

reaching the III rib on the right. Puncture had experienced antenatal fetal death due
blood contained clots. What is the optimal to hemolytic disease. What is the optimal
treatment tactics? tactics of pregnancy management?
A. Emergency thoracotomy A. Early delivery
B. Pleural puncture B. Delivery at 37 weeks of gestation
C. Thoracentesis and thoracostomy C. Screening for Rh-antibodies 2 weeks
D. Hemostatic therapy later and early delivery in case of further
E. Medical thoracoscopy titer rise
D. Introduction of anti-Rh (D)
198. A 10-year-old patient complains of immunoglobulin
skin itch that occurs at night. Objecti- E. Ultrasound for signs of hemolytic
vely: multiple paired papules, burrow disease of the fetus
tracks in the interdigital skin folds, on
the anterolateral surfaces of abdomen and 200. A 7-year-old female child has
buttocks. Specify the period of regular developed an acute condition. She
medical check-up for pupils of the class complains of a headache, two onsets of
where the patient learns: vomiting. Objectively: deferred reactions,
body temperature - 39, 3o C, pronounced
A. 2 months hyperesthesia, nuchal rigidity, positive
B. 2 weeks superior and inferior Brudzinski’s signs,
C. 5 days symmetric Kernig’s sign. What is the
D. 6 months provisional diagnosis?
E. 1 year
A. Meningitis
199. Examination of a Rh-negative B. Food toxicoinfection
pregnant woman at 32 weeks of gestation C. Craniocerebral trauma
revealed a four-time rise of Rh-antibody D. Toxic encephalopathy
titer within 2 weeks, the titer was 1:64. E. Encephalitis
In the first two pregnancies the patient
Krok 2 Medicine 2014 1

1. Head of a department and a trade-union A. Schede-type vertical suspension


group have appealed to the head of a hospital B. Closed reduction
about dismissal of the senior nurse who has C. Intramedullary osteosynthesis
17-year record of service. The facts of charge D. Use of Ilizarov apparatus
were confirmed and recognized by the nurse E. Periosteal osteosynthesis
herself. The nurse lives with a daughter (who is
5. What juice should be included in a complex
divorced and unemployed) and a 9-month-old drug and dietary therapy for patients with
grandson. Make an administrative decision: gastric ulcer or duodenal ulcer and increased
A. To continue the worker in office with a gastric juice acidity in order to accelerate the
warning of dismissal in case of repeated violati- ulcer healing?
on of labor discipline A. Potato, potato and carrot
B. To discharge the worker, i.e. to satisfy B. Apple, birch and apple
demands of the collective C. Pumpkin
C. To issue the sick leave D. Cabbage, cabbage and carrot
D. To embark other officials or public organi- E. Celery, parsley
zations with this problem
E. - 6. A 7-year-old child complains of itchi-
ng, papular erythematous rash, dry skin.
2. A 50-year-old female patient complains of Objectively: there is lichenification in the
aching pain in the lower abdomen. She has popliteal fossae and antecubital spaces.
a history of normal menstrual cycle. At the What immunologic indicator if found in the
age of 40, the patient underwent a surgery for blood serum will verify the diagnosis (atopic
gastric ulcer. Examination findings: abdomen
is soft, in the hypogastrium there is a well- dermatitis)?
defined nodular tumor of limited mobility. A. Total IgE
Vaginal examination findings: the cervix is B. Secretory IgA
clean, of cylindrical shape. Body of the uterus C. IgM
cannot be palpated separately. On both sides D. IgG
of the uterus palpation reveals tight tumors wi- E. IgD
th an uneven surface. The tumors are immobi-
le andl fill the whole pelvic cavity. What is the 7. A 7-year-old patient presents with body
most likely diagnosis? temperature rise up to 39o C , dry cough, pain
in the lateral abdomen. Objectively: there is
A. Krukenberg tumor cyanosis of the nasolabial triangle, inspiratory
B. Ovarian fibroid dyspnea with accessory muscle recruitment.
C. Ovarian granulosa cell tumor Percussion reveals pulmonary dullness; among
D. Bilateral pioovarium auscultation findings there are diminished
E. Subserous metrofibroma breath sounds, crepitant rales. Respiratory
3. A 38-year-old male patient complains of rate is of 50/min, HR- 120/min. Evaluate the
marked dyspnea that escalates with physical grade of respiratory failure in the patient:
exertion. The problems, namely acute chest A. II
pain on the left and cough, arose unexpectedly B. I
2 hours before at work. The pain abated, C. III
but there were progressing dyspnea, dizzi- D. IV
ness, pallor, cold sweat, cyanosis. Auscultation E. 0
reveals the absence of vesicular breath sounds,
radiograph shows a shadow on the left. What 8. A 43-year-old alcohol abuser had not
pathology can be suspected? consumed alcohol for the last two days. In
the evening he claimed to see rats and feel
A. Left-sided spontaneous pneumothorax like they bite his feet. The patient is disori-
B. Pulmonary infarction ented, agitated, all the time attempts to run
C. Pleurisy somewhere. Specify the psychopathological
D. Left-sided pneumonia syndrome:
E. Lung abscess
A. Delirious
4. A 2-year-old boy has been admitted to B. Amential
the casualty department for the pain and C. Oneiroid
deformity of his right thigh. Radiograph D. Choreatic
shows a femoral fracture with longitudinal E. Ganser’s syndrome
displacement. What method of treatment is
indicated for the patient? 9. A 67-year-old male patient complains of
rash, severe pain in the subscapular regi-
on on the right. Objectively: skin in the ri-
ght subscapular region is covered with li-
nearly arranged pink-red edematous lesions
that are somewhat infiltrated, and have clear
Krok 2 Medicine 2014 2

boundaries. On the lesion surface there are of intense pain in the right hypochondrium
vesicles with transparent exudate. What is the irradiating to the right supraclavicular region.
most likely diagnosis? Skin and sclerae are icteric. There is tension
and tenderness in the right hypochondrium
A. Herpes zoster on palpation. Body temperature is 38, 8o C .
B. Duhring dermatitis Blood test results: WBC- 11, 2 · 109 /l, total bi-
C. Erysipelas lirubin - 112 mmol/l (conjugated - 86 mmol/l,
D. Atopic dermatitis unconjugated - 26 mmol/l). What is the most
E. Impetigo likely diagnosis?
10. An 8-year-old boy was brought to the A. Cholangitis
admission department by his parents. Parents B. Acute pancreatitis
report that he has had pain in the right knee C. Acute appendicitis
for the last 9 months, recently mother has noti- D. Pancreatic tumor
ced some limitation of motion in his right leg, E. Perforated duodenal ulcer
and morning stiffness that doesn’t last till the
evening. What is the most likely diagnosis? 15. A 3-year-old girl has had an increase
in body temperature up to 38, 5o C for four
A. Juvenile rheumatoid arthritis days. The child refuses to eat. Over the last
B. Rheumatism two days, nose and mouth breathing has
C. Osteomyelitis of the knee joint become difficult. Mesopharyngoscopy reveals
D. Reactive arthritis hyperthermia and enlargement of tonsils, as
E. Traumatic arthritis well as hyperemia and bulging of the posteri-
11. A 13-year-old girl was admitted to the or wall of the oropharynx, which significantly
gynecology department for having a signifi- narrows the oropharyngeal lumen. What
cant bleeding from the genital tract for 10 complication of quinsy occurred in the pati-
days. The patient has a history of irregular ent?
menstrual cycle since menarche. Menarche A. Retropharyngeal abscess
occurred at the age of 11. Recto-abdominal B. Paratonsillar abscess
examination revealed no pathology. What is C. Parapharyngeal abscess
the provisional diagnosis? D. Phlegmon of the mouth floor
A. Juvenile uterine bleeding E. Laryngostenosis
B. Adenomyosis 16. Within a year, in a maternity hospital there
C. Injury of the external genitalia were 616 livebirths, one stillbirth, one infant
D. Werlhof’s disease died on the 5th day of life. What index can
E. Endometrial polyp most accurately describe this situation?
12. A 64-year-old male patient has a 35- A. Perinatal mortality
year history of chronic pancreatitis. In B. Total mortality
the last 5 years, he claims to observe the C. Birthrate
pain abatement, bloating, frequent bowel D. Infant mortality
movements up to 3-4 times a day, grayish, E. Natural increase
glossy stool with undigested food rests, the
progressive loss of body weight. Change of 17. In one of the surgical departments the
symptoms in the patient is due to overlay of: quality assurance testing of sterilization of
surgical instruments was performed. After
A. Exocrine pancreatic insufficiency an instrument had been treated with 1%
B. Endocrine pancreatic insufficiency phenolphthalein, the solution turned pink.
C. Lactase deficiency syndrome This indicates that the instrument has:
D. Irritable bowel syndrome
E. Chronic enterocolitis A. Synthetic detergent residues
B. Residual blood
13. During the doctor’s round, a 56-year-old C. Drugs residues
male patient with decompensated cirrhosis D. Residual tissue
complains of dizziness, palpitations, moving E. Disinfectant residues
black specks seen before the eyes, general
weakness. The patient is pale, Ps- 110/min, 18. A 21-year-old female patient consulted a
AP- 90/50 mm Hg. What complication is most gynecologist about itching, burning, watery
likely to have occurred in the patient? vaginal discharges with a fish-like smell.
Speculum examination revealed that the cervi-
A. Bleeding from esophageal varices cal and vaginal mucosa was of a normal pi-
B. Hepatocellular insufficiency nk color. Vaginal examination revealed no
C. Hepatic encephalopathy alterations of the uterus and appendages.
D. Acute coronary syndrome Gram-stained smears included clue cells.
E. Paroxysmal tachycardia What is the most likely pathology?
14. A 57-year-old female patient complains
Krok 2 Medicine 2014 3

A. Bacterial vaginosis (gardnerellosis) mitive judgments, is unabe to perform si-


B. Chlamydiosis mple arithmetic operations or explain simple
C. Gonorrhea metaphors. The patient is untidy, takes no
D. Trichomoniasis interest in anything, passive. Considers hi-
E. Candidiasis mself to be completely healthy. Qualify mental
condition of the patient:
19. A 9-year-old patient has measles. On
the 6th day after the rash appeared, the boy A. Total dementia
developed a condition manifested by dyspnea, B. Lacunar (dysmnestic) dementia
barking cough, stenotic respiration. Objecti- C. Somnolentia
vely: the rash on the face, neck and torso D. Korsakoff’s (amnesic) syndrome
turned brown. There is a branny desquamati- E. Hysterical pseudodementia
on. Respiratory rate is 22/min. What compli-
cation should be diagnosed? 23. A 13-year-old boy with hypertrophic cardi-
omyopathy complains of dyspnea on mini-
A. Laryngotracheitis mal exertion. EhoCG reveals asymmetric left
B. Bronchitis ventricular hypertrophy, signs of pulmonary
C. Pneumonia hypertension, dilatation of the left atrium. EF
D. Pharyngitis is 64%. The revealed alterations are indicative
E. Quinsy of:
20. A 26-year-old secundipara at 40 weeks of A. Diastolic heart failure
gestation arrived at the maternity ward after B. Systolic heart failure
the beginning of labor activity. 2 hours before, C. Primary pulmonary hypertension
bursting of waters occurred. The fetus was D. Primary arterial hypertension
in a longitudinal lie with cephalic presentati- E. Symptomatic arterial hypertension
on. Abdominal circumference was 100 cm,
fundal height - 42 cm. Contractions occurred 24. Study of natural illumination for a
every 4-5 minutes and lasted 25 seconds each. workplace in a secondary school classroom
Internal obstetric examination revealed cervi- revealed that the angle of sunlight incidence
cal effacement, opening by 4 cm. Fetal bladder was 25o , window opening angle - 3o , window-
was absent. Fetal head was pressed against the to-floor area ratio - 1:4, daylight ratio - 0,5%,
pelvic inlet. What complication arose in chi- depth ratio - 2. What indicators do not meet
ldbirth? hygienic standards?
A. Early amniorrhea A. Daylight ratio
B. Primary uterine inertia B. Window opening angle
C. Secondary uterine inertia C. Window-to-floor area ratio
D. Discoordinated labor D. Depth ratio
E. Clinically narrow pelvis E. Angle of incidence
21. Examination of a 35-year-old patient wi- 25. Bakers at bread production work in condi-
th rheumatism revealed that the right heart tions of high temperature and high heat radi-
border was 1 cm displaced outwards from the ation. What is used to increase the body’s
right parasternal line, the upper border was resistance to the unfavorable effects of these
on the level with inferior margin of the 1st harmful work environment factors?
rib, the left border was 1 cm in from the left
midclavicular line. Auscultation revealed atri- A. Vitamin preparations
al fibrillation, loud apical first sound, diastolic B. Milk
shock above the pulmonary artery. Echocardi- C. Pectin
ocopy revealed abnormal pattern of the mitral D. Therapeutic and preventive diet № 1
valve motion. What heart disease is characteri- E. Therapeutic and preventive diet number № 3
zed by these symptoms?
26. A 24-year-old male patient got a puncture
A. Mitral stenosis injury below the Poupart’s ligament, which
B. Mitral valve prolapse was accompanied by intense arterial bleedi-
C. Mitral valve insufficiency ng. The best method to temporarily stop the
D. Aortic stenosis bleeding in the patient would be:
E. Tricuspid valve insufficiency
A. Compression band
22. A 54-year-old male patient works as B. Esmarch’s tourniquet
an engineer. At the age of 35, he got C. Maximum limb bending
infected with syphilis and treated it with D. Compressing a blood vessel with a clamp
"traditional remedies". About 5 years ago, E. Wound suturing
he became forgetful, unable to cope with
work, told cynical jokes, bought useless thi- 27. 5 days before, a 26-year-old female patient
ngs, collected cigarette butts in the street. developed an acute condition. Objectively:
Objectively: the patient is indifferent, has marked headache, vomiting, weakness, poor
slow speech, dysarthria, can make only pri- appetite, temperature up to 39o C . Objecti-
Krok 2 Medicine 2014 4

vely: the patient is in a moderately grave agnosed with hidradenitis. What is the most
condition, excited. The face is hyperemic, likely causative agent of this disease?
sclerae are injected. The tongue is coated with
brown fur. The trunk and limbs are covered A. Staphylococci
with plentiful roseolous and petechial rash. B. Streptococci
Hepatosplenomegaly is present. Complement C. Proteus vulgaris
binding reaction with Rickettsia prowazekii D. Pseudomonas aeruginosa
is positive with the titer of 1:640. What drug E. Mixed infection
should be administered?
32. A 36-year-old female has a 7-year history of
A. Doxycycline pollen allergy. Over the last 2 years in August
B. Chloramphenicol and September (during ragweed flowering),
C. Penicillin the patient has had 2-3 asthma attacks that
D. Streptomycin could be treated with one dose of salbutamol.
E. Metronidazole Objectively: body temperature - 36, 5o C ,
respiratory rate - 18/min, Ps- 78/min, AP-
28. A 39-year-old female patient complains of 115/70 mm Hg. There is vesicular breathing
dyspnea when walking, palpitation, edemata above the lungs. Cardiac sounds are sonorous,
in the evening. The patient’s height is 164 cm, of regular rhythm. What drug would be most
weight - 104 kg. Objectively: overnutrition. effective to prevent asthma attacks during the
Heart sounds are weak, and tachycardia is critical season for the patient?
present. The menstrual cycle is not broken.
Blood sugar is 5,6 mmol/l, ACTH-response A. Intalum inhalation
tests revealed no alterations. X-ray of the B. Berotec inhalation
Turkish saddle revealed no pathology. What C. Atrovent inhalation
disease is it? D. Suprastin administration
E. Theopecum administration
A. Alimentary obesity
B. Climax 33. A study of the structure of death causes
C. Pituitary obesity in the urban population revealed that cardi-
D. Diabetes mellitus ovascular diseases accounted for 55,0% of all
E. Cushing’s syndrome (primary hypercorti- deaths. What statistic value represents these
solism) data?

29. A 26-year-old male patient complains of a A. Extensive index


rash on the upper lip skin, which arose on B. Intensive index
a background of influenza with high-grade C. Index of evidence
fever and is accompanied by pain and burning. D. Index of correlation
The rash has been present for 3 days. Objecti- E. Correspondence index
vely: the skin of the upper lip is edematic and
erythematous, grouped vesicles are filled with 34. A 25-year-old female patient complains of
serous fluid and have a rough surface. What is marked weakness, sleepiness, blackouts, di-
the most likely diagnosis? zziness, taste disorder. The patient has a hi-
story of menorrhagia. Objectively: the pati-
A. Herpetic vesicular dermatitis ent has marked weakness, pale skin, cracks in
B. Eczema the corners of mouth, peeling nails, systolic
C. Contact dermatitis apical murmur. Blood test results: RBC -
D. Dermatitis herpetiformis 3, 4 · 1012 /l, Hb- 70 g/l, color index - 0,75,
E. Erythema multiforme platelets - 140 · 109 /l, WBC- 6, 2 · 109 /l. What is
the most likely diagnosis?
30. A 6-year-old boy complains of paroxysmal
pain that occurs after a mental stress, consumi- A. Chronic posthemorrhagic anemia
ng cold drinks or ice cream. After clinical and B. Acute leukemia
instrumental examination the boy has been di- C. Acute posthemorrhagic anemia
agnosed with hypertensive biliary dyskinesia. D. B12 -deficiency anemia
The drugs of the following groups should be E. Werlhof’s disease
administered in the first place:
35. A 51-year-old female is a weaving factory
A. Antispasmodics and choleretics worker with 15 years of service record. During
B. Choleretics and cholekinetics a regular preventive examination she complai-
C. Sedatives and cholekinetics ned of frequent headaches, poor sleep, tingli-
D. Antioxidants ng in the heart, irritability, rapid fatigabili-
E. Antibiotics ty, hearing impairment. For years, the noise
level has exceeded the maximum allowable
31. A 15-year-old patient consulted a concentration by 10-15 dB. A year ago, the
dermatologist about a painful lump in patient underwent a course of treatment for
the armpit. Objectively: there is a walnut- essential hypertension. Specify the most likely
sized node, lymphadenitis, infiltration of the diagnosis:
surrounding tissues. The patient has been di-
Krok 2 Medicine 2014 5

A. Noise disease
B. Essential hypertension A. Hemolytic anemia
C. Neurasthenia B. Iron-deficiency anemia
D. Asthenic-vegetative syndrome C. Protein-deficiency anemia
E. Arteriosclerotic encephalopathy D. B12 -deficiency anemia
E. Hereditary elliptocytosis
36. A 49-year-old male patient who had been
scheduled for a surgery for gastric cancer 40. A lumbar puncture was performed for a
underwent preoperative infusion therapy. Up newborn suspected of having an intracranial
to 3,0 liters of liquid was introduced into the birth injury. Bloody cerebrospinal fluid was
right cubital vein. The following day, he felt a obtained. What hemorrhage occurred in this
dragging pain in the right shoulder. Objecti- case?
vely: on the inner surface of the shoulder
there is a longitudinal hyperemic zone, edema A. Subarachnoid
of skin, a tender cord. What complication B. Cephalohematoma
occurred in the patient? C. Epidural
D. Supratentorial
A. Acute thrombophlebitis E. Subtentorial
B. Venepuncture and edema of paravenous
tissue 41. A 36-year-old female patient complains
C. Paravenous tissue necrosis of intense pain in the knee joints and neck.
D. Acute lymphangitis In the morning she experiences pain in the
E. Paravenous tissue phlegmon interscapular region and leg joints; pain subsi-
des after warm-up gymnastics. The patient is
37. Some of the population of a city distri- overnourished, there is a clicking sound in the
ct have uneven teeth color. The individuals knees when squatting, the knees are somewhat
have white spots, transverse brown stripes on disfigured, painful on palpation. Blood test
the incisors. Occurrence of these symptoms is results: ESR- 18 mm/h, WBC- 8, 0 · 109 /l. Radi-
associated with the quality of drinking water ography reveals subchondral sclerosis in the
from a deep well. Which of the following left knee. What is the basis of this pathology?
components of water can be the cause of the
disease? A. Degenerative processes in cartilage
B. Autoimmune process in the synovium
A. F C. Deposition of urates (tophi) in the articular
B. Ca tissues
C. Mg D. Beta-haemolytic streptococcus
D. J E. Hemarthrosis
E. Fe
42. A 35-year-old female reports heart pain
38. A 22-year-old female patient has been (aching and drilling) occurring mainly in the
delivered by an ambulance team to a surgi- morning in autumn and spring and irradi-
cal clinic with symptoms of acute intestinal ating to the neck, back and abdomen; rapid
obstruction. It is known from the past hi- heartbeat; low vitality. Occurrence of this
story that 2 years ago she was operated for condition is not associated with physical acti-
acute destructive appendicitis. For two years, vity. In the evening, the patient’s condition
she has repeatedly complained of bloating improves. Study of somatic and neurological
and abdominal pain. Which of the followi- status, and ECG reveal no pathology. What
ng etiological factors has led to the intestinal pathology is most likely to have caused these
obstruction in the patient? clinical presentations?
A. Abdominal adhesions A. Somatization depression
B. Ileal diverticulum B. Resting stenocardia
C. Dolichosigma C. Pseudoneurotic schizophrenia
D. Diet violation D. Neurocirculatory asthenia
E. Helminthiasis E. Hypochondriacal depression
39. Mother of a 10-month-old baby reports 43. An hour before an elective surgery, a 56-
significant pallor, poor appetite, enlarged year-old patient of the surgical department
abdomen in the baby. As a neonate, the child got a dramatic increase in blood pressure,
underwent treatment in the in-patient hospi- tachycardia, hand tremor. The patient is
tal for jaundice and anemia. Objectively: the confused, anxious, depressed, fearful, is pessi-
skin is pale and jaundiced, teeth are absent, mistic about the operation outcome, refuses
abdomen is enlarged, spleen is palpable. the surgery. What tactics should be chosen by
Blood test results: Hb- 90 g/l, RBC- 3, 0·1012 /l, a surgeon?
color index - 0,9, microspherocytosis, reti-
culocytosis up to 20%, serum bilirubin - 37
mmol/l, unconjugated bilirubin - 28 mmol/l.
What type of anemia has occurred in the pati-
ent?
Krok 2 Medicine 2014 6

A. Start the surgery after correction of blood has been a smoker since childhood. Objecti-
pressure vely: to - 37, 4o C , respiratory rate is 26/min,
B. Isolate the patient Ps- 82/min, rhythmic. AP- 130/85 mm Hg.
C. Predict the psychological state of the patient There is limited breathing movement in the
D. Organize monitoring of the patient by right side of chest cavity, as well as percussive
medical personnel and mental health counselor dullness and diminished breath sounds. Radi-
E. Organize monitoring of the patient by his ograph shows a homogeneous opacity of the
family members pulmonary field on the right with the mediasti-
num displacement to the affected side. What
44. A 33-year-old female complains of escalati- is the most likely diagnosis?
ng spastic pain in the abdomen after the
psycho-emotional stress. The patient has A. Central lung cancer
intermittent bowel movements, that is 2-3 B. Pleural effusion
bowel movements after waking up alternate C. Pleuropneumonia
with constipation lasting for 1-2 days. Objecti- D. Pulmonary tuberculosis
vely: body weight is unchanged, there is E. Bronchiectasis
moderate pain on palpation of the sigmoid
colon. Hb- 130 g/l, WBC- 5, 2 · 109 /l, ESR- 9 48. A 33-year-old male patient developed a
mm/h. Proctosigmoidoscopy causes pain due condition that had a stormy clinical course:
to spastic bowel condition, intestinal mucosa chills, fever up to 39o C , vomiting, epigastric
is not changed. In the lumen there is a lot of pain, diarrhea with watery smelly feces. 6
mucus. What is the most likely diagnosis? hours before, he ate a raw egg, fried potatoes
with stewed meat, drank some juice. What
A. Irritable bowel syndrome pathogen is likely to have caused this conditi-
B. Crohn’s disease on?
C. Non-specific ulcerative colitis
D. Acute bowel ischemia A. Salmonella
E. Malabsorption syndrome B. Colibacillus
C. Campylobacter
45. An infant is 2 days old. He was born D. Shigella
full-term with signs of intrauterine infection, E. Vibrio cholerae
and therefore receives antibiotics. Neonates
should be given antibiotics at longer intervals 49. Chief physician of a polyclinic encharged
and lower doses compared to older children a district doctor with a task to determine the
and adults because: pathological prevalence of disease N in his
district. What document allows to estimate
A. Neonates have lower glomerular filtration the disease prevalence in the population of a
B. Neonates have lower concentration of medical district?
protein and albumin in blood
C. Neonates have a reduced activity of A. Prophylactic examinations register
glucuronyl transferase B. Statistic coupons (+)
D. Neonates have a decreased blood pH C. Statistic coupons (-)
E. Neonates have higher hematocrit D. Statistic coupons (+) and (-)
E. Vouchers for medical appointments
46. After a holiday in the Crimea, a 36-year-
old female patient presents with severe pain 50. A 76-year-old male consulted a therapi-
in the elbow joints, dyspnea and weakness. st about slow discharge of urine with a small
The body temperature is of 37, 6o C , the skin jet. The patient reported no cardiac problems.
is pale, there is erythema of cheeks and nose, Examination revealed atrial fibrillation with
lower lip ulceration. Visual inspection reveals a heart rate of 72/min and without pulse defi-
no changes in the joints, the right elbow cit. There are no signs of heart failure. ECG
movement is limited. There is murmur and confirms the presence of atrial fibrillation.
pleural friction in the lungs below the ri- From history we know that the arrhythmia
ght angle of the scapula. Cardiac sounds are was detected three years ago. What tactics for
muffled, there is tachycardia, gallop rhythm, the treatment of atrial fibrillation in the pati-
Ps- 114/min. AP- 100/60. What is the most li- ent should be chosen?
kely diagnosis?
A. Does not require treatment
A. SLE B. Digoxin
B. Rheumatic heart disease C. Verapamil
C. Rheumatoid arthritis D. Obzidan
D. Infectious allergic myocarditis E. Ajmaline
E. Dry pleurisy
51. A 53-year-old male has been admitted to
47. A 63-year-old male patient complains of a hospital for an attack of renal colic whi-
cough with expectoration of mucous blood- ch has repeatedly occurred throughout the
streaked sputum, asthma, low-grade fever, year. Objectively: in the region of auricles
general weakness. These presentations have and the right elbow some nodules can be
been observed for 3 months. The patient seen that are covered with thin shiny skin.
Krok 2 Medicine 2014 7

Ps- 88/min, AP- 170/100 mm Hg. There is bi- since he was 16, abuses alcohol, has a history
lateral costovertebral angle tenderness (posi- of CHD. The left lower extremity is colder
tive Pasternatsky’s symptom). The patient than the right one, the skin of extremities is
has been scheduled for examination. What dry, pedal pulse cannot be palpated, femoral
laboratory value would be most helpful for pulse is preserved. What is the most likely di-
making a diagnosis? agnosis?

A. Uric acid A. Obliterating endarteritis


B. Rheumatoid factor B. Diabetic angiopathy
C. ESR C. Leriche syndrome
D. Urine sediment D. Raynaud’s disease
E. Lactic acid E. Deep thrombophlebitis

52. A 58-year-old female patient complai- 56. A 38-year-old male complains of tonic
ns of spontaneous bruises, weakness, bleedi- tension of the masticatory muscles, so that
ng gums, dizziness. Objectively: the mucous he cannot open his mouth. 12 days before, he
membranes and skin are pale with numerous was bitten by an unknown dog. Objectively:
hemorrhages of various time of origin. Lymph there is pronounced tension and twitching of
nodes are not enlarged. Ps- 100/min, AP- the masticatory muscles. What is the most li-
110/70 mm Hg. There are no changes of kely diagnosis?
internal organs. Blood test results: RBC -
A. Tetanus
3,0·1012 /l, Нb - 92 g/l, colour index - 0,9, B. Rabies
anisocytosis, poikilocytosis, WBC - 10·109 /l, C. Hysteria
eosinophils - 2%, stab neutrophils - 12%, D. Trigeminal neuralgia
segmented neutrophils - 68%, lymphocytes E. Apyretic tetanus
- 11%, monocytes - 7%, ESR - 12 mm/h. What
laboratory test is to be determined next for 57. A 72-year-old male had had a moderate
making a diagnosis? headache. Two days later, he developed the
progressing speech disorders and weakness in
A. Platelets the right extremities. The patient has a history
B. Reticulocytes of myocardial infarction, arrhythmia. Study
C. Clotting time of the neurologic status revealed elements of
D. Osmotic resistance of erythrocytes motor aphasia, central paresis of the VII and
E. Fibrinogen XII cranial nerves on the right, central hemi-
paresis on the same side and hyperaesthesia.
53. A 48-year-old male in-patient undergoes What is the most likely diagnosis?
treatment for essential hypertension of II-
B stage. It is known from history that he A. Ischemic stroke
works in a design engineering office. His job B. Hemorrhagic stroke
involves neuro-emotional stress. Which of C. Transient ischemic attack
these foodstuffs do not stimulate the central D. Epidural hematoma
nervous system and can be recommended for E. Brain tumor
the patient?
58. A 28-year-old female patient has been
A. Whole milk admitted to the gynecology department for
B. Meat broths abdominal pain, spotting before and after
C. Mushroom broths menstruation for 5 days. The disease is associ-
D. Vegetable broths ated with the abortion which she had 2 years
E. Carbonated beverages ago. Anti-inflammatory treatment had no
effect. Bimanual examination findings: the
54. In an urban settlement situated on the ri- uterus is enlarged, tight, painful, smooth.
verbank, an outbreak of hepatitis A was regi- Hysteroscopy reveals dark red holes in the
stered. The disease might have water origin. fundus with dark blood coming out of them.
This assumption can be confirmed by growth What diagnosis can be made on the grounds
of the following values of water quality: of these clinical presentations?
A. Number of coli-phages A. Inner endometriosis
B. Escherichia coli index B. Polymenorrhea
C. Oxidability C. Hypermenorrhea
D. Presence of benign leptospirosis pathogen D. Submucous fibromatous node
E. Index of fecal coli-forms E. Dysfunctional uterine bleeding
55. A 48-year-old male patient complains of 59. 2 weeks after having quinsy, a 26-
pain in the lower extremities, especially when year-old male patient got facial edemata,
walking, intermittent claudication, numbness moderate pain in the sacrum. Objectively:
in the fingers, cold extremities, inability to body temperature is 37, 5o C , AP- 100/80 mm
walk more than 100 meters. Sleeps with his Hg. Urinalysis results: RBC- up to 100 fresh
leg lowered. The patient has been a smoker cells in per HPF, protein - 2,2 g/l, hyaline cyli-
Krok 2 Medicine 2014 8

nders - up to 10 per HPF, relative density - the heart’s need for oxygen without aggravati-
1002. What is the most likely diagnosis? ng the disease?
A. Acute glomerulonephritis A. Isosorbide dinitrate
B. Nephroma B. Corinfar
C. Acute pyelonephritis C. Atenolol
D. Urolithiasis D. Streptokinase
E. Chronic glomerulonephritis E. Aminophylline
60. Examination of a full-term 6-day-old infant 64. A 5-year-old girl has had thirst, polyuria,
revealed that different areas of skin had increased appetite for two months. At the
erythemas, flaccid bubbles, eroded surface, same time, there is a 3 kg decrease in body
cracks, peeling of the epidermis looking like weight. During the last week, these presentati-
being scalded with boiling water. There was ons got accompanied by nocturnal enuresis.
positive Nikolsky’s symptom. General condi- Examination revealed hyperglycemia at the
tion of the child was serious. The child was rate of 14 mmol/l. The child has been di-
restless, hypersensitive, febrile. What is the agnosed with type I diabetes. What is the most
most likely diagnosis in this case? likely genesis of this disease?
A. Ritter’s exfoliative dermatitis A. Autoimmune
B. Neonatal phlegmon B. Viral
C. Finger’s pseudofurunculosis C. Bacterial
D. Neonatal pemphigus D. Neurogenic
E. Epidermolysis E. Viral and bacterial
61. A 39-year-old female patient complains of 65. An 8-year-old child with a 3-year history
rapid fatigability, drowsiness, dry skin, hair of diabetes was hospitalized in hyperglycemic
loss, swelling of the face. A month ago, she coma. Specify the initial dose of insulin to be
underwent a surgery for thyrotoxicosis. The administered:
patient has the following gland dysfunction:
A. 0,1-0,2 U/kg of body weight per hour
A. Thyroid (hypothyroidism), due to B. 0,05 U/kg of body weight per hour
inadequate operative technique C. 0,2-0,3 U/kg of body weight per hour
B. Pituitary, due to a tumor D. 0,3-0,4 U/kg of body weight per hour
C. Adrenal E. 0,4-0,5 U/kg of body weight per hour
D. Parathyroid, due to the gland removal
during surgery 66. A 12-year-old girl undergoes regular
E. Ovarian, due to a tumor gastroenterological check-ups for duodenal
ulcer, biliary dyskinesia. What is the
62. A 27-year-old patient has a severe recommended frequency of anti-relapse
headache, nausea and vomiting. Objecti- treatment?
vely: body temperature is 38, 9o C , there is
a haemorrhagic stellate rash on the legs. A. Twice a year
The patient takes meningeal pose in bed. B. Every two months
Meningeal symptoms are strongly positive. C. Every 3 months
Deep reflexes are brisk, uniform. Pathologi- D. Once a year
cal reflexes are absent. It has been suspected E. Three times a year
that the patient has epidemic cerebrospinal
meningitis. Which of additional tests should 67. On the 2nd day of disease a 27-year-old
be performed in the first place to verify the patient complains of unbearable headache,
diagnosis? repeated vomiting. Objectively: the patient
is in a grave condition. He is conscious but
A. Lumbar puncture adynamic. Lies in a forced position with his
B. Echoencephalography head thrown back. There is no skin rash.
C. Rheoencephalography Nuchal muscles are evidently rigid, there are
D. Electroencephalography Kernig’s and Brudzinski’s signs. to - 39, 5o C ,
E. Survey craniogram Ps- 120/min, AP- 130/80 mm Hg. The leading
syndrome of this disease is caused by:
63. 3 hours before, a 68-year-old male pati-
ent got a searing chest pain radiating to the A. Liquor hypertension
neck and left forearm, escalating dyspnea. Ni- B. Liquor hypotension
troglycerin failed to relieve pain but somewhat C. Affection of the cranial nerve nuclei
reduced dyspnea. Objectively: there is cri- D. Haemorrhages in the adrenal glands
mson cyanosis of face. Respiratory rate is E. Hyperthermy
28/min. The patient has vesicular breathing
with isolated sibilant rales. Heart sounds are 68. Two years ago, a 46-year-old patient was
muffled, with a gallop rhythm. Ps- 100/min, diagnosed with stage I silicosis. Currently
AP- 100/65 mm Hg. ECG shows negative T- the patient complains of escalating dyspnea,
wave in V 2 − V 6 leads. What drug can reduce pain in the infrascapular regions. Radiograph
Krok 2 Medicine 2014 9

shows a diffuse enhancement and distorti-


on of lung markings, as well as multiple A. Rheumatoid arthritis
nodular shadows 2-4 mm in diameter. There is B. Osteoarthritis
interlobar pleural density on the right. Dense C. Gout
shadows are found in the hilar regions. Specify D. Pseudogout
the form of radiographic pulmonary fibrosis in E. Multiple myeloma
this case:
73. A 30-year-old female patient has been
A. Nodular delivered to a hospital for sudden dyspnea
B. Interstitial progessing to asthma, sensation of having a
C. Interstitial nodular "lump in the throat", hand tremor, fear of
D. Nodal death. The attack has developed for the first
E. Tumor-like time and is associated with a strong emotion.
There is no previous history. Objectvely: respi-
69. A 63-year-old female complains of general ratory rate - 28/min, Ps- 104/min, rhythmic,
weakness, a feeling of heaviness, compressi- AP- 150/85 mm Hg. The patient has rapid
on in the epigastrium, postprandial fullness, superficial vesicular breathing with extended
nausea, belching after meals. These symptoms expiration. Percussion findings: heart borders
have been observed for about 15 years. are not changed. Cardiac sounds are loud,
Objectively: body temperature is 36, 4o C , rhythmic. What is the most likely diagnosis?
respiratory rate - 20/min, Ps - 88/min, blood
pressure - 115/75 mm Hg. Skin and mucous A. Neurocirculatory asthenia
membranes are pale. Blood test results: RBC - B. Bronchial asthma
2,0·1012 /l, Hb - 100 g/l. Tests revealed parietal- C. Hypertensive crisis
cell antibodies. What is the most likely reason D. Cardiac asthma
for the development of anemia in this patient? E. Thyrotoxic crisis

A. Production of antibodies to intrinsic factor 74. A 42-year-old male patient wth essential
B. Disruption of hemoglobin synthesis hypertension presents with headache, palpi-
C. Disruption of erythropoietin synthesis tations, unexplained fear. Objectively: Ps-
D. Impaired iron absorption 100/min, AP- 200/100 mm Hg, the left border
E. Increased loss of iron of cardiac dullness is displaced by 1,5 cm to
the left, vesicular breathing is present. ECG
70. During dressing of a poorly-granulating shows sinus tachycardia, signs of left ventri-
wound Pseudomonas aeruginosa infection was cular hypertrophy. What drug should be admi-
revealed. What medication would be optimal nistered as an emergency?
for the wound d-bridement?
A. Obzidan
A. Boric acid solution B. Dibazol
B. Biogenic stimulators C. Reserpine
C. Sulfonamides D. Magnesium sulfate
D. Salicylic acid E. Furosemide
E. Antibiotics
75. A 37-year-old male patient has a histrory
71. A 39-year-old male patient complains of of diabetes of moderate severity. On the left
moderate pain and weakness in the shoulder, side of face the patient has a carbuncle. What
back and pelvic girdle muscles, that has been severe complication might have occurred in
progressing for the last 3 weeks; great di- the patient?
fficulty in getting out of bed, going up and
down the stairs, shaving. Dermatomyositis has A. Cavernous sinus thrombosis
been suspected. Blood test results: Hb- 114 g/l, B. Lymphangitis
C. Endarteritis
WBC- 10, 8 · 109 /l, eosinophils - 9%, ESR -22 D. Thrombophlebitis
mm/h, C-reactive protein - (++). The alterati- E. Thromboembolism
on in the following laboratory value wil be of
decisive diagnostic significance: 76. A 42-year-old female patient complains of
a dull pain in her left side, low-grade fever,
A. Creatine phosphokinase accelerated painful urination in small porti-
B. Ceruloplasmin ons. These presentations have been observed
C. Sialic acids for three years. For a long time, the pati-
D. dsDNA antibodies ent has had cystitis with frequent exacerbati-
E. Gamma-globulins ons, there is pulmonary tuberculosis in the
72. A 60-year-old female patient complains of past history. Urinalysis results: microscopic
recurrent pain in the proximal interphalangeal hematuria, leukocyturia. What is the most li-
and wrist joints, their periodic swelling and kely provisional diagnosis?
reddening that have been observed for 4 years.
X-ray picture represents changes in form of
osteoporosis, joint space narrowing and single
usuras. What is the most likely diagnosis?
Krok 2 Medicine 2014 10

A. Renal tuberculosis before the onset of the disease the patient had
B. Urolithiasis examined a dead calf. What is the most likely
C. Chronic pyelonephritis diagnosis?
D. Renal tumor
E. Chronic cystitis A. Cutaneous anthrax
B. Bubonic plague
77. A woman at 30 weeks pregnant has had C. Carbuncle
an attack of eclampsia at home. On admissi- D. Lymphocutaneous tularemia
on to the maternity ward AP- 150/100 mm E. Erysipelas
Hg. Predicted fetal weight is 1500 g. There
is face and shin pastosity. Urine potein is 81. An employee of a petrol station with 15
0, 66o /oo . Parturient canal is not ready for deli- years of service record having contact wi-
very. An intensive complex therapy has been th ethylated gasoline presents with memory
started. What is the correct tactics of this case impairment, bradycardia, sensation of havi-
management? ng a hair in the mouth, skin paresthesia. In
this case, one can assume intoxication with the
A. Delivery by cesarean section following substance:
B. Continue therapy and prolong pregnancy
for 1-2 weeks A. Tetraethyl lead
C. Continue therapy and prolong pregnancy B. Lead chloride
for 3-4 weeks C. Organophosphates
D. Labor induction by intravenous oxytocin or D. Benzene
prostaglandins E. Nitrobenzene
E. Treat preeclampsia and achieve the delivery
by way of conservative management 82. A 38-year-old female suddenly developed
acute inflammatory rash in form of roseolas,
78. Examination of a dead man who died papules, vesicles that are scattered on the skin
from hanging revealed that cadaver spots di- of trunk in irregular and predominantly focal
sappeared when pressed upon and restored manner. The rash appeared a few hours after
after 50 seconds, rigor mortis was moderately visiting a restaurant. The patient complains of
expressed only in the masticatory muscles itching skin. What is the most likely diagnosis?
and the muscles of neck and fingers. Body
temperature was 31o C . Specify the time of A. Toxicodermatosis
death: B. Atopic dermatitis
C. Contact dermatitis
A. 6-7 hours D. Eczema
B. 1-2 hours E. -
C. 16-24 hours
D. 8-10 hours 83. The institutions which take part in medi-
E. 10-18 hours cal examinations can be prevention and
treatment facilities, medical board of Mini-
79. A 65-year-old male patient complains of stry of Defense, medical board of Ministry of
dyspnea that is getting worse with exerti- Home Affairs, medico-social expert commi-
on, morning cough with expectoration of ssions, forensic medical boards etc. What insti-
mucous sputum. For about 15 years, he has tutions are responsible for temporary disabili-
been subject to regular medical check-up ty examination?
for chronic bronchitis. The patient takes
berodual (16 inhaled doses per day). Objecti- A. Prevention and treatment facilities
vely: body temperature is 36, 8o C , RR- 24/min, B. Sanitary-and-prophylactic institutions
Ps- 110/min, AP- 145/90 mm Hg. Auscultati- C. Medico-social expert commissions
on reveals a lot of dry rales above the lungs. D. Medical boards of the Ministry of Defense
FEV1- 65%. What is the optimal tactics of E. Medical boards of the Ministry of Home
further management of the patient? Affairs

A. To administer inhalation corticosteroids 84. After lifting a load, a 36-year-old male


B. To administer antibiotics patient has experienced a severe pain in the
C. To administer theophylline lumbar region, which spread to the right
D. To increase the daily dose of berodual leg and was getting worse when he moved
E. To include short-acting β2 -agonists in the his foot or coughed. Objectively: the long
therapy back muscles on the right are strained. Achi-
lles jerk is reduced on the right. There is a
80. A 49-year-old countryman got an itching pronounced tenderness of paravertebral poi-
papule on the dorsum of his right hand. In nts in the lumbar region. The straight leg raise
the centre there is a vesicle with serosangi- (Lasegue’s sign) is positive on the right. What
nous exudate. Within the next 2 days the additional tests should be performed in the fi-
patient developed a painless edema of hand rst place?
and forearm. On the 4th day the temperature
rose to 38, 5o C , in the right axillary region a
large painful lymph node was found. One day
Krok 2 Medicine 2014 11

A. Radiography of the spinal column A. Central-unit


B. Computed tomography B. Centralized
C. Magnetic resonance tomography C. Decentralized
D. Electromyography D. Free
E. Lumbar puncture E. Combined
85. A 17-year-old male patient consulted a 89. A 43-year-old female patient complai-
therapist about malaise, chills, runny nose, ns of dyspnea, swelling of legs, abdomen
aching muscles and joints, nausea and di- enlargement, pricking heart pain. She has a
arrhea. The patient asks to prescribe him history of tuberculous bronchadenitis, qui-
a lot of painkillers and sedatives (tramadol nsies. The patient’s condition deteriorated 6
or solpadein that help the best, and di- months ago. Objectively: cyanosis, bulging
azepam). Pharyngeal mucosa is pale pink, neck veins, vesicular breathing. Heart borders
clean. Auscultation reveals vesicular breathi- are not displaced. Heart sounds are muffled,
ng. Tachycardia is present. The pupils are di- Ps- 106/min, liver is +4 cm, ascites is present.
lated, there is sluggish response to light. There Low voltage on the ECG has been revealed.
are injection marks on the forearm skin. Duri- Radiograph shows a thin layer of calcium
ng examination, the patient’s manner is vulgar, deposits along the left contour of heart. What
irritable, rude and untruthful. Make a di- treatment should be recommended to the pati-
agnosis: ent?

A. Opioid addiction A. Treatment by a cardiac surgeon


B. Painkillers addiction B. Digitalis preparations
C. Sedative drug addiction C. Anti-TB drugs
D. Acute respiratory disease D. Diuretics
E. Food-born toxic infection E. Vasodilators, nitrates

86. A 4-year-old boy had untimely vacci- 90. A 26-year-old female patient has an 11-
nation. He complains of painful swallowing, year history of rheumatism. Four years ago
headache, inertness, fever. Objectively: the she suffered 2 rheumatic attacks. Over the last
child is pale, has enlarged anterior cervical 6 months there have been paroxysms of atri-
lymph nodes, swollen tonsils with cyanotic al fibrillation every 2-3 months. What option
hyperemia, tonsils are covered with gray-white of antiarrhythmic therapy or tactics should be
pellicles which cannot be easily removed. proposed?
When the pellicles are forcibly removed, the
tonsils bleed. What is the most likely di- A. Prophylactic administration of cordarone
agnosis? B. Immediate hospitalization
C. Defibrillation
A. Oropharyngeal diphtheria D. Lidocaine administration
B. Lacunar tonsillitis E. Heparin administration
C. Pseudomembranous tonsillitis
D. Infectious mononucleosis 91. Blood typing resulted in positive
E. Follicular tonsillitis isohemagglutination reaction with standard
sera of A(II) and B(III) groups and negative
87. Public nurseries are designed as a single or reaction with sera of 0(I) and AB(IV) groups.
several one- or two-storey buildings linked What is this result indicative of?
by covered walkways. Planning the structure
of preschool instiutions is based upon the A. Faulty standard sera
following principle: B. The first blood group
C. The second blood group
A. Principle of group isolation D. The third blood group
B. Principle of autonomy E. The fourth blood group
C. Principle of age distribution
D. Principle of sex distribution 92. A 9-year-old girl has been admitted to
E. Principle of age-sex distribution a hospital for an elevated body temperature
(39, 8o C ), painful dry cough, abdominal pain
88. A city somatic hospital with 300 beds has a on the right. Examination reveals dullness on
main building which houses the therapeutic percussion on the right, diminished breath
and surgical departments. Several separate sounds, crepitus. What study is required to
buildings house the maternity, pediatric and make a diagnosis?
radiologic departments that are connected to
the main building by underground walkways A. Radiography of the chest cavity
and above-ground covered skybridges. Speci- B. USI of the chest cavity
fy the building system of the hospital: C. Pleural puncture
D. Bronchoscopy
E. Bronhography
93. A newborn has purulent discharges from
the umbilical wound, the skin around the navel
Krok 2 Medicine 2014 12

is swollen. The baby’s skin is pale, with a is 0,586 mmol/l, plasma potassium - 7,2 mmol/l.
yellow-gray tint, generalized hemorrhagic rash What treatment is necessary for this patient?
is present. What is the most likely diagnosis?
A. Hemodialysis
A. Sepsis B. Large doses of verospiron
B. Hemorrhagic disease of the newborn C. Plasma volume expanders
C. Hemolytic disease of the newborn D. Glucocorticosteroids
D. Thrombocytopathy E. Heparin
E. Omphalitis
98. A 10-year-old child with a history of
94. A pregnant 26-year-old woman was admi- nonrheumatic carditis has periodic attacks
tted to a hospital for abdominal pain and manifested by heart pain, dyspnea, pallor, hi-
bleeding from the genital tract. Bimanual gh blood pressure, a dramatic increase in heart
examination revealed that uterus was the si- rate up to 180/min. What drug would be most
ze of 9 weeks of pregnancy, the cervical canal effective to treat this patient?
let a finger through. Fetal tissues could be
palpated in the orifice. There was moderate A. Obsidan
vaginal bleeding. What is the tactics of choice? B. Procainamide
C. Lidocaine
A. Instrumental extraction of fetal tissue D. Verapamil
B. Surveillance E. Ajmaline
C. Administration of hormones
D. Hemostatic and antianemic therapy 99. A 45-year-old male patient with acute
E. Therapy for the maintenance of pregnancy abscess of the left lung has suddenly
developed acute chest pain and dyspnea whi-
95. On the second day of the disease a 22- le coughing, tachycardia has increased. The
year-old male patient complains of high-grade control Ro-gram shows left lung collapse, the
fever, headache in the region of forehead and air in the left pleural cavity and a horizontal
superciliary arches, and during eye movement; fluid level. What is the mechanism of this
aching muscles and joints. Objectively: body complication?
temperature is 39o C . Face is hyperemic,
sclerae are injected. The mucous membrane A. Abscess burst into the pleural cavity
of the soft palate and posterior pharyngeal B. Bullae rupture of the left lung
wall is bright hyperemic and has petechial C. Inflammation spread to the visceral pleura
hemorrhages. What changes in the hemogram D. Atelectasis of the left lung
are typical for this disease? E. Acute cardiovascular insufficiency

A. Leukopenia 100. A 24-year-old male patient had been


B. Leukocytosis diagnosed with class III diffuse toxic goi-
C. Neutrocytosis ter. There is moderate hyperthyroidism. A
D. Anemia surgery was suggested, and the patient agreed
E. Accelerated ESR to it. What preoperative measures should be
taken for prevention of thyrotoxic crisis in the
96. A 44-year-old male patient complains of postoperative period?
severe non-localized abdominal pain, pain in
the right shoulder girdle, repeated vomiting, A. Administration of antithyroid drugs
red urine. The onset of the disease is associ- B. Minimally invasive surgical techniques
ated with alcohol consumption. The face is C. Bed rest
hyperemic. AP- 70/40 mm Hg. Abdominal D. Detoxification therapy
radiography reveals no pathological shadows. E. Administration of corticosteroids
Hemodiastase is 54 mg/h/l. Prothrombin is
46%. What is the provisional diagnosis? 101. A 26-year-old male patient complains of
pain in the right knee, which is getting worse in
A. Acute pancreatitis the morning. Two weeks before, he consulted
B. Acute myocardial infarction an urologist about prostatitis. Objectively:
C. Perforated gastric ulcer conjunctivitis is present. There is also peri-
D. Thrombosis of mesenteric vessels articular edema of the knee joint, redness of
E. Aneurysm of the abdominal aorta the overlying skin. Rheumatoid factor was not
detected. Until further diagnosis is specified,
97. A 41-year-old male patient was delivered it would be reasonable to start treatment with
to a hospital unconscious. During the previ- the following antibiotic:
ous 7 days he had been taking large doses
of biseptolum for a cold. The night before, A. Tetracyclines
he began complaining of dyspnea, especially B. Cephalosporins
when lying down, swollen legs, 2-day urinary C. Penicillins
retention. In the morning he had seizures and D. Aminoglycosides
lost consciousness. Objctively: noisy breathi- E. Lincosamides
ng at the rate of 30/min, edematous legs and
lumbar region, Ps- 50/min. Plasma creatinine 102. A 66-year-old female patient has been
Krok 2 Medicine 2014 13

admitted to a hospital for massive gross A. Furosemide


hematuria with release of shapeless blood B. Mannitol
clots, frequent painful urination. The pati- C. Hydrochlorthiazide
ent also reports a moderate weight loss wi- D. Spironolactone
thin 3-4 months. Gross hematuria that was E. Moduretic
not accompanied by pain and dysuria first
occurred three months ago for no apparent 106. For 3 days, a 28-year-old emale patient
reason, and after a few days the bleeding had had the body temperature increase up to
subsided independently. What is the most li- 38o C , weakness, poor appetite, nausea, a si-
kely diagnosis? ngle vomiting. On the 4th day the temperature
was normal, the condition improved, but the
A. Bladder tumor jaundice developed. Objectively: moderate
B. Urolithiasis ictericity of skin, +3 cm enlarged liver of
C. Renal tumor elastic consistency. Ortner’s, Kehr’s and
D. Chronic cystitis Voznesensky’s symptoms are negative. What
E. Acute cystitis test will verify the diagnosis?

103. A 36-year-old female pesented to a A. IgM Anti-HAV detection


gynecological hospital with a significant B. Complete blood count
bleeding from the genital tract and a 1-month C. Ultrasound of the abdomen
delay of menstruation. Bimanual examinati- D. Total bilirubin
on revealed soft barrel-shaped cervix. Uterus E. AST activity
was of normal size, somewhat softened.
Appendages were unremarkable on both si- 107. An 11-year-old girl has been immunized
des. Speculum examination revealed that the according to her age and in compliance with
cervix was cyanotic, enlarged, with the the the calendar dates. What vaccinations should
external orifice disclosed up to 0,5 cm. Uri- the children receive at this age?
ne hCG test was positive. What is the most A. Diphtheria and tetanus
likely diagnosis? B. TB
A. Cervical pregnancy C. Polio
B. Uterogestation D. Hepatitis B
C. Abortion in progress E. Pertussis
D. Threatened miscarriage 108. A 40-year-old male patient has had heavi-
E. Ectopic pregnancy ness in the epigastric region for the last 6
104. A 47-year-old female patient has an 8- months. He has not undergone any exami-
year history of ulcerative colitis, has been nations. The night before, he abused vodka. In
treated with glucocorticoids. She complai- the morning there was vomiting, and 30 mi-
ns of cramping pain in the umbilical regi- nutes after physical activity the patient experi-
on and left iliac region which has signifi- enced dizziness and profuse hematemesis.
cantly increased during the past 2 weeks, di- What pathology should be suspected in the
arrhea with mucus and blood 4-6 times a day, first place?
elevated body temperature up to 38 − 39o C , A. Mallory-Weis’s syndrome
headache and pain in the knee joints. Objecti- B. Menetrier’s disease
vely: the patient is in moderate condition, C. Gastric ulcer
Ps- 108/min, AP- 90/60 mm Hg; heart and D. Perforated ulcer
lungs are unremarkable; the tongue is moi- E. Zollinger-Ellison syndrome
st; abdominal muscle tone is significantly
decreased; peristaltic noises are absent. What 109. A 55-year-old patient complains of severe
complication developed in the patient? itching, burning and pain in the eyes, skin
redness in the outer corners of the palpebral
A. Toxic dilatation of the colon fissure. Objectively: skin around the outer
B. Perforation of the colon corners of the palpebral fissure is macerated,
C. Enterorrhagia eczematous, there are single moist cracks.
D. Stricture of the colon Palpebral conjunctiva is hyperemic, quaggy.
E. Colon carcinoma There are minor discharges in form of stringi-
105. A 10-year-old child has been admitted ng mucus. What is the most likely diagnosis?
to a hospital with a closed craniocerebral A. Chronic conjunctivitis
injury with a suspected cerebral edema. The B. Acute conjunctivitis
patient is in grave condition, unconscious. C. Sty
The dyspnea, tachycardia, hypertension are D. Blepharitis
present. Muscle tone is increased, there is E. Atopic eyelid dermatitis
nystagmus, pupillary and oculomotor reacti-
ons are impaired. The mandatory component 110. A 3-month-old infant has occipital
of intensive care is dehydration. What diuretic alopecia, restless sleep, excessive sweating.
is adequate in this case? What disease can you think of?
Krok 2 Medicine 2014 14

A. Rickets A. Apply a bandage, give an injection of


B. Spasmophilic diathesis vasodilators
C. Anemia B. Administer heart medications
D. Phosphate diabetes C. Put the feet into hot water
E. Chondrodystrophy D. Rub the feet with snow
E. Apply an alcohol compress
111. A 50-year-old male in a grave conditi-
on has been admitted to the intensive care 116. A 21-year-old male patient got a deep cut
unit. It is known from life history that the pati- wound in his right thigh. In the emergency
ent works in agriculture, and 3 hours ago was room a surgeon on duty performed pri-
engaged into insecticide treatment of crops mary debridement of the wound and pri-
for control of colorado potato beetle. Conditi- mary wound closure with a suture. After 4
on on admission: acrocyanosis, bronchorrhea, days, there appeared pain, redness, edema,
tachypnea, AP- 100/60 mm Hg, Ps- 44/min. purulent discharge from the wound gap, body
What method of efferent therapy would be temperature rose up to 39o C . What kind of
most appropriate at this stage? wound complication can you think of and what
actions should be taken?
A. Hemosorbtion
B. Hemodialysis A. Wound abscess, remove the sutures and
C. Plasmapheresis drain the wound
D. Lymphosorption B. Infiltration, apply a hot compress
E. Plasma dialysis C. Erysipelas, prescribe antibiotics
D. Tetanus, active-passive immunization agai-
112. An 80-year-old patient complains of nst tetanus
constantly urinating small amounts, a feeling E. Lymphangitis, apply a hot compress
of pressure in the lower abdomen. Objectively:
there is a suprapubic spherical bulging with 117. A general practitioner visited a 2-year-old
percussion dullness over it. What syndrome child and diagnosed him with measles. The
occurred in the patient? child attends a nursery, has a 5-year-old si-
ster. What document must be filled in for the
A. Paradoxical ischuria effective antiepidemic measures in the given
B. Urinary incontinence health locality?
C. Dysuria
D. Enuresis A. Emergency notification on infectious di-
E. Pollakisuria sease (form № 058/o)
B. Carer’s leave certificate
113. Carpathian region is characterized by C. Infant’s record (report form № 112/o)
permanently high (over 80%) air humidi- D. House call record (form № 031/o)
ty. In the cold season the population of this E. Sick leave
region feels very cold at moderately low
temperatures. This is due to an increase in 118. An employee has been diseased for 4
the heat transfer by: months, further treatment is necessary, the
patient is incapacitated. Who is authorized to
A. Convection provide further disability examination of this
B. Emission patient?
C. Evaporation
D. Conduction A. Medical and social expert board
E. Radiation B. Medical consultative board
C. Physician in charge and chief of department
114. An 18-year-old girl complains of breast D. Chief physician of a medical facility
pain and engorgement, headaches, irri- E. Deputy chief responsible for disability
tability, swelling of the lower extremities. examination
These symptoms have been observed since
menarche and occur 3-4 days before the 119. It has been suspected that a newborn has
regular menstruation. Gynecological exami- congenital diaphragmatic hernia (asphyctic
nation revealed no pathology. Make a di- incarceration). What study will allow to confi-
agnosis: rm the diagnosis?
A. Premenstrual syndrome A. Plan radiography of the chest cavity
B. Neurasthenia B. Plan abdominal radiography
C. Renal disease C. Irrigography
D. Mastopathy D. Pneumoirrigoscopy
E. Cardiovascular disorder E. Fibroesophagogastroduodenoscopy
115. A 56-year-old male patient has been 120. A 25-year-old female presented to a
delivered to the emergency department wi- women’s welfare clinic and reported the
th frostbite on both feet. What aid should be inability to get pregnant within 3 years of
rendered to the victim? regular sexual activity. Examination revealed
Krok 2 Medicine 2014 15

increased body weight, male pattern of she hears the voice of her brother who tells
pubic hair growth, excessive pilosis of thi- her to go home. The patient is anxious, suspi-
ghs, dense enlarged ovaries, monophasic basal cious, looks around all the time. Specify the
temperature. What is the most likely di- psychopathological syndrome:
agnosis?
A. Hallucinatory
A. Polycystic ovarian syndrome B. Generalized anxiety disorder
B. Adnexitis C. Paranoiac
C. Adrenogenital syndrome D. Paraphrenic
D. Premenstrual syndrome E. Depressive
E. Gonadal dysgenesis
125. During the first home visit to a full-term
121. A factory’s sectorial doctor selects a boy after his discharge from the maternity
group of persons who often fall ill for thorough hospital a pediatrician revealed a symmetri-
monitoring. At the same time he takes into cal swelling of mammae without skin changes
consideration the number of etiologically over them, swelling of the scrotum. The body
related cases with temporary disability in temperature was of 36, 5o C . The baby was
each of the employees over the last year. An calm, sucked the mother’s breast actively.
employee falls into this group if the number of What condition should you think of?
sickness cases is:
A. Hormonal crisis of the newborn
A. 4 or more B. Neonatal mastitis
B. 1 or more C. Sclerema
C. 2 or more D. Necrotic neonatal phlegmon
D. 3 or more E. Congenital adrenal dysfunction
E. 6 or more
126. A 30-year-old male patient complains
122. A 23-year-old female consulted a of inertness, low-grade fever, bleeding gums,
gynecologist on the 20th day postpartum peri- frequent quinsies, aching bones. Objecti-
od about pain in the left breast, purulent vely: the patient has pale skin and mucous
discharge from the nipple. Objectively: Ps- membranes, sternalgia, +2 cm liver, +5 cm
120/min, t - 39 C . The left breast is painful, painless spleen. Blood test results: RBC-
larger than the right one, hyperemic. In the 2, 7 · 1012 /l, Нb- 80 g/l, WBC- 3 · 109 /l,
upper quadrant there is an infiltrate sized eosinophils - 4%, basophils - 5%, blasts -
10x15 cm with a softening inside. Blood test 4segmented neutrophils - 17%, lymphocytes -
results: ESR- 50 mm/h, WBC- 15, 0 · 109 /l. 29%, myelocytes - 25%, promyelocytes - 12%,
What is the tactics of choice? monocytes - 2%, platelets - 80 · 109 /l, ESR -
57 mm/h. What test should be performed to
A. Refer to the surgical department for operati- verify the diagnosis?
ve treatment
B. Refer to the gynecology department A. Sternal puncture
C. Refer to the postpartum department B. Trephine biopsy
D. Refer to a polyclinic surgeon for conservati- C. Lymph node biopsy
ve treatment D. Lumbar puncture
E. Lance the breast abscess in the women’s E. Chest X-ray
health clinic
127. A 46-year-old male patient complains of
123. A family lives in the town situated within periodic epigastric pain that occurs at night.
the zone of radiation pollution. A 6-year-old Objectively: HR- 70/min, AP- 125/75 mm Hg,
child had been ill with ARVI for 19 days. The tenderness in the epigastric region is present.
child was undergoing outpatient treatment EGD confirms duodenal ulcer of 0,6 cm in
and was nursed by his mother, a cafe worker. diameter. Test for H. Pylori is positive. Whi-
Specify the order of disability examination: ch of the given antisecretory drugs will be
a compulsory element of the treatment regi-
A. Sick leave is granted for the entire period of men?
the child’s illness
B. Sick leave is granted for a total of 14 days, A. Omeprazole
and after that period is over, a carer’s leave B. Famotidine
certificate is issued C. Pirenzepine
C. Sick leave is not granted, only a carer’s leave D. Atropine
certificate is issued E. Maalox
D. Sick leave is granted for 14 days, after this
period is over, no document is issued 128. A 47-year-old male patient has been lately
E. Sick leave is granted for 7 days, after this complaining of compressing chest pain that
period is over, a carer’s leave certificate is issued occurs when he walks a distane of 700-800 m.
Once a week, he drinks 2 liters of beer. Rise in
124. A 49-year-old female patient with schi- arterial pressure has been observed for the last
zophrenia is all the time listening to somethi- 7 years. Objectively: Ps- 74/min, AP- 120/80
ng, insists that "there is a phone in her head"as mm Hg. The bicycle ergometry performed at
Krok 2 Medicine 2014 16

workload of 75 watts shows 2 mm ST -segment A. Botulism


depression in V 4 − V 6 leads. What is the most B. Shigellosis
likely diagnosis? C. Salmonellosis
D. Cholera
A. Exertional stenocardia, II functional class E. Yersiniosis
B. Exertional stenocardia, III functional class
C. Exertional stenocardia, IV functional class 133. A full-term neonate weighing 4500 g was
D. Vegetative-vascular dystonia of hypertensi- born asphyxiated with Apgar score of 4-6 poi-
ve type nts. During the delivery shoulder dystocia
E. Alcoholic cardiomyopathy occurred. Neurologic assessment revealed
non-focal neurologic symptoms, total flaccid
129. A puerperant is 28 years old. It’s the 3rd paresis of the upper extremities since the arm
day post-partum after a second, normal, term was atonic and pronated. Grasping, Babki-
delivery. The body temperature is of 36, 8o C , n’s and Moro’s reflexes were absent. What
Ps- 72/min, AP- 120/80 mm Hg. Mammary segments of spinal cord had been affected?
glands are moderately engorged, the nipples
are clean. Abdomen is soft, painless. The A. V - T hI
fundus is 3 fingers’ breadth below the navel. B. I - II
Moderate bloody lochia are present. What di- C. III - IV
agnosis can be made? D. T hI - T hV
E. T hV I - T hV 
A. Physiological course of the postpartum
period 134. A newborn (mother’s I pregnancy) wei-
B. Subinvolution of uterus ghing 3500 g has been found to have jaundice,
C. Postpartum metroendometritis lethargy, reduced reflexes. Objectively: second
D. Remains of placental tissue after childbirth grade jaundice of skin with saffron tint, li-
E. Lactostasis ver - +2 cm, spleen - +1 cm. Urine and feces
are yellow. Blood count: Hb- 100 g/l, RBC-
130. A 29-year-old unconscious patient has 3, 2 · 1012 /l, WBC- 18, 7 · 109 /l, mother’s blood
been delivered to a hospital. Objectively: skin type - 0(I) Rh(+), baby’s blood type - A(II)
and mucous membranes are pale, cyanotic, Rh(-), bilirubin - 170 mmol/l, indirect fraction.
breath sounds are dramatically diminished ALT, AST rates are normal. What is the most
on the right and cannot be auscultated in the likely disease in the child?
lower parts, at the level of the 6 rib along the
anterior axillary line there is a wound hole wi- A. Hemolytic disease of newborn, AB0-conflict
th moderate bleeding and passage of air duri- B. Perinatal hepatitis
ng inspiration. Radiography reveals a bullet in C. Hemolytic disease of newborn, Rh-conflict
the pleural cavity. What is the medical tactics D. Biliary atresia
of choice? E. Physiologic jaundice
A. Emergency thoracotomy 135. As a result of a road accident a 45-
B. Thoracoscopy with removal of bullet year-old male patient got multiple injuries,
C. Converting a tension pneumothorax into a namely closed fractures of the right humerus
simple (open) pneumothorax and the left antebrachial bones with a di-
D. Thoracostomy splacement of bone fragments, a closed blunt
E. Tight bandage on a wound abdominal injury. The patient was delivered
to the emergency department 30 minutes after
131. A patient had four generalized convulsive the injury. Objectively: the skin is pale. AP-
seizures within a day. Between the seizures the 90/20 mm Hg, there is pain and deformati-
patient did not come to waking consciousness on at the fracture sites. Abdomen is tense,
(was in a coma or stupor). Specify his state: palpation causes severe pain, there is rebound
tenderness (positive Blumberg’s sign). What is
A. Status epilepticus the treatment tactics of choice?
B. Frequent generalized seizures
C. Frequent jacksonian seizures A. Urgent diagostic laparotomy
D. Hysterical attacks B. Infusion therapy to stabilize blood pressure
E. Frequent complex partial seizures C. Fracture immobilization, analgesia
D. Local anesthetic blockade of fractures
132. A 12-year-old boy presents with nausea, E. Additional tests to specify the diagnosis
frequent repeated vomiting that first occurred
after eating canned vegetables. Objectively: 136. A 34-year-old male visited Tajikistan.
the patient has dry mucous membranes, After return, he complains of fever up
muscular hypotonia, anisocoria, mydriasis, to 40o C which occurs every second day
dysphagia and dysarthria. What is the most and is accompanied by chills, sweating.
likely diagnosis? Hepatosplenomegaly is present. Blood test
results: RBC- 3 · 1012 /l, Нb- 80 g/l, WBC- 4 ·
109 /l, eosinophils - 1%, stab neutrophils - 5%,
segmented neutrophils - 60%, lymphocytes -
Krok 2 Medicine 2014 17

24%, monocytes - 10%, ESR - 25 mm/h. What A. Pericardiocentesis and immediate


is the provisional diagnosis? thoracotomy
B. Oxygen inhalation
A. Malaria C. Puncture of the pleural cavity on the left
B. Infectious mononucleosis D. Conservative treatment, infusion of
C. Sepsis adrenomimetics
D. Typhoid fever E. Pleural cavity drainage
E. Leptospirosis
141. A puerperant is 32 years old, it’s her first
137. A 23-year-old male patient consulted childbirth, term precipitate labor, the III peri-
a doctor about pain occurring in the lower od is unremarkable, the uterus is contracted,
third of the thigh with weight bearing acti- tight. Examination of the birth canal revealed
vities and unloaded. The patient denies any a rupture in the left posterior vaginal wall that
injuries to the region. Objectively: the skin was closed with catgut. Two hours later, the
is of normal color, deep palpation reveals patient complained of a feeling of pressure on
pastosity and tenderness, movements of the the anus, pain in the perineum, minor vaginal
knee joint are limited. Radiograph of the di- discharges, edema of the vulva. These clinical
stal femoral metaepiphyseal region shows a presentations are indicative most likely of:
zone of degradation and spicules. In blood:
immature cells are present, there are no signs A. Vaginal hematoma
of inflammation. What is the most likely di- B. Hysterocervicorrhexis
agnosis? C. Hemorrhoids
D. Hysterorrhesis
A. Osteosarcoma E. Hypotonic bleeding
B. Hyperparathyroid dystrophy
C. Chronic osteomyelitis 142. A hospital nutrition unit received a batch
D. Multiple myeloma of beef. Sanitation physician examined the
E. Marble bone disease meat and revealed the presence of 5 bladder
worms per 40 cm2 of meat. Give the hygienic
138. It is planned to organize a rural outpati- assessment of meat:
ent clinic. The patients will be able to visit the
doctors of the following specialities: A. Liable to technical utilization
B. Conditionally admissible
A. Therapeutist, dentist, pediatrician, C. Adulterated
obstetrician-gynecologist D. Good-quality
B. Therapeutist, pediatrician, neurologist E. Poor-quality
C. Pediatrician, obstetrician-gynaecologist,
ophthalmologist 143. A 47-year-old female patient complains of
D. Pediatrician, therapeutist, ophthalmologist having pain on swallowing and difficult passi-
E. Obstetrician-gynaecologist, therapeutist ng of solid food for two months. The pati-
ent has taken to the liquid and semi-liquid
139. A 12-year-old girls has minor functi- food. During the last week the liquid food has
onal and morphological abnormalities: 1,0 D barely passed through. General condition is
myopia, reduced body resistance. The patient satisfactory, the patient is undernourished, the
has no history of chronic diseases. Over the appetite is preserved, there is a fear of eating.
last year, there were 4 cases of respiratory di- What is the provisional diagnosis?
seases. The girl belongs to the following health
group: A. Esophageal carcinoma
B. Esophageal stricture
A. II C. Esophageal foreign body
B. I D. Esophageal varices
C. III E. Esophageal achalasia
D. IV
E. V 144. A 31-year-old female patient complai-
ns of infertility, amenorrhea for 2 years after
140. 4 weeks after a myocardial infarction, the artificial abortion that was complicated by
a 56-year-old male patient developed acute endometritis. Objectively: examination of the
heart pain, pronounced dyspnea. Objectively: external genitalia revals no pathology, there is
the patient’s condition is extremely grave, female pattern of hair distribution. According
there is marked cyanosis of face, swelling and to the functional tests, the patient has biphasic
throbbing of neck veins, peripheral pulse is ovulatory cycle. What form of infertility is the
absent, the carotid artery pulse is rhythmic, case?
130 bpm, AP- 60/20 mm Hg. Auscultation
of heart reveals extremely muffled sounds, A. Uterine
percussion reveals heart border extension in B. Ovarian
both directions. What is the optimal treatment C. Pituitary
tactis for this patient? D. Hypothalamic
E. Immunological
Krok 2 Medicine 2014 18

145. A 49-year-old male patient consulted analyses of the daily urinary excretion of
a doctor about difficult swallowing, voice albumin revealed microalbuminuria (200-
hoarseness, weight loss. These symptoms have 300 mg/day). Glomerular filtration rate
been gradually progressing for the last 3 is 105 ml/min. Blood pressure is within
months. Objectively: the patient is exhausted, normal range. Normalization of the followi-
supraclavicular lymph nodes are enlarged. ng indicator should be the first-priority task
Esophagoscopy revealed no esophageal in the secondary prevention of diabetic
pathology. Which of the following studies is nephropathy:
most appropriate in this case?
A. Glycosylated hemoglobin
A. Computed tomography of chest and medi- B. C-peptide
astinum C. Blood insulin
B. X-ray of lungs D. Fasting glucose
C. Multiplanar imaging of esophagus E. Glycemia 2 hours after a meal
D. Radioisotope investigation of chest
E. Ultrasound investigation of mediastinum 150. 10 days after birth, a newborn developed
a sudden fever up to 38, 1o C . Objectively:
146. A 36-year-old male patient complains of the skin in the region of navel, abdomen
having headache, obstructed nasal breathing, and chest is erythematous; there are multi-
purulent nasal discharge for 2 weeks. A month ple pea-sized blisters with no infiltration at the
before, he had his right maxillary premolar fi- base; single bright red moist erosions with epi-
lled. Radiolography revealed an intense opaci- dermal fragments on the periphery. What is
ty of the right maxillary sinus. Diagnostic your provisional diagnosis?
puncture revealed a large amount of thick
malodorous crumbly pus. What is the most A. Epidemic pemphigus of newborn
likely diagnosis? B. Syphilitic pemphigus
C. Streptococcal impetigo
A. Chronic suppurative odontogenic sinusitis D. Vulgar impetigo
B. Acute purulent sinusitis E. Atopic dermatitis
C. Chronic purulent sinusitis
D. Chronic atrophic sinusitis 151. An emergency physician arrived to provi-
E. Tumor of the maxillary sinus de medical care for a hangman taken out
of the loop by his relatives. The doctor
147. A 5-year-old boy has a history of repeated revealed no pulse in the carotid arteries, lack
pneumonia, frequent acute respiratory viral of consciousness, spontaneous breathing and
diseases. Objectively: exertional dyspnea, mi- corneal reflexes; cadaver spots on the back
nor fatigabilty. There is a systolic murmur and posterior parts of extremities. A person
having its epicenter in the IV intercostal can be declared dead if the following sign is
space on the left. Left relative dullness is present:
found along the midclavicular line. Accordi-
ng to the findings of instrumental methods of A. Cadaver spots
examination (electrocardiography, echocardi- B. Lack of spontaneous breathing
ography), the patient has been diagnosed wi- C. Lack of corneal reflexes
th ventricular septal defect, subcompensation D. Pulselessness
stage. What is the main method of treatment? E. Unconsciousness

A. Operative therapy 152. A 69-year-old male patient has been


B. Phytotherapy hospitalized with hypothermia. Objectively:
C. Does not require treatment the patient is pale, has shallow breathing. AP-
D. Conservative treatment 100/60 mm Hg, Ps- 60/min. Palpation of the
E. Indomethacin abdomen and chest reveals no pathological
signs. The body temperature is of 34, 8o C .
148. A patient with autoimmune thyroi- The patient’s breath smells of alcohol. Give
ditis accompanied by multinodular goiter treatment recommendations:
underwent the right lobe ectomy and subtotal
resection of the left lobe. What drug should A. Warm bath + intravenous administration of
be administered to prevent postoperative warm solutions
hypothyroidism? B. Body warming with hot-water bags
C. Forced diuresis
A. L-thyroxine D. Rubbing with alcohol and snow
B. Merkazolil E. Antishock therapy
C. Iodomarin
D. Lithium drugs 153. A 26-year-old male patient consulted a
E. Insulin doctor abut sore throat, fever up to 38, 2o C .
A week before, the patient had quinsy, di-
149. A 49-year-old female patient has type dn’t follow medical recommendations. On
1 diabetes of moderate severity. The di- examination, the patient had forced position
sease is complicated by retinopathy and of his head, trismus of chewing muscles. Left
polyneuropathy. Besides that, repeated peritonsillar region is markedly hyperemic,
Krok 2 Medicine 2014 19

swollen. What is the provisional diagnosis? upper extremities and decreased pressure
in the lower ones, extension of the left
A. Left-sided peritonsillar abscess heart border, blowing systolic murmur in the
B. Meningitis interscapular region. ECG shows the hori-
C. Phlegmonous tonsillitis zontal axis of heart. Radiography reveals left
D. Pharyngeal diphtheria cardiomegaly, costal usuration. What is the
E. Tonsil tumour most likely diagnosis?
154. A 21-year-old female patient has been A. Aortarctia
hospitalized on an emergency basis because B. Aortic stenosis
of severe dyspnea, pain in the left side of C. Patent ductus arteriosus
chest. Body temperature is 38, 8o C . The condi- D. Ventricular septal defect
tion developed three days ago. Respiratory E. Atrial septal defect
rate is 42/min, auscultation reveals shallow
breathing. There is percussive dullness on 158. During the preventive examinati-
the right starting from the middle of the on a 17-year-old young man reports no
blade, breath sounds cannot be heard. The left health problems. Objectively: the patient is
border of heart is 3 cm displaced outwards. undernourished, asthenic; blood pressure is
Embryocardia is present, HR is 110/min. The 110/70 mm Hg, Ps- 80/min. Heart borders
right hypochondrium is painful on palpation. are within normal range. Auscultation reveals
What urgent therapeutic measures should be three apical heart sounds, murmurs are absent.
taken in this situation? ECG shows no pathological changes, PCG
registers the S3 occurring 0,15 seconds after
A. Emergency puncture of the pleural cavity the S2. How can you interpret these changes?
B. Administration of penicillin antibiotics
C. Injection of Lasix A. Physiologic S3
D. Injection of cardiac glycosides B. Fout-ta-ta-rou (three-component rhythm)
E. Transferring the patient to the thoracic C. Protodiastolic gallop rhythm
surgery department D. Presystolic gallop rhythm
E. Physiologic S4
155. A 77-year-old male patient complains of
inability to urinate, bursting pain above the 159. After having the flu, a 39-year-old
pubis. The patient developed acute condi- male patient with a history of Addison’s
tion 12 hours before. Objectively: full uri- disease developed a condition manifested
nary bladder is palpable above the pubis. by weakness, depression, nausea, vomi-
Rectal prostate is enlarged, dense and elastic, ting, diarrhea, hypoglycemia. AP- 75/50
well-defined, with no nodes. Interlobular mm Hg. Blood test results: low corti-
sulcus is distinct. Ultrasonography results: costerone and cortisol, 13-oxycorticosteroids,
prostate volume is 120 cm3 , it projects into the 17-oxycorticosteroids levels. What condition
bladder cavity, has homogeneous parenchyma. developed in the patient?
Prostate-specific antigen rate is of 5 ng/ml.
What is the most likely disease that caused A. Acute adrenal insufficiency
acute urinary retention? B. Acute gastritis
C. Acute enterocolitis
A. Prostatic hyperplasia D. Collapse
B. Prostate carcinoma E. Diabetes mellitus
C. Sclerosis of the prostate
D. Tuberculosis of the prostate 160. A female patient complains of being
E. Acute prostatitis unable to get pregnant for 5 years. A complete
clinical examination brought the following
156. At the first minute of life a full-term results: hormonal function is not impaired,
infant born with umbilical cord entangled urogenital infection hasn’t been found, on
around his neck has total cyanosis, apnea, hysterosalpingography both tubes were filled
HR- 80/min, hypotonia and areflexia. There with the contrast medium up to the isthmic
are no signs of meconium aspiration. After segment, abdominal contrast was not visuali-
the airway suctioning the newborn did not zed. The patient’s husband is healthy. What
start breathing. What is the next action of the tactics will be most effective?
doctor?
A. In-vitro fertilization
A. ALV with a 100% O2 mask B. Insemination with husband’s sperm
B. Intravenous administration of adrenaline C. ICSI within in-vitro fertilization program
C. Intravenous administration of etamsylate D. Hydrotubation
D. Tracheal intubation and ALV E. Laparoscopic tubal plasty
E. Stimulation of the skin along the spine
161. A 19-year-old primiparous woman wi-
157. Examination of an 11-year-old boy th a body weight of 54,5 kg gave birth at
revealed frequent nosebleeds, fatigue when 38 weeks gestation to a full-term live girl
walking, underdevelopment of the lower half after a normal vaginal delivery. The girl’s wei-
of the body, increased blood pressure in the ght was 2180,0 g, body length - 48 cm. It is
Krok 2 Medicine 2014 20

known from history that the woman has been suspected?


a smoker for 8 years, and kept smoking duri-
ng pregnancy. Pregnancy was complicated by A. Heavy-metal coniosis
moderate vomiting of pregnancy from 9 to 12 B. Silicosis
weeks pregnant, edemata of pregnancy from C. Silicatosis
32 to 38 weeks. What is the most likely cause D. Carbon pneumo coniosis
of low birth weight? E. Bronchitis

A. Fetoplacental insufficiency 166. A 12-year-old boy periodically has short


B. Low weight of the woman episodes (10-15 seconds) of a brief loss of
C. Woman’s age awareness with a dazed look and eyes stare in
D. First trimester preeclampsia an upright position, blank expression of face,
E. Third trimester preeclampsia absence of motions and subsequent amnesia.
Specify the described state:
162. In a cold weather, the emergency room
admitted a patient pulled out of the open A. Absence seizure
water. There was no respiratory contact wi- B. Obnubilation
th the water. The patient is excited, pale, C. Trance
complains of pain, numbness of hands and D. Fugue
feet, cold shiver. Respiratory rate is 22/min, E. Sperrung
AP- 120/90 mm Hg, Ps- 110/min, rectal
temperature is 34, 5o C . What kind of warming 167. Evaluation results of sanitary and hygiene
is indicated for this patient? conditions in a 4-bed ward were as follows:
ward area - 30 m2 , height - 3,2 m, temperature
A. Passive warming - 20o C , humidity - 55%, air velocity - 0,1 m/s,
B. Infusion of 37o C solutions window-to-floor area ratio - 1:5, daylight ratio
C. Hot compresses - 0,6%, concentration of carbon dioxide in the
D. Warm bath air - 0,1%. Which of the given indicators does
E. Hemodialysis with blood warming not meet hygienic requirements?
163. A week before, a 65-year-old male pati- A. Daylight ratio
ent suffered an acute myocardial infarction. B. Air velocity
His general condition has deteriorated: he C. Window-to-floor area ratio
complains of dyspnea at rest, pronounced D. Concentration of carbon dioxide in the air
weakness. Objectively: edema of the lower E. Ward area
extremities, ascites is present. Heart borders
are extended, paradoxical pulse is 2 cm di- 168. A 25-year-old patient works as a tractor
splaced from the apex beat to the left. What is driver. Four days ago, he got pain in the left
the most likely diagnosis? axillary region, general weakness, fever up to
38o C . He hadn’t sought medical helf until a
A. Acute cardiac aneurysm painful solid lump appeared in this region.
B. Recurrent myocardial infarction Objectively: in the left axilla there is a very
C. Acute pericarditis painful cone-shaped mass sized 3x2,5 cm, wi-
D. Cardiosclerotic aneurysm th a destruction in the center of the pointed
E. Pulmonary embolism vertex. The surrounding skin is hyperemic,
there are purulent discharges. What is the
164. A 41-year-old patient cosulted a most likely diagnosis?
dermatologist about discoloration, thickening,
brittleness of toenails. These symptoms have A. Hydradenitis
been present for about five years. Objecti- B. Carbuncle
vely: nail plates in all toes are thickened, of C. Furuncle
dirty yellow color, lustreless, crumble over the D. Lymphadenitis
edge. Microscopy of the nail plate material E. Abscess
treated with alkali revealed mycelial filaments.
Material inoculation onto Sabouraud medium 169. A 19-year-old male patient complains of
resulted in growth of the Trichophyton rubrum intense pain in the left knee joint. Objectively:
colony. What is the most likely diagnosis? the left knee joint is enlarged, the overlyi-
ng skin is hyperemic, the joint is painful on
A. Rubromycosis of toenails palpation. Blood test results: RBC- 3, 8 · 1012 /l,
B. Candidal onychia Hb- 122 g/l, lymphocytes - 7, 4 · 109 /l, platelets
C. Psoriasis of the nails - 183 · 109 /l. ESR- 10 mm/h. Duke bleeding
D. Nail dystrophy time is 4 minutes, Lee-White clotting time - 24
E. Epidermophytosis of nails minutes. A-PTT is 89 sec. Rheumatoid factor
165. Examination of an electric welder with is negative. What is the most likely diagnosis?
15 years of service record revealed dry rales
in the lower lung fields. Radiograph shows
diffuse nodules sized 3-4 mm in the middle
and lower lung fields. What disease can be
Krok 2 Medicine 2014 21

A. Hemophilia, hemarthrosis A. Sorting, medical assistance, evacuation


B. Werlhof’s disease B. Sorting, evacuation, treatment
C. Rheumatoid arthritis C. Medical assistance, evacuation, isolation
D. Thrombocytopathy D. Isolation, rescue activity, recovery
E. Hemorrhagic vasculitis, articular form E. Sorting, recovery, rescue activity
170. A 29-year-old female patient complai- 174. A 36-year-old injured has been taken
ns of dyspnea and palpitations on exertion. to the emergency station with open tibi-
According to her mother, as a child she had al fractures. Examination reveals bleeding:
heart murmur, did not undergo any exami- pulsating blood spurts out of the wound. What
nations. Objectively: the patient has pale skin, medical manipulations are required at this
Ps- 94/min, rhythmic. AP- 120/60 mm Hg. In stage of medical care?
the II intercostal space on the left auscultation
reveals a continuous rasping systolodiastolic A. Apply a tourniquet on the thigh proximal
murmur, diastolic shock above the pulmonary to the source of bleeding and transport the
artery. Blood and urine are unremarkable. patient to the operating room
What is the most likely diagnosis? B. Immobilize the fracture and transport the
patient to the operating room
A. Patent ductus arteriosus C. Apply a tourniquet on the thigh distal to the
B. Atrial septal defect source of bleeding and transport the patient to
C. Ventricular septal defect the operating room
D. Aortarctia D. Stop the bleeding by a compressive bandage
E. Tetralogy of Fallot and transport the patient to the operating room
E. Transport the patient to the operating room
171. A 6-year-old boy had had a quinsy. 9
days later, there appeared edemata of the 175. The outpatient department of a city
face, extremities and trunk, general health hospital works also as a 60-bed day hospital
condition deteriorated. Urine became turbid. for somatic patients. The department operates
Objectively: expressive edemata, ascites. AP- in a single-shift mode. What specialist selects
100/55 mm Hg, diuresis - 0,2 l of urine per patients for admission to the day hospital?
day. Results of the biochemical blood analysis:
total protein - 50 g/l, cholesterol - 11,28 A. District doctor
mmol/l, urea - 7,15 mmol/l, creatinine - 0,08 B. Resident doctor
mmol/l. Urinalysis results: leukocytes - 3-5 per C. Chief of the outpatient department
HPF, red blood cells are absent. What is the D. Deputy chief of the hospital
provisional diagnosis? E. Chief physician of the hospital

A. Acute glomerulonephritis 176. A 67-year-old female patient with


B. Acute pyelonephritis hypertensive crisis has asthma, cough with
C. Urolithiasis expectoration of frothy pink sputum, moist
D. Acute renal failure rales in the lungs. The patient stays in sitti-
E. Chronic glomerulonephritis ng position, respiratory rate is 40/min, AP-
214/136 mm Hg, heart rate - 102/min. What
172. A 13-year-old boy has had abdominal is the most rational tactics of this patient
pain, bloating, nausea, liquid fatty gray stool management?
with putrid smell for the last 3 years. Palpati-
on reveals epigastric tenderness, as well as A. Intravenous administration of furosemide
tenderness in the Desjardins’ pancreatic point, B. Urgent pneumography
Chauffard’s triangle; there is positive Mayo- C. Bed rest, lying position
Robson’s sign. Failure of exocrine pancreatic D. Intravenous administration of a β -blocker
function has been suspected. What is the most E. Tactics can be determined after ECG and
informative method for evaluating the state of chest radiography
exocrine pancreatic function?
177. A 23-year-old primigravida at 39 weeks
A. Fecal elastase-1 determination gestation has been admitted to the maternity
B. Blood serum trypsin determination ward with irregular contractions. The intensi-
C. Sonography of the pancreas ty of uterine contractions is not changing,
D. Blood and urine amylase determination the intervals between them stay long. Bi-
E. Scatological study manual examination reveals that the cervix
is centered, soft, up to 1,5 cm long. There is no
173. Explosion of a tank with benzene at a cervical dilatation. What diagnosis should be
chemical plant has killed and wounded a large made?
number of people. There are over 50 victims
with burns, mechanical injuries and intoxicati-
on. Specify the main elements of medical care
and evacuation of population in this situation:
Krok 2 Medicine 2014 22

A. Pregnancy I, 39 weeks, preliminary period A. Iron-deficiency anemia


B. Pregnancy I, 39 weeks, labor I, 1 period, the B. Acute leukemia
latent phase C. B12 -deficiency anemia
C. Pregnancy I, 39 weeks, labor I, period 1, the D. Vegetative-vascular dystonia
active phase E. Aplastic anemia
D. Pregnancy I, 39 weeks, birth I, 1 period, the
acceleration phase 182. During the ultrasound study of carotid
E. Pregnancy I, 39 weeks, pathological prelimi- and vertebral arteries a 74-year-old pati-
nary period ent developed a condition manifested by di-
zziness, weakness, nausea, transient loss of
178. A 49-year-old male patient complains of consciousness. Objectively: pale skin, AP-
retrosternal pain, heartburn, weight loss of 8 80/60 mm Hg, Ps- 96/min of poor volume.
kg over the last year, constipation, weakness. ECG shows sinus tachycardia, left ventricular
The patient has been a smoker for 20 years, hypertrophy. Focal neurological symptoms
and has a 10-year history of gastroesophageal were not found. What is the provisional di-
reflux disease. The patient is asthenic, has dry agnosis?
skin. EGD revealed an ulcer in the lower
third of the esophagus and esophageal stri- A. Carotid sinus syncope
cture accompanied by edema, hyperemia and B. Orthostatic syncope
multiple erosions of the mucosa. What study C. Morgagni-Adams-Stokes attack
is required for more accurate diagnosis? D. Complete atrioventricular block
E. Acute cerebrovascular accident
A. Biopsy of the esophageal mucosa
B. X-ray examination of the esophagus 183. On the 10th day postpartum a puerperant
C. Respiratory test for Helicobacter pylori woman complains of pain and heaviness in
D. pH-metry of the esophagus and the stomach the left breast. Body temperature is 38, 8o C ,
E. Fecal occult blood test Ps- 94 bpm. The left breast is edematic, the
supero-external quadrant of skin is hyperemic.
179. A 63-year-old male patient with persi- Fluctuation symptom is absent. The nipples di-
stent atrial fibrillation complains of moderate scharge drops of milk when pressed. What is a
dyspnea. Objectively: peripheral edemata are doctor’s further tactics?
absent, vesicular breathing is present, heart
rate - 72/min, AP- 140/90 mm Hg. What combi- A. Antibiotic therapy, immobilization and
nation of drugs will be most effective for the expression of breast milk
secondary prevention of heart failure? B. Compress to both breasts
C. Inhibition of lactation
A. Beta-blockers, ACE inhibitors D. Physiotherapy
B. Beta-blockers, cardiac glycosides E. Opening of the abscess and drainage of the
C. Cardiac glycosides, diuretics breast
D. Cardiac glycosides, ACE inhibitors
E. Diuretics, beta-blockers 184. During the breast self-exam a 37-year-old
female patient revealed a lump in the lower
180. A 57-year-old male patient had an attack inner quadrant of her left breast. Palpation
of retrosternal pain that lasted more than confirms presence of a mobile well-defined
1,5 hours. Objectively: the patient is inert, neoplasm up to 2 cm large. Peripheral lymph
adynamic, has pale skin, cold extremities, poor nodes are not changed. What is the way of
volume pulse, heart rate - 120/min, AP- 70/40 further management?
mm Hg. ECG shows ST elevation in II, III,
aVF leads. What condition are these changes A. Ultrasound examination of breasts,
typical for? mammography, fine-needle aspiration bi-
opsy
A. Cardiogenic shock B. Anti-inflammatory therapy, physiotherapy
B. Arrhythmogenic shock C. Radical mastectomy
C. Perforated gastric ulcer D. Ultrasound monitoring of genitals during
D. Acute pericarditis the entire course of antiestrogens therapy,
E. Acute pancreatitis systemic enzyme therapy, phytotherapy
E. Case follow-up
181. During an exam, a 22-year-old female
student fainted. She grew up in a family with 185. An 8-year-old child was hospitalized
many children, has a history of frequent acute for fever up to 39, 8o C , inertness, moderate
respiratory infections. Objectively: the patient headache, vomiting. Examination revealed
has pale skin and mucous membranes, split- meningeal symptoms. Lumbar puncture was
end hair, brittle nails. Blood test results: RBC- performed. The obtained fluid had raised
2, 7 · 1012 /l, Hb- 75 g/l, color index - 0,7, WBC- opening pressure, it was transparent, with
3, 2 · 109 /l, platelets - 210 · 109 /l, ESR- 30 mm/h. the cell count of 450 cells per 1 mcL (mai-
Blood serum iron is 6 mmol/l. What is the most nly lymphocytes - 90%), glucose level of 2,6
likely diagnosis? mmol/l. What causative agent might have
caused the disease in the child?
Krok 2 Medicine 2014 23

A. Enterovirus
B. Meningococcus A. Chronic adrenal insufficiency
C. Koch’s bacillus B. Diabetes mellitus
D. Staphylococcus C. Coronary artery disease
E. Pneumococcus D. Chronic pancreatitis
E. Pulmonary tuberculosis
186. A 25-year-old female has a self-detected
tumor in the upper outer quadrant of her ri- 190. A 42-year-old male patient has been
ght breast. On palpation there is a painless, delivered to a hospital in a grave condition
firm, mobile lump up to 2 cm in diameter, with dyspnea, cough with expectoration of
peripheral lymph nodes are not changed. In purulent sputum, fever up to 39, 5o C . The
the upper outer quadrant of the right breast first symptoms appeared 3 weeks ago. Two
ultrasound revealed a massive neoplasm with weeks ago, a local therapist diagnosed him wi-
increased echogenicity sized 21x18 mm. What th acute right-sided pneumonia. Over the last
is the most likely diagnosis? 3 days, the patient’s condition deteriorated:
there was a progress of dyspnea, weakness,
A. Fibroadenoma lack of appetite. Chest radiography confirms a
B. Lactocele rounded shadow in the lower lobe of the right
C. Diffuse mastopathy lung with a horizontal fluid level, the right si-
D. Mammary cancer nus is not clearly visualized. What is the most
E. Mastitis likely diagnosis?
187. A 49-year-old female patient complai- A. Abscess of the right lung
ns of itching, burning in the external geni- B. Acute pleuropneumonia
tals, frequent urination. The symptoms have C. Right pulmonary empyema
been present for the last 7 months. The pati- D. Atelectasis of the right lung
ent has irregular menstruation, once every E. Pleural effusion
3-4 months. Over the last two years she has
had hot flashes, sweating, sleep disturbance. 191. An 11-year-old boy complains of general
Examination revealed no pathological weakness, fever up to 38, 2o C , pain and swelli-
changes of the internal reproductive organs. ng of the knee joints, feeling of irregular
Complete blood count and urinalysis showed heartbeat. 3 weeks ago, the child had quinsy.
no pathological changes. Vaginal smear Knee joints are swollen, the overlying skin
contained 20-25 leukocytes per HPF, mixed and skin of the knee region is reddened, local
flora. What is the most likely diagnosis? temperature is increased, movements are li-
mited. Heart sounds are muffled, extrasystole
A. Menopausal syndrome is present, auscultation reveals apical systolic
B. Cystitis murmur that is not conducted to the left ingui-
C. Trichomonas colpitis nal region. ESR is 38 mm/h. CRP is 2+, anti-
D. Vulvitis streptolysin O titre - 400. What is the most
E. Bacterial vaginosis likely diagnosis?
188. A 45-year-old male patient complains of A. Acute rheumatic fever
acute pain in his right side irradiating to the B. Vegetative dysfunction
right thigh and crotch. The patient claims also C. Non-rheumatic carditis
to have frequent urination with urine which D. Juvenile rheumatoid arthritis
resembles a meat slops. The patient has no E. Reactive arthritis
previous history of this condition. There is
costovertebral angle tenderness on the right 192. A 28-year-old male patient complains
(positive Pasternatsky’s symptom). What is of sour regurgitation, cough and heartburn
the most likely diagnosis? that occurs every day after having meals,
when bending forward or lying down. These
A. Urolithiasis problems have been observed for 4 years.
B. Acute appendicitis Objective status and laboratory values are
C. Acute pyelonephritis normal. FEGDS revealed endoesophagitis.
D. Acute cholecystitis. Renal colic What is the leading factor in the development
E. Acute pancreatitis of this disease?
189. After a holiday in the Crimea, a 49- A. Failure of the lower esophageal sphincter
year-old male patient with a history of lung B. Hypersecretion of hydrochloric acid
tuberculosis felt increased weakness, peri- C. Duodeno-gastric reflux
odic dizziness, easing bowel movements with D. Hypergastrinemia
abdominal pain, the need for additional salti- E. Helicobacter pylori infection
ng his meals. The patient has noted that his
condition improves after some sweet tea and 193. On admission a 35-year-old female
validol taken sublingually. Objectively: there reports acute abdominal pain, fever up to
is an intense darkening of skin, AP- 70/50 mm 38, 8o C , mucopurulent discharges. The pati-
Hg, glycemia is 3,0 mmol/l. What is the possi- ent is nulliparous, has a history of 2 artificial
ble cause of health deterioration: abortions. The patient is unmarried, has sexual
Krok 2 Medicine 2014 24

contacts. Gynecological examination reveals scan showed a destruction area in the center
no uterus changes. Appendages are enlarged, of the shadow. Sputum analysis revealed
bilaterally painful. There is profuse purulent MTB. The patient was diagnosed with focal
vaginal discharge. What study is required to tuberculosis. What phases of tuberculosis are
confirm the diagnosis? the identified changes typical for?
A. Bacteriologic and bacteriascopic studies A. Infiltration and disintegration
B. Hysteroscopy B. Infiltration and dissemination
C. Curettage of uterine cavity C. Resorption and scarring
D. Vaginoscopy D. Disintegration and dissemination
E. Laparoscopy E. Calcification and resorption
194. A 20-year-old female consulted a 198. A 43-year-old female complains of si-
gynecologist about not having menstrual gnificant weakness, sore throat, occurrence
period for 7 months. History abstracts: of multiple unexplained bruises on her skin.
early childhood infections and frequent These symptoms have been present for a
tonsillitis, menarche since 13 years, regular week, the disease is associated with quinsy
monthly menstrual cycle of 28 days, painless which she had some time before. Objectively:
menstruation lasts 5-6 days. 7 months ago the body temperature - 38, 9o C , respiratory rate -
patient had an emotional stress. Gynecologi- 24/min, Ps- 110/min, AP- 100/65 mm Hg. The
cal examination revealed no alterations in the patient has pale skin, petechial rash on the
uterus. What is the most likely diagnosis? extremities, enlarged lymph nodes. Blood test
results: Hb- 80 g/l, RBC- 2, 2 · 1012 /l; WBC-
A. Secondary amenorrhea
B. Primary amenorrhea 3, 5 · 109 /l; blasts - 52%; eosinophils - 2%;
C. Algomenorrhea stab neutrophils - 3%; segmented neutrophils
D. Spanomenorrhea - 19%; lymphocytes - 13%; monocytes - 1%;
E. Cryptomenorrhea platelets - 35 · 109 /l. ESR - 47 mm/h. What test
is required to specify the diagnosis?
195. A 48-year-old female has been admi-
tted to the gynecology department for pain in A. Immunophenotyping
the lower right abdomen and low back pain, B. Protein electrophoresis
constipations. Bimanual examination findings: C. Lymph node biopsy
the uterus is immobile, the size of a 10-week D. Determination of anti-platelet antibody
pregnancy, has uneven surface. Aspirate from titer
the uterine cavity contains atypical cells. What E. Cytogenetic study
diagnosis can be made?
199. A 47-year-old male patient complains of
A. Hysterocarcinoma compressive chest pain that occurs both at rest
B. Cervical cancer and during light physical activity; irregular
C. Metrofibroma heartbeat. These problems arose 3 months
D. Colon cancer ago. The patient’s brother died suddenly at the
E. Chorionepithelioma age of 30. Objectively: Ps- 84/min, arrhythmic,
AP- 130/80 mm Hg. ECG confirms signs of
196. A 27-year-old male patient consulted left ventricular hypertrophy, abnormal Q-
a doctor about pain in the lower third waves in V 4 − V 6 leads. EchoCG reveals that
of the thigh with weight bearing activiti- interventricular septum is 1,7 cm, left ventri-
es and unloaded. Two years ago, the pati- cular wall thickness is 1,2 cm. What is the most
ent underwent treatment in the casualty likely diagnosis?
depatment for the open fracture of the lower
third of femur. The fracture healed slowly, the A. Hypertrophic cardiomyopathy
healing process was accompanied by prulent B. Neurocirculatory asthenia
inflammation. Objectively: edema of the lower C. Exertional angina
third of the thigh, elevated local temperature. D. Myocarditis
Radiograph shows signs of destruction and E. Pericarditis
sequestration. What is the most likely di-
agnosis? 200. Within the structure of the region’s
population the share of persons aged 0 to 14
A. Chronic post-traumatic osteomyelitis years is 25%, the share of persons aged 50
B. Osteosarcoma years and older is 30%. What concept most
C. Tuberculosis of femur accurately describes this demographic situati-
D. Hematogenous osteomyelitis on?
E. Multiple myeloma
A. Regressive type of population age structure
197. A 30-year-old male patient had been B. Progressive type of population age structure
admitted to the TB hospital because of the C. Cohort reproduction
following changes detected by fluorography: D. Stationary type of population age structure
an ill-defined shadow of low intensity up to 1 E. Immigration of population
cm in diameter in the S1 of the right lung. CT
Крок 2 Medicine (англомовний варiант, iноземнi студенти) 2015 рiк 1

1. 13 months after the first labor a 24- A. Droperidolum of 0,25% - 2,0 ml


year-old patient complained of amenorrhea. B. Dibazolum (Bendazole hydrochloride) of
Pregnancy ended in Caesarian section because 1% - 6,0 ml
of premature detachment of normally positi- C. Papaverine hydrochloride of 2% - 4,0 ml
oned placenta which resulted in blood loss at D. Hexenalum of 1% - 2,0 ml
the rate of 2000 ml due to disturbance of blood E. Pentaminum of 5% - 4,0 ml
clotting. Choose the most suitable investigati-
on: 5. A patient with high temperature came to
a first-aid post in the evening. The fact of
A. Estimation of gonadotropin rate temporary disability was established. Indicate
B. US of small pelvis the measure to be taken in this case:
C. Progesteron assay
D. Computer tomography of head A. The night duty doctor should issue a medical
E. Estimation of testosteron rate in blood serum certificate, which will be subsequently used for
issuing a sick list from the date of the previous
2. A 24-year-old primipara was hospitalised day
with complaints of discharge of the amniotic B. The sick list for 1 day should be issued
waters. The uterus is tonic on palpation. The C. The sick list for up to 3 days should be issued
position of the fetus is longitudinal, it is pressed D. The sick list for 3 days should be issued
with the head to pelvic outlet. Palpitation of E. No document should be issued
the fetus is rhythmical, 140 bpm, auscultated
on the left below the navel. Internal examinati- 6. A 10-year-old boy complains of pain in his
on: cervix of the uterus is 2,5 cm long, dense, left eye and strong photophobia after he has
the external opening is closed, light amniotic injured his left eye with a pencil at school. Left
waters are discharged. Point out the correct eye examination revealed: blepharospasm, ci-
component of the diagnosis: liary and conjunctival congestion, cornea is
transparent, other parts of eyeball have no
A. Antenatal discharge of the amniotic waters changes. Visus 0,9. Right eye is healthy, Visus
B. Early discharge of the amniotic waters 1,0. What additional method would you choose
C. The beginning of the 1st stage of labour first of all?
D. The end of the 1st stage of labour
E. Pathological preterm labour A. Staining test with 1% fluorescein
B. X-ray examination of orbit
3. A patient was delivered to a surgical C. Tonometria
department after a road accident with a closed D. Gonioscopia
trauma of chest and right-sided rib fracture. E. Cornea sensation-test
The patient was diagnosed with right-sided
pneumothorax; it is indicated to perform drai- 7. An infant was born with body mass 3 kg and
nage of pleural cavity. Pleural puncture should body length 50 cm. Now he is 3 years old. His
be made: brother is 7 years old, suffers from rheumatic
fever. Mother asked a doctor for a cardiac
A. In the 2nd intercostal space along the middle check up of the 3-year-old son. Where is the
clavicular line left relative heart border located?
B. In the 6th intercostal space along the posteri-
or axillary line A. 1 cm left from the left medioclavicular line
C. In the 7th intercostal space along the scapular B. 1 cm right from the left medioclavicular line
line C. Along the left medioclavicular line
D. In the projection of pleural sinus D. 1 cm left from the left parasternal line
E. In the point of the greatest dullness on E. 1 cm right from the left parasternal line
percussion 8. A baby was born at 36 weeks of gestation.
4. A primagravida with pregnancy of 37-38 Delivery was normal, by natural way. The baby
weeks complains of headache, nausea, pain in has a large cephalohematoma. The results of
epigastrium. Objectively: the skin is acyanotic. blood count are: Hb- 120g/l, Er- 3, 5 · 1012 /l,
Face is hydropic, there is short fibrillar twi- total serum bilirubin - 123 mmol/l, direct bili-
tching of blepharons, muscles of the face and rubin - 11 mmol/l, indirect - 112 mmol/l. What
the inferior extremities. The stare is fixed. BP are the causes of hyperbilirubinemia in this
- 200/110 mm Hg; sphygmus is of 92 bpm, case?
intense. Respiration rate is 32/min. Heart acti-
vity is rhythmical. Appreciable edemas of A. Erythrocyte hemolysis
the inferior extremities are present. Urine is B. Intravascular hemolysis
cloudy. What medication should be admini- C. Disturbance of the conjugative function of
stered? liver
D. Bile condensing
E. Mechanical obstruction of the bile outflow
9. A patient has been hospitalised. The onset of
the disease was gradual: nausea, vomiting, dark
urine, аcholic stool, yellowness of the skin and
scleras. The liver is enlarged by 3 cm. Jaundice
Крок 2 Medicine (англомовний варiант, iноземнi студенти) 2015 рiк 2

developed on the 14th day of the disease. The 14. A 13-year-old girl complains of periodic
liver diminished in size. What complication of prickly pain in the heart region. Percussi-
viral hepatitis caused deterioration of the pati- on revealed no changes of cardiac borders.
ent’s condition? Auscultation revealed arrhythmic enhanced
heart sounds, extrasystole at the 20-25 cardi-
A. Hepatic encephlopathy ac impulse. ECG showed the sinus rhythm,
B. Meningitis impaired repolarization, single supraventri-
C. Relapse of viral hepatitis cular extrasystoles at rest. What is the most
D. Cholangitis likely diagnosis?
E. Infectious-toxic shock
A. Somatoform autonomic dysfunction
10. An 18-year-old patient was admitted B. Rheumatism
to a hospital with complaints of headache, C. Nonrheumatic carditis
weakness, high fever, sore throat. Objectively: D. Myocardial degeneration
enlargement of all groups of lymph nodes was E. Intoxication syndrome
revealed. The liver is enlarged by 3 cm, spleen
- by 1 cm. In blood: leukocytosis, atypical 15. A 24-year-old patient visited a doctor
lymphocytes - 15%. What is the most probable complaining of enlargement of his submaxi-
diagnosis? llary lymph nodes. Objectively: submaxillary,
axillary and inguinal lymph nodes are enlarged.
A. Infectious mononucleosis Chest X-ray shows: enlarged lymph nodes
B. Acute lymphoid leukosis of mediastinum. Blood test: erythrocytes -
C. Diphtheria 3, 4 · 1012 /l, Hb- 100 g/l, blood colour index
D. Angina
E. Adenoviral infection - 0,88, platelets - 190 · 109 /l, leucocytes -
7, 5 · 109 /l, eosinophiles - 8%, band neutrophi-
11. A 60-year-old woman, mother of 6 children, les - 2%, segmented neutrophiles - 67%,
developed a sudden onset of upper abdomi- lymphocytes - 23%, ESR - 22 mm/h. What
nal pain radiating to the back, accompani- test must be prescribed to verify the cause of
ed by nausea, vomiting, fever and chills. lymphadenopathy?
Subsequently, she noticed yellow discolorati-
on of her sclera and skin. On physical exami- A. Open biopsy of lymph nodes
nation the patient was found to be febrile with B. Ultrasonography of abdominal cavity
temperature of 38, 9o C , along with right upper C. Mediastinum tomography
quadrant tenderness. The most likely diagnosis D. Puncture biopsy of lymph nodes
is: E. Sternal puncture

A. Choledocholithiasis 16. A 60-year-old woman has been sufferi-


B. Benign biliary stricture ng from arterial hypertension for 15 years.
C. Malignant biliary stricture After recurrent stroke she started complaini-
D. Carcinoma of the head of the pancreas ng about unmotivated bad mood, problems
E. Choledochal cyst with attention concentration; she forgets to
close the entrance door, cannot recall events
12. What juice is recommended to be included of the past day. Computer tomography shows
in a complex drug and diet therapy for patients areas of postinfarction changes in the cortical
suffering from gastric or duodenal ulcer and postfrontal areas. What is the most probable
high acidity of gastric juice to speed up ulcer diagnosis?
healing?
A. Vascular dementia
A. Potato, potato-carrot B. Alzheimer’s disease
B. Apple, apple-birch C. Huntington’s disease
C. Pumpkin D. Pick’s disease
D. Cabbage, cabbage-carrot E. Dissociative amnesia
E. Celery, parsley
17. Clinic of a research instutute for occupati-
13. A 40-year-old woman has been hospitalized onal diseases examined a worker who works at
for attacks of asphyxia, cough with phlegm. She a concentration plant and diagnosed him with
has a 4-year history of the disease. The first chronic dust bronchitis. The case is investigated
attack of asphyxia occurred during her stay in by a commission including the representatives
the countryside. Further attacks occurred whi- of: the plant, medical unit, territorial sanitation
le cleaning the room. After 3 days of inpatient center, department of Social Insurance Fund,
treatment the patient’s condition has signifi- trade union. According to the "regulation on
cantly improved. What is the most likely eti- investigation of. . . ", the commission should be
ological factor? headed by the representative of the following
authority:
A. Household allergens
B. Pollen
C. Infectious
D. Chemicals
E. Psychogenic
Крок 2 Medicine (англомовний варiант, iноземнi студенти) 2015 рiк 3

A. Territorial sanitation center mes joined by hemoptysis and pain in the ri-
B. Plant ght side of chest. Breathing is vesicular. X-ray
C. Social Insurance Fund shows darkening and sharp decrease in size of
D. Trade union the lower lobe distinctly visible on the X-ray
E. Medical unit image as a streak 2-3 cm wide situated at the
angle from lung root to the frontal costodi-
18. 4 days after a patient received a gunshot aphragmatic recess. The most likely diagnosis
wound of the middle third of the thigh soft ti- is:
ssues his condition suddenly began deteriorati-
ng. There are complaints of bursting pain in the A. Peripheral lung cancer
wound; pain increases during the last 12 hours. B. Bronchiectasis
Edema of skin and hypodermic tissue quickly C. Pneumonia
grows. Body temperature is 38, 2o C , heart rate D. Middle lobe syndrome
is 102/min. The wound edges gape, are dull in E. Interlobular pleurisy
color; the muscles, viable as of day before, now
protrude into the wound, look boiled, are dull 23. A 52-year-old patient, who has been sufferi-
in colour, have dirty-grey coating and fall apart ng from angina pectoris for 2 weeks, has
when being held with forceps. What infection more and more frequent pain attacks in the
has developed in the wound? area behind his sternum and his need for ni-
troglycerine increased. Objectively: the condi-
A. Anaerobic tion is of moderate severity. Skin is pale. Heart
B. Aerobic gram-negative sounds are weakened, rhythmic. Heart rate is
C. Putrid 84 per minute. ECG shows no signs of focal
D. Aerobic gram-positive myocardial damage. What is the most likely di-
E. Diphtheria of wound agnosis?
19. Bacterial analysis of air in a living space in A. Progressive angina pectoris
winter period by means of Krotov’s apparatus B. First-time angina pectoris
revealed that total number of microorganisms C. Stable FC II angina pectoris
in 1m3 of air was 7200. What is the allowed D. Variant angina pectoris
number of microorganisms for the air to be E. Acute cardiac infarction
characterized as "pure"?
24. An 8-year-old boy during preventive exami-
A. Up to 4500 nation was determined to have changes in
B. Up to 2500 his spine curvature in frontal plane: the ri-
C. Up to 3500 ght shoulder is lowered and flat, scapulae
D. Up to 5500 angles are of different height due to the ri-
E. Up to 7500 ght scapula being shifted down. Waist triangles
are pronounced on the both sides; longitudinal
20. Maximum permissible concentration of muscles of the back form muscle cushion on
carbon dioxide in the air is considered to be the left. What type of posture is detected in the
a sanitary index of air purity in a classroom. child?
What concentration of carbon dioxide in the
air is accepted as a permissible maximum? A. Scoliotic
B. Kyphotic
A. 0,1% C. Lordotic
B. 0,05% D. Stooping
C. 0,15% E. Corrected
D. 0,2%
E. 0,3% 25. A patient with cardiac infarction is on sick
leave with his medical certificate being conti-
21. When examining the parameters of desk nuously extended for 4 months according to a
natural lighting in a school class it was determi- standard procedure. The disease is progressi-
ned that: light angle is 25o , window angle is ng. The issue is put forward to refer the pati-
3o , window-to-floor area ratio is 1:4, dayli- ent to the industrial injury assessment board.
ght factor is 0,5%, and window-head-to-room- Who among the medical staff of medical and
depth ratio is 2. What parameter does not preventive treatment facility is authorized to
correspond to hygienic norms? do this?
A. Daylight factor A. Medical expert committee
B. Window angle B. Attending medical doctor
C. Window-to-floor area ratio C. Head doctor of hospital department
D. Window-head-to-room-depth ratio D. Deputy head doctor for assessment of
E. Light angle temporary disability
E. Head doctor
22. A 29-year-old patient works as a motor
mechanic. Anamnesis shows frequent exposure 26. A 9-year-old boy has been suffering from
to cold, exacerbation of chronic bronchitis bronchoectasis since he was 3. Exacerbations
attended by cough with relativly small amount occur quite often, 3-4 times a year. Conservati-
of mucopurulent sputum, subfebrility, someti- ve therapy results in short periods of remi-
Крок 2 Medicine (англомовний варiант, iноземнi студенти) 2015 рiк 4

ssion. The disease is progressing, the child 31. A 46-year-old patient once took part in eli-
has physical retardation. The child’s skin is mination of breakdown at an atomic power
pale, acrocyanotic, he has "watch glass"nail plant. Currently he is being treated at an in-
deformation. Bronchography revealed saccular patient hospital unit. He was diagnosed with
bronchiectases of the lower lobe of his right progressing somatoform autonomic dysfuncti-
lung. What is the further treatment tactics? on. This disease relates to the following group
of ionizing radiation effects:
A. Surgical treatment
B. Further conservative therapy A. Somato-stochastic
C. Physiotherapeutic treatment B. Somatic
D. Sanatorium-and-spa treatment C. Genetic
E. Tempering of the child’s organism D. Hormesis
E. Heterosis
27. A 29-year-old female patient complains of
dyspnea, heaviness and chest pain on the ri- 32. A patient complains of fatigue, lack of
ght, body temperature rise up to 37, 2o C . The appetite, pain and burning sensation in the
disease is associated with a chest trauma recei- tongue, numbness of the distal limbs, diarrhea.
ved 4 days ago. Objectively: skin is pale and Objectively: pale skin with lemon-yellow tint,
moist. Heart rate is 90 bpm, regular. Palpation face puffiness, brown pigmentation in the form
reveals a dull sound on the right, auscultati- of a "butterfly", bright red areas on the tongue.
on reveals significantly weakened vesicular The liver is 3 cm below the costal margin, soft.
breathing. In blood: RBCs - 2, 8 · 1012 /l, colour Blood count: RBCs - 1, 5 · 1012 /l, colour index -
index - 0,9, Hb- 100 g/l, WBCs - 8, 0 · 109 /l, ESR 1,2, WBCs - 3, 8·109 /l, thrombocytes - 180·109 /l,
- 17 mm/h. What results of diagnostic puncture eosinophiles - 0%, stab neutrophiles - 1%,
of the pleural cavity can be expected? segmented neutrophiles - 58%, lymphocytes -
38% monocytes - 3%, RBC macrocytosis. ESR
A. Haemorrhagic punctate - 28 mm/h. What diagnosis are these presentati-
B. Chylous liquid ons typical for?
C. Exudate
D. Transudate A. B12 -deficiency anemia
E. Purulent punctate B. Iron deficiency anemia
C. Aplastic anemia
28. A 54-year-old man had been drowning at D. Acute erythromyelosis
sea, when he was found and evacuated to the E. Chronic adrenal failure
shore. Objectively: unconscious, pale face, no
breathing can be auscultated, thready pulse. 33. A 25-year-old female patient complains of
Resuscitation measures allowed to save the marked weakness, sleepiness, blackouts, di-
man. What complication can develop in him zziness, taste disorder. The patient has a hi-
in the nearest future? story of menorrhagia. Objectively: the pati-
ent has marked weakness, pale skin, cracks
A. Pulmonary edema in the corners of her mouth, peeling nails,
B. Respiratory arrest systolic apical murmur. Blood test results: RBC
C. Encephalopathy - 3, 4 · 1012 /l, Hb- 70 g/l, colour index - 0,75,
D. Cardiac arrest
E. Bronchial spasm platelets - 140 · 109 /l, WBC- 6, 2 · 109 /l. What is
the most likely diagnosis?
29. A 7-year-old child complains of cramping
pain occuring after mental exertion, cold dri- A. Chronic posthemorrhagic anemia
nks and eating ice-cream. Instrumental exami- B. Acute leukemia
nation allowed to diagnose biliary dyskinesia of C. Acute posthemorrhagic anemia
hypertensive type. What group of drugs should D. B12 -deficiency anemia
be prescribed for treatment? E. Werlhof’s disease

A. Antispasmodics and choleretics 34. A 39-year-old patient complains of morning


B. Choleretics and cholekinetics headache, appetite loss, nausea, morning vomi-
C. Sedatives and cholekinetics ting, periodic nasal haemorrhages. The patient
D. Antioxydants had acute glomerulonephritis at the age of 15.
E. Antibiotics Examination revealed rise of arterial pressure
up to 220/130 mm Hg, skin haemorrhages on
30. Caries morbidity rate is 89% among resi- his arms and legs, pallor of skin and mucous
dents of a community. It is determined that membranes. What biochemical parameter is
fluorine content in water is 0,1 mg/l. What the most important for making diagnosis in
preventive measures should be taken? this case?

A. Water fluorination A. Blood creatinine


B. Tooth brushing B. Blood bilirubin
C. Fluorine inhalations C. Blood sodium
D. Sealant application D. Uric acid
E. Introduce more vegetables to the diet E. Fibrinogen
Крок 2 Medicine (англомовний варiант, iноземнi студенти) 2015 рiк 5

35. A child was taken to a hospital with focal treatment is going to last for another 1-2
changes in the skin folds. The child was anxi- months. Who is authorised to extend the
ous during examination, examination revealed duration of medical certificate for this patient?
dry skin with solitary papulous elements and
ill-defined lichenification zones. Skin erupti- A. Medical advisory commission after medico-
on was accompanied by strong itch. The child social expert commission examination
usually feels better in summer, his condition B. Medical superintendent
is getting worse in winter. The child has been C. Medical advisory commission after inpatient
on bottle feeding since he was 2 months old. treatment
He has a history of exudative diathesis. His D. District doctor by agreement with a
grandmother on his mother’s side has bronchi- department chief
al asthma. What is the most likely diagnosis? E. Medico-social expert commission

A. Atopic dermatitis 40. A 30-year-old woman complains of pain


B. Contact dermatitis in the heart area ("aching, piercing pain")
C. Seborrheal eczema that arises primarily in the morning hours in
D. Strophulus autumn and spring. Pain irradiates into the
E. Urticaria neck, back, abdomen and is attended by rapid
heart rate and low vital tonus. This conditi-
36. An electro-gas welding operator worki- on occurs independently from physical exerti-
ng at a machine workshop performs welding on. In the evening her condition improves.
and cutting of metal, which is accompanied by Somatic and neurologic state and ECG have no
intense UV-radiation. His welding station is pathologies. What pathology is likely to result
equipped with effective mechanical ventilati- in such clinical presentation?
on. What occupational disease is most likely to
develop in the electro-gas welding operator? A. Somatized depression
B. Rest angina pectoris
A. Photoelectric ophthalmia C. Pseudoneurotic schizophrenia
B. Heatstroke D. Somatoform autonomic dysfunction
C. Vegetative-vascular dystonia E. Hypochondriacal depression
D. Chronic overheating
E. Pneumoconiosis 41. A 59-year-old female patient attended
a maternity welfare clinic with complains
37. 2 weeks after recovering from tonsillitis an of bloody discharge from the genital tracts.
8-year-old boy developed edemas of face and Postmenopause is 12 years. Vaginal exami-
lower limbs. Objectively: the patient is in grave nation revealed that external genital organs
condition, BP - 120/80 mm Hg. Urine is of dark had signs of age involution, uterus cervix
brown colour. Oliguria is present. On urine was not erosive, small amount of bloody di-
analysis: specific gravity - 1,015, protein - 1,2 scharge came from the cervical canal. Uterus
g/l, RBCs are leached and cover the whole vi- is of normal size, uterine appendages are
sion field, granular casts - 1-2 in the vision field, unpalpable. Fornices were deep and painless.
salts are represented by urates (large quantity). What method should be applied for the di-
What is the most likely diagnosis? agnosis specification?
A. Acute glomerulonephritis with nephritic A. Separated diagnosic curretage
syndrome B. Laparoscopy
B. Acute glomerulonephritis with nephrotic C. Puncture of abdominal cavity through
syndrome posterior vaginal fornix
C. Acute glomerulonephritis with nephrotic D. Extensive colposcopy
syndrome, hematuria and hypertension E. Culdoscopy
D. Acute glomerulonephritis with isolated
urinary syndrome 42. A 41-year-old patient, a hunter, complai-
E. Nephrolithiasis ns of heaviness in the right subcostal area.
No other complaints recorded. Anamnesis
38. A 48-year-old patient has the followi- states causeless urticaria and skin itching that
ng symptoms: diffuse enlargement of thyroid occurred 1 year ago. Objectively: liver margin
gland, exophthalmus, weight loss up to 4 kg wi- is rounded, painless, and can be palpated 3
thin 2 months, excessive sweating. Objectively: cm below the costal arch. No other pathology
heart rate 105/min, BP - 180/70 mm Hg. Stool is was detected during physical examination of
normal. What therapy is advisable in the given the patient. Body temperature is normal. X-ray
case? reveals a hemispherical protrusion in the right
cupula of the diaphragm. What disease can be
A. Mercazolil (Thiamazole) suspected in the given case?
B. Potassium iodide
C. Propranolol A. Hydatid disease of liver
D. Iodomarin B. Liver abscess
E. Thyroxin C. Hepatocellular carcinoma
D. Metastatic tumor
39. A patient is on sick leave for 4 months E. Subdiaphragmatic abscess
continuously from the date of injury. The
Крок 2 Medicine (англомовний варiант, iноземнi студенти) 2015 рiк 6

43. A 14-year-old girl has been presenting wi- Objectively: focal neurological symptoms
th irritability and tearfulness for about a year. were not found. Pronounced meningeal
A year ago she was also found to have diffuse symptoms were revealed. BP - 120/80 mm
enlargement of the thyroid gland (II grade). Hg. According to clinical and liquorologi-
This condition was regarded as a pubertal cal findings the patient was diagnosed wi-
manifestation, the girl did not undergo any th subarachnoid haemorrhage. After admi-
treatment. The girl’s irritability gradually gave nistration of dehydrants the patient’s condi-
place to a complete apathy. The girl got tion somewhat improved. What is the main
puffy face, soft tissues pastosity, bradycardia, component of further emergency care?
constipations. Skin pallor and gland density
progressed, the skin got a waxen hue. What di- A. Coagulants
sease may be assumed? B. Anticoagulants
C. Antiaggregants
A. Autoimmune thyroiditis D. Fibrinolytics
B. Diffuse toxic goiter E. Corticosteroids
C. Thyroid carcinoma
D. Subacute thyroiditis 48. A 42-year-old man has undergone gastric
E. Juvenile basophilism analysis. Free hydrochloric acid is absent
at all phases. On endoscopy: pallor, gastric
44. A 26-year-old woman, who delivered a child mucosa loss, gastric folds are smoothed out.
7 months ago, has been suffering from nausea, On microscopy: glands atrophy with intestinal
morning vomiting, sleepiness for the last 2 metaplasia. What disease can be characterized
weeks. She breastfeeds the child, menstruation by such presentation?
is absent. She has not applied any contracepti-
ves. What method should be applied in order A. Chronic gastritis type A
to specify her diagnosis? B. Chronic gastritis type B
C. Chronic gastritis type C
A. Ultrasonic examination D. Menetrier’s disease
B. X-ray of small pelvis E. Stomach cancer
C. Palpation of mammary glands and pressing-
out of colostrum 49. A 24-year-old woman, teacher by professi-
D. Bimanual vaginal examination on, complains of dizziness and heart pain
E. Speculum examination irradiating to the left nipple. Pain is not associ-
ated with physical activity and cannot be reli-
45. Over a current year among workers of an eved by nitroglycerin, it abates after taking
institution 10% have not been ill a single time, Valocordin and lasts for an hour or more. The
30% have been ill once, 15% - twice, 5% - 4 patient has a nearly 2-year history of this di-
times, the rest - 5 and more times. What is the sease. Objectively: Heart rate - 76 bpm. BP
percentage of workers belonging to the I health - 110/70 mm Hg. Heart borders are normal,
group? heart sounds are clear. The ECG shows respi-
ratory arrhythmia. X-ray of the cervicothoracic
A. 55% spine shows no pathology. Lungs, abdomen are
B. 10% unremarkable. What changes in blood formula
C. 40% can be expected?
D. 60%
E. 22% A. No changes
B. Leukocytosis
46. A 34-year-old patient after vacation in C. Thrombocytopenia
Crimea has developed severe pain in her D. Leukemic hiatus
elbow joints, dyspnea and weakness. Body E. Increased ESR
temperature is 37, 6o C , skin pallor and
erythema on the cheeks and bridge of nose 50. Chief physician of a polyclinic tasked a di-
are observed, lip mucosa is ulcerated. The joi- strict doctor with determining the pathologi-
nts are not visibly deformed, movement of the cal prevalence of disease N in his district.
right elbow joint is restricted. Pleura friction What document allows to estimate the disease
sound is detected in the lungs on the right side prevalence in the population of a medical di-
below the angle of scapula. Heart sounds are strict?
dull, tachycardia, gallop rhythm, heart rate -
114/min, BP - 100/60 mm Hg. The most likely A. Prophylactic examinations register
diagnosis is: B. Statistic coupons (+)
C. Statistic coupons (-)
A. Systemic lupus erythematosus D. Statistic coupons (+) and (-)
B. Rheumatic carditis E. Vouchers for medical appointments
C. Rheumatoid arthritis
D. Infectious allergic myocarditis 51. A 32-year-old woman complains of di-
E. Dry pleurisy zziness, headache, palpitation, tremor. For
the last several months she has been under
47. Several hours before, a 28-year-old pati- outpatient monitoring for increased arterial
ent suddenly developed acute headache and pressure. Recently such attacks have become
repeated vomiting, then lost consciousness. more frequent and severe. Objectively: skin
Крок 2 Medicine (англомовний варiант, iноземнi студенти) 2015 рiк 7

is covered with clammy sweat, tremor of the A. Coliphage number


extremities is present. Heart rate - 110/min, BP B. Colibacillus index
- 220/140 mm Hg. Heart sounds are weakened. C. Oxidability
In blood: WBCs - 9, 8 · 109 /l, ESR - 22 mm/h. D. Non-icteric leptospirosis agent
Blood glucose - 9,8 millimole/l. What disease is E. Fecal coliform bacteria index
the most likely cause of this crisis? 56. A 25-year-old patient was delivered to
A. Pheochromocytoma an infectious diseases unit on the 3rd day
B. Essential hypertension of illness with complaints of headache, pain
C. Preeclampsia in lumbar spine and gastrocnemius muscles,
D. Primary hyperaldosteronism high fever, chill. Objectively: condition of
E. Diabetic glomerulosclerosis moderate severity. Scleras are icteric. Pharynx
is hyperemic. Tongue is dry with dry brown
52. A 45-year-old patient complains of pain coating. Abdomen is distended. Liver is
in the epigastric region, left subcostal area, enlarged by 2 cm. Spleen is not enlarged.
abdominal distension, diarrhea, loss of wei- Palpation of muscles, especially gastrocnemi-
ght. He has been suffering from this condition us muscles, is painful. Urine is dark in colour.
for 5 years. Objectively: tongue is moist with Stool is normal in colour. The most likely di-
white coating near the root; deep palpation of agnosis is:
abdomen reveals slight pain in the epigastric
region and Мауо-Robson’s point. Liver is pai- A. Leptospirosis
nless and protrudes 1 cm from the costal arch. B. Viral hepatitis type A
Spleen cannot be palpated. What disease can C. Malaria
be primarily suspected? D. Infectious mononucleosis
E. Yersiniosis
A. Chronic pancreatitis
B. Atrophic gastritis 57. A patient suffering from acute
C. Peptic stomach ulcer posttraumatic pain received an injection of
D. Chronic cholecystitis morphine that brought him a significant relief.
E. Chronic enteritis Which of the following mechanisms of action
provided for antishock effect of morphine in
53. A 58-year-old woman complains of this patient?
spontaneous bruises, weakness, bleeding gums,
dizziness. Objectively: the mucous membranes A. Stimulation of opiate receptors
and skin are pale with numerous hemorrhages B. Block of central cholinergic receptors
of various time of origin. Lymph nodes are not C. Stimulation of benzodiazepine receptors
enlarged. Heart rate - 100/min, BP- 110/70 D. Inhibition of dopamine mediation
mm Hg. There are no changes of internal E. Intensification of GABA-ergic reactions
organs. Blood test results: RBC - 3,0·1012 /l, 58. A 9-month-old child presents with fever,
Нb - 92 g/l, colour index - 0,9, anisocytosis, poi- cough, dyspnea. The symptoms appeared 5
kilocytosis, WBC - 10·109 /l, eosinophiles - 2%, days ago after a contact with a person havi-
stab neutrophiles - 12%, segmented neutrophi- ng URTI. Objectively: the child is in grave
les - 68%, lymphocytes - 11%, monocytes - 7%, condition. Temperature of 38o C , cyanosis of
ESR - 12 mm/h. What laboratory test should be nasolabial triangle is present. Respiration rate
determined additionally to make a diagnosis? - 54/min, nasal flaring while breathing. There
was percussion dullness on the right below the
A. Platelets scapula angle, and tympanic sound over the
B. Reticulocytes rest of lungs. Auscultation revealed bilateral fi-
C. Clotting time ne moist rales (crackles) predominating on the
D. Osmotic resistance of erythrocytes right. What is the most likely diagnosis?
E. Fibrinogen
A. Acute pneumonia
54. After examination a 46-year-old patient was B. URTI
diagnosed with left breast cancer T2N2M0, cli- C. Acute laryngotracheitis
nical group II-a. What will be the treatment D. Acute bronchitis
plan for this patient? E. Acute bronchiolitis
A. Radiation therapy + operation + 59. A 50-year-old patient complains about havi-
chemotherapy ng pain attacks in the right subcostal area for
B. Operation only about a year. The pain arises mainly after
C. Operation + radiation therapy taking fatty food. Over the last week the
D. Radiation therapy only attacks occurred daily and became more pai-
E. Chemotherapy only nful. On the 3rd day of hospitalization the pati-
55. In river-side urban community there was an ent presented with icteritiousness of skin and
outbreak of hepatitis type A possibly spread by scleras, light-coloured feces and dark urine. In
water. What indexes of river water quality can blood: neutrophilic leukocytosis - 13, 1 · 109 /l,
confirm this theory? ESR - 28 mm/h. What is the most likely di-
agnosis?
Крок 2 Medicine (англомовний варiант, iноземнi студенти) 2015 рiк 8

A. Chronic calculous cholecystitis other day. Two days ago the patient arrived
B. Chronic recurrent pancreatitis from Africa. Objectively: pale skin, subicteric
C. Fatty degeneration of liver sclera, significantly enlarged liver and spleen.
D. Chronic cholangitis, exacerbation stage What is the cause of fever attacks in this di-
E. Hypertensive dyskinesia of gallbladder sease?
60. A 52-year-old woman complains of face di- A. Erythrocytic schizogony
stortion. It appeared 2 days ago after exposure B. Tissue schizogony
to cold. Objectively: body temperature is C. Exotoxin of a causative agent
38, 2o C . Face asymmetry is present. Frontal D. Endotoxin of a causative agent
folds are flattened. Left eye is wider than the ri- E. Gametocytes
ght one and cannot close. Left nasolabial fold is
flattened, mouth corner is lowered. Examinati- 65. A 49-year-old patient 2 years ago was di-
on revealed no other pathology. Blood count: agnosed with stage 1 silicosis. He complains of
leukocytes - 10 · 109 /l, ESR - 20 mm/h. What is increased dyspnea and pain in the infrascapular
the most probable diagnosis? regions. X-ray shows diffuse intensification
and distortion of lung pattern with numerous
A. Facial neuritis nodular shadows 2-4 mm in diameter. Hardeni-
B. Trigeminus neuralgia ng of right interlobar pleura is detected. Lung
C. Hemicrania (migraine) roots are hardened and expanded. What X-ray
D. Ischemic stroke type of pneumosclerosis does the patient have?
E. Brain tumour
A. Nodular
61. A 28-year-old female patient with a six-year B. Interstitial
history of Raynaud’s syndrome has recently C. Interstitial-nodular
developed pain in the small joints of hands, di- D. Nodal
fficult movement of food down the esophagus. E. Tumor-like
What disease can be suspected in this case?
66. A 24-year-old patient complains of general
A. Systemic scleroderma weakness, dizziness, body temperature ri-
B. Periarteritis nodosa se up to 37, 5o C , sore throat, neck edema,
C. Rheumatoid arthritis enlargement of submaxillary lymph nodes.
D. Systemic lupus erythematosus Objectively: mucous membrane of oropharynx
E. Pseudotrichiniasis is edematic and cyanotic, tonsils are enlarged
and covered with films that spread beyond
62. An 8-year-old boy, who has been sufferi- the tonsils and cannot be easily removed.
ng from diabetes mellitus for 3 years, was What is the leading mechanism of this illness
delivered to a hospital in a condition of development?
hyperglycemic coma. Primary dose of insulin
should be prescribed basing on the following A. Action of bacterial exotoxin
calculation: B. Action of bacterial endotoxin
C. Allergic
A. 0,1-0,2 units/kg of body mass per hour D. Accumulation of suboxidated products
B. 0,05 units/kg of body mass per hour E. Bacteriemia
C. 0,2-0,3 units/kg of body mass per hour
D. 0,3-0,4 units/kg of body mass per hour 67. A 40-year-old woman with a history of
E. 0,4-0,5 units/kg of body mass per hour combined mitral valve disease with predomi-
nant stenosis complains of dyspnea, asthma
63. A 43-year-old female patient complains of attacks at night, heart problems. At present,
rash on the skin of her right leg, pain, weakness, she is unable to do even easy housework. What
body temperature rise up to 38o C . The disease is the optimal tactics of the patient treatment?
is acute. Objectively: there is an edema on the
skin of her right leg in the foot area, a well- A. Mitral commissurotomy
defined bright red spot in the irregular shape B. Implantation of an artificial valve
of flame tips, which is hot to touch. There are C. Antiarrhythmia therapy
isolated vesicles in the focus. What is your D. Treatment of heart failure
provisional diagnosis? E. Antirheumatic therapy
A. Erysipelas 68. A 30-year-old woman with a long history
B. Microbial eczema of chronic pyelonephritis complains of consi-
C. Contact dermatitis derable weakness, sleepiness, decrease in di-
D. Toxicoderma uresis down to 100 ml per day. BP is 200/120
E. Haemorrhagic vasculitis mm Hg. In blood: creatinine - 0,62 millimole/l,
hypoproteinemia, albumines - 32 g/l, potassi-
64. A 47-year-old patient came to see a doctor um - 6,8 millimole/l, hypochromic anemia,
on the 7th day of disease. The disease is increased ESR. What is the first step in the
acute: after the chill body temperature rose patient treatment tactics?
to 40o C and lasted up to 7 hours, then dropped
abruptly, which caused profuse sweat. There
were three such attacks occurring once every
Крок 2 Medicine (англомовний варiант, iноземнi студенти) 2015 рiк 9

A. Haemodialysis 73. A 12-year-old cild has been hit on the


B. Antibacterial therapy stomach. The patient is in moderately grave
C. Enterosorption condition, has a forced position in bed. The
D. Haemosorption skin is pale, heart rate - 122/min. Stress on
E. Blood transfusion the left costal arch causes a slight pain. Wei-
nert and Kulenkampff symptoms are positive.
69. A 43-year-old patient had right-sided deep Macroscopically the urine is unchanged. What
vein thrombosis of iliofemoral segment 3 years is the most likely diagnosis?
ago. Now he is suffering from the sense of
heaviness, edema of the lower right extremi- A. Spleen rupture, abdominal bleeding
ty. Objectively: moderate edema of shin, brown B. Left kidney rupture, retroperitoneal
induration of skin in the lower third of shin, hematoma
varix dilatation of superficial shin veins are C. Rupture of the pancreas
present. What is the most probable diagnosis? D. Liver rupture, abdominal bleeding
E. Rupture of a hollow organ, peritonitis
A. Postthrombophlebitic syndrome, varicose
form 74. Establishments participating in medical
B. Acute thrombosis of right veins examinations include: medical and preventi-
C. Lymphedema of lower right extremity ve treatment facility, hygiene and preventive
D. Parkes-Weber syndrome treatment facility, sociomedical expert commi-
E. Acute thrombophlebitis of superficial veins ttees, Ministry of Defence medical commi-
ttees, Ministry of Domestic Affairs medical
70. A 28-year-old woman has bursting pain committees, forensic medicine agency, etc.
in the lower abdomen during menstruati- Specify what service deals with sociomedical
on; chocolate-like discharges from vagina are assessment of temporary disability:
observed. It is known from the anamnesis that
the patient suffers from chronic adnexitis. Bi- A. Medical and preventive treatment facility
manual examination revealed a tumour-like B. Hygiene and preventive treatment facility
formation of heterogenous consistency 7х7 cm C. Sociomedical expert committees
large to the left from the uterus. The formation D. Ministry of Defence medical committees
is restrictedly movable, painful when moved. E. Ministry of Domestic Affairs medical commi-
What is the most probable diagnosis? ttees
A. Endometrioid cyst of the left ovary 75. An unconscious 35-year-old patient has
B. Follicular cyst of the left ovary been delivered by an ambulance to the intensi-
C. Fibromatous node ve care unit. Objectively: the patient is in
D. Exacerbation of chronic adnexitis semicoma. Moderate mydriasis is present. The
E. Tumour of sigmoid colon reaction of pupils to light is reduced. The pati-
ent is non-responsive to verbal instructions. BP
71. A 6-year-old child complains of frequent li- is150/100 mm Hg, tachycardia is present. Blood
quid stool and vomiting. On the 2nd day of contains methanol. What antidote should be
desease the child presented with inertness, administered?
temperature rise up to 38, 2o C , heart rate -
150 bpm, scaphoid abdomen, palpatory painful A. Ethanol
sigmoid colon, defecation 10 times a day with B. Unithiol
liquid, scarce stool with mucus and streaks of C. Thiamine chloride
green. What is a provisional diagnosis? D. Tavegil (Clemastine)
E. Naloxone
A. Shigellosis
B. Salmonellosis 76. After lifting a load, a 36-year-old male pati-
C. Escherichiosis ent has experienced a severe pain in the lumbar
D. Intestinal amebiasis region, which spread to the right leg and was
E. Yersiniosis getting worse when he moved his foot or
coughed. Objectively: the long back muscles on
72. A 65-year-old patient complains of pain in the right are strained. Achilles jerk is reduced
the lumbar spine, moderate disuria. He has on the right. There is a pronounced tenderness
been suffering from this condition for about of paravertebral points in the lumbar region.
half a year. Prostate volume is 45 cm3 (there The straight leg raise (Lasegue’s sign) is posi-
are hypoechogenic nodes in both lobes, capsule tive on the right. What additional tests should
invasion). Prostate-specific antigen is 60 ng/ml. be performed in the first place?
Prostate biopsy revealed an adenocarcinoma.
Which of the supplementary examination A. Radiography of the spinal column
methods will allow to determine the stage of B. Computed tomography
neoplastic process in this patient? C. Magnetic resonance tomography
D. Electromyography
A. Computer tomography of small pelvis E. Lumbar puncture
B. Lumbar spine X-ray
C. Excretory urography 77. A patient is active, lively, fussy. He resi-
D. Bone scintigraphy sts the examination. His speech is fast and
E. Chest X-ray loud, his statements are spontaneous and
Крок 2 Medicine (англомовний варiант, iноземнi студенти) 2015 рiк 10

inconsequent. Specify the psychopathological diagnosis is:


state of this patient:
A. Uterine rupture
A. Psychomotor agitation B. Initial uterine rupture
B. Catatonic agitation C. Threatened uterine rupture
C. Delirium D. Premature detachment of normally positi-
D. Behavioral disorder oned placenta
E. Paranoid syndrome E. Compression of inferior pudendal vein
78. A 4-year-old boy was vaccinated in vi- 82. A 26-year-old female patient has an 11-
olation of his vaccination schedule. There year history of rheumatism. Four years ago
are complaints of pain in the throat during she suffered 2 rheumatic attacks. Over the last
swallowing, headache, inertness, fever. Objecti- 6 months there have been paroxysms of atri-
vely: the child is pale; anterior cervical lymph al fibrillation every 2-3 months. What option
nodes are enlarged; tonsils are swollen, with of antiarrhythmic therapy or tactics should be
cyanotic hyperemy, and covered with grey- proposed?
white coating that cannot be removed; if it is
forcibly removed, tonsils bleed. The most li- A. Prophylactic administration of cordarone
kely diagnosis is: B. Immediate hospitalization
C. Defibrillation
A. Oropharyngeal diphtheria D. Lidocaine administration
B. Lacunar tonsillitis E. Heparin administration
C. Pseudomembranous (Vincent’s) tonsillitis
D. Infectious mononucleosis 83. A 47-year-old female patient complains of
E. Follicular tonsillitis cough with purulent sputum, pain in the lower
left chest, periodical body temperature rise.
79. A 26-year-old male patient complains of pi- She has been suffering from this condition for
ercing pain during breathing, cough, dyspnea. about 10 years. Objectively: "drumstick"distal
Objectively: to - 37, 3o C , respiration rate is phalanges. What examination would be the
19/min, heart rate is 92/min; BP is 120/80 mm most informative for making a diagnosis?
Hg. Vesicular respiration is observed. In the
inferolateral parts of chest auscultation in both A. Bronchography
inspiration and expiration phase revealed noi- B. Bronchoscopy
se that was getting stronger at phonendoscope C. Survey radiograph of lungs
pressing and could be still heard after cough. D. Pleural puncture
ECG showed no pathological changes. What is E. Bacteriological analysis of sputum
the most likely diagnosis?
84. A patient is 60-year-old, retired, worked
A. Acute pleuritis as deputy director of a research institute.
B. Intercostal neuralgia Behavioural changes appeared 2 years ago
C. Subcutaneous emphysema after the death of her husband: she stopped
D. Spontaneous pneumothorax looking after herself and leaving the house;
E. Pericarditis sicca then she stopped to clean the apartment and
cook. Mental status: temporal disorientation.
80. A child is 3-week-old. Since his birth peri- The patient does not understand many of the
odical vomiting is observed occuring several questions, is confused; does not know how
minutes after feeding. Vomit mass does not to cook soup or fasten a button. Her speech
exceed previous feeding volume. Body mass is is characterized by stumbling and logoclonia.
appropriate to the child’s age. What is the most She does not recognize doctors, fellow pati-
likely cause for the symptoms described? ents. She cries a lot but can not explain the
reason for tears. What is the mechanism of this
A. Pylorospasm pathology?
B. Esophageal chalasia
C. Adrenogenital syndrome A. Atrophy of the cerebral cortex
D. Pylorostenosis B. Atherosclerotic changes in cerebral vessels
E. Esophageal achalasia C. Serotonin deficiency
D. Impaired conversion of dopamine to
81. A 30-year-old parturient woman was deli- noradrenaline
vered to a maternity hospital with full-term E. Disorder of melatonin metabolism
pregnancy. She complains of severe lancinati-
ng pain in the uterus that started 1 hour ago, 85. An 8-year-old girl periodically has sudden
nausea, vomiting, cold sweat. Anamnesis states short-term heart pain, sensation of chest
cesarean section 2 years ago. Uterine contracti- compression, epigastric pain, dizziness, vomiti-
ons stopped. Skin and mucous membranes are ng. Objectively: the patient is pale, respiratory
pale. Heart rate is 100/min, BP is 90/60 mm Hg. rate - 40/min, jugular pulse is present. Heart
Uterus has no clear margins, is sharply painful. rate - 185 bpm, of poor volume. BP - 75/40
No heartbeat can be auscultated in the fetus. mm Hg. ECG taken during an attack shows
Moderate bloody discharge from the uterus ectopic P waves, QRS wave is not deformed.
can be observed. Uterus cervix is 4 cm open. At the end of an attack a compensatory pause
Presenting part is not visible. The most likely is observed. The most likely cause of the attack
Крок 2 Medicine (англомовний варiант, iноземнi студенти) 2015 рiк 11

is: A. Thermograph
B. Alcohol thermometer
A. Paroxysmal atrial tachycardia C. Mercury thermometer
B. Sinus tachycardia D. August’s psychrometer
C. Paroxysmal ventricular tachycardia E. Assmann psychrometer
D. Complete AV-block
E. Atrial fibrillation 91. Against the background of angina a patient
has developed pain in tubular bones. Exami-
86. During examination a patient is unconsci- nation revealed generalized enlargement
ous, his skin is dry and hot, face hyperemia of lymph nodes, hepatolienal syndrome,
is present. The patient has Kussmaul’s respi- sternalgia. In blood: RBCs - 3, 6 · 1012 /l, Hb-
ration, there is also smell of acetone in the 87 g/l, thrombocytes - 45 · 109 /l, WBCs -
air. Symptoms of peritoneum irritation are
positive. Blood sugar is 33 millimole/l. What 13 · 109 /l, blasts - 87%, stab neutrophiles - 1%,
emergency actions should be taken? segmented neutrophiles - 7%, lymphocytes -
5%, ESR - 55 mm/h. What is the most likely
A. Intravenous infusion of short-acting insulin diagnosis?
B. Intravenous infusion of glucose along with
insulin A. Acute leukemia
C. Introduction of long-acting insulin B. Erythremia
D. Intravenous infusion of neohaemodesum C. Chronic lymphocytic leukemia
along with glutamic acid D. Chronic myeloid leukemia
E. Intravenous infusion of sodium chloride E. Multiple myeloma
saline 92. For a long time to eradicate weeds on
87. A patient complains of evaginations in the agricultural lands herbicides retaining in envi-
region of anus that appear during defecation ronment have been used. Point out the most
and need resetting. Examination with anoscope probable way of herbicides reaching human
revealed 1x1 cm large evaginations of mucosa organism from soil:
above the pectineal line. What is the most A. Soil-plants-human
probable diagnosis? B. Soil-microorganisms-human
A. Internal hemorrhoids C. Soil-animals-human
B. Acute paraproctitis D. Soil-protozoa-human
C. External hemorrhoids E. Soil-insects-human
D. Anal fissure 93. A 52-year-old patient complains of pain in
E. - the right part of her chest, dyspnea, cough wi-
88. During the newborn infant’s examinati- th a lot of albuminoid sputum emitting foul
on redness of the skin was detected, which smell of "meat slops". Objectively: the pati-
occurred immediately after birth and became ent’s condition is grave, cyanosis is observed,
the most pronounced on the second day of the breathing rate is 31/min, percussion sound
infant’s life. What provisional diagnosis can be above the right lung is shortened, auscultation
made? revealed various moist rales (crackles). What is
the most probable diagnosis?
A. Simple erythema
B. Toxic erythema A. Lung gangrene
C. Transient erythema B. Lung abscess
D. Erythema nodosum C. Pleura empyema
E. Erythema anulare centrifugum D. Multiple bronchiectasis
E. Chronic pneumonia
89. A baby was born by a young smoker. The
labour was complicated by uterine inertia, diffi- 94. A 15-year-old patient suffers from
cult delivery of the baby’s head and shoulders. headache, nasal haemorrhages, sense of lower
The baby’s Apgar score was 4. Which of the extremity coldness. Objectively: muscles of
following is a risk factor for a spinal cord shoulder girdle are developed, lower extremi-
injury? ties are hypotrophied. Pulsation on the pedal
and femoral arteries is sharply dampened. BP
A. Difficult delivery of the head and shoulders is 150/90 mm Hg, 90/60 in the legs. Systolic
B. Young age of the mother murmur can be auscultated above carotid
C. Pernicious habits arteries. What is the most probable diagnosis?
D. Uterine inertia
E. Chronic hypoxia A. Aorta coarctation
B. Aorta aneurism
90. In order to study impact of microclimate on C. Aortal stenosis
the human organism it is necessary to make D. Aortal insufficiency
systematic observation of air temperature E. Coarctation of pulmonary artery
during the period of 3 days. Choose a devi-
ce that will allow to make the most precise 95. A patient with frostbite of both feet was
temperature records: delivered to an admission ward. What actions
should be taken?
Крок 2 Medicine (англомовний варiант, iноземнi студенти) 2015 рiк 12

A. Disturbed urine outflow from the kidney


A. To apply a bandage, to introduce vasodilating B. Increase in urine specific gravity
medications C. Ureteric twists
B. To administer cardiac medications D. Destruction of renal glomerules
C. To put feet into hot water E. Renal artery spasm
D. To rub feet with snow
E. To apply an alcohol compress 101. A 26-year-old woman has attended
maternity center complaining of her inability
96. A patient in a clinical death condition is bei- to become pregnant despite 3 years of regular
ng resuscitated through mouth-to-mouth artifi- sex life. Examination revealed the followi-
cial pulmonary ventilation and external cardi- ng: increased body weight; male-type pubic
ac massage. A doctor noticed that air does not hair; excessive pilosis of thighs; ovaries are
flow into the patient’s airways and his head and dense and enlarged; basal body temperature
torso are positioned at the same level. Why is is monophasic. The most likely diagnosis is:
artificial respiration not effective in the given
case? A. Ovaries sclerocystosis
B. Inflammation of uterine appendages
A. Tongue retraction C. Adrenogenital syndrome
B. Low breathing volume D. Premenstrual syndrome
C. External cardiac massage E. Gonadal dysgenesis
D. Probe is absent from stomach
E. The patient’s mouth is too small 102. A workshop resident doctor makes a li-
st of workers, who are often ill, for special
97. Hygienic assessment of a sample taken supervision. He takes into account the number
from the batch of grain revealed 2% of grains of etiologically connected cases resulting in
infected with microscopic Fusarium fungi. On temporary disability that occurred in the span
the grounds of laboratory analyses this batch of a year for each worker. How many such cases
of grain should be: should a worker have to be included in this
group?
A. Sold without restrictions
B. Tested for toxicity A. 4 and more
C. Used for forage production B. 1 and more
D. Used for ethanol production C. 2 and more
E. Destroyed D. 3 and more
E. 6 and more
98. Residents of an industrial community si-
tuated near a plant suffer from increased 103. A 28-year-old woman complains of
morbidity rate caused by nervous and endocri- increased intermenstrual periods up to 2
ne system conditions and kidney diseases. months, hirsutism. Gynaecological examinati-
Blood test: decrease of sulfhydric groups on revealed that the ovaries were enlarged,
content in blood. The pathologies developed painless, compact, uterus had no peculariti-
can be caused by environment being polluted es. Pelvic ultrasound revealed that the ovari-
by the following: es were 4-5 cm in diameter and had multiple
enlarged follicles on periphery. X-ray of skull
A. Mercury base showed that sellar region was dilated.
B. Cadmium What is the most probable diagnosis?
C. Boron
D. Chromium A. Stein-Leventhal syndrome (Polycystic ovary
E. Lead syndrome)
B. Algodismenorrhea
99. Various population groups mortality has C. Sheehan’s syndrome
been studied for a long time; territorial distri- D. Premenstrual syndrome
bution of population and resulting changes in E. Morgagni-Stewart syndrome
mortality trends are considered. What statstical
method can be applied? 104. A woman consulted a therapeutist
about fatigability, significant weight loss,
A. Time series analysis weakness, loss of appetite. She has been having
B. Standardized ratio analysis amenorrhea for 8 months. A year ago she born
C. Correlation and regression analysis a full-term child. Haemorrhage during labour
D. Assessment of parameter difference reliabi- made up 2 l. She got blood and blood substi-
lity tute transfusions. What is the most probable
E. Analysis of relative values diagnosis?
100. A 45-year-old patient with urolithiasis had A. Sheehan’s syndrome
an attack of renal colic. What is the mechanism B. Stein-Leventhal syndrome
of the attack development? C. Shereshevsky-Turner’s syndrome
D. Homological blood syndrome
E. Vegetovascular dystonia
105. The correlation between service record
Крок 2 Medicine (англомовний варiант, iноземнi студенти) 2015 рiк 13

and eosinophiles concentration in blood was A. Alimentary constitutive obesity, I stage,


studied in workers at dyeing shops of texti- android type
le factories. What index will be the most B. Itsenko-Cushing hypothalamic obesity, II
informative for the analysis of this data? stage, gynoid type
C. Alimentary constitutive obesity, III stage,
A. Correlation factor gynoid type
B. Student’s criterion D. Alimentary constitutive obesity, II stage,
C. Standardized index android type
D. Fitting criterion E. Itsenko-Cushing hypothalamic obesity, I
E. Sign index stage, android type
106. A 39-year-old woman suffering from schi- 110. A parturient woman is 27 year old, it was
zophrenia constantly strains to hear somethi- her second labour, delivery was at full-term,
ng insisting that "there is a phone connecti- normal course. On the 3rd day of postpartum
on in her brain and she hears her brother’s period body temperature is 36, 8o C , heart rate
voice demanding that she come back home". - 72/min, BP - 120/80 mm Hg. Mammary glands
The patient is anxious, suspicious, constantly are moderately swollen, nipples are clean.
looking around. Specify the psychopathologic Abdomen is soft and painless. Fundus of uterus
syndrome. is 3 fingers below the umbilicus. Lochia are
bloody, moderate. What is the most probable
A. Hallucinatory diagnosis?
B. Anxiety
C. Paranoid A. Physiological course of postpartum period
D. Paraphrenic B. Subinvolution of uterus
E. Depressive C. Postpartum metroendometritis
D. Remnants of placental tissue after labour
107. A 3-month-old girl has rhinitis, dyspnea, E. Lactostasis
dry cough. She has been sick for 2 days.
Objectively: pale skin, acrocyanosis, hypopnoe; 111. A 40-year-old patient suffers from
breathing rate is 80/min; over the whole influenza. On the 5th day of illness there are
pulmonary surface there is vesiculotympanitic pain behind sternum, cough with sputum,
(bandbox) resonance observed with numerous inertness. Temperature is 39, 5o C . Face is
bubbling rales (crackles). The most likely di- pale. Mucosa of conjunctivas and pharynx is
agnosis is: hyperemic. Heart rate is 120/min, breathing
rate is 38/min. In the lower lung segments
A. Acute bronchiolitis shortening of percussion sound and moist rales
B. Pneumonia (crackles) can be detected. What additional
C. Mucoviscidosis investigation should be performed first of all to
D. Foreign body in airways specify the diagnosis?
E. Acute bronchitis
A. Lung X-ray
108. A 30-year-old patient was hospitalised in B. ECG
an intensive care unit with a diagnosis of multi- C. Heart US
ple bee stings. Skin is pale and covered with D. Mantoux test
cold sweat. Pulse can be palpated only at the E. Spirography
carotid arteries and is 110/min; breating rate is
24/min, rhytmical, weakened. What drug must 112. A 45-year-old patient complains of body
be administered immediately? temperature rise up to 40o C , general weakness,
headache, painfulness and spastic muscle
A. Adrenaline hydrochloride intravenously contractions around the wound in the shin. He
B. Prednisolone intravenously received this wound 5 days ago when worki-
C. Adrenaline hydrochloride intramuscularly ng in his garden. He requested no medical
D. Dopamine intravenously care back then. What wound infection can be
E. Tavegyl (Clemastine) intravenously suspected?
109. A 24-year-old patient complains of gaining A. Tetanus
body mass and increased appetite. Objectively: B. Anthrax
built of hypersthenic type, body mass index C. Erysipelas
is 33,2 kg/m2 , waist circumference is 100 cm. D. Gram-positive
Waist to hips circumference ratio is 0,95. What E. Gram-negative
is the provisional diagnosis?
113. A 60-year-old woman started feeling
weakness, vertigo, rapid fatigability during the
last year. Recently there have been dyspnea
and paresthesia observed. Objectively: skin
and mucous membranes are pale and icteric.
Lingual papillae are smoothed out. Liver and
spleen are situated at the edge of costal arch.
Blood test: Hb- 70 g/l, erythrocytes - 1, 7·1012 /l,
blood color index - 1,2, macrocytes. What drug
Крок 2 Medicine (англомовний варiант, iноземнi студенти) 2015 рiк 14

can be prescribed on pathogenetic grounds? A. Acute bartholinitis


B. Furuncle of outer labia
A. Vitamin B12 C. Acute vulvovaginitis
B. Vitamin B6 D. Bartholin’s cyst
C. Ascorbic acid E. Carcinoma of vulva
D. Iron preparations
E. Vitamin B1 118. In a rural health care area there is a case of
child dying during the first month of his life. To
114. A 34-year-old patient complains of profuse analyse this situation, among other measures,
sweating at night, skin itching, weight loss an expert assessment of medical records is
(9 kg within the last 3 months). Examinati- performed. What medical document should be
on revealed malnutrition, skin pallor. Palpati- considered first?
on of neck and inguinal areas revealed dense
elastic lymph nodes of about 1 cm in diameter, A. Child development history
nonmobile, non-adhering to skin. What is the B. Infant development history
most probable diagnosis? C. Vaccination card
D. Outpatient medical record
A. Lymphogranulomatosis E. Child’s medical record
B. Chronic lymphadenitis
C. Lymphosarcoma 119. Examination of placenta revealed a defect.
D. Burkitt’s lymphoma An obstetrician performed manual investi-
E. Cancer metastases gation of uterine cavity, uterine massage.
Prophylaxis of endometritis in the postpartum
115. A 57-year-old woman complains of having period should involve the following actions:
a sensation of esophageal compresion, palpi-
tation, breathing difficulties when eating solid A. Antibacterial therapy
food, occasional vomiting with a full mouth, B. Instrumental revision of uterine cavity
"wet pillow"sign at night for the last 6 months. C. Haemostatic therapy
Objectively: body tempearture - 39o C , height - D. Contracting agents
168 cm, weight - 72 kg, heart rate - 76/min, BP- E. Intrauterine instillation of dioxine
120/80 mm Hg. X-ray revealed considerable di- 120. A 15-year-old boy has developed pain in
lation of esophagus and its constriction in the the area of his left knee joint. Objectively:
cardial part. What pathology is most likely to thigh soft tissues in the painful area are infi-
have caused dysphagia in this patient? ltrated, joint function is reduced. X-ray: there is
A. Achalasia cardiae a focus of destruction in the left distal femoral
B. Primary esophagism metaphysis, with periosteum detachment and
C. Hiatal hernia formation of Codman triangle in the bone
D. Esophageal carcinoma cortical layer at the margin of the defect. Chest
E. Reflux esophagitis X-ray reveals numerous microfocal metastases.
What pathology is the most likely to cause such
116. A 25-year-old man complains of pain presentation?
in the lower third of his left thigh, which
occurs both with and without physical exertion. A. Osteogenic sarcoma
Possibility of trauma is denied by the patient. B. Fibrosarcoma
Objectively: skin colour is normal; pastosity C. Chondrosarcoma
and pain can be detected with deep palpati- D. Ewing’s sarcoma
on; knee joint mobility is reduced. X-ray of E. Juxtacortical sarcoma
distal femoral metaphysis shows an area of 121. A 74-year-old man complains of pain in
destruction and spicule. Blood test: immature the left foot and dark color of the 1st toe. He
cells, no signs of inflammation. The most likely has been sick for 6 days. Objectively: heart
diagnosis is: rate is 84/min. Body temperature is 37, 8o C .
A. Osteogenic sarcoma The left foot resembles "chicken claw"due
B. Osteitis fibrosa cystica to soft tissues decreasing in volume, skin is
C. Chronic osteomyelitis dark red. The 1st toe is black, small amount of
D. Multiple myeloma dark substance is being discharged from under
E. Marble-bone disease the nail. Pulse over the peripheral arteries of
extremity is absent. The most likely diagnosis
117. A 37-year-old woman complains of sharp is:
pains in her external genitalia, edema of the
vulvar lips, pain when walking. Objectively: A. Gangrene
body temperature is 38, 7o C , heart rate is B. Phlegmon
98/min. Inside the right vulvar lip there is a C. Panaritium
dense, painful, tumor-like growth 5,0х4,5 cm in D. Erysipelas
size; skin and mucosa of the external genitalia E. Erysipeloid
are hyperemic, copious foul-smelling discharge 122. Head circumference of a 1-month-old boy
is observed. The most likely diagnosis is: with signs of excitement is 37 cm, prefontanel
is 2x2 cm large. After feeding the child regurgi-
tates small portions of milk; stool is normal
Крок 2 Medicine (англомовний варiант, iноземнi студенти) 2015 рiк 15

in its volume and composition. Muscle tone is


within norm. What is the most likely diagnosis? A. Diabetes mellitus type 2 with visceral
neuropathy
A. Pylorospasm B. Diabetes mellitus type 1 with ketoacidosis
B. Meningitis C. Diabetes mellitus type 1 with
C. Pylorostenosis encephalomyelopathy
D. Microcephaly D. Diabetes mellitus type 2 with nephropathy
E. Craniostenosis E. Diabetes mellitus type 2 with polyneuropathy
123. A 49-year-old patient complains of di- 127. A young woman suffering from seborrhea
fficulties when swallowing food, hoarse voi- oleosa has numerous light-brown and whi-
ce, weight loss. The patient has been suffering te spots on the skin of her torso and
from these symptoms for the last 3 months. The shoulders. The spots have clear margins,
symptoms are gradually progressing. Objecti- branny desquamation, no itching. What provi-
vely: the patient is emaciated, supraclavicular sional diagnosis can be made?
lymph nodes are enlarged. Esophagoscopy
revealed no pathologies of esophagus. What A. Pityriasis versicolor
investigation would be the most advisable in B. Torso dermatophytosis
the given case? C. Seborrheic dermatitis
D. Pityriasis rosea
A. Computed tomogram of chest and mediasti- E. Vitiligo
num
B. Lungs X-ray 128. 10 days after birth an infant developed
C. Multiplanar fluoroscopy of esophagus a sudden fever up to 38, 1o C . Objectively:
D. Radioisotopic examination of chest and the skin of navel, abdomen and chest is
mediastinum erythematous; there are multiple pea-sized
E. US of mediastinum blisters with no infiltration at the base; si-
ngle bright red moist erosions with epidermal
124. A 36-year-old male patient complains of fragments on the periphery. What is your provi-
having headache, obstructed nasal breathing, sional diagnosis?
purulent nasal discharge for 2 weeks. A month
before, he had his right maxillary premolar A. Epidemic pemphigus of newborn
filled. X-ray revealed an intense opacity of B. Syphilitic pemphigus
the right maxillary sinus. Diagnostic puncture C. Streptococcal impetigo
revealed a large amount of thick malodorous D. Vulgar impetigo
crumbly pus. What is the most likely diagnosis? E. Atopic dermatitis

A. Chronic suppurative odontogenic sinusitis 129. In a traffic accident two persons died. An
B. Acute purulent sinusitis appointed forensic medical expert was called
C. Chronic purulent sinusitis on duty to another accident location; therefore,
D. Chronic atrophic sinusitis according to the crininal law in force in Ukrai-
E. Tumor of the maxillary sinus ne, an investigator called in the following speci-
alist for examination of the location and bodi-
125. During a surgical operation necessitated es:
by the patient’s suffering from autoimmune
thyroiditis with concomitant multinodular goi- A. Nearest available medical doctor
ter the right lobe was removed and subtotal B. Medical assistant
resection of the left lobe was performed. C. Surgical nurse
What should be prescribed to the patient for D. Nearest available dispensing chemist
postoperative hypothyroidism prevention? E. Nearest available dental mechanic

A. L-thyroxin 130. A 35-year-old female patient has gained


B. Mercazolil (Thiamazole) 20 kg weight within a year with the normal di-
C. Iodomarin (Potassium iodide) et. She complains of chill, sleepiness, dyspnea.
D. Lithium preparations The patient’s mother and sister are corpulent.
E. Insulin Objectively: height - 160 cm, weight - 92 kg,
BMI - 35,9. Obesity is uniform, there are no
126. A 54-year-old patient has been suffering striae. The face is amimic. The skin is dry. The
from diabetes mellitus for 5 years, with diet tongue is thickened. Heart sounds are muffled.
being his only treatment. Within the last half Heart rate - 56/min, BP - 140/100 mm Hg. The
a year he lost 7 kg of body weight, complai- patient has constipations, amenorrhea for 5
ns of thirst, vertigo when raising from bed, months. TSH - 28 mkME/l (normal rate - 0,32-
decrease of erectile function, frequent stool, 5). Craniogram shows no pathology. What is
especially at night. Objectively: malnutrition, the etiology of obesity?
dry skin. BP in lying position is 160/90 mm
Hg; BP in standing position is 170/85 mm Hg. A. Hypothyroid
No edemas. Fasting plasma glucose level is B. Hypo-ovarian
12 mmol/l. Glycated hemoglobin accounts for C. Hypothalamic-pituitary
11%. Albumin excreted with urine is 20 mg per D. Alimentary and constitutive
day. The most likely diagnosis is: E. Hypercorticoid
Крок 2 Medicine (англомовний варiант, iноземнi студенти) 2015 рiк 16

131. A 45-year-old patient has been sufferi- A. Ileocecal invagination


ng from essential hypertension for 10 years. B. Gastrointestinal haemorrhage
After hot bath he suddenly developed sharp C. Wilm’s tumour
headache and recurrent vomiting. Objecti- D. Helminthic invasion
vely: pronounced meningeal symptoms; BP E. Pylorostenosis
is 180/110 mm Hg. He was hospitalised in a
neurology unit. What additional tests should 135. A 68-year-old patient complains of
be primarily prescribed? pain, slight swelling and creaking in the di-
stal joints of her fingers and knee joints.
A. Lumbar puncture, cerebrospinal fluid test She has been suffering from this condition
B. Ventriculopuncture for 4 years. Objectively: thickening of knee
C. Echoencephalography and interphalangeal joints, restricted mobi-
D. Rheoencephalography lity of these joints. Blood test: leukocytes -
E. Electroencephalography 5, 4 · 109 /l, ESR - 12 mm/h, fibrinogen - 3,5
g/l; Rheumatoid factor - 1:32; anti-streptolysin
132. A 77-years-old patient complains of inabi- O - 160 units; seromucoid - 0,20 units.; CRP+.
lity to urinate and bursting pain above his What mechanism of development is likely for
pubis. Acute onset of his condition occurred this condition?
12 hours ago. Objectively: overfilled urinary
bladder can be palpated above the pubis. On A. Glycosaminoglycanes synthesis disruption
rectal examination: prostate is enlarged, dense- B. Rheumatoid factor development
elastic, with clear margins and no nodules. C. Native DNA antibodies hyperproduction
Interlobar sulcus is pronounced. US examinati- D. Increased uric acid sunthesis
on: prostate volume is 120 cm3 , it protrudes E. Immune reaction to streptococcosis
into urinary bladder cavity, parenchyma is
homogeneous. Prostate specific antigen is 5 136. A 30-year-old male patient consulted a
ng/ml. What disease is the most likely to cause family doctor 2 months after he had been
acute urinary retention? operated for an open fracture of brachial bone.
Objectively: the patient’s condition is sati-
A. Prostatic hyperplasia sfactory; in the area of the operative wound
B. Prostate cancer there is a fistula with some purulent discharge,
C. Prostate sclerosis redness, fluctuation. X-ray image shows brachi-
D. Prostate tuberculosis al bone destruction with sequestra. What
E. Acute prostatitis complication arose in the postoperative peri-
od?
133. A patient with uterine fibromyoma si-
zed up to 8-9 weeks of pregnancy consulted A. Posttraumatic osteomyelitis
a gynaecologist about acute pain in the lower B. Hematogenic osteomyelitis
abdomen. Examination revealed pronounced C. Wound abscess
positive symptoms of peritoneal irritation, hi- D. Posttraumatic phlegmon
gh leukocytosis. Vaginal examination revealed E. Suture sinus
that the uterus was enlarged up to 9 weeks of
pregnancy due to the fibromatous nodes, one 137. A 40-year-old female patient complains of
of which was mobile and extremely painful. having a bulge on the anterior surface of neck
Appendages were not palpable. Discharges for 5 years. Objectively: Heart rate - 72 bpm,
were mucous, coming in moderate amounts. arterial pressure - 110/70 mm Hg, in the right
What is the treatment tactics? lobe of thyroid gland palpation reveals a mobi-
le 4x2 cm node, the left lobe is not palpable, the
A. Urgent surgery (laparotomy) basal metabolic rate is 6%. What is the most li-
B. Surveillance and spasmolytic therapy kely diagnosis?
C. Fractional diagnostic curettage of the uterine
cavity A. Nodular euthyroid goiter
D. Surgical laparoscopy B. Nodular hyperthyroid goiter
E. Surveillance and antibacterial therapy C. Riedel’s thyroiditis
D. Mixed euthyroid goiter
134. A 4-month-old child was admitted to E. The median cervical cyst
a surgical department 8 hours after the first
attack of anxiety. The attacks happen every 138. Estimation of community health level
10 minutes and last for 2-3 minutes, vomi- involved analysis of a report on diseases regi-
ting occurred once. Objectively: the child’s stered among the population of catchment area
condition is grave. Abdomen is soft, palpati- (reporting form 12). What index is calculated
on reveals a tumour-like formation in the right basing on this report?
iliac area. After rectal examination the doctor’s A. Common morbidity rate
finger was stained with blood. What is the most B. Index of pathological affection
probable diagnosis? C. Index of morbidity with temporary disability
D. Index of hospitalized morbidity
E. Index of basic non-epidemic morbidity
139. A tractor driver with the record of servi-
Крок 2 Medicine (англомовний варiант, iноземнi студенти) 2015 рiк 17

ce of 24 years has undergone palestesiometry vely: tuberous slightly movable lump can be
test (Vibration Sensitivity Measurement). palpated in the right subcostal area; veins
Test revealed increased vibration sensitivity of spermatic cord and scrotum are dilated.
threshold at the frequencies of 63-125-259 Hz Blood test: Hb - 86 g/l, ESR - 44 mm/h. Uri-
to 25 dB. Dynamometry is 20 kg on the right ne test: specific gravity - 1020, proteine - 0,99
and 16 kg on the left. Cold stimulus test is posi- g/l, erythrocytes - all field of vision, leukocytes
tive, time of hand temeperature restoration is - 4-6 in the field of vision. The provisional di-
52 minutes. Blanching at pressure symptom is agnosis is:
positive and equals 21. Hypesthesia of upper
and lower limbs is observed and can be classifi- A. Kidney tumor
ed as "gloves"and "socks"polyneuritic pattern. B. Urolithiasis
Make the provisional diagnosis. C. Acute pyelonephritis
D. Acute glomerulonephritis
A. Vibration disease, I stage, caused by combi- E. Nephroptosis
ned vibration
B. Vibration disease, I stage, caused by local 144. A 58-year-old patient complains of
vibration headache in the occipital region, nausea, choki-
C. Vibration disease, II stage, caused by combi- ng, opplotentes. The presentations appeared
ned vibration after a physical exertion. Objectively: the pati-
D. Vibration disease, II stage, caused by local ent is excited. Face is hyperemic. Skin is pale.
vibration Heart sounds are regular, the 2nd aortic sound
E. Vibration disease, I stage, caused by general is accentuated. BP - 240/120 mm Hg, heart
vibration rate - 92/min. Auscultation reveals some fine
moist rales (crackles) in the lower parts of the
140. A patient’s X-ray image (anteroposterior lungs. Liver is not enlarged. ECG shows signs
projection) shows deformation of lung pattern, of hypertrophy and left ventricular overload.
pneumofibrosis, reticular (honeycomb) lung What is the most likely diagnosis?
pattern of lower pulmonary segments, cylindric A. Complicated hypertensic crisis
and fusiform lumps. The most likely diagnosis B. Acute myocardial infarction, pulmonary
is: edema
A. Multiple bronchiectasis C. Bronchial asthma exacerbation
B. Right lung abscess D. Uncomplicated hypertensic crisis
C. Non-hospital-acquired pneumonia E. Community-acquired pneumonia
D. Lungs maldevelopment 145. A 37-year-old patient complains of pain
E. Diffuse pulmonary fibrosis in the lumbar spine, which increases during
141. A 41-year-old patient with Addi- walking, limited mobility, edema of the right si-
son’s disease had influenza. After that he de of abdomen. Focal tuberculosis is recorded
developed adynamia, depression, nausea, in the anamnesis. X-ray shows destruction
vomiting, diarrhea and hypoglycemia. BP of adjacent surfaces of the bodies of the 1-
is 75/50 mm Hg. Blood test: decreased 2 lumbar vertebrae, vertebral bodies height
content of corticosterone, hydrocortisone, is decreased, intervertebral fissure cannot be
13-oxycorticosteroids, 17-oxycorticosteroids. detected. Abdominal US reveals a growth
What condition has developed in the patient? 15x20 cm in size in the retroperitoneal space
and echo signs of liquid. What diagnosis can be
A. Acute adrenal gland insufficiency suspected?
B. Acute gastritis
C. Acute enterocolitis A. Lumbar tuberculous spondylitis
D. Collapse B. Fracture of the 1-2 lumbar vertebrae body
E. Diabetes mellitus C. Metastatic damage of vertebral column
D. Lumbar spondylolisthesis
142. After a contact with chemicals a plant E. Osteochondrosis
worker has suddenly developed stridor, voi-
ce hoarseness, barking cough, progressi- 146. A 45-year-old patient (14-year-long work
ng dyspnea. Objective examination reveals record as a house painter) upon the contact
acrocyanosis. What is your provisional di- with synthetic paint develops skin reddeni-
agnosis? ng, edema, severe itching and oozing lesions
on her face. Symptoms disappear after the
A. Laryngeal edema contact with this chemical substance stops
B. Laryngeal carcinoma but even the smell of paint alone is enough
C. PATE to make them reappear each time. Each
D. Pulmonary atelectasis recurrence is characterised by increased severi-
E. Pneumothorax ty of symptoms. What provisional diagnosis can
be made?
143. A 58-year-old patient complains of general
weakness, weight loss up to 10 kg within the
last 1,5 months, progressive pain in the small
of the back, raise of blood pressure to 220/160
mm Hg, subfebrile body temperature. Objecti-
Крок 2 Medicine (англомовний варiант, iноземнi студенти) 2015 рiк 18

A. Professional eczema A. Pyloric stenosis


B. Simple contact dermatitis B. Penetration of gastric ulcer
C. Allergic contact dermatitis C. Functional pyloric spasm
D. Urticaria D. Foreign body in the stomach (bezoar)
E. Toxicodermia E. Malignization of gastric ulcer
147. In cold season a patient was delivered to 151. A 5-year-old child has body temperature
a hospital. He was rescued from drowning in a risen up to febrile numbers, suffers from
body of water. There was no contact of his ai- inertness, weakness. Examination revealed
rways with water. The patient is anxious, pale, hemorrhage on the skin of limbs and torso.
complains of pain and numbness of limbs, has Enlargement of cervical and axillary lymph
cold shivering. Breathing rate is 22/min, BP nodes can be detected. The liver is 4 cm below
is 120/90 mm Hg, heart rate is 110/min, rectal the costal arch; the spleen is 6 cm below
temperature is 34, 5o C . What kind of warming the costal arch. Blood test: erythrocytes -
should be prescribed to the patient? 2, 3 · 1012 /l, Hb- 60 g/l, platelets - 40 · 109 /l,
A. Passive warming leukocytes - 32, 8 · 109 /l, eosinophiles - 1%,
band neutrophiles - 1%, segmented neutrophi-
B. Infusion of 37O c solutions les - 12%, lymphocytes - 46%, monocytes - 1%,
C. Hot compresses blasts - 40%, Duke’s bleeding time test result is
D. Heat bath 9 min. What examination is necessary to make
E. Hemodialysis with blood warming the diagnosis?
148. Survey radiograph of a 52-year-old worker A. Myelogram
of an agglomeration plant (28 years of experi- B. Lymph nodes biopsy
ence, the concentration of metal dust is 22-37 C. US of abdominal cavity
mg/m3 ) shows mildly pronounced interstitial fi- D. Detection of hepatitis markers
brosis with diffused contrast well-defined small E. Investigation of platelets dynamic functions
nodular shadows. The patient has no complai-
nts. Pulmonary function is not compromised. 152. A child from primipregnancy was born in
What is the provisional diagnosis? a term labor and has body weight of 4000 g
and body length of 57 cm. When born, he was
A. Siderosis nonresponsive to examination. Diagniosis is di-
B. Silicosis ffuse. Heart rate is 80/min. What resuscitation
C. Anthraco-silicatosis measures should be prescribed?
D. Silicatosis
E. Anthracosis A. Begin ALV with mask
B. Introduce 100% oxygen
149. A week before, a 65-year-old male patient C. Intubate and begin ALV
suffered an acute myocardial infarction. His D. Tactile stimulation
general condition has deteriorated: he complai- E. Administer naloxone
ns of dyspnea at rest, pronounced weakness.
Objectively: edema of the lower extremities, 153. A 20-year-old patient was delivered to a
ascites is present. Heart borders are extended, surgical unit complaining of an incised wound
paradoxical pulse is 2 cm displaced from the on his right forearm that has been bleeding
apex beat to the left. What is the most likely for 1,5 days. Suffers from general weakness,
diagnosis? vertigo, cold sweat, opplotentes. Skin and visi-
ble mucous membranes are pale. Heart rate is
A. Acute cardiac aneurysm 110/min, BP is 100/70 mm Hg. Blood test: Hb
B. Recurrent myocardial infarction is 100 g/l, erythrocytes 2, 5 · 1012 /l. What is the
C. Acute pericarditis cause for the paient’s general condition?
D. Cardiosclerotic aneurysm
E. Pulmonary embolism A. Posthemorrhagic anemia
B. Aplastic anemia
150. A 60-year-old patient complains of nearly C. Wound infection
permanent sensation of heaviness and fullness D. Concomitant disease
in the epigastrium, that increases after eating, E. Acute trombophlebitis
foul-smelling eructation, occasional vomiting
with food consumed 1-2 days ago, weight loss. 154. A 20-year-old parturient woman has the I
12 years ago he was first found to have an ulcer labor stage. The pregnancy is full-term. Labors
of pyloric channel. The patient has taken rani- occur every 3 minutes and last for 55 seconds.
tidine for periodic hunger pain. The patient’s Fetus presentation is polar, the head is pressed
condition has been deteriorating over the last to the small pelvis entrance. Heart rate of the
3 months. Objectively: splashing sound in the fetus is 150/min, distinct and rhythmic. Vagi-
epigastrium is present. What kind of compli- na examination: uterus cervix is smoothed out;
cation is it? mouth of the womb is 2 cm open; fetal bladder
is intact; the head is presented over the I plane
of small pelvis; moderate mucous-bloody di-
scharge is observed. What phase of the I labor
stage is it?
Крок 2 Medicine (англомовний варiант, iноземнi студенти) 2015 рiк 19

A. Latent A. Echocardiography
B. Active B. Electrocardiography
C. Slowing-down C. X-ray
D. Physiological preliminary period D. Phonocardiography
E. Primary uterine inertia E. Rheography
155. A 23-year-old woman after stress has 159. A 64-year-old patient has been hospitali-
developed thirst, polydipsia, polyuria, weight sed with complaints of progressive jaundice
loss, increasing fatigue. Later she developed that developed over 3 weeks ago without pain
nausea and somnolence, lost consciousness syndrome, general weakness, loss of appetite.
and was hospitalised. Glycemia is 27 mmol/l, Objectively: temperature is 36, 8o C , heart rate
acetone in urine is sharply positive. Treatment is 78/min, abdomen is soft and painless, peri-
for ketoacidotic coma was initiated. When toneum irritation symptoms are not detected,
would it be advisable to start preventive palpation reveals sharply enlarged tense
treatment of hypoglycemia by introduction of gallbladder. What disease can be characteri-
5% glucose solution? sed with these symptoms?
A. After glycemia rate drops to 13-14 mmol/l A. Cancer of pancreas head
B. 2 hours after beginning of insulinotherapy B. Duodenal ulcer
C. When patient becomes conscious C. Acute cholecystitis
D. After glycemia rate becomes normal D. Chronic cholecystitis
E. If glycemia decreases with the rate over 5 E. Cholecystitis caused by lambliasis
mmol/l per hour
160. A 22-year-old woman on a reduced diet,
156. A 53-year-old woman complains of weight vegetarian, attended a hospital with complai-
loss up to 10 kg within the last 2 years, liquid nts of distorted smell and taste perception and
foul-smelling stool two times a day that poorly lesions in the angles of her mouth. Objecti-
washes off the toilet, periodic bouts of nausea, vely: sclera is distinctly blue. Diagnosis: iron-
girdle pain in the upper abdomen. Objecti- deficiency anemia. What clinical syndrome is
vely: pain in Gubergrits zone (on the right from expressed primarily?
navel) and Mayo-Robson’s point. Biochemical
blood analysis: glucose - 3,2 mmol/l, bilirubin - A. Sideropenic
16,5 micromole/l, crude protein - 56,4 g/l. Urine B. Anemic
diastase - 426 g/h/l. D-xylose test (oral admi- C. Hemorrhagic
D. Hemolytic
nistration of 25 g of d-xylose) after 5 hours E. Myelodysplasic
reveals 3 g of xylose in urine. The most likely
diagnosis is: 161. A 15-year-old patient complains of
excessive body weight, headache, irritability,
A. Pancreatitis. Malabsorption syndrome rapid fatigability. Significant increase of body
B. Pseudomembranous colitis weight occurred at the age of 14. Objectively:
C. Nonspecific ulcerative colitis weight is 90 kg; height is 160 sm, proportional
D. Irritable bowel syndrome body built. Fatty tissue is distributed evenly.
E. Chronic gastritis
There are thin pink striae (stretch marks) on
157. A 23-year-old patient after intake of brake the thighs, abdomen and mammary glands. BP
fluid has developed anuria that has been lasting - 145/90 mm Hg. Provisional diagnosis is:
for 5 days already. Creatinine level increased up
to 0,769 mmol/l. What treatment tactics should A. Pubertate dyspituitarism
be chosen in the given case? B. Alimentary constitutive obesity
C. Somatoform autonomic dysfunction
A. Hemodialysis D. Itsenko-Cushing’s disease
B. Detoxification therapy E. Cushing’s syndrome
C. Antidotal therapy
D. Diuretics 162. A 6-year-old girl attended a general practi-
E. Plasmapheresis tioner with her mother. The child complains
of burning pain and itching in her external
158. A 15-year-old teenager has undergone genitalia. The girl was taking antibiotics the
medical examination in military recruitment day before due to her suffering from acute
center. The following was revealed: interval bronchitis. On examination: external genitalia
systolic murmur at the cardiac apex, accent are swollen, hyperemic, there is white deposit
of the II heart sound over the pulmonary accumulated in the folds. The most likely di-
artery, tachycardia. What additional examinati- agnosis is:
on method will be the most informative for
determining diagnosis? A. Candidal vulvovaginitis
B. Trichomoniasis
C. Nonspecific vulvitis
D. Helminthic invasion
E. Herpes vulvitis
163. A 44-year-old man complains of dyspnea
Крок 2 Medicine (англомовний варiант, iноземнi студенти) 2015 рiк 20

with sensation of lack of air on inhale, palpi- are detected, the skin covering them is bluish-
tations occurring during slight physical exerti- red in colour. Some nodules have fistulous
on, and shin edemas that appear in eveni- openings producing thick purulent mass. Body
ng and resolve in morning. His condition has temperature is 38, 5O c, general malaise. The
been lasting for 5 months already, deteriorati- most likely diagnosis is:
on is gradual. What method of instrumental
diagnostics allows to verify decrease of systolic A. Hydradenitis
function in this patient? B. Carbuncle
C. Cutaneous tuberculosis
A. Echocardiography D. Necrotizing ulcerative trichophytosis
B. Electrocardiography E. Pyoderma chancriformis
C. Computed tomogram
D. Phonocardiography 168. A patient complains of weight gain, chill,
E. Holter blood pressure monitoring edemas, xeroderma, somnolence, difficulties
with focusing. Objectively: height is 165 cm;
164. When playing in a kindergarten a 3- weight is 90 kg; body proportions are of female
year-old child sudenly developed dyspnea, type, to - 35, 8o C , heart rate - 58/min, BP -
paroxysmal compulsive dry cough. The face 105/60 mm Hg. Heart sounds are weakened,
is cyanotic, the eyes are tearful. Vomiting bradycardia is observed. Other internal organs
occurred several times. Breathing is weakened have no changes. Thyroid gland cannot be
over the whole right side of the chest. The palpated. Milk secretion from mammary
provisional diagnosis is: glands is observed. Hormone investigation
revealed increased levels of thyroid-stimulating
A. Foreign body
B. Obstructive bronchitis hormone (TSH) and prolactin, and decreased
C. Bronchial asthma level of thyroxine (T4). Which one is the cause
D. Hysteria fit for obesity?
E. Stenosing laryngotracheitis A. Primary hypothyroidism
165. A patient with otopyosis is in sharply B. Secondary hypothyroidism
deteriorating condition: he developed C. Prolactinoma
headache, vomiting, febrile temperature, D. Hypopituitarism
general hyperesthesia. Meningeal symptoms E. Adiposogenital dystrophy
and stagnant optic disks are observed. There 169. A 54-year-old patient complains of
is no focal symptoms. Liquor is turbid, blood weakness, jaundice, itching skin. Disease onset
pressure is high, albuminocytological dissoci- was 1,5 months ago: fever up to 39o C appeared
ation occurs with neutrophils. What disease at first, with progressive jaundice developed 2
can be suspected? weeks later. On hospitalisation jaundice was
A. Secondary purulent meningitis severely progressed. Liver cannot be palpated.
B. Meningoencephalitis Gallbladder is enlarged and painless. Blood
C. Serous meningitis bilirubin is 190 micromole/l (accounting for
D. Primary purulent meningitis direct bilirubin). Stool is acholic. What is the
E. Subarachnoid hemorrhage most likely jaundice genesis in this patient?

166. A 25-year-old patient is not married and A. Mechanical jaundice


has sexual relations with several partners. Duri- B. Hepatocellular jaundice
ng the last 3 months he noticed small amount of C. Hemolytic jaundice
mucoserous secretions produced from urethra. D. Caroli syndrome
Subjectively: periodical itching or burning pain E. Gilbert’s syndrome
in urethra. Two months ago pain in knee join
developed. Possibility of trauma or exposure 170. A 22-year-old patient complains
to cold is denied by the patient. During the of 8-months-long delay of menstruation.
last week eye discomfort is noted - lacrimation Anamnesis: menarche since the age of 12,5.
and itching. What provisional diagnosis can be Since the age of 18 menstruations are irregular.
suggested? No pregnancies. Mammary glands have normal
development; when the nipples are pressed,
A. Reactive arthritis milk drops are discharged. On gynecological
B. Rheumatoid arthritis examination: moderate uterus hypoplasia. On
C. Seasonal pollinosis hormonal examination: prolactin level exceeds
D. Bacterial nonspecific urethral conjunctivitis the norm two times. On computed tomogram
E. URTI with conjunctiva and joints affected of the sellar region: a space-occupying lesion
4 mm in diameter is detected. The most likely
167. A 19-year-old patient complains of severe diagnosis is:
pain in axillary crease. Condition onset was a
week ago after her swimming in cold river and
epilation. The next day painful "boil"appeared
that was becoming larger every day and
became a plum-sized tumor. Upon examinati-
on nodular conical growths joined together
Крок 2 Medicine (англомовний варiант, iноземнi студенти) 2015 рiк 21

A. Pituitary tumor 175. An emergency doctor has diagnosed a 32-


B. Lactation amenorrhea year-old woman with generalized convulsive
C. Stein–Leventhal syndrome (Polycystic ovary status epilepticus. The deterioration in the pati-
syndrome) ent’s condition is caused by a sudden gap in the
D. Sheehan’s syndrome epilepsy treatment. Specify the doctor’s further
E. Cushing’s disease tactics:
171. A 2-year-old child in a satisfactory conditi- A. Hospitalization in the intensive care unit
on periodically presents with moderate protei- B. Hospitalization in the department of
nuria, microhematuria. US results: the left ki- neurology
dney is undetectable, the right one is enlarged, C. Hospitalization in the department of
there are signs of double pyelocaliceal system. neurosurgery
What investigation is required to specify the D. Outpatient monitoring by a neuropathologist
diagnosis? E. Outpatient monitoring by a neurosurgeon
A. Excretory urography 176. A 19-year-old patient complains of
B. Micturating cystography dyspnea during physical exertion. He often has
C. Retrograde urography bronchitis and pneumonia. Cardiac murmur
D. Doppler study of renal vessels has been observed since his childhood. On
E. Radioisotope renal scan auscultation: there is splitting of II heart sound
over pulmonary arteria, systolic murmur in the
172. For the last 3 years a 12-year-old boy has third intercostal space near the left sternum
been suffering from stomachache, abdominal margin. ECG test shows right bundle-branch
distension, nausea, periodical liquid fatty stool, block. What is the provisional diagnosis?
grey in color, with rotten smell. On palpation:
pain in the epigastrium, Desjardins’ pancreatic A. Interatrial septum defect
point and Chauffard’s triangle; positive Mayo- B. Open arterial duct
Robson’s sign. Insufficiency of pancreas exocri- C. Coarctation of aorta
ne function is suspected. What method is the D. Aortic stenosis
most informative for pancreas exocrine functi- E. Mitral valve insufficiency
on assessment?
177. A 30-year-old patient was in a car accident.
A. Detection of elastase-1 in feces He is unconscious, pale, has thready pulse. In
B. Determining tripsin content in blood serum the middle third of the right thigh there is
C. Pancreas echography an extensive laceration with ongoing profuse
D. Determining amylase content in blood and external arterial bleeding. What urgent actions
urine must be taken to save the life of the patient?
E. Scatological test
A. Tourniquet above the wound of the right
173. A 34-year-old patient was delivered to thigh
a first-aid center with open fracture of the B. Tourniquet below the wound of the right
lower leg bones. Upon examination bleeding thigh
is stated: blood flows in pulsating bursts. What C. Artificial lung ventilation
actions should be taken at this stage of medical D. Precordial thump
aid? E. Plaster bar
A. Apply tourniquet to the thigh proximally to 178. A 75-year-old male patient complains of
the place of hemorrhage and deliver the patient slight pain in the right iliac region. The abdomi-
to operating room nal pain arose 6 days ago and was accompani-
B. Provide immobilization and deliver the ed by nausea. Surgical examination revealed
patient to a hospital moist tongue, heart rate - 76 bpm. BP - 130/80
C. Apply tourniquet to the thigh distally to the mm Hg. Abdomen is soft, slightly painful in
place of hemorrhage and deliver the patient to the right iliac region on deep palpation, the
operating room symptoms of the peritoneum irritation are
D. Apply pressure dressing to stop hemorrhage doubtful. In blood: RBCs - 4, 0 · 1012 /l, Hb-
and deliver the patient to operating room 135 g/l, WBCs - 9, 5 · 109 /l, stab neutrophiles
E. Deliver the patient to operating room - 5%, segmentonuclear - 52%, lymphocytes -
174. A 26-year-old patient with affective bi- 38%, monocytes - 5%, ESR - 20 mm/h. Specify
polar disorder has developed a condition mani- the doctor’s further tactics:
fested by mood improvement, behavioural A. Emergency operation for acute appendicitis
and sexual hyperactivity, verbosity, active body B. Hospitalization, dynamic surveillance
language, reduced need for sleep. Which of the C. Send the patient home
following drugs are most effective in this case? D. Refer the patient to a district therapist
A. Neuroleptics with a sedative effect E. Administration of additional examination:
B. Antidepressants with an activating effect abdominal ultrasound, x-ray contrast study of
C. Neuroleptics with an activating effect the gastrointestinal tract
D. Tranquilizers 179. On the 10th day postpartum a puerperant
E. Antidepressants with a sedative effect woman complains of pain and heaviness in
Крок 2 Medicine (англомовний варiант, iноземнi студенти) 2015 рiк 22

the left mammary gland. Body temperature is 183. A 70-year-old patient consulted a doctor
38, 8o C , Ps- 94 bpm. The left mammary gland is about arrhythmic cardiac activity, dyspnea.
edematic, the supero-external quadrant of skin Objectively: BP - 150/90 mm Hg, extrasystole
is hyperemic. Fluctuation symptom is absent. arrhythmia (10-12 beats per minute), left
The nipples discharge drops of milk when ventricular systolic dysfunction (ejection fracti-
pressed. What is a doctor’s further tactics? on at the rate of 42%). Which of anti-
arrhythmic drugs should be administered as
A. Antibiotic therapy, immobilization and initial therapy in this case?
expression of breast milk
B. Compress to both mammary glands A. Amiodarone
C. Inhibition of lactation B. Flecainide
D. Physiotherapy C. Encainide
E. Opening of the abscess and drainage of the D. Moracizine
mammary gland E. Digoxin
180. An 8-year-old child was hospitalized 184. A 45-year-old male patient complains of
for fever up to 39, 8o C , inertness, moderate acute pain in his right side irradiating to the
headache, vomiting. Examination revealed right thigh and crotch. The patient claims also
meningeal symptoms. Lumbar puncture was to have frequent urination with urine which
performed. The obtained fluid had raised resembles a "meat slops"in color. The patient
opening pressure, it was transparent, with has no previous history of this condition. There
the cell count of 450 cells per 1 mcL (mai- is costovertebral angle tenderness on the right
nly lymphocytes - 90%), glucose level of 2,6 (positive Pasternatsky’s symptom). What is the
mmol/l. What causative agent might have most likely diagnosis?
caused the disease in the child?
A. Urolithiasis
A. Enterovirus B. Acute appendicitis
B. Meningococcus C. Acute pyelonephritis
C. Koch’s bacillus D. Acute cholecystitis. Renal colic
D. Staphylococcus E. Acute pancreatitis
E. Pneumococcus
185. A 38-year-old man works within the
181. A 25-year-old patient during self- range of ionizing radiation. At a routine medi-
examination detected a tumor in the upper cal examination he presents no problems. In
external quadrant of the right mammary gland. blood: RBCs - 4, 5 · 1012 /l, Hb- 80 g/l, WBCs -
On palpation: painless, dense, mobile growth 2
cm in diameter is detected in the mammary 2, 8 · 109 /l, thrombocytes - 30 · 109 /l. Decide if
gland; no changes in the peripheral lymph this person can work with sources of ionizing
nodes are observed. On mammary glands US: radiation:
in the upper external quadrant of the right A. Working with radioactive substances and
mammary gland there is a space-occupying other sources of ionizing radiation is contraindi-
lesion of increased echogenicity 21х18 mm in cated
size. The most likely diagnosis is: B. The patient is allowed to work with radioacti-
A. Fibrous adenoma ve substances
B. Lacteal cyst C. The patient can only work with radioactive
C. Diffuse mastopathy substances of low activity
D. Breast cancer D. The patient can be allowed to work after an
E. Mastitis extended medical examination
E. The patient is allowed to work with radioacti-
182. A 20-year-old woman complains of feeling ve substances for the limited period of time
of air shortage, lingering dull pain in the heart
area, irritability. Objectively: general condition 186. A maternity patient breastfeeding for 1,5
is satisfactory, heart rate lability, BP is 130/60 weeks has attended a doctor. She considers
mm Hg. ECG reveals disruption of repolari- the onset of her disease to be when proporti-
zation proceses. The patient is diagnosed with onal breast engorgement occurred. Mammary
somatoform autonomic dysfunction of cardi- glands are painful. Body temperature is
al type. Specify the conditions of the patient 36, 6o C . Expression of breast milk is hindered.
treatment: The most likely diagnosis is:

A. Out-patient treatment A. Lactostasis


B. In-patient treatment in therapeutics B. Infiltrative mastitis
department C. Suppurative mastitis
C. In-patient treatment in cardiology D. Chronic cystic mastitis
department E. Gangrenous mastitis
D. In-patient treatment in cardiac surgery 187. A 12-year-old girl complains of general
department weakness, rise of body temperature up to
E. In-patient treatment in psychiatric 38, 2o C , pain and swelling of knee joints, feeli-
department ng of cardiac rhythm disruption. The chi-
Крок 2 Medicine (англомовний варiант, iноземнi студенти) 2015 рiк 23

ld had tonsillitis 3 weeks ago. The knee joi- ultrastructural change of ciliated epithelium.
nts are swollen, local raise of temperature is What is the basis of the given syndrome?
observed, mobility is reduced. Heart sounds
are weakened, extrasystole is present; at the A. Primary ciliary diskinesia
cardiac apex systolic noise can be auscultated, B. Proteoglycans insufficiency
which is not conducted to the left axillary regi- C. Surfactant deficit
on. ESR is 38 mm/h. CRP 2+. Antistreptolysin- D. Muscle cells atony
O titers are 400. The most likely disease is: E. Alpha-1-antitrypsin deficiency

A. Acute rheumatic fever 192. A newborn infant (the first labor, lasted
B. Somatoform autonomic dysfunction for 26 hours) is 1-day-old, postmature; body
C. Non-rheumatic carditis weight is 3850 g; body length is 52 cm. Deli-
D. Juvenile rheumatoid arthritis very was performed by applying obstetrical
E. Reactive arthritis forceps in sincipital presentation, Apgar score
is 1/3. The face is bluish-pale. The head is
188. A 60-year-old man has a diet consisting of thrown back; severe birth trauma is present; the
unvaried food staples: mostly cereals, potato, infant is excitable, shrill "cerebral scream"is
pasta; few vegetables and little fats (especially present; the eyes are half-open; facial expressi-
animal fats). During medical examination he on is attentive; hyperesthesia, hypersthenia and
complains of deterioration of his twilight visi- readiness for convulsions are present. Liquor
on. This condition can be caused by lack of: has high content of erythrocytes, lymphocytic
cytosis occurs. The most likely diagnosis is:
A. Retinol
B. Amino acids A. Subarachnoid hemorrhage
C. Fats B. Epidural hemorrhage
D. Calcium C. Subdural hemorrhage
E. Carbohydrates D. Intraventricular hemorrhage
E. Intracerebral hemorrhage
189. A 45-year-old patient was hospitalised
in surgical in-patient unit with intermuscular 193. A 74-year-old patient visited a urologist
phlegmon of the right thigh on the 6th day from with complaints of pain above the pubis and
the onset of disease. Abscess formed under the inability to urinate for 8 hours. At home he
fascia lata was diagnosed, lanced and widely had taken antispasmodics and had a warm bath
drained, necrotic tissues were removed. Anti- but no improvement occurred. Objectively:
biotic therapy with cephalosporines and the abdomen is soft and painful above the pubis;
III generation fluoroquinolones was prescri- dullness of percussion sound is observed above
bed, as well as immune corrective and detoxifi- the pubis. Murphy’s (Pasternatski’s) punch sign
cation infusion therapy (2,5 liters per day), di- is negative on the both sides. What condition
uresis stimulation, UV irradiation of blood and does the patient have?
plasmapheresis. What is the main component
of sepsis prevention? A. Acute urinary retention
B. Paradoxal ischuria
A. Surgical invasion C. Shronic urinary retention
B. General antibiotic therapy D. Anuria
C. Topical antibiotic therapy E. Oliguria
D. Immune correction therapy
E. Detoxification therapy 194. A 37-year-old patient has been undergoing
treatment of diskogenic lumbosacral radiculitis
190. An Rh-negative woman with 32-week- for a month. There is skin numbness observed
long term of pregnancy has been exami- at the lateral surface of the right lower extremi-
ned. It was observed that Rh-antibodies ti- ty, Achilles jerk is absent. MRI of lumbar spine
ter had increased four times within the last revealed intervertebral disk L5 -S1 prolapse up
2 weeks and was 1:64. First two pregnancies to 8 mm. Choose the further tactics for patient
ended in antenatal death of fetus caused by treatment:
hemolytic disease. What tactics of pregnancy
management should be chosen? A. Surgical extraction of intervertebral disk
B. Conservative treatment
A. Preterm delivery C. Stabilizing spinal surgery
B. Delivery at 37 weeks term D. Corset
C. Rh-antibody test in 2 weeks; if Rh-antibodies E. Manual therapy
increase in number conduct delivery
D. Introduction of anti-Rh immunoglobulin 195. A 40-year-old patient complains of
E. US examination to determine signs of fetal constant moderate pain in the lumbar spine
erythroblastosis and significantly reduced mobility. The pati-
ent has been suffering from this condition for
191. A 2-year-old child has been suffering the last 7 years since pain appeared first in the
since birth from recurring inflammatory di- sacrum area. X-ray: ankylosis of sacroiliac arti-
seases of lungs, purulent pansinusites, heari- culation, significant narrowing of intervertebral
ng deterioration, multiple cylindrical bronchi- joint fissures of lumbar vertebrae and calcifi-
ectases. Dextrocardia is observed. On biopsy: cation of spinal ligaments. What pathology is
Крок 2 Medicine (англомовний варiант, iноземнi студенти) 2015 рiк 24

most likely to cause such X-ray image? 198. A 30-year-old woman complains of
irregular copious painful menstruations, pain
A. Ankylosing spondylitis irradiates to the rectum. Anamnesis states 10-
B. Spinal tuberculosis year-long infertility. On bimanual examination:
C. Spinal osteochondrosis uterus is of normal size; uterine appendages
D. Vertebral osteochondropathy on the both sides are corded, with rectricted
E. Rheumatoid arthritis mobility, painful; there are dense nodular pai-
nful growths detected in the posterior fornix. A
196. A 7-year-old child became ill again 2 doctor suspects endometriosis. What method
weeks after he had tonsillitis. There are the allows to verify this diagnosis?
following complaints: temperature rise up to
38o C , hemorrhagic rash on the extremities, A. Laparoscopy
enlargement of the ankle joints. Blood test: B. Diagnostic curettage of uterine cavity
hemoglobin is 120 g/l, platelets are 170 · 109 /l, C. Paracentesis of posterior fornix
ESR is 30 mm/h. Urine test: proteinuria up D. Uterine probing
to 0,7 g/l, cylinders - 5-6 in the field of visi- E. Hysteroscopy
on, erythrocytes - 8-10 in the field of vision.
What mechanism of hemorrhagic syndrome is 199. A 14-year-old girl complains of tooth
present in the given case? caries; the tooth should be filled. Anamnesis
states that artificial mitral valve was installed 2
A. Vessel wall damage caused by immune years ago due to mitral insifficiency. What anti-
complexes bacterial drug should be prescribed to prevent
B. Platelet dysfunction infective endocarditis?
C. Suppression of hematopoietic stem cells
D. Decrease of adhesive-aggregative function A. Amoxicillin
of platelets B. Lincomycin
E. Vessel wall damage caused by bacteria C. Ceftriaxone
D. Erythromycin
197. A 48-year-old patient was delivered to a E. Midecamycin
hospital in-patient unit with uterine bleeding
that occurred after the 2-week-long delay of 200. A 30-year-old woman complains of inferti-
menstruation. Anamnesis states single birth. lity during her 10-year-long married life.
Examination of the uterine cervix with mirrors Menstruations occur since she was 14 and are
revealed no pathologies. On bimanual exami- irregular, with delays up to a month and longer.
nation: uterus is of normal size, painless, mobi- Body mass is excessive. Hirsutism is observed.
le; uterine appendages have no changes. Di- On bimanual examination: uterine body is
scharge is bloody and copious. What primary decreased in size; ovaries are increased in si-
hemostatic measure should be taken in the gi- ze, dense, painless, and mobile. The most likely
ven case? diagnosis is:
A. Fractional curettage of uterine cavity A. Stein–Leventhal syndrome (Polycystic ovary
B. Hormonal hemostasis syndrome)
C. Hemostatics B. Follicular cyst of ovaries
D. Uterine tamponade C. Genital endometriosis
E. Uterotonics D. Genital tuberculosis
E. Inflammatory tumor of ovaries
Крок 2 Medicine (англомовний варiант, iноземнi студенти) 2016 рiк 1

1. You are a doctor on duty. A patient after Common blood analysis: erythrocytes - 2,8
a successful resuscitation (drowning) was deli- Т/l, Hb- 80 g/l, color index - 0,78, anisocytosis,
vered to an admission room. BP is 90/60 mm poikilocythemia, serum iron - 10 mcmol/l.
Hg, heart rate is 120/min., respiration rate is What diagnosis is most likely?
26/min. The patient is unconscious, pupils are
moderately dilated, general clonic and tonic A. Iron-deficiency anemia
convulsions are observed. Make the diagnosis: B. B12 -deficient anemia
C. Autoimmune hemolytic anemia
A. Postresuscitation disease D. Aplastic anemia
B. Apparent death E. Hypoplastic anemia
C. Coma of unknown origin
D. Unconsciousness 6. A 24-year-old patient visited a doctor
E. Vegetative state complaining of enlargement of his submaxi-
llary lymph nodes. Objectively: submaxi-
2. A 32-year-old welder complains of llary, axillary and inguinal lymph nodes
weakness and fever. His illness initially are enlarged. Chest X-ray shows: enlarged
presented as tonsillitis one month earlier. On lymph nodes of mediastinum. Blood test:
examination: BT- 38, 9o C , RR- 24/min., HR- erythrocytes - 3, 4 · 1012 /l, Hb- 100 g/l, blood
100/min., BP- 100/70 mm Hg, hemorrhages colour index - 0,88, platelets - 190 · 109 /l,
on the legs, enlargement of the lymph nodes. leucocytes - 7, 5·109 /l, eosinophiles - 8%, band
CBC shows Hb- 70 g/l, RBC- 2, 2 · 1012 /l, neutrophiles - 2%, segmented neutrophiles -
WBC- 3, 0 · 109 /l with 32% of blasts, 1% of 67%, lymphocytes - 23%, ESR- 22 mm/hour.
eosinophiles, 3% of bands, 36% of segments, What test must be prescribed to verify the
20% of lymphocytes, and 8% of monocytes, cause of lymphadenopathy?
ESR- 47 mm/hour. What is the cause of
anemia? A. Open biopsy of lymph nodes
B. Ultrasonography of abdominal cavity
A. Acute leukemia C. Mediastinum tomography
B. Chronic lympholeukemia D. Puncture biopsy of lymph nodes
C. Aplastic anema E. Sternal puncture
D. B12 -deficient anemia
E. Chronic hemolytic anemia 7. A 32 year old patient complains of cardiac
irregularities, dizziness, dyspnea at physical
3. A regional cardiologist is tasked with the exertion. He has never suffered from such
development of a plan for medioprophylactic condition before. Objectively: Ps- 74/min.,
measures aimed at decrease of cardiovascular rhythmic. BP- 130/80 mm Hg. Auscultati-
mortality. What measures should be planned on revealed systolic murmur above aorta,
for secondary prevention? the first heart sound was normal. ECG
showed hypertrophy of the left ventricle, si-
A. Prevention of recurrences and complicati- gns of repolarization disturbance in the I , V5
ons and V6 leads. Echocardiogram revealed that
B. Referring patients for sanatorium-and-spa interventricular septum was 2 cm. What is the
treatment most probable diagnosis?
C. Prevention of diseases
D. Referring patients for in-patient treatment A. Hypertrophic cardiomyopathy
E. Optimization of life style and living conditi- B. Aortic stenosis
ons C. Essential hypertension
D. Myocardium infarction
4. An 8-year-old boy developed a temperature E. Coarctation of aorta
of 37, 5o C two days after his recovery from
the case of URTI. He complains of suffocati- 8. A 35-year-old patient’s wound with
on, heart pain. Objectively: the skin is pale, suppurative focus was surgically cleaned.
tachycardia, the I heart sound is weakened, On the 8th day after the surgery the wound
short systolyc murmur in the 4th intercostal cleared from its purulo-necrotic content and
area near the left edge of the breastbone. granulations appeared. However, against the
What heart disorder such clincal presentati- bacground of antibacterial therapy the body
on is characteristic of? temperature keeps at 38,5-39,5o C . There are
chills, excessive sweating, euphoria, heart rate
A. Nonrheumatic myocarditis is 120/min. What complication of local pyoi-
B. Primary rheumatic carditis nflammatory process can it be?
C. Myocardiodystrophy
D. Fallot’s tetrad A. Sepsis
E. Cardiomyopathy B. Purulent absorption fever
C. Trombophlebitis
5. A woman complains of muscle weakness D. Meningitis
and general fatigue, dyspnea, vertigo, bri- E. Pneumonia
ttleness of her hair and nails, an urge to
eat chalk. Anamnesis states uterine fibroid. 9. A 60-year-old woman has been sufferi-
Крок 2 Medicine (англомовний варiант, iноземнi студенти) 2016 рiк 2

ng from arterial hypertension for 15 years. A. Tuberculous spondylitis of the thoracic


After recurrent stroke she started complaini- spine
ng of unmotivated bad mood, problems with B. Spinal tumor
attention concentration; she forgets to close C. Multiple sclerosis
the entrance door, cannot recall events of D. Metastases into the spine
the past day. Computer tomography shows E. Osteochondrosis
areas of postinfarction changes in the cortical
postfrontal areas. What is the most probable 13. A pregnant woman is 28 years old.
diagnosis? Anamnesis: accelerated labor complicated by
the II degree cervical rupture. The followi-
A. Vascular dementia ng two pregnancies resulted in spontaneous
B. Alzheimer’s disease abortions at the terms of 12 and 14 weeks.
C. Huntington’s disease On mirror examination: the uterine cervix
D. Pick’s disease is scarred from previous ruptures at 9 and 3
E. Dissociative amnesia hours, the cervical canal is gaping. On vagi-
nal examination: the cervix is 2 cm long, the
10. Clinic of a research instutute for occupati- external orifice is open 1 cm wide, the internal
onal diseases examined a worker who works orifice is half-open; the uterus is enlarged to
at a concentration plant and diagnosed him the 12th week of pregnancy, soft, mobile, pai-
with chronic dust bronchitis. The case is nless, the appendages are without changes.
investigated by a commission including the What diagnosis would you make?
representatives of: the plant, clinic, local
SES, department of Social Insurance Fund, A. Isthmico-cervical insufficiency, habitual
trade union. According to the ”regulation on noncarrying of pregnancy
investigation of. . . ”, the commission should be B. Threatened spontaneous abortion
headed by the representative of the following C. Incipient abortion, habitual noncarrying of
authority: pregnancy
D. Cervical hysteromyoma, habitual noncarryi-
A. Local SES ng of pregnancy
B. Plant E. Cervical pregnancy, 12 weeks
C. Social Insurance Fund
D. Trade union 14. A 56-year-old patient complains of pain in
E. Clinic the epigastrium after eating, eructation, loss
of appetite, slight loss of weight, fatigabili-
11. A 37-year-old woman complains of ty. The patient smokes; no excessive alcohol
headaches, nausea, vomiting, spasms. The consumption. Objectively: pale mucosa, BP-
onset of the disease occurred the day before 110/70 mm Hg. The tongue is ”lacquered”.
due to her overexposure to cold. Objecti- The abdomen is soft, sensitive in the epi-
vely: fever up to 40o C ; somnolence; rigid gastric area. Blood test: erythrocytes - 3,0 T/l,
neck; Kernig’s symptom is positive on the Hb- 110 g/l, color index - 1,1; macrocytosis;
both sides; general hyperesthesia. Blood test: leukocytes - 5,5 g/l, ESR- 13 mm/hour. On
leucocytosis, increased ESR. Cerebrospinal fibrogastroduodenoscopy: atrophy of fundic
fluid is turbid, yellow-tinted. What changes of mucosa. What pathogenesis does this disorder
the cerebrospinal fluid are most likely? have?
A. Neutrophilic pleocytosis A. Producing antibodies to parietal cells
B. Lymphocytic pleocytosis B. Н.pylori persistence
C. Blood in the cerebrospinal fluid C. Alimentary factor
D. Xanthochromia in the cerebrospinal fluid D. Chemical factor
E. Albuminocytological dissociation E. Gastropathic effect
12. A 48-year-old woman complains of pain 15. A 26-year-old secundipara at 40 weeks of
in the thoracic spine, sensitivity disorder gestation arrived at a maternity ward after
in the lower body, disrupted motor functi- the beginning of labor activity. The bursti-
on of the lower limbs, body temperature ng of waters occurred 2 hours prior. The
rise up to 37, 5o C . She has been suffering fetus was in a longitudinal lie with cephalic
from this condition for 3 years. Treatment presentation. Abdominal circumference was
by various specialists was ineffective. X-ray 100 cm, fundal height - 42 cm. Contracti-
reveals destruction of adjacent surfaces of ons occurred every 4-5 minutes and lasted
the VIII and IX vertebral bodies. In the right 25 seconds each. Internal obstetric examinati-
paravertebral area at the level of lesion there on revealed cervical effacement, opening by 4
is an additional soft tissue shadow. What di- cm. Fetal bladder was absent. Fetal head was
agnosis is most likely? pressed against the pelvic inlet. What compli-
cation arose in the childbirth?
Крок 2 Medicine (англомовний варiант, iноземнi студенти) 2016 рiк 3

A. Early amniorrhea A. Peripheral lung cancer


B. Primary uterine inertia B. Bronchiectasis
C. Secondary uterine inertia C. Pneumonia
D. Discoordinated labor D. Middle lobe syndrome
E. Clinically narrow pelvis E. Interlobular pleurisy
16. Bacterial analysis of air in a living space in 20. A 62-year-old patient has been hospitali-
winter period by means of Krotov’s apparatus zed with complaints of pain in the thorax
revealed that total number of microorganisms on the right during breathing, dyspnea, dry
in 1 m3 of air was 7200. What is the allowed cough. Ten days ago he slipped and fell hitti-
number of microorganisms for the air to be ng his right side. On examintaion: the pati-
characterized as ”pure”? ent lies on the left side. The right side of the
thorax lags during breathing. On the right
A. Up to 4500 there are crepitation and pain in the III-IV ri-
B. Up to 2500 bs. Dullness of percussion sound and sharply
C. Up to 3500 diminished breath sounds can be observed.
D. Up to 5500 On X-ray: signs of exudate, fracture of the III-
E. Up to 7500 IV ribs. On pleurocentesis: blood is detected.
Choose the further tactics:
17. A 24-year-old patient received a
puncture injury below the Poupart’s ligament A. Transfer to a thoracic surgery department
accompanied by intense arterial bleeding. The B. Prescribe conservative therapy
best method to temporarily stop the bleeding C. Recurrent pleurocentesis
in the patient would be: D. Fixed bandage of the rib cage
E. Refer to a traumatologist
A. Compression band
B. Esmarch’s tourniquet 21. A 29-year-old female patient complains of
C. Maximum limb bending dyspnea, heaviness and chest pain on the ri-
D. Compressing a blood vessel with a clamp ght, body temperature rise up to 37, 2o C . The
E. Wound suturing disease is associated with a chest trauma recei-
ved 4 days ago. Objectively: skin is pale and
18. A 42-year-old woman has been hospitali- moist. Ps- 90/min., regular. Palpation reveals
zed with complaints of intense pain attacks a dull sound on the right, auscultation reveals
in the lumbar and right iliac areas, which significantly weakened vesicular breathing.
irradiate to the vulvar lips, frequent urination, In blood: RBCs- 2, 8 · 1012 /l, colour index -
nausea. The pain onset was acute. Objecti-
vely: the abdomen is soft, moderately pai- 0,9, Hb- 100 g/l, WBCs- 8, 0 · 109 /l, ESR- 17
nful in the right subcostal area, costovertebral mm/hour. What results of diagnostic puncture
angle tenderness on the right. Common uri- of the pleural cavity can be expected?
ne analysis: specific gravity - 1016, traces of A. Haemorrhagic punctate
protein, leukocytes - 6-8 in the vision field, B. Chylous liquid
erythrocytes - 12-16 in the vision field, fresh. C. Exudate
What diagnosis can be made? D. Transudate
A. Right-sided renal colic E. Purulent punctate
B. Acute right-sided pyelonephritis 22. Caries morbidity rate is 89% among resi-
C. Acute right-sided adnexitis dents of a community. It is determined that
D. Acute cholecystitis fluorine content in water is 0,1 mg/l. What
E. Acute appendicitis preventive measures should be taken?
19. A 29-year-old patient works as a A. Water fluorination
motor mechanic. Anamnesis shows frequent B. Tooth brushing
exposure to cold, exacerbation of chronic C. Fluorine inhalations
bronchitis attended by cough with relati- D. Sealant application
vly small amount of mucopurulent sputum, E. Introduce more vegetables to the diet
subfebrility, sometimes joined by hemoptysis
and pain in the right side of chest. Breathing 23. Examination of a group of persons living
is vesicular. X-ray shows shadows and sharp on the same territory revealed the following
decrease in size of the lower lobe distinctly vi- common symptoms: dark-yellow pigmentati-
sible on the X-ray image as a streak 2-3 cm wi- on of the tooth enamel, diffuse osteoporosis
de situated at the angle from lung root to the of bone apparatus, ossification of ligaments
frontal costodiaphragmatic recess. The most and joints, functional disorders of the central
likely diagnosis is: nervous system. This condition may be caused
by the excessive concentration of the followi-
ng microelement in food or drinking water:
Крок 2 Medicine (англомовний варiант, iноземнi студенти) 2016 рiк 4

A. Fluorine aside from tachycardia (102/min.) no other


B. Copper changes. On ECG: pathologic wave Q in I,
C. Nickel аVL, QS in V1, V2, V3 leads and ’domed’ ST
D. Iodine elevation with negative T. What diagnosis is
E. Cesium most likely?
24. In a pre-school educational establishment A. Acute left ventricular anterior myocardial
the menu consists of the following dishes: milk infarction
porridge from buckwheat, pasta with minced B. Variant angina pectoris
meat, cucumber salad, kissel (thin berry jelly), C. Aortic dissection
rye bread. What dish should be excluded from D. Acute left ventricular posterior myocardial
the menu? infarction
E. Exudative pericarditis
A. Pasta with minced meat
B. Milk porridge from buckwheat 29. A 48-year-old woman has thermal burns
C. Kissel (thin berry jelly) of both hands. The epidermis of the palms
D. Rye bread and backs of her hands is exfoliating, and bli-
E. Cucumber salad sters filled with serous liquid are forming. The
forearms are intact. What diagnosis is most
25. An infant has been born at the 41st week likely?
of gestation. The pregnancy was complicated
with severe gestosis of the second semester. A. 2-3A degree thermal burn
The weight of the baby is 2400 g, the height is B. 4 degree thermal burn
50 cm. Objectively: the skin is flabby, the layer C. 1 degree thermal burn
of subcutaneous fat is thin, hypomyotonia, D. 3B degree thermal burn
neonatal reflexes are weak. The internal E. 1-2 degree thermal burn
organs are without pathologic changes. This
newborn can be estimated as a: 30. A 39-year-old patient complains of morni-
ng headache, appetite loss, nausea, morning
A. Full-term infant with prenatal growth vomiting, periodic nasal haemorrhages. The
retardation patient had a case of acute glomerulonephri-
B. Premature infant tis at the age of 15. Examination revealed rise
C. Immature infant of arterial pressure up to 220/130 mm Hg, skin
D. Postmature infant haemorrhages on his arms and legs, pallor of
E. Full-term infant with normal body weight skin and mucous membranes. What biochemi-
cal parameter is most important for making
26. A patient suffering from infiltrati- diagnosis in this case?
ve pulmonary tuberculosis was prescribed
streptomycin, rifampicin, isoniazid, pyrazi- A. Blood creatinine
namide, vitamin C. One month after the begi- B. Blood bilirubin
nning of the treatment the patient started C. Blood sodium
complaining of reduced hearing and tinnitus. D. Uric acid
What drug has such a side effect? E. Fibrinogen
A. Streptomycin 31. A 30-year-old patient, who has
B. Isoniazid been suffering from headaches, suddenly
C. Rifampicin developed extreme headache after lifting a
D. Pyrazinamide heavy load, as if he had been hit over the
E. Vitamin C head. Nausea, vomiting, and slight dizzi-
ness are observed. In a day he developed
27. A woman has developed sudden thoracic pronounced meningeal syndrome and body
pain on the right with expectoration of pi- temperature up to 37, 6o C . A doctor suspects
nk sputum and body temperature rise up to subarachnoid hemorrhage. What additional
37, 7o C on the 4th day after the surgery for examination is necessary to confirm this di-
cystoma of the right ovary. On lung examinati- agnosis?
on: dullness of the lung sound on the lower
right is observed. Isolated moist crackles can A. Lumbar puncture with investigation of the
be auscultated in the same area. What compli- spinal fluid
cation is the most likely? B. Skull X-ray
C. Computed tomography of the brain
A. Pulmonary infarction D. Rheoencephalography
B. Pneumonia E. Angiography of the brain vessels
C. Pulmonary abscess
D. Exudative pleurisy 32. A worker of a blowing shop complains of
E. Pneumothorax headache, irritability, sight impairment - he
sees everything as if through a ”net”. Objecti-
28. A 58-year-old patient was delivered to an vely: hyperemic sclera, thickened cornea,
admission room with complaints of pain in the decreased opacity of pupils, visual acuity is
thorax on the left. On clinical examination:
Крок 2 Medicine (англомовний варiант, iноземнi студенти) 2016 рiк 5

0,8 in the left eye, 0,7 in the right eye. The operator?
worker uses no means of personal protection.
What diagnosis is most likely? A. Photoelectric ophthalmia
B. Heatstroke
A. Cataract C. Vegetative-vascular dystonia
B. Conjunctivitis D. Chronic overheating
C. Keratitis E. Pneumoconiosis
D. Blepharospasm
E. Progressive myopia 37. A 48-year-old patient was found to have
diffuse enlargement of the thyroid gland,
33. A 45-year-old woman is undergoing exophthalmia, weight loss of 4 kg in 2 months,
treatment for active rheumatism, combined sweating. Objectively: HR- 105/min., BP-
mitral valve failure. During her morning 140/70 mm Hg. Defecation act is normal.
procedures she suddenly sensed pain in the What kind of therapy is recommended in this
left hand, which was followed by numbness. case?
Pain and numbness continued to aggravate.
Objectively: the skin of the left hand is pale A. Mercazolil
and comparatively cold. Pulse in the hand B. Radioiodine
arteries is absent along the whole length. C. Propranolol
What treatment tactics is most efficient? D. Lugol’s solution
E. Thyroxine
A. Urgent embolectomy
B. Prescription of fibrinolytics and anti- 38. A woman addressed a doctor with
coagulants complaints of increased body temperature up
C. Prescription of antibiotics and antii- to 37, 8o C and moderately sore throat for the
nflammatory agents last 3 days. Objectively: mandibular lymph
D. Cardiac catheterization nodes are enlarged up to 3 cm. Palatine tonsils
E. Urgent thrombintimectomy are hypertrophied, covered with grey coating
that spreads to the uvula and anterior pillars
34. A 10-year-old boy had a case of viral of the fauces. What diagnosis is most likely?
hepatitis type B four years ago. Currently the
assumption was made about the formation of A. Oropharyngeal diphtheria
hepatic cirrhosis in the patient. What additi- B. Infectious mononucleosis
onal investigation can clarify the diagnosis? C. Pseudomembranous (Vincent’s) tonsillitis
D. Agranulocytosis
A. Renal needle biopsy E. Oropharyngeal candidiasis
B. Proteinogram
C. Echocholecystography 39. A 48-year-old man complains of constant
D. Markers of viral hepatitis type B pain in the upper abdomen, predominantly
E. Transaminase level measurement on the left, which aggravates after eating,
diarrhea, loss of weight. The patient has
35. A 40-year-old patient has acute onset alcohol use disorder. Two years ago he had
of disease caused by overexposure to cold. a case of acute pancreatitis. Blood amylase
Temperature has increased up to 39o C . is 4 g/hour·l. Feces analysis: steatorrhea,
Foul-smelling sputum is expectorated duri- creatorrhea. Blood sugar is 6,0 mmol/l. What
ng coughig. Various moist crackles can be treatment should be prescribed?
auscultated above the 3rd segment on the ri-
ght. Blood test: leukocytes - 15, 0 · 109 /l, stab A. Panzinorm forte (Pancreatin)
neutrophils - 12%, ESR- 52 mm/hour. On X- B. Insulin
ray: in the 3rd segment on the right there is a C. Gastrozepin (Pirenzepine)
focus of shadow 3 cm in diameter, low density, D. Contrykal (Aprotinin)
with fuzzy smooth margins and a clearing in E. No-Spa (Drotaverine)
its center. What disease is most likely in the
given case? 40. In 10 hours after eating canned
mushrooms a 27-year-old patient has
A. Pneumonia complicated by an abscess developed diplopia, bilateral ptosis, di-
B. Infiltrative tuberculosis srupted swallowing, shallow breathing with
C. Peripheral pulmonary cancer respiratory rate 40/min., muscle weakness,
D. Cystic echinococcosis enteroparesis. What measure should be taken
E. Pulmonary cyst first?
36. An electro-gas welding operator worki- A. Intubation of the trachea for artificial
ng at a machine workshop performs welding respiration
and cutting of metal, which is accompanied B. Gastrointestinal lavage
by intense UV-radiation. His welding station C. Introduction of antibotulinic serum
is equipped with efficient mechanical venti- D. Introduction of glucocorticosteroids
lation. What occupational disease is most E. Intravenous detoxication therapy
likely to develop in an electro-gas welding
41. A 32-year-old patient complains of
Крок 2 Medicine (англомовний варiант, iноземнi студенти) 2016 рiк 6

reddening, burning, and sensation of a foreign labor was diagnosed with cephalohematoma.
body in the right eye. The disease is acute. On On the 2nd day of life the child developed
examination: visual acuity of the both eyes jaundice; on the 3rd day of life there appeared
is 1,0. In the right eye there are hyperemy neurological changes: nystagmus, Graefe
and swelling of the conjunctiva, superficial syndrome. Urine is yellow, feces are golden-
injection. There is purulent discharge in the yellow. The mother’s blood group is А (II)
conjunctival sac. The cornea is clear. The color Rh− , the child’s - А (II) Rh+ . On the 3rd
and pattern of the iris are uncanged, the pupil day the results of the child’s blood test are as
is mobile. What diagnosis is most likely? follows: Hb- 200 g/l, erythrocytes - 6, 1 · 1012 /l,
A. Acute conjunctivitis blood bilirubin - 58 mcmol/l due to the
B. Acute iridocyclitis presence of its unconjugated fraction, Ht- 0,57.
C. Acute attack of glaucoma In this case the jaundice is caused by:
D. Foreign body of the cornea A. Craniocerebral birth injury
E. Acute dacryocystitis B. Physiologic jaundice
42. A patient is on the sick leave for 4 months C. Hemolytic disease of newborn
continuously from the date of injury. The D. Atresia of bile passages
treatment is going to last for another 1-2 E. Fetal hepatitis
months. Who has the right to extend the 46. On the 4th day after recovering from a
duration of medical certificate for this pati- cold a patient was hospitalized with complai-
ent? nts of solitary spittings of mucoid sputum.
A. Medical advisory commission after medico- On the 2nd day there was a single discharge
social expert commission examination of about 250 ml of purulent blood-streaked
B. Medical superintendent sputum. Objectively: the patient’s conditi-
C. Medical advisory commission after inpati- on is moderately severe. Respiratory rate -
ent treatment 28-30/min., Ps- 96/min., BP- 110/70 mm Hg.
D. District doctor upon agreement with a Respiration above the left lung is vesicular,
department chief weak above the right lung. There are vari-
E. Medico-social expert commission ous moist crackles above the lower lobe and
amphoric breath near the angle of scapula.
43. Monthly dysentery morbidity in the region What is the most likely diagnosis?
given in absolute figures is as follows: January
- 6; February - 9; March - 11; April - 10; May - A. Acute pulmonary abscess
16; June - 23; July - 19; August - 33; September B. Exudative pleuritis
- 58; October - 19; November - 11; December C. Acute focal pneumonia
- 5. Annual total is 220 cases. What graphic D. Pleural empyema
presentation would provide the best visual for E. Pyopneumothorax
monthly deviations of dysentery morbidity 47. A 46-year-old patient with temporari-
from the average? ly undetermined diagnosis was prescribed
A. Radar chart pleurocentesis based on the results of the X-
B. Map ray. The puncture yielded 1000 ml of a liquid
C. Cartogram with the following properties: clear, specific
D. Pie chart gravity - 1,010, protein content - 1%, Rivalta’s
E. Bar chart test is negative, erythrocytes - 2-3 in the field
of vision. What disorder are these pathologic
44. A 30-year-old woman complains of pain changes characteristic of?
in the heart area (”aching, piercing pain”) A. Cardiac failure
that arises primarily in the morning hours in B. Pleuropneumonia
autumn and spring. Pain irradiates into the C. Pleural mesothelioma
neck, back, abdomen and is attended by rapid D. Pulmonary tuberculosis
heart rate and low vital tonus. This condition E. Pulmonary cancer
occurs independently from physical exerti-
on. In the evening her condition improves. 48. A 14-year-old girl has been presenting wi-
Somatic and neurologic state and ECG have th irritability and tearfulness for about a year.
no pathologies. What pathology is likely to A year ago she was also found to have diffuse
result in such clinical presentation? enlargement of the thyroid gland (II grade).
A. Somatized depression This condition was regarded as a pubertal
B. Rest angina pectoris manifestation, the girl did not undergo any
C. Pseudoneurotic schizophrenia treatment. The girl’s irritability gradually gave
D. Somatoform autonomic dysfunction place to a complete apathy. The girl developed
E. Hypochondriacal depression puffy face, soft tissues pastosity, bradycardia,
constipations. Skin pallor and gland density
45. A full term baby born from the 1st progressed, the skin developed a waxen hue.
noncomplicated pregnancy with complicated What disease can be suspected?
Крок 2 Medicine (англомовний варiант, iноземнi студенти) 2016 рiк 7

A. Autoimmune thyroiditis 53. A 56-year-old patient with diffuse toxic


B. Diffuse toxic goiter goiter has ciliary arrhythmia, heart rate is 110-
C. Thyroid carcinoma 120/min., arterial hypertension, BP is 165/90
D. Subacute thyroiditis mm Hg. What drug besides Mercazolil (Thi-
E. Juvenile basophilism amazole) should be prescribed in this case?
49. In a maternity hospital a newborn had A. Propranolol
been presenting with cough attacks after eati- B. Radioactive iodine
ng. The child was discharged from the hospital C. Novocainamide (procainamide)
on the 18th day due to a case of pneumonia. D. Verapamil
During the further 1,5 months the child had E. Corinfar
2 cases of pneumonia. Periodically there are
cough attacks after eating, especially if the 54. A patient has been provisionally di-
child lies on the left side. Objectively: the II agnosed with pheochromocytoma at the stage
degree hypotrophy, isolated moist crackles, of intermission. BP is within norm, there is
dyspnea. Stool and diuresis are not disrupted. a tendency towards tachycardia. No urine
What diagnosis is most likely? pathologies. The decision has been made to
perform a provocative test with histamine.
A. Tracheoesophageal fistula What drug should be kept close at hand for
B. Mucoviscidosis emergency aid in case of positive test result?
C. Posthypoxic encephalopathy
D. Hernia of the esophageal opening A. Phentolamine
E. Tracheobronchomalacia B. Pipolphen
C. Nifedipine
50. A 26-year-old woman, who gave bi- D. Mesaton (Phenylephrine)
rth 7 months ago, has been suffering from E. Prednisolone
nausea, morning sickness, somnolence for
the last 2 weeks. The patient breasfeeds; no 55. A 54-year-old patient complains of
menstruation. She has been using no means of weakness, weight loss despite the unchanged
contraception. What method would be most appetite, frequent urination, skin itch for six
efficient in clarification of the diagnosis? months. Some time ago the patient underwent
treatment for furunculosis. She has not been
A. Ultrasound examined recently. Objectively: malnutriti-
B. Small pelvis radiography on, dry skin with scratch marks. Small lymph
C. Palpation of mammary glands and squeezi- nodes can be palpated in the axillary regions.
ng out colostrum Changes in the internal organs are absenr.
D. Bimanual abdominovaginal examination What test must be performed in the first
E. Mirror examination place?
51. A 2-year-old girl has a medical history A. Fasting blood sugar
of recurrent obstructive pneumonia. In the B. Complete blood count
lungs various moist and dry crackles can C. Endoscopy of stomach
be auscultated, breath sounds are dimini- D. Lymph node biopsy
shed. Sputum is thick, viscous and difficult E. Blood sterility testing
to expectorate. Drumstick fingers and physi-
cal developmental retardation are observed. 56. A child is 1 month 10 days old. ”Gushing”
What preliminary diagnosis can be made? vomiting has been observed since the age of 3
weeks. The vomit volume exceeds the volume
A. Pulmonary mucoviscidosis of the previous feeding. Objectively: the chi-
B. Recurrent bronchitis ld is inert. Skin elasticity and tissue turgor
C. Bronchial asthma are decreased. Hour-glass deformity sign is
D. Congenital polycystic lungs positive. The preliminary diagnosis is pyloric
E. Pulmonary tuberculosis stenosis. What treatment tactics should be
52. After a case of purulent otitis a 1-year-old chosen?
boy has developed pains in the upper third of A. Surgery
the left thigh, body temperature up to 39o C .
Objectively: swelling of the thigh in its upper B. Prescription of Cerucal (Metoclopramide)
third and smoothed out inguinal fold. The li- C. Atropinization
mb is in semiflexed position. Active and passi- D. Internal administration of Novocaine
ve movements are impossible due to severe E. Prescription of Pipolphen
pain. What diagnosis is most likely? 57. A 46-year-old woman complains of pain
A. Acute hematogenous osteomyelitis attacks in the right lumbar area, which irradi-
B. Acute coxitis ate to the lower abdomen, and nausea.
C. Intermuscular phlegmon This kind of pain attacks has never been
D. Osteosarcoma detected in the patient before. Survey X-
E. Brodie’s abscess ray of the abdominal cavity reveals no
pathologic shadows. Ultrasonic scan detects
Крок 2 Medicine (англомовний варiант, iноземнi студенти) 2016 рiк 8

a hyperechogenic growth 1,5 cm in diameter, 27 10 . What drug would be most efficient in
which reflects sound wave, in the enlarged ri- the treatment of this patient?
ght renal pelvis. What diagnosis is most likely?
A. Cryoprecipitate
A. Renal calculus B. Calcium chloride
B. Benign renal tumor C. Concentrated red cells
C. Renal cyst D. Aminocapronic acid
D. Renal tuberculosis E. Vicasol (Menadione)
E. Malignant renal tumor
62. A 67-year-old man complains of dyspnea
58. Several hours before, a 28-year-old pati- on exertion, attacks of retrosternal pain, di-
ent suddenly developed acute headache and zziness. He has no history of rheumatism.
repeated vomiting, then lost consciousness. Objectively: pale skin, acrocyanosis. There are
Objectively: focal neurological symptoms crackles in the lower lungs. There is systolic
were not found. Pronounced meningeal thrill in the II intercostal space on the ri-
symptoms were revealed. BP- 120/80 mm ght, coarse systolic murmur conducted to the
Hg. According to clinical and liquorologi- vessels of neck. BP- 130/90 mm Hg, heart rate
cal findings the patient was diagnosed wi- - 90/min., regular rhythm. The liver extends 5
th subarachnoid haemorrhage. After admi- cm from under the edge of costal arch, shin
nistration of dehydrants the patient’s condi- edemas are present. Specify the suspected
tion somewhat improved. What is the main valvular defect:
component of further emergency care?
A. Aortic stenosis
A. Coagulants B. Pulmonary artery stenosis
B. Anticoagulants C. Mitral insufficiency
C. Antiaggregants D. Ventricular septal defect
D. Fibrinolytics E. Tricuspid regurgitation
E. Corticosteroids
63. A 24-year-old female teacher complai-
59. A 24-year-old man has developed increasi- ns of dizziness and heart pain irradiating
ng headaches, vertigo, diplopia, paresis of the to the left nipple. Pain is not associated
facial muscles on the right, choking during with physical activity and cannot be reli-
swallowing. The signs appeared on the 5th eved by nitroglycerin, it abates after taki-
day of respiratory disorder. He was diagnosed ng Valocordin and lasts an hour or more.
with acute viral encephalitis. Determine the The patient has a nearly 2-year history of
main direction of emergency therapy: this disease. Objectively: Ps- 76/min., BP-
110/70 mm Hg. Heart borders are normal,
A. Zovirax (Aciclovir) heart sounds are clear. The ECG shows respi-
B. Glucocorticoids ratory arrhythmia. Radiograph of the cervi-
C. Ceftriaxone cothoracic spine reveals no pathology. Lungs,
D. Lasix (Furosemide) abdomen are unremarkable. What changes in
E. Neohaemodes blood formula can be expected?
60. A 23-year-old woman, who works as a A. No changes
milk and dairy inspector, after the miscarri- B. Leukocytosis
age suffers from high fever up to 38, 6o C , C. Thrombocytopenia
recurring chills, excessive sweating. Objecti- D. Leukemic hiatus
vely: polyadenitis, pain in the lumbosacral E. Increased ESR
spine, swollen left knee joint, enlarged liver
and spleen. What diagnosis is most likely? 64. A 51-year-old female patient complains of
frequent defecation and liquid blood-streaked
A. Brucellosis stools with mucus admixtures, diffuse pain in
B. Sepsis the inferolateral abdomen, 6 kg weight loss
C. Toxoplasmosis within the previous month. Objectively: body
D. Polyarticular rheumatoid arthritis temperature - 37, 4o C , malnutrition, skin is
E. Yersiniosis pale and dry. Abdomen is soft, sigmoid is pai-
nful and spasmodic, makes a rumbling sound.
61. A 16-year-old adolescent has been hospi- Liver is dense, painful, extends 3 cm below
talized with complaints of unceasing nasal the costal margin. What is the most likely di-
hemorrhage and unbearable pain in his ri- agnosis?
ght elbow joint. Objectively: the large joint
is enlarged and defigurated, the skin over A. Non-specific ulcerative colitis
the joint is hyperemic. Arthropathy signs can B. Bacillary dysentery
be observed in the other joints. Ps- 90/min. C. Sprue
Blood test: erythrocytes - 3, 9 · 1012 /l, Нb- 130 D. Intestinal enzymopathy
g/l, color index - 1,0, leukocytes - 5, 6 · 109 /l, E. Helminthic invasion
platelets - 220 · 109 /l, ESR- 6 mm/hour.
65. Chief physician of a polyclinic charged a
Lee-White coagulation time: start- 24 , end-
Крок 2 Medicine (англомовний варiант, iноземнi студенти) 2016 рiк 9

district doctor with a task to determine the A. Platelets


pathological prevalence of disease N in his B. Reticulocytes
district. What document allows to estimate C. Clotting time
the disease prevalence in the population of a D. Osmotic resistance of erythrocytes
medical district? E. Fibrinogen

A. Prophylactic examinations register 69. A 49-year-old patient complains of


B. Statistic coupons (+) swallowing disorder that intensifies duri-
C. Statistic coupons (-) ng eating solid food, hiccups, hoarse voice,
D. Statistic coupons (+) and (-) nausea, regurgitation, significant weight loss
E. Vouchers for medical appointments (15 kg within 2,5 months). Objectively: body
weight is reduced; the skin is pale and dry;
66. A 32-year-old woman complains of di- vesicular respiration; heart sounds are suffi-
zziness, headache, palpitation, tremor. For ciently sonorous; heart rate is rhythmic. The
the last several months she has been under abdomen is soft, no pain on palpation. The
outpatient observation for increased arteri- liver is not enlarged. What investigation is
al pressure. Since recently such attacks have most necessary for making the diagnosis in
become more frequent and severe. Objecti- this case?
vely: the skin is covered with clammy sweat,
tremor of the extremities is present. HR- A. Esophagoduodenoscopy with biopsy
110/min., BP- 220/140 mm Hg. Heart sounds B. Clinical blood analysis
are muffled. Blood test results: WBCs- 9, 8 · C. X-ray of the gastrointestinal tract
D. X-ray in the Trendelenburg position
109 /l, ESR- 22 mm/hour. Blood glucose - 9,8 E. Investigation of gastric secretion
millimole/l. What disease is the most likely
cause of this crisis? 70. A 9-month-old child presents with fever,
cough, dyspnea. The symptoms appeared
A. Pheochromocytoma
5 days ago after a contact with a person
B. Essential hypertension suffering from URTI. Objectively: the child
C. Preeclampsia is in grave condition. Temperature of 38o C ,
D. Primary hyperaldosteronism
cyanosis of nasolabial triangle is present.
E. Diabetic glomerulosclerosis Respiration rate - 54/min., nasal flaring duri-
67. A 45-year-old patient complains of pain ng breathing. There was percussion dullness
in the epigastric region, left subcostal area, on the right below the scapula angle, and
abdominal distension, diarrhea, loss of wei- tympanic sound over the rest of lungs.
ght. He has been suffering from this condition Auscultation revealed bilateral fine moist
for 5 years. Objectively: tongue is moist with crackles predominating on the right. What is
white coating near the root; deep palpation of the most likely diagnosis?
abdomen reveals slight pain in the epigastric
A. Acute pneumonia
region and Мауо-Robson’s point. Liver is pai- B. URTI
nless and protrudes 1 cm from the costal arch. C. Acute laryngotracheitis
Spleen cannot be palpated. What disease can
D. Acute bronchitis
be primarily suspected? E. Acute bronchiolitis
A. Chronic pancreatitis
71. A 50-year-old man, who works as a poli-
B. Atrophic gastritis sher at a combine-building factory, addressed
C. Peptic stomach ulcer the factory’s sectorial doctor with complai-
D. Chronic cholecystitis
nts of general fatigue, sensations of numbness
E. Chronic enteritis and pain in his fingers. Objectively: the skin
68. A 58-year-old woman complains of of his fingers is pale. Reaction to pain, tactile
spontaneous bruises, weakness, bleeding and thermal stimuli was revealed to be slightly
gums, dizziness. Objectively: the mucous disrupted. No disruptions can be observed wi-
membranes and skin are pale with numerous thin the other organs and systems. What di-
hemorrhages of various time of origin. Lymph sorder is most likely?
nodes are not enlarged. Heart rate - 100/min., A. Pneumatic hammer disease
BP- 110/70 mm Hg. There are no changes
B. Multiple neuritis
of internal organs. Blood test results: RBC- C. Raynaud’s disease
3, 0 · 1012 /l, Нb- 92 g/l, colour index - 0,9, D. Syringomyelia
anisocytosis, poikilocytosis, WBC - 10 · 109 /l, E. Deforming arthrosis
eosinophiles - 2%, stab neutrophiles - 12%,
segmented neutrophiles - 68%, lymphocytes 72. A 27-year-old woman complains of
- 11%, monocytes - 7%, ESR- 12 mm/hour. bleeding gums, nasal hemorrhages, multiple
What index should be determined additi- hematomas on the skin of her limbs and on
onally by a laboratory to make a diagnosis? the front of her torso, extreme general fati-
gue. Blood test: Hb- 64 g/l, erythrocytes -
2, 5 · 1012 /l, reticulocytes - 16%, platelets -
Крок 2 Medicine (англомовний варiант, iноземнi студенти) 2016 рiк 10

30 · 109 /l, ESR- 22 mm/hour. What approach adrenal glands, cranium X-ray. The tests
would be most efficient for treatment of this revealed no pathologies.The diagnosis of
pathology? exogenic constitutive obesity has been made.
What direction of therapy should be prioriti-
A. Splenectomy zed?
B. Dicynone (Etamsylate)
C. Platelet concentrate transfusion A. Reducing diet and exercise
D. Cytostatics B. Sanatorium-and-spa treatment
E. Group B vitamins C. Anorectic drugs
D. Dehydration therapy
73. A 23-year-old patient had taken 1 g of E. ”Fat-burning” methods
aspirin to treat acute respiratory infection.
After that he developed an asthmatic fit wi- 77. An 8-year-old child with a 3-year history
th labored expiration that was arrested by of diabetes was hospitalized in hyperglycemic
introduction of aminophylline. The patient’s coma. Specify the initial dose of insulin to be
medical history is not burdened with allergies. administered:
The patient has undergone two surgeries for
nasal polyposis in the past. What diagnosis is A. 0,1-0,2 U/kg of body weight per hour
most likely? B. 0,05 U/kg of body weight per hour
C. 0,2-0,3 U/kg of body weight per hour
A. Aspirin-induced asthma D. 0,3-0,4 U/kg of body weight per hour
B. Atopic bronchial asthma E. 0,4-0,5 U/kg of body weight per hour
C. Infectious allergic bronchial asthma
D. Exercise-induced asthma 78. A worker, who was involved in fire fi-
E. Symptomatic bronchospasm ghting inside the building that stored 2 kg
of mercury, has been delivered to a hospital
74. A 58-year-old woman undergoing with complaints of emotional expansiveness,
chemotherapy for her oncologic disorder has palpitations, excessive sweating, body tremor,
developed sore throat. Examination revealed heart pain. Within one day his condition
necrotic areas on the mucosa of the pharynx aggravated. Objectively: the skin is pale and
and tonsils. Many of her teeth are afflicted wi- moist. The patient is depressed. Permanent
th caries. In blood: neutrophilic granulocytes red dermographism, erethism, unstable BP
are practically absent against the background are observed. What drug is the serum in this
of leukopenia. Leukocytes are represented case?
mainly by lymphocytes and monocytes. What
disease can be suspected in the given case? A. Unithiol
B. Atropine sulfate
A. Agranulocitar tonsillitis C. Calcium tetacine
B. Lacunar tonsillitis D. Amyl nitrite
C. Pseudomembranous (Vincent’s) tonsillitis E. Dipyroxime
D. Syphilitic tonsillitis
E. Diphtheria 79. During meat testing Trichinella was
detected in diaphragm crura in one of the two
75. A patient complains of constant dull muscular tissue samples. What tactics should
pain in the perineum and suprapubic area, a doctor choose regarding this meat?
weak flow of urine, frequent difficult pai-
nful urination, nocturia. The patient has A. Technolgical disposal
been suffering from this condition for several B. Incineration
months, during which urination was becomi- C. Boiling under 1,5 atmosphere
ng increasingly difficult, and pain in the peri- D. Preservation in 10% salt solution
neum has developed. On rectal examinati- E. Freezing under -12o C
on: the prostate is enlarged (mainly its right 80. A 40-year-old patient was bitten by a stray
lobe), dense, asymmetrical, central fissure is dog an hour ago. On the left shin there is a
smoothed out, the right lobe is of stony densi- bite mark - the wound is 4x2x0,5 cm in size.
ty, painless, tuberous. What disease is it? What surgical aid would be most efficient in
A. Prostate cancer this case?
B. Prostate sclerosis A. Lavage with soapy water, retension sutures
C. Urolithiasis, prostatolith of the right lobe B. Aseptic dressing
D. Prostate tuberculosis C. Salve dressing
E. Chronic congestion prostatitis D. Blind suture
76. A child is 10 years old. The weight is E. Retension sutures
46 kg. Since birth the child has been gai- 81. A 30-year-old patient has been hospitali-
ning excessive weight. The parents are full- zed with diagnosis of intestinal obstruction.
bodied. The child has undergone the followi- The surgery revealed the obstruction of the
ng tests: carbohydrate tolerance, level of small intestine to be caused by a helminth
17-ketosteroids, blood electrolytes, US of ball. What kind of helminth is it?
Крок 2 Medicine (англомовний варiант, iноземнi студенти) 2016 рiк 11

A. Shigellosis
A. Ascaris B. Salmonellosis
B. Guinea worm C. Escherichiosis
C. Filariidae D. Intestinal amebiasis
D. Cysticercus E. Yersiniosis
E. Pinworm
86. The institutions that take part in medi-
82. Mother of a newborn suffers from chronic cal examinations include prevention and
pyelonephritis; she had a case of URTI before treatment facilities, medical board of Mini-
the delivery. The delivery is at term, for a stry of Defense, medical board of Ministry of
long period waters were not breaking. On Home Affairs, medico-social expert commi-
the 2nd day of life the child developed an ssions, forensic medical boards etc. What insti-
erythematic rash, later the rash developed tutions are responsible for temporary disabi-
into blisters approximately 1 cm in diameter lity examination?
filled with serous purulent substance. Ni-
kolsky’s symptom is positive. After the bli- A. Prevention and treatment facilities
sters had been lanced, erosions developed in B. Sanitary-and-prophylactic institutions
their place. The child is inert. The child’s body C. Medico-social expert commissions
temperature is subfebrile. What diagnosis is D. Medical boards of Ministry of Defense
most likely? E. Medical boards of Ministry of Home Affairs

A. Impetigo neonatorum 87. After a lengthy march an army regi-


B. Vesiculopustulosis ment has set camp for 3 days near a
C. Pseudofurunculosis settlement. Sanitary-hygienic investigation
D. Sepsis detected several water sources. Choose the
E. Ritter’s disease source that would satisfy the demands for
potable water the most under the given fi-
83. Three weeks after a case of acute tonsilli- eld conditions:
tis the patient is still weak, inert, subfebrile,
his retromaxillary lymph nodes are enlarged. A. Artesian well
Tonsils are flabby, stick together with arches, B. Brook
there are purulent plugs in lacunae. What is C. River
the most probable diagnosis? D. Rain water
E. Melt water
A. Chronic tonsillitis
B. Chronic pharyngitis 88. Clinical statistical investigation was
C. Acute lacunar tonsillitis performed to determine efficiency of a new
D. Paratonsillitis pharmacological preparation for patients wi-
E. Tonsillar tumour th ischemic heart disease. What parametric
test (coefficient) can be used to estimate reli-
84. The following indicators were calculated ability of the results?
to analyse population health and treatment
quality in a cardiological hospital: primary A. Student’s t-distribution
cardiovascular morbidity - 62%; total cardi- B. Sign test
ovascular morbidity - 483,55%; cardiovascular C. Matching factor
mortality - 10,9%; proportion of cardi- D. Wilcoxon signed-rank test
ovascular mortality within total mortality - E. Kolmogorov-Smirnov test
67,0%; primary disablement caused by cardi-
ovascular diseases - 16,2 per 10.000 populati- 89. A 30-year-old parturient woman was deli-
on. What indicator is an extensive value? vered to a maternity hospital with full-term
pregnancy. She complains of severe lanci-
A. Proportion of cardiovascular mortality nating pain in the uterus that started 1 hour
within total mortality ago, nausea, vomiting, cold sweat. Anamnesis
B. Primary cardiovascular morbidity states cesarean section 2 years ago. Uteri-
C. Total cardiovascular morbidity ne contractions stopped. Skin and mucous
D. Primary disablement caused by cardi- membranes are pale. Heart rate is 100/min.,
ovascular diseases BP is 90/60 mm Hg. Uterus has no clear margi-
E. Cardiovascular mortality ns, is sharply painful. No heartbeat can be
auscultated in the fetus. Moderate bloody
85. A 6-year-old child complains of frequent discharge from the uterus can be observed.
liquid stool and vomiting. On the 2nd day Uterus cervix is 4 cm open. Presenting part is
of disease the child presented with inertness, not visible. The most likely diagnosis is:
temperature rise up to 38, 2o C , Ps- 150/min.,
scaphoid abdomen, palpatory painful sigmoid
colon, defecation 10 times a day with liquid,
scarce stool with mucus and streaks of green.
What is the provisional diagnosis?
Крок 2 Medicine (англомовний варiант, iноземнi студенти) 2016 рiк 12

A. Uterine rupture
B. Initial uterine rupture A. Testicular feminization syndrome
C. Threatened uterine rupture B. Mayer-Rokitansky-Kuster-Hauser
D. Premature detachment of normally positi- syndrome
oned placenta C. Cushing’s syndrome
E. Compression of inferior pudendal vein D. Sheehan syndrome
E. Cushing’s disease
90. A parturient woman is 23 years old.
Internal obstetric examination shows the 94. A planner designs a heating system for a
uterine cervix to be completely open. Fetal pre-school educational establishment. The hi-
bladder is absent. Cephalic presentation is ghest air temperature should be in the followi-
observed in the plane of the small pelvic ng room:
outlet. Sagittal suture is at the longitudi-
nal section of the small pelvic outlet, small A. Game room of a nursery group
fontanel is situated closer to the uterus. What B. Common room of a preschool group
cephalic position will the newborn have duri- C. Bedroom of a nursery group
ng birth in this case? D. Bedroom of a preschool group
E. Gymnasium
A. Minor oblique lie
B. Longitudinal lie 95. A 27-year-old sexually active woman
C. Transverse lie complains of numerous vesicles on the right
D. Medium oblique lie sex lip, itch and burning. Eruptions regularly
E. Major oblique lie appear before menstruation and disappear
8-10 days later. What is the most likely di-
91. During the dynamic observation of a agnosis?
parturient woman in the second stage of labor
it was registered that the fetal heart rate A. Herpes simplex virus
decreased to 90-100/min. and did not normali- B. Bartholinitis
ze after contractions. Vaginal examination C. Primary syphilis
revealed the complete cervical dilatation, the D. Cytomegalovirus infection
fetal head filling the entire posterior surface E. Genital condylomata
of the pubic symphysis and sacral hollow;
the sagittal suture was in the anteroposteri- 96. A 35-year-old woman addressed a
or diameter of the pelvic outlet, the posterior gynecological in-patient department with
fontanelle was in front under the pubic arch. complaints of regular pains in her lower
What plan for further labour management abdomen, which increase during menstruati-
should be recommended? on, and dark-brown sticky discharge from
the genital tracts. On bimanual examinati-
A. Application of forceps minor on: the uterine body is slightly enlarged, the
B. Caesarean section appendages are not palpated. Mirror exami-
C. Episiotomy nation of the uterine cervix reveals bluish
D. Application of cavity forceps spots. What diagnosis is most likely?
E. Stimulation of labour activity through
intravenous injection of oxytocin A. Cervical endometriosis
B. Cervical erosion
92. An 8-year-old girl periodically has sudden C. Cervical polyp
short-term heart pains, sensation of chest D. Cervical cancer
compression, epigastric pain, dizziness, vomi- E. Cervical fibroid
ting. Objectively: the patient is pale, respi-
ratory rate - 40/min., jugular pulse is present. 97. A 10-year-old patient has a history of mild
Ps- 185/min., of poor volume. BP- 75/40 bronchial asthma. During a regular check-up
mm Hg. ECG taken during an attack shows the patient should be recommended:
ectopic P waves, QRS wave is not deformed. A. To avoid allergenic food
At the end of an attack a compensatory pause B. To avoid body tempering procedures
is observed. The most likely cause of the C. To avoid sports
attack is: D. To avoid spa treatment
A. Paroxysmal atrial tachycardia E. To avoid going to the seaside
B. Sinus tachycardia 98. A baby was born by a young smoker. The
C. Paroxysmal ventricular tachycardia labour was complicated by uterine inertia,
D. Complete AV-block difficult delivery of the baby’s head and
E. Atrial fibrillation shoulders. The baby’s Apgar score was 4.
93. A 16-year-old girl has primary Which of the following is a risk factor for a
amenorrhea, no pubic hair growth, normally spinal cord injury?
developed mammary glands; her genotype is
46 ХY; uterus and vagina are absent. What is
your diagnosis?
Крок 2 Medicine (англомовний варiант, iноземнi студенти) 2016 рiк 13

A. Difficult delivery of the head and shoulders te, depression. Objective examination reveals
B. Young age of the mother no pathologic changes of uterus and uterine
C. Pernicious habits appendages. What pathogenesis is characteri-
D. Uterine inertia stic of this disorder?
E. Chronic hypoxia
A. Decreased production of gonadotropin
99. An emergency situation at a chemical B. Hyperproduction of estrogen
plant caused acute occupational intoxicati- C. Hyperproduction of androgen
on. A doctor who revealed the case of ”acute D. Decreased production of progesterone
occupational disease (intoxication)” must E. Hyperproduction of prolactin
notify the following authority:
104. A patient in a clinical death condition is
A. Sanitary and epidemiological station being resuscitated through mouth-to-mouth
B. Plant administration artificial pulmonary ventilation and external
C. Trade union committee of the plant cardiac massage. A doctor noticed that air
D. Medical unit of the plant does not flow into the patient’s airways and
E. Ministry of Public Health of Ukraine his head and torso are positioned at the same
level. Why is artificial respiration not effective
100. A 27-year-old woman has been treated in in the given case?
a surgical department for pleural empyema
for 6 months. Multiple paracenteses of the A. Tongue retraction
pleural cavity were performed along with anti- B. Low breathing volume
bacterial treatment. The patient’s condition is C. External cardiac massage
slowly aggravating; attempts to fully stretch D. Probe is absent from stomach
the lung were unsuccessful. Choose the tacti- E. The patient’s mouth is too small
cs:
105. A 45-year-old patient with urolithiasis
A. Decortication of the lung had an attack of renal colic. What is the
B. Change antibiotics mechanism of the attack development?
C. Set constant active suction drain
D. Pulmonectomy A. Disturbed urine outflow from the kidney
E. Include hyperbaric oxygenation in the B. Increase in urine specific gravity
treatment C. Ureteric twists
D. Destruction of glomerules
101. A prematurely born girl is now 8 E. Renal artery spasm
months old. She has dyspnea, tachycardia,
hepatosplenomegaly, physical developmental 106. A 26-year-old woman has attended
lag, limb cyanosis. There is parasternal cardiac maternity center complaining of her inability
hump, auscultation revealed systolodiastolic to become pregnant despite 3 years of regular
murmur in the II intercostal space on the sex life. Examination revealed the followi-
left. BP is 90/0 mm Hg. What disease can be ng: increased body weight; male-type pubic
suspected? hair; excessive pilosis of thighs; ovaries are
dense and enlarged; basal body temperature
A. Patent ductus arteriosus is monophasic. The most likely diagnosis is:
B. Coarctation of aorta
C. Stenosis of aortal valve A. Ovaries sclerocystosis
D. Stenosis of pulmonary artery B. Inflammation of uterine appendages
E. Nonclosure of interventricular septum C. Adrenogenital syndrome
D. Premenstrual syndrome
102. Three days ago a boy underwent removal E. Gonadal dysgenesis
of a foreign body from under a nail plate. 2
days later he deeloped acute pulsating pain at 107. The process of open-cut mining requi-
the end of the nail bone which aggravated at res drilling and blasting operations, rock
pressing. Nail fold became hyperemic, body and ore excavation, transportation of ore
temperature rose up to 37, 5o C , there was a to fragmentation and sorting factories and
change in nail plate colour. What is the most transportation of barren rock to slag-heaps,
likely diagnosis? road building and maintenance, repair works.
What factor of production is most important
A. Subungual panaritium for miner’s health?
B. Erysipelas
C. Paronychia A. High content of dust in the air
D. Erysipeloid B. High content of explosion gas
E. Abscess C. Vibration
D. Noise
103. A 25-year-old woman complains of E. Adverse microclimate
menstruation retention lasting for 3 years.
The patient explains it by a difficult childbirth 108. A woman addressed a gynecologist
complicated with profuse hemorrhage, weight on the 20th day of puerperal period with
loss, brittleness and loss of hair, loss of appeti- complaints of pain in the left mammary gland,
Крок 2 Medicine (англомовний варiант, iноземнi студенти) 2016 рiк 14

puruent discharge from the nipple. Objecti- at the seaside. Objectively: face erythema,
vely: Ps- 120/min., body temperature is 39o C . edema of shin muscles. Heart sounds are
The left mammary gland is painful, larger than muffled, BP is 100/70 mm Hg. In blood:
the right one, the skin there is hyperemic; in ASAT activity is 0,95 millimole/h·l, ALAT-
the upper quadrant there is an infiltrate 10x15 1,3 micromole/h·l, aldolase - 9,2 IU/l, creati-
cm in size with soft center. Blood test: ESR- ne phosphokinase - 2,5 micromole Р/g·l. What
50 mm/hour, leukocytes - 15, 0 · 109 /l. What method of study would be most specific?
would be the treatment tactics?
A. Muscle biopsy
A. Transfer to a surgical department for surgi- B. ECG
cal treatment C. Echocardiogram
B. Refer to a gynecology department D. Electromyography
C. Refer to a postnatal department E. Determination of cortisol concentration in
D. Refer to a surgeon for conservative blood and urine
treatment
E. Lance the mammary gland abscess in a 113. A 14-year-old girl has fainted during a
maternity department meeting. The day before she complained of a
headache. The skin is pale, the limbs are cold,
109. The correlation between the service shallow breathing, heart sounds are muffled;
record and eosinophil concentration in blood heart rate is 51/min.; BP is 90/50 mm Hg. The
was studied in workers at dyeing shops of abdomen is soft. Meningeal symptoms are
textile factories. What index will be most negative. Make the provisional diagnosis:
informative for the analysis of this data?
A. Collapse
A. Correlation factor B. Unconsciousness
B. Student’s criterion C. Acute left ventricular failure
C. Standardized index D. Acute right ventricular failure
D. Fitting criterion E. Respiratory failure
E. Sign index
114. For the last 15 years a 48-year-old patient
110. A 3-month-old girl has rhinitis, dyspnea, has been working at the factory producing
dry cough. She has been sick for 2 synthetic resins. Lately he has been complai-
days. Objectively: pale skin, acrocyanosis, ning of significant general fatigue, headaches,
hypopnoe; breathing rate is 80/min.; over frequent urination (predominantly during the
the whole pulmonary surface there is day), red color of urine. What complication
vesiculotympanitic (bandbox) resonance of benzene nitrocompounds poisoning can be
observed with numerous bubbling crackles. suspected?
The most likely diagnosis is:
A. Malignant tumor of the urinary bladder
A. Acute bronchiolitis B. Chronic cystitis
B. Pneumonia C. Chronic prostatitis
C. Mucoviscidosis D. Acute glomerulonephritis
D. Foreign body in airways E. Chronic pyelonephritis
E. Acute bronchitis
115. A 60-year-old woman developed
111. A 24-year-old patient complains of gaini- weakness, vertigo, rapid fatigability during the
ng body mass and increased appetite. Objecti- last year. Recently there have been dyspnea
vely: built of hypersthenic type, body mass and paresthesia observed. Objectively: skin
index is 33,2 kg/m2 , waist circumference is and mucous membranes are pale and icteric.
100 cm. Waist to hips circumference ratio is Lingual papillae are smoothed out. Liver
0,95. What is the provisional diagnosis? and spleen are situated at the edge of costal
arch. Blood test: Hb- 70 g/l, erythrocytes -
A. Alimentary constitutive obesity, I stage, 1, 7·1012 /l, blood color index - 1,2, macrocytes.
android type What drug can be prescribed on pathogenetic
B. Itsenko-Cushing hypothalamic obesity, II grounds?
stage, gynoid type
C. Alimentary constitutive obesity, III stage, A. Vitamin B12
gynoid type B. Vitamin B6
D. Alimentary constitutive obesity, II stage, C. Ascorbic acid
android type D. Iron preparations
E. Itsenko-Cushing hypothalamic obesity, I E. Vitamin B1
stage, android type
116. In the morning upon waking a 65-year-
112. A 36 year old female patient complains old patient developed weakness in the right-
ofgeneral weakness, edemas of her face and side limbs, speech disorder, decreased sensi-
hands, rapid fatigability during walking, diffi- tivity of the left side of the body. On exami-
cult diglutition, cardiac irregularities. These nation: conscious, BP- 100/60 mm Hg, motor
symptoms developed 11 days after holiday aphasia, right-sided central hemiparesis and
Крок 2 Medicine (англомовний варiант, iноземнi студенти) 2016 рiк 15

hemihypalgesia. Make the preliminary di- mmol/l, potassium - 6,5 mmol/l, glomerular
agnosis: filtration rate - 10 ml/min. What tactics would
be leading in the patient’s treatment?
A. Ishemic stroke
B. Hemorrhagic stroke A. Hemodialysis
C. Encephalitis B. Antibacterial therapy
D. Brain tumor C. Sorbent agents
E. Subarachnoid hemorrhage D. Blood transfusion
E. Hypotensive therapy
117. The objective of a statistical study was to
find out the extent of seeking medical care by 121. In a rural health care area there is a case
the population. For this purpose 300 residents of child dying during the first month of life. To
of the area were interviewed. Information was analyse this situation, among other measures,
collected by means of a special questionnaire. an expert assessment of medical records is
What method of collecting information was performed. What medical document should
used by researchers? be considered first?
A. Anamnestic A. Child development history
B. Immediate registration B. Infant development history
C. Immediate examination C. Vaccination card
D. Doing extracts D. Outpatient medical record
E. - E. Child’s medical record
118. A 57-year-old woman complains of havi- 122. A region attended by a central regional
ng a sensation of esophageal compresion, hospital demonstrates increased hemorrhagic
palpitation, difficult breathing during eating stroke morbidity. Essential hypertension
solid food, occasional vomiting with a full morbidity, however, remains at the same level
mouth, ”wet pillow” sign at night for the last and is below the average level registered wi-
6 months. Objectively: body tempearture - thin the larger area. What managerial decision
39o C , height - 168 cm, weight - 72 kg, Ps- should be made in this case?
76/min., BP- 120/80 mm Hg. X-ray revealed
a considerable dilation of the esophagus and A. To design and implement measures for
its constriction in the cardial part. What early diagnostics of arterial hypertension
pathology is most likely to have caused B. To design and implement measures for
dysphagia in this patient? primary prevention of essential hypertension
C. To design and implement measures for
A. Achalasia cardiae secondary prevention of hypertension compli-
B. Primary esophagism cations
C. Hiatal hernia D. To design and implement measures for
D. Esophageal carcinoma professional development of medical workers
E. Reflux esophagitis E. To design and implement improved di-
spensary system for hypertensive patients
119. A 25-year-old man complains of pain
in the lower third of his left thigh, which 123. A 42-year-old man was delivered to a
occurs both with and without physical exerti- surgical in-patient department with complai-
on. Possibility of trauma is denied by the nts of icteric skin, pain in the right subcostal
patient. Objectively: skin colour is normal; area. Biochemical blood analysis: total bili-
pastosity and pain can be detected with deep rubin - 140 mcmol/l, direct bilirubin - 112
palpation; knee joint mobility is reduced. X- mcmol/l. On US: choledoch duct - 1,4 cm,
ray of distal femoral metaphysis shows an a concrement is detected in the distal area.
area of destruction and spicule. Blood test: Gallbladder is 40 cm, no concrements. What
immature cells, no signs of inflammation. The treatment tactics should be chosen?
most likely diagnosis is:
A. Endoscopic papillosphincterotomy
A. Osteogenic sarcoma B. Laparoscopic cholecystectomy
B. Osteitis fibrosa cystica C. Laparotomy with choledoch duct drain
C. Chronic osteomyelitis D. Laparotomy with cholecystectomy
D. Multiple myeloma E. Threatment in an infectious diseases hospi-
E. Marble-bone disease tal
120. A 30-year-old woman suffers from 124. 4 weeks after myocardial infarction a 56-
polycystic renal disease. She has been admi- year-old patient developed acute heart pain,
tted with signs of fatigue, thirst and nocturia. pronounced dyspnea. Objectively: the pati-
Diuresis is up to 1800 ml per day. BP is ent’s condition is extremely grave, there is
200/100 mm Hg. Blood test: erythrocytes - marked cyanosis of the face, swelling and
1, 8 · 109 /l, Hb- 68 g/l. Urine analysis: specific throbbing of the neck veins, peripheral pulse
gravity - 1005, leukocytes - 50-60, erythrocytes is absent, carotid artery pulse is rhythmic,
- 3-5 in the vision field, creatinine - 0,82 130/min., BP is 60/20 mm Hg. Auscultation of
Крок 2 Medicine (англомовний варiант, iноземнi студенти) 2016 рiк 16

the heart reveals extremely muffled sounds, ght shin and dorsal surface of foot. Objecti-
percussion reveals heart border extension in vely: weakness of the anterior tibial muscle,
both directions. What is the optimal treatment long extensor muscle of the right toes, short
tactis for this patient? extensor muscle of the right toes. Low Achi-
lles reflex on the right. Positive Lasegue’s si-
A. Pericardiocentesis and immediate gn. What examination method would be most
thoracotomy effective for specification of the diagnosis of
B. Oxygen inhalation L5 root discogenic compression?
C. Puncture of the pleural cavity on the left
D. Conservative treatment, infusion of A. Magnetic resonance scan
adrenomimetics B. Spinal column X-ray
E. Pleural cavity drainage C. Electromyography
D. Angiography
125. A 45-year-old man was delivered to E. Lumbar puncture
a hospital with complaints of vomiting wi-
th streaks of blood, loss of weight. On 129. A 23-year-old female patient has been
esophagofiberscopy a cauliflower-shaped suffering from mental disorder since the
mucosal growth was detected in the abdomi- age of 18, the course of disorder has no
nal esophagus. The mucosa there bleeds on remission periods. At hospital the patient
contact. What preliminary diagnosis can be mostly presents with non-purposeful fooli-
made? sh excitation: she makes stereotypic gri-
maces, exposes herself, publically masturbati-
A. Esophageal tumor ng, loudly laughs, repeating stereotypical
B. Barrett esophagus obscene shouts. The patient should be prescri-
C. Abdominal esophagitis bed:
D. Esophageal diverticulum
E. Esophageal achalasia A. Neuroleptics
B. Antidepressants
126. A 35-year-old man complains of intense C. Tranquilizers
lumbar pain irradiating to the inguinal area, D. Nootropics
external genitalia, thigh; frequent urination, E. Mood stabilizers
chill, nausea, vomiting. Objectively: positive
Pasternatsky’s symptom (costovertebral angle 130. During preventive ultrasound scan of
tenderness). Urine analysis revealed that abdomen performed during regular check-
RBCs and WBCs covered the total field of up in a school the following was revealed in
microscope; the urine exhibited high protein an 11-year-old student of the 5th grade: the
concentration. These clinical presentations left kidney is 3 cm below the normal posi-
were most likely caused by the following tion, its shape, size and structure are within
pathology: the norm, the contralateral kidney cannot be
observed at its proper place. The preliminary
A. Urolithiasis, renal colic diagnosis is as follows: congenital anomaly of
B. Cholelithiasis, biliary colic renal development, dystopic left kidney, right
C. Renal infarct kidney is absent or pelvic dystopic. What X-
D. Intestinal obstruction ray method would be required for making the
E. Osteochondrosis, acute radicular syndrome final diagnosis and determining the functional
capacity of both kidneys?
127. A 20-year-old patient complains of pain
in the left lumbar region, arterial pressure rise A. Renal dynamic scintigraphy
up to 160/110 mm Hg. US revealed that the B. Radioimmunoassay
structure and size of the right kidney were wi- C. Radionuclide renography
thin age norms, there were signs of 3 degree D. Thermography
hydronephrotic transformation of the left ki- E. Excretory urography
dney. Doppler examination revealed an addi-
tional artery running to the lower pole of the 131. A young woman suffering from
kidney. Excretory urogram shows a narrowing seborrhea oleosa has numerous light-brown
in the region of ureteropelvic junction. Speci- and white spots on the skin of her torso
fy the treatment tactics: and shoulders. The spots have clear margi-
ns, branny desquamation, no itching. What
A. Surgical intervention provisional diagnosis can be made?
B. Administration of spasmolytics
C. Administration of ACE inhibitors A. Pityriasis versicolor
D. Kidney catheterization B. Torso dermatophytosis
E. Administration of β -blockers C. Seborrheic dermatitis
D. Pityriasis rosea
128. As a result of load lifting a 68-year-old E. Vitiligo
woman developed acute pain in the lumbar
region, in a buttock, posterolateral surface 132. In a traffic accident two persons died.
of her right thigh, external surface of the ri- An appointed forensic medical expert was
Крок 2 Medicine (англомовний варiант, iноземнi студенти) 2016 рiк 17

called on duty to another accident locati- A. Hypothyroid


on; therefore, according to the crininal law B. Hypo-ovarian
in force in Ukraine, an investigator called in C. Hypothalamic-pituitary
the following specialist for examination of the D. Alimentary and constitutive
location and bodies: E. Hypercorticoid

A. Nearest available medical doctor 136. A 10 week pregnant woman was admi-
B. Medical assistant tted to a hospital for recurrent pain in the
C. Surgical nurse lower abdomen, bloody discharges from the
D. Nearest available dispensing chemist genital tracts. The problems developed after
E. Nearest available dental mechanic a case of URTI. The woman was registered
for antenatal care. Speculum examination
133. A patient is 28 years old. He has been revealed cyanosis of vaginal mucosa, clean
suffering from mental disorder since he was cervix, open cervical canal discharging blood
22. His current condition has changed acutely: and blood clots; the lower pole of the gestati-
for 3 days the patient has been refusing to onal sac was visible. What tactics should be
leave his home. He claims that there is a chosen?
”telepathy” occurring between him and other
people, through which he receives ”thoughts A. Curettage of the uterus
of strangers” and transmits his own thoughts B. Pregnancy maintenance therapy
for everyone to hear. He thinks his thoughts C. Expectant management, surveillance
and actions are manipulated through this D. Hysterectomy
”telepathy”. Make the preliminary diagnosis: E. Antiviral therapy

A. Paranoid schizophrenia 137. A patient with fibromyoma of uterus si-


B. Depressive episode zed up to 8-9 weeks of pregnancy consulted
C. Manic episode a gynaecologist about acute pain in the lower
D. Organic delirium abdomen. Examination revealed pronounced
E. Acute reaction to stress positive symptoms of peritoneal irritation, hi-
gh leukocytosis. Vaginal examination revealed
134. A 40-year-old patient is registered in a that the uterus was enlarged correspondi-
narcological dispensary. Somatically: skin is ng to 9 weeks of pregnancy due to the fi-
dramatically hyperemic, sclera are injected, bromatous nodes, one of which was mobile
hyperhidrosis is present. BP- 140/100 mm and extremely painful. Appendages were not
Hg, heart rate - 100/min. Mental state: palpable. There were moderate mucous di-
autopsychic orientation is intact, allopsychic scharges. What is the optimal treatment tacti-
orientation is distorted. The patient presents cs?
with motor anxiety. There is an expression
of fear on his face. He refuses to talk about A. Urgent surgery (laparotomy)
his problems and asks to release him immedi- B. Surveillance and spasmolytic therapy
ately, because he ”may be killed”. This state C. Fractional diagnostic curettage of the uteri-
developed in a day after one of his regular ne cavity
drinking bouts. What is your provisional di- D. Surgical laparoscopy
agnosis? E. Surveillance and antibacterial therapy
A. Delirium tremens 138. A 4-month-old child was admitted to
B. Organic delirium a surgical department 8 hours after the first
C. Paranoia attack of anxiety. The attacks happen every
D. Alcoholic hallucinosis 10 minutes and last for 2-3 minutes, vomi-
E. Alcoholic paranoid ting occurred once. Objectively: the child’s
condition is grave. Abdomen is soft, palpation
135. A 35-year-old female patient has gai- reveals a tumor-like formation in the right ili-
ned 20 kg weight within a year with the ac area. After rectal examination the doctor’s
normal diet. She complains of chill, sleepi- finger was stained with blood. What is the
ness, dyspnea. The patient’s mother and sister most probable diagnosis?
are corpulent. Objectively: height - 160 cm,
weight - 92 kg, BMI- 35,9. Obesity is uniform, A. Ileocecal invagination
there are no striae. The face is amimic. The B. Gastrointestinal haemorrhage
skin is dry. The tongue is thickened. Heart C. Wilm’s tumour
sounds are muffled. Heart rate - 56/min., D. Helminthic invasion
BP- 140/100 mm Hg. The patient has consti- E. Pylorostenosis
pations, amenorrhea for 5 months. TSH- 28
mkME/l (normal rate - 0,32-5). Craniogram 139. A woman has focal encephalitis in the
shows no pathology. What is the etiology of anamnesis. Her spatial orientation is not di-
obesity in this case? srupted. She has a sensation, as if ”everythi-
ng seems surreal: buildings are small, round
or distorted; trees are upside down; people
are very tall with thin limbs”. Determine the
Крок 2 Medicine (англомовний варiант, iноземнi студенти) 2016 рiк 18

psychopathologic syndrome: A. Streptoderma


B. Herpes
A. Derealization C. Atopic cheilitis
B. Depersonalization D. Dermatitis
C. Hallucinatory E. Eczema
D. Oneiric
E. Cenestopathic 144. After a contact with chemicals a plant
worker has suddenly developed stridor, voi-
140. A 56-year-old woman complains of itchi- ce hoarseness, barking cough, progressi-
ng skin of her torso, constant nausea, consti- ng dyspnea. Objective examination reveals
pation, sensation of heaviness and pain in acrocyanosis. What is your provisional di-
the right subcostal area, extreme general fati- agnosis?
gue. The patient suffers from biliary cirrhosis.
The skin is pale icteric. The abdomen is soft, A. Laryngeal edema
the liver protrudes 2,0 cm from under the B. Laryngeal carcinoma
margin of the right costal arch, sensitive on C. PATE
palpation. Biochemical investigation: total bi- D. Pulmonary atelectasis
lirubin - 142,0 mcmol/l, conjugated bilirubin E. Pneumothorax
- 139,0 mcmol/l, alanine aminotransferase -
0,98 mmol/hour·l, aspartate aminotransferase 145. An 18-year-old woman complains of
- 0,82 mmol/hour·l, alkaline phosphatase - 8,7 pains in her lower abdomen, purulent di-
mmol/hour·l. What drug should be prescribed scharge from the vagina, temperature rise
in the first place? up to 37, 8o C . Anamnesis states that she
had random sexual contact the day before
A. Ursodeoxycholic acid the signs appeared. She was diagnosed wi-
B. Sirepar th acute bilateral adnexitis. On additional
C. Allochol examination: leukocytes in the all field of visi-
D. Essentiale forte (Phospholipides) on, bacteria, diplococci with intracellular and
E. Livolin forte extracellular position. What agent is most li-
kely in the given case?
141. A 40-year-old female patient complai-
ns of having a bulge on the anterior surface A. Gonococcus
of neck for 5 years. Objectively: Ps- 72/min., B. Colibacillus
arterial pressure - 110/70 mm Hg, in the ri- C. Chlamydia
ght lobe of thyroid gland palpation reveals a D. Trichomonad
mobile node 4x2 cm in size, the left lobe is E. Staphylococcus
not palpable, the basal metabolic rate is 6%.
What is the most likely diagnosis? 146. A 45-year-old man has been suffering
from duodenal ulcer disease for 5 years. He
A. Nodular euthyroid goiter complains of weakness, dizziness, dryness
B. Nodular hyperthyroid goiter of the skin. Objectively: the skin and vi-
C. Riedel’s thyroiditis sible mucosa are pale, chapped lips; heart
D. Mixed euthyroid goiter rate is 100/min., BP- 100/70 mm Hg, systolic
E. Median cervical cyst murmur at all points on heart auscultati-
on. All other internal organs are unchanged.
142. Estimation of community health level Fecal occult blood test is positive. Blood test:
involved analysis of a report on diseases regi- erythrocytes - 3, 1 · 1012 /l, Hb- 88 g/l, color
stered among the population of district under index - 0,7, leukocytes - 4, 6 · 109 /l, platelets -
charge (reporting form 12). What index is 350·109 /l, ESR- 21 mm/hour, anisocytosis, poi-
calculated based on this report? kilocythemia, serum iron - 9,5 mcmol/l. What
A. Common morbidity rate treatment tactics would you choose?
B. Index of pathological affection A. Iron preparations, balanced diet
C. Index of morbidity with temporary disabili- B. Concentrated red cells transfusion
ty C. Intramuscular introduction of 500 mkg of
D. Index of hospitalized morbidity cyanocobalamin
E. Index of basic non-epidemic morbidity D. Corticosteroids, cytostatics
143. A 32-year-old woman addressed a E. Ascorbic acid, calcium chloride
dermatologist with complaints of slightly 147. A 58-year-old patient complains of a
itching rashes in the mouth angles. She has headache in the occipital region, nausea,
been suffering from this condition for 3 choking, opplotentes. The presentations
days. Objectively: there are isolated small appeared after a physical exertion. Objecti-
phlyctenas and superficial erosions covered vely: the patient is excited. Face is hyperemic.
in honey-yellow scabs against the background Skin is pale. Heart sounds are regular, the
of slight hyperemia. Make the diagnosis: 2nd aortic sound is accentuated. BP- 240/120
mm Hg, HR- 92/min. Auscultation reveals
some fine moist crackles in the lower lungs.
Крок 2 Medicine (англомовний варiант, iноземнi студенти) 2016 рiк 19

Liver is not enlarged. ECG shows signs of apex beat to the left. What is the most likely
hypertrophy and left ventricular overload. diagnosis?
What is the most likely diagnosis?
A. Acute cardiac aneurysm
A. Complicated hypertensic crisis B. Recurrent myocardial infarction
B. Acute myocardial infarction, pulmonary C. Acute pericarditis
edema D. Cardiosclerotic aneurysm
C. Bronchial asthma exacerbation E. Pulmonary embolism
D. Uncomplicated hypertensic crisis
E. Community-acquired pneumonia 152. A 60-year-old patient complains of nearly
permanent sensation of heaviness and fullness
148. A 45-year-old patient (14-year-long work in the epigastrium, which increases after eati-
record as a house painter) upon the contact ng, foul-smelling eructation, occasional vomi-
with synthetic paint develops skin reddeni- ting with food consumed 1-2 days ago, wei-
ng, edema, severe itching and oozing lesi- ght loss. 12 years ago he was first found to
ons on her face. Symptoms disappear after have an ulcer of pyloric channel. The patient
the contact with this chemical substance has taken ranitidine for periodic hunger pains.
stops but even the smell of paint alone is The patient’s condition has been deteriorating
enough to make them reappear each time. over the last 3 months. Objectively: splashing
Each recurrence is characterised by increased sound in the epigastrium is present. What ki-
severity of symptoms. What provisional di- nd of complication is it?
agnosis can be made?
A. Pyloric stenosis
A. Professional eczema B. Penetration of gastric ulcer
B. Simple contact dermatitis C. Functional pyloric spasm
C. Allergic contact dermatitis D. Foreign body in the stomach (bezoar)
D. Urticaria E. Malignization of gastric ulcer
E. Toxicodermia
153. A 5-year-old child has body temperature
149. A 32-year-old woman addressed a increased up to febrile numbers, suffers from
maternity clinic with complaints of inferti- inertness, weakness. Examination revealed
lity that has been lasting for 7 years. hemorrhage on the skin of limbs and torso.
Her menstrual cycle occurs in two phases. Enlargement of cervical and axillary lymph
Hysterosalpingography reveals obstruction of nodes can be detected. The liver is 4 cm
the uterine tubes in the ampullar areas, an below the costal arch; the spleen is 6 cm
adhesive process in the small pelvis can be below the costal arch. Blood test: erythrocytes
observed. What treatment is most advisable - 2, 3 · 1012 /l, Hb- 60 g/l, platelets - 40 ·
in this case? 109 /l, leukocytes - 32, 8 · 109 /l, eosinophiles
- 1%, band neutrophiles - 1%, segmented
A. Laparoscopy neutrophiles - 12%, lymphocytes - 46%,
B. Laparotomy monocytes - 1%, blasts - 40%, Duke’s bleedi-
C. Tubectomy ng time is 9 min. What examination is
D. Adnexectomy necessary to make the diagnosis?
E. Hydrotubation
A. Myelogram
150. Survey radiograph of a 52-year-old B. Lymph nodes biopsy
worker of an agglomeration plant (28 years C. US of abdominal cavity
of experience, the concentration of metal dust D. Detection of hepatitis markers
is 22-37 mg/m3 ) shows mildly pronounced E. Investigation of platelets dynamic functions
interstitial fibrosis with diffused contrast well-
defined small nodular shadows. The patient 154. A child from primipregnancy was born in
has no complaints. Pulmonary function is a term labor and has body weight of 4000 g
not compromised. What is the provisional di- and body length of 57 cm. When born, he was
agnosis? nonresponsive to examination. Diagniosis is
diffuse. Heart rate is 80/min. What resuscitati-
A. Siderosis on measures should be taken?
B. Silicosis
C. Anthraco-silicatosis A. Begin ALV with mask
D. Silicatosis B. Introduce 100% oxygen
E. Anthracosis C. Intubate and begin ALV
D. Tactile stimulation
151. A week ago a 65-year-old patient E. Administer naloxone
suffered an acute myocardial infarction, his
general condition deteriorated: he complai- 155. A 15-year-old teenager has undergone
ns of dyspnea at rest, pronounced weakness. medical examination in military recruitment
Objectively: edema of the lower extremities, center. The following was revealed: interval
ascites is present. Heart borders are extended, systolic murmur at the cardiac apex, accent
paradoxical pulse is observed 2 cm from the of the II heart sound over the pulmonary
Крок 2 Medicine (англомовний варiант, iноземнi студенти) 2016 рiк 20

artery, tachycardia. What additional exami- is noted - lacrimation and itching. What provi-
nation method will be most informative for sional diagnosis can be suggested?
determining a diagnosis?
A. Reactive arthritis
A. Echocardiography B. Rheumatoid arthritis
B. Electrocardiography C. Seasonal pollinosis
C. X-ray D. Bacterial nonspecific urethral conjunctivitis
D. Phonocardiography E. URTI with conjunctiva and joints affected
E. Rheography
160. A 19-year-old patient complains of severe
156. A 64-year-old patient has been hospi- pain in axillary crease. Condition onset was a
talised with complaints of progressive jaundi- week ago after her swimming in a cold ri-
ce that developed over 3 weeks ago without ver and epilation. The next day painful ”boi-
pain syndrome, along with general weakness, l” appeared that was becoming larger every
loss of appetite. Objectively: temperature is day and became a plum-sized tumor. Upon
36, 8o C , heart rate is 78/min., abdomen is soft examination nodular conical growths joined
and painless, peritoneum irritation symptoms together are detected, the skin covering them
are not detected, palpation reveals sharply is bluish-red in color. Some nodules have fi-
enlarged tense gallbladder. What disease can stulous openings producing thick purulent
be characterised with these symptoms? mass. Body temperature is 38, 5o C , general
malaise. The most likely diagnosis is:
A. Cancer of pancreas head
B. Duodenal ulcer A. Hydradenitis
C. Acute cholecystitis B. Carbuncle
D. Chronic cholecystitis C. Cutaneous tuberculosis
E. Cholecystitis caused by lambliasis D. Necrotizing ulcerative trichophytosis
E. Pyoderma chancriformis
157. A 6-year-old girl attended a general
practitioner with her mother. The child 161. Annual report of an in-patient ward
complains of burning pain and itching in her presents data about the number of patient
external genitalia. The girl was taking antibi- days and the number of patients, who have
otics the day before due to her suffering from undergone treatment within a year. What
acute bronchitis. On examination: external work indicator of the in-patient ward can be
genitalia are swollen, hyperemic, there is whi- calculated based on these data?
te deposit accumulated in the folds. The most
likely diagnosis is: A. Average duration of in-patient treatment of
a patient
A. Candidal vulvovaginitis B. Bed occupancy rate
B. Trichomoniasis C. Hospital bed turnover rate
C. Nonspecific vulvitis D. Mortality
D. Helminthic invasion E. Efficient use of the hospital bed capacity
E. Herpes vulvitis
162. ECG revealed the following in a 10-
158. A patient with otopyosis is in sharply year-old child: sharp acceleration of the heart
deteriorating condition: he developed rate - 240/min., P wave overlaps with T wave
headache, vomiting, febrile temperature, and deforms it, moderate lengthening of PQ
general hyperesthesia. Meningeal symptoms interval, QRS complex is without changes.
and stagnant optic disks are observed. There What pathology does this child have?
are no focal symptoms. Liquor is turbid, blood
pressure is high, albuminocytological dissoci- A. Paroxysmal atrial tachycardia
ation occurs with neutrophils. What disease B. Atrial hypertrophy
can be suspected? C. Ventricular hypertrophy
D. WPW syndrome
A. Secondary purulent meningitis E. Extrasystole
B. Meningoencephalitis
C. Serous meningitis 163. A 54-year-old patient complains of
D. Primary purulent meningitis weakness, jaundice, itching skin. Disease
E. Subarachnoid hemorrhage onset was 1,5 months ago: fever up to 39o C
appeared at first, with progressive jaundice
159. A 25-year-old patient is not married and developed 2 weeks later. On hospitalisati-
has sexual relations with several partners. on jaundice was severely progressed. Liver
During the last 3 months he noticed small cannot be palpated. Gallbladder is enlarged
amount of mucoserous secretions produced and painless. Blood bilirubin is 190 mi-
from urethra. Subjectively: periodical itching cromole/l (accounting for direct bilirubin).
or burning pain in urethra. Two months ago Stool is acholic. What is the most likely jaundi-
pain in the knee join developed. Possibility of ce genesis in this patient?
trauma or exposure to cold is denied by the
patient. During the last week eye discomfort
Крок 2 Medicine (англомовний варiант, iноземнi студенти) 2016 рiк 21

A. Mechanical jaundice A. Neuroleptics with a sedative effect


B. Hepatocellular jaundice B. Antidepressants with an activating effect
C. Hemolytic jaundice C. Neuroleptics with an activating effect
D. Caroli syndrome D. Tranquilizers
E. Gilbert’s syndrome E. Antidepressants with a sedative effect
164. A 22-year-old patient complains of 8- 168. A 32-year-old man complains of pain in
month delay of menstruation. Anamnesis: the chest on the left, dyspnea, temperature
menarche since the age of 12,5. Since the rise up to 38, 0o C , slight cough. The disease
age of 18 menstruations are irregular. No onset was 2 weeks ago after overexposure to
pregnancies. Mammary glands have normal cold. He had suffered from bronchoadenitis
development; when the nipples are pressed, in his childhood. The affected side lags during
milk drops are discharged. On gynecologi- breating; percussion reveals dull sound with
cal examination: moderate uterus hypoplasia. oblique margin in the lower left lung, where
On hormonal examination: prolactin level breathing is absent. The right heart border is
exceeds the norm two times. On computed displaced outwards. Mantoux test with 2 TU
tomogram of the sellar region: a space- resulted in a papule 16 mm in size. What di-
occupying lesion 4 mm in diameter is agnosis is most likely?
detected. The most likely diagnosis is:
A. Tuberculous pleurisy
A. Pituitary tumor B. Central carcinoma of the left lung
B. Lactation amenorrhea C. Congestion pneumonia
C. Stein-Leventhal syndrome D. Thromboembolism of the pulmonary artery
D. Sheehan’s syndrome branches
E. Cushing’s disease E. Community-acquired pneumonia
165. A 2-year-old child in a satisfactory condi- 169. An emergency doctor has diagnosed a
tion periodically presents with moderate 32-year-old woman with generalized convulsi-
proteinuria, microhematuria. US results: the ve status epilepticus. The deterioration in the
left kidney is undetectable, the right one is patient’s condition is caused by a sudden gap
enlarged, there are signs of double pyelocali- in the epilepsy treatment. Specify the doctor’s
ceal system. What study is required to specify further tactics:
the diagnosis?
A. Hospitalization in the intensive care unit
A. Excretory urography B. Hospitalization in the department of
B. Micturating cystography neurology
C. Retrograde urography C. Hospitalization in the department of
D. Doppler study of renal vessels neurosurgery
E. Radioisotope renal scan D. Outpatient monitoring by a neuropathologi-
st
166. A 13-year-old girl complains of fatigabili- E. Outpatient monitoring by a neurosurgeon
ty, frequent headaches, cardialgia. Eight years
ago she had a case of pyelonephritis. Urine 170. A 19-year-old patient complains of
analyses periodically revealed leukocyturia. dyspnea during physical exertion. He often
The child has undergone no further treatment. has bronchitis and pneumonia. Cardiac
On examination: increased BP up to 150/100 murmur has been observed since his chi-
mm Hg. Ultrasound investigation revealed si- ldhood. On auscultation: there is splitting of
gnificant reduction of the right kidney. What the II heart sound over pulmonary arteria,
process is leading in arterial hypertension systolic murmur in the third intercostal space
pathogenesis in this case? near the left sternum margin. ECG test shows
right bundle-branch block. What is the provi-
A. Hyperactivity of renin-angiotensin system sional diagnosis?
B. Disruption of water-electrolytic balance
C. Disruption of renal circulation A. Interatrial septum defect
D. Hypersympathicotonia B. Open arterial duct
E. Increased cortisol level C. Coarctation of aorta
D. Aortic stenosis
167. A 26-year-old patient with affective bi- E. Mitral valve insufficiency
polar disorder has developed a condition
manifested by mood improvement, behavi- 171. A 75-year-old male patient complains
oural and sexual hyperactivity, verbosity, acti- of slight pain in the right iliac region. The
ve body language, reduced need for sleep. abdominal pain arose 6 days ago and was
Which of the following drugs are most effi- accompanied by nausea. Surgical examination
cient in this case? revealed moist tongue, Ps- 76/min., BP- 130/80
mm Hg. Abdomen was soft, slightly painful in
the right iliac region on deep palpation, the
symptoms of the peritoneum irritation were
doubtful. In blood: RBCs- 4, 0 · 1012 /l, Hb- 135
Крок 2 Medicine (англомовний варiант, iноземнi студенти) 2016 рiк 22

g/l, WBCs- 9, 5 · 109 /l, stab neutrophils - 5%, lymph nodes are observed. On mammary
segmentonuclear - 52%, lymphocytes - 38%, glands US: in the upper external quadrant of
monocytes - 5%, ESR- 20 mm/hour. Specify the right mammary gland there is a space-
the doctor’s further tactics: occupying lesion of increased echogenicity
21х18 mm in size. The most likely diagnosis
A. Emergency operation for acute appendicitis is:
B. Hospitalization, dynamic surveillance
C. Send the patient home A. Fibrous adenoma
D. Refer the patient to a district therapist B. Lacteal cyst
E. Administration of additional examination: C. Diffuse mastopathy
abdominal ultrasound, x-ray contrast study of D. Breast cancer
the gastrointestinal tract E. Mastitis
172. An 8-year-old boy has a 2-year history of 176. 2 weeks after labour a parturient
blotchy itchy rash appearing after eating ci- woman developed breast pain being observed
trus fruit. The first eruption occurred at the for 3 days. Examination revealed body
age of 6 months after the introduction of jui- temperature at the rate of 39o C , chills,
ces to the baby’s diet. Father has a history of weakness, hyperaemia, enlargement, pain and
bronchial asthma, mother - that of allergic rhi- deformity of the mammary gland. On palpati-
nitis. What is the most likely diagnosis? on the infiltrate was found to have an area of
softening and fluctuation. What is the most
A. Atopic dermatitis likely diagnosis?
B. Psoriasis
C. Pityriasis Rosea A. Infiltrative-purulent mastitis
D. Urticaria B. Phlegmonous mastitis
E. Quincke’s edema C. Lactostasis
D. Serous mastitis
173. A 25-year-old patient has been admi- E. Mastopathy
tted to the hospital with the following
problems: weakness, sweating, itching, wei- 177. A 48-year-old woman has been suffering
ght loss, enlarged submandibular, cervical, from chronic pancreatitis for the last 7 years.
axillary, inguinal lymph nodes. Objectively: Lately she has been noticing an increase in
hepatomegaly. Lymph node biopsy revealed daily feces with foul smell, abdominal di-
giant Berezovsky-Reed-Sternberg cells, stention, gurgling. The patient complains of
polymorphocellular granuloma composed of diarrhea, weakness, fatigability, loss of appeti-
lymphocytes, reticular cells, neutrophils, eosi- te, loss of weight. What syndrome can be
nophils, fibrous tissue, plasma cells. What is suspected in this case?
the most likely diagnosis?
A. Malabsorption
A. Lymphogranulomatosis B. Irritable colon
B. Lymph node tuberculosis C. Maldigestion
C. Lymphoreticulosarcoma D. Exudative enteropathy
D. Cancer metastases to lymph nodes E. Endocrine gland failure
E. Macofollicular reticulosis
178. A 37-year-old man suffers from attacks
174. An 8-year-old child was hospitalized of unconsciousness, dyspnea during physi-
for fever up to 39, 8o C , inertness, moderate cal exertion, periodical sensations of heart
headache, vomiting. Examination revealed rate disorder. Father of the patient died
meningeal symptoms. Lumbar puncture was suddenly at the age of 45. Objectively: heart
performed. The obtained fluid had raised rate is 90/min., BP is 140/90 mm Hg. On
opening pressure, it was transparent, with heart US: ejection fraction - 49%, significant
the cell count of 450 cells per 1 mcl (mai- myocardium thickening of the left ventri-
nly lymphocytes - 90%), glucose level of 2,6 cle and interventricular septum. What drug
mmol/l. What causative agent might have should be prescribed for the treatment?
caused the disease in the child?
A. Bisoprolol
A. Enterovirus B. Enalapril
B. Meningococcus C. Phenyhydinum (Nifedipine)
C. Koch’s bacillus D. Hydrochlorothiazide
D. Staphylococcus E. Furosemide
E. Pneumococcus
179. During a regular check-up of a 50-year-
175. A 25-year-old patient during self- old woman a tumor was detected in her ri-
examination detected a tumor in the upper ght mammary gland. The tumor is 5 cm in
external quadrant of the right mammary diameter, dense, without clear margins. The
gland. On palpation: painless, dense, mobi- skin over the tumor resembles lemon rind,
le growth 2 cm in diameter is detected in the the nipple is inverted. The lymph node can
mammary gland; no changes in the peripheral be palpated in the axillary region. What di-
Крок 2 Medicine (англомовний варiант, iноземнi студенти) 2016 рiк 23

agnosis is most likely? muffled, extrasystole is present, auscultati-


on reveals apical systolic murmur that is not
A. Breast cancer conducted to the left inguinal region. ESR is
B. Lacteal cyst 38 mm/hour. CRP is 2+, antistreptolysin O ti-
C. Diffuse mastopathy tre - 400. What is the most likely diagnosis?
D. Mastitis
E. Breast lipoma A. Acute rheumatic fever
B. Vegetative dysfunction
180. A 20-year-old woman complains of C. Non-rheumatic carditis
sensation of air shortage, lingering dull pain D. Juvenile rheumatoid arthritis
in the heart area, irritability. Objectively: E. Reactive arthritis
general condition is satisfactory, heart rate
lability, BP is 130/60 mm Hg. ECG reveals di- 184. An 18-year-old woman complains
sruption of repolarization proceses. The pati- of mammary glands swelling, headaches,
ent is diagnosed with somatoform autonomic tearfulness, abdominal distension occurring
dysfunction of cardial type. Specify the condi- the day before menstruation. The symptoms
tions of the patient treatment: disappear with the beginning of menstruation.
Menstruations are regular, last for 5-6 days wi-
A. Out-patient treatment th interval of 28 days in between. Gynecologi-
B. In-patient treatment in therapeutics cal examination revealed no changes of
department internal genitals. What is your diagnosis?
C. In-patient treatment in cardiology
department A. Premenstrual syndrome
D. In-patient treatment in cardiac surgery B. Sheehan syndrome
department C. Stein-Leventhal syndrome
E. In-patient treatment in psychiatric D. Asherman’s syndrome
department E. Adrenogenital syndrome
181. A 38-year-old man’s workplace is within 185. A 24-year-old patient had been delivered
the area of effect of ionizing radiation. Duri- to a thoracic department with a chest injury,
ng regular medical check-up he expresses a fracture of the IV, V, VI ribs on the right.
no complaints. Blood test: erythrocytes - Plan radiography shows the fluid level in the
4, 5 · 1012 /l, Нb- 80 g/l, leukocytes - 2, 8 · 109 /l, pleural cavity reaching the III rib on the right.
platelets - 30 · 109 /l. Can this person continue Puncture blood contained clots. What is the
to work with sources of ionizing radiation? optimal treatment tactics?

A. Work with radioactive substances and other A. Emergency thoracotomy


sources of radiation is contraindicated B. Pleural puncture
B. Work with radioactive substances is allowed C. Thoracentesis and thoracostomy
C. Only work with low-level radioactive D. Hemostatic therapy
substances is allowed E. Medical thoracoscopy
D. Work with radioactive substances can be 186. A 30-year-old man was delivered to
allowed after detailed medical examination a neurosurgical department with complai-
E. Work with radioactive substances is allowed nts of constant headaches, nausea, vomi-
only for limited periods of time ting, fever, weakness of the right-side li-
182. A maternity patient breastfeeding for 1,5 mbs. Anamnesis states that one month ago
weeks has attended a doctor. She considers the patient had a surgery for left-sided
the onset of her disease to be when proporti- suppurative otitis and mastoiditis. He has
onal breast engorgement occurred. Mammary been undergoing treatment in an ENT
glands are painful. Body temperature is department. Approximately 2 weeks ago
36, 6o C . Expression of breast milk is hindered. the temperature increased, and the patient
The most likely diagnosis is: developed headaches. Objectively: heart rate
- 98/min., BP- 140/90 mm Hg, temperature -
A. Lactostasis 38, 3o C . Neurologically pronounced stiff neck:
B. Infiltrative mastitis bilateral Kernig’s symptom, unsteadiness
C. Suppurative mastitis during the Romberg’s maneuver. Computer
D. Chronic cystic mastitis tomography of the brain revealed a three-
E. Gangrenous mastitis dimensional growth with a capsule in the left
hemisphere. Make the diagnosis:
183. An 11-year-old boy complains of general
weakness, fever up to 38, 2o C , pain and swelli- A. Cerebral abscess
ng of the knee joints, sensation of irregular B. Echinococcus
heartbeat. 3 weeks ago, the child had a case C. Hemorrhage
of tonsillitis. Knee joints are swollen, the D. Hydrocephalus
overlying skin and skin of the knee region E. Arnold-Chiari malformation
is reddened, local temperature is increased,
movements are limited. Heart sounds are 187. Examination of a Rh-negative pregnant
Крок 2 Medicine (англомовний варiант, iноземнi студенти) 2016 рiк 24

woman at 32 weeks of gestation revealed A. Mitral valve stenosis


a four-time rise of Rh-antibody titer wi- B. Mitral valve failure
thin 2 weeks, the titer was 1:64. In the first C. Aortic outflow stenosis
two pregnancies the patient had experienced D. Aortic valve failure
antenatal fetal death due to hemolytic di- E. Tricuspid valve stenosis
sease. What is the optimal tactics of pregnancy
management? 191. A 64-year-old woman has been suffering
from diabetes mellitus for the last 14 years.
A. Early delivery Approximately 3 days ago the skin on the
B. Delivery at 37 weeks of gestation distal phalanx of the I toe on the left foot
C. Screening for Rh-antibodies 2 weeks later became cold and bluish-black in color. Mild
and early delivery in case of further titer rise pain is observed in the affected area. Pulse on
D. Introduction of anti-Rh (D) the pedal arteries cannot be detected, pulse
immunoglobulin on the popliteal artery is retained. Glycemia
E. Ultrasound for signs of hemolytic disease of is 12,4 mmol/l. US scan: stenosis of the shin
the fetus arteries, collateral compensated blood flow.
Ankle-brachial pressure index is 0,7. Foot X-
188. A 74-year-old patient visited a urologi- ray: destruction of the distal phalanx of the
st with complaints of pain above the pubis I toe. Determine the grade of diabetic foot
and inability to urinate for 8 hours. At home according to Wagner:
he had taken antispasmodics and had a
warm bath but no improvement occurred. A. IV
Objectively: the abdomen is soft and pai- B. I
nful above the pubis; dullness of percussion C. II
sound is observed above the pubis. Murphy’s D. III
(Pasternatski’s) punch sign is negative on the E. V
both sides. What condition does the patient 192. A 4-month-old boy has been undergoi-
have? ng in-patient treatment for pneumocystic
A. Acute urinary retention pneumonia for 4 weeks. The diagnosis has
B. Paradoxal ischuria been made based on clinical signs, typi-
C. Chronic urinary retention cal X-ray presentation, presence of severe
D. Anuria hypoxemia, positive dynamics caused by
E. Oliguria intravenous introduction of Biseptol (Co-
trimoxazole). Anamnesis states that enzyme-
189. A 7-year-old child became ill again 2 linked immuno sorbent assay (ELISA)
weeks after a case of tonsillitis. There are the detected antibodies to HIV in the umbilical
following complaints: temperature rise up to blood. Polymerase chain reaction (PCR) was
38o C , hemorrhagic rash on the extremities, performed on the child at the ages of 1 month
enlargement of the ankle joints. Blood test: and 3 months, and proviral DNA was detected
hemoglobin is 120 g/l, platelets are 170 · 109 /l, in the child’s blood. Viral load and number of
ESR is 30 mm/hour. Urine test: proteinuria CD4+-lymphocytes was not measured. Make
up to 0,7 g/l, cylinders - 5-6 in the field of vi- the diagnosis:
sion, erythrocytes - 8-10 in the field of vision.
What mechanism of hemorrhagic syndrome is A. HIV/AIDS
present in the given case? B. Infectious mononucleosis
C. Pneumonia
A. Vessel wall damage caused by immune D. Tuberculosis
complexes E. Adenovirus infection
B. Platelet dysfunction
C. Suppression of hematopoietic stem cells 193. A 35-year-old patient undergoing
D. Decrease of adhesive-aggregative function treatment for heart disorder in cardiologi-
of platelets cal department has developed complaints
E. Vessel wall damage caused by bacteria of acute sudden pain in the epigastrium,
temperature rise up to 38, 3o C . Blumberg’s
190. A 33-year-old patient has developed and Razdolsky’s (abduction of femur) signs
dyspnea during physical exertion, palpitati- are positive. What necessitates surgical aid in
ons, disruptions of heart rate, swollen legs. the given case?
In the childhood the patient had a case of
acute rheumatic fever that required in-patient A. Progress as a surgical disease
treatment. There were no further requests B. Extent of congenital and acquired
for medical care. Objectively: heart rate is development disorders
92/min., rhythmic; BP is 110/70 mm Hg. At C. Severity of the concomitant pathology
the apex the I heart sound is increased, triple D. Urgent aid is required to save the patient’s
rhythm, diastolic murmur. What heart disease life
is most likely? E. Activity of the heart disorder
194. A 5-year-old girl was hospitalized with
Крок 2 Medicine (англомовний варiант, iноземнi студенти) 2016 рiк 25

complaints of pain and swelling in the right A. Doxazosin


knee joint, temperature rise up to 38, 4o C and B. Propranolol
a rash diagnosed as erythema annulare centri- C. Indapamide
fugum. The signs developed 3 days after the D. Amlodipine
recovery from a case of acute respiratory di- E. Captopril
sease. Name the etiotropic drug to be prescri-
bed: 198. On the 9th day after childbirth the
obstetric patient developed high fever up
A. Augmentin to 38o C . She complains of pain in the right
B. Methotrexate mammary gland. The examination revealed
C. Metypred (Methylprednisolone) the following: a sharply painful infiltrate can
D. Diclofenac sodium be palpated in the right mammary gland, the
E. Captopril skin over the infiltrate is red, subareolar area
and nipple are swollen and painful. What is
195. During medical examination of a group your diagnosis?
of children under 4 years carried out by a
pediatric team in one of the African countri- A. Abscess of the right mammary gland
es a set of similar pathological signs was B. Mastopathy
detected in some of the children. The si- C. Cancer of the right mammary gland
gns are as follows: growth inhibition, mental D. Serous mastitis
changes, muscle atrophy, swellings, changes E. Fibrous cystic degeneration of the right
in hair and skin pigmentation. These children mammary gland
were diagnosed with kwashiorkor. What food
products should be added to the diet to treat 199. A 16-year-old girl addressed a doctor wi-
this disorder? th complaints of fatigability and dizziness. On
heart auscultation: systolic murmur in the II
A. Fish, vegetables, cereals intercostal area along the breastbone edge
B. Milk, meat, vegetables on the left. ECG revealed signs of the right
C. Vegetables, fruit ventricular hypertrophy. X-ray revealed di-
D. Cereals, fruit, berries latation of the the pulmonary artery trunk,
E. Poultry, fruit, berries enlargement of the right heart. What heart di-
sorder is it?
196. An 8-year-old girl was diagnosed wi-
th signs of Morgagni-Adams-Stokes disease A. Pulmonary artery outflow stenosis
that developed against the background of the B. Fallot’s tetrad
III degree atrioventricular heart block. What C. Defect of the interatrial septum
drug should be introduced intravenously for D. Coarctation of aorta
emergency aid? E. Pulmonary artery valve failure

A. Atropine 200. A 32-year-old patient complains of


B. Prednisolone excessive appetite, excess weight, dyspnea
C. Dobutamine during physical exertion. There are fat deposi-
D. Digoxin ts in the area of abdomen and shoulder gi-
E. Potassium chloride rdle. The skin is pale-pink, adult male pattern
of hair distribution is observed on the torso,
197. A 62-year-old man addresed a urologist no stretch marks. Heart rate is 90/min., BP
with complaints of frequent urination at night is 120/80 mm Hg, body build index equals
(5-6 times per night), sensation of incomplete 35. Blood sugar is 4,9 mmol/l, cholesterol is
voiding of the urinary bladder, pain in the 6,2 mmol/l. On ophthalmoscopy: fundus of
lower abdomen, slow urination. Anamnesis: the eye without changes. What provisional di-
the II degree essential hypertension (peak agnosis can be made?
BP is 160/100 mm Hg). Current case: the II
degree enlargement of the prostate gland, A. Primary alimentary constitutive obesity,
PSA is 2,2 ng/ml. Select the drug suitable for android type
long-term therapy of the patient’s combined B. Primary alimentary constitutive obesity,
pathology: gynoid type
C. Secondary hypothalamic obesity
D. Secondary neuroendocrine obesity
E. Secondary endocrine hypo-ovarian obesity
MINISTRY OF PUBLIC HEALTH OF UKRAINE

Department of human resources policy, education and science

Testing Board

Student ID Last name

Variant ___________________

Test items for licensing examination

Krok 2
MEDICINE
General Instruction
Every one of these numbered questions or unfinished statements in
this chapter corresponds to answers or statements endings. Choose the
answer (finished statements) that fits best and fill in the circle with the
corresponding Latin letter on the answer sheet.

Authors of items: Ahafonova O.O., Ambrozevych Z.M., Barannyk S.I., Berezniuk V.V., Berezov V.M.,
Bilenko O.A., Bilyk O.V., Bilyk V.D., Blikhar V.Ye., Bolbot Yu.K., Borzova O.Yu., Bukhtieieva E.R.,
Bukhtiyarova O.H., Buriak V.M., Butina L.I., Butvyn I.M., Chaika O.O., Chekanov S.L., Chelpan L.L.,
Chonka I.I., Chuiko Yu.M., Dankyna I.A., Dashchuk A.M., Demchenko T.V., Desiatska Yu.V., Drin T.M.,
Duplenko P.Yu., Dzis N.P., Filippova O.Yu., Franchuk O.A., Hahara V.F., Havrylova L.O., Henyk N.I.,
Herasymenko O.I., Hovalenkova O.L., Hrydasova V.D., Hubka O.V., Hyrla Ya.V., Kalinovska I.V.,
Kaliy V.V., Kalyberdenko V.B., Kandyba V.P., Karliychuk O.O., Kirieieva T.V., Klymenko A.V.,
Koliush O.I., Kompaniyets K.M., Kondratenko P.H., Kondratiev V.O., Konopkina L.I., Konovalova N.V.,
Kopchak O.V., Korovka S.Ya., Kovalchuk P.Ye., Kovalenko S.V., Kovtunenko R.V., Krut Yu.Ya.,
Kryvenko Z.F., Kryvonosov M.V., Kryzhanovsky Yu.M., Kudria V.I., Lavrinchuk I.O., Lakusta N.M.,
Lebediuk M.M., Leshchenko K.A., Liulka O.M., Makieieva N.I., Malovany V.V., Marushko Yu.V.,
Martsynik Ye.M., Melnychuk L.V., Mierkulova N.F., Mitiunina N.I., Miziuk V.V., Moroz I.M.,
Muravska O.M., Murtazin L.M., Mysak A.I., Nadraha O.B., Nechytailo Yu.M., Nerianov K.Yu.,
Nikolaichuk O.M., Nishkumai O.I., Nykoniuk T.R., Olifierovska R.P., Parashchuk Yu.S., Pashchenko I.V.,
Pertseva N.O., Petrynych V.V., Pisotska L.A., Pohorelov O.V., Proniv L.M., Pryshliak O.Ya.,
Radchenko O.M., Reitmaier M.Y., Riapolova T.L., Romanenko V.N., Rudai V.V., Rudenko M.M.,
Rudenko S.M., Ruzhytska O.O., Samardakova H.O., Semeniak A.V., Semenukha K.V., Sennikov I.A.,
Serheta I.V., Shapkin V.Ye., Shkrobanets I.D., Shorikov Ye.I., Shusterman T.Y., Shvyhar L.V.,
Sidykh N.M., Sierkova V.K., Sikorska M.V., Smachylo I.V., Smoliak L.L., Snizhko S.S., Soboleva N.P.,
Sokolov O.B., Soldak I.I., Sonnyk H.T., Sorochan V.V., Sotnik Yu.P., Suk V.H., Svyrydova V.V.,
Svystunov I.V., Sylenko H.Ya., Sychova V.V., Synoverska O.B., Tarasova V.I., Teliushchenko O.D.,
Tiuieva N.V., Todoriko L.D., Tokariev A.V., Tonkohlas O.A., Tretiakevych Z.M., Troian V.I.,
Tykhonova S.A., Ukhal M.I., Vankhanen N.V., Vatanska I.Yu., Volianska A.H., Voloshyna L.O.,
Vorokhta Yu.M., Vykhovanets T.A., Yakovenko I.K., Yermachenko T.P., Yevtushenko V.V.,
Yurchenko I.V., Yutanova A.V., Zakharov V.K., Zheliba M.D., Zoria A.V. and Committees of professional
expertise

Item reviewers. Agafonova O.O., Anisimov Ye.M., Bab’yak T.Ye., Chursina T.Ya., Dyndar O.A.,
Grygorov Yu.B., Grynzovs’ky A.M., Gubka V.O., Gutsalenko O.O., Kalinina S.Yu., Karapetyan K.G.,
Khrapach V.V., Kolesnyk O.M., Kolosovych I.V., Kondratyuk V.Ye., Kopcha V.S., Kravchenko O.V.,
Kutovy O.B., Kuz’mina I.Yu., Malanchuk L.M., Martynyuk L.P., Mishchenko V.P., Muravs’ka O.M.,
Petrushenko V.V., Prokhorova M.P., Pryshlyak O.Ya., Puzanova O.G., Pyptyuk O.V., Shestakova I.V.,
Shevtsova T.I., Stovban I.V., Tsvirenko S.M., Usenko S.G., Vakal’uk I.P., Volyans’ka A.G.

The book includes test items for use at licensing integrated examination “Krok 2. Medicine” and
further use in teaching.

The book has been developed for students of medical, pediatric and medical-and-prophylactic
faculties and academic staff of higher medical educational establishments.

Approved by Ministry of Public Health of Ukraine as examination and teaching


publication based on expert conclusions (Orders of MPH of Ukraine of
14.08.1998 №251, of 27.12.1999 №303, of 16.10.2002 №374, of 29.05.2003 №233).

© Copyright Testing Board.


Крок 2 Medicine (англомовний варiант, iноземнi студенти) 2017 рiк 1

1. A 32-year-old welder complains of 5. 5 weeks after hypothermia a 22-year-


weakness and fever. His illness initially old patient developed fever, weakness,
presented as tonsillitis one month earlier. On muscle pain, inability to move independently.
examination: body temperature - 38,9o C , RR- Objectively: tenderness, induration of
24/min., HR- 100/min., BP- 100/70 mm Hg, shoulder and shin muscles, restricted active
hemorrhages on the legs, enlargement of the movements, erythema on the anterior surface
lymph nodes. Complete blood count: Hb- 70 of the chest. There is a periorbital edema
g/l, RBC- 2, 2 · 1012 /l, WBC- 3, 0 · 109 /l wi- with heliotropic erythema. Gottron’s sign is
th 32% of blasts, 1% of eosinophils, 3% of present. What investigation is required for the
band neutrophils, 36% of segments, 20% of diagnosis verification?
lymphocytes, and 8% of monocytes, ESR- 47
mm/hour. What is the cause of anemia? A. Muscle biopsy
B. Aminotransferase activity
A. Acute leukemia C. Pneumoarthrography
B. Chronic lympholeukemia D. ASO titer
C. Aplastic anema E. Rheumatoid factor
D. B12 -deficient anemia
E. Chronic hemolytic anemia 6. A 37-year-old woman complains of
headaches, nausea, vomiting, spasms. The
2. After a 5-day-long celebration of his onset of the disease occurred the day before
daughter’s wedding a 65-year-old patient due to her overexposure to cold. Objecti-
”saw” in his yard many cats, chickens, and vely: fever up to 40o C ; somnolence; rigid
rats. He tried to chase them away, but was neck; Kernig’s symptom is positive on the
scared off when the animals started to scold both sides; general hyperesthesia. Blood test:
him and tried to harm him. Make the di- leucocytosis, increased ESR. Cerebrospinal
agnosis: fluid is turbid, yellow-tinted. What changes of
the cerebrospinal fluid are most likely?
A. Delirium tremens
B. Senile psychosis A. Neutrophilic pleocytosis
C. Schizophrenia B. Lymphocytic pleocytosis
D. Organic brain syndrome C. Blood in the cerebrospinal fluid
E. Reactive hallucinosis D. Xanthochromia in the cerebrospinal fluid
E. Albuminocytological dissociation
3. An 8-year-old boy developed a temperature
of 37, 5o C two days after his recovery from 7. A 44-year-old patient complains of diffi-
the case of URTI. He complains of suffocati- cult urination, sensation of incomplete uri-
on, heart pain. Objectively: the skin is pale, nary bladder emptying. Sonographic exami-
tachycardia, the I heart sound is weakened, nation of the urinary bladder near the
short systolyc murmur in the 4th intercostal urethra entrance revealed an oval well-
area near the left edge of the breastbone. defined hyperechogenic formation 2x3 cm
What heart disorder such clincal presentati- large that was changing its position during the
on is characteristic of? examination. What conclusion can be made?
A. Nonrheumatic myocarditis A. Stone
B. Primary rheumatic carditis B. Malignant tumour of the urinary bladder
C. Myocardiodystrophy C. Urinary bladder polyp
D. Fallot’s tetrad D. Prostate adenoma
E. Cardiomyopathy E. Primary ureter tumour
4. A 28-year-old woman has been delivered 8. 4 days after a patient received a gunshot
to a hospital with acute pain in the lower wound to the soft tissues of middle third
abdomen. There was a brief syncope. The of the thigh, his condition suddenly began
delay of menstruation is 2 months. Objecti- deteriorating. There are complaints of bursti-
vely: the patient has pale skin, BP- 90/50 mm ng pain in the wound; pain has been increasi-
Hg, Ps- 110/min. Lower abdomen is extremely ng during the last 12 hours. Edema of skin
painful. Vaginal examination reveals uterus and hypodermic tissue quickly grows. Body
enlargement. Promtov’s sign (pain during bi- temperature is 38,2o C , heart rate is 102/min.
manual gynecological examination) is posi- The wound edges gape, are dull in color;
tive. Right uterine appendages are enlarged the muscles, viable as of day before, now
and very painful. Posterior vault hangs over. protrude into the wound, look boiled, are dull
What is the most likely diagnosis? in color, have dirty-gray coating, and fall apart
when held with forceps. What infection has
A. Right-sided tubal pregnancy developed in the wound?
B. Right ovary apoplexy
C. Acute right-sided salpingoophoritis
D. Pelvioperitonitis
E. Incipient abortion
Крок 2 Medicine (англомовний варiант, iноземнi студенти) 2017 рiк 2

A. Anaerobic A. Surgical intervention


B. Aerobic gram-negative B. Continuation of conservative therapy
C. Putrid C. Physiotherapy
D. Aerobic gram-positive D. Sanatorium-and-spa treatment
E. Diphtheria of the wound E. Physical training
9. A 35-year-old patient has been sufferi- 13. Caries morbidity rate is 89% among resi-
ng from an illness for 3 days. 5 days ago he dents of a community. It is determined that
returned from a trip to Africa. The onset of fluorine content in water is 0,1 mg/l. What
disease was accompanied by fever up to 40o C , preventive measures should be taken?
chills, acute headache, myalgia. In the axi-
llary region the lymph node enlarged up to A. Water fluorination
3x6 cm can be palpated. The lymph node is B. Tooth brushing
dense, intensely painful, slightly mobile, wi- C. Fluorine inhalations
thout clear margins; the skin over the node D. Sealant application
is hyperenic and tight. Tachycardia is present. E. Introduction of more vegetables to the diet
Make the preliminary diagnosis:
14. A patient received flame burns of both
A. Plague hands. On the dorsal and palmar surface of
B. Sepsis the hands there are blisters filled with serous
C. Tularemia fluid. The wrist joint region is hyperemic. The
D. Lymphadenitis forearms were not injured. What is the provi-
E. Anthrax sional diagnosis?

10. A parturient woman complains of pain in A. II degree flame burn of the hands, 4% of
her mammary gland. In the painful area there body surface area
is an infiltration 3x4 cm in size with softened B. II degree flame burn of the hands, 2% of
center. Body temperature is 38,5o C . What is body surface area
the most likely diagnosis? C. IIIa degree flame burn of the hands, 4% of
body surface area
A. Acute suppurative mastitis D. III degree flame burn of the hands, 4% of
B. Pneumonia body surface area
C. Pleurisy E. IIb degree flame burn of the hands, 2% of
D. Milk retention body surface area
E. Birth trauma
15. During the periodic medical examination
11. A 52-year-old patient, who has been an assembly fitter (works on soldering detai-
suffering from angina pectoris, for 2 weeks ls) didn’t report any health problems. Closer
has increasingly frequent pain attacks in the examination revealed signs of asthenic-
area behind his sternum and his need for ni- vegetative syndrome. Blood included red
troglycerine has increased. Objectively: the blood cells with basophilic aggregations
condition is of moderate severity. The skin is and a somewhat higher number of reti-
pale. Heart sounds are weakened, rhythmic. culocytes, urine had a high concentration of
Heart rate is 84/min. ECG shows no signs of delta-aminolevulinic acid. The complex of
focal myocardial injury. What is the most li- symptoms indicates the initial stage of chronic
kely diagnosis? intoxication with:
A. Progressive angina pectoris A. Lead
B. First-time angina pectoris B. Manganese
C. Stable NYHA functional class II angina C. Mercury
pectoris D. Tin
D. Variant angina pectoris E. Ethanol
E. Acute cardiac infarction
16. A 29-year-old woman came to a
12. A 9-year-old boy has been suffering gynecologist with complaints of irritabili-
from multiple bronchiectasis since he was 3 ty, tearfulness, headache, nausea, occasional
years old. Exacerbations occur frequently (3- vomiting, pain in the heart area, tachycardia
4 times a year), after conservative therapy attacks, memory impairment, meteorism.
there are short remission periods. The di- These signs appear 6 days before menstruati-
sease progresses, the child is physically on and disappear the day before menstruation
underdeveloped, presents with pale skin, or during its first 2 days. On vaginal exami-
acrocyanosis, deformed nail plates in the nation: the uterus and uterine appendages
shape of ”clock-face”. Bronchography reveals are without alterations. What diagnosis is the
saccular bronchiectases in the lower lobe of most likely?
the right lung. What further treatment tactics
should be chosen?
Крок 2 Medicine (англомовний варiант, iноземнi студенти) 2017 рiк 3

A. Premenstrual syndrome A. Blood creatinine


B. Algodismenorrhea B. Blood bilirubin
C. Ovarian apoplexy C. Blood sodium
D. Genital endometriosis D. Uric acid
E. Neurosis E. Fibrinogen
17. Examination of a group of persons living 21. A worker of a glass-blowing workshop
on the same territory revealed the following complains of headache, irritability, visual
common symptoms: dark-yellow pigmentati- impairment - he sees everything as if through
on of the tooth enamel, diffuse osteoporosis a ”net”. Objectively: hyperemic sclera, thi-
of bone apparatus, ossification of ligaments ckened cornea, decreased opacity of pupi-
and joints, functional disorders of the central ls, visual acuity is 0,8 in the left eye, 0,7 in
nervous system. This condition may be caused the right eye. The worker uses no means of
by the excessive concentration of the followi- personal protection. What is the most likely
ng microelement in food or drinking water: diagnosis?
A. Fluorine A. Cataract
B. Copper B. Conjunctivitis
C. Nickel C. Keratitis
D. Iodine D. Blepharospasm
E. Cesium E. Progressive myopia
18. An infant has been born at the 41st week 22. For a week a 42-year-old patient has been
of gestation. The pregnancy was complicated suffering from fever attacks followed by high
with severe gestosis of the second semester. temperature, which occur each 48 hours. Body
The weight of the baby is 2400 g, the height is temperature raises up to 40o C and decreases
50 cm. Objectively: the skin is flabby, the layer in 3-4 hours with excessive sweating. The pati-
of subcutaneous fat is thin, hypomyotonia ent presents with loss of appetite and general
is observed, neonatal reflexes are weak. fatigue. The skin is pale and sallow. The li-
The internal organs are without pathologic ver and spleen are enlarged and dense on
alterations. This newborn can be assessed as palpation. What method of diagnosis verifi-
a: cation would be most efficient?
A. Full-term infant with prenatal growth A. Microscopy of blood smear and thick blood
retardation film
B. Premature infant B. Complete blood count
C. Immature infant C. Bacteriological analysis
D. Postmature infant D. Immune-enzyme assay
E. Full-term infant with normal body weight E. Microscopy of hanging blood drop
19. A patient suffering from infiltrati- 23. A 28-year-old patient is a drug addict. He
ve pulmonary tuberculosis was prescribed has been sick for a year, when noticed general
streptomycin, rifampicin, isoniazid, pyrazi- weakness, increased sweating, and weight
namide, vitamin C . One month after the begi- loss. He often had cases of respiratory di-
nning of the treatment the patient started seases. Within the last 2 days he demonstrates
complaining of reduced hearing and tinnitus. intermittent fever with profuse night sweati-
What drug has such a side effect? ng, increased general weakness, developed di-
arrhea with mucus and blood admixtures. On
A. Streptomycin examination: polylymphadenopathy, herpetic
B. Isoniazid rashes in the oral cavity; on abdominal
C. Rifampicin palpation: the liver and spleen are enlarged.
D. Pyrazinamide What is the most likely diagnosis?
E. Vitamin C
A. HIV-infection
20. A 39-year-old man complains of morni- B. Herpetic stomatitis
ng headaches, appetite loss, nausea, morni- C. Chronic lymphatic leukemia
ng vomiting, periodic nasal hemorrhages. The D. Colon cancer
patient had a case of acute glomerulonephritis E. Chronic sepsis
at the age of 15. Examination revealed rise of
arterial pressure up to 220/130 mm Hg, skin 24. A 40-year-old patient has acute onset
hemorrhages on his arms and legs, pallor of of disease caused by overexposure to cold.
skin and mucous membranes. What biochemi- Temperature has increased up to 39o C .
cal parameter is the most important for maki- Foul-smelling sputum is expectorated duri-
ng diagnosis in this case? ng coughig. Various moist crackles can be
auscultated above the 3rd segment on the ri-
ght. Blood test: leukocytes - 15, 0 · 109 /l, stab
neutrophils - 12%, ESR- 52 mm/hour. On X-
ray: in the 3rd segment on the right there is a
Крок 2 Medicine (англомовний варiант, iноземнi студенти) 2017 рiк 4

focus of shadow 3 cm in diameter, low density, A. Risk of late abortion with hemorrhage
with fuzzy smooth margins and a clearing in B. Risk of late abortion without hemorrhage
its center. What disease is most likely in the C. The process of late abortion
given case? D. Incomplete late abortion
E. Attempted late abortion
A. Pneumonia complicated by an abscess
B. Infiltrative tuberculosis 29. A 65-year-old man was diagnosed with
C. Peripheral pulmonary cancer B12 -deficient anemia and the treatment was
D. Cystic echinococcosis prescribed. A week later control blood test
E. Pulmonary cyst was performed. What would be the early indi-
cator of the therapy effectiveness?
25. A 48-year-old patient was found to have
diffuse enlargement of the thyroid gland, A. Increased number of reticulocytes
exophthalmia, weight loss of 4 kg in 2 months, B. Increased hemoglobin level
sweating. Objectively: HR- 105/min, BP- C. Megaloblastic hematopoiesis
140/70 mm Hg. Defecation act is normal. D. Normoblastic hematopoiesis
What kind of therapy is recommended in this E. Increased erythrocyte number
case?
30. A 35-year-old woman complains of heart
A. Mercazolil pain (”aching and drilling”) occurring mai-
B. Radioactive iodine nly in the morning in autumn and spring and
C. Propranolol irradiating to the neck, back and abdomen;
D. Lugol’s solution rapid heartbeat; low vitality. Occurrence of
E. Thyroxine this condition is not associated with physical
activity. In the evening, the patient’s condition
26. A 26-year-old woman is suspected to improves. Study of somatic and neurological
suffer from systemic lupus erythematosus due status and ECG reveal no pathology. What
to systemic lesions of skin, vessels, joints, pathology is most likely to have caused these
serous tunics, and heart that developed after clinical presentations?
photosensitization. The following is detected:
LE cells, antibodies to native DNA, isolated A. Somatization depression
anti-centromere antibodies, rheumatoid B. Resting stenocardia
factor is 1:100, Wassermann reaction is posi- C. Pseudoneurotic schizophrenia
tive, circulating immune complex is 120 uni- D. Neurocirculatory asthenia
ts. What immunological indicators are consi- E. Hypochondriacal depression
dered to be specific to this disease?
31. On the 4th day after recovery from a
A. DNA antibodies cold a patient was hospitalized with complai-
B. Rheumatoid factor nts of solitary spittings of mucoid sputum.
C. Anti-centromere antibodies On the 2nd day there was a single discharge
D. Immunoglobulin A of about 250 ml of purulent blood-streaked
E. Increased circulating immune complex sputum. Objectively: the patient’s conditi-
on is moderately severe. Respiratory rate -
27. A woman came to a doctor with complai- 28-30/min., Ps- 96/min., BP- 110/70 mm Hg.
nts of increased body temperature up to Respiration over the left lung is vesicular,
37,8o C and moderately sore throat for the last over the right lung - weakened. There are
3 days. Objectively: mandibular lymph nodes various moist crackles over the lower lobe
are enlarged up to 3 cm. Palatine tonsils are and amphoric breath sounds near the angle of
hypertrophied, with gray coating that spreads scapula. What is the most likely diagnosis?
to the uvula and anterior pillars of the fauces.
What is the most likely diagnosis? A. Acute pulmonary abscess
B. Exudative pleuritis
A. Oropharyngeal diphtheria C. Acute focal pneumonia
B. Infectious mononucleosis D. Pleural empyema
C. Pseudomembranous (Vincent’s) tonsillitis E. Pyopneumothorax
D. Agranulocytosis
E. Oropharyngeal candidiasis 32. A 65-year-old woman on abdominal
palpation presents with a tumor in the umbi-
28. A primigravida at the term of 20 weeks lical region and above it; the tumor is 13x8 cm
complains of pain in her lower abdomen, in size, moderately painful, non-mobile, pulsi-
smearing blood-streaked discharge from the ng. On auscultation systolic murmur can be
genital tracts. Uterine tone is increased, fetus observed. What is the most likely diagnosis?
is mobile. On vaginal examination: the uterus
is enlarged according to the term, uterine A. Abdominal aneurysm
cervix is shortened to 0,5 cm, external cervical B. Gastric tumor
orifice is open by 2 cm. What is the most likely C. Arteriovenous aneurysm
diagnosis? D. Tricuspid insufficiency
E. Bicuspid insufficiency
Крок 2 Medicine (англомовний варiант, iноземнi студенти) 2017 рiк 5

33. A 45-year-old man complains of cough fi- 37. A patient with suspected pheochromocytoma
ts and tickling in his nasopharynx. He had has normal blood pressure in the periods
been staying for 10 days in the polluted area between the atacks and a tendency towards
created by the Chornobyl nuclear power plant tachycardia. Urine test revealed no pathologi-
accident. Rhinoscopy shows signs of severe es. It was decided to use a provocative test
nasopharynx irritation. What radionuclide is with histamine. What medication should be
the cause of this irritation? prepared to provide emergency care in case
of positive test result?
A. Radioactive iodine
B. Radioactive cesium A. Phentolamine
C. Radioactive strontium B. Pipolphen
D. Radioactive plutonium C. Nifedipine
E. Radioactive cobalt D. Mesatonum
E. Prednisolone
34. A 20-year-old patient complains of severe
headache, double vision, weakness, fever, 38. A 42-year-old woman complains of severe
irritability. Objectively: body temperature is pulsing headache in the frontoparietal area,
38,1o C , the patient is reluctant to contact, vertigo, palpitations. She has been sufferi-
sensitive to stimuli. There are ptosis of ng from hypertension for 3 years. Significant
the left eyelid, exotropia, anisocoria S>D, increase of BP occurs 2-3 times per month
pronounced meningeal syndrome. On lumbar and lasts for 3-8 hours. The left ventricle is
puncture the cerebrospinal fluid flowed out enlarged, heart sounds are clear, heart rate
under a pressure of 300 mm Hg, the fluid - 105/min., BP- 225/115 mm Hg. ECG: signs
is clear, slightly opalescent. 24 hours later of left ventricular hypertrophy. What drug
there appeared fibrin film. Protein - 1,4 g/l, would be the most effective for termination
lymphocytes - 600/3 per mm3 , sugar - 0,3 of cerebral crisis attack?
mmol/l. What is the provisional diagnosis?
A. Labetalol
A. Tuberculous meningitis B. Hydrochlorothiazide (Hypothiazide)
B. Meningococcal meningitis C. Captopril
C. Lymphocytic (Armstrong’s) meningitis D. Losartan
D. Syphilitic meningitis E. Clonidine (Clophelin)
E. Mumps meningitis
39. A 45-year-old patient complains of pain
35. A 37-year-old worker during a fire ended in the epigastric region, left subcostal area,
up in the area of high CO concentration. abdominal distension, diarrhea, loss of wei-
He was delivered to a hospital in unconsci- ght. He has been suffering from this condition
ous state. Objectively: the skin of his face for 5 years. Objectively: the tongue is moist
and hands is crimson. Respiration rate is with white coating near the root; deep palpati-
20/min. ECG: alterations specific for hypoxic on of abdomen reveals slight pain in the epi-
myocardium. Hourly diuresis is 40 ml. Blood gastric region and Мауо-Robson’s point. Li-
test: erythrocytes - 4, 5 · 1012 /l, Нb- 136 ver is painless and protrudes by 1 cm from the
g/l, color index - 0,89, ESR- 3 mm/hour, costal arch. Spleen cannot be palpated. What
carboxyhemoglobin - 5%. What criterion disease can be primarily suspected?
allows determining the severity of the pati- A. Chronic pancreatitis
ent’s condition? B. Atrophic gastritis
A. Carboxyhemoglobin concentration C. Peptic stomach ulcer
B. Respiratory disorders D. Chronic cholecystitis
C. ECG results E. Chronic enteritis
D. Extent of trophic disorders 40. For three years a 31-year-old woman has
E. Development of chronic renal failure been complaining of pain and swelling of
36. After a case of purulent otitis a 1-year-old her radiocarpal and metacarpophalangeal
boy has developed pain in the upper third of articulations, their reduced mobility in the
the left thigh, body temperature up to 39o C . morning, which persisted up to 1,5 hours.
Objectively: swelling of the thigh in its upper Two weeks ago she developed pain, swelli-
third and smoothed out inguinal fold. The li- ng, and reddening of her knee joints, her
mb is in semiflexed position. Active and passi- body temperature increased up to 37,5oC .
ve movements are impossible due to severe The treatment was belated. Examination of
pain. What diagnosis is the most likely? the internal organs revealed no pathologic
alterations. Diagnosis of rheumatoid arthritis
A. Acute hematogenous osteomyelitis was made. What alterations are the most li-
B. Acute coxitis kely to be visible on the arthrogram?
C. Intermuscular phlegmon
D. Osteosarcoma
E. Brodie’s abscess
Крок 2 Medicine (англомовний варiант, iноземнi студенти) 2017 рiк 6

A. Joint space narrowing, usuration resins. Objectively: lesion foci are symmetri-
B. Joint space narrowing, subchondral cally localized on both hands. Against the
osteosclerosis background of erythema with blurred margi-
C. Cysts in the subchondral bone ns there are papulae, vesicles, erosions, crusts,
D. Numerous marginal osteophytes and scales. What is the most likely pathology?
E. Epiphyseal osteolysis
A. Occupational eczema
41. A 58-year-old woman complains of B. Idiopathic eczema
spontaneous bruises, weakness, bleeding C. Allergic dermatitis
gums, dizziness. Objectively: the mucous D. Simple contact dermatitis
membranes and skin are pale with numerous E. Erythema multiforme
hemorrhages of various time of origin. Lymph
nodes are not enlarged. Ps- 100/min, BP- 45. A 10-year-old boy with symptoms of
110/70 mm Hg. There are no alterations of arthritis and myocarditis was delivered into
internal organs. Blood test results: RBC- a hospital. Based on clinical examination the
3, 0 · 1012 /l, Нb- 92 g/l, color index - 0,9, preliminary diagnosis of juvenile rheumatoid
anisocytosis, poikilocytosis, WBC- 10 · 109 /l, arthritis was made. What symptom is the most
eosinophils - 2%, stab neutrophils - 12%, contributive for the diagnostics of this di-
segmented neutrophils - 68%, lymphocytes sease?
- 11%, monocytes - 7%, ESR- 12 mm/h. What A. Reduced mobility of the joints in the
laboratory test should be performed additi- morning
onally to make the diagnosis? B. Regional hyperemia of the joints
A. Platelets C. Affection of the large joints
B. Reticulocytes D. Enlarged heart
C. Clotting time E. Increased heart rate
D. Osmotic resistance of erythrocytes 46. A 25-year-old patient was delivered to
E. Fibrinogen an infectious diseases unit on the 3rd day of
42. After tonsillectomy a woman with illness with complaints of headache, pain in
systemic lupus erythematosus, who has been the lumbar spine and gastrocnemius muscles,
taking prednisolone for a year, developed high fever, chills. Objectively: condition is of
acute weakness, nausea, vomiting, pain in the moderate severity. Scleras are icteric. Pharynx
right iliac area, watery stool up to 5 times is hyperemic. Tongue is dry with dry brown
per day. Ps- 96/min., BP- 80/50 mm Hg. What coating. Abdomen is distended. Liver is
preventive therapy should have been admini- enlarged by 2 cm. Spleen is not enlarged.
stered prior to the surgery? Palpation of muscles, especially gastrocnemi-
us muscles, is painful. Urine is dark in color.
A. Increase of prednisolone dosage Feces are normal in color. What is the most
B. Prescription of desoxycorticosterone likely diagnosis?
acetate
C. Prescription of antibiotics A. Leptospirosis
D. Administration of Ringer’s solution B. Viral hepatitis type A
E. Administration of 10% N aCl solution C. Malaria
D. Infectious mononucleosis
43. A 45-year-old woman complains of E. Yersiniosis
paroxysmal intolerable facial pain on the left
with attacks that last for 1-2 minutes. Attacks 47. A 28-year-old woman complains of
are provoked by chewing. The disease onset skin hemorrhages after minor traumas and
was two month ago after overexposure to spontaneous appearance of hemorrhages on
cold. Objectively: pain at the exit points of the the front of her torso and extremities. On
trigeminal nerve on the left. Touching near examination: the skin is variegated (old and
the wing of nose on the left induces new pain new hemorrhages), bleeding gums. Blood
attack with tonic spasm of the facial muscles. platelets - 20·109 /l; in the bone marrow there is
What is the most likely diagnosis among those increased number of megakaryocytes and no
listed? platelet production. Treatment with steroid
hormones was effective. What disease is it?
A. Trigeminal neuralgia
B. Glossopharyngeal neuralgia A. Idiopathic thrombocytopenic purpura
C. Temporomandibular joint arthritis B. Hemophilia
D. Facial migraine C. Rendu-Osler-Weber disease (Hereditary
E. Maxillary sinusitis hemorrhagic telangiectasia)
D. Disseminated intravascular coagulation
44. A 28-year-old man complains of skin E. Acute vascular purpura
rash and itching on the both of his hands.
The condition persists for 1,5 years. The 48. A 74-year-old patient was delivered into
exacerbation of his condition he ascribes to admission room with clinical presentations of
the occupational contact with formaldehyde acute deep vein thrombosis of the shin. What
Крок 2 Medicine (англомовний варiант, iноземнi студенти) 2017 рiк 7

symptom is the most typical of this pathology? A. Renal amyloidosis


B. Chronic glomerulonephritis
A. Homans’ sign C. Acute glomerulonephritis
B. Rovsing’s sign D. Heart failure
C. Courvoisier’s sign E. Chronic pyelonephritis
D. Mayo-Robson’s sign
E. Grey Turner’s sign 52. A 23-year-old man had taken 1 g of aspirin
to treat acute respiratory infection. After that
49. A 9-month-old child presents with fever, he developed an asthmatic fit with labored
cough, dyspnea. The symptoms appeared 5 expiration that was arrested by introduction
days ago after a contact with a person wi- of aminophylline. The patient has no medical
th URTI. Objectively: the child is in grave history of allergies. The patient has undergone
condition. Temperature is 38o C , cyanosis of two surgeries for nasal polyposis in the past.
nasolabial triangle is present. RR- 54/min, What is the most likely diagnosis?
nasal flaring during breathing is observed.
There was percussion dullness on the right A. Aspirin-induced asthma
below the scapula angle and tympanic sound B. Atopic bronchial asthma
over the other areas of lungs. Auscultati- C. Infectious allergic bronchial asthma
on revealed bilateral fine moist crackles D. Exercise-induced asthma
predominating on the right. What is the most E. Symptomatic bronchospasm
likely diagnosis?
53. A 50-year-old patient was delivered to
A. Pneumonia a hospital with complaints of blood traces
B. URTI in urine. Urination is painless and undi-
C. Acute laryngotracheitis sturbed. Macrohematuria had been observed
D. Acute bronchitis for 3 days. Objectively: kidneys cannot be
E. Acute bronchiolitis palpated, suprapubic area is without alterati-
ons, external genitalia are non-pathologic. On
50. The mother of a 3-month-old child came rectal investigation: prostate is not enlarged,
to a family doctor with complaints of her painless, has normal structure. Cystoscopy
child being physically underdeveloped and revealed no alterations. What is the most li-
suffering from cough attacks and dyspnea. kely diagnosis?
Anamnesis: the child is the result of the
second full-term pregnancy with the risk of A. Renal carcinoma
miscarriage (the first child died of pulmonary B. Bladder tuberculosis
pathology at the age of 4 months, according C. Varicocele
to the mother). Body mass at birth is 2500 D. Dystopic kidney
g. Cough attacks were observed from the fi- E. Necrotic papillitis
rst days of life, twice the child was treated 54. A man complains of constant dull pain
for bronchitis. Considering the severity of the in the perineum and suprapubic area, weak
child’s condition the doctor made the referral flow of urine, frequent difficult painful
for hospitalization. What diagnosis was most urination, nocturia. The patient has been
likely stated in the referral? suffering from this condition for several
A. Mucoviscidosis (Cystic fibrosis) months, during which urination was becomi-
B. Acute obstructive bronchitis ng increasingly difficult, and pain in the peri-
C. Recurrent obstructive bronchitis neum has developed. On rectal examinati-
D. Pertussis on: the prostate is enlarged (mainly its right
E. Acute obstructive pneumonia lobe), dense, asymmetrical, central fissure is
smoothed out, the right lobe is of stony densi-
51. A 46-year-old man notes swollen legs, ty, painless, tuberous. What disease is it?
weakness, sensation of fullness and heavi-
ness in the right subcostal area; it is the A. Prostate cancer
first occurrence of these signs in the pati- B. Prostate sclerosis
ent. The patient has 20-year-long history of C. Urolithiasis, prostatolith of the right lobe
rheumatoid arthritis. The liver and spleen D. Prostate tuberculosis
are enlarged and dense. Blood creatinine - E. Chronic congestion prostatitis
0,23 mmol/l, proteinemia - 68 g/l, cholesterol
- 4,2 mmol/l, urine specific gravity - 1012, 55. A boy was born at 32 weeks of gestation. 2
proteinuria - 3,3 g/l, isolated wax-like cyli- hours after the birth he developed respiratory
nders, leached erythrocytes in the vision field, distress (RD). The RD severity assessed by
leukocytes - 5-6 in the vision field. What is the Silverman score was 5. The respiratory di-
most likely complication? sorders progressed, respiratory failure could
not be eliminated by Martin-Bouyer CPAP
(continuous positive airway pressure). X-ray
of lungs shows reticular and nodular pattern,
air bronchogram. What is the most likely
cause of respiratory distress syndrome?
Крок 2 Medicine (англомовний варiант, iноземнi студенти) 2017 рiк 8

60. An infant is 2,5 months old. The onset of


A. Hyaline membrane disease the disease was gradual, the child had normal
B. Segmental atelectasis body temperature but presented with slight
C. Bronchopulmonary dysplasia cough. Within a week the cough intensified,
D. Congenital pulmonary emphysema especially at night; on the 12th day the child
E. Edematous hemorrhagic syndrome developed cough fits occurring up to 20 times
per day and followed by vomiting. There was
56. An 8-year-old child with a 3-year-long one instance of respiratory arrest. Make the
history of diabetes was hospitalized in diagnosis:
hyperglycemic coma. Specify the initial dose
of insulin to be administered: A. Pertussis
B. Parainfluenza
A. 0,1-0,2 U/kg of body weight per hour C. Congenital stridor
B. 0,05 U/kg of body weight per hour D. Respiratory syncytial infection
C. 0,2-0,3 U/kg of body weight per hour E. Adenovirus infection
D. 0,3-0,4 U/kg of body weight per hour
E. 0,4-0,5 U/kg of body weight per hour 61. Vaginal examination reveals the head of
the fetus, which fills the posterior surface of
57. A 17-year-old young man complains of symphysis pubis and hollow of the sacrum.
general weakness, trismus, twitching of the The lower edge of symphysis pubis, ischi-
muscles in his left shin. 7 days ago he pierced adic spines, and sacrococcygeal joint can be
his foot with a nail. Objectively: at the sole palpated. Where in the lesser pelvis is the fetal
of the foot there is a wound, 0,3х0,2 mm in head situated?
size, with small amount of serous-purulent
discharge, the skin around the wound is A. In the narrow plane of lesser pelvis
hyperemic. What is the most likely diagnosis? B. In the wide plane of lesser pelvis
C. Above pelvic inlet
A. Tetanus D. In the area of brim
B. Phlegmon E. In the area of pelvic outlet
C. Osteomyelitis
D. Infected wound 62. A man works in casting of nonferrous
E. Erysipelas metals and alloys for 12 years. In the air
of working area there was registered high
58. A patient with trauma of the lower thi- content of heavy metals, carbon monoxi-
rd of the forearm volar surface caused by a de, and nitrogen. During periodic health
glass shard came to a first-aid center. Objecti- examination the patient presents with
vely: flexion of the IV and V fingers is asthenovegetative syndrome, sharp pains
impaired, sensitivity of the inner dorsal and in the stomach, constipations, pain in the
palmar surfaces of the hand and IV finger is hepatic area. In urine: aminolevulinic acid
decreased. What nerve is damaged? and coproporphyrin are detected. In blood:
A. Ulnar reticulocytosis, low hemoglobin level. Such
B. Radial intoxication is caused by:
C. Median A. Lead and lead salts
D. Musculocutaneous B. Tin
E. Axillary C. Carbon monoxide
59. A 58-year-old patient complains of D. Nitric oxide
pain in the lower left extremity, which E. Zinc
aggravates during walking, sensation of cold 63. A 30-year-old multigravida has been
and numbness in the both feet. The patient in labour for 18 hours. 2 hours ago the
has been suffering from this condition for 6 pushing stage began. Fetal heart rate is
years. Objectively: the skin is pale and dry, clear, rhythmic, 136/min. Vaginal examinati-
with hyperkeratosis. On the left shin hair is on reveals complete cervical dilatation, the
scarce, ”furrow” symptom of inflamed ingui- fetal head in the pelvic outlet plane. Sagittal
nal lymph nodes is positive. Pulse cannot be suture is in line with obstetric conjugate, the
detected over the pedal and popliteal arteri- occipital fontanel is near the pubis. The pati-
es and is weakened over the femoral artery. ent has been diagnosed with primary uterine
In the right limb popliteal artery pulsation is inertia. What is the further tactics of labour
retained. What is the most likely diagnosis? management?
A. Arteriosclerosis obliterans of the lower A. Outlet forceps
extremities B. Labour stimulation
B. Obliterating endarteritis C. Cesarean section
C. Femoral artery thrombosis D. Skin-head Ivanov’s forceps
D. Raynaud’s disease E. Vacuum extraction of the fetus
E. Buerger’s disease (thromboangiitis obli-
terans) 64. A patient, who had eaten canned
Крок 2 Medicine (англомовний варiант, iноземнi студенти) 2017 рiк 9

mushrooms (honey agaric) three days A. Lactational mastitis


ago, developed vision impairment (di- B. Abscess
plopia, mydriasis), speech disorder, disturbed C. Erysipelas
swallowing. What type of food poisoning D. Dermatitis
occurred in the patient? E. Tumor

A. Botulism 69. A 3-year-old girl is being treated at a


B. Food toxicoinfection resuscitation unit with diagnosis ”acute ki-
C. Fusariotoxicosis dney failure, oligoanuric stage”. ECG: high T
D. Honey agaric poisonong wave, extended QRS complex, displacement
E. Lead salts poisoning of S-T interval downwards below the isoline.
What electrolyte imbalance is it?
65. An infant is 3 weeks old. Since birth
there has been observed periodical vomiti- A. Hyperkalemia
ng within a few minutes after feeding. The B. Hypokalemia
amount of vomitive masses does not exceed C. Hypocalcemia
the volume of previous feeding. The infant D. Hypercalcemia
has age-appropriate body weight. What is the E. Hyperphosphatemia
most likely cause of this symptom? 70. Mother of an 8-year-old girl complains
A. Pylorospasm that the child is too short and has excessi-
B. Esophageal chalasia ve body weight. Objectively: obesity with fat
C. Adrenogenital syndrome deposits on the torso and face (round moon-
D. Pyloristenosis like face), acne, striae on the thighs and lower
E. Esophageal achalasia abdomen, hirsutism. What hormone can cause
such symptoms, when in excess?
66. A newborn with gestational age of 31
weeks presents with hypotonia and depressed A. Cortisol
consciousness. Hematocrit is 35%, general B. Thyroxin
cerebrospinal fluid analysis shows increased C. Testosterone
content of erythrocytes and protein, and low D. Insulin
glucose. These data correspond with the clini- E. Glucagon
cal presentation of:
71. A 9-year-old girl complains of fever up to
A. Intracranial hemorrhage 37,5oC , headache, inertness, weakness, loss
B. Meningitis of appetite, stomachache, and frequent pai-
C. Sepsis nful urination. Provisional diagnosis of acute
D. Anemia pyelonephritis is made. Clinical urine analysis:
E. Prenatal infection specific gravity - 1018, no protein, leukocytes
- 10-15 in the vision field. What investigati-
67. From urine of a 14-year-old boy with on method can verify the diagnosis of urinary
the exacerbation of secondary obstructive system infection?
pyelonephritis Pseudomonas aeruginosa was
isolated with a titer of 1000000 microbes per A. Bacteriological inoculation of urine
1 ml. What antibiotic is the most advisable in B. Rehberg test (creatinine clearance test)
this case? C. Zymnytsky test (measurement of daily
diuresis)
A. Ciprofloxacin D. Complete blood count
B. Ampicillin E. Clinical urine analyses, dynamic testing
C. Cefazolinum
D. Azithromycin 72. During assessment of work conditions
E. Chloramphenicol at the mercury thermometer manufacture,
content of mercury vapors in the air of worki-
68. A 20-year-old woman on the 10th day ng area is revealed to exceed maximum
after her discharge from the maternity ward concentration limit. Specify the main way of
developed fever up to 39o C and pain in her mercury penetration into the body:
left mammary gland. On examination the
mammary gland is enlarged, in its upper outer A. Respiratory organs
quadrant there is a hyperemic area. In this B. Intact skin
area a dense spot with blurred margins can be C. Damaged skin
palpated. The patient presents with lactostasis D. Gastrointestinal tract
and no fluctuation. Lymph nodes in the ri- E. Mucous tunics
ght axillary crease are enlarged and painful.
Specify the correct diagnosis: 73. During health assessment of car drivers
and police officers on point duty, the physici-
ans detected carboxyhemoglobin in the blood
of the patients, weakened reflex responses,
disturbed activity of a number of enzymes.
Крок 2 Medicine (англомовний варiант, iноземнi студенти) 2017 рiк 10

Revealed professional health disorders are A. Sheehan’s syndrome (postpartum hypopi-


most likely to be associated with the effect of: tuitarism)
B. Ovarian amenorrhea
A. Carbon monoxide C. Turner’s syndrome
B. Sulfurous anhydride D. Ovarian exhaustion syndrome
C. Mental stress E. Galactorrhea-amenorrhea syndrome
D. Aromatic hydrocarbons
E. Nitric oxide 78. On laboratory investigation of a pork
sample there is 1 dead trichinella detected
74. On the 3rd day of life a newborn, in 24 sections. This meat should be:
who had suffered birth asphyxia, developed
hemorrhage from the umbilical wound. A. Handed over for technical disposal
Laboratory analysis reveals hypocoagulati- B. Allowed for sale with no restrictions
on, thrombocytopenia, and hypothrombi- C. Processed and sold through public catering
nemia. What is the cause of such clinical network
developments? D. Processed for boiled sausage production
E. Frozen until the temperature of - 10o C is
A. Disseminated intravascular coagulation reached in the deep layers, with subsequent
B. Hemorrhagic disease of newborn exposure to cold for 15 days
C. Congenital angiopathy
D. Thrombocytopenic purpura 79. Examination of a 43-year-old man
E. Umbilical vessel trauma objectively revealed pallor of skin and
mucous membranes, loss of tongue papillae,
75. A 46-year-old woman came to a materni- transverse striation of fingernails, cracks in
ty clinic with complaints of moderate blood the mouth corners, tachycardia. Blood test
discharge from the vagina, which developed results: Hb- 90 g/l, anisocytosis, poikilocytosis.
after the menstruation delay of 1,5 months. The most likely causative factor of this condi-
On vaginal examination: the cervix is clean; tion is the inadequate intake of:
the uterus is not enlarged, mobile, painless;
appendages without changes. Make the di- A. Iron
agnosis: B. Copper
C. Zinc
A. Dysfunctional uterine bleeding D. Magnesium
B. Adenomyosis E. Selene
C. Ectopic pregnancy
D. Submucous uterine myoma 80. A 52-year-old woman presents with
E. Cancer of the uterine body affected mucosa in the mouth angles where
fissures, erosions, and ulcers develop; verti-
76. A patient has the second and third degree cal fissures appear on the lips during their
burns of the 15% of the body surface. On the closing (cheilosis); there are tongue alterati-
20th day after the trauma the patient presents ons (glossitis), angular stomatitis, seborrheic
with sharp increase of body temperature, dermatitis around the mouth and wings of
general weakness, rapid vesicular respiration; the nose, and pericorneal injection. The listed
facial features are sharpened, BP is 90/50 mm symptoms are characteristic of:
Hg, heart rate is 112/min. What complication
is it? A. B2 -hypovitaminosis
B. B1 -hypovitaminosis
A. Sepsis C. C -hypovitaminosis
B. Pneumonia D. P P -hypovitaminosis
C. Acute intoxication E. A-hypovitaminosis
D. Purulent bronchitis
E. Anaerobic infection 81. Two days ago a woman fell from the hei-
ght of 1,5 m. She complains of severe thoracic
77. A 30-year-old woman complains of pain on the left and dyspnea. Chest X-ray
amenorrhea that lasts for 2 years after she reveals hydropneumothorax on the left with
has given birth, loss of hair and body weight. fluid level at the 7th rib and the lung collapsed
The labor was complicated with hemorrhage by 1/3. The 6th-7th ribs are fractured along
caused by uterine hypotonia. Objectively the the scapular line. Serohemorrhagic fluid was
patient is of asthenic type, her external geni- obtained during thoracic puncture. What
talia are hypoplastic, the uterine body is small treatment tactics should be chosen?
in size and painless. No uterine appendages
can be detected. What is the most likely di-
agnosis?
Крок 2 Medicine (англомовний варiант, iноземнi студенти) 2017 рiк 11

A. Thoracocentesis on the left at the 7th shoulders, the head is bowed forward, the
intercostal space thorax is flattened, the stomach is bulging.
B. Thoracocentesis at the 2nd intercostal space In the vertebral column there are deepened
C. Repeated puncture of the pleural cavity cervical and lumbar flexures. What posture
D. Intubation of trachea with artificial does the child have?
pulmonary ventilation
E. Alcohol-novocaine block of the fracture A. Kyphosis
area B. Lordosis
C. Stooping
82. A patient complains of suppuration from D. Corrected
the ear and impaired hearing of the left ear, E. Normal
which have been observed for the past 6
years. The patient had periodical headaches, 86. A 46-year-old woman has been hospitali-
general indisposition, fever. Objectively: zed with open fracture of the left thigh in
otoscopy of the external auditory meatus its middle third. She underwent the surgery
revealed mucopurulent odorless substance. - fixation with extraosseous osteosynthesis
The eardrum is of normal color, with central plates. On the 4th day after the surgery
perforation. What is the most likely di- she developed pain in the wound, body
agnosis? temperature rose over 39o C . What measures
should be taken in this case?
A. Chronic mesotympanitis
B. Otosclerosis A. Undo the sutures, drain the wound, and
C. Acute otitis media prescribe antibiotics
D. Chronic epitympanitis B. Prescribe broad spectrum antibiotics and
E. Chronic sensorineural hearing loss hormonal agents
C. Administer antibiotics intraosseously and
83. In 2 hours after a traffic accident a 28-year- hypothermia locally
old man in grave condition was delivered to a D. Inject antibiotics into the area surroundi-
hospital. The patient complains of abdominal ng the wound, prescribe spasmolytics and
pain. He received a blow to the abdomen wi- analgesics
th the steering wheel. Objective examination E. Remove the fixation, prescribe sulfanilami-
revealed the following: the abdomen does not des
participate in respiration, is tense and acutely
painful on palpation; the abdominal muscles 87. A multigravida on the 38th week of her
are defensively tense, peritoneal irritation si- pregnancy complains of increased BP up to
gns are positive, hepatic dullness is absent. BP 140/90 mm Hg, edema of the shins for 2 weeks.
is 90/60 mm Hg, heart rate is 120/min. What In the last month she gained 3,5 kg of weight.
further treatment tactics should be chosen? Urine analysis: protein - 0,033 g/l. Make the
diagnosis:
A. Laparotomy
B. Laparoscopy A. Mild preeclampsia
C. Cold to the abdomen B. Moderate preeclampsia
D. Ultrasound investigation C. Pregnancy hypertension
E. Laparocentesis D. Severe preeclampsia
E. Pregnancy edema
84. A 28-year-old woman complains of
increased intervals between menstruations, 88. A 44-year-old patient with postinfarcti-
up to 2 months, and hirsutism. Gynecological on cardiosclerosis presents with frequent
examination revealed the following: ovaries heart rate disorders and lower extremity
are enlarged, painless, and dense; no alterati- edema. Objectively: Ps- 95/min., irregular,
ons of the uterus. US of the lesser pelvis: 10-12 extrasystoles per minute. BP- 135/90
ovaries are 4-5 cm in diameter, with numerous mm Hg. The 1st heart sound at the apex is
enlarged follicles on the periphery. X-ray of weakened. Pulmonary respiration is rough.
the skull base: sellar region is widened. What The liver is enlarged +2 cm. ECG: irregular
is the most likely diagnosis? sinus rhythm, heart rate - 95/min, frequent
polytopic ventricular extrasystoles. What anti-
A. Stein-Leventhal syndrome (polycystic arrhythmic drug is advisable in this case for
ovarian syndrome) treatment and prevention of extrasystole?
B. Algodismenorrhea
C. Sheehan syndrome (postpartum hypopitui- A. Amiodarone
tarism) B. Lidocaine
D. Premenstrual syndrome C. Mexiletine
E. Morgagni-Stewart-Morel syndrome D. Quinidine
(metabolic craniopathy) E. Novocainamide (Procainamide)

85. Posture of an 11-year-old boy was determi- 89. A 60 year-old woman has been suffering
ned during preventive examination. The chi- from weakness, dizziness, and fatigue over
ld presents with curled forward rounded the last year. Recently she has also developed
Крок 2 Medicine (англомовний варiант, iноземнi студенти) 2017 рiк 12

dyspnea, paresthesia. Objectively: the skin A. Hepatocellular carcinoma of the liver


and mucous membranes are pale and slightly B. Hepatocellular insufficiency
icteric. The tongue is smooth due to the loss of C. Hypersplenism
lingual papillae. Liver and spleen are located D. Cholestasis
at the costal margin. Blood count: Hb- 70 g/l, E. Congestive heart failure
RBC- 1, 7·1012 /l, color index - 1,2, macrocytes. 93. A 43-year-old man has undergone a
Administer the patient a pathogenetically surgery for osteomyelitis of the left thigh.
justified drug: On the 6th day the patient’s condition was
A. Vitamin B12 complicated with sepsis. Despite complex
B. Vitamin B6 therapy of sepsis on the 9th day the patient
C. Ascorbic acid persistently presents with temperature up to
D. Iron preparations 40o C , heart rate is 110/min., respiration rate
E. Vitamin B1 is 23/min., BP is 100/60 mm Hg. Blood test:
leukocytes - 16 · 109 /l, band neutrophils - 16%.
90. After excessive consumption of fatty food What phase of clinical course is it?
a 60-year-old woman suddenly developed
pain in her right subcostal area, nausea, bi- A. Catabolic
le vomiting, sharp bitter taste in her mouth. B. Anabolic
In 2 days she developed jaundice, her uri- C. Rehabilitation
ne darkened. Objectively: sclera and skin are D. Functional
icteric, the abdomen is distended, the liver is E. Tension
enlarged by 3 cm, soft and painful on palpati-
on, Ortner’s, Murphy’s, Kehr’s, Zakharyin’s, 94. A 23-year-old woman presents with di-
Mayo-Robson’s signs are positive. What di- abetes mellitus type 1. She complains of
agnostic technique should be used in the first weakness, headache, nausea, and vomiting.
place? Objectively: temperature is 37,6o C , heart rate
is 98/min., BP is 95/65 mm Hg, respiration rate
A. US of the gallbladder and bile duct is 32/min., loud. Smell of acetone is detected,
B. Fibrogastroduodenoscopy heart sounds are muffled, pulse is rhythmic.
C. X-ray of the abdomen The stomach is sensitive in the epigastrium.
D. Radionuclide scanning of the liver and Costovertebral angle tenderness (Murphy’s
gallbladder punch sign) is present. Blood glucose is 28,5
E. Laparoscopy mmol/l; blood leukocytes - 16, 5 · 109 /l. In uri-
ne: acetone ++, leukocytes - 25-40 in the vision
91. A 42-year-old woman working at a poultry field. Blood pH is 7,1. What correction method
farm complains of dyspnea, thoracic pain on would be the most advisable?
the left, increased body temperature up to 38-
39o C in the evening, and cough. The patient A. Infusion of normal saline + insulinotherapy
suffers from essential hypertension. Objecti- in small portions
vely: vesicular respiration in the lungs, vesi- B. Infusion of 5% glucose + insulinotherapy in
cular resonance without alterations. X-ray large portions
of both lungs reveals numerous small low- C. Infusion of 2,5% sodium bicarbonate +
intensity foci, 2-3 mm in size, located in a row insulinotherapy
along the blood vessels. ESR- 32 mm/hour. D. Intravenous administration of antibiotics +
What is the most likely diagnosis? insulinotherapy
E. Infusion of dextran solutions + insuli-
A. Acute disseminated tuberculosis notherapy
B. Infiltrative tuberculosis
C. Pulmonary carcinomatosis 95. A 57-year-old woman complains of a
D. Focal tuberculosis sensation of esophageal compression, palpi-
E. Community-acquired pneumonia tations, difficult breathing when eating solid
food, occasional vomiting with a full mouth,
92. A 65-year-old patient has been suffering ”wet pillow” sign at night for the last 6
from liver cirrhosis associated with hepati- months. Objectively: body tempearture -
tis C virus for 7 years. During the last 3 39o C , height - 168 cm, weight - 72 kg, Ps-
weeks the patient developed severe edema 76/min, BP- 120/80 mm Hg. X-ray study
of the lower extremities, the abdomen is si- revealed considerable dilation of esophagus
gnificantly distended with fluid. Ultrasound: and its constriction in the cardial part. What
signs of liver cirrhosis, portal hypertensi- pathology is the most likely to have caused
on, lymph nodes are in the area of hepatic dysphagia in this patient?
portal. Concentration of α-fetoprotein in
blood serum is 285 ng/ml. What complicati- A. Achalasia cardiae
on could have developed in this case? B. Primary esophageal spasm
C. Hiatal hernia
D. Esophageal carcinoma
E. Reflux esophagitis
Крок 2 Medicine (англомовний варiант, iноземнi студенти) 2017 рiк 13

96. A 47-year-old man is employed at the neck veins, peripheral pulse is absent, the
weaving workshop, has 15-year-long record carotid artery pulse is rhythmic, 130/min., BP
of service at this factory; his work conditi- is 60/20 mm Hg. Auscultation of the heart
ons are associated with high-frequency and reveals extremely muffled sounds, percussion
high-intensity noise. During periodical exami- reveals heart border extension in both directi-
nation he was diagnosed with occupational ons. What is the optimal treatment tactics for
deafness. What are the grounds for making this patient?
such a diagnosis?
A. Pericardiocentesis and immediate
A. Audiometry data and hygienic assessment thoracotomy
of working environment B. Oxygen inhalation
B. Record of service at this factory C. Puncture of the pleural cavity on the left
C. Noise characteristic at this factory D. Conservative treatment, infusion of
D. Central nervous system examination results adrenomimetics
E. Inner ear examination results E. Pleural cavity drainage
97. A 37-year-old woman complains of acute 101. A patient with chronic pancreatitis
pain in the genital area, swelling of the complains of diarrhea occurring up to 5 ti-
labia, pain when walking. Objectively: body mes per day (no blood traces), loss of body
temperature is 38,7o C , Ps- 98/min. In the weight, abdominal distention, dryness of skin,
interior of the right labia there is a dense, loss of hair, thirst, bleeding gums, convulsi-
painful tumor-like formation 5,0x4,5 cm in si- ons. Complete blood count: leukocytes -
ze, the skin and mucous membrane of genitals 5, 8 · 109 /l; Hb- 86 g/l; ESR- 15 mm/g; Blood
are hyperemic, there is profuse foul-smelling protein test: protein - 48 g/l; albumins - 28
discharge. What is the most likely diagnosis? g/l. What indicators of coprological analysis
would accompany this syndrom?
A. Acute bartholinitis
B. Labial furuncle A. Steatorrhea, creatorrhea
C. Acute vulvovaginitis B. Large amount of mucus, amylorrhea
D. Bartholin gland cyst C. Large amount of starch grains and cellulose
E. Carcinoma of vulva D. Gas bubbles, acid reaction
E. Large numbers of iodinophilous microbes
98. A woman undergoing in-patient treatment
for viral hepatitis type B developed headache, 102. A 35-year-old patient complains of
nausea, recurrent vomiting, memory lapses, heartburn, gasseous and sour eructation,
flapping tremor of her hands, rapid pulse. burning constricting pain behind the sternum
Sweet smell from the mouth is detected. Body and along the esophagus, developing when
temperature is 37,6o C , heart rate is 89/min. bowing the torso to the front. No previ-
What complication developed in the patient? ous examination; the patient takes almagel
at his own discretion, after which he notes
A. Acute liver failure improvement of his general state. Make the
B. Ischemic stroke provisional diagnosis:
C. Gastrointestinal hemorrhage
D. Hypoglycemic shock A. Gastroesophageal reflux disease
E. Meningoencephalitis B. Functional dyspepsia
C. Cardiospasm
99. A 42-year-old man was delivered to a D. Ulcer disease of the stomach
surgical in-patient department with complai- E. Ulcer disease of the duodenum
nts of icteric skin, pain in the right subcostal
area. Biochemical blood analysis: total bili- 103. The left hand of a newborn is extended
rubin - 140 mcmol/l, direct bilirubin - 112 in all its joints, stretched along the torso, and
mcmol/l. On US: choledoch duct - 1,4 cm, pronated in the forearm. Active movements
a concrement is detected in the distal area. of the shoulder joint are retained. The hand
Gallbladder is 40 cm, no concrements. What is flattened, atrophied, cold to touch, hangs
treatment tactics should be chosen? passively. Grasping and Babkin’s reflexes are
absent at the affected side. Hemogram indi-
A. Endoscopic papillosphincterotomy cators are normal. Make the most likely di-
B. Laparoscopic cholecystectomy agnosis:
C. Laparotomy with choledoch duct drain
D. Laparotomy with cholecystectomy A. Inferior distal obstetrical paralysis
E. Threatment in an infectious diseases hospi- B. Osteomyelitis
tal C. Proximal obstetrical paralysis
D. Complete obstetrical paralysis
100. 4 weeks after myocardial infarction a 56- E. Hypoxic-ischemic encephalopathy
year-old patient developed acute heart pain,
marked dyspnea. Objectively: the patient’s 104. A 34-year-old man is being treated for
condition is extremely grave, there is marked schizophrenia exacerbation in a psychiatric
cyanosis of face, swelling and throbbing of unit. Objectively: the patient remains in bed,
Крок 2 Medicine (англомовний варiант, iноземнi студенти) 2017 рiк 14

is sluggishly mobile, unresponsive, does not Objectively: the patient is exhausted, there
react to questions. His position is unvari- are enlarged supraclavicular lymph nodes.
ed, hypomimic, snout reflex and Dupre’s Esophagoscopy revealed no esophageal
symptom are present, muscles exhibit waxy pathology. Which of the following investigati-
flexibility. He has been remaining in this ons is the most appropriate in this case?
state for approximately a week. Feeding is
parenteral. Determine the neuromotor di- A. Computed tomography of chest and medi-
sturbance: astinum
B. X-ray of lungs
A. Catatonic stupor C. Multiplanar imaging of esophagus
B. Depressive stupor D. Radioisotope investigation of chest
C. Psychogenic stupor E. Ultrasound investigation of mediastinum
D. Anergic stupor
E. Exogenic stupor 109. A 15-year-old girl complains of dizzi-
ness and sensation of lack of air that she
105. A 14-year-old girl has been delivered to develops in emotionally straining situations.
a gynecological department with complaints Relief occurs after she takes corvalol. Objecti-
of profuse blood discharge from her genital vely: hyperhidrosis and marble-like pattern
tract for 2 weeks. Anamnesis: menstruation si- of the skin of her palms and feet. Clinical
nce 13, irregular, painful, profuse; the last one and instrumental examination revealed no
was 2 months ago. Objectively: pale skin and organic alterations of the central nervous,
mucosa, BP- 100/60 mm Hg, Hb- 108 g/l. The cardiovascular, and respiratory systems. What
abdomen is soft and painless on palpation. provisional diagnosis can be made?
Rectal examination revealed no pathologies
of reproductive organs. What condition is it? A. Somatoform autonomic dysfunction
B. Obstructive bronchitis
A. Juvenile uterine hemorrhage (Dysfuncti- C. Bronchial asthma
onal) D. Stenosing laryngotracheitis
B. Hypomenstrual syndrome E. Acute epiglottitis
C. Inflammation of uterine appendages (Pelvic
110. A 14-year-old patient complains of
inflammatory disease) alopecia foci on his scalp. The patient has
D. Pelviperitonitis been presenting with this condition for 2
E. Endometritis weeks. Objectively: on the scalp there are
106. An infant cries during urination, the several small oval foci with blurred margins.
foreskin swells and urine is excreted in The skin in the foci is pink-red, the hairs are
drops. What approach to treatment should broken off at 4-5 mm length or at skin level.
be chosen? Under Wood’s lamp there are no foci of green
luminescence detected. What disease is it?
A. Create an opening into the foreskin cavity
B. Prescription of α-adrenergic blocking A. Trichophytosis capitis
agents B. Syphilitic alopecia
C. Prescription of antispasmodic agents C. Alopecia areata
D. Urinary bladder catheterization D. Scleroderma
E. Epicystostomy E. Psoriasis

107. A 20-year-old patient complains of pain 111. A 38-year-old patient has been delivered
in the left lumbar region, arterial pressure rise by an ambulance to a surgical department
up to 160/110 mm Hg. US revealed that the with complaints of general weakness, indi-
structure and size of the right kidney were wi- sposition, black stool. On examination the
thin age norms, there were signs of the 3rd patient is pale, there are dotted hemorrhages
degree hydronephrotic transformation of the on the skin of his torso and extremiti-
left kidney. Doppler examination revealed es. On digital investigation there are black
an additional artery running to the lower feces on the glove. Blood test: Hb- 108 g/l,
pole of the kidney. Excretory urogram shows thrombocytopenia. Anamnesis states that si-
a narrowing in the region of ureteropelvic milar condition was observed 1 year ago.
junction. Specify the treatment tactics: Make the diagnosis:

A. Surgical intervention A. Thrombocytopenic purpura


B. Administration of spasmolytics B. Hemophilia
C. Administration of ACE inhibitors C. Ulcerative bleeding
D. Kidney catheterization D. Rectal tumor
E. Administration of β -blockers E. Nonspecific ulcerative colitis

108. A 49-year-old patient consulted a doctor 112. A 35-year-old patient developed an epi-
about difficult swallowing, voice hoarseness, leptic attack with tonoclonic spasms that
weight loss. These symptoms have been lasted for 3 minutes. After the attack the pati-
gradually progressing for the last 3 months. ent fell asleep but in 5 minutes the second
Крок 2 Medicine (англомовний варiант, iноземнi студенти) 2017 рiк 15

attack occurred. The first step of emergency 22. His current condition has changed acutely:
aid would be to: for 3 days the patient has been refusing
to leave his home. He claims that there is
A. Ensure patency of airways a ”telepathy” occurring between him and
B. Take blood from the vein for analysis the other people, through which he receives
C. Introduce diazepam intravenously ”thoughts of strangers” and transmits his own
D. Prescribe antiepileptic drugs thoughts for everyone to hear. He is convi-
E. Administer chloral hydrate via an enema nced that his thoughts and actions are being
manipulated through this ”telepathy”. Make
113. A 23-year-old woman has been suffering the preliminary diagnosis:
from a mental disease since the age of 18,
the course of disease has no remission peri- A. Paranoid schizophrenia
ods. At a hospital the patient mostly presents B. Depressive episode
with non-purposeful foolish excitation: she C. Manic episode
makes stereotypic grimaces, exposes herself, D. Organic delirium
publicly masturbates with a loud laughter, E. Acute reaction to stress
repeates stereotypical abusive shouts. The
patient should be prescribed: 118. A 35-year-old woman has gained 20 kg
weight within a year with the normal diet.
A. Neuroleptics She complains of chills, sleepiness, shortness
B. Antidepressants of breath. The patient’s mother and sister are
C. Tranquilizers corpulent. Objectively: height - 160 cm, wei-
D. Nootropics ght - 92 kg, BMI- 35,9. Obesity is uniform,
E. Mood stabilizers there are no striae. The face is amimic. The
skin is dry. The tongue is thickened. Heart
114. A young woman suffering from sounds are muffled. HR- 56/min, BP- 140/100
seborrhea oleosa has numerous light-brown mm Hg. The patient has been suffering from
and white spots on the skin of her torso amenorrhea for 5 months, has constipations.
and shoulders. The spots have clear margi- TSH- 28 mcIU/l (norm is 0,32-5). Craniogram
ns, branny desquamation, no itching. What shows no pathology. What is the etiology of
provisional diagnosis can be made? obesity?
A. Pityriasis versicolor A. Hypothyroid
B. Torso dermatophytosis B. Hypo-ovarian
C. Seborrheic dermatitis C. Hypothalamic-pituitary
D. Pityriasis rosea D. Alimentary and constitutive
E. Vitiligo E. Hypercorticoid
115. 10 days after birth a newborn developed 119. A patient complains of painless ”sores”
sudden fever up to 38,1o C . Objectively: the on his penis and inguinal lymph nodes
skin in the region of navel, abdomen and enlargement. Synthomycin emulsion that the
chest is erythematous; there are multiple pea- patient have been applying to the ”sores” was
sized blisters with no infiltration at the base; ineffective. Objectively: on the inner leaf of
isolated bright red moist erosions with epi- the foreskin there are three closely situated
dermal fragments are observed on the peri- rounded erosions, 0,5 cm in diameter, with
phery. What is the provisional diagnosis? dense infiltration that can be palpated at their
A. Epidemic pemphigus of newborn bases. Make the preliminary diagnosis:
B. Syphilitic pemphigus A. Primary syphilis
C. Streptococcal impetigo
D. Vulgar impetigo B. Herpes simplex (Herpes pro genitalis)
E. Atopic dermatitis C. Candidiasis of the inner leaf of the foreskin
D. Shingles
116. A man came to an urologist with E. Erythema multiforme
complains of painful urination, discharge
from urethra. The patient has been suffering 120. A 1,5-month-old child on breasfeedi-
from this condition for a week. Objectively: ng presents from birth with daily vomiting,
hyperemic urinary meatus, edema, purulent irregular liquid foamy feces, and meteori-
discharge. Microscopy of smears detected sm, which are resistant to antibacterial and
gram-negative bacteria. Specify the diagnosis: probiotic therapy; no increase of body mass
is observed. The child’s condition improved,
A. Acute gonorrheal urethritis when breastmilk was substituted with ”NAN
B. Trichomonas urethritis low lactose” formula. What pathology is it?
C. Candidal urethritis
D. Chlamydial urethritis
E. Chronic gonorrhea
117. A patient is 28 years old. He has been
suffering from mental disorder since he was
Крок 2 Medicine (англомовний варiант, iноземнi студенти) 2017 рiк 16

A. Lactase deficiency crackles in the lower segments of the lungs.


B. Intestinal lambliasis (Giardiasis) Heart sounds are weakened, the II heart
C. Infectious enteritis sound is accentuated over the pulmonary
D. Drug-induced enteritis artery. The liver is +3 cm. What complicated
E. Functional dyspepsia the clinical course of COPD in this patient?
121. A 12-year-old girl after a case of respi- A. Chronic pulmonary heart
ratory infection developed dyspnea at rest, B. Pulmonary embolism
paleness of skin. Heart rate is 110/min., BP C. Acute left ventricular failure
is 90/55 mm Hg. Heart sounds are muffled. D. Diffuse pneumosclerosis
Borders of relative heart dullness: right - the E. Community-acquired pneumonia
parasternal line, upper - the III rib, left -
1,0 cm outwards from the midclavicular line. 125. A multigravida at 39 weeks of gestation
Make the provisional diagnosis: presenting with regular labour activity for 8
hours has been delivered to a hospital; the
A. Infectious myocarditis waters broke an hour ago. She complains of
B. Functional cardiopathy headache, seeing spots. BP is 180/100 mm Hg.
C. Somatoform autonomic dysfunction Urine test results: protein - 3,3 g/l, hyaline
D. Hypertrophic cardiomyopathy cylinders. Fetal heart rate is 140/min, rhythmi-
E. Exudative pericarditis cal. Vaginal examination reveals complete
cervical dilatation, the fetal head is on the
122. A 48-year-old man complains of fati- pelvic floor, sagittal suture is in line with
gue, excessive sweating, severe skin itchi- obstetric conjugate, the occipital fontanel is
ng, undulant fever, enlarged cervical and under the pubis. What is the optimal tactics of
supraclavicular lymph nodes. Objectively: labour management?
paleness of skin and mucosa, cervical lymph
nodes are mobile, dense, elastic, walnut-sized, A. Outlet forceps
painless, not attached to the skin. Complete B. Cavity forceps
blood count: erythrocytes - 3, 0 · 1012 /l, Hb- C. Cesarean section
100 g/l, leukocytes - 14 · 109 /l, eosinophils - D. Vacuum extraction of the fetus
6%, basophils - 3%, band neutrophils - 11%, E. Conservative labour management
segmented neutrophils - 69%, lymphocytes -
126. After the contact with chemicals a plant
7, monocytes - 4%, platelets - 280 · 109 /l, ESR- worker has suddenly developed stridor, voi-
37 mm/hour. What method should be applied ce hoarseness, barking cough, progressi-
to verify the diagnosis? ng dyspnea. Objective examination reveals
A. Lymph node biopsy acrocyanosis. What is the provisional di-
B. Sternal puncture agnosis?
C. Muscle biopsy A. Laryngeal edema
D. Chest X-ray B. Laryngeal carcinoma
E. Lumbar puncture C. PATE
123. A patient after a blow to the head D. Pulmonary atelectasis
developed general symptoms of cerebral di- E. Pneumothorax
sturbance, nausea, vomiting, focal signs - 127. An 18-year-old woman complains of pain
hemi-hyperreflexia S>D, hemihyperesthesia in her lower abdomen, profuse purulent di-
on the left, marked meningeal syndrome. Nei- scharge from the vagina, temperature rise up
ther cranial X-ray nor computer tomography to 37,8o C . Anamnesis states that she had a
revealed any pathologies. What examination random sexual contact the day before the si-
method would allow making and clarification gns appeared. She was diagnosed with acute
of the diagnosis? bilateral adnexitis. On additional examinati-
A. Lumbar puncture on: leukocytes are present throughout all visi-
B. Echoencephalography on field, bacteria, diplococci with intracellular
C. Electroencephalography and extracellular position. What is the most
D. Angiography likely agent in the given case?
E. Pneumoencephalography A. Neisseria gonorrhoeae
124. A 72-year-old man complains of lower B. Escherichia coli
extremity edema, sensation of heaviness in C. Chlamydia trachomatis
the right subcostal area, dyspnea of rest. D. Trichomona vaginalis
For over 25 years he has been sufferi- E. Staphylococcus aureus
ng from COPD. Objectively: orthopnea, 128. A 58-year-old man complains of general
jugular venous distention, diffuse cyanosis, weakness, loss of 10 kg of weight within 1,5
acrocyanosis. Barrel chest is observed, months, progressive pain in the lumbar regi-
on percussion there is vesiculotympanitic on, increased blood pressure up to 220/160
(bandbox) resonance, sharply weakened vesi- mm Hg, low grade fever. Objectively: in the
cular respiration on both sides, moist crepitant
Крок 2 Medicine (англомовний варiант, iноземнi студенти) 2017 рiк 17

right hypochondrium deep palpation reveals a A. Cause of death


formation with uneven surface and low mobi- B. Manner of death
lity; veins of the spermatic cord and scrotum C. Time of death
are dilated. Blood test results: Hb- 86 g/l, D. Mode of death
ESR- 44 mm/h. Urine test results: specific E. Mechanism of death
gravity - 1020, protein - 0,99 g/l, RBC cover
the whole field of vision, WBC- 4-6 in the field 132. It is the 3rd day after the normal term
of vision. What is the provisional diagnosis? labor; the infant is rooming-in with the
mother and is on breastfeeding. Objectively:
A. Renal tumour the mother’s general condition is satisfactory.
B. Urolithiasis Temperature is 36,4o C , heart rate is 80/min.,
C. Acute pyelonephritis BP is 120/80 mm Hg. Mammary glands
D. Acute glomerulonephritis are soft and painless; lactation is moderate,
E. Nephroptosis unrestricted milk flow. The uterus is dense,
the uterine fundus is located by 3 fingers width
129. Anamnesis of a 30-year-old patient below the navel. Lochia are sanguino-serous,
includes closed thoracic injury. Lately the moderate in volume. Assess the dynamics of
patient has been suffering from increasing uterine involution:
dyspnea, sensation of heaviness in the right
subcostal area, and heart rate disturbances. A. Physiological involution
Objectively: acrocyanosis, bulging cervical B. Subinvolution
veins, ascites, edema of the lower extremiti- C. Lochiometra
es. Heart auscultation reveals muffled heart D. Pathologic involution
sounds, additional III heart sound is detected. E. Hematometra
Provisional diagnosis of constrictive peri-
carditis was made. What diagnostic technique 133. Survey radiograph of a 52-year-old
would NOT confirm the diagnosis? worker of an agglomeration plant (28-year-
long record of service, the concentration
A. US of abdomen of metal dust is 22-37 mg/m3 ) shows mi-
B. Computer tomography ldly pronounced interstitial fibrosis with di-
C. Echocardiography ffused contrasting well-defined small nodular
D. Magnetic resonance imaging shadows. The patient has no complaints.
E. Chest X-ray Pulmonary function is not compromised.
What is the provisional diagnosis?
130. A 72-year-old woman suffers from di-
abetes mellitus type 2, concomitant diseases A. Siderosis
are stage 2 hypertension and stage 2B heart B. Silicosis
failure. She takes metformin. Hypertensic cri- C. Anthraco-silicatosis
sis had occurred the day before, after whi- D. Silicatosis
ch the patient developed extreme weakness, E. Anthracosis
myalgias, thirst, dry mouth, polyuria. BP
is 140/95 mm Hg, heart rate is 98/min., 134. A 53-year-old woman complains of wei-
no edemas or smell of acetone detected. ght loss up to 10 kg within the last 2 years,
What measures should be taken to prevent liquid foul-smelling stool two times a day that
development of comatose state in the patient? poorly washes off the toilet, periodic bouts
of nausea, girdle pain in the upper abdomen.
A. Stop metformin, prescribe short-acting Objectively: pain in Gubergrits zone (on the
insulin right from navel) and at Mayo-Robson’s poi-
B. Double the dosage of metformin nt. Biochemical blood analysis: glucose - 3,2
C. Apply hypotonic solution of sodium chlori- mmol/l, bilirubin - 16,5 mcmol/l, crude protein
de - 56,4 g/l. Urine diastase/amylase - 426 g/h/l.
D. Additionally prescribe long-acting insulin D-xylose test (oral administration of 25 g of
E. Prescribe glibenclamide d-xylose) after 5 hours reveals 3 g of xylose in
131. The body of a 24-year-old woman with urine. The most likely diagnosis is:
probable signs of poisoning has been found on A. Pancreatitis. Malabsorption syndrome
the street. Forensic medical examination was B. Pseudomembranous colitis
requested by an investigator during exami- C. Nonspecific ulcerative colitis
nation of the site and the body. According D. Irritable bowel syndrome
to the Criminal Procedure Code currently in E. Chronic gastritis
force in Ukraine, forensic medical examinati-
on is required when it is necessary to determi- 135. A 15-year-old teenager has undergone
ne the: medical examination in military recruitment
center. The following was revealed: interval
systolic murmur at the cardiac apex, accent
of the II heart sound over the pulmonary
artery, tachycardia. What additional exami-
nation method will be the most informative
Крок 2 Medicine (англомовний варiант, iноземнi студенти) 2017 рiк 18

for determining diagnosis? A. Reactive arthritis


B. Rheumatoid arthritis
A. Echocardiography C. Seasonal pollinosis
B. Electrocardiography D. Bacterial nonspecific urethral conjunctivitis
C. X-ray E. Upper respiratory tract infection (URTI)
D. Phonocardiography that affects conjunctiva and joints
E. Rheography
140. A 19-year-old woman complains of
136. A 64-year-old patient has been hospitali- severe pain in the axillary crease. Conditi-
zed with complaints of progressive jaundice on onset occurred a week ago after her swi-
that developed over 3 weeks without pain mming in a cold river and epilation. The next
syndrome and is accompanied by general day a painful ”boil” appeared. The ”boil” was
weakness and loss of appetite. Objectively: increasing in size every day and became a
temperature is 36,8o C , heart rate is 78/min, plum-sized tumor. Upon examination there
abdomen is soft and painless, peritoneum irri- are nodular conical growths joined together
tation symptoms are not detected, palpati- detected, the skin covering them is bluish-red
on reveals sharply enlarged tense gallbladder. in color. Some nodules have fistulous openi-
What disease can be characterised by these ngs producing thick purulent mass. Body
symptoms? temperature is 38, 5o C , general malaise. What
is the most likely diagnosis?
A. Cancer of pancreas head
B. Duodenal ulcer A. Hydradenitis
C. Acute cholecystitis B. Carbuncle
D. Chronic cholecystitis C. Cutaneous tuberculosis
E. Cholecystitis caused by lambliasis D. Necrotizing ulcerative trichophytosis
E. Pyoderma chancriformis
137. A 6-year-old girl came to a general practi-
tioner with her mother. The child complains 141. A woman complains of weight gain,
of burning pain and itching in her external chills, edema, xeroderma, somnolence, diffi-
genitalia. The girl was taking antibiotics the culties with focusing. Objectively: height is
day before due to her suffering from acute 165 cm; weight is 90 kg; body proportions
bronchitis. On examination: external genitalia are of female type, to - 35,8o C , heart rate -
are swollen, hyperemic, there is white deposit 58/min, BP- 105/60 mm Hg. Heart sounds
accumulated in the folds. The most likely di- are weakened, bradycardia is observed. Other
agnosis is: internal organs have no alterations. Thyroid
gland cannot be palpated. Milk secretion
A. Candidal vulvovaginitis from mammary glands is observed. Hormone
B. Trichomoniasis test revealed increased levels of thyroid-
C. Nonspecific vulvitis stimulating hormone (TSH) and prolactin,
D. Helminthic invasion and decreased level of thyroxine (4 ). What
E. Herpes vulvitis is the cause of obesity?
138. A 37-year-old patient complains of pain A. Primary hypothyroidism
in the spinal column, reduced mobility. The B. Secondary hypothyroidism
condition persists for 7 years. ”Sway back” is C. Prolactinoma
observed, there is no movement in all spi- D. Hypopituitarism
nal regions. On X-ray: ”bamboo spine” is E. Adiposogenital dystrophy
detected. What is the most likely diagnosis?
142. ECG revealed the following in a 10-
A. Ankylosing spondylitis year-old child: sharp acceleration of the heart
B. Osteochondrosis rate - 240/min., P wave overlaps with T wave
C. Spondylitis deformans and deforms it, moderate lengthening of PQ
D. Tuberculous spondylitis interval, QRS complex is without alterations.
E. Spondylolisthesis What pathology does this child have?
139. A 25-year-old patient is not married and A. Paroxysmal atrial tachycardia
has sexual relations with several partners. B. Atrial hypertrophy
During the last 3 months he noticed a small C. Ventricular hypertrophy
amount of mucoserous discharge from the D. WPW syndrome
urethra. Subjectively: periodical itching or E. Extrasystole
burning pain in the urethra. Two months ago
pain in the knee joint developed. Possibility 143. A 54-year-old patient complains of
of trauma or exposure to cold is denied by the weakness, jaundice, itching skin. Disease
patient. During the last week eye discomfort onset was 1,5 months ago: fever up to 39o C
is noted - lacrimation and itching. What provi- appeared at first, with progressive jaundice
sional diagnosis can be made? developed 2 weeks later. On hospitalizati-
on jaundice was severely progressed. Liver
cannot be palpated. Gallbladder is enlarged
Крок 2 Medicine (англомовний варiант, iноземнi студенти) 2017 рiк 19

and painless. Blood bilirubin is 190 mcmol/l analyses periodically revealed leukocyturia.
(accounting for direct bilirubin). Stool is The child has undergone no further treatment.
acholic. What is the most likely jaundice On examination: increased BP up to 150/100
genesis in this patient? mm Hg. Ultrasound investigation revealed si-
gnificant reduction of the right kidney. What
A. Mechanical jaundice process is leading in arterial hypertension
B. Hepatocellular jaundice pathogenesis in this case?
C. Hemolytic jaundice
D. Caroli syndrome A. Hyperactivity of renin-angiotensin system
E. Gilbert’s syndrome B. Disturbance of water-electrolytic balance
C. Disturbance of renal circulation
144. During hemotransfusion the patient D. Hypersympathicotonia
developed nausea, tremor, lumbar and E. Increased cortisol level
retrosternal pain. On examination the skin
is hyperemic, later developed pallor; the pati- 148. A 3-month-old child presents with
ent presents with hyperhidrosis, labored respi- saffron-yellow coloring of the skin, sclera,
ration, pulse is 110/min., BP is 70/40 mm Hg. and mucous membranes. The abdomen is
Urine is black colored. What complication enlarged, hepatomegaly and splenomegaly
developed in the patient? are observed. In blood there is conjugated
bilirubin-induced hyperbilirubinemia. On
A. Posttransfusion shock intravenous cholangiocholecystography:
B. Acute renal failure opacified bile is discharged into the intesti-
C. Pulmonary embolism ne. Transaminase activity is normal. What is
D. Anaphylactic shock the most likely diagnosis?
E. Hypotonic crisis
A. Biliary atresia
145. A 22-year-old man suddenly developed B. Physiologic jaundice
extreme weakness, nausea, vomiting with C. Hemolytic disease of newborn
traces of blood. The patient is known to D. Crigler-Najjar syndrome
suffer from peptiv ulcer disease of duodenum E. Congenital hepatitis
and hemophilia A. Objectively: heart rate -
102/min., BP- 100/60 mm Hg. Complete blood 149. A 51-year-old woman complains of
count: erythrocytes - 3, 2 · 1012 /l, Hb- 98 g/l, headache, trembling, paresthesiae, palpi-
color index - 0,92, leukocytes - 7, 4 · 109 /l, tations, increased blood pressure up to
280/160 mm Hg. The day before she experi-
platelets - 240 · 109 /l, ESR- 11 mm/hour. What enced exhausting headache, vascular pulsati-
measure would most effectively decrease on, palpitations, asphyxia, stomachache,
hemorrhaging in this case? unbearable fear of coming death. The pati-
A. Cryoprecipitate ent paled and broke out in cold sweat. In
B. Aminocapronic acid urine there is increased content of vani-
C. Native plasma llylmandelic acid. What disease causes such
D. Direct transfusion of donor blood clinical presentation in the patient?
E. Platelet concentrate transfusion A. Pheochromocytoma
146. A 22-year-old woman complains of B. Conn’s syndrome (primary
amenorrhea for 8 months. Anamnesis states hyperaldosteronism)
that menarche occured at the age of 12,5. Si- C. Cushing’s syndrome
nce the age of 18 the patient has a history of D. Primary hypertension
irregular menstruation. The patient is nulli- E. Cushing’s disease
gravida. The mammary glands are developed
properly, nipples discharge drops of milk 150. A 52-year-old patient suffers from
when pressed. Hormone test: prolactin level marked dyspnea during physical exertion,
is 2 times higher than normal. CT reveals a non-productive cough. The patient’s condi-
bulky formation with diameter of 4 mm in tion has been persisting for 8 months. The
the region of sella. What is the most likely di- patient has been a smoker for 30 years. In
agnosis? the lungs there are cellophane-type crackles
auscultated on both sides. Respiration rate is
A. Pituitary tumour 26/min., oxygen saturation of blood is 92%.
B. Lactational amenorrhea On spirometry: moderate restrictive-type di-
C. Stein-Leventhal syndrome (polycystic ovary sturbance of external respiration. What is the
syndrome) most likely diagnosis?
D. Sheehan’s syndrome (postpartum hypopi-
tuitarism)
E. Cushing’s disease
147. A 13-year-old girl complains of fatigabili-
ty, frequent headaches, cardialgia. Eight years
ago she had a case of pyelonephritis. Urine
Крок 2 Medicine (англомовний варiант, iноземнi студенти) 2017 рiк 20

A. Idiopathic fibrosing alveolitis A. Lymphogranulomatosis (Hodgkin’s


B. Chronic obstructive pulmonary disease lymphoma)
(COPD) B. Lymph node tuberculosis
C. Chronic bronchitis C. Lymphoreticulosarcoma
D. Community-acquired pneumonia D. Cancer metastases to the lymph nodes
E. Sarcoidosis E. Macofollicular reticulosis
151. A 26-year-old patient with affective bi- 155. An 9-year-old child was hospitalized
polar disorder has developed a condition for fever up to 39,8o C , inertness, moderate
manifested by mood improvement, behavi- headache, vomiting. Examination revealed
oural and sexual hyperactivity, verbosity, acti- meningeal symptoms. Lumbar puncture was
ve body language, reduced need for sleep. performed. The obtained fluid was characteri-
Which of the following drugs would be most sed by increased opening pressure, was
effective in this case? transparent, with the cell count of 450 cells per
1 mcL (mainly lymphocytes - 90%), glucose
A. Neuroleptics with sedative effect level of 3,6 mmol/l. What agent could have
B. Antidepressants with activating effect caused the disease in the child?
C. Neuroleptics with activating effect
D. Tranquilizers A. Enterovirus
E. Antidepressants with sedative effect B. Neisseria meningitidis
C. Mycobacterium tuberculosis
152. A 19-year-old patient complains of D. Staphylococcus aureus
dyspnea on exertion. He often has bronchitis E. Streptococcus pneumoniae
and pneumonia. Since childhood the patient
presents with cardiac murmur. Auscultation 156. A 25-year-old woman has a self-detected
revealed splitting of the II sound above the tumor in the upper outer quadrant of her ri-
pulmonary artery, systolic murmur in the 3rd ght breast. On palpation there is a painless
intercostal space at the left sternal border. firm mobile lump up to 2 cm in diameter, peri-
ECG detected right bundle branch block. pheral lymph nodes are without alterations. In
What is the provisional diagnosis? the upper outer quadrant of the right breast
ultrasound revealed a massive neoplasm with
A. Atrial septal defect increased echogenicity sized 21x18 mm. What
B. Open ductus arteriosus is the most likely diagnosis?
C. Aortarctia
D. Aortic stenosis A. Fibroadenoma
E. Mitral insufficiency B. Lactocele
C. Diffuse mastopathy
153. A 30-year-old patient was in a car acci- D. Mammary cancer
dent. He is unconscious, pale, has thready E. Mastitis
pulse. In the middle third of the right thigh
there is an extensive laceration with ongoi- 157. A 68-year-old man complains of inabili-
ng profuse external arterial bleeding. What ty to urinate for a day. On attempt of urinary
urgent actions must be taken to save the life bladder catheterization there was detected a
of the patient? rough stricture in the membranous portion of
the urethra. What first aid tactics should be
A. Tourniquet above the wound of the right applied in this case?
thigh
B. Tourniquet below the wound of the right A. Troacar cystostomy
thigh B. Adenomectomy
C. Artificial lung ventilation C. Optical internal urethrotomy
D. Precordial thump D. α-adrenoblockers
E. Application of plaster bar E. Urinary bladder catheterization
154. A 25-year-old patient has been admi- 158. Heart X-ray of a 31-year-old man has
tted to the hospital with the following revealed the following: with tightly filled
problems: weakness, sweating, itching, wei- opacified esophagus there is a marginal filli-
ght loss, enlarged submandibular, cervical, ng defect in its middle third on the posterior
axillary, inguinal lymph nodes. Objectively: wall; the defect is 1,8x1,3 cm in size with clear
hepatomegaly. Lymph node biopsy revealed oval border. Mucosal folds are retained and
giant Berezovsky-Reed-Sternberg cells, envelop the defect; wall peristalsis and elasti-
polymorphocellular granuloma composed of city are not affected. There are no complai-
lymphocytes, reticular cells, neutrophils, eosi- nts regarding the condition of the patient’s
nophils, fibrous tissue, and plasma cells. What alimentary canal. Make the provisional di-
is the most likely diagnosis? agnosis:
Крок 2 Medicine (англомовний варiант, iноземнi студенти) 2017 рiк 21

A. Esophageal tumor general fatigue, low-grade fever persisting


B. Achalasia cardiae for 4 months, lumbar pain, and dysuria.
C. Esophageal burns Anamnesis includes frequent acute respi-
D. Diverticulum ratory diseases, overexposure to cold, low-
E. Barrett esophagus calorie diet, a case of pulmonary tuberculosis
in childhood. Clinical urine analysis: pH-
159. A patient with signs of general 4,8, leukocyturia, hematuria. Complete blood
overexposure to cold presenting with local count: leukocytosis, lymphocytosis, increased
frostbites of fingers has been delivered into ESR. Urography concludes: dilatation of
an admission room. Objectively: conscious, renal pelvis and calyceal system of both ki-
inert, speech is slow, the skin of the face is dneys, foci of calcification in the projection of
cold, body temperature is 34o C , heart rate is right kidney parenchyma. What is the most
68/min. What would be the actions of a doctor likely diagnosis?
on call?
A. Nephrotuberculosis
A. Hospitalize the patient to the surgical B. Right renal cyst
department C. Right renal carcinoma
B. Hospitalize the patient to the therapeutics D. Acute glomerulonephritis
department E. Chronic pyelonephritis
C. Hospitalize the patient to the traumatology
department 164. A full-term newborn (born with the body
D. Let the patient go home weight of 3900 g at gestational age of 39
E. Refer to a family doctor on the next day weeks) on the first day of his life developed
160. A 22-day-old infant developed respiratory disturbances: dyspnea, arrhythmic
subcutaneous red nodes from 1,0 to 1,5 cm in respiration, cyanosis attacks. On examination
size on the scalp; later the nodes suppurated. there is paradoxical respiration observed and
Temperature increased up to 37,7o C , intoxi- left side of the chest lags behind in the act of
cation symptoms appeared, regional lymph breathing. On auscultation the respiration is
nodes enlarged. Complete blood count: weakened in the lungs on the left. Neurologi-
anemia, leukocytosis, neutrocytosis, increased st diagnosed the patient with left-sided Erb-
ESR. What diagnosis will you make? Duchenne palsy. Complete blood count shows
no changes. What is the most likely diagnosis?
A. Pseudofurunculosis
B. Pemphigus A. Left-sided diaphragm paresis
C. Vesiculopustulosis B. Congenital pneumonia
D. Scalp phlegmon C. Left-sided pneumothorax
E. - D. Respiratory distress syndrome
E. Transient tachypnea of the newborn
161. During examination of a healthy infant,
the child takes a toy into his hands, turns 165. A 12-year-old child had three attacks of
from the back to the side; when lying on the acute rheumatic fever accompanied by cardi-
stomach he can firmly prop himself up on his tis. Examination revealed the symptoms of
forearms; the child laughs and makes joyful chronic tonsillitis, mitral insufficiency, and
exclamations. The age of the child is: carious teeth. What is the optimal method
of secondary prophylaxis?
A. 4 months
B. 2 months A. Year-round bicillin prophylaxis until the age
C. 5 months of 25
D. 1 month B. Course of cardiotrophic drugs twice a year
E. 6 months C. Year-round bicillin prophylaxis for 3 years
D. Tonsillectomy
162. After significant physical exertion a 66- E. Oral cavity sanitation
year-old man with deep vein thrombosis of
the extremities developed shortness of breath, 166. A 24-year-old patient had been delivered
intense pain in the chest on the left, marked to a thoracic department with a chest injury,
palpitations. The patient’s condition is grave, fracture of the IV, V, VI ribs on the right.
his face is cyanotic, the cervical veins are Plan radiography showed the fluid level in
swollen, BP is 60/40 mm Hg. What investi- the pleural cavity to be reaching the III rib
gation method would be the most advisable in on the right. Puncture contained blood clots.
this case? What is the optimal treatment tactics?

A. Selective angiopneumography A. Emergency thoracotomy


B. Chest X-ray B. Pleural puncture
C. Echocardiography C. Thoracentesis and thoracostomy
D. Magnetic resonance imaging of the chest D. Hemostatic therapy
E. Fiber-optic bronchoscopy E. Medical thoracoscopy

163. A 32-year-old woman complains of 167. A 3-year-old child has been delivered to
Крок 2 Medicine (англомовний варiант, iноземнi студенти) 2017 рiк 22

a hospital with complaints of pain in the legs, 171. Examination of a Rh-negative pregnant
fever, loss of appetite. Objectively: pale skin woman at 32 weeks of gestation revealed
and mucosa, hemorrhagic rash. Lymph nodes a four-time rise of Rh-antibody titer within
are enlarged, painless, dense and elastic, not 2 last weeks; the titer is 1:64. The first two
matted together. Bones, joints, and abdomen pregnancies resulted in antenatal fetal death
are painful. The liver and spleen are enlarged. due to hemolytic disease. What is the optimal
Hemogram: Hb- 88 g/l, color index - 1,3, tactics of pregnancy management?
platelets - 80 · 109 /l, leukocytes - 25, 8 · 109 /l,
lymphoblasts - 70%, ESR- 52 mm/hour. Make A. Early delivery
the provisional diagnosis: B. Delivery at 37 weeks of gestation
C. Screening for Rh-antibodies in 2 weeks and
A. Acute leukemia urgent delivery in case of further increase of
B. Thrombocytopenic purpura antibody titer
C. Acute rheumatic fever D. Introduction of anti-Rh (D)
D. Infectious mononucleosis immunoglobulin
E. Hemorrhagic vasculitis (Henoch-Schonlein E. Ultrasound for signs of hemolytic disease of
purpura) the fetus
168. A 60-year-old man has a diet consisting of 172. During last several weeks an 11-year-
unvaried food staples: mostly cereals, potato, old girl has been complaining of dyspnea and
pasta; few vegetables and little fats (especially edema of shins and feet after physical exerci-
animal fats). During medical examination he se. After a long rest or sleep through the night
complains of deterioration of his twilight visi- her edemas diminish significantly. On clini-
on. This condition can be caused by lack of: cal examination there are enlarged liver and
rasping systolic murmur over the cardiac area.
A. Retinol Blood and urine analyses are without changes.
B. Amino acids What is the most likely cause of the child’s
C. Fats edema?
D. Calcium
E. Carbohydrates A. Heart failure
B. Angioneurotic edema
169. A 14-year-old girl came to a general C. Acute pyelonephritis
practitioner with complaints of weakness, D. Hepatocirrhosis
loss of appetite, headache, rapid fatigabili- E. Nephrotic syndrome
ty. Her last menstruation was profuse and
lasted for 14 days after previous delay of 2 173. A 74-year-old man visited an urologist
months. Objectively: the skin is pale, heart with complaints of pain above the pubis and
rate is 90/min., BP is 110/70 mm Hg, Hb is inability to urinate for 8 hours. At home he
88 g/l. Rectal examination: the uterus and its had taken antispasmodics and had a warm
appendages are without changes, no discharge bath but no improvement occurred. Objecti-
from the genital tracts. What complication vely: the abdomen is soft and painful above
occurred in the patient? the pubis; dullness of percussion sound is
observed above the pubis. Murphy’s punch si-
A. Posthemorrhagic anemia gn (costovertebral angle tenderness) is negati-
B. Somatoform autonomic dysfunction of ve on both sides. What condition does the
hypotonic type patient have?
C. Migraine
D. Gastritis A. Acute urinary retention
E. Dysmenorrhea B. Paradoxal ischuria
C. Chronic urinary retention
170. A postpartum woman on the 12th day D. Anuria
after the normal delivery complains of pain E. Oliguria
localized in her left gastrocnemius muscle.
Body temperature is 37,2o C ; pulse is 85/min, 174. During preventive examination a 58-year-
rhythmic; blood pressure is 128/80 mm Hg. old man on chest X-ray presents with multi-
Mammary glands are soft and painless. The ple globular pale shadows 3 cm in diameter
uterus is behind the pubis. The left leg in the within parenchyma of the both lungs. Exami-
area of gastrocnemius muscle is by 3 cm larger nation in the oncologic hospital: the primary
than the right leg in the diameter. Internal focus is not found; transbronchial biopsy wi-
organs present no pathologies. What compli- th cytologic investigation detected cells of
cation can be suspected? glandular neoplasm. What tactics should the
physician choose?
A. Deep vein thrombosis of the shin
B. Iliofemoral thrombosis
C. Varicose veins of lower extremities
D. Endometritis
E. Myositis
Крок 2 Medicine (англомовний варiант, iноземнi студенти) 2017 рiк 23

A. Polychemotherapy courses on on the right, which developed late in


B. Exploratory laparotomy the night, and impaired walking. Objecti-
C. Exploratory thoracotomy vely: the metatarsophalangeal articulation
D. Laparoscopy is swollen, hyperemic, hot to touch, pai-
E. Symptomatic treatment at home nful on movement. In blood: erythrocytes -
175. A 36-year-old man developed a disease 5, 1·1012 /l, Нb- 155 g/l, leukocytes- 13, 0·109 /l,
with acute onset 6 hours ago. The patient ESR- 50 mm/hour, CRP- 46 mg/dl, uric
presents with pain in the epigastric, ileocecal, acid - 720 mcmol/l. X-ray of feet articulati-
and paraumbilical areas, vomiting, weakness, ons: osteoporosis, narrowing of interarti-
nausea, and body temperature of 38,5o C . cular spaces, numerous punched-out erosions.
Stool is liquid, profuse, frequent, retains fecal Make the preliminary diagnosis:
nature, foul-smelling, frothy, colored dark A. Gout
green. The stomach is moderately distended B. Osteoarthritis
and painful on palpation. The patient attri- C. Reactive arthritis
butes his disease to eating raw chicken eggs D. Rheumatoid arthritis
one day before the clinical signs of the disease E. Psoriatic arthritis
appeared. What is the most likely diagnosis?
179. A woman in her early- to mid-thirties
A. Salmonellosis has lost her consciousness 3-5 minutes ago.
B. Shigellosis On examination: the skin is pale, no pulse
C. Typhoid fever over the carotid arteries, no spontaneous
D. Cholera respiration, pupils are dilated; the patient is
E. Food toxicoinfection nonresponsive, presents with atony. The pati-
176. A 55-year-old woman came to a ent’s condition can be determined as:
gynecologist with complaints of leukorrhea A. Apparent death
and bloody discharge from the vagina after B. Natural death
5 years of menopause. Anamnesis states C. Syncope
no pregnancies. Bimanual examination: the D. Brain death
uterus and uterine appendages are without E. Coma
changes. During diagnostic curettage of the
uterine cavity the physician scraped off 180. A 32-year-old woman complains of body
enchephaloid matter. What is the most likely weight loss despite her increased appetite,
diagnosis in this case? nervousness, and tremor of the extremities.
Objectively: the skin is moist; the thyroid
A. Endometrial carcinoma gland is diffusely enlarged, painless, soft, and
B. Adenomyosis mobile. Blood test: increased level of T3,
C. Subserous uterine myoma T4, and thyroid-stimulating hormone (THS).
D. Cervical carcinoma What is the most likely diagnosis?
E. Ovarian carcinoma
A. Diffuse toxic goiter
177. A 10-year-old boy is delivered into a B. Thyroid carcinoma
polytrauma unit after he received a blunt C. Autoimmune (Hashimoto’s) thyroiditis
trauma of the thorax, having fallen from D. Thyroid adenoma
the bicycle. Upon hospitalization his blood E. Diffuse nontoxic goiter
pressure is 110/80 mm Hg, heart rate is 96/min.
Chest X-ray is noncontributive to the di- 181. A 57-year-old patient complains of
agnosis. Echocardiogram shows free liquid sensation of dryness and pain during
in the pericardial cavity, in the amount of up swallowing, frequent unbearable cough, the
to 100 ml. In an hour after the hospitalization voice is hoarse. Disease onset was abrupt. On
the patient started to develop increasing si- laryngoscopy: laryngeal mucosa is hyperemic,
gns of heart failure: jugular venous distention, vocal folds are swollen, laryngeal lumen
decreased blood pressure down to 90/70 mm contains viscous secretion. What diagnosis is
Hg, tachycardia up to 120/min. On auscultati- it?
on muffled heart sounds. What would be the
primary tactics of a physician? A. Acute laryngitis
B. Acute stenosing laryngotracheitis
A. Pericardiocentesis C. Bronchial asthma
B. Cardiac glycosides intravenously D. Flegmonous laryngitis
C. Constant oxygenotherapy E. Laryngeal diphtheria
D. Diuretics intravenously
E. Antibiotics intravenously 182. A 24-year-old pregnant woman on her
37th week of pregnancy has been delivered to
178. After the celebratory feast that took a maternity obstetric service with complaints
place the day before, a 35-year-old man was of weak fetal movements. Fetal heartbeats are
hospitalized with complaints of marked pain 95/min. On vaginal examination the uterine
within the I metatarsophalangeal articulati- cervix is tilted backwards, 2 cm long, external
Крок 2 Medicine (англомовний варiант, iноземнi студенти) 2017 рiк 24

orifice allows inserting a fingertip. Biophysi- A. Total protein content in the pleural fluid
cal profile of the fetus equals 4 points. What below 25 g/l
tactics of pregnancy management should be B. Presence of atypical cells
chosen? C. Total protein content in the pleural fluid
above 30 g/l
A. Urgent delivery via a cesarean section D. Specific gravity exceeding 1015
B. Treatment of placental dysfunction and E. Positive Rivalta’s test
repeated analysis of the fetal biophysical
profile on the next day 186. A 28-year-old woman complains of gi-
C. Doppler measurement of blood velocity in rdle pain in her epigastric and left subcostal
the umbilical artery areas with irradiation to the back, nausea,
D. Urgent preparation of the uterine cervix for and vomiting without relief. On examinati-
delivery on a surgeon observes stomach distension
E. Treatment of fetal distress, if ineffective, and meteorism. There are positive Mondor’s,
then elective cesarean section on the next day Mayo-Robson’s, and Cullen’s symptoms.
What is the most likely diagnosis?
183. During regular preventive gynecologi-
cal examination a 30-year-old woman was A. Acute pancreatitis
detected to have dark blue punctulated B. Acute cholecystitis
”perforations” on the vaginal portion of C. Acute intestinal obstruction
the uterine cervix. The doctor suspects D. Aortic dissecting aneurysm
endometriosis of the vaginal portion of the E. Splenic infarction
uterine cervix. What investigation method
would be most informative for diagnosis 187. A 36-year-old woman complains of
confirmation? pain in her lumbar area, which irradiates to
her lower right limb and increases during
A. Colposcopy, target biopsy of the cervix movements, and sensation of numbness in
B. US of small pelvis her limb. Objectively: palpation of the shin
C. Hysteroscopy and thigh muscles is painful, positive stretch
D. Curettage of the uterine cavity symptom on the right. MRI scan: herniati-
E. Hormone testing on of intervertebral disk L5-S1 4 mm in size.
What is the most likely diagnosis?
184. A 26-year-old woman came to a
gynecologist for a regular check-up. She has A. Vertebrogenic lumbago
no complaints. Per vaginum: the uterus lies in B. Vertebrogenic radicular syndrome of L5-S1
anteflexion, not enlarged, dense, mobile, pai- on the right
nless. On the left from the uterus in the area of C. Endarteritis of lower extremities
uterine appendages there is a mobile painless D. Spinal stroke
outgrowth that can be moved independently E. Acute myelitis
from the uterus. On the right the appendages
cannot be detected. What additional investi- 188. During routine medical examinati-
gation would be informative for diagnosis on a 35-year-old woman presents with
clarification? enlarged cervical and mediastinal lymph
nodes. Her overall health is satisfactory.
A. US of lesser pelvis ESR is 30 mm/hour. Cervical node biopsy
B. Metrosalpingography was performed. In the specimen there are
C. Examination for urogenital infection granulomas composed of epithelial and gi-
D. Colposcopy ant cells, no caseous necrosis detected. What
E. Colonoscopy is the most likely diagnosis?

185. A 57-year-old patient complains of A. Sarcoidosis


dyspnea at rest. The patient presents with B. Lymphogranulomatosis
orthopnea, acrocyanosis, bulging cervical vei- C. Infectious mononucleosis
ns. On percussion: dull sound over the lower D. Nonspecific lymphadenitis
lung segments; on auscultation: no respiratory E. Lymph node tuberculosis
murmurs. Heart rate is 92/min. Right-sided
cardiac dilatation is observed. The liver is 189. A 39-year-old woman complains of vi-
enlarged by 7 cm. Shins are swollen. Pleural olent pain in her left lumbar area with irradi-
effusion is suspected. What indicator would ation to the right iliac area. Several years
confirm the presence of transudate in this ago she was diagnosed with cholelithiasis
case? and urolithiasis. The patient’s condition is
moderately severe, the skin is dry. Ortner’s
symptom is negative; costovertebral angle
tenderness is observed on the right. The most
reasonable treatment tactics would be:
Крок 2 Medicine (англомовний варiант, iноземнi студенти) 2017 рiк 25

A. Spasmolytics and analgesics 194. A 32-year-old woman complains of epi-


B. Urgent hemodialysis sodes of intense fear that occur without visible
C. Peritoneal dialysis cause and last for 10-20 minutes; the episodes
D. Laparoscopic cholecystectomy are characterized by rapid pulse, sweating,
E. Antibiotics labored breathing, and vertigo. Specify the li-
kely diagnosis:
190. An 18-year-old patient complains of skin
rash. The patient has been suffering from this A. Panic disorder
condition for 5 years. The first instance of this B. Paranoid syndrome
disease occurred after a car accident. Objecti- C. Manic syndrome
vely: the patient presents with papular rash D. Simple schizophrenia
covered in silvery scales, ”thimble” symptom E. Claustrophobia
(small pits on the nails), affected joints. What
is the most likely diagnosis? 195. A resuscitation unit received a 46-year-
old woman, who has been suffering from
A. Psoriasis diabetes mellitus type 1 for approximately
B. Panaritium 30 years. Objectively: the skin is pale, heart
C. Onychomycosis sounds are weakened, BP is 170/100 mm
D. Lupus erythematosus Hg, lower limbs are markedly swollen. Blood
E. Rheumatism creatinine - 1125 mcmol/l, urea - 49,6 mmol/l,
potassium - 6.3 mmol/l, glucose - 7,6 mmol/l,
191. A woman complains of frequent, li- glomerular filtration rate - 5 ml/min. What
quid stool (up to 9-10 times per day) with treatment is indicated for the patient in the
mucus and blood admixtures, dull pain in the first place?
hypogastrium, weight loss of 4 kg within the
last year. Objectively: malnutrition, dry skin, A. Hemodialysis
low turgor, aphthous stomatitis. The stomach B. Kidney transplantation
is soft, the sigmoid colon is spastic and painful C. Hemofiltration
on palpation. Occult blood test is positive. Fi- D. Enterosorption
brocolonoscopy: edema, hyperemia, mucosal E. Conservative detoxification therapy
granulation, pseudopolyps, small ulcers with
irregular edges. Make the diagnosis: 196. A woman is on the 32nd week of her
second pregnancy. She complains of fever,
A. Nonspecific ulcerative colitis chills, nausea, vomiting, lumbar pain, and
B. Chronic enterocolitis dysuria. Costovertebral angle tenderness is
C. Colon cancer present on both sides. Urine analysis: pyuria,
D. Irritable bowel syndrome bacteriuria. Blood test: leukocytosis. What is
E. Crohn’s disease (regional enteritis) the most likely diagnosis?
192. A 48-year-old woman has been hospitali- A. Gestational pyelonephritis
zed due to development of tachysystolic atrial B. Cystitis
fibrillation. She has lost 5 kg of body weight C. Pyelitis
within 2 months. On palpation there is a node D. Glomerulonephritis
in the left lobe of the thyroid gland. What E. Latent bacteriuria
pathology resulted in the development of this
condition? 197. A 19-year-old young man complains of
cough with expectoration of purulent sputum
A. Toxic nodular goiter in amount of 100 ml per day, hemoptysis,
B. Aterosclerotic cardiosclerosis dyspnea, increased body temperature up
C. Chronic thyroiditis to 37,8oC , general weakness, weight loss.
D. Nontoxic nodular goiter The patient’s condition lasts for 4 years.
E. Autoimmune thyroiditis Exacerbations occur 2-3 times a year.
The patient presents with malnutrition,
193. On the 9th day after childbirth the pale skin, cyanosis of the lips, drumstick
obstetric patient developed high fever up (clubbed) fingers. Tympanic percussion sound
to 38o C . She complains of pain in the right in the lungs, weakened respiration, vari-
mammary gland. On examination a sharply ous numerous moist crackles in the lower
painful infiltrate can be palpated in the right pulmonary segments on the left can be
mammary gland, the skin over the infiltrate observed. In blood: erythrocytes - 3, 2 · 1012 /l,
is red, subareolar area and nipple are swollen
and painful. What is your diagnosis? leukocytes - 8, 4 · 109 /l, ESR- 56 mm/hour.
On X-ray: lung fields are emphysematous, the
A. Abscess of the right mammary gland left pulmonary root is deformed and dilated.
B. Mastopathy What is the most likely diagnosis?
C. Cancer of the right mammary gland
D. Serous mastitis
E. Fibrous cystic degeneration of the right
mammary gland
Крок 2 Medicine (англомовний варiант, iноземнi студенти) 2017 рiк 26

A. Multiple bronchiectasis of the left lung her son with violence. She had opened her
B. Chronic left-sided pneumonia window (on the 8th floor) and tried to jump
C. Chronic abscess of the left lung out, resisted the people, who were holding her
D. Left-sided pulmonary cystic dysplasia back. What drugs should be administered to
E. Suppuration of the cyst in the left lung terminate such condition of the patient?
198. A 37-year-old man working as a A. Antipsychotics
typesetter in a print shop complains of B. Antidepressants
rapid fatigability, paroxysmal attacks of C. Tranquilizers
stomachache, weak drooping hands. Exami- D. Nootropic agents
nation of neurological status revealed E. Antihypertensive drugs
hypotrophy of the forearm muscles.
Carporadial reflexes are sharply weakened. 200. A 43-year-old man, who has been abusi-
Sensitivity is not disturbed. Gums present ng alcohol and suffering from pulmonary
with dark blue border. What neurological tuberculosis, in the course of two weeks
pathology is it? gradually developed general weakness,
headache, diplopia, vomiting. Objectively:
A. Lead polyneuropathy ptosis on the left, anisocoria S>D, exotropia
B. Guillain-Barre syndrome (postinfectious of the left eye, neck stiffness; Kernig’s
polyneuritis) and Brudzinski’s signs are positive. In
C. Shingles cerebrospinal fluid: lymphocytic pleocytosis,
D. Ulnar neuropathy low glucose, precipitation of cerebrospinal
E. Brachial plexitis fluid resulted in production of fibrin film.
What is the most likely diagnosis?
199. A 54-year-old woman takes anti-
hypertensive drugs for hypertension. Havi- A. Tuberculous meningitis
ng discovered that her son was arrested and B. Subarachnoid hemorrhage
is under investigation, became agitated and C. Brainstem encephalitis
extremely anxious. She lost her orientation D. Acute myelitis
in place, stopped recognizing her relatives, E. Basal arachnoiditis
started hearing ”voices” threatening her and
INSTRUCTIONAL BOOK

Testing Board

TEST ITEMS FOR LICENSING EXAMINATION: KROK 2. MEDICINE.

Kyiv. Testing Board.


(English language).

Approved to print 24.04/№217. Paper size 60х84 1/8


Offset paper. Typeface. Times New Roman Cyr. Offset print.
Conditional print pages 24. Accounting publishing pages 28.
Issue. 2610 copies
List of abbreviations
List of abbreviations
ACTH Adrenocorticotrophic hormone
ADP Adenosine diphosphate
ALT, ALAT Alanin aminotranspherase
ALV Artificial lung ventilation
AP Arterial (blood) pressure
AST, AspAT Aspartat aminotranspherase
ATP Adenosine triphosphate
ВСG Bacillus Calmette-Guérin
BP Blood (arterial) pressure
CK Creatine kinase
CNS Central nervous system
СоА Coenzyme A
CRP C-reactive protein
CT Computed tomography
COX Cyclooxygenase
DIC Disseminated intravascular coagulation
DTP (DPT) Diphtheria-tetanus-pertussis vaccine
EDTA Ethylenediamine tetra-acetic acid
ELISA Enzyme-linked immunosorbent assay
ENT Ear, nose, and throat (as a department in a hospital)
EPR (ER) Endoplasmic reticulum
ESR Erythrocyte sedimentation rate
EV Enterovirus
FC Functional class
GABA Gamma aminobutyric acid
Hct, Ht Hematocrit
HDL High-density lipoproteins
HR Heart rate
IDL Intermediate-density lipoproteins
IU International unit
IUPAC International Union of Pure and Applied Chemistry
LDH Lactate dehydrogenase
LDL Low-density lipoproteins
LOX Lipoxygenase
MAO Monoamine oxidase
MRI Magnetic resonance imaging
NSAID Nonsteroidal anti-inflammatory drug
PE (PATE) Pulmonary embolism (Pulmonary artery thromboembolism)
PSA Prostate-specific antigen
RBC Red blood count
RR Respiratory rate
SES Sanitary-and-epidemiologic station
STD Sexually transmitted disease
STI Sexually transmitted infection
T/l Trillion/liter
TABT Typhoid-paratyphoid A and B plus tetanus toxoid vaccine
TMJ Temporomandibular joint
TSH Thyroid-stimulating hormone
TU Tuberculin unit
URTI Upper respiratory tract infection
V/f Vision field
VLDL Very-low-density lipoproteins
WBC White blood count
WPW Wolff-Parkinson-White syndrome
MINISTRY OF PUBLIC HEALTH OF UKRAINE

Department of human resources policy, education and science

Testing Board

Student ID Last name

Variant ___________________

Test items for licensing examination

Krok 2
MEDICINE
General Instruction
Every one of these numbered questions or unfinished statements in
this chapter corresponds to answers or statements endings. Choose the
answer (finished statements) that fits best and fill in the circle with the
corresponding Latin letter on the answer sheet.

Authors of items: Afanasievska M.S., Baburina O.A., Bahirian I.O., Bedeniuk A.D., Beriozov V.M.,
Besh L.V., Beziuk M.M., Bezruk T.O., Bielska O.A., Bilyk V.D., Bobrytska V.V., Borysova T.P.,
Budaieva I.V., Bukhtieieva E.R., Bukhtiyarova O.H., Buriak O.H., Butina L.I., Chaika O.O.,
Chebanenko R.O., Chemodanov E.B., Chichirelio-Konstantynovych K.D., Chuiko A.P., Dankyna I.A.,
Davydenko O.M., Desiatska Yu.V., Detsyk O.Z., Dobrovolska L.M., Dotsenko T.M., Dovhaliuk Yu.P.,
Duplenko P.Yu., Furman V.M., Halicheva N.O., Harahulia I.S., Herasymenko O.I., Hlumcher F.S.,
Hnatko O.P., Honcharova I.M., Hordieieva H.D., Hresko M.M., Hyrla Ya.V., Ivakhno O.P., Ivanchuk P.R.,
Kadochnykov V.S., Kalinovska I.V., Kalsada I.N., Kalutsky I.V., Kandyba V.P., Kanikovsky O.Ye.,
Kanovska L.V., Karliychuk O.O., Khomaziuk T.A., Kitura O.Ye., Koliush O.I., Koloskova O.K.,
Koltsova N.I., Kompaniyets K.M., Kondratenko P.H., Kondratiev V.O., Konopliova L.F., Konovalova N.V.,
Kopchak O.V., Korniyets N.H., Korobchansky V.O., Korobko O.A., Kovalchuk P.Ye., Kovtun A.I.,
Kriachkova L.V., Krylova V.Yu., Kucherenko O.S., Kudelia I.V., Kudintseva T.Z., Kudrevych O.M.,
Kudria V.I., Kuzmenko S.A., Lakusta N.M., Lavrinchuk I.O., Lebediuk M.M., Leshchenko K.A.,
Loskutova I.Ye., Lysenko D.A., Makieieva N.I., Malkovych N.M., Martsynik Ye.M., Martyniuk L.P.,
Mavrutenkov V.V., Mazorchuk S.H., Medvediev M.V., Miakinkova L.O., Mierkulova N.F., Mitiunina N.I.,
Mostovy Yu.M., Mysak A.I., Nadraha O.B., Nechytailo Yu.M., Nykoniuk T.R., Osetrova M.S., Osiadla E.S.,
Ospanova T.S., Ostropolets M.S., Ostrovsky I.M., Pavlovych L.V., Petriuk B.V., Pisotska L.A., Prus L.O.,
Pryshliak O.Ya., Radchenko O.M., Radutna O.A., Rak L.M., Reva V.B., Riabenko E.B., Riapolova T.L.,
Romanenko V.N., Rud V.O., Seleznova S.V., Semeniak A.V., Semenukha K.V., Serheta I.V.,
Shapkin V.Ye., Shcherbinin O.V., Shkondina O.F., Shkrobanets I.D., Shorikov Ye.I.,
Shostakovych-Koretska L.R., Shtyker S.Yu., Shumko B.I., Shvyhar L.V., Sikorska M.V., Sliepichko Yu.M.,
Slyva V.I., Smoliak L.L., Soboleva N.P., Sokolov O.B., Soldak I.I., Soloviov I.Ye., Sorochan V.V.,
Sotnik Yu.P., Suk V.H., Sushkov M.T., Sychova V.V., Sylenko H.Ia., Synoverska O.B., Syvozhelizov A.V.,
Talalaienko Yu.O., Todoriko L.D., Tokariev A.V., Tonkohlas O.A., Toropchyn V.I., Troian V.I.,
Tsariova N.M., Tsertiy V.P., Tsiapa N.A., Tsyvenko O.I., Tuchkina I.O., Vankhanen V.D., Volianska A.H.,
Vorokhta Yu.M., Yakovenko I.K., Yermolenko T.O., Yurieva L.M., Zakharchenko Yu.B., Zheliba M.D.,
Znakhurenko L.S., Zotov O.S. and Committees of professional expertise

Item reviewers. Ahafonova O.O., Anisimov Ye.M., Babiak T.Ye., Bobrytska V.V., Borysova T.M.,
Chursina T.Ya., Dyndar O.A., Holovanova I.A., Hryhorov Yu.B., Hubka V.O., Hutsalenko O.O.,
Kalinina S.Yu., Kanikovsky O.Ye., Karapetian K.H., Khomaziuk T.A., Kolesnyk O.M., Koloskova O.K.,
Kolosovych I.V., Kopcha V.S., Kravchenko O.V., Kutovy O.B., Kuzmina I.Yu., Malanchuk L.M.,
Martyniuk L.P., Mavrutenkov V.V., Medvediev M.V., Mishchenko V.P., Moroz L.V., Muravska O.M.,
Petrushenko V.V., Prokhorova M.P., Pryshliak O.Ya., Puzanova O.H., Pyptiuk O.V., Radchenko O.M.,
Shestakova I.V., Stovban I.V., Sydorchuk R.I., Tsvirenko S.M., Usenko S.H., Vakaliuk I.P., Volianska A.H.

The book includes test items for use at licensing integrated examination “Krok 2. Medicine” and
further use in teaching.

The book has been developed for students of medical, pediatric and medical-and-prophylactic
faculties and academic staff of higher medical educational establishments.

Approved by Ministry of Public Health of Ukraine as examination and teaching


publication based on expert conclusions (Orders of MPH of Ukraine of
14.08.1998 №251, of 27.12.1999 №303, of 16.10.2002 №374, of 29.05.2003 №233).

© Copyright Testing Board.


Krok 2 Medicine (англомовний варiант, iноземнi студенти) 2018 рiк 1

1. A patient with Morgagni-Adams-Stokes A. Spontaneous pneumothorax


syndrome has fainted when walking up the B. Hemothorax
stairs. The skin is pale, the pupils are dilated, C. Lobar pneumonia
tonoclonic spasms are observed, rib cage is D. Pulmonary embolism
immobile. Make the diagnosis: E. Acute pleurisy

A. Clinical death 6. A 34-year-old man on the 3rd day of ceftri-


B. Social death axone treatment for acute otitis (daily dosage
C. Preagony - 2 grams) developed diarrhea occurring 5-6 ti-
D. Agony mes per day. Feces are without mucus or blood
E. Biological death admixtures. Temperature is 36.6o C . Gregersen
reaction (occult blood in feces) is negative.
2. A 32-year-old welder complains of weakness Stool culture detected no pathogenic germs.
and fever. His illness initially presented as What is the most likely cause of diarrhea in this
tonsillitis one month earlier. On examination: case?
temperature - 38.9o C , respirations - 24/min.,
pulse - 100/min., blood pressure - 100/70 mm A. Antibiotic-associated diarrhea
Hg, hemorrhages on the legs, enlargement of B. Intestinal dysbiosis
the lymph nodes. Complete blood count shows C. Bacterial overgrowth syndrome
Hb- 70 g/L, RBC- 2.2 · 1012 /L, WBC- 3.0 · 109 /L D. Ulcerative colitis
with 32% of blasts, 1% of eosinophiles, 3% of E. Crohn’s disease (regional enteritis)
bands, 36% of segments, 20% of lymphocytes,
and 8% of monocytes, ESR- 47 mm/hour. What 7. A 24-year-old patient visited a doctor
is the cause of anemia in this case? complaining of enlargement of his submaxi-
llary lymph nodes. Objectively: submaxillary,
A. Acute leukemia axillary and inguinal lymph nodes are enlarged.
B. Chronic lympholeukemia Chest X-ray shows: enlarged lymph nodes
C. Aplastic anema of mediastinum. Blood test: erythrocytes -
D. B12 -deficient anemia 3.4 · 1012 /L, Hb- 100 g/L, blood colour index
E. Chronic hemolytic anemia - 0.88, platelets - 190 · 109 /L, leucocytes -
3. After a 5-day-long celebration of his 7.5 · 109 /L, eosinophiles - 8%, band neutrophi-
daughter’s wedding a 65-year-old patient ”saw” les - 2%, segmented neutrophiles - 67%,
in his yard many cats, chickens, and rats. He tri- lymphocytes - 23%, ESR - 22 mm/hour. What
ed to chase them away, but was scared off when test must be prescribed to verify the cause of
the animals started scolding him and tried to lymphadenopathy?
harm him. What is the likely diagnosis? A. Open biopsy of the lymph nodes
A. Delirium tremens B. Abdominal US
B. Senile psychosis C. Mediastinum tomography
C. Schizophrenia D. Puncture biopsy of the lymph nodes
D. Organic brain syndrome E. Sternal puncture
E. Reactive hallucinosis 8. During medical examination a cadet in the
4. A regional cardiologist is given a task to naval college was detected to have a painless
develop a plan for preventive measures ai- dense ulcer 1.5x0.5 in size in his perianal area at
med at decreasing cardiovascular mortality the 2 o’clock position. The ulcer floor resembles
rates. What measures should be planned for ”old fat”. What is the provisional diagnosis?
secondary prevention? A. Hard syphilitic chancre of the rectum
A. Prevention of recurrences and complications B. Rectal fissure
B. Referring patients for sanatorium-and-spa C. Rectal fistula
treatment D. Anal cancer
C. Prevention of diseases E. Anal crypt suppuration
D. Referring patients for inpatient treatment 9. A 7-year-old boy has severe pulmonary
E. Optimization of lifestyle and living conditions mucoviscidosis (cystic fibrosis). He complai-
5. During physical exertion a man suddenly ns of dyspnea and blood expectoration.
developed acute chest pain on the right and Objectively he presents with lagging physi-
dyspnea. Objectively he assumes forced half- cal development, acrocyanosis, hepatomegaly,
sitting position in the bed, presents with diffuse drumstick fingers, and nail plates resembling
cyanosis, resting tachypnea of 38/min., the ri- a ”clock face”. Provisional diagnosis of chronic
ght side of the thorax is enlarged and does not pulmonary heart disease is made. What exami-
participate in the respiratory process; percussi- nation would be the most informative for di-
on on the right reveals tympanic resonance and agnosis confirmation?
absence of respiration.What is the most likely A. Doppler echocardiography
diagnosis in this case? B. Electrocardiography
C. Chest X-ray
D. Rheography of the pulmonary artery
E. Ultrasound of the liver
Krok 2 Medicine (англомовний варiант, iноземнi студенти) 2018 рiк 2

10. A 10-year-old boy, who was outdoors in chills, acute headache, myalgia. In the axillary
windy and cold weather, developed moderate region the lymph node enlarged up to 3x6 cm
pain and tingling in his fingers and toes. When can be palpated. The lymph node is dense,
he had returned home, his parents noticed that intensely painful, slightly mobile, without clear
the tips of his fingers and toes were white and margins; the skin over the node is hyperemic
their sensitivity was lost. The affected areas and tight. Tachycardia is present. Make the
are warming up, the fingers are tingling and preliminary diagnosis:
in pain. Skin pallor changed into redness, ti-
ngling stopped, slight itching and swelling of A. Plague
the fingers appeared. Determine the frostbite B. Sepsis
degree in this child: C. Tularemia
D. Lymphadenitis
A. Frostbite of the I degree E. Anthrax
B. Perniosis
C. Frostbite of the II degree 15. Caries morbidity rate is 89% among resi-
D. Frostbite of the III degree dents of a community. It is determined that
E. Frostbite of the IV degree fluorine content in water is 0.1 mg/L. What
preventive measures should be taken?
11. A 25-year-old woman complains of fati-
gue, dizziness, hemorrhagic rashes on the skin. A. Water fluorination
She has been presenting with these signs for a B. Tooth brushing
month. Blood test: erythrocytes - 1.0 · 1012 /L, C. Fluorine inhalations
Hb- 37 g/L, colour index - 1.1, leukocytes - D. Sealant application
1.2 · 109 /L, platelets - 42 · 109 /L. What analysis E. Introduce more vegetables to the diet
would be the most advisable for diagnosis- 16. During appointment with the doctor
making in this case? a man complains of painful itching rashes
that appeared on his skin under the beard
A. Sternal puncture (bone marrow biopsy) and moustache one year ago, with frequent
B. Splenic biopsy exacerbations occurring throughout the year.
C. Liver biopsy Objectively the skin of the facial hair growth
D. Coagulation studies areas is bluish-purple, thickened, with pustules,
E. US of the gastrointestinal tract erosions, and scabs covering its moist surface.
12. A 12-year-old boy with hypertrophic cardi- The fistulous tracts are surrounded by the
omyopathy complains of dyspnea caused by area of loose pink-red granulation and di-
the slightest physical exertion. Echocardi- scharge pus. The skin resembles mulberry in
ography detected asymmetrical left ventricular appearance. Make the diagnosis:
hypertrophy, signs of pulmonary hypertension, A. Sycosis
and left ventricular dilatation, its ejection fracti- B. Acne rosacea
on is 59%. These developments are indicative C. Lupus erythematosus
of: D. Deep trichophytosis
A. Heart failure with preserved ejection fraction E. Tuberculous lupus
B. Heart failure with reduced ejection fraction 17. A 60-year-old man presents with ischemic
C. Primary pulmonary hypertension heart disease and heart failure of the IV class
D. Essential hypertension according to NYHA (New York Heart Associ-
E. Symptomatic arterial hypertension
ation) that manifests as dyspnea at rest. There
13. A 35-year-old man complains of rapidly are moist crackles in the patient’s lungs. Liver
incresing fatigue, palpitations, ”visual snow”, +4 cm, lower limbs are swollen. Ejection fracti-
dizziness. He has a history of peptic ulcer of the on is 25%. What sign is the most indicative
stomach. Objectively the skin is pale. Vesicular when determining functional class of heart fai-
respiration is observed in the lungs. Systolic lure according to NYHA?
murmur is detected over the cardiac apex, heart
rate is 100/min., BP is 100/70 mm Hg. The epi- A. Degree of dyspnea
gastrium is slightly tender on palpation. Blood B. Moist crackles in the lungs
C. Swollen lower limbs
test: erythrocytes - 3.2 · 1012 /L, Нb- 100 g/L, D. Decrease of ejection fraction
color index - 0.94. What type of anemia is the E. Extent of liver enlargement
most likely present in this case?
18. A 39-year-old man, a battery attendant,
A. Posthemorrhagic anemia suddenly developed weakness, loss of appeti-
B. Sideroblastic anemia te, nonlocalized colicky abdominal pains, and
C. Iron-deficiency anemia nausea. Objectively his skin is gray; there is
D. Hemolytic anemia pink-gray stripe on his gums; the stomach is
E. Hypoplastic anemia soft and sharply painful. Blood test detected
14. A 35-year-old patient has been sufferi- erythrocytes with basophilic stippling and
ng from an illness for 3 days. 5 days ago he anemia. The patient has a history of peptic
returned from a trip to Africa. The onset of di- ulcer disease of the stomach. There is tendency
sease was accompanied by fever up to 40o C , to constipation. What is the most likely provisi-
onal diagnosis?
Krok 2 Medicine (англомовний варiант, iноземнi студенти) 2018 рiк 3

5th intercostal nerves on the left the skin is


A. Saturnism (lead poisosning) hyperemic and there are tight clusters of small
B. Acute appendicitis vesicles filled with clear serous content. What is
C. Perforation of gastric ulcer the most likely diagnosis?
D. Acute cholecystitis
E. Chronic alcoholism A. Herpes zoster
B. Herpes simplex
19. A 9-month-old infant presents with delayed C. Streptococcal impetigo
tooth eruption and fontanel closure, weakness, D. Pemphigus
and excessive sweating. What type of hypovi- E. Dermatitis herpetiformis (Duhring’s disease)
taminosis is the most likely in this child?
24. A worker of a blowing shop complains of
A. Hypovitaminosis D headache, irritability, sight impairment - he sees
B. Hypovitaminosis C everything as if through a ”net”. Objectively:
C. Hypovitaminosis B1 hyperemic sclera, thickened cornea, decreased
D. Hypovitaminosis B6 opacity of pupils, visual acuity is 0.8 in the left
E. Hypovitaminosis A eye, 0.7 in the right eye. The worker uses no
means of personal protection. What diagnosis
20. A 10-year-old girl exhibits high level of

physical development ( + 3σ ), her body
is the most likely?
length increased by 10 cm within a year (whi- A. Cataract
ch is double the norm for her age group), the B. Conjunctivitis
number of permanent teeth corresponds wi- C. Keratitis
th the age norm (20), the development of her D. Blepharospasm
secondary sex characteristics is three years E. Progressive myopia
ahead of her age (Ма, Р, Ах, Menarche).
Development rate ahead of her biological age 25. For a week a 42-year-old patient has been
can occur due to: suffering from fever attacks followed by high
temperature, which occur every 48 hours. Body
A. Endocrine disorders temperature raises up to 40o C and decreases in
B. Acceleration 3-4 hours with excessive sweating. The patient
C. Certain components of her diet presents with loss of appetite and general fati-
D. Sports training gue. The skin is pale and sallow. The liver and
E. Deficient hygienic education spleen are enlarged and dense on palpation.
What method of diagnosis verification would
21. An infant has been born at the 41st week of be most efficient?
gestation. The pregnancy was complicated wi-
th severe gestosis of the second semester. The A. Microscopy of blood smear and thick blood
weight of the baby is 2400 g, the height is 50 film
cm. Objectively: the skin is flabby, the layer B. Complete blood count
of subcutaneous fat is thin, hypomyotonia, C. Bacteriological analysis
neonatal reflexes are weak. The internal organs D. Immune-enzyme assay
are without pathologic changes. This newborn E. Microscopy of hanging blood drop
can be assessed as a:
26. A 16-year-old adolescent living in a rural
A. Full-term infant with intrauterine growth area has been bitten in the shin by a stray
retardation dog. The wound is superficial. Regular vacci-
B. Premature infant nation against tetanus was received 3 months
C. Immature infant ago. What treatment tactics would be the most
D. Postmature infant advisable in this case?
E. Full-term infant with normal body weight
A. Antirabies vaccination
22. A patient suffering from infiltrati- B. Antirabies immunoglobulin
ve pulmonary tuberculosis was prescribed C. Tetanus toxoid adsorbed
streptomycin, rifampicin, isoniazid, pyrazi- D. Antitetanus serum
namide, vitamin C. One month after the begi- E. Antitetanus immunoglobulin
nning of the treatment the patient started
complaining of reduced hearing and tinnitus. 27. A 26-year-old woman is suspected to
What drug has such a side effect? suffer from systemic lupus erythematosus with
systemic lesions of skin, vessels, joints, serous
A. Streptomycin tunics, and heart tissue that developed after
B. Isoniazid photosensitization. The following is detected in
C. Rifampicin blood analysis: LE cells, antibodies to native
D. Pyrazinamide ds-DNA, isolated anti-centromere antibodies,
E. Vitamin C rheumatoid factor is 1:100, Wassermann reacti-
on is positive, circulating immune complex is
23. The doctor has an appointment with a pati- 120 units. What immunological indicators are
ent, who 2 days ago developed severe chest pain considered to be specific to this disease?
on the left, general weakness, high temperature,
and headache. Objectively along the 4th and
Krok 2 Medicine (англомовний варiант, iноземнi студенти) 2018 рiк 4

A. ds-DNA antibodies is 106/min., rhythmic. Blood pressure is 90/60


B. Rheumatoid factor mm Hg. Cardiac borders are unchanged, heart
C. Anti-centromere antibodies sounds are weakened, at the cardiac apex there
D. Immunoglobulin A is soft systolic murmur. What factor would be
E. Increased circulating immune complex the most indicative of the likely disease eti-
ology?
28. A group of 5 had been resting in a forest,
they were drinking alcohol and eating canned A. Anti-streptolysin O
mushrooms and cured fish. The next day two B. C-reactive protein
of them were hospitalized with disturbed visi- C. Creatine kinase
on, swallowing and respiration; the third one D. Rheumatoid factor
presented with acute general weakness and dry E. Seromucoid
mouth. The remaining two were healthy. A tick
was detected on the skin of one of the healthy 33. A 20-year-old patient complains of severe
group members. What is the most likely di- headache, double vision, weakness, fever,
agnosis? irritability. Objectively: body temperature is
38.1o C , the patient is reluctant to contact, sensi-
A. Botulism tive to stimuli. There are ptosis of the left
B. Tick-borne encephalitis eyelid, exotropia, anisocoria S>D, pronounced
C. Alcohol poisoning meningeal syndrome. On lumbar puncture
D. Mushroom poisoning the cerebrospinal fluid flowed out under a
E. Lyme borreliosis pressure of 300 mm Hg, the fluid is clear, sli-
ghtly opalescent. 24 hours later there appeared
29. A 23-year-old man complains of facial fibrin film. Protein - 1.4 g/L, lymphocytes - 600/3
edema, headache, dizziness, low urinary output,
per mm3 , sugar - 0.3 mmol/L. What is the provi-
urine discoloration (dark red). These complai- sional diagnosis?
nts arose after the patient had had a case of
acute tonsillitis. On examination there are faci- A. Tuberculous meningitis
al edema, the skin is pale, temperature is 37.4o C ; B. Meningococcal meningitis
heart rate is 86/min., blood pressure is 170/110 C. Lymphocytic (Armstrong’s) meningitis
mm Hg. Heart sounds are muffled, the II heart D. Syphilitic meningitis
sound is accentuated over the aorta. What eti- E. Mumps meningitis
ological factor is the most likely in this case?
34. A 32-year-old woman complains of increasi-
A. Beta-hemolytic streptococcus ng spastic pains in her lower abdomen that
B. Staphylococcus aureus occur after emotional stress. Bowel movements
C. Streptococcus viridans are intermittent: 2-3 defecations after waking
D. Streptococcus pyogenes in the morning alternate with constipations
E. Staphylococcus saprophyticus that last for 1-2 days. Objectively body mass
30. During examination a 4-month-old chi- is retained, palpation of the sigmoid colon is
ld with meningococcemia presents with moderately painful. Hb- 130 g/L, leukocytes -
acrocyanosis, cold extremities, tachypnea, and 5.2 g/L, ESR- 9 mm/hour. Rectoromanoscopy is
thready pulse, blood pressure of 30/0 mm Hg, painful due to spastic condition of the intestine,
anuria, and sopor. What clinical syndrome is it? intestinal mucosa is without changes. Intestinal
lumen contains large amounts of mucus. What
A. Septic shock is the most likely diagnosis in this case?
B. Neurotoxicosis
C. Exicosis A. Irritable bowel syndrome
D. Encephalic syndrome B. Crohn’s disease (regional enteritis)
E. Acute renal failure C. Nonspecific ulcerative colitis
D. Acute mesenteric ischemia
31. The pregnancy is full term. The body wei- E. Malabsorption syndrome
ght of the parturient woman is 62 kg. Fetus
is in a longitudinal lie, the head is engaged to 35. The inpatient surgery unit has introduced
the pelvic inlet. Belly circumference is 100 cm. the method of laparoscopic cholecystectomy
Uterine fundus height is 35 cm. What body wei- into its practice. As the result the average
ght of the fetus can be expected? duration of postoperative care provided to
the patients could be reduced to 3.4±0.8 days
A. 3.5 kg compared to 7.3±1.1 days that were required
B. 4 kg after non-laparoscopic cholecystectomy. What
C. 2.5 kg method of medical statistics can confirm the
D. 3 kg statistical significance of the difference between
E. 4.5 kg these two estimates?
32. An 18-year-old young man complains of
pain in his knee and elbow joints and body
temperature up to 39.5o C . One week and a half
earlier developed sore throat. On examinati-
on his body temperature is 38.5o C . Swelling of
the knee and elbow joints is observed. Pulse
Krok 2 Medicine (англомовний варiант, iноземнi студенти) 2018 рiк 5

A. Calculation of Student’s confidence interval This kind of pain attacks has never been
B. Calculation of correlation coefficient detected in the patient before. Plain abdomi-
C. Calculation of standardized ratio nal X-ray reveals no pathologic shadows.
D. Calculation of average values (measures of Ultrasound detects a hyperechogenic mass 1.5
central tendency) cm in diameter, which reflects sound wave, in
E. Calculation of relative values the enlarged right renal pelvis. What diagnosis
is the most likely?
36. A 37-year-old worker during a fire ended up
in the area of high CO concentration. He was A. Renal calculus
delivered to a hospital in unconscious state. B. Benign renal tumor
Objectively: the skin of his face and hands is cri- C. Renal cyst
mson. Respiration rate is 20/min. ECG: alterati- D. Renal tuberculosis
ons specific for hypoxic myocardium. Hourly E. Malignant renal tumor
diuresis is 40 ml. Blood test: erythrocytes -
4.5 · 1012 /L, Нb- 136 g/L, color index - 0.9, ESR- 41. A 60-year-old woman complains of
3 mm/hour, carboxyhemoglobin - 5%. What unbearable pain in her right subcostal area. She
criterion allows determining the severity of the has a history of acute pancreatitis. Temperature
patient’s condition? is 38.2o C . Objectively her sclera are icteric.
There are no signs of peritoneal irritation.
A. Carboxyhemoglobin concentration Ortner’s and Gubergrits’ symptoms are positi-
B. Respiratory disorders ve. Urine diastase is 320 U/L. What is the most
C. ECG results likely diagnosis?
D. Extent of trophic disorders
E. Development of chronic renal failure A. Exacerbation of chronic pancreatitis
B. Acute cholangitis
37. The right arm of a newborn is stretched C. Chronic cholecystitis
along the torso with all its joints extended; the D. Acute cholecystitis
shoulder is rotated inwards, while the forearm E. Pancreatic cancer
is pronated, the hand is in the position of
palmar flexion. Spontaneous movements are 42. A 64-year-old man complains of cough wi-
absent in the shoulder and elbow joints, passive th expectoration consisting of blood-streaked
movements are painless. What is the most likely mucus, dyspnea, low grade fever and general
diagnosis? fatigue. He has been presenting with these
symptoms for 3 months. He has been smoki-
A. Duchenne-Erb palsy, superior proximal type ng since early adolescence. Objectively to
B. Dejerine-Klumpke palsy, inferior distal type is 37.4o C , respirations are 26/min., pulse is
C. Total obstetric palsy 82/min., rhythmic, blood pressure is 130/85 mm
D. Osteomyelitis of the right humerus Hg. The right side of the thorax lags behi-
E. Poliomyelitis nd in the respiratory process, dull percussion
sound and acute decrease of breathing activity
38. In April during the medical examination are observed there. X-ray shows homogeneous
of various population groups, 27% of indivi- shadow of the lung field on the right with medi-
duals presented with low working ability and astinum displacement towards the affected side.
rapid fatigability. The following symptoms were What is the most likely diagnosis?
observed in the affected individuals: swollen
friable gingiva that bleeds when pressed, A. Central lung cancer
hyperkeratosis follicularis not accompanied by B. Exudative pleuritis
skin dryness. These symptoms most likely result C. Pleuropneumonia
from the following pathology: D. Pulmonary tuberculosis
E. Multiple bronchiectasis
A. C -hypovitaminosis
B. Parodontosis 43. For 4 days a 35-year-old man has been


C. A-hypovitaminosis treated in the resuscitation unit for acute renal
D. 1 -hypovitaminosis failure caused by compartment syndrome. The
E. Polyhypovitaminosis patient is disoriented. ECG shows high T
waves and right ventricular extrasystoles. His
39. A 1-year-old child with a case of URTI central venous pressure is 159 mmH2 O; for the
suddenly developed noisy respirations with di- last 3 hours auscultation has been detecting
fficult inspiration, intercostal retractions, and isolated moist crackles in the lungs. Respirati-
barking cough on the 2nd night after the di- ons are 32/min. Blood test: residual nitrogen -
sease onset. What is the most likely diagnosis? 62 mmol/L, K + - 7.1 mmol/L, Cl− - 78 mmol/L,
A. Stenosing laryngotracheobronchitis N a+ - 120 mmol/L, Ht- 0.32 L/L, Hb- 100 g/L,
blood creatinine - 0.9 mmol/L. In this case the
B. Acute pulmonary inflammation most advisable would be to perform:
C. Bronchial asthma
D. Acute bronchitis A. Hemodialysis
E. Acute bronchiolitis B. Plasmasorption
40. A 46-year-old woman complains of severe C. Hemosorption
D. Plasmafiltration
pain attacks in the right lumbar area, which E. Ultrafiltration
irradiate to the lower abdomen, and nausea.
Krok 2 Medicine (англомовний варiант, iноземнi студенти) 2018 рiк 6

44. A 55-year-old woman, a cook, complai- with tonic spasm of the facial muscles. What is
ns of pain in her right knee joint that has the most likely diagnosis among those listed?
been troubling her for a month and intensifi-
es in the evening. Objectively she is overwei- A. Trigeminal neuralgia
ght, the knee joint is swollen, creaks during B. Glossopharyngeal neuralgia
movement, palpation reveals localized pain. C. Temporomandibular joint arthritis
The 1st metatarsophalangeal articulation is D. Facial migraine
deformed on the both feet. No changes in blood E. Maxillary sinusitis
and urine are detected. What should be visible
on the patient’s X-ray? 48. A 10-year-old boy with symptoms of arthri-
tis and myocarditis was delivered into a hospi-
A. Joint space narrowing, marginal osteophytes tal. Based on clinical examination the prelimi-
B. Osteoporosis, joint space narrowing, singular nary diagnosis of juvenile rheumatoid arthritis
usurations was made. What symptom is the most contri-
C. Joint space narrowing, multiple usurations, butive for the diagnostics of this disease?
subluxations
D. Epiphyseal erosions, bony ankylosis A. Reduced mobility of the joints in the morning
E. Joint space narrowing, round bone defects B. Regional hyperemia of the joints
C. Affection of the large joints
45. A 52-year-old woman has been suffering D. Enlarged heart
for 2 years from dull, occasionally exacerbating E. Increased heart rate
pain in her right subcostal area, occurring after
eating high-fat foods, bitter taste in her mouth 49. A 25-year-old patient was delivered to
in the morning, constipations, and flatulence. an infectious diseases unit on the 3rd day
Objectively she has excess weight, her body of illness with complaints of headache, pain
temperature is 36.9o C ; there is a coating on the in lumbar spine and gastrocnemius muscles,
root of her tongue; the abdomen is moderately high fever, chill. Objectively: condition of
distended and painful in the area of gallbladder moderate severity. Scleras are icteric. Pharynx is
projection. What examination would be the hyperemic. Tongue is dry with dry brown coati-
most helpful for diagnosis-making? ng. Abdomen is distended. Liver is enlarged
by 2 cm. Spleen is not enlarged. Palpation of
A. Ultrasound muscles, especially gastrocnemius muscles, is
B. Duodenal intubation painful. Urine is dark in colour. Stool is normal
C. Cholecystography in colour. The most likely diagnosis is:
D. Duodenoscopy
E. Liver scanning A. Leptospirosis
B. Viral hepatitis type A
46. A 57-year-old woman complains of C. Malaria
weakness, dyspnea, loss of appetite, and li- D. Infectious mononucleosis
quid feces. She has been suffering from this E. Yersiniosis
condition for 2 years. Objectively she presents
with pale skin, subicteric sclera, and bright- 50. A 28-year-old woman complains of
red fissured tongue. Lymph nodes are not skin hemorrhages after minor traumas and
enlarged. Pulse - 100/min. BP- 105/70 mm Hg. spontaneous appearance of hemorrhages on
Liver +3 cm, the spleen cannot be palpated. the front of her torso and extremities. On
examination: the skin is variegated (old and
Blood test: erythrocytes - 1.2 · 1012 /L, Нb- 56
g/L, color index - 1.4, macrocytes, leukocytes new hemorrhages), bleeding gums. Blood
- 2, 5 · 109 /L, eosinophils - 1%, juvenile - 1%, platelets - 20 · 109 /L; in the bone marrow there
metamyelocytes - 1%, band neutrophils - 8%, is increased number of megakaryocytes and
segmented neutrophils - 47%, lymphocytes - no platelet production. Treatment with steroid
38%, monocytes - 4%, reticulocytes - 0.1%, hormones was effective. What is the likely di-
agnosis?
platelets - 100 · 109 /L, ESR- 30 mm/hour, indi-
rect bilirubin - 26 mmol/L. What changes can be A. Idiopathic thrombocytopenic purpura
expected in the bone marrow puncture materi- B. Hemophilia
al? C. Rendu-Osler-Weber disease (Hereditary
A. Prevalence of megaloblasts hemorrhagic telangiectasia)
B. Increased number of sideroblasts D. Disseminated intravascular coagulation
C. Erythroid hyperplasia E. Acute vascular purpura
D. Presence of blast cells 51. A 7-year-old boy has been an inpatient for
E. Prevalence of lymphoid tissue 1.5 months. He had been delivered to the hospi-
47. A 45-year-old woman complains of tal with complaints of edemas all over his body,
paroxysmal intolerable facial pain on the left low urine output, and headache. Clinical uri-
with attacks that last for 1-2 minutes. Attacks nalysis: proteins - 7.1 g/L, leukocytes - 1-2 in the
are provoked by chewing. The disease onset vision field, erythrocytes - 3-4 in the vision fi-
was two month ago after overexposure to cold. eld. During the course of treatment the edemas
Objectively: pain at the exit points of the tri- gradually dissipated, headache abated, diuresis
geminal nerve on the left. Touching near the normalized. Daily urine proteins - 3 g/L. Bi-
wing of nose on the left induces new pain attack ochemical blood test: total protein - 43.2 g/L,
Krok 2 Medicine (англомовний варiант, iноземнi студенти) 2018 рiк 7

urea - 5.2 mmol/L, cholesterol - 9.2 mmol/L. suprapubic area is without alterations, external
What glomerulonephritis syndrome is the most genitalia are non-pathologic. On rectal investi-
likely to be present in the patient? gation: prostate is not enlarged, painless, has
normal structure. Cystoscopy revealed no
A. Nephrotic alterations. What is the most likely diagnosis?
B. Nephritic
C. Isolated urinary A. Renal carcinoma
D. Hematuric B. Bladder tuberculosis
E. Mixed C. Varicocele
D. Dystopic kidney
52. The mother of a 3-month-old child came to E. Necrotic papillitis
a family doctor with complaints of her child
being physically underdeveloped and suffering 56. A 36-year-old man has been complaini-
from cough attacks and dyspnea. Anamnesis: ng of marked weakness, low appetite, and
the child is the result of the second full-term nausea for the last year, year and a half. Wi-
pregnancy with the risk of miscarriage (the first thin the last year he has lost 10 kg of body
child died of pulmonary pathology at the age of mass. The skin is darkened, especially on his
4 months, according to the mother). Body mass face, neck, and arms. Skin folds and nipples
at birth is 2500 g. Cough attacks were observed are hyperpigmented, there are pigment spots
from the first days of life, twice the child was on the patient’s inner thighs. Pulse is 60/min.,
treated for bronchitis. Considering the severi- blood pressure is 80/50 mm Hg. What is the
ty of the child’s condition the doctor made the provisional diagnosis?
referral for hospitalization. What diagnosis was
most likely stated in the referral? A. Adrenal insufficiency
B. Diabetes mellitus
A. Mucoviscidosis (Cystic fibrosis) C. Chronic gastritis
B. Acute obstructive bronchitis D. Hemochromatosis
C. Recurrent obstructive bronchitis E. Cholestatic hepatitis
D. Pertussis
E. Acute obstructive pneumonia 57. A 5-year-old child that contacts with vi-
ral hepatitis in the kindergarten presents wi-
53. A 10-year-old girl complains of th increased body temperature up to 38o C ,
stomachache that appears and intensifies after weakness, low appetite, single case of vomiti-
she eats rough or spicy food, sour eructation, ng, dull pain in the subcostal area on the right.
heartburn, frequent constipations, headaches, The child is provisionally diagnosed with viral
irritability. She has been presenting with these hepatitis. What examination would be the most
signs for 12 months. Her meals are irregular and informative for diagnosis confirmation?
consist of dry food. Objectively her diet is suffi-
cient in calories. The tongue is moist with white A. ALT activity in blood
coating near the root. The abdomen is soft and B. Urine analysis for bile pigments
painful in the epigastrium. What method would C. Feces analysis for stercobilin
be optimal for diagnosis-making in this case? D. Blood test for bilirubin
E. Thymol turbidity test
A. Esophagogastroduodenoscopy
B. Intragastric pH-metry 58. A patient with trauma of the lower thi-
C. Fractional gastric analysis (Fractional test rd of the forearm volar surface caused by a
glass shard came to a first-aid center. Objecti-
meals) vely: flexion of the IV and V fingers is impai-
D. Phase-contrast X-ray imaging red, sensitivity of the inner dorsal and palmar
E. Biochemical blood test surfaces of the hand and IV finger is decreased.
54. A 23-year-old patient had taken 1 g of aspi- What nerve is damaged?
rin to treat acute respiratory infection. After A. Ulnar
that he developed an asthmatic fit with labored B. Radial
expiration that was arrested by introduction of C. Median
aminophylline. The patient’s medical history is D. Musculocutaneous
not burdened with allergies. The patient has E. Axillary
undergone two surgeries for nasal polyposis in
the past. What diagnosis is most likely? 59. A man diagnosed with closed-angle
glaucoma, grade IIa, of the right eye is regi-
A. Aspirin-induced asthma stered for regular medical check-ups. In the
B. Atopic bronchial asthma evening an acute glaucoma attack occurred in
C. Infectious allergic bronchial asthma his right eye; an ambulance was called. What
D. Exercise-induced asthma emergency aid would be optimal in this case?
E. Symptomatic bronchospasm
55. A 50-year-old patient was delivered to a
hospital with complaints of blood traces in
urine. Urination is painless and undisturbed.
Macrohematuria had been observed for 3
days. Objectively: kidneys cannot be palpated,
Krok 2 Medicine (англомовний варiант, iноземнi студенти) 2018 рiк 8

A. Pilocarpine, Diacarb (Acetazolamide), lytic A. Pertussis


mixture B. Parainfluenza
B. Atropine eye drops C. Congenital stridor
C. Antibiotic eye drops, broad-spectrum D. Respiratory syncytial infection
D. Sulfacetamide sodium eye drops E. Adenovirus infection
E. Dexamethasone eye drops
64. A man works in casting of nonferrous
60. A 58-year-old patient complains of pain in metals and alloys for 12 years. In the air of
the lower left extremity, which aggravates duri- working area there was registered high content
ng walking, and sensation of cold and numbness of heavy metals, carbon monoxide, and ni-
in the both feet. The patient has been sufferi- trogen. During periodic health examination
ng from this condition for 6 years. Objectively: the patient presents with asthenovegetati-
the skin is pale and dry, with hyperkeratosis. ve syndrome, sharp pains in the stomach,
On the left shin hair is scarce. Pulse cannot be constipations, pain in the hepatic area. In uri-
detected over the pedal and popliteal arteries ne: aminolevulinic acid and coproporphyrin
and is weakened over the femoral artery. On are detected. In blood: reticulocytosis, low
the right limb pulsation of the popliteal artery hemoglobin level. Such intoxication is caused
is retained. What is the most likely diagnosis? by:
A. Atherosclerosis obliterans of the lower A. Lead and lead salts
extremities B. Tin
B. Obliterating endarteritis C. Carbon monoxide
C. Femoral artery thrombosis D. Nitric oxide
D. Raynaud’s disease E. Zinc
E. Buerger’s disease (thromboangiitis obli-
terans) 65. Due to introduction of a new treatment
method, average duration of therapy in
61. A 22-year-old woman, gravida 1, para 0 the experimental group was 12.3±0.2 days
arrived with complaints of sharply painful compared to 15.4±0.4 days in the control group
contractions that occur every 4-5 minutes and that was treated by the old method. What
last for 25-30 seconds. Amniotic fluid did calculations should be made to estimate the
not burst. The fetus is in transverse lie, fetal statistical significance of the difference in the
heartbeats are not affected. Contraction ring is results?
acutely painful, located obliquely at the umbili-
cus. What is the most likely diagnosis? A. T-test (Student’s t-distribution)
B. Sign test (Z-test)
A. Impending uterine rupture C. Matching factor (chi-squared test)
B. Uterine tetany D. Wilcoxon T-test
C. Excessive uterine activity during labor E. Kolmogorov-Smirnov test
D. Discoordinated labor
E. Uterine rupture 66. Establishments participating in medical
examinations include: medical and preventi-
62. During regular check-up the doctor exami- ve treatment facilities, hygiene and preventi-
nes a young woman, a student, with height of ve treatment facilities, sociomedical expert
162 cm and weight of 59 kg. She complains committees, Ministry of Defence medical
that in the evening she becomes unable to see committees, Ministry of Domestic Affairs
clearly the objects around her. Objectively her medical committees, forensic medicine agency,
skin is dry, presents with hyperkeratosis. Her etc. Specify what service deals with sociomedi-
daily ration has the following vitamin content: cal assessment of temporary disability:
vitamin A - 0.5 mg, vitamin B1 - 2.0 mg, vitamin
B2 - 2.5 mg, vitamin B6 - 2 mg, vitamin C - 70 A. Medical and preventive treatment facilities
mg. Make the diagnosis: B. Hygiene and preventive treatment facilities
C. Sociomedical expert committees
A. A-hypovitaminosis D. Ministry of Defence medical committees
B. B1 -hypovitaminosis E. Ministry of Domestic Affairs medical commi-
C. B2 -hypovitaminosis ttees
D. B6 -hypovitaminosis
E. C -hypovitaminosis 67. A 14-year-old boy presents with moderate
bronchial asthma in its exacerbation period.
63. An infant is 2.5 months old. The onset of the What drug should be prescribed to stop an
disease was gradual, the child had normal body acute attack of expiratory dyspnea?
temperature but presented with slight cough.
Within a week the cough intensified, especially A. Salbutamol
at night; on the 12th day the child developed B. Cromolyn sodium (Cromoglicic acid)
cough fits occurring up to 20 times per day and C. Dexamethasone
followed by vomiting. There was one instance D. Lasolvan (Ambroxol)
of respiratory arrest. Make the diagnosis: E. Strophanthine (cardiac glycosides)
68. A newborn with gestational age of 31
weeks presents with hypotonia and depressed
consciousness. Hematocrit is 35%, general
Krok 2 Medicine (англомовний варiант, iноземнi студенти) 2018 рiк 9

cerebrospinal fluid analysis shows increased should the infant be put to the breast?
content of erythrocytes and protein, and low
glucose. These data correspond with the clini- A. In the delivery room
cal presentation of: B. After 12 hours
C. After 2 hours
A. Intracranial hemorrhage D. On the 2nd day
B. Meningitis E. On the 3rd day
C. Sepsis
D. Anemia 74. During assessment of work conditions at the
E. Prenatal infection factory manufacturing mercury thermometers,
the content of mercury vapors in the air of the
69. A 16-year-old girl has primary amenorrhea, working area is revealed to exceed the maxi-
no pubic hair growth, normally developed mum concentration limit. Specify the main
mammary glands; her genotype is 46 ХY; uterus pathway of human body exposure to mercury:
and vagina are absent. What is your diagnosis?
A. Respiratory organs
A. Testicular feminization syndrome B. Intact skin
B. Mayer-Rokitansky-Kuster-Hauser syndrome C. Damaged skin
C. Cushing syndrome D. Gastrointestinal tract
D. Sheehan syndrome E. Mucous tunics
E. Cushing disease
75. In the process of hiring, a prospective
70. 6 hours ago the waters of a 30-year-old employee has undergone preventive medical
gravida 1, para 0, burst; her preliminary peri- examination and was declared fit to work in
od was pathologic and lasted for over 2 days; this manufacturing environment. What type of
the term of pregnancy is 39 weeks. No labor preventive medical examination was it?
activity is observed. Fetal head presents above
the pelvic inlet. Fetal heartbeats are 142/min., A. Preliminary
clear and rhytmic. On vaginal examination the B. Scheduled
uterine cervix is not dilated. What further tacti- C. Periodical
cs should the doctor choose? D. Specific
E. Comprehensive
A. Perform cesarean section
B. Induce cervical dilation with prostaglandins 76. A 47-year-old woman came to the admissi-
C. Stimulate the labor with oxytocin on room with complaints of general weakness,
D. Wait for the onset of spontaneous labor dizziness, vomiting with blood clots. Conditi-
E. Prolong the pregnancy, while providing on onset was 3 hours ago. The patient has no
antibacterial treatment preceding illnesses. Blood pressure is 90/60
mm Hg, pulse is 106/min., of poor volume.
71. Mother of an 8-year-old girl complains that The abdomen is soft, with mild tenderness
the child is too short and has excessive body in the epigastrium. Blood test: erythrocytes -
weight. Objectively: obesity with fat deposits on 2.1·1012 /L, Нb- 70 g/L, hematocrit - 28%. What
the torso and face (round moon-like face), acne, tactics should the doctor on duty choose?
striae on the thighs and lower abdomen, hirsuti-
sm. What hormone can cause such symptoms, A. Consult the surgeon
when in excess? B. Refer the patient to the family doctor
C. Give spasmolytics
A. Cortisol D. Perform gastric lavage
B. Thyroxin E. Make an appointment for colonoscopy
C. Testosterone
D. Insulin 77. A 23-year-old woman came the the
E. Glucagon gynecologist with complaints of blood smears
from her genital tracts that have been observed
72. A 9-year-old girl complains of fever up for a long time. Her menstruation has been
to 38.5o C , headache, inertness, weakness, loss delayed for 8 weeks. Examination shows the
of appetite, stomachache, and frequent pai- uterine body to be enlarged up to 14 weeks of
nful urination. Provisional diagnosis of acute pregnancy. US detected a vesicular mole. What
pyelonephritis is made. Clinical urine analysis: tactics should the doctor choose?
specific gravity - 1016, no protein, leukocytes
- 10-15 in the vision field. What investigation A. Curettage of the uterine cavity
method can verify the diagnosis of urinary tract B. Hormonal treatment
infection? C. Hemostatic treatment
D. Supravaginal uterine amputation
A. Bacteriological inoculation of urine E. Uterectomy
B. Rehberg test (creatinine clearance test)
C. Zymnytsky test (density measurement of 78. A 52-year-old patient complains of pain
daily diuresis) in the right part of her chest, dyspnea, cough
D. Complete blood count with large amounts of foamy sputum emitti-
E. Clinical urine analyses, dynamic testing ng foul smell and resembling ”meat slops”.
Objectively: the patient’s condition is grave,
73. A newborn has Apgar score of 9. When cyanosis is observed, breathing rate is 31/min.,
Krok 2 Medicine (англомовний варiант, iноземнi студенти) 2018 рiк 10

percussion sound above the right lung is 83. A population of a small town often
shortened, auscultation revealed various moi- presents with registered cases of juvenile
st rales (crackles). What is the most likely di- cardiomyopathy, atherosclerosis, hypertensi-
agnosis? on, endocrinopathy, chronic dermatitis, and
arthralgia - signs of Keshan disease. What is
A. Lung gangrene the most likely cause of this pathology?
B. Lung abscess
C. Empyema of pleura A. Selenium deficiency in the environment
D. Multiple bronchiectasis B. Excessive iron in the environment
E. Chronic pneumonia C. Strontium deficiency in the environment
D. Excessive zinc in the environment
79. A man complains of sore throat on the left, E. Excessive manganese in the environment
pain in his left ear, to up to 39o C , and nasal
sound of his voice. Disease onset was 5 days 84. In 2 hours after a traffic accident a 28-year-
ago. Marked trismus and increased salivation old man in grave condition was delivered to a
are observed. The head tilts to the left shoulder. hospital. The patient complains of abdominal
Left side of the soft palate presents with swelli- pain. He received a blow to the abdomen wi-
ng, hyperemia, and infiltration. Retromandi- th the steering wheel. Objective examination
bular lymph nodes on the left are acutely pai- revealed the following: the abdomen does not
nful on palpation. Otoscopy results are normal. participate in respiration, is tense and acutely
Make the diagnosis: painful on palpation; abdominal guarding is
present, peritoneal irritation signs are positive,
A. Left-sided peritonsillar abscess hepatic dullness is absent. BP is 90/60 mm Hg,
B. Retropharyngeal abscess heart rate is 120/min. What further treatment
C. Parapharyngeal phlegmon tactics should be chosen?
D. Peritonsillitis on the left
E. Cervical phlegmon on the left A. Laparotomy
B. Laparoscopy
80. A patient has the second and third degree C. Cold to the abdomen
burns of the 15% of the body surface. On the D. Abdominal X-ray
20th day after the trauma the patient presents E. Laparocentesis
with sharp increase of body temperature,
general weakness, rapid vesicular respiration; 85. A 48-year-old woman was arrived to the
facial features are sharpened, BP is 90/50 mm surgical unit with wounds in her thigh. On
Hg, heart rate is 112/min. What complication is examination the wound surface has dirty-gray
it? coating with unpleasant sweet smell. Wound
content resembles raspberry jelly. Skin ti-
A. Sepsis ssues around the wound are glossy and turgid.
B. Pneumonia Palpation reveals moderate crepitation in the
C. Acute intoxication tissues. What microflora is the most likely to
D. Purulent bronchitis cause such inflammation?
E. Anaerobic infection
A. Anaerobic clostridial
81. A patient in the state of clinical death is bei- B. Anaerobic non-clostridial
ng resuscitated through mouth-to-mouth artifi- C. Streptococci
cial pulmonary ventilation and external cardiac D. Staphylococci
massage. A doctor noticed that air does not E. Blue pus bacillus
flow into the patient’s airways and his head and
torso are positioned at the same level. Why 86. The gynecology unit received a patient
is artificial respiration ineffective in the given with uterine bleeding that started 6 hours
case? after induced abortion at the term of 11-12
weeks. Objectively the skin is pale, pulse is
A. Tongue retraction 100/min., blood pressure is 100/70 mm Hg. On
B. Low breathing volume vaginal examination the uterus is painless, its
C. External cardiac massage enlargement corresponds to the 10th week of
D. Probe is absent from the stomach pregnancy; uterine cervix is dilated enough to
E. The patient’s mouth is too small let in one finger, there are fragments of the ferti-
lized ovum. What actions should be taken next:
82. An employee has been sick for 4 months,
further treatment is necessary, the patient is A. Urgent repeated curettage of the uterine
unable to work. Who is authorized to provide cavity
further disability examination of this patient? B. Uterotonic drugs
C. Treatment for acute anemia
A. Sociomedical expert committee D. Antibacterial agents
B. Medical consultative board E. Prescribe rest and continue to monitor the
C. Physician in charge and the head of the patient’s condition
department
D. Chief physician of a medical facility 87. A woman came to the general practitioner
E. Deputy chief responsible for disability exami- with complaints of fatigability, significant wei-
nation ght loss, weakness, and loss of appetite. She has
been presenting with amenorrhea for the last
Krok 2 Medicine (англомовний варiант, iноземнi студенти) 2018 рiк 11

8 month. One year ago she gave birth to a live A. Undo the sutures, drain the wound, and
full-term child. Blood loss during delivery was prescribe antibiotics
2 liters. The woman received blood transfusion B. Prescribe broad spectrum antibiotics and
and blood components. What is the most likely hormonal agents
diagnosis? C. Administer antibiotics intraosseously and
hypothermia locally
A. Sheehan’s syndrome (postpartum hypopitui- D. Inject antibiotics into the area surrounding
tarism) the wound, prescribe spasmolytics and analgesi-
B. Stein-Leventhal syndrome (polycystic ovary) cs
C. Turner’s syndrome E. Remove the fixation, prescribe sulfanilamides
D. Homologous blood syndrome
E. Somatoform autonomic dysfunction 92. A woman with blood group B(III) Rh(+)
gave birth to a full-term healthy boy. Exami-
88. To assess the effectiveness of medical nation on the 3rd day of the infant’s life shows
technologies and determine the power and di- him to have icteric tint to his skin. The chi-
rection of their effect on the public health indi- ld has no problems with suckling, sleep is
cators, the research was conducted to study the nondisturbed. The abdomen is soft, the li-
immunization rate of children and measles inci- ver protrudes by 2 cm from under the costal
dence rate by district. What method of statisti- margin. Complete blood count: hemoglobin -
cal analysis should be applied in this case? 200 g/L, erythrocytes - 5.5 · 1012 /L, total bi-
lirubin - 62 mcmol/L, indirect bilirubin - 52
A. Calculation of correlation coefficient mcmol/L. What condition can be suspected?
B. Calculation of morbidity index among the
nonvaccinated A. Physiologic jaundice
C. Calculation of coefficient of agreement B. Congenital hepatitis
D. Calculation of standardized ratio C. Hemolytic disease of the newborn due to Rh
E. Calculation of statistical significance of the incompatibility
difference between two estimates D. Biliary atresia
E. Hemolytic disease of the newborn due to
89. A 22-year-old woman complains of itchi- ABO incompatibility
ng and profuse discharge from her genital
tracts. The condition developed 10 days ago 93. A 45-year-old man developed constricting
after a sexual contact. Bacterioscopy of a di- retrosternal pain that occurs during walks at
scharge sample detected trichomonads. What the distance of 200 m. Objectively heart rate
drug should be prescribed for treatment in this is 80/min., BP is 160/90 mm Hg. During cardi-
case? opulmonary exercise test at 50 W there is a
depression of S-T segment by 3 mm below the
A. Metronidazole isoline in V3-V4. What is the provisional di-
B. Ampicillin agnosis?
C. Erythromycin
D. Zovirax (Acyclovir) A. Exertional angina pectoris, functional class
E. Valcyclovir III
B. Exertional angina pectoris, functional class
90. A 30-year-old patient was hospitalized in an IV
intensive care unit with a diagnosis of multi- C. Exertional angina pectoris, functional class II
ple bee stings. Skin is pale and covered wi- D. Somatoform autonomic dysfunction,
th cold sweat. Pulse can be palpated only at hypertension type
the carotid arteries and is 110/min.; breathing E. Alcoholic myocardiodystrophy
rate is 24/min., rhytmical, weakened. What drug
must be administered immediately? 94. A multigravida on the 38th week of her
pregnancy complains of increased BP up to
A. Epinephrine hydrochloride 140/90 mm Hg, edema of the shins for 2 weeks.
B. Prednisolone In the last month she gained 3.5 kg of weight.
C. Norepinephrine hydrochloride Urine analysis: protein - 0.033 g/L. Make the
D. Dopamine diagnosis:
E. Tavegyl (Clemastine)
A. Mild preeclampsia
91. A 46-year-old woman has been hospitalized B. Moderate preeclampsia
with open fracture of the left thigh in its middle C. Pregnancy hypertension
third. She underwent the surgery - fixation wi- D. Severe preeclampsia
th extraosseous osteosynthesis plates. On the E. Pregnancy edema
4th day after the surgery she developed pain in
the wound, body temperature rose over 39o C . 95. For the last 15 years a 48-year-old pati-
What measures should be taken in this case? ent has been working at the factory producing
synthetic resins. Lately he has been complai-
ning of significant general fatigue, headaches,
frequent urination (predominantly during the
day), red color of urine. What complication
of benzene nitrocompounds poisoning can be
suspected?
Krok 2 Medicine (англомовний варiант, iноземнi студенти) 2018 рiк 12

A. Anamnestic (history-taking)
A. Malignant tumor of the urinary bladder B. Immediate registration
B. Chronic cystitis C. Immediate examination
C. Chronic prostatitis D. Doing extracts
D. Acute glomerulonephritis E. -
E. Chronic pyelonephritis
100. A 30-year-old woman complains of
96. A 44-year-old patient with postinfarcti- increased body weight and problems with
on cardiosclerosis presents with frequent physical exertion. Her parents are of increased
heart rate disorders and lower extremity body weight as well; typical meals in their fami-
edema. Objectively: Ps- 95/min., irregular, 10- ly are high in fats and carbohydrates. Objecti-
12 extrasystoles per minute. BP- 135/90 mm Hg. vely her blood pressure is 135/80 mm Hg, pulse
The 1st heart sound at the apex is weakened. is 89/min., weight is 87 kg, height is 165 cm.
Pulmonary respiration is rough. The liver is The patient’s skin is clear, with even distributi-
enlarged +2 cm. ECG: irregular sinus rhythm, on of subcutaneous fat; the thyroid gland is
heart rate - 95/min., frequent polytopic ventri- not enlarged; there are no menstrual cycle di-
cular extrasystoles. What antiarrhythmic drug sturbances. What obesity prevention methods
is advisable in this case for treatment and would be the most advisable in this case?
prevention of extrasystole?
A. Dietary treatment, graduated exercise
A. Amiodarone B. Intensive training regimen
B. Lidocaine C. Gastroplasty or gastrojejunal shunt
C. Mexiletine D. Inhibitors of gastrointestinal lipases
D. Quinidine E. Anorectic drugs
E. Novocainamide (Procainamide)
101. A 43-year-old woman complains of pain in
97. A 60-year-old woman started feeling the lumbar area, which irradiates to her left leg
weakness, vertigo, rapid fatigability during the and aggravates on movement, and sensation
last year. Recently she has developed dyspnea of numbness in this leg. Objectively palpati-
and paresthesia observed. Objectively: skin and on of her shin and thigh is painful, there are
mucous membranes are pale and icteric. Li- painful stretch symptoms of on the left and
ngual papillae are smoothed out. Liver and gastrocnemius cramps. There is no sensory loss
spleen are at the edge of costal arch. Blood test: or weakening of reflex responses. Make the di-
Hb- 70 g/L, erythrocytes - 1.7 · 1012 /L, blood agnosis:
color index - 1.2, macrocytes. What drug can be
prescribed on pathogenetic grounds? A. Vertebrogenous lumbar ischialgia on the left
B. Vertebrogenous radicular syndrome of L5-S1
A. Vitamin B12 on the left
B. Vitamin B6 C. Left-sided coxitis
C. Ascorbic acid D. Endarteritis of the lower extremities
D. Iron preparations E. Spinal stroke
E. Vitamin B1
102. A 3-day-old infant with hyperbili-
98. After excessive consumption of fatty food rubinemia (428 mcmol/L) developed di-
a 60-year-old woman suddenly developed pain sturbances manifesting as periodical excitati-
in her right subcostal area, nausea, bile vomi- on and convulsions against the background of
ting, sharp bitter taste in her mouth. In 2 days inertness, hypotension, hypodynamia, and inhi-
she developed jaundice, her urine darkened. bition of unconditioned reflexes, convergent
Objectively: sclera and skin are icteric, the strabismus, rotational nystagmus, and setting-
abdomen is distended, the liver is enlarged by sun eye phenomenon. What is the most likely
3 cm, soft and painful on palpation, Ortner’s, cause of such symptoms?
Murphy’s, Kehr’s, Zakharyin’s, Mayo-Robson’s
signs are positive. What diagnostic technique A. Bilirubin encephalopathy
should be used in the first place to confirm the B. Craniocerebral injury
diagnosis? C. Brain tumor
D. Hydrocephalus
A. Ultrasound of the gallbladder and bile duct E. Infantile cerebral paralysis
B. Fibrogastroduodenoscopy
C. Abdominal X-ray 103. A woman undergoing in-patient treatment
D. Radionuclide scanning of the liver and for viral hepatitis type B developed headache,
gallbladder nausea, recurrent vomiting, memory lapses,
E. Laparoscopy flapping tremor of her hands, rapid pulse.
Sweet smell from the mouth is detected. Body
99. The objective of a statistical research was to temperature is 37.6o C , heart rate is 89/min.
find out to what extent the population peruses What complication developed in the patient?
the available medical services. For this purpose
300 residents of the area were interviewed.
Information was collected by means of a speci-
al questionnaire. What method of collecting
information was used by the researchers?
Krok 2 Medicine (англомовний варiант, iноземнi студенти) 2018 рiк 13

A. Acute liver failure 108. A 38-year-old patient has been delivered


B. Ischemic stroke by an ambulance to a surgical department with
C. Gastrointestinal hemorrhage complaints of general weakness, indisposition,
D. Hypoglycemic shock black stool. On examination the patient is pale,
E. Meningoencephalitis there are dotted hemorrhages on the skin of his
torso and extremities. On digital investigation
104. A patient with chronic pancreatitis there are black feces on the glove. Blood test:
complains of diarrhea occurring up to 5 ti- Hb- 108 g/L, thrombocytopenia. Anamnesis
mes per day (no blood traces), loss of body states that similar condition was observed 1
weight, abdominal distention, dryness of skin, year ago. Make the diagnosis:
loss of hair, thirst, bleeding gums, convulsions.
Complete blood count: leukocytes - 5.8 · 109 /L; A. Thrombocytopenic purpura
Hb- 86 g/L; ESR- 15 mm/g; Blood protein B. Hemophilia
test: total protein - 48 g/L; albumins - 28 g/L. C. Bleeding from an ulcer
What indicators of coprological analysis would D. Rectal tumor
accompany this syndrom? E. Nonspecific ulcerative colitis
A. Steatorrhea, creatorrhea 109. A 35-year-old patient developed an epi-
B. Large amount of mucus, amylorrhea leptic attack with tonoclonic spasms that lasted
C. Large amount of starch grains and cellulose for 3 minutes. After the attack the patient
D. Gas bubbles, acid reaction fell asleep but in 5 minutes the second attack
E. Large numbers of iodinophilous microbes occurred. The first step of emergency aid would
be to:
105. A 43-year-old woman complains of persi-
stent stomachache with recurrent pain attacks, A. Ensure patency of airways
nausea, repeated vomiting with stagnant bowel B. Take blood from the vein for analysis
content, abdominal distension, and flatulence. C. Introduce diazepam intravenously
She has been presenting with these signs for D. Prescribe antiepileptic drugs
7 hours. Pulse is 116/min. The tongue is dry E. Administer chloral hydrate via enema
and brown. The abdomen is symmetrically
distemded, soft, painful. Percussion reveals 110. A young woman suffering from seborrhea
tympanitis. On auscultation there are bowel oleosa has numerous light-brown and whi-
sounds with metallic overtone, splashing, and te spots on the skin of her torso and
dripping. Make the diagnosis: shoulders. The spots have clear margins, branny
desquamation, no itching. What provisional di-
A. Acute intestinal obstruction agnosis can be made?
B. Acute necrotizing pancreatitis
C. Acute destructive cholecystitis A. Pityriasis versicolor
D. Acute erosive gastritis B. Torso dermatophytosis
E. Acute nonspecific colitis C. Seborrheic dermatitis
D. Pityriasis rosea
106. After overexposure to cold a 45-year-old E. Vitiligo
woman developed acute pain in her suprapubic
and lumbar areas during urination, sharp pains 111. A patient is 28 years old. He has been
at the end of urination, false urges to urinate. suffering from mental disorder since he was
Urine is turbid with blood streaks. The doctor 22. His current condition has changed acutely:
suspects urinary tract infection. What results of for 3 days the patient has been refusing to
laboratory analysis would be the most indicati- leave his home. He claims that there is a
ve of such infection? ”telepathy” occurring between him and other
people, through which he receives ”thoughts
A. Leukocyturia, gross hematuria of strangers” and transmits his own thoughts
B. Gross hematuria for everyone to hear. He thinks his thoughts
C. Increased blood creatinine and blood urea and actions are manipulated through this
D. Daily proteinuria under 3.0 ”telepathy”. Make the preliminary diagnosis:
E. Daily proteinuria over 3.0
A. Paranoid schizophrenia
107. A 15-year-old adolescent girl came the B. Depressive episode
the gynecologist with complaints of painful C. Catatonic episode
menstruations that are accompanied by nausea, D. Organic delirium
vomiting, and dizziness. Her menarche was at E. Acute reaction to stress
12. Menstruations became painful since she was
14, remain regular. What treatment should be 112. A 3-year-old child presents with sharp
prescribed in this case? deterioration of his general condition. He has
a history of purulent otitis. His temperature is
A. Analgesics, antispasmodics, antiprostaglandi- now 38.5o C . The left leg is pressed to the torso,
ne therapy active movements are absent, the lower thi-
B. Antiinflammatory treatment only rd of the thigh and knee joint are thickened,
C. Antihemorrhagic agents hyperemic, with localized fever. Axial load
D. Antiandrogen therapy leads to acute discomfort of the patient. What
E. Vitamin supplements is the most likely diagnosis?
Krok 2 Medicine (англомовний варiант, iноземнi студенти) 2018 рiк 14

A. Epiphyseal osteomyelitis on the left of the lungs. Heart sounds are weakened, the II
B. Left hip fracture heart sound is accentuated over the pulmonary
C. Rheumatoid arthritis artery. The liver is +3 cm. What complicated the
D. Osteogenic sarcoma clinical course of COPD in this patient?
E. Hygroma of the knee
A. Chronic pulmonary heart
113. A 19-year-old student was urgently hospi- B. Pulmonary embolism
talized due to marked dyspnea and chest pain C. Acute left ventricular failure
on the left. Her body temperature is 38.8o C . D. Diffuse pneumosclerosis
She has been presenting with these signs for E. Community-acquired pneumonia
3 days. Respiratory rate is 42/min., shallow.
Percussion sound is dull to the left from the 117. A burn victim with flame burns of the
center of the scapula, no respiration can be IIIA-B and IV degrees on his face, neck, and
auscultated. The left heart border is displaced anterior surface of the thorax was brought into
outwards by 3 cm. Embryocardia and heart rate the admission room. The hairs in his nostri-
of 110/min are observed. Palpation of the right ls are burnt, his labial and glossal mucosa are
subcostal area is painful. What urgent measures gray-white. The voice is hoarse; respirations are
should be taken in this case? frequent and shallow; the patient has trumpet-
like cough that produces soot-streaked sputum.
A. Urgent thoracocentesis The signs of respiratory failure were progressi-
B. Prescription of penicillin antibiotics ng, while the patient was being transported into
C. Administration of furosemide the intensive care unit. What emergency care
D. Administration of cardiac glycosides must be provided to this patient?
E. Referral into thoracic surgery unit
A. Intubation of the trachea and mechanical
114. A 20-year-old woman, gravida 2, para 1 has ventilation
been in labor for 4 hours. Her condition is sati- B. Tracheostomy
sfactory. Moderately painful contractions occur C. Administration of bronchial spasmolytics
every 3 minutes and last for 35-40 seconds. D. Administration of respiratory analeptics
The waters have not burst yet. The fetus is E. Inhalation of moisturized oxygen
in longitudinal position. Fetal heartbeats are
136/min., clear and rhytmic. Major segment of 118. A 72-year-old man with pnaumonia
the fetal head is engaged to the pelvic inlet. complains of marked dyspnea, chest pain,
Vaginal examination shows smooth cervix of severe cough with expectoration, to is 39.5-
6cm, amniotic sac is intact, sagittal suture is in 40o C , no urination for a whole day. Objecti-
the left oblique diameter, occipital fontanel is vely the patient is conscious. Respiratory rate
on the right near the symphysis pubis. What is 36/min. Over the right lower pulmonary lobe
stage of the labor is it? percussion sound is dull; on auscultation there
is bronchial respiration and numerous moist
A. Active phase of the first stage of normal labor crackles. Blood pressure is 80/60 mm Hg. Heart
B. Latent phase of the first stage of normal labor rate is 120/min. Heart sounds are muffled, there
C. The second stage of normal labor is tachycardia. What tactics should the family
D. Precursors of childbirth doctor choose in the management of this pati-
E. Preliminary stage ent?
115. A 1.5-month-old child on breastfeedi- A. Hospitalization into intensive care unit
ng presents from birth with daily vomiting, B. Outpatient treatment
irregular liquid foamy feces, and flatulence, C. Treatment in the day patient facility
which are resistant to antibacterial and probi- D. Hospitalization into pulmonology unit
otic therapy; no increase of body mass is E. Hospitalization into neurology unit
observed. The child’s condition improved,
when breastmilk was substituted. What 119. Estimation of community health level
pathology is it? involved analysis of a report on diseases regi-
stered among the population of district under
A. Lactase deficiency charge (reporting form 12). What index is
B. Intestinal lambliasis (Giardiasis) calculated based on this report?
C. Infectious enteritis
D. Drug-induced enteritis A. Prevalence
E. Functional dyspepsia B. Index of pathological affection
C. Index of morbidity with temporary disability
116. A 72-year-old man complains of lower D. Index of hospitalized morbidity
extremity edema, sensation of heaviness in the E. Index of basic non-epidemic morbidity
right subcostal area, dyspnea in rest. For over
25 years he has been suffering from COPD. 120. A 72-year-old woman suffers from diabetes
Objectively: orthopnea, jugular venous di- mellitus type II, concomitant diseases are stage
stention, diffuse cyanosis, acrocyanosis. Barrel II hypertension and stage IIB heart failure.
chest is observed, on percussion there is vesi- She takes metformin. Hypertensic crisis had
culotympanitic (bandbox) resonance, sharply occurred the day before, after which the patient
weakened vesicular respiration on both sides, developed extreme weakness, myalgias, thirst,
moist crepitant crackles in the lower segments dry mouth, polyuria. BP is 140/95 mm Hg, heart
rate is 98/min., no edemas or smell of acetone
Krok 2 Medicine (англомовний варiант, iноземнi студенти) 2018 рiк 15

detected. What measures should be taken to complex. BP is 185/105 mm Hg. What additi-
prevent development of comatose state in the onal examination would you recommend to the
patient? patient in the first place?
A. Stop metformin, prescribe short-acting A. Lumbar puncture
insulin B. Ventriculopuncture
B. Double the dosage of metformin C. Echoencephalography
C. Apply hypotonic solution of sodium chloride D. Rheoencephalography
D. Additionally prescribe long-acting insulin E. Electroencephalography
E. Prescribe glibenclamide
125. During regular examination of a 2-year-
121. The body of a 24-year-old woman with old boy, he presents with enlarged left ki-
probable signs of poisoning has been found on dney, painless on palpation. The right ki-
the street. Forensic medical examination was dney was undetectable on palpation. Excretory
requested by an investigator during examinati- urography shows no contrast on the right.
on of the site and the body. According to the Cytoscopy detected hemiatrophy of the urinary
Criminal Procedure Code currently in force in bladder trigone, the right ureteral orifice is not
Ukraine, forensic medical examination is requi- detected. What pathology is it?
red when it is necessary to determine the:
A. Agenesis of the right kidney
A. Cause of death B. Dystopia of the right kidney
B. Manner of death C. Hypoplasia of the right kidney
C. Time of death D. Agenesis of the right ureter
D. Mode of death E. Ectopic right ureteral orifice
E. Mechanism of death
126. A 5-year-old child has body temperature
122. It is the 3rd day after the normal term risen up to febrile numbers, suffers from
labor; the infant is rooming-in with the mother inertness, weakness. Examination revealed
and is on breastfeeding. Objectively: the hemorrhage on the skin of limbs and torso.
mother’s general condition is satisfactory. Enlargement of cervical and axillary lymph
Temperature is 36.4o C , heart rate is 80/min., nodes can be detected. The liver is 4 cm
BP is 120/80 mm Hg. Mammary glands are soft below the costal arch; the spleen is 6 cm
and painless; lactation is moderate, unrestri- below the costal arch. Blood test: erythrocytes
cted milk flow. The uterus is dense, the uterine - 2.3 · 1012 /L, Hb- 60 g/L, platelets - 40 · 109 /L,
fundus is located by 3 fingers width below the leukocytes - 32.8 · 109 /L, eosinophiles - 1%,
navel. Lochia are sanguino-serous, moderate band neutrophiles - 1%, segmented neutrophi-
in volume. Assess the dynamics of uterine les - 12%, lymphocytes - 46%, monocytes - 1%,
involution: blasts - 40%, Duke’s bleeding time test result is
9 min. What examination is necessary to make
A. Physiological involution the diagnosis?
B. Subinvolution
C. Lochiometra A. Myelogram (bone marrow biopsy)
D. Pathologic involution B. Lymph nodes biopsy
E. Hematometra C. Abdominal US
123. A 27-year-old man was hospitalized in D. Detection of hepatitis markers
severe condition 50 minutes after receiving a E. Analysis of dynamic platelet function
penetrating wound to the left side of the chest. 127. A 36-year-old man complains of marked
Objectively the patient is in a stupor, his skin is dyspnea and cardiac pain. He ascribes his di-
pale and acrocyanotic. Pulse is 120/min., of poor sease to the case of influenza that he had 2
volume, weak. Blood pressure is 80/40 mm Hg. weeks ago. Objectively he leans forward when
Heart sounds are muffled, cardiac borders are sitting. The face is swollen, cyanotic, cervical
markedly expanded. In the III intercostal area veins are swollen. Heart borders are extended
along the parasternal line on the left there is on the both sides, heart sounds are muffled,
a stab-incised wound. Plain chest X-ray shows heart rate = Ps = 118/min., BP is 90/60 mm Hg.
enlarged heart shadow with smoothed out wai- Blood test: ESR is 16 mm/hour. ECG shows
st of the heart, there is hemothorax on the left low voltage. X-ray shows trapezoidal cardiac
to the 5th rib. What contributes the most to the silhouette and signs of pulmonary congestion.
severity of the patient’s condition? Choose the treatment tactics:
A. Cardiac tamponade A. Pericardial puncture (pericardiocenthesis)
B. Acute heart failure B. Diuretics
C. Cardiac rhythm disturbance C. Antibiotics
D. Blood loss D. Pericardectomy
E. Hemothorax and acute respiratory failure E. Glucocorticosteroids
124. A 59-year-old patient suffering from 128. A 25-year-old patient is not married and
hypertension was delivered to the hospital has sexual relations with several partners. Duri-
with complaints of acute headache, nausea, ng the last 3 months he noticed small amount of
recurrent vomiting. On examination she mucoserous secretions produced from urethra.
presents with acute meningeal symptom
Krok 2 Medicine (англомовний варiант, iноземнi студенти) 2018 рiк 16

Subjectively: periodical itching or burning pain 132. A 28-year-old woman complains of nausea,
in urethra. Two months ago pain in knee join stomachache, pain in her tongue, and liquid
developed. Possibility of trauma or exposure feces. Three days ago she ate poorly salted
to cold is denied by the patient. During the pike caviar. Objectively her skin is pale, the
last week eye discomfort is noted - lacrimation tongue looks ”lacquered” (bald tongue). Pulse
and itching. What provisional diagnosis can be is 100/min., with muffled heart sounds and
suggested? systolic murmur over the cardiac apex. Blood
pressure is 95/50 mm Hg. The liver is enlarged
A. Reactive arthritis by 3 cm. Hemogram shows anemia, eosinophi-
B. Rheumatoid arthritis ls - 18%. Oval helminth eggs were detected in
C. Seasonal pollinosis feces. Make the provisional diagnosis:
D. Bacterial nonspecific urethral conjunctivitis
E. URTI with conjunctiva and joints affected A. Diphyllobothriasis
B. Trichinosis
129. A woman complains of weight gain, chi- C. Teniasis
lls, edema, xeroderma, somnolence, difficulti- D. Taeniarhynchosis
es with focusing. Objectively: height is 165 E. Ascaridiasis
cm; weight is 90 kg; body proportions are of
female type, to - 35,8o C , heart rate - 58/min., BP- 133. A 30-year-old woman complains of milk
105/60 mm Hg. Heart sounds are weakened, discharge from her breasts and no menstruati-
bradycardia is observed. Other internal organs on for the last 5 months. One physiologic
have no alterations. Thyroid gland cannot be childbirth was 4 years ago. There are no
palpated. Milk secretion from mammary glands maldevelopments of mammary glands. Bi-
is observed. Hormone test revealed increased manual examination revealed diminished
levels of thyroid-stimulating hormone (TSH) uterus and normal sized ovaries. MRI-
and prolactin, and decreased level of thyroxine

scan shows no brain pathologies. Thyroid-
( 4 ). What is the cause of obesity? stimulating hormone is within normal limits.
Serum prolactin is high. What is the most likely
A. Primary hypothyroidism diagnosis?
B. Secondary hypothyroidism
C. Prolactinoma A. Hyperprolactinemia
D. Hypopituitarism B. Hypothyroidism
E. Adiposogenital dystrophy C. Polycystic ovaries
D. Pituitary adenoma
130. A 54-year-old patient complains of E. Sheehan’s syndrome (postpartum hypopitui-
weakness, jaundice, itching skin. Disease onset tarism)
was 1.5 months ago: fever up to 39o C appeared
at first, with progressive jaundice developed 2 134. A 25-year-old woman during self-
weeks later. On hospitalisation jaundice was examination detected a tumor in the upper
severely progressed. Liver cannot be palpated. external quadrant of her right mammary gland.
Gallbladder is enlarged and painless. Blood bi- On palpation: painless, dense, mobile growth 2
lirubin is 190 mcmol/L (accounting mainly for cm in diameter is detected in the mammary
direct bilirubin). Stool is acholic. What is the gland; no changes in the peripheral lymph
most likely reason for jaundice in this patient? nodes are observed. On US of the mammary
glands: in the upper external quadrant of the ri-
A. Mechanical jaundice ght mammary gland there is a space-occupying
B. Hepatocellular jaundice lesion of increased echogenicity 21х18 mm in
C. Hemolytic jaundice size. The most likely diagnosis is:
D. Caroli syndrome
E. Gilbert’s syndrome A. Fibrous adenoma
B. Breast cyst
131. A 23-year-old man came to the surgeon C. Diffuse mastopathy
with complaints of pain, redness of the skin, D. Breast cancer
and swelling in the area of his proximal E. Mastitis
interphalangeal joint of the III finger on the
right hand. Six days ago he pricked his finger 135. A 45-year-old woman underwent one year
with a wire. Objectively the III finger on the ago mastectomy followed by chemo- and radi-
right hand is swollen, hyperemic, prominent in ation therapy. She now complains of dyspnea at
the projection of interphalangeal joint, sharply rest and temperature up to 37.2o C . Her general
painful on touch and during movements. Finger condition is severe, acrocyanosis is observed.
mobility is reduced. Fluctuation sign is present. The right side of her chest practically does not
What diagnosis corresponds to the given clini- participate in respiration. Percussion reveals
cal presentation? a dull sound below the 3rd rib; auscultation
detects acute weakening of the respiratory
A. Articular panaritium sounds. Pleural puncture on the right has yi-
B. Bone panaritium elded a large amount of hemorrhagic exudate.
C. Subcutaneous panaritium What complication has developed in the pati-
D. Pandactylitis ent?
E. Finger furuncle
Krok 2 Medicine (англомовний варiант, iноземнi студенти) 2018 рiк 17

A. Carcinomatous pleuritis A. Nephrotuberculosis


B. Acute pleural empyema B. Right renal cyst
C. Acute right-sided pleuropneumonia C. Right renal carcinoma
D. Right lung abscess D. Acute glomerulonephritis
E. Pulmonary embolism E. Chronic pyelonephritis
136. A 37-year-old man suffers from attacks 140. A 45-year-old woman came to the materni-
of unconsciousness, dyspnea during physical ty clinic with complaints of periodical pains in
exertion, periodical sensations of heart rate di- her mammary glands that start 1 day before
sorder. Father of the patient died suddenly at menstruation and stop after the menstruati-
the age of 45. Objectively: heart rate is 90/min., on begins. Palpation of the mammary glands
BP is 140/90 mm Hg. On heart US: ejection detects diffuse nodes predominantly in the
fraction - 55%, significant myocardium thi- upper outer quadrants. What is the most likely
ckening of the left ventricle and interventri- diagnosis?
cular septum. What drug should be prescribed
for the treatment? A. Fibrocystic mastopathy
B. Breast cancer
A. Bisoprolol C. Mastitis
B. Enalapril D. Hyperprolactinemia
C. Phenyhydinum (Nifedipine) E. Breast cyst
D. Hydrochlorothiazide
E. Furosemide 141. A woman complains of temperature
increase up to 39o C , sharp pains in her lower
137. A 60-year-old man complains of discomfort abdomen, and sanguinopurulent discharge
when swallowing solid food, which he has been from her genital tracts. From her case history it
observing for a month. He changed his diet is known that 6 days ago she underwent illegal
to semiliquid food products. At first the di- abortion. Objectively her blood pressure is
scomfort had abated but later it renewed despi- 100/60 mm Hg, pulse is 110/min. Abdominal ri-
te the change in the diet. The patient developed gidity, rebound tenderness (Bloomberg’s sign),
gaseous eructation and hoarse voice. What and painful palpation of the lower abdomen are
examination should be performed to clarify the observed. On bimanual examination the uterus
diagnosis? is enlarged up to 7 weeks of pregnancy, painful,
and soft; posterior vaginal fornix overhangs.
A. Esophagoscopy with biopsy Make the diagnosis:
B. Urea breath test for H. pylori
C. Diurnal variations of ECG parameters A. Pelviperitonitis
D. Esophageal pH monitoring B. Endometritis
E. Abdominal US C. Acute adnexitis
D. Pyosalpinx
138. A 22-day-old infant developed E. Metroendometritis
subcutaneous red nodes from 1.0 to 1.5 cm in
size on the scalp; later the nodes suppurated. 142. An 8-year-old girl with complaints of pai-
Temperature increased up to 37.7o C , intoxicati- nful urination, frequent low-volume urinati-
on symptoms appeared, regional lymph nodes on, and leukocyturia was diagnosed with acute
enlarged. Complete blood count: anemia, cystitis. 10 days before the disease onset she
leukocytosis, neutrocytosis, increased ESR. was treated by the gynecologist for acute vulvi-
What diagnosis will you make? tis. 5 days ago she presented with mild catarrhal
symptoms. Her mother ascribes the child’s di-
A. Pseudofurunculosis sease to her overexposure to cold. Specify the
B. Pemphigus most likely infection route:
C. Vesiculopustulosis
D. Scalp phlegmon A. Ascending
E. - B. Descending
C. Hematogenic
139. A 32-year-old woman complains of D. Contact
general fatigue, low-grade fever persisting E. Lymphogenic
for 4 months, lumbar pain, and dysuria.
Anamnesis includes frequent acute respi- 143. A 74-year-old patient visited a urologist
ratory diseases, overexposure to cold, low- with complaints of pain above the pubis and
calorie diet, a case of pulmonary tuberculosis inability to urinate for 8 hours. At home he
in childhood. Clinical urine analysis: pH- had taken antispasmodics and had a warm bath
4.8, leukocyturia, hematuria. Complete blood but no improvement occurred. Objectively:
count: leukocytosis, lymphocytosis, increased abdomen is soft and painful above the pubis;
ESR. Urography concludes: dilatation of renal dullness of percussion sound is observed above
pelvis and calyceal system of both kidneys, foci the pubis. Murphy’s (Pasternatski’s) punch sign
of calcification in the projection of right kidney is negative on the both sides. What condition
parenchyma. What is the most likely diagnosis? does the patient have?
Krok 2 Medicine (англомовний варiант, iноземнi студенти) 2018 рiк 18

A. Acute urinary retention A. Pericardiocentesis (pericardial puncture)


B. Paradoxal ischuria B. Cardiac glycosides intravenously
C. Chronic urinary retention C. Constant oxygenotherapy
D. Anuria D. Diuretics intravenously
E. Oliguria E. Antibiotics intravenously
144. An excavator operator with 20 years of 148. A patient has gradually lost consciousness.
work experience at the opencast ore mine The skin is pale and dry. There is smell of
undergoes regular medical examination. He ammonia from the mouth. Respirations are
presents with signs of pneumoconiosis. What deep and noisy. Heart sounds are muffled, peri-
type of pneumoconiosis is the most likely in cardial friction rub is present. Blood pressure
this case? is 180/130 mm Hg. Blood test: Нb- 80 g/L,
leukocytes - 12 · 109 /L, blood glucose - 6.4
A. Siderosis mmol/L, urea - 50 mmol/L, creatinine - 1200
B. Silicosis mcmol/L, blood osmolality - 350 mOsmol/kg
C. Anthracosis H2 O. No urinary excretion. Make the diagnosis:
D. Asbestosis
E. Silicatosis A. Uremic coma
B. Hyperglycemic coma
145. A 55-year-old woman came to a C. Acute renal failure
gynecologist with complaints of leukorrhea and D. Acute disturbance of cerebral circulation
bloody discharge from the vagina after 5 years E. Hyperosmolar coma
of menopause. Anamnesis states no pregnanci-
es. Bimanual examination: the uterus and uteri- 149. A 26-year-old woman has been undergoing
ne appendages are without changes. During treatment for community-acquired pneumonia
diagnostic curettage of the uterine cavity the for 10 days. It is known that her husband had
physician scraped off enchephaloid matter. been treated for drug addiction. Sequenti-
What is the most likely diagnosis in this case? al intravenous administration of Amoksi-
klav (Amoxicillin+Clavunate) + Levofloxacin
A. Endometrial carcinoma combination and vancomycin in the prescribed
B. Adenomyosis dosage was ineffective. Within the last two days
C. Subserous uterine myoma the patient’s dyspnea and intoxication acutely
D. Cervical carcinoma exacerbated, bilateral pulmonary infiltrates are
E. Ovarian carcinoma observed. What is the most likely cause of the
146. A 5-year-old boy complains of severe medication ineffectiveness?
dyspnea and sensation of lack of air. Objecti- A. HIV infection and pneumocystic pneumonia
vely the child assumes orthopneic position, B. Tuberculosis mycobacterium infection with
presents with marked peripheral cyanosis, development of tuberculosis
drumstick fingers, nail plates resembling a C. Idiopathic fibrosing alveolitis
”clock face”, the borders of cardiac dullness are D. Infection with polyresistant bacterial strains
bilaterally extended; coarse systolic murmur E. Cancer metastases in the pulmonary tissues
can be detected over the whole surface of the
heart and is especially pronounced in the II 150. A 35-year-old man complains of persisti-
intercostal area on the left near the sternum. ng enlargement of his peripheral lymph nodes
What disease can be characterized by such that cause him no discomfort. The case history
presentations? states that the first lymph nodes to enlarge
were cervical, supraclavicular, and axillary;
A. Fallot’s tetrad new groups of lymph nodes emerged. Objecti-
B. Dilated cardiomyopathy vely the lymph nodes are soft and elastic on
C. Defect of the interventricular septum palpation, enlarged, painless, not fixed to the
D. Primary bacterial endocarditis surrounding tissue. What examination method
E. Defect of the interatrial septum would be the most informative for early di-
147. A 10-year-old boy is delivered into a agnostics of this disease?
polytrauma unit after he received a blunt A. Needle biopsy
trauma of the thorax, having fallen from the bi- B. Magnetic resonance tomography
cycle. Upon hospitalization his blood pressure C. Radioisotope scanning of the skeleton
is 110/80 mm Hg, heart rate is 96/min. Chest D. Ultrasound
X-ray is noncontributive to the diagnosis. E. X-ray
Echocardiogram shows free liquid in the peri-
cardial cavity, in the amount of up to 100 151. A 32-year-old woman complains of body
ml. In an hour after the hospitalization the weight loss despite her increased appetite,
patient started to develop increasing signs nervousness, and tremor of the extremities.
of heart failure: jugular venous distention, Objectively: the skin is moist; the thyroid gland
decreased blood pressure down to 90/70 mm is diffusely enlarged, painless, soft, and mobi-
Hg, tachycardia up to 120/min. On auscultation le. Blood test: increased level of T3, T4, and
muffled heart sounds. What would be the pri- decreased thyroid-stimulating hormone (TSH).
mary tactics of a physician? What is the most likely diagnosis?
Krok 2 Medicine (англомовний варiант, iноземнi студенти) 2018 рiк 19

A. Diffuse toxic goiter A. Colposcopy, target biopsy of the cervix


B. Thyroid carcinoma B. US of the lesser pelvis
C. Autoimmune (Hashimoto’s) thyroiditis C. Hysteroscopy
D. Thyroid adenoma D. Curettage of the uterine cavity
E. Diffuse nontoxic goiter E. Hormone testing
152. A 64-year-old man suddenly sensed pain in 156. A 26-year-old woman came to a
his occipital area, dizziness, general weakness. gynecologist for a regular check-up. She has
He has a 15-year-long history of hypertension. no complaints. Per vaginum: the uterus lies in
Objectively the skin and mucosa are of normal anteflexion, not enlarged, dense, mobile, pai-
color. Auscultation reveals vesicular respiration nless. On the left from the uterus in the area of
across the lung surface. At the cardiac apex the uterine appendages there is a mobile painless
I heart sound is weakened, the II heart sound outgrowth that can be moved independently
is accentuated over the aorta. Pulse is 84/min., from the uterus. On the right the appendages
blood pressure is 180/100 mm Hg. Other body cannot be detected. What additional investi-
organs and systems are unaffected. What drug gation would be informative for diagnosis clari-
should be prescribed in the first place? fication?
A. Captopril A. Ultrasound of the lesser pelvis
B. Ramipril B. Metrosalpingography
C. Urapidil C. Examination for urogenital infection
D. Perindopril D. Colposcopy
E. Amlodipine E. Colonoscopy
153. During winter epidemics of influenza 157. A 38-year-old man underwent surgical
caused predominantly by virus А/California/04/2009 treatment of a wound with a suppuration
(H1N1), on the 2nd day after the disease onset focus. On the 8th day after the procedure the
a 30-year-old hospitalized man presented with wound cleared of purulo-necrotic discharge
high fever, dry cough, myalgia, headache, and and granulations appeared. However, against
general weakness. What should be prescribed the background of antibacterial therapy, the
as etiotropic treatment in this case? patient’s body temperature persists as high
as 38.5-39.5o C ; chills, excessive sweating, and
A. Neuraminidase inhibitors (Oseltamivir) euphoria are observed in the patient; heart
B. Antibiotics rate is 120/min. What complication of the
C. Immunoglobulin local suppurative inflammatory process can be
D. Interferon inducers suspected?
E. Acyclovir
A. Sepsis
154. A 24-year-old pregnant woman on her B. Purulent-resorptive fever
37th week of pregnancy has been delivered C. Trombophlebitis
to a maternity obstetric service with complai- D. Meningitis
nts of weak fetal movements. Fetal heartbeats E. Pneumonia
are 95/min. On vaginal examination the uterine
cervix is tilted backwards, 2 cm long, external 158. A man complains of high fever, pain in the
orifice allows inserting a fingertip. Biophysical area of his right mastoid bone, and purulent
profile of the fetus equals 4 points. What tactics discharge from the right ear. One week ago
of pregnancy management should be chosen? he had a case of URTI. Objectively the ri-
ght auricle protrudes, the skin behind the ear
A. Urgent delivery via cesarean section is hyperemic and pastose; on palpation of the
B. Treatment of placental dysfunction and mastoid bone the pain intensifies; the auditory
repeated analysis of the fetal biophysical profile meatus is filled with thick pus, posterosuperior
on the next day meatal wall sags; the tympanic membrane is red
C. Doppler measurement of blood velocity in and perforated. Make the diagnosis:
the umbilical artery
D. Urgent preparation of the uterine cervix for A. Acute mastoiditis
delivery B. Furuncle of the external auditory meatus
E. Treatment of fetal distress, if ineffective, then C. Acute otitis media
elective cesarean section on the next day D. Acute otitis externa diffusa
E. Exacerbation of chronic mesotympanitis
155. During regular preventive gynecological
examination a 30-year-old woman was detected 159. A 52 year old man came to see his fami-
to have dark blue punctulated ”perforations” ly physician complaining of pain in the chest.
on the vaginal portion of the uterine cervix. After taking history and performing physical
The doctor suspects endometriosis of the vagi- exam the doctor decided to direct the patient
nal portion of the uterine cervix. What investi- to cardiologist for a consultation. What level of
gation method would be most informative for medical care is being proposed to the patient?
diagnosis confirmation?
Krok 2 Medicine (англомовний варiант, iноземнi студенти) 2018 рiк 20

A. Secondary healthcare Examination revealed her mammary gland


B. Emergency healthcare to be significantly enlarged and deformed;
C. Primary healthcare breast tissue fluctuations and lymphadenitis are
D. Tertiare healtcare observed. What type of mastitis is the most li-
E. Palliative care kely?
160. A 30-year-old woman made an appoi- A. Phlegmonous mastitis
ntment with the family doctor for scheduled B. Serous mastitis
vaccination of her 2-year-old child. What type C. Infiltrative mastitis
of healthcare provides such medical services? D. Lactostasis
E. Mammary edema
A. Primary healthcare
B. Emergency aid 165. When her car collided with a tree, a 37-
C. Secondary healthcare year-old woman felt sharp pain in her left hip
D. Tertiary healthcare joint. She was unable to get out of the car.
E. Palliative care Her position is forced, the hip is pressed to
the abdomen, fixed, and rotated inwards; the li-
161. A 28-year-old man after car accident recei- mb is flexed in the knee, any attempt to change
ved a wound to the right side of his chest infli- the position results in sharp pain. Make the di-
cted by a sharp metal object. A foamy liquid agnosis:
flows out from the wound, there are tympani-
tis and acutely weakened respirations in the A. Closed dislocation of the left hip
right. Blood pressure is 70/30 mm Hg, pulse is B. Contusion of the left hip joint
120/min., Hb is 28 g/L. X-ray shows collapsed C. Hemarthrosis of the left hip joint
right lung, horizontal fluid level is at the 3rd rib. D. Arthritis of the left hip joint
What treatment tactics should be chosen? E. Closed cervical fracture of the left hip
A. Urgent thoracotomy 166. On the 9th day after childbirth the
B. Delayed thoracotomy obstetric patient developed high fever up to
C. Drain the right pleural cavity 38o C . She complains of pain in the right
D. Apply occlusive dressing to the wound mammary gland. The examination revealed the
E. Conservative therapy following: a sharply painful infiltrate can be
palpated in the right mammary gland, the skin
162. A 38-year-old woman has been working over the infiltrate is red, subareolar area and
as a milker for 15 years. She made an appoi- nipple are swollen and painful. What is your
ntment with the doctor due to development of diagnosis?
red rashes on her hands, predominantly in the
interdigital space. The rashes are weeping, itchi- A. Abscess of the right mammary gland
ng, and expanding on her skin. Examination of B. Mastopathy
her hands shows her nail plates to be yellow C. Cancer of the right mammary gland
and brittle. These presentations aggravate duri- D. Serous mastitis
ng work. Make the provisional diagnosis: E. Fibrous cystic degeneration of the right
mammary gland
A. Occupational eczema
B. Scabies 167. During reanimation procedures an
C. Pemphigus attempt to intubate trachea was made. Nei-
D. Pyoderma ther glottis nor epiglottis can be visualized via
E. Dermatophytosis laryngoscopy. What tactics should be chosen in
this case?
163. An 18-year-old patient complains of skin
rash. The patient has been suffering from this A. Insert a laryngeal mask
condition for 5 years. The first instance of this B. Perform a bag-valve-mask ventilation with
disease occurred after a car accident. Objecti- Ambu bag
vely: the patient presents with papular rash C. Continue the attempts until the intubation is
covered in silvery scales, ”thimble” symptom successful
(small pits on the nails), affected joints. What is D. Temporarily cease the chest compressions to
the most likely diagnosis? make intubation easier
E. Discontinue the attempts for mechanical
A. Psoriasis ventilation
B. Panaritium
C. Onychomycosis 168. A 48-year-old woman developed insomnia,
D. Lupus erythematosus depressive mood, anxiety, fears and suicidal
E. Rheumatism thoughts after the death of her husband that
occurred one month ago. During her stay in the
164. A postparturient woman, who has been hospital she speaks in a low voice, is depressed,
breastfeeding for 3 weeks, made an appoi- anxious, avoids sleeping, refuses to eat. What
ntment with the doctor. For the last 6 days medications should be prescribed in this case?
she has been feeling unwell, complains of body
temperature of 38-39o C , general weakness; wi-
thin the last 2 days she developed pain and
redness in the area of her right mammary gland.
Krok 2 Medicine (англомовний варiант, iноземнi студенти) 2018 рiк 21

A. Antidepressants 173. On ultrasound of the thyroid gland, a 47-


B. Antipsychotics year-old woman presents with a hypoechoic
C. Group B vitamins node 1.6 cm in diameter with blurred margi-
D. Nootropics ns and intranodular hypervascularization.
E. Anticonvulsants The doctor suspects thyroid carcinoma. What
method should be used to verify the diagnosis?
169. A 39-year-old man suffers from chronic
adrenal insufficiency and receives replacement A. Fine-needle aspiration biopsy
glucocorticoid therapy (hydrocortisone - 15 B. Thyroid scintigraphy
mg/day). He is to undergo elective surgery C. Case monitoring
for calculous cholecystitis. What medication D. Determine TSH level in the blood
adjustment should be made on the day of the E. Positron emission tomography (PET)
surgery to prevent the development of acute
adrenal insufficiency? 174. A 19-year-old young man complains of
cough with expectoration of purulent sputum
A. Increase the dosage by 2-3 times in amount of 100 ml per day, hemoptysis,
B. Cancel the drug for the day of the surgery dyspnea, increased body temperature up to
C. Add mineralocorticoid 37.8o C , general weakness, weight loss. The
D. Add antibiotic patient’s condition has been persisting for
E. Prescribe large volume intravenous fluid 4 years. Exacerbations occur 2-3 times per
infusion year. The patient presents with malnutriti-
on, pale skin, cyanosis of the lips, drumsti-
170. A 22-year-old man at 18:00 developed ck (clubbed) fingers. Tympanic percussi-
persisting dull pain in the epigastrium. Three on sound in the lungs, weakened respirati-
hours later nausea appeared, he vomited on, various numerous moist crackles in the
once. By the morning the pain shifted to lower pulmonary segments on the left can be
the right iliac area. Body temperature rose observed. In blood: erythrocytes - 3.2 · 1012 /L,
to 38.6o C , developed tachycardia of 110/min.
On examination there are muscle rigidity and leukocytes - 8.4 · 109 /L, ESR- 56 mm/hour. On
X-ray: lung fields are emphysematous, the left
Bloomberg’s sign (rebound tenderness) in the pulmonary root is deformed and dilated. What
right iliac area of the anterior abdominal wall. is the most likely diagnosis?
Plain x-ray of the abdomen shows no fluid
levels, free air under the diapragm on the ri- A. Multiple bronchiectasis of the left lung
ght. Make the diagnosis: B. Chronic left-sided pneumonia
C. Chronic abscess of the left lung
A. Perforation of a gastric ulcer D. Left-sided pulmonary cystic dysplasia
B. Renal colic E. Suppuration of the cyst in the left lung
C. Acute appendicitis
D. Acute pancreatitis 175. An 8-year-old child presents with blood
E. Acute cholecystitis pressure up to 180/100 mm Hg in the upper
limbs accompanied by headaches, tinnitus,
171. 40-50 minutes after the completion of occasional nosebleeds, and high fatigabili-
repair works conducted in a closed garage, ty. On examination there is no pulse over
with car engine running, the repair workers the leg arteries. ECG shows left ventricular
developed severe headache in the temporal hypertrophy. MRI-scan shows aortic narrowi-
area, nausea, tinnitus, vertigo, etc. These ng to 5 mm in the typical place. Coarctation
symptoms are characteristic of acute poisoni- of aorta is diagnosed. What kind of treatment
ng with: should be prescribed in this case?
A. Carbon monoxide A. Surgical
B. Aldehydes B. Conservative
C. Organochlorides C. Physical therapy
D. Hydrogen sulfide D. Case monitoring
E. Fluoride E. Abstain from surgery in favor of complex
172. After a long drive with the window open conservative therapy
a man developed facial asymmetry; he cannot 176. A 6-year-old girl arrived to the in-patient
close his right eye, his right nasolabial fold is unit with complaints of enlargement of the
smoothed out, movements of expression are lower third of her right thigh. According to the
absent on the right, there is a disturbance of case history, she has been stepping carefully
gustatory sensation in the tongue on the ri- on her right leg and limping for 6 months.
ght. No other neurological pathologies were Blood test detected anemia. X-ray of the ri-
detected. What disease can be provisionally di- ght thigh shows a round bone defect with clear
agnosed in this patient? margins resembling melting sugar in the distal
A. Neuropathy of the facial nerve metaphysis. What provisional diagnosis can be
B. Neuropathy of the trigeminal nerve made in this case?
C. Trigeminal ganglionitis
D. Neuropathy of the oculomotor nerve
E. Ischemic stroke
Krok 2 Medicine (англомовний варiант, iноземнi студенти) 2018 рiк 22

A. Osteogenic sarcoma of the right femur


B. Rheumatoid arthritis of the right knee joint A. Scabies
C. Acute hematogenous osteomyelitis of the B. Pyoderma
distal femoral metaphysis on the right C. Dermatitis
D. Tuberculous osteitis of the distal femoral D. Eczema
metaphysis on the right E. Shingles
E. Giant cell tumor of the right femur
181. A 28-year-old man, a teacher, after emoti-
177. A 37-year-old man working as a typesetter onal stress developed painful muscle spasms in
in a print shop complains of rapid fatigabili- his right hand that occur during writing; now
ty, paroxysmal attacks of stomachache, weak he has to hold the pen between the second and
drooping hands. Examination of neurologi- third fingers. He has no problems with typing
cal status revealed hypotrophy of the forearm or writing on the blackboard; no other motor
muscles. Carporadial reflexes are sharply disturbances or neurological pathologies are
weakened. Sensitivity is not disturbed. Gums detected. What is the most likely diagnosis?
present with dark blue border. What neurologi-
cal pathology is it? A. Writer’s cramp
B. Cortical agraphia
A. Lead polyneuropathy C. Parkinsonism
B. Guillain-Barre syndrome (postinfectious D. Neuropathy of the right ulnar nerve
polyneuritis) E. Neuropathy of the right radial nerve
C. Shingles
D. Ulnar neuropathy 182. A 32-year-old pregnant woman at the term
E. Brachial plexitis of 5-6 weeks was vaccinated against influenza
along with her whole family. At that time she
178. The mother of an epileptic son complai- was not aware of her pregnancy. The pregnancy
ns of her son periodically presenting with irri- is wanted. The woman needs an advice from
table depression, when he becomes excited and the family doctor regarding the maintenance of
prone to physical agression and violence. These her pregnancy, namely whether there is a risk of
moods last for 5-10 minutes, after which the fetal malformations because of received vacci-
patient exhausts himself and falls asleep. Upon nation. What advice should the doctor give in
waking he is depressed, sad, cannot recall his this case?
actions or recalls them only partially. What
psychopathologic condition is it? A. Vaccination against influenza is safe during
pregnancy
A. Dysphoria B. Therapeutic abortion is recommended
B. Dysthymia C. Immediate ultrasound of the lesser pelvis is
C. Epileptic delirium necessary
D. Pathologic affect D. Test for antibodies against influenza virus is
E. Ambulatory automatism necessary
E. An infectious diseases specialist must be
179. A 44-year-old woman has undergone consulted
subtotal thyroid resection due to diffuse toxic
goiter. On the second day after the surgery the 183. A 15-year-old girl suffers from systemic
patient’s condition deteriorated; she developed lupus erythematosus and has been receiving
palpitations, dyspnea, sweating, and diarrhea, prednisolone therapy in the daily dosage of 2
and became fearful The patient is anxious, mg/kg for the last 6 weeks. The plans are made
her skin is moist and hot to the touch. Her to gradually lower the dosage of the medicine.
temperature is 39.2o C . Heart sounds are No clinical signs of her disease are observed.
muffled, tachycardia is observed, pulse is Previously she has received no immunization
160/min., blood pressure is 85/40 mm Hg. The against measles. Due to measles outbreak it is
stomach is soft and painless. What should be necessary to develop the immunity against this
measured to clarify the patient’s condition? infection in the patient. When can she be vacci-
nated?
A. Thyroid hormones
B. Blood sugar A. At the present time
C. 17-KS and 17-OCS in urine B. After 2 weeks of prednisolone therapy in the
D. Blood epinephrine and norepineprine dosage lower than 2 mg/kg/24 hours
E. Serum transaminases C. 1 month after the prednisolone therapy is
complete
180. The dermatologist has an appointment wi- D. Immediately after the prednisolone therapy
th a 30-year-old man that complains of severely is complete
itching rashes that especially disturb him at ni- E. Never due to medical contraindications
ght. The rashes developed 2 weeks ago, after
he had returned from a travel. Objectively 184. A 45-year-old veterinary worker has made
on the lateral surfaces of his fingers, hands, an appointment with the doctor for regular
wrists, elbows, lower abdomen, genitals, and examination. In his duties he frequently deals
thighs there are paired papulovesicles, single with animals, however he denies working wi-
pustules, and scratch marks. What disease can th rabies-affected animals. Previously he has
be suspected? received no antirabic vaccination. What should
the doctor recommend in this case?
Krok 2 Medicine (англомовний варiант, iноземнi студенти) 2018 рiк 23

tral heart disease with prevalence of III degree


A. Preventive immunization with antirabic stenosis; heart failure IIA with retained left
vaccine ventricular ejection fraction, functional class
B. Vaccination in case of contact with sick III (NYHA). What tactics of vaccination agai-
animal nst respiratory infections should be chosen to
C. Preventive immunization with rabies provide secondary prevention of exacerbations
immunoglobulin and to avoid heart failure decompensation in
D. Administration of antirabic vaccine and this patient?
rabies immunoglobulin
E. Preventive immunization with anti-rabies A. Scheduled yearly vaccination against
serum influenza and pneumococci
B. Vaccination should be combined with antibi-
185. A healthy child 1 year and 5 months of otic administration
age is being vaccinated against hepatitis B. The C. Vaccination is contraindicated due to severe
child did not receive the first dose of the vacci- heart failure
ne previously, while in the maternity hospital. D. Any vaccination is contraindicated due to
The doctor makes an individual vaccination elderly age of the patient
schedule for this child and plans the admini- E. Any vaccination is contraindicated due to
stration of the next dose of the vaccine. What is mitral valve disease
the minimum interval between doses of vaccine
in this case? 189. A 69-year-old woman was diagnosed wi-
th the following: ischemic heart disease; stable
A. 1 month exertional angina pectoris, FC III; heart fai-
B. 6 months lure IIA with retained left ventricular ejecti-
C. 2 months on fraction, functional class III (NYHA).
D. 3 months What vaccine should be chosen for influenza
E. 12 months prevention and to avoid destabilization of the
186. A 46-year-old man came to the surgeon’s patient’s condition?
office. He complains of twitching sensation A. Type of influenza vaccine is not important
in the wound on his left foot, insomnia, and B. Inactivated influenza vaccine (IIV)
anxiety. According to the patient, he received
this wound 5 days ago, when he accidentally C. Recombinant influenza vaccine (RIV)
stepped on a glass shard, while on the beach. He D. Vaccination is contraindicated due to severe
requested no medical assistance. Objectively heart failure
the patient’s general condition is satisfactory, E. Vaccination is contraindicated due to elderly
pulse is 75/min., blood pressure is 130/80 mm age of the patient
Hg, temperature is 36.9o C . On the plantar 190. A 60-year-old man presents with
surface of his foot there is a wound 1.5 cm subcompensated viral liver cirrhosis (HCV),
long and up to 3 cm deep. The wound edges Child-Pugh class B. What tactics should be
are moderately hyperemic, no discharge from chosen regarding the vaccination against
the wound is observed. What disease can be influenza in this case?
suspected in this patient?
A. Tetanus A. Scheduled yearly vaccination
B. In case of influenza outbreak
B. Diphtheria C. Combined with antiviral drugs
C. Anthrax D. Contraindicated due to disease progression
D. Fasciitis stage, as shown by Child-Pugh class
E. Phlegmon E. Contraindicated due to elderly age of the
187. A 45-year-old woman has been suffering patient
from rheumatoid arthritis for 10 years and takes 191. A 20-year-old student was brought to
methotrexate twice a week. What statement the first-aid center. He has a closed fracture
regarding vaccination against pneumococci of the left forearm and a contused lacerated
(23-valent vaccine) would conform to the wound on his left shin. After the patient recei-
recommendations for the management of ved initial wound management, he presented
rheumatoid arthritis issued by the European the documents confirming that he has recei-
League Against Rheumatism in 2010? ved all the necessary preventive vaccination
A. Vaccination is recommended as scheduled. What should the doctor do to
B. Vaccination is not recommended prevent tetanus in this patient?
C. Vaccination is contraindicated to the patients A. Dynamic case monitoring
who take methotrexate B. Administration of tetanus immunoglobulin
D. Vaccination is contraindicated in cases when C. Administration of anti-tetanus serum
inflammatory process is active D. Antibiotic therapy
E. Vaccination necessitates increase in the E. Administration of tetanus toxoid
dosage of the long-term medicines
192. A 17-year-old girl has made an appoi-
188. A 65-year-old woman was diagnosed with ntment with the doctor. She plans to begin her
the following: chronic rheumatic heart disease, sex life. No signs of gynecological pathology
I degree of rheumatic activity; combined mi-
Krok 2 Medicine (англомовний варiант, iноземнi студенти) 2018 рiк 24

were detected. In the family history there was a sease?


case of cervical cancer that occurred to the pati-
ent’s grandmother. The patient was consulted A. Rubella virus
about the maintenance of her reproductive B. Epstein-Barr virus
health. What recommendation will be the C. Streptococcus
most helpful for prevention of invasive cervi- D. Mumps virus
cal cancer? E. Neisseria meningitis

A. Vaccination against human papillomavirus 197. During administration of planned DPT


(HPV) vaccination the child suddenly developed acute
B. Vitamins, calcium, omega-3 anxiety, signs of pain response, dyspnea, grunti-
C. Immunomodulators ng respirations, cutis marmorata, cold sweat.
D. Antiviral and antibacterial drugs Objectively the child’s consciousness is di-
E. Timely treatment of sexually transmitted sturbed, heart rate is 150/min., blood pressure
diseases is 60/40 mm Hg, heart sounds are muffled. The
child was diagnosed with anaphylactic shock.
193. A 6-month-old infant is not vaccinated. What drug should be administered first?
The physician recommends a DPT (diphtheria,
pertussis, tetanus) vaccination but the mother A. Epinephrine
is absolutely against this procedure. Choose the B. Lasix (Furosemide)
most substantial argument in favor of vaccinati- C. Suprastin (Chloropyramine)
on: D. Euphylline (Aminophylline)
E. Analgin (Metamizole)
A. Risk of lethal consequences
B. Epidemic risk for the others 198. Among first-year schoolchildren there was
C. Personal professional experience a case of measles registered. A 7-year-old boy
D. High quality of vaccines from the same group was not vaccinated against
E. - measles due to refusal of his parents. His clini-
cal history has no cases of measles in the past
194. In autumn a 45-year-old man was and is not contraindicatory to immunobiologi-
recommended an elective surgery for coronary cal agents. Choose the most rational tactics of
artery bypass grafting due to multivessel measles prevention in this schoolboy:
coronary artery disease. The patient has never
received anti-influenza vaccination. Why would A. Measles-Mumps-Rubella vaccine
the family doctor offer a scheduled yearly vacci- B. Isolation for 20 days
nation against influenza to this patient? C. Antiviral agents
D. Antibiotics
A. Decrease the risk of mortality due to E. Immunomodulators
pneumonia and heart failure
B. Secondary prevention of exacerbations of 199. A 40-year-old man developed fever up to
chronic ischemic heart disease 37.5o C and macular rash 10 days after the fi-
C. Immunoprophylaxis of postoperative rst dose of MMR (Measles-Mumps-Rubella)
pulmonary complications vaccine was administered. The vaccination was
D. Primary prevention of influenza during considered necessary as there was a measles
postoperative care outbreak in the city and the patient had not
E. Prevention of seasonal influenza received MMR vaccination in his childhood. Is
revaccination with MMR vaccine possible?
195. A 28-year-old woman has made an
appointment with the family doctor to recei- A. Possible
ve vaccination against influenza. However, B. Forbidden
having collected the patient’s medical hi- C. After a course of glucocorticoids treatment
story, the doctor claimed this procedure to D. Simultaneously with antihistamines
be absolutely contraindicated for this woman. E. Under supervision in the infectious diseases
What anamnestic data is the absolute contrai- inpatient ward
ndication to vaccination?
200. A 26-year-old man is undergoing a regular
A. Egg white intolerance check-up. One year ago he had a case of tonsi-
B. Pregnancy at 30 weeks llar diphtheria complicated with myocarditis.
C. Blood hemoglobin - 109 g/L Presently his condition is satisfactory, no si-
D. Body temperature - 37.2o C gns of cardiovascular failure; ECG shows first-
E. Psoriasis in the remission phase degree atrioventricular block. What vaccine
was administered to this man according to his
196. A 26-year-old man complains of chills, age?
rhinitis, dry cough, and fever up to 38o C .
Examination shows him to be in a moderately A. Adsorbed diphtheria tetanus vaccine (modi-
severe condition; there are small pale pink fied)
non-merging spots on the skin of his back, B. Acellular DPT vaccine
abdomen, and extremities. Palpation reveals C. Tetanus anatoxin
enlarged occipital and axillary lymph nodes. No D. Oral polio vaccine (OPV)
information about vaccination history could be E. BCG vaccine
obtained. What is the likely etiology of this di-
Krok 2 Medicine (англомовний варiант, iноземнi студенти) 2018 рiк 25

Laboratory values
Complete blood count
Laboratory value Normal reference range
Male: 4.3 − 5.9 · 1012 /L
Erythrocyte count
Female: 3.5 − 5.5 · 1012 /L
Male: 135-175 g/L
Hemoglobin, blood
Female: 120-160 g/L
Color index 0.85- 1.05
Reticulocyte count 0.5 - 1.5%
Platelet count 150 − 400 · 109 /L
Leukocyte count 4.0 − 9.0 · 109 /L
Basophils 0 − 0.065 · 109 /L (0-1%)
Eosinophils 0.02 − 0.30 · 109 /L (0.5 - 5.0%)
Band neutrophils 0.04 − 0.30 · 109 /L (1 - 6%)
Segmented neutrophils 2.0 − 5.50 · 109 /L (47 - 72%)
Monocytes 0.09 − 0.60 · 109 /L (3 - 11%)
Lymphocytes 1.2 − 3.0 · 109 /L (19 - 37%)
Erythrocyte sedimentation rate Male: 0 - 15 mm/h
(ESR) Female: 0 - 20 mm/h
Male: 41 - 53%
Hematocrit
Female: 36 - 46%
Biochemical blood analysis (Metabolic panel)
Total proteins 60-78 g/L
Albumin 35-50 g/L (52-65%)
Globulin: 23-35 g/L (35-48%)
α1 -Globulin 2-4 g/L (4.2-7.2%)
α2 -Globulin 5-9 g/L (6.8-12%)
β -Globulin 6-11 g/L (9.3-15%)
γ -Globulin 11-15 g/L (15-19%)
Immunoglobulins:
IgD 0 - 0.15 g/L
IgG 6.5-15 g/L
IgM 0.4-3.45 g/L
IgA 0.76-3.90 g/L
IgE 0-380 kU/L
Bilirubin:
Total 2-17 mcmol/L
Indirect (unconjugated) 2-17 mcmol/L
Direct (conjugated) 0-5 mcmol/L
Triglycerides 0.59-1.77 mmol/L
Total cholesterol 3.9-6.2 mmol/L
Lipoproteins:
high-density lipoproteins (β -Lipoproteins) <4.2 mmol/L
low-density lipoproteins (α-Lipoproteins) 0.8-1.8 mmol/L
Glucose, blood 3.3-6.1 mmol/L (fasting)
Glycated hemoglobin 6%
Iron, blood 9-30 mcmol/L
Potassium, plasma 3.5-5.0 mmol/L
Sodium, plasma 136-145 mmol/L
Calcium, plasma 0.75-2.5 mmol/L
Krok 2 Medicine (англомовний варiант, iноземнi студенти) 2018 рiк 26

Magnesium, plasma 0.75 - 1 mmol/L


Phosphorus (inorganic), serum 0.646-1.292 mmol/L
Chloride, blood 95-105 mmol/L
Urea, serum 3.33 - 8.32 mmol/L
Creatinine 53 - 106 mcmol/L
Male: 0.12-0.38 mcmol/L
Uric acid
Female: 0.12-0.46 mcmol/L
Lactate dehydrogenase (LDH) 45-90 U/L
α-Amylase (diastase), blood 25-125 U/L
Aspartate aminotransferase (AST) 15-40 U/L
Alanine aminotransferase (ALT) 10-40 U/L
Phosphatase (alkaline) 30-115 U/L
Coagulogram
Prothrombin time 80-100% (< 12 seconds)
Fibrinogen 2-4 g/L
Partial thromboplastin time (activated) < 28 seconds
Lee-White coagulation time 5-10 minutes
Bleeding time (Duke method) < 4 minutes
Acid-base balance
pH arterial blood 7.35-7.45
Pco2 arterial blood 33-45 mm Hg
Po2 arterial blood 75 - 105 mm Hg
Bicarbonate, blood (HCO3 ) 22-28 mmol/L
Other blood values
Cortisol, serum 230 -750 nmol/L
Osmolality, serum 275 - 295 mOsmol/kg H2 O
Thyroid-stimulating hormone, serum or plasma 0.5-5 mIU/L
Thyroxine (T4), serum 65-155 nmol/L
Triiodothyronine (T3), serum 1.77 - 2.43 nmol/L
Male: 15-200 mcg/L
Ferritin, serum
Female: 12-150 mcg/L
Thymol turbidity test <5U
C-reactive protein (CRP) <6 mg/dL
Antistreptolysin 0 (ASL0) 250 U
Antistreptohyaluronidase (ASH) 250 U
Urine
Urine specific gravity 1.016-1.022
Nechiporenko’s test:
leukocytes < 2 · 106 /л
erythrocytes < 1 · 106 /л
Proteins, total <45.0 - 75.0 mg/24 h (<0.033 g/L)
Calcium 2.5 - 7.5 mmol/24 h
Male: 97 - 137 mL/min
Creatinine clearance
Female: 88 - 128 mL/min
Oxalate 90 - 445 mcmol/L
Male: 27.7 - 79.7 mcmol/24 h
17-Ketosteroids (17-KS)
Female: 17.4 - 55.4 mcmol/24 h
17-Hydroxycorticosteroids (17-OCS) 5.5 - 27.6 mcmol/24 h
α-Amylase (diastase), urine 28-160 U/L
INSTRUCTIONAL BOOK
Testing Board

TEST ITEMS FOR LICENSING EXAMINATION: KROK 2. MEDICINE.

Kyiv. Testing Board.


(English language).

Approved to print 12.04/№214. Paper size 60х84 1/8


Offset paper. Typeface. Times New Roman Cyr. Offset print.
Conditional print pages 28. Accounting publishing pages 32.
Issue. 2493 copies
List of abbreviations
ACTH Adrenocorticotrophic hormone
ADP Adenosine diphosphate
ALT, ALAT Alanin aminotranspherase
ALV Artificial lung ventilation
AP Arterial (blood) pressure
AST, AspAT Aspartat aminotranspherase
ATP Adenosine triphosphate
ВСG Bacillus Calmette-Guérin
BP Blood (arterial) pressure
CK Creatine kinase
CNS Central nervous system
СоА Coenzyme A
CRP C-reactive protein
CT Computed tomography
COX Cyclooxygenase
DIC Disseminated intravascular coagulation
DTP (DPT) Diphtheria-tetanus-pertussis vaccine
EDTA Ethylenediamine tetra-acetic acid
ELISA Enzyme-linked immunosorbent assay
ENT Ear, nose, and throat (as a department in a hospital)
EPR (ER) Endoplasmic reticulum
ESR Erythrocyte sedimentation rate
EV Enterovirus
FC Functional class
GABA Gamma aminobutyric acid
Hct, Ht Hematocrit
HDL High-density lipoproteins
HR Heart rate
IDL Intermediate-density lipoproteins
IU International unit
IUPAC International Union of Pure and Applied Chemistry
LDH Lactate dehydrogenase
LDL Low-density lipoproteins
LOX Lipoxygenase
MAO Monoamine oxidase
MRI Magnetic resonance imaging
NSAID Nonsteroidal anti-inflammatory drug
PE (PATE) Pulmonary embolism (Pulmonary artery thromboembolism)
PSA Prostate-specific antigen
RBC Red blood count
RR Respiratory rate
SES Sanitary-and-epidemiologic station
STD Sexually transmitted disease
STI Sexually transmitted infection
T/l Trillion/liter
TABT Typhoid-paratyphoid A and B plus tetanus toxoid vaccine
TMJ Temporomandibular joint
TSH Thyroid-stimulating hormone
TU Tuberculin unit
URTI Upper respiratory tract infection
V/f Vision field
VLDL Very-low-density lipoproteins
WBC White blood count
WPW Wolff-Parkinson-White syndrome
1. Human body receives from atmosphere a number of chemicals. What type the of
action results in the combined effect that is less than the sum of isolated effects of these
chemicals on the body?

A. Synergistic action

B. Complex action

C. Isolated action

D. *Antagonism

E. Potentiation

2. A 30-year-old woman came to the gynecological department. She complains of


sharp pain in her lower abdomen and temperature of 38.8°C. She has a history of
extramarital sexual activity and 2 artificial abortions. On gynecological examination
the uterus is unchanged. The appendages are bilaterally enlarged and painful. Profuse
purulent discharge is being produced from the vagina. What examination needs to be
conducted to clarify the diagnosis?

A. *Bacteriological and bacterioscopic analysis

B. Laparoscopy

C. Colposcopy

D. Hysteroscopy

E. Curettage of the uterine cavity

3. A 16-year-old patient has made an appointment with an otolaryngologist. He


complains of elevated body temperature and sore throat. Disease onset was 2 days ago
after the patient ate two portions of ice-cream. Pharyngoscopy shows hyperemic
mucosa of the palatine tonsils, with purulent exudate in the lacunae. Make the
provisional diagnosis

A. Pseudomembranous (Vincent’s) tonsillitis


B. Acute pharyngitis

C. Diphtheria

D. *Lacunar tonsillitis

E. Follicular tonsillitis

4. At the railroad crossing a passenger train collided with a bus. In this collision 26 bus
passengers died, another 18 passengers received mechanical injuries of varying
severity. Where will be professional medical aid provided for the victims of this
accident? Who will provide this aid?

A. At the site of the accident; specialized second-response emergency teams

B. In medico-prophylactic institutions; specialized second-response emergency teams

C. *In medico-prophylactic institutions general physicians and surgeons

D. At the site of the accident; first-response emergency teams

E. In medical institutions: all listed types of healthcare workers

5. A 35-year-old man complains of rapidly increasing fatigue, palpitations, “visual


snow and dizziness. He has a history of peptic ulcer of the stomach. Objectively the
skin is pale. Vesicular respiration is observed in the lungs. Systolic murmur is detected
over the cardiac apex, heart rate is 100/min., BP is 100/70 mm Hg. The epigastrium is
slightly tender on palpation. Blood test: erythrocytes .3.2.1017/L, Hb. 100 g/L, color
index – 0.95 What type of anemia is the most likely present in this case?

A. Hemolytic anemia

B. Sideroblastic anemia

C. Hypoplastic anemia

D. *Posthemorrhagic anemia

E. Chronic iron-deficiency anemia


6. A 63-year-old man complains of unmotivated weakness and pressing and bursting
sensation in the left subcostal area. According to him, these signs have been present
for a year already. Previously he was healthy. He took part in containment measures
during the accident at the Chornobyl Nuclear Power Plant. Objectively the skin is pale,
peripheral lymph nodes are not enlarged, the liver is +3 cm, the spleen is +10 cm.
Complete blood count: erythrocytes – 3.1 . 109/L, Hb- 100 g/L, leukocytes. 46. 10°/L,
blasts -2%. promyelocytes – 10% myelocytes – 18%, band neutrophils – 27%
segmented neutrophils – 10%, lymphocytes – 12%, eosinophils – 6%, basocytes – 3%,
monocytes – 2%, erythrocyte sedimentation rate – 20 mm/hour. What is the most likely
diagnosis?

A. Hepatic cirrhosis

B. Chronic lymphatic leukemia

C. Acute leukemia

D. *Chronic myeloleukemia

E. Hemolytic anemia

7. A 57-year-old patient complains of dyspnea at rest. The patient presents with


orthopnea. acrocyanosis, bulging cervical veins. On percussion: dull sound over the
lower lung segments. On auscultation: no respiratory sounds. Heart rate is 92/min.
Right-sided cardiac dilatation is observed. The liver is +7 cm. Shins are swollen.
Pleural effusion is suspected. What indicator would confirm the presence of transudate
in this case?

A. *Total protein content in the pleural fluid below 25 g/L

B. Positive Rivalta’s test

C. Specific gravity exceeding 1015

D. Total protein content in the pleural fluid exceeding 30 g/L

E. Presence of atypical cells


8. To assess the effectiveness of medical technologies and determine the power and
direction of their effect on the public health indicators, the research was conducted to
study the immunization rate of children and measles incidence rate by district. What
method of statistical analysis should be applied in this case?

A. Calculation of matching factor

B. Calculation of standardized ratio

C Calculation of morbidity Index among the nonvaccinated

D. Calculation of statistical significance of the difference between two estimates

E *Calculation of correlation coefficient

9. A 59-year-old woman was brought into the rheumatology unit. Extremely severe
case of scleroderma is suspected. Objectively she presents with malnourishment,
mask-like lace, and acro-osteolysis. Blood: erythrocytes 22. 10°, erythrocyte
sedimentation rate 40 mm/hour. Urine: elevated levels of free Oxyproline. Name one
of the most likely pathogenetic links in this case:

A. Formation of antibodies to native DNA

B, *Formation of antibodies to collagen

C. Formation of antibodies to transversal striated muscles

D. Formation of antibodies to vessel wall

E. Formation of antibodies to blood corpus

10.A 10-year-old boy was brought to the hospital with complaints of expiratory
dyspnea, respirations are 30/min. He explains his state by a change in the weather
conditions For the last 4 years the boy has been registered for regular check-ups due
to his diagnosis of third degree persistent bronchial asthma. To provide emergency aid
for this child, first he needs to be given:

A. *Salbutamol or short-acting B2-agonists


B. Claritin (Loratadine)

C. Dexamethasone

D. Adrenaline
E. Euphylline (Aminophylline)

11. A newborn has Apgar score of 9. When should this infant be put to the breast?

A. *In the delivery room

B. On the 3rd day

C. On the 2nd day

D. After 2 hours

E. After 12 hours

12. A 55-year-old woman complains of pain and popping sounds in her left knee joint,
which occur when she climbs the stairs. Occasionally during movements her joint
becomes “stuck”. 5 years ago she suffered a trauma of her left knee. Complete blood
count and biochemical blood analysis show normal results. X-ray shows marked
osteosclerosis and osteophytes. The joint space is narrowed. Make the provisional
diagnosis:

A. Reactive arthritis

B. Psoriatic arthritis

C. *Osteoarthritis

D. Gouty arthritis

E. Rheumatoid arthritis

13. A 40-year-old man. a welder, uses manganese electrodes in his line of (18 years 0f
experience). He complains of gait abnormalities, problems with speech and hand
tremors. Objectively the following is observed in the patient hypomimia Increased
muscle tone of plastic type, and quiet monotonous speech, tremor of the tongue, pill-
rolling tremor of the fingers, and retropulsion. What syndrome developed in this
patient due to manganese poisoning?
A. Vestibular syndrome

B. Polyneuritic syndrome

C. Hypothalamic syndrome

D. Meningism

E. *Parkinsonism

14. A 45-year-old man developed constricting retrosternal pain that occurs during
walks at the distance of 200 m. Objectively, heart rate is 80/min., BP is 160/90 mm Hg
During cardiopulmonary exercise test at 50 W there is a depression of S-T segment by
3 mm below the isoline in V3-V4. What is the provisional diagnosis?

A. Exertional angina pectoris, functional class IV

B. *Exertional angina pectoris, functional class III

C. Alcoholic myocardiodystrophy

D. Exertional angina pectoris, functional class II

E. Somatoform autonomic dysfunction, hypertension type

15. A 13-year-old girl for the last two weeks has been complaining of dyspnea and
shin and foot edemas that appear after a physical exertion. In the morning the edemas
significantly decrease. Clinical examination revealed enlarged liver and coarse systolic
murmur over the heart area. Blood tests and urinalysis are without changes. What is
the most likely cause of edemas in this child?

A. Angioneurotic edema

B. Hepatic cirrhosis

C. *Heart failure

D. Acute pyelonephritis

E. Nephrotic syndrome
16. A 14-year-old girl came to the general practitioner with complaints of weakness,
loss of appetite, headache, rapid fatigability. Her last menstruation was profuse and
lasted for 14 days after the previous delay of 2 months. Objectively: the skin is pale,
heart rate is 90/min., BP is 110/70 mm Hg. Hb is 88 g/L. Rectal examination the uterus
and its appendages are without changes, no discharge from the genital tracts. What
complication occurred in the patient?

A. *Posthemorrhagic anemia

B. Dysmenorrhea

C. Somatoform autonomic dysfunction of hypotonic type

D. Migraine

E. Gastritis

17. Heart X-ray of a 31-year-old man has revealed the following with tightly field
opacified esophagus there is a marginal filling defect in its middle third on the posterior
wall; the defect is 1.8×1.3 cm in size with clear oval border. Mucosal folds are retained
and envelop the defect; wall peristalsis and elasticity are not affected. There are no
complaints regarding the condition of the patient’s alimentary canal. Make the
provisional diagnosis

A. Barrett esophagus

B. Diverticulum

C. Esophageal burns

D. *Esophageal tumor

E. Achalasia cardiae

18. For three years a 31-year-old woman has been complaining of pain and swelling
of her radiocarpal and metacarpophalangeal articulations and their reduced mobility in
the morning, which persisted up to 1.5 hours Two weeks ago she developed pain,
swelling, and reddening of her knee joints, her body temperature increased up to
375°C. “The treatment was untimely. Examination of the internal organs revealed no
pathologic alterations. Diagnosis of rheumatoid arthritis was made. What changes are
most likely to be visible on the arthrogram?

A. Joint space narrowing. subchondral osteosclerosis

B. Numerous marginal osteophytes

C. *Joint space narrowing, usuration

D. Cysts in the subchondral bone

E. Epiphyseal osteolysis

19. A 58-year-old man complains of weakness and tumor-like formations that


appeared on the anterior surface of his neck and in the inguinal region. Palpation
detects soft painless mobile cervical and inguinal lymph nodes up to 2 cm in diameter.
The liver protrudes by 2 cm from the edge of the costal margin, the lower splenic pole
is at the umbilical level. In blood: erythrocytes – 3.5 – 10/1, Hb-88 L leukocytes – 86.
10/L, band neutrophils – 1% segmented neutrophils – 10%, lymphocytes – 85%,
eosinophils – 2%, basocytes – 0%, monocytes – 2%, erythrocyte sedimentation rate –
15 mm/hour, Gumprecht shadows. What is the most likely diagnosis?

A. Lymphogranulomatosis

B. Lymphocytic leukemoid reaction

C. *Chronic lymphocytic leukemia

D. Acute leukemia

E. Chronic myeloid leukemia

20. A 20-year-old man, a calibrator of dosimetry equipment committed gross violation


of safety regulations when he put two ampoules with cobalt-60, each with the
Radioactivity of 7 MCi In the pockets of his trousers and jacket. He has been keeping
the ampoules in his pockets for hours. The tissues at the distance of 0.5 cm from the
Source received the done of 30 Gy (3000 R). while the tissues at the distance of 20 cm
2R Did this man develop radiation sickness?

A. Yes, he developed a moderate form of acute radiation syndrome

B. *No, he did not

C. Yes, he developed chronic syndrome radiation

D. Yes, he developed a severe form of acute radiation syndrome

E. Yes, he developed a mild form of acute radiation syndrome

21. A 46-year-old woman has diarrhea with abdominal distension, loss of body mass,
and large amounts of porridge-like foul smelling stool without blood streaks or
tenesmus. Objective examination detects moderate tenderness in the mesogastrium and
left-abdominal flank. Feces analysis detects steatorrhea with neutral fat and
creatorrhea. What prescription would be the most advisable in this case?

A. Antacids and antispasmodics

B. Cholinergic antagonists and antibacterial agents

C. Cholinergic antagonists

D. *Multi-enzyme preparations

E. Metronidazole and loperamide

22. After overexposure to cold a 45-year old woman developed acute pain in her
suprapubic and lumbar areas during urination sharp pains at the end of urination, false
urges to urinate. Urine is turbid with blood streaks. The doctor suspects urinary tract
infection. What results of laboratory analysis would be the most indicative of such
infection?

A. Daily proteinuria under 3.0

B. Increased blood creatinine and blood urea


C. Daily proteinuria over 3.0

D. *Leukocyturia, gross hematuria

E. Gross hematuria

23. A 32-year-old woman complains of body weight loss despite her increased
appetite, nervousness, and tremor of the extremities. Objectively: the skin is moist: the
thyroid gland is diffusely enlarged, painless, soft, and mobile. Blood test: increased
level of T3, T4, and thyroid-stimulating hormone (THS). What is the most likely
diagnosis?

A. *Diffuse toxic goiter

B. Autoimmune (Hashimoto’s) thyroiditis

C. Diffuse nontoxic goiter

D. Thyroid carcinoma

E. Thyroid adenoma

24. A 48-year-old woman has been hospitalized due to the development of


tachysystolic atrial fibrillation. She has lost 5 kg of body weight within 2 months. On
palpation there is a node in the left lobe of the thyroid gland. What pathology resulted
in the development of this condition?

A. Autoimmune thyroiditis

B. Chronic thyroiditis

C. Nontoxic nodular goiter

D. *Toxic nodular goiter

E. Aterosclerotic cardiosclerosis
25. A 19-year-old young man complains of cough with expectoration of purulent
sputum in the amount of 100 mL per day, hemoptysis, dyspnea, increased body
temperature up to 37.8°C, general weakness, weight loss. The patient’s condition lasts
for 4 years. Exacerbations occur 2-3 times a year. The patient presents with
malnutrition. pale skin, cyanosis of the lips, drumstick (clubbed) fingers. Tympanic
percussion sound in the lungs, weakened respiration, numerous various moist crackles
in the lower pulmonary segments on the left can be observed. In blood: erythrocytes –
3.2 . 1012/L, leukocytes – 8.4. 10°/L, ESR-56 mm/hour. On X-ray: lung fields are
emphysematous, the left pulmonary root is deformed and dilated. What is the most
likely diagnosis?

A. Chronic abscess of the left lung

B. *Multiple bronchiectasis of the left lung

C. Left-sided pulmonary cystic dysplasia

D. Chronic left-sided pneumonia

E. Suppuration of the cyst in the left lung

26. A 30-year-old man came to the family physician. 2 months ago he underwent
surgery for an open fracture of the humerus. On examination the patient’s condition is
satisfactory, in the area of the postoperative wound there is a fistula that discharges a
small amount of pus; the area itself is red: fluctuation is detected. X-ray shows
destruction of the humerus with sequestra. What complication did the patient develop
during the postoperative period?

A. Wound suppuration

B. Suture sinus

C. Posttraumatic phlegmon

D. Hematogenous osteomyelitis

E. *Posttraumatic osteomyelitis
27. After eating shrimps, a 25-year-old man suddenly developed skin itching, some
areas of his skin became hyperemic or erupted into vesicles. Make the diagnosis:

A. Psoriasis

B. Urticaria pigmentosa

C. *Acute urticaria

D. Scabies

E. Hemorrhagic vasculitis (Henoch-Schonlein purpura)

28. A woman came to the doctor with complaints of increased body temperature up to
378°C and moderately sore throat for the last 3 days. Objectively: mandibular lymph
nodes are enlarged up to 3 cm. Palatine tonsils are hypertrophied and covered with
gray coating that spreads to the uvula and anterior pillars of the fauces. What is the
most likely diagnosis?

A. Agranulocytosis

B. Infectious mononucleosis

C. Oropharyngeal candidiasis

D. Pseudomembranous (Vincent’s) tonsillitis

E. *Oropharyngeal diphtheria

29. Indicators of work of inpatient departments in the city hospitals for the past year
were analyzed. After that the meeting was held at the central city hospital and, based
on the results of the analysis, a decision was made to decrease the number of beds in
the inpatient departments, and instead open daycare units with partial hospitalization
in the city polyclinics. What is the main goal of this decision?

A. *Rational use of the bed fund

B. Decrease of the length of inpatient stay

C. Optimization of the average bed occupancy rate per year


D. Intensification of the bed fund usage

E. Decreased mortality during inpatient treatment

30. A 30-year-old woman made an appointment with the family doctor for a scheduled
vaccination of her 2-year-old child. What type of healthcare provides such medical
services?

A. Tertiary healthcare

B. *Primary healthcare

C. Secondary healthcare

D. Emergency aid

E. Palliative care

31. A 2-year-old child with persistent cough and subfebrile body temperature after a
case of URTI developed dyspnea, cyanosis of the nasolabial triangle, percussion
dullness and weakened respiration in the lower lobe of the right lung, and a slight
mediastinal displacement to the left. What pulmonary pathology is likely to cause this
clinical presentation?

A. Emphysema

B. Atelectasis

C. *Pleurisy

D. Bronchitis

E. Pneumonia

32. A 30-year-old multigravida has been in labour for 18 hours. 2 hours ago the
pushing stage began. Fetal heart rate is clear, rhythmic, 136/min. Vaginal examination
reveals complete cervical dilatation, the fetal head in the pelvic outlet plane. Sagittal
suture is in line with obstetric conjugate, the occipital fontanel is near the pubís. The
patient has been diagnosed with primary uterine inertia. What are the further tactics of
labor management?

A. Cesarean section

B. Labour stimulation

C. *Outlet forceps

D. Vacuum extraction of the fetus

E. Skin-head Ivanov’s forceps

33. A 40-year-old man with Bekhterev disease (ankylosing spondylitis) complains of


elevated body temperature up to 378°C, back pain and stiffness, especially observed
during the second half of the night. This condition has been lasting for 2 years.
Objectively: reduced spinal mobility, painful sacroiliac joint, erythrocyte
sedimentation rate – 45 mm/hour. X-ray shows narrowing of the intervertebral disc
space and of the sacroiliac joint. What eye pathology is often associated with this type
of disease progression?

A. Cataract

B. *Iridocyclitis

C. Blepharitis

D. Retinal detachment

E. Optic nerve atrophy

34. In the air of the feed kitchen at the poultry factory at the area where formula feed
is being mixed, the dust concentration reaches 200 mg/m”. Air microflora is
represented predominantly by Aspergillus and Mucor fungi. What effect determines
pathogenic properties of the dust?

A. *Allergenic
B. Mutagenic

C. Toxic

D. Teratogenic

E. Fibrogenic

35. A 25-year-old woman was brought into the gynecological department with profuse
bloody discharge from her genital tracts. She is 12 weeks pregnant, the pregnancy is
planned. Within the last 3 days she was experiencing pains in her lower abdomen that
eventually started resembling cramps, she developed bleeding. Her skin is pale, pulse
88/min., blood pressure – 100/60 mm Hg, body temperature – 36.8°C. Vaginal
examination: the uterus size corresponds with 11 weeks of pregnancy, the cervical
canal allows inserting 1 finger and contains fragments of the fertilized ovum, the
discharge is bloody and profuse. What is the most likely diagnosis?

A. *12-week pregnancy, spontaneous abortion in progress

B. Full-term pregnancy, term labor

C. 12-week pregnancy, threatened spontaneous abortion

D. Disturbed menstrual cycle, amenorrhea

E. Disturbed menstrual cycle, hyperpolymenorrhea

36. A 7-year-old boy has severe pulmonary mucoviscidosis (cystic fibrosis). He


complains of dyspnea and blood expectoration. Objectively he presents with lagging
physical development.acrocyanosis, hepatomegaly, drumstick fingers, and nail plates
resembling a “clock face Provisional diagnosis of chronic pulmonary heart disease is
made. What examination would be the most informative for diagnosis confirmation?

A. Chest X-ray

B. *Doppler echocardiography

C. Rheography of the pulmonary artery


D. Ultrasound of the liver

E. Electrocardiography

37. In the factory cafeteria there was an outbreak of food poisoning. Clinical
presentation indicates staphylococcal etiology of this disease. 15 people are sick. To
confirm the diagnosis of food poisoning, samples need to be sent to the laboratory.
What samples should be obtained for analysis?

A. Blood for hemoculture

B. Urine

C. *Vomit masses

D. Saliva

E. Blood (complete blood count)

38. A 47-year-old man developed the signs of decompensated laryngeal stenosis


against the background of acute phlegmonous laryngitis. He presents with inspiratory
dyspnea at rest, forced position, cyanotic skin covered in cold sweat, tachycardia,
deficient pulse, and low blood pressure. What urgent treatment tactics should be
chosen?

A. Oxygen therapy

B. *Tracheostomy

C. Intravenous administration of dehydrating agents

D. Administration of glucocorticoid hormones

E. Oral administration of hyposensitization substances and broncholytics

39. A woman with the pregnancy term of 8 weeks complains of elevated temperature
up to 37.6°C, skin rash that can be characterized as macular exanthema, enlargement
of posterior cervical and occipital lymph nodes, small amount of bloody discharge
from the genital tracts. She was examined by the infectious diseases specialist and
diagnosed with rubella. What tactics should the obstetrician-gynecologist choose?

A. Prescription of antibacterial therapy

B. Treatment of incipient abortion

C. Prescription of hemostatic therapy

D. *Abortion

E. Prescription of antiviral therapy

40. A 20-year-old man was hospitalized on the 9th day of the disease. He attributes his
disease to eating insufficiently thermally processed pork. At its onset this condition
manifested as periorbital edema and fever. Objectively his body temperature is 38.5°C.
The face is puffy and the eyelids are markedly swollen. Palpation of gastrocnemius
muscles is sharply painful. Blood test shows hypereosinophilia. What is the etiology
of this disease?

A. *Trichinella

B. Echinococci

C. Leptospira

D. Ascarididae

E. Trichuris

41. A 34-year-old man complains of pale edema of the face, feet, shins, and lumbar
area. elevated blood pressure up to 160/100 mm Hg. and general weakness. He has a
clinical history of nonspecific ulcerative colitis. Objectively: pulse – 84/min.,
rhythmic, blood pressure – 165/100 mm Hg: edemas all over the body: the skin is pale
and dry with low turgor. The kidneys cannot be palpated, on an attempt to palpate them
they are painless. Blood test: erythrocytes – 3.0 – 1012/L, Hb- 100 L erythrocyte
sedimentation rate – 50 mm/hour. Urinalysis: proteins – 3.5 L, erythrocytes – 7-10 in
the vision field, leukocytes – 5-6 in the vision field. Daily proteinuria – 6 grams. What
analysis should be conducted additionally to verify the diagnosis?

A. *Gingival biopsy for the diagnosis of amyloid disease

B. Renal ultrasound

C. Urinalysis for Bence-Jones protein

D. Radioisotopic examination of kidneys

E. Survey and excretory urography

42. A 26-year-old woman is suspected to suffer from systemic lupus erythematosus


due to systemic lesions of skin, vessels, joints, serous tunics, and heart that developed
after photosensitization. The following is detected: LE cells, antibodies to native DNA,
isolated anti-centromere antibodies, rheumatoid factor is 1:100, Wassermann reaction
is positive, the circulating immune complex is 120 units. What immunological
indicators are considered to be specific to this disease?

A. Immunoglobulin A

B. *DNA antibodies

C. Rheumatoid factor

D. Increased circulating immune complex

E. Anti-centromere antibodies

43. A patient is being treated in the tuberculosis clinic. Throughout the last 3 weeks he
has been suffering from headaches of increasing intensity. Neurological examination
detects nuchal rigidity without focal signs. Make the provisional diagnosis:

A. Chorea minor

B. Brain tumor

C. Myelitis
D. *Tuberculous meningitis

E. Convexital arachnoiditis

44. A 13-year-old girl has 30% of excessive body mass, she started to gain weight at
the age of 3. She has a family history of obesity, Her height and sexual development
are normal for her age. The appetite is excessive. She complains of periodical
headaches. Blood pressure – 120/80 mm Hg. Subcutaneous fat is evenly distributed,
she has no stretch marks. There is juvenile acne on her face. What type of obesity is
it?

A. Hypothalamic obesity

B. *Alimentary constitutive obesity

C. Adrenal obesity

D. Hypothalamic syndrome of puberty

E. Hypothyroid obesity

45. A 45-year-old man, a farmer, presents with acute onset of a disease. He complains
of headache, high temperature, pain in the gastrocnemius muscles, icteric face, and
dark urine. Objectively: body temperature – 38°C. blood pressure – 100/70 mm Hg,
conjunctival hemorrhages, hepatosplenomegaly, and oliguria. What is the most likely
provisional diagnosis?

A. Viral hepatitis

B. Brucellosis

C. Pseudotuberculosis

D. Trichinosis

E. *Leptospirosis
46. During analysis of morbidity in the city, it was determined that age structure of
population is different in each district. What statistical method allows to exclude this
factor, so that it would not skew the morbidity data?

A. *Standardization

B. Dynamic time series analysis

C. Wilcoxon signed-rank test

D. Correlation-regression analysis

E. Analysis of average values

47. A 52-year-old woman has been suffering for 2 years from dull, occasionally
exacerbating pain in her right subcostal area, occurring after eating high-fat foods,
bitter taste in her mouth in the morning. constipations, and flatulence. Objectively she
has excess weight, her body temperature is 36.9°C; there is a coating on the root of her
tongue; the abdomen is moderately distended and painful in the area of gallbladder
projection. What examination would be the most helpful for diagnosis-making?

A. Cholecystography

B. Duodenal intubation

C. Liver scanning

D. Duodenoscopy

E. *Ultrasound

48. A 50-year-old patient was brought to a hospital with complaints of blood in urine.
Urination is painless and undisturbed. Macrohematuria had been observed for 3 days.
Objectively: kidneys cannot be palpated. The suprapubic area is without alterations,
external genitalia are non-pathologic. On rectal investigation: prostate is not enlarged,
painless, has normal structure. Cystoscopy revealed no changes. What is the most
likely diagnosis?

A. Necrotic papillitis
B. *Renal carcinoma

C Dystopic kidney

D. Bladder tuberculosis

E. Varicocele

49. During examination a 4-month-old child with meningococcemia presents with


acrocyanosis, cold extremities, tachypnea, and thready pulse, blood pressure of 30/0
mm Hg. anuria, and sopor. What clinical syndrome is it?

A. *Toxic shock syndrome

B. Encephalic syndrome

C. Exicosis

D. Acute renal failure

E. Neurotoxicosis

50. A district doctor has diagnosed one of his patients with dysentery. What accounting
document reflects this type of morbidity?

A. Certificate of temporary disability

B. *Urgent report

C. Report on a major non-epidemic disease

D. Control card of a patient registered for regular check-ups

E. Statistical report

51. An 18-year-old young man complains of pain in his knee and elbow joints and
body temperature up to 39.5°C. One week and a half earlier he developed a sore throat.
On examination his body temperature is 38.5°C. Swelling of the knee and elbow joints
is observed. Pulse is 106/min., rhythmic. Blood pressure is 90/60 mm Hg. Cardiac
borders are unchanged, heart sounds are weakened, at the cardiac apex there is a soft
systolic murmur. What factor would be the most indicative of the likely disease
etiology?

A. Seromucoid

B. Rheumatoid factor

C. Creatine kinase

D. *Anti-streptolysin O

E. C-reactive protein

52. A 24-year-old pregnant woman on her 37th week of pregnancy has been brought
to the maternity obstetric service with complaints of weak fetal movements. Fetal
heartbeats are 95/min. On vaginal examination the uterine cervix is tilted backwards,
2 cm long, external orifice allows inserting a fingertip. Biophysical profile of the fetus
equals 4 points. What tactics of pregnancy management should be chosen?

A. Urgent preparation of the uterine cervix for delivery

B. *Urgent delivery via a cesarean section

C. Doppler measurement of blood velocity in the umbilical artery

D. Treatment of placental dysfunction and repeated analysis of the fetal biophysical


profile on the next day

E. Treatment of fetal distress: if ineffective, then elective cesarean section on the next
day

53. A 48-year-old woman complains of disturbed menstrual cycle: her periods last for
7.9 days and are excessively profuse throughout the last half-year. She notes
occasional hot flashes in her head, insomnia, irritability, and headaches. Her skin is of
normal color. Blood pressure – 150/90 mm Hg. pulse – 90/min., rhythmic. The
abdomen is soft and painless. Bimanual examination shows no uterine enlargement,
the appendages cannot be detected. The vaginal fornices are free. What is the most
likely diagnosis?

A. Uterine myoma

B. Stein-Leventhal syndrome (polycystic ovary syndrome)

C. Adrenogenital syndrome

D. Premenstrual syndrome

E. *Climacteric syndrome

54. A 38-year-old woman after physical overexertion suddenly developed palpitations,


dyspnea, and a dull pain in the cardiac area. For 10 years she has been registered for
regular check-ups due to rheumatism and mitral valve disease with non-disturbed
blood circulation. Objectively her pulse is 96/min., of unequal strength. Blood pressure
is 110/70 mm Hg, heart rate is 120/min. ECG registers small unevenly-sized waves in
place of P-waves, R R intervals are of unequal length. What is the most likely
diagnosis?

A. Respiratory arrhythmia

B. Paroxysmal supraventricular tachycardia

C. *Atrial fibrillation

D. Paroxysmal ventricular tachycardia

E. Atrial flutter

55. A 38-year-old woman works in flax processing, she dries fax. She came to the
hospital complaining of difficult breathing, constricting sensation in her chest, and
cough attacks. These signs appear on the first day of her working week and gradually
diminish on the following days. What respiratory disease is likely in this case?

A. *Byssinosis

B. Asthmatic bronchitis
C. Bronchial asthma

D. Allergic rhinopharyngitis

E. Silicosis

56. At night a 2-year-old child with upper respiratory tract infection suddenly
developed dyspnea with labored inspiration. Objectively the skin is pale, perioral
cyanosis and slight acrocyanosis are observed. Breathing is loud, respiration rate is
32/min. Jugular, supra- and infraclavicular fossae retract during breathing Respiration
is coarse on auscultation. Heart sounds are clear and sonorous, heart rate is 120/min.
What condition was complicated by the development of the upper respiratory tract
infection?

A. *Stenosing laryngotracheitis

B. Bronchiolitis

C. Bronchial asthma

D. Airway foreign body

E. Obstructive bronchitis

57. A 62-year-old woman was brought into the admission room with complaints of
severe burning retrosternal pain and asphyxia. She has a 10-year-long history of
essential hypertension. Objectively her condition is moderately severe. She presents
with skin pallor, cyanotic lips, and vesicular respiration over her lungs. The II heart
sound is accentuated over the aorta. Blood pressure – 210/120 mm Hg, heart rate
(pulse) – 76/min. ECG shows elevation of ST segment in the leads I. AVL, and V5-
V6. What is the most likely diagnosis?

A. Hypertensive crisis complicated with acute left ventricular failure

B. Hypertensive crisis complicated with unstable angina pectoris

C Uncomplicated hypertensive crisis

D. Pulmonary embolism
E. *Hypertensive crisis complicated with acute myocardial infarction

58. A child is 1 year old. After solid food was introduced into the diet, within the last
several months the child developed loss of appetite, diarrhea with large amount of
feces, and occasional vomiting. Body temperature remains normal. Body weight is 7
kg. The child is very pale, has leg edemas and an extremely distended abdomen. Feces
analysis detects high levels of fatty acids and soups. Diagnosis of celiac disease was
made and gluten-free diet was prescribed. What should be excluded from the diet in
this case?

A. Milk and dairy products

B. Easily digestible carbohydrates

C. *Cereals – wheat, oats

D. Animal protein

E. Fruits

59. A 35-year-old woman complains of high body temperature and pain in the upper
outer quadrant of her right buttock, which developed after an injection. She has been
presenting with this condition for 3 days. At the site of injection the skin is hyperemic,
there is a painful infiltrate with an area of softening in its center. The woman is
diagnosed with a post injection abscess of the right buttock. What tactics should the
surgeon choose in this case?

A. *Abscess incision, sanation and drainage of the cavity

B. Hospitalization, prescription of antibiotics, UHF

C. Antipyretic agents, massage, and application of dry heat to the right buttock

D. Abscess puncture, pus removal followed by application of antiseptics

E. 10-15 minutes of low-intensity laser radiation directed at the right buttock


60. A 57-year-old woman during a regular ultrasound examination presented with a
space-occupying heterogeneous lesion in the right kidney. What is the most
informative method of renal tumor diagnostics?

A. Radioisotope renography

B. *Spiral computed tomography

C. Three glass urine test

D. Excretory urography

E. Retrograde pyelography

61. A 23-year-old man has accidentally swallowed brake fluid. After that he has been
presenting with anuria for 5 days already: his creatinine levels elevated up to 0.569
mmol/L. What treatment tactics should be chosen in this case?

A. Antidotal therapy

B. Diuretics

C. Detoxication therapy

D. Plasmapheresis

E. *Hemodialysis

62. A 35-year-old man suffers from insulin dependent diabetes mellitus and chronic
cholecystitis. He takes NPH insulin: 20 units in the morning and 12 units in the
evening. After a meal he developed pain in the right subcostal area, nausea, vomiting,
sleepiness. and increased polyuria. What prehospital measures will be the most
effective for prevention of crisis within the next several hours?

A. *Change insulin regimen

B. Decrease carbohydrates in the diet

C. Take analgesics
D. Exclude fats from the diet

E. Take cholagogues

63. A 7-year-old boy has been an inpatient for 1.5 months. He had been brought to the
hospital with complaints of edemas all over his body, low urine output, and headache.
Clinical urinalysis: proteins – 71 g/L 1-2 in the vision leukocytes – field, erythrocytes.
3-4 in the vision field. During the course of treatment the edemas gradually dissipated,
headache abated, diuresis normalized. Daily urine proteins – 3 L. Biochemical blood
test: total protein – 43.2 g/L, urea – 5.2 mmol/L, cholesterol – 9.2 mmol/L. What
glomerulonephritis syndrome is the most likely to be present in the patient?

A. Isolated urinary

B. Nephritic

C. *Nephrotic

D. Mixed

E. Hematuric

64. A chronic alcoholic was hospitalized into the therapeutic inpatient unit due to
pneumonia. On the day 5 of his hospitalization he became disoriented in time and
space, developed fear-inducing visual hallucinations and motor agitation. Full body
tremor and tremor of the limbs are observed. X-ray and physical examinations detect
the signs of his convalescence from pneumonia. What tactics should be chosen
regarding this patient?

A. Discharge from the hospital

B. *Transfer into the inpatient narcology department

C. Continue the treatment in the therapeutic department

D. Transfer into the neuro resuscitation department

E Compulsory medical treatment for alcoholism


65. A 34-year-old man on the 3rd day of ceftriaxone treatment for acute otitis (daily
dosage – 2 grams) developed diarrhea occurring 5-6 times per day. Feces are without
mucus or blood admixtures. Temperature is 36.6°C. The Gregersen reaction (occult
blood in feces) is negative. Stool culture detected no pathogenic germs. What is the
most likely cause of diarrhea in this case?

A. *Antibiotic-associated diarrhea

B. Crohn’s disease (regional enteritis)

C. Ulcerative colitis

D. Bacterial overgrowth syndrome

E. Intestinal dysbiosis

66. A 5-year-old child was brought to the ENT department by an ambulance. The child
presents with cough and difficult respiration From the patient’s history it is known that
the child was playing with a toy construction set, when suddenly started coughing and
developed labored breathing Examination detects periodical cough, labored
expiration, and respiratory lag in the left side of the child’s thorax. Auscultation:
diminished respiration on the left. Percussion: tympanitis. X-ray shows a displacement
of the mediastinal organs to the right. Make the diagnosis:

A. A foreign body in the left bronchus, complete bronchostenosis

B. A foreign body in the trachea

C. A foreign body in the right bronchus, valvular bronchostenosis

D. *A foreign body in the left bronchus, valvular bronchostenosis

E. A foreign body in the right bronchus, partial bronchostenosis


67. On laboratory investigation of a pork sample there is 1 dead trichinella detected in
24 sections. This meat should be:

A. *Sent for technical disposal

B. Processed for boiled sausage production

C. Allowed for sale with no restrictions

D. Processed and sold through public catering network

E. Frozen until the temperature of -10°C is reached in the deep layers, with subsequent
exposure to cold for 15 days

68. A 22-day-old infant developed subcutaneous red nodes from 1.0 to 1.5 cm in size
on the scalp: later the nodes suppurated. Temperature increased up to 327°C
intoxication symptoms appeared. regional lymph nodes enlarged. Complete blood
count: anemia, leukocytosis. neutrocytosis, increased ESR. What diagnosis can be
made?

A. –

B. Pemphigus

C. Scalp phlegmon

D. Vesiculo Pustulosis

E. *Pseudo Furunculosis

69. A 9-month-old infant presents with delayed tooth eruption and fontanel closure,
weakness, and excessive sweating. What type of hypovitaminosis is the most likely in
this child?

A. Hypovitaminosis A

B. Hypovitaminosis B1

C. Hypovitaminosis B6
D. *Hypovitaminosis D

E. Hypovitaminosis C

70. A 58-year-old woman came to the gynecological clinic. She complains of bloody
discharge from her genital tracts. Menopause is 8 years. Gynecological examination:
the uterus is slightly enlarged, dense to touch. with limited mobility: the uterine
appendages cannot be detected; parametrium is free. Fractional curettage of the uterine
cavity yields a significant amount of medullary substance in the scrape. What is the
most likely diagnosis?

A. Adenomyosis

B. *Uterine corpus cancer

C. Choriocpithehoma

D. Hormone-producing ovarian tumor

E. Uterine cervix cancer

71. A 72-year-old man diagnosed with ischemic heart disease presents with diffuse
cardiosclerosis, permanent tachysystolic atrial fibrillation, heart failure lla, FCI
Objective examination of vital signs blood pressure is 135/80 mm Hg, heart rate is
160/min., pulse is 125/min. Left ventricular ejection fraction is 32%. What drug is
indicated in this case and should be prescribed to the patient?

A. Verapamil

B. *Digoxin

C. Ivabradine

D. Procainamide (Novocainamide)

E. Isadrine (Isoprenaline)

72. A pregnant woman is 28 years old. Anamnesis: accelerated labor complicated by


the II degree cervical rupture. The following two pregnancies resulted in spontaneous
abortions at the terms of 12 and 14 weeks. On mirror examination: the uterine cervix
is scarred from previous ruptures at 9 and 3 hours, the cervical canal is gaping. On
vaginal examination: the cervix is 2 cm long, the external orifice is open 1 cm wide,
the internal orifice is half-open: the uterus is enlarged to the 12th week of pregnancy,
soft, mobile, painless, the appendages are without changes. What diagnosis can be
made?

A. *Isthmico-cervical insufficiency, habitual non carrying of pregnancy

B. Threatened spontaneous abortion

C. Cervical pregnancy, 12 weeks

D. Cervical hysteromyoma. habitual non carrying of pregnancy

E. Incipient abortion, habitual noncarrying of pregnancy

73. A 40-year-old victim of a traffic accident sustained the following injuries: closed
diaphyseal femur fracture, brain concussion, multiple rib fractures.
hemopneumothorax degloving shin injuries. What injuries ress the most urgent
attention?

A. Brain concussion

B. Degloving shin injuries

C. All injuries are equivalent

D. Closed diaphyseal femur fracture

E. *Multiple rib fractures, hemopneumothorax

74. A 16-year-old girl has primary amenorrhea, no pubic hair growth, normally
developed mammary glands: her genotype is 46 XY, uterus and vagina are absent.
What is your diagnosis?

A. Cushing disease

B. Mayer-Rokitansky-Kuster-Hauser syndrome
C. *Testicular feminization syndrome

D. Sheehan syndrome

E. Cushing syndrome

75. A 10-year-old boy with symptoms of arthritis and myocarditis was brought to a
hospital. Based on clinical examination the provisional diagnosis of juvenile
rheumatoid arthritis was made. What symptom is the most contributive for the
diagnostics of this disease?

A. Affection of the large joints

B. *Reduced mobility of the joints in the morning

C. Enlarged heart

D. Regional hyperemia of the joints

E. Increased heart rate

76. An 11-year-old boy for a month has been presenting with increasing pain in the
right femur. In the painful area there is a nonmobile painful tumor with unclear
margins. The child complains of general indisposition, weakness, increased body
temperature up to 39°C. X-ray shows widened medullary cavity small foci of
cancellous bone destruction, and onion-like lameliar exfoliation of the cortical layer.
What is the most likely pathology resulting in such clinical presentation?

A. Juxtacortical sarcoma

B. Fibrosarcoma

C. *Ewing sarcoma

D. Chondrosarcoma

E. Osteogenic sarcoma
77. At night a 63-year-old woman suddenly developed an asphyxia attack. She has a
15year-long history of essential hypertension and had a myocardial infarction 2 years
ago. Objectively her position in bed is orthopneic, the skin is pale, the patient is
covered with cold sweat, acrocyanosis is observed. Pulse – 104/min. Blood pressure –
210/130 mm Hg, respiration rate – 38/min. Pulmonary percussion sound is clear, with
slight dullness in the lower segments, throughout the lungs single dry crackles can be
heard that become bubbling and non-resonant in the lower segments. What is the most
likely complication in this patient?

A. Paroxysmal tachycardia

B. Pulmonary embolism

C. Acute right ventricular failure

D. Bronchial asthma attack

E. *Acute left ventricular failure

78. A 27-year-old man complains of pain in his leg joints, purulent discharge from the
eyes, and painful burning sensations during urination. Disease onset was acute. He has
a history of influenza. The patient smokes and drinks alcohol in excess. In his line of
work he is often away on business trips. What is the most likely etiological factor of
this disease?

A. Adenovirus

B. *Chlamydia

C. Staphylococci

D. Streptococci

E. Candida

79. The 5-year-old child has been ill for 2 weeks. Cough attacks developed first and
were then followed by reprises. During coughing the child’s face turns red and cervical
veins bulge. The cough attacks induce vomiting, Xray shows intensified bronchial
pattern. Blood test: leukocytes – 16 . 10°/L . lymphocytes • 72%, erythrocyte
sedimentation rate – 4 mm/hour. What is the most likely diagnosis?

A. Adenovirus infection

B. Pneumonia

C. Foreign body

D. *Pertussis

E. Obstructive bronchitis

80. During medical examination a cadet in the naval college was detected to have a
painless dense ulcer 1.5×0.5 in size in his perianal area at the 2 o’clock position. The
ulcer floor resembles “old fat” What is the provisional diagnosis?

A. Rectal fissure

B. Anal cancer

C. Rectal fistula

D. Anal crypt suppuration

E. *Hard syphilitic chancre of the rectum

81. A 53-year-old man complains of general weakness, loss of appetite, and painful
vesicles appearing on his skin. The disease onset occurred suddenly, after
hyperinsolation one week ago. Examination detects isolated vesicles with wrinkled
opercula and occasional painful erosions on the skin of the patient’s torso and limbs.
The Nikolsky sign is positive. What is the most likely diagnosis?

A. Non Acantholytic pemphigus

B. *Acantholytic pemphigus

C. Toxicodermia

D. Duhring’s disease (dermatitis herpetiformis)


E. Herpes

82. A 23-year-old man complains of severe pain in his left knee joint. Objectively the
left knee joint is enlarged with hyperemic skin, painful on palpation. Complete blood
count: erythrocytes – 3.8. 101/L, HD-122 L leukocytes – 74-10P/L. platelets – 183 10
IL Erythrocyte sedimentation rate – 10 mm/hour. Bleeding time (Duke method) – 4
min., Lee White coagulation time. 24 min. Partial thromboplastin time (activated) –
89 seconds Rheumatoid factor – negative. What is the most likely diagnosis?

A. Rheumatoid arthritis

B. Werlhof disease (immune thrombocytopenia)

C. Hemorrhagic vasculitis (Henoch-Schonlein purpura), articular form

D. Thrombocytopathy

E. *Hemophilia, hemarthrosis

83. A 32-year-old woman complains of episodes of intense fear that occur without
visible cause and last for 10-20 minutes, the episodes are characterized by rapid pulse.
sweating, labored breathing, and vertigo. Specify the likely diagnosis:

A. *Panic disorder

B. Simple schizophrenia

C. Paranoid syndrome

D. Claustrophobia

E. Manic syndrome

84. During medical examination of high and middle school students, the doctors vere
assessing correlation between biological and calendar age of the school students based
on the following criteria: height growth rate per year, ossification of the carpal bones,
the number of permanent teeth. What additional development criterion should be
assessed at this age?

A. Vital capacity of lungs

B. Hand strength

C. Chest circumference

D. Body mass

E. *Development of secondary sex characteristics

85. During an outdoor school event in hot weather, a 10-year-old girl lost
consciousness. Body temperature – 36.7°C. Objectively her skin is pale and cold to
touch, her pupils are dilated. Blood pressure – 90/50 mm Hg. Heart rate – 58/min.
What pathology occurred in this case?

A. Sunstroke

B. Paralytic collapse

C. Sympathicotonic collapse

D. –

E. *Syncope

86. A 78-year-old man with a prostate adenoma underwent a herniotomy for a direct
inguinal hernia. After the surgery he presents with absent urination. Enlarged urinary
bladder is detectable above the patient’s pubis. What measures should be taken in this
case?

A. Prescribe proserin (neostigmine) intramuscularly

B. Prescribe antispasmodics subcutaneously

C. Prescribe processing of the postoperative wound with UHF field

D. *Bladder catheterization
E. Apply cold to the urinary bladder area

87. A 27-year-old woman complains of foul smelling discharge from her genital tracts,
pain in her lower abdomen, and elevated temperature. The complaints arose 2 days
ago. She has a history of surgical abortion at the term of 8 weeks one week ago. Mirror
examination: the uterine cervix is clear, external orifice produces foul-smelling
discharge. Vaginal examination: the uterus lies in anteflexion, is mobile, painful, and
slightly enlarged. The appendages are without changes. Make the provisional
diagnosis:

A. Appendicitis

B. Salpingo Oophoritis

C. *Postabortal endometritis

D. Acute respiratory disease

E. Enterocolitis

88. A woman with atopic bronchial asthma was found to have one allergen to dog hair
+++. Carpets were removed from the apartment, the apartment was renovated, and the
air conditioner was installed. However, recurrent asphyxia attacks still occur every
night, despite the patient undergoing pathogenetic therapy. What long-term treatment
tactics can help this patient to decrease her sensitivity to the allergen?

A. Continuation of prior treatment

B. Antihistamine therapy

C. Buteyko breathing technique

D. *Specific hyposensitization

E. Referral for speleotherapy


89. A 45-year-old man with thrombophlebitis of the deep veins in his legs suddenly
after physical exertion developed sharp pain in his thorax on the right, dyspnea, and
hemoptysis. Objectively his condition is severe; he presents with acrocyanosis,
shortening of pulmonary percussion sound on the right, and weakened respiration.
Respiration is 30/min.. blood pressure is 110/80 mm Hg ECG shows sinus tachycardia,
heart rate is 120/min., electric axis of the heart deviates to the right, S-O What is the
most likely diagnosis?

A. Spontaneous pneumothorax

B. Right-sided exudative pleurisy

C. *Pulmonary embolism

D. Cancer of the right lung

E. Community-acquired right-sided pneumonia

90. A 40-year-old man claims that his wife is cheating on him and presents a “proof of
her infidelity. He repeatedly initiated scandals with his wife at home and at work,
demanding that she confess her infidelity, insulted her, and threatened to kill her. What
preventive measures should be taken against socially dangerous actions on his part?

A. Consultation with the general practitioner

B. *Consultation with the psychiatrist

C. Outpatient treatment

D. Consultation with the psychologist

E. Family counseling

91. A 1.5-month-old child on breastfeeding presents from birth with daily vomiting.
irregular liquid foamy feces, and meteorism, which are resistant to antibacterial and
probiotic therapy: no increase of body mass is observed. The child’s condition
improved, when breastmilk was substituted with “NAN low lactose” formula. What
pathology is it?
A. Drug-induced enteritis

B. Infectious enteritis

C. Functional dyspepsia

D. *Lactase deficiency

E. Intestinal lambliasis (Giardiasis)

92. A 37-year-old patient complains of pain in the spinal column, reduced mobility.
The condition persists for 7 years. “Sway back” is observed, there is no movement in
all spinal regions. X-ray shows “bamboo spine” vertebral column. What is the most
likely diagnosis?

A. Spondylolisthesis

B. Tuberculous spondylitis

C. *Ankylosing spondyloarthritis

D. Osteochondrosis

E. Spondylitis deformans

93. After a long drive with the window open a man developed facial asymmetry, he
cannot close his right eye, his right nasolabial fold is smoothed out, movements of
expression are absent on the right, there is a disturbance of gustatory sensation in the
tongue on the right. No other neurological pathologies were detected. What disease
can be provisionally diagnosed in this patient?

A. Trigeminal ganglionitis

B. *Neuropathy of the facial nerve

C. Neuropathy of the trigeminal nerve

D. Neuropathy of the oculomotor nerve

E. Ischemic stroke
94. A man was brought into the admission room after overexposure to cold. He
complains of sharp pain in the small of his back and elevated body temperature up to
38°C. He took some aspirin. Blood test: leukocytes – 10.5 1012/L, eosinophils – 5%.
band neutrophils – 8%, segmented neutrophils 51%, lymphocytes – 32%, monocytes .
4% erythrocyte sedimentation rate – 28 mm/hour Urinalysis: protein – 0.6 L,
leukocytes – cover the whole vision field, large amount of mucus What is the most
likely diagnosis?

A. *Acute pyelonephritis

B. Acute glomerulonephritis

C. Chronic pyelonephritis

D. Subacute malignant glomerulonephritis

E. Tubulointerstitial nephritis

95. A 28-year-old woman complaining of irregular menstruations and infertility came


to the gynecological clinic. Menstruations occur since the age of 15, irregular, with
delays up to 2 months. On examination she presents with marked hirsutism and
excessive body weight. On vaginal examination the uterus is reduced in size and
painless. The ovaries on both sides are dense and enlarged. Ultrasound shows
microcystic changes in the ovaries, the ovaries are 5×4 cm and 4.5×4 cm in size with
dense ovarian capsule. Basal body temperature is monophasic. What is the most likely
diagnosis?

A. Krukenberg tumor

B. Endometrioid cysts

C. Ovarian carcinoma

D. Bilateral adnexitis

E. *Polycystic ovary syndrome


96. A newborn with gestational age of 31 weeks presents with hypotonia and depressed
consciousness. Hematocrit is 35%, general cerebrospinal fluid analysis shows
increased content of erythrocytes and protein, and low glucose. These data correspond
with the clinical presentation of:

A. Sepsis

B. *Intracranial hemorrhage

C. Anemia

D. Meningitis

E. Intrauterine infection

97. A 25-year-old woman has been suffering from diabetes mellitus since she was 9.
She was admitted into the nephrology unit with significant edemas of the face, arms,
and legs. Blood pressure – 200/110 mm Hg. Hb- 90 g/L, blood creatinine – 850
mcmol/L, urine proteins – 10 g/L, leukocytes – 10-15 in the vision field. Glomerular
filtration rate – 10 mL/min. What tactics should the doctor choose

A. Active conservative therapy for diabetic nephropathy

B. Transfer into the endocrinology Clinic

C. *Transfer into the hemodialysis unit

D. Dietotherapy

E. Renal transplantation

98. A 43-year-old man, a coal-face worker with a 15-year-long record of work,


complains of cough, thoracic pain, and dyspnea. The cough is mild, usually dry, and
occurs mostly in the morning. The pain is localized in the interscapular region and
aggravates during deep intake of breath. Dyspnea occurs during physical exertion.
Vesicular respiration in the lungs is weakened. Heart sounds are rhythmic. heart rate
is 86/min., blood pressure is 135/80 mm Hg. The abdomen is soft and painless, Xray
shows micronodular pulmonary fibrosis. Make the provisional diagnosis:
A. Berylliosis

B. Metal pneumoconiosis

C. *Carboconiosis

D. Byssinosis

E. Siderosis

99. A 26-year-old woman presents with amenorrhea. 10 months ago she gave birth for
a second time. In her early postpartum period she developed a massive hypotonic
hemorrhage. No breastfeeding. Lately she has been presenting with loss of weight, loss
of hair, and indisposition. Gynecological examination revealed atrophy of the external
genitals, the uterus is abnormally small, no uterine appendages can be detected. What
is the most likely diagnosis?

A. Galactorrhea-amenorrhea syndrome

B. *Sheehan syndrome (postpartum pituitary gland necrosis)

C. Suspected progressing ectopic pregnancy

D. Stein-Leventhal syndrome (polycystic ovary syndrome)

E. Physiological amenorrhea

100. A 7-year-old girl has been twice treated with antibacterial agents for urinary tract
infection. US shows no severe renal defects. The child presents with recurrence of
leukocyturia and bacteriuria, elevated body temperature up to 38.5°C, and pain in her
left lumbar area. What examination should be conducted first to clarify the cause of
urinary infection recurrence?

A. Retrograde pyelography

B. Radioisotope renography

C. Excretory urography
D. Immunogram

E. *Micturating cystourethrography

101. A 3-month-old child with signs of ricketspresents with positive Chvostek,


Trousseau, and Maslov signs. One day ago the parents witnessed a cyanotic attack in
their child – the child broke into a cold sweat, the eyes bulged, and respiratory arrest
occurred. One minute later the child drew in a loud breath and the child’s condition
normalized again. What is the cause of the described signs of the disease?

A. Increase of blood calcium levels

B. Increase of blood phosphorus levels

C. *Decrease of blood calcium levels

D. Decrease of blood phosphorus levels

E. Metabolic acidosis

102. A 25-year-old man was hospitalized with complaints of pain in his lower
abdomen and right lumbar area that appeared one hour ago. Patient’s general state is
moderately severe. Body temperature – 38.2°C, heart rate – 102/min. The tongue is
dry. The abdomen is painful on deep palpation in the right iliac area and in the Petit
triangle. Aure-Rozanov and Gabay signs are positive. Make the provisional diagnosis:

A. Intestinal obstruction

B. *Acute appendicitis

C. Cecal tumor

D. Acute cholecystitis

E. Right-sided renal colic


103. Clinical statistical investigation was performed to determine effectiveness of a
new pharmacological preparation for patients with ischemic heart disease. What
parametric test (coefficient can be used to estimate the reliability of the results?

A. *Student’s t-distribution

B. Kolmogorov-Smirnov test

C. Matching factor

D. Sign test

E. Wilcoxon signed-rank test

104. A 45-year-old man underwent cardiac surgery one week ago. His general state
has been deteriorating since then: dyspnea at rest, retrosternal pain that irradiates to
the neck, marked weakness. Objectively his body temperature is hectic. His cardiac
borders are expanded. apical beat is weakened. Auscultation detects pericardial friction
rub. What is the most likely diagnosis?

A. Acute myogenic dilatation of the heart

B. Acute cardiac aneurysm

C. *Acute pericarditis

D. Myocardial infarction

E. Pulmonary embolism

105. A surgery unit received a person with an incised stab wound on the upper third of
the right thigh. Examination detects an incised stab wound 3.0×0.5×2.0 cm in size on
the inner surface of the upper third of the right thigh. Bright-red blood flows from deep
within the wound in a pulsing stream. Characterize this type of bleeding:

A. Mixed

B. Capillary
C. Venous

D. Parenchymatous

E. *Arterial

106. A 33-year-old man developed multiple rashes on the skin of his torso and extensor
surfaces of his upper and lower limbs. The rashes itch and occasionally fuse together
and form plaques. The elements of rash are covered with silver-white fine scales that
easily flake off when scratched. Grattage test results in three sequential phenomena:
stearin spot, terminal film, and punctate hemorrhage. What diagnosis can be
suspected?

A. Parapsoriasis

B. Pyoderma

C. Lichen ruber planus

D. Secondary papular syphilid

E. *Psoriasis

107. A 22-year-old postparturient woman on the 12th day after the normal childbirth
informs of elevated body temperature up to 39°C for the last 3 days and pain in her
right mammary gland. The right mammary gland is enlarged, hot to touch, tense,
hyperemic, and painful. Palpation reveals there a dense infiltration 8×8 cm with a
fluctuation in its center. What is the most likely diagnosis?

A. *Postpartum period, day 12. Right-sided infiltrative-purulent mastitis

B. Postpartum period, day 12. Right-sided serous mastitis

C. Postpartum period, day 12. Right-sided gangrenous mastitis

D. Postpartum period, day 12. Right-sided phlegmonous mastitis

E. Postpartum period, day 12. Right-sided lactostasis


108. A 45-year-old woman complains of intolerable paroxysmal facial pain on the left
with attacks that last for 1-2 minutes. Attacks are provoked by chewing. The disease
onset was two months ago after the overexposure to cold. Objectively: pain at the exit
points of the trigeminal nerve on the left. Touching near the wing of the nose on the
left induces a pain attack with tonic spasm of the facial muscles. What is the most
likely diagnosis?

A. Glossopharyngeal neuralgia

B. Maxillary sinusitis

C. *Trigeminal neuralgia

D. Temporomandibular joint arthritis

E. Facial migraine

109. A 15-year-old girl complains of dizziness and sensation of lack of air that she
develops in emotionally straining situations. Relief occurs after she takes corvalol.
Objectively hyperhidrosis and marble-like pattern of the skin of her palms and feet.
Clinical and instrumental examination revealed no organic changes in the central
nervous, cardiovascular, and respiratory systems. What provisional diagnosis can be
made?

A. *Somatoform autonomic dysfunction

B. Acute epiglottitis

C. Stenosing laryngotracheitis

D. Obstructive bronchitis

E. Bronchial asthma

110. A 32-year-old woman complains of tumor-like formation on the anterior surface


of her neck that appeared 2 years ago. Within the last 3 months the tumor has been
rapidly growing. It hinders swallowing and impairs speech; the tumor causes a
sensation of pressure. Objectively the skin moisture is normal, pulse is 80/min.,
rhythmic, blood pressure is 130/80 mm Hg. In the right lobe of the thyroid gland there
is a dense lumpy node 3.0×3.5 cm that moves during swallowing. Scanning image
shows a “cold nodule” in the thyroid gland. Make the provisional diagnosis:

A. Autoimmune thyroiditis

B. Thyroid adenoma

C. Thyroid cyst

D. Nodular goiter

E. *Thyroid cancer

111. A 3-week-old infant developed large, flaccid vesicles with purulent contents on
the skin of chest and abdomen. The vesicles rupture quickly. Make the provisional
diagnosis:

A. Toxic erythema

B. *Pemphigus neonatorum

C. Pseudo Furunculosis

D. Vesiculo Pustulosis

E. Pemphigus syphiliticus

112. Disease onset was acute. A developed general weakness, pain in the joints, and
elevated temperature. Later these signs became accompanied by itching skin rash
manifested as erythematous spots 25 mm in size. The rash gradually turned
hemorrhagic. Large joints are painful and swollen: pain attacks periodically occur in
the periumbilical area; there are signs of intestinal hemorrhage. What is the most likely
diagnosis?

A. Scarlet fever

B. *Hemorrhagic vasculitis (Henoch-Schonlein purpura)

C. Hemorrhagic meningoencephalitis
D. Streptococcal impetigo

E. Rheumatism

113. A 23-year-old woman came to the gynecological clinic. She complains of pain,
itching and burning in her vulva, general weakness, indisposition, elevated body
temperature up to 322°C, and headache. On examination in the vulva there are multiple
vesicles up to 2-3 mm in diameter with clear contents against the background of
hyperemia and mucosal edema. Make the provisional diagnosis:

A. Papillomavirus infection

B. Cytomegalovirus infection

C. Vulvar cancer

D. *Genital herpes infection

E. Primary syphilis

114. A 72-year-old man complains of lower extremity edema, sensation of heaviness


in the right subcostal area, dyspnea at rest. For over 25 years he has been suffering
from COPD. Objectively: orthopnea, jugular venous distention, diffuse cyanosis,
acrocyanosis. Barrel chest is observed, on percussion there is a vesicular tympanitic
(bandbox) resonance, sharply weakened vesicular respiration on both sides, moist
crepitant crackles in the lower segments of the lungs. Heart sounds are weakened, the
II heart sound is accentuated over the pulmonary artery. The liver is +3 cm. What
complicated the clinical course of COPD in this patient?

A. Pulmonary embolism

B. Diffuse pneumosclerosis

C. Acute left ventricular failure

D. Community-acquired pneumonia

E. *Chronic pulmonary heart


115. A woman is 40 weeks pregnant. The fetus is in the longitudinal lie and cephalic
presentation. Pelvic size: 26-29-31-20. Expected weight of the fetus is 4800 gram. The
labor contractions have been lasting for 12 hours, within the last 2 hours they were
extremely painful. The parturient woman is anxious. The waters broke 4 hours ago.
On external examination the contraction ring is located 2 finger widths above the
navel, Henkel-Vasten sign is positive. Fetal heart rate is 160/min., muffled. On internal
examination the uterine cervix is fully open, the head is engaged and pressed to the
entrance into the lesser pelvis. What is the most likely diagnosis?

A. Anatomically contracted pelvis

B. Hyperactive uterine contractions

C. *Threatened uterine rupture

D. Abruption of the normally positioned

E. Complete uterine rupture

116. A 17-year-old girl has made an appointment with the doctor. She plans to begin
her sex life. No signs of gynecological pathology were detected. In the family history
the patient’s grandmother had cervical cancer. The patient was consulted about the
maintenance of her reproductive health. What recommendation will be the most
helpful for prevention of invasive cervical cancer?

A. Timely treatment of sexually transmitted diseases

B. *Vaccination against human papillomavirus (HPV)

C. Antiviral and antibacterial drugs

D. Immunomodulators

E. Vitamins, calcium, omega-3

117. A 52-year-old man for the last 3 years has been suffering from difficult
swallowing of solid food, burning retrosternal pain that aggravated during eating, loss
of body mass, and occasional vomiting with undigested food. Esophageal X-ray shows
S-shaped deformation of the esophagus and its dilation; at the cardiac orifice the
esophagus is constricted: esophageal mucosa is smooth, without signs of peristalsis.
Make the provisional diagnosis

A. Diaphragmatic hernia

B. Reflux esophagitis

C. Esophageal achalasia

D. Esophageal diverticulum

E. *Esophageal carcinoma

118. A 30-year-old man was brought to the neurosurgical department with complaints
of constant headaches, nausea, vomiting, fever, and weakness of the right-side limbs.
Anamnesis states that one month ago the patient had surgery for left-sided suppurative
otitis and mastoiditis. He has been undergoing treatment in an ENT department.
Approximately 2 weeks ago the temperature increased, and the patient developed
headaches. Objectively: heart rate – 98/min., BP-140/90 mm Hg, temperature –
38.3°C. Neurologically manifested stiff neck: bilateral Kernig’s symptom.
unsteadiness during the Romberg’s maneuver. Computer tomography of the brain
revealed a three dimensional growth with a capsule in the left hemisphere. Make the
diagnosis:

A. Arnold-Chiari malformation

B. Hemorrhage

C *Cerebral abscess

D. Hydrocephalus

E. Echinococcus

119. A 65-year-old man complains of dyspnea, severe cough with expectoration of


small amounts of blood-streaked sputum, weight loss, body temperature 37.2°C. loss
of appetite, and weakness. He has been suffering from this condition for years. The
patient’s condition deteriorated one year ago, dyspnea developed 3 weeks ago. The
patient is a lifelong smoker. He is a carpenter by occupation. Objectively he is of
normal body type but emaciated. Right side of the chest is retracted, excursions are
limited, accessory muscles take part in the respiration, respiratory rate is 22 /min
percusion detects dull sound over the upper segment. Chest X-ray shows shrunken
right upper lobe with homogenous shadow connected to the root of the lung: the root
is deformed: mediastinal organs are displaced to the right. What is the most likely
diagnosis?

A. Pulmonary tuberculosis

B. Pulmonary sarcoidosis

C. Complete right-sided pneumothorax

D. Fibrosing alveolitis

E. *Obstructive atelectasis

120. A 20-year-old student after failing an exam developed complaints of a sensation


of a round foreign body in her throat, difficult swallowing. She fixates on her
condition, limits her diet, often cries, seeks attention, exhibits demonstrative attitude.
She is highly susceptible to psychotherapeutic suggestions. What psychiatric diagnosis
can be made in this case?

A. *Hysterical neurosis

B. Depressive neurosis

C. Paranoid personality disorder

D. Hypochondriacal neurosis

E. Obsessive neurosis

121. Having studied the relationship between the distance from villages to the local
outpatient clinics and frequency of visits to the clinics among the rural population of
this area it was determined that the rank correlation coefficient in this case equals -0.9.
How can this relationship be characterized?
A. Moderate inverse relationship

B. –

C. *Strong inverse relationship

D. Moderate direct relationship

E. Strong direct relationship

122. A 39-year-old man suffers from chronic adrenal insufficiency and receives
replacement glucocorticoid therapy (hydrocortisone – 15 mg/day). He is to undergo
elective surgery for calculous cholecystitis. What medication adjustment should be
made on the day of the surgery to prevent the development of acute adrenal
insufficiency?

A. Add an antibiotic

B. *Increase the dosage by 2-3 times

C. Add a mineralocorticoid

D. Prescribe a large volume intravenous fluid infusion

E. Cancel the drug for the day of the surgery

123. After a pain attack in the right subcostal area, a 38-year-old woman with
overnutrition developed icteric skin and sclera, light colored feces, and dark urine. Her
abdomen is distended and painful on palpation in the right subcostal area. Palpation
detects liver enlargement by 2-3 cm. Blood test: total bilirubin – 90 mcmol/l,
conjugated bilirubin60 mcmol/L, What method of examination will be the most
informative for diagnosis clarification?

A. Intravenous cholegraphy

B. *Retrograde cholangiopancreatography

C. Infusion cholegraphy
D. Percutaneous transhepatic cholegraphy

E. US of the hepatopancreatobiliary zone

124. A 65-year-old woman on abdominal palpation presents with a tumor in the


umbilical region and above it; the tumor is 13×8 cm in size, moderately painful,
nonmobile, pulsing. On auscultation systolic murmur can be observed. What is the
most likely diagnosis?

A. Bicuspid insufficiency

B. Tricuspid insufficiency

C. *Abdominal aortic aneurysm

D. Arteriovenous aneurysm

E. Gastric tumor

125. A woman has been provisionally diagnosed with pheochromocytoma. At the


stage of intermission her BP is within norm: there is a tendency towards tachycardia.
No urine pathologies. The decision has been made to perform a provocative test with
histamine. What drug should be kept close at hand for emergency aid in case of a
positive test result?

A. Nifedipine

B. *Phentolamine

C. Prednisolone

D. Pipolphen (Promethazine)

E. Mesaton (Phenylephrine)
126. A middle school teacher with a 4-year long record of work was issued a medical
certificate for pregnancy and childbirth leave. What amount of pay will she receive for
the duration of her leave in this case?

A. 50% of average salary

B. 70% of average salary

C. 60% of average salary

D. 80% of average salary

E. *100% of average salary

127. A 38-year-old woman developed a medical condition 7 days after her return from
Bangladesh. Periodical elevation of temperature was accompanied by chills and
excessive sweating. She was diagnosed with tropical malaria. Next day her condition
further deteriorated: body temperature – 38°C. inertness, periodical loss of
consciousness, generalized seizures, tachycardia. hypotension, and icteric skin. What
complication can be suspected in this case?

A. Serous meningitis

B. Purulent meningitis

C. Acute hepatic failure

D. Acute heart failure

E. *Cerebral coma

128. A 45-year-old man was brought by an ambulance into the emergency hospital.
He complains of sudden pain in the lumbar area frequent painful urination, and
vomiting Examination detects pain in the lumbar area. costovertebral angle tenderness,
pain on palpation of kidneys and along the ureter on the right. Urine test: proteins,
fresh erythrocytes, leukocytes. Make the provisional diagnosis:

A. Polycystic kidney disease


B. Acute glomerulonephritis

C. Acute pyelonephritis

D. *Urolithiasis, renal colic

E. Acute renal failure

129. An 8-year-old girl complains of frequent painful urination in small amounts and
urinary incontinence. The signs have been present for 2 days already. She explains her
disease by overexposure to cold. Costovertebral angle tenderness is absent. Complete
blood count is without pathologies. Urine test: leukocytes – 20-30 in the vision field,
erythrocytes – 40-50 in the vision field, unchanged, bacteriuria. What is the most likely
diagnosis?

A. Glomerulonephritis

B. Vulvitis

C. Pyelonephritis

D. Urolithiasis

E. *Cystitis

130. After a surgery for a left thigh phlegmon the disease progression was complicated
by sepsis. On the 7th day after the surgery there are marked signs of a generalized
inflammatory reaction, in blood there are signs of toxic anemia and progressing
hypoproteinemia, bilirubin levels are 40 mcmol/L, AST and ALT exceed the norm by
2.5 times. Oliguria persists (700 mL of urine per day). Name the phase of sepsis
progression:

A. Mixed phase

B. Stress phase

C. *Catabolic phase

D. Recovery phase

E. Anabolic phase
131. 3 hours after a trauma, a young man developed bradycardia of 46/min., anisocoria
D>s, hemi-hyperreflexia S>D, hemihypesthesia on the left, and a convulsive disorder.
The character of this process needs to be clarified. What method of examination will
be the most accurate for this purpose?

A. Electroencephalography

B. *Brain CT

C. Echoencephalography

D. Skull X-ray

E. Lumbar puncture

132. Having examined a 52-year-old patient, the doctor diagnosed him with obesity
(body mass index – 34 kg/m2. waist circumference – 112 cm) and arterial hypertension
(170/105 mm Hg). 2-hour postprandial blood sugar is 10.8 mmol/L. What biochemical
blood analysis needs to be conducted to diagnose the patient with metabolic syndrome
X?

A. Bilirubin

B. Creatinine and urea

C. Calcium and phosphorus

D. Electrolytes

E. *Lipid profile

133. Mother of a 5-year-old child noticed on the head of her child a round “bald” spot
3 cm in diameter. All the hairs in the focus are broken off at the length of 5-6 mm. The
day before the child was petting a stray cat. Make the diagnosis:

A. Superficial trichophytosis
B. Psoriasis

C. Alopecia areata

D. Deep trichophytosis

E. *Microsporia

134. A 23-year-old man complains of facial edemas, headache, dizziness, low urinary
output, and urine discoloration (dark red). These complaints arose after a case of acute
tonsillitis. On examination there are facial edemas, the skin is pale, temperature is
374°C; heart rate is 86/min., blood pressure is 170/110 mm Hg. Heart sounds are
muffled, the II heart sound is accentuated over the aorta. What etiological factor is the
most likely in this case?

A. Streptococcus pyogenes

B. Streptococcus viridans

C. Staphylococcus aureus –

D. *Beta-hemolytic streptococcus

E. Staphylococcus saprophyticus

135. A 1-year-old child with a case of URTI suddenly developed noisy respirations
with difficult inspiration, intercostal retractions, and barking cough on the 2nd night
after the disease onset. What is the most likely diagnosis?

A. Acute bronchitis

B. *Stenosing laryngotracheobronchitis

C. Acute pulmonary inflammation

D. Bronchial asthma

E. Acute bronchiolitis
136. A 35-year-old patient developed an epileptic attack with tonic clonic spasms that
lasted for 3 minutes. After the attack the patient fell asleep but in 5 minutes the second
attack occurred. The first step of emergency aid would be to:

A. Introduce diazepam intravenously

B. Prescribe antiepileptic drugs

C. *Ensure patency of airways

D. Take blood from the vein for analysis

E. Administer chloral hydrate via an enema

137. A young man has made an appointment with the dermatologist. He complains of
a painful facial rash in the beard and mustache area. This condition has been persisting
for several weeks already. After shaving, the patient’s condition aggravates. The
diagnosis of sycosis is made. What primary morphological elements can be observed
in the rash in this case?

A. Maculae, nodes

B. Pustules, bumps

C. Phlyctenae, maculae

D. Nodes, nodules

E. *Pustules, papulae

138. A 26-year-old man complains of chills, rhinitis, dry cough, and fever up to
Examination shows him to be in a moderately severe condition, there are small pale
pink non-merging spots on the skin of his abdomen, and extremities. Palpation
revealed enlarged occipital and axillary lymph nodes. No information about
vaccination history could be obtained. What is the likely etiology of this disease?

A. Mumps virus

B. Streptococcus
C. Epstein-Barr virus

D. Neisseria meningitis

E. *Rubella virus

139. A 37-year-old man suddenly developed an acute headache accompanied by


nausea. vomiting, and impaired consciousness. Objectively blood pressure is 190/120
mm Hg. The face is hyperemic. Patient’s consciousness is clouded, his answers to the
questions are short, monosyllabic. Movement and sensory disturbances are absent.
Meningeal signs are positive. Cerebrospinal fluid contains blood. What provisional
diagnosis can be made?

A. *Subarachnoid hemorrhage

B. Meningitis

C. Encephalitis

D. Ischemic stroke

E. Cerebral vascular embolism

140. A 45-year-old man diagnosed with acute pulmonary abscess suddenly developed
sharp pain in his chest on the right and dyspnea up to 30/min. Examination detects
facial cyanosis and shallow rapid respiration. Auscultation reveals acutely weakened
respiration throughout the whole right lung percussion reveals a vesicular tympanitic
(bandbox) resonance at the lung apex and dullness in the lower lobe. What
complication developed in this patient?

A. Pleuropneumonia

B. Pneumothorax

C. Acute mediastinitis

D. *Pyopneumothorax

E. Esophageal perforation
141. 2 hours after a traffic accident a 28-year old man in a grave condition was brought
to a hospital. The patient complains of abdominal pain. He received a blow to the
abdomen with the steering wheel. Objective examination revealed the following: the
abdomen does not participate in respiration, is tense and acutely painful on palpation:
the abdominal muscles are defensively tense, peritoneal irritation signs are positive,
hepatic dullness is absent. BP is 90/60 mm Hg, heart rate is 120/min. What further
treatment tactics should be chosen?

A. *Laparotomy

B. Ultrasound investigation

C. Laparocentesis

D. Cold to the abdomen

E. Laparoscopy

142. A 48-year-old woman developed insomnia, depressive mood, anxiety, fears and
suicidal thoughts after the death of her husband that occurred one month ago. During
her stay in the hospital she speaks in a low voice, is depressed, anxious, avoids
sleeping, refuses to eat. What medications should be prescribed in this case?

A. *Antidepressants

B. Group B vitamins

C. Antipsychotics

D. Nootropics

E. Anticonvulsants

143. A woman has been working as a polisher for a year and a half. Her workstation
is equipped with a grinding machine (grinding wheels). She complains of white
discoloration of her fingers and toes that appears when she is nervous. Objectively
there are no changes in the coloration of the distal segments of her limbs. Grip strength
measured with a dynamometer is 25 kg, algesimetry findings are 0.1; 0.3; 0.5. Cold
stimulus is extremely positive on the upper and lower limbs. Internal organs are
without pathologies. Make the diagnosis:

A. Polyneuritis

B. Raynaud disease

C. Raynaud syndrome

D. Syringomyelia

E. *Vibration disease

144. In April during the medical examination of various population groups, 27% of
individuals presented with low working ability and rapid fatigability. The following
symptoms were observed in the affected individuals swollen friable gingiva that bleeds
when pressed, hyperkeratosis follicularis not accompanied by skin dryness. These
symptoms most likely result from the following pathology

A. Poly Hypovitaminosis

B. Parodontosis

C. *C-hypovitaminosis

D. Bi-hypovitaminosis

E. A-hypovitaminosis

145. A 39-year-old man, a battery attendant, suddenly developed weakness, loss of


appetite, non localized colicky abdominal pains, and nausea. Objectively his skin is
gray. there is a pink-gray stripe on his gums, the stomach is soft and sharply painful.
Blood test detected erythrocytes with basophilic stippling and anemia. The patient has
a history of peptic ulcer disease of the stomach. Constipation tends to occur every 3-4
days. What is the most likely provisional diagnosis?

A. Acute appendicitis
B. Acute cholecystitis

C. Perforation of gastric ulcer

D. *Saturnism (lead poisoning)

E. Chronic alcoholism

146. A 62-year-old patient has been hospitalized with complaints of pain in the thorax
on the right during breathing, dyspnea, and dry cough. Ten days ago he slipped and
fell hitting his right side. On examination: the patient lies on the left side. The right
side of the thorax lags during breathing. On the right there is crepitation and pain in
the III-IV ribs. Dullness of percussion sound and sharply diminished breath sounds
can be observed. On X-ray: signs of exudate, fracture of the IIIIV ribs. On
pleurocentesis: blood is detected. Choose the further tactics.

A. *Transfer to a thoracic surgery department

B. Prescribe conservative therapy

C. Perform repeated pleural taps

D. Apply a fixation bandage to the rib cage

E. Refer to a traumatologist

147. A 38-year-old patient has been brought by an ambulance to the surgical


department with complaints of general weakness, indisposition. black stool. On
examination the patient is pale, there are dotted hemorrhages on the skin of his torso
and extremities. On digital investigation there are black feces on the glove. Blood test:
Hb. 108 g/L thrombocytopenia. Anamnesis states that a similar condition was
observed 1 year ago. Make the diagnosis:

A. Hemophilia

B. Ulcerative bleeding

C. *Thrombocytopenic purpura
D. Rectal tumor

E. Nonspecific ulcerative colitis

148. A 10-year-old boy, who was outdoors in Windy and cold weather, developed pain
and tingling in his fingers and toes when he returned home. His parents noticed that
the tips of his fingers and toes were white and their sensitivity was lost. As the affected
areas were warming up the fingers and toes developed tingling and painful sensations.
Skin pallur changed into redness, tingling stopped. Mild itching and swelling of the
fingers appeared. Determine the frostbite degree in this child!

A. Frostbite of the III degree

B. Frostbite of the IV degree

C. Frostbite of the II degree

D. *Frostbite of the I degree

E. Perniosis

149. A 24-year-old woman, a kindergarten teacher, has been sick for 2 days already
Disease onset was acute. She presents with elevated body temperature up to 38.0°C,
pain attacks in her lower left abdomen, liquid stool in small amounts with blood and
mucus admixtures 10 times a day. Pulse – 98/min.. blood pressure – 110/70 mm Hg.
Her tongue is moist and coated with white deposits. The abdomen is soft, the sigmoid
colon is painful and spastic. Make the provisional diagnosis

A. Escherichiosis

B. Salmonellosis

C. *Shigellosis

D. Yersiniosis

E. Rotavirus infection
150. A 36-year-old man complains of marked dyspnea and cardiac pain. He ascribes
his disease to the case of influenza that he had 2 weeks ago. Objectively he leans
forward when sitting. The face is swollen. cyanotic, cervical veins are distended. Heart
borders are extended on both sides, heart sounds are muffled, heart rate = Ps =
118/min., BP is 90/60 mm Hg. Blood test: ESR is 46 mm/hour. ECG shows low
voltage. X-ray shows trapezoidal cardiac silhouette and signs of pulmonary
congestion. Choose the treatment tactics:

A. Diuretics

B. Pericardiectomy

C. *Pericardial puncture (pericardiocentesis)

D. Antibiotics

E. Glucocorticosteroids

151. A 73-year-old woman came to the family physician for one of her regular follow-
up examinations. Three months ago she was found to have type 2 diabetes mellitus.
She was keeping to her diet and exercise plan and taking phytopreparations. On
examination her fasting glucose was within the range of 7.8 – 8.6 mmol/L, HbAlc –
79%. Height – 134 cm, weight – 83 kg. What blood sugar-controlling medicine should
she be prescribed first in the course of her pharmacological therapy?

A. Insulin

B. *Metformin

C. Glimepiride

D. Glibenclamide

E. Gliclazide

152. A newborn girl has an Apgar score of 7-8 points at the 1-5 minutes after birth.
During the labor there was a brief difficulty with extraction of the shoulder girdle.
After birth the baby presents with disturbed function of the proximal segment and
forced position of the right arm. The shoulder is inwards, the elbow is extended, the
forearm pronated, and the whole upper limb resembles the arm of a doll. What is the
most likely clinical diagnosis in this case?

A. Thoracic spine trauma

B. *Erb-Duchenne palsy

C. Intracranial hemorrhage

D. Soft tissue injury of the right arm

E. Osteomyelitis of the right arm

153. 2 hours after eating unknown mushrooms a 28-year-old man sensed a decrease in
his mobility and deterioration of his ability to focus. This condition was then followed
by a state of agitation and aggression. On examination he is disoriented and his speech
is illegible. 4 hours later he developed fetor hepaticus and lost his consciousness. What
syndrome can be observed in this patient?

A. *Acute hepatic failure

B. Cholestatic syndrome

C. Hepatolienal syndrome

D. Cytolytic syndrome

E. Portal hypertension

154. A 45-year-old woman came to the maternity clinic with complaints of periodical
pains in her mammary glands that start 1 day before menstruation and stop after the
menstruation begins. Palpation of the mammary glands detects diffuse nodes
predominantly in the upper outer quadrants What is the most likely diagnosis?

A. Breast cyst

B. *Fibrocystic mastopathy

C. Hyperprolactinemia
D. Mastitis

E. Breast cancer

155. A 32-year-old man complains of pain in his legs that intensifies during walking
was intermittent claudication, numbness of his toes, extremity coldness, and inability
to walk more than 100 meters. When he sleeps his leg usually hangs down. The patient
has been smoking since he was 16. He drinks alcohol in excess. The left leg is colder
than the right one; the skin of the extremities is dry. No pulse can be detected on the
pedal arteries, while pulsation of the femoral arteries is retained. What is the most
likely diagnosis?

A. *Obliterating endarteritis

B. Raynaud disease

C. Deep thrombophlebitis

D. Leriche syndrome (aortoiliac occlusive disease)

E. Diabetic angiopathy

156. A 46-year-old woman came to the maternity clinic with complaints of moderate
blood discharge from the vagina, which developed after the menstruation delay of 1.5
months. On vaginal examination: the cervix is clean; the uterus is not enlarged, mobile,
painless; appendages without changes. Make the diagnosis:

A. Cancer of the uterine body

B. Ectopic pregnancy

C. Adenomyosis

D. *Dysfunctional uterine bleeding

E. Submucous uterine myoma


157. A woman undergoing in-patient treatment for viral hepatitis type B developed a
headache. nausea. recurrent vomiting, memory lapses, flapping tremor of her hands.
and rapid pulse. Sweet smell from her mouth is detected. Body temperature is 376°C
and the heart rate is 89/min. What complication developed in the patient?

A. Ischemic stroke

B. Hypoglycemic shock

C. *Acute liver failure

D. Meningoencephalitis

E. Gastrointestinal hemorrhage

158. An 18-year-old patient always obeys others and adapts his needs to the demands
of the people on whom he depends. He excessively defers to their wishes and makes
them responsible for his well being, cannot defend his interests and needs support from
other people. Such a psychic profile has been formed in childhood, and remains
unchanged. and hinders adaptation. What psychic disorder is observed in this patient?

A. Anxiety (avoidant) personality disorder

B. Anankastic personality disorder

C. Psychopathy-like state

D. *Dependent personality disorder

E. Markedly accentuated personality

159. It is the 3rd day after the first normal term labor; the infant is rooming-in with the
mother and is on breastfeeding. Objectively: the mother’s general condition is
satisfactory Temperature is 36.4°C, heart rate is 80/min., BP is 120/80 mm Hg.
Mammary glands are soft and painless, lactation is moderate, unrestricted milk flow.
The uterus is dense, the uterine fundus is located 3 finger widths below the navel.
Lochia are sanguino-serous. moderate in volume. Assess the dynamics of uterine
involution:
A. Hematometra

B. Subinvolution

C. *Physiological involution

D. Pathologic involution

E. Lochiometra

160. A boy had a foreign body removed from under his nail plate. 3 days later he
developed a sharp throbbing pain at the end of his distal phalanx, which intensifies
when the phalanx is pressed, hyperemia of the nail fold, elevated body temperature up
to 38.5°C, and nail plate discoloration. Make the diagnosis:

A. *Subungual panaritium

B. Paronychia

C. Abscess

D. Erysipeloid

E. Erysipelas

161. A 42-year-old man, a dispatcher, suffers from peptic ulcer disease of the
duodenum. The disease is of moderate severity. He wants to be assigned a disability
group. Make the conclusion regarding his working ability:

A. Capable of working, non-employable

B. Second group of disability

C. Third group of disability

D. First group of disability

E. *Capable of working, employable


162. A 48-year-old woman has arrived to the surgical unit with wounds in her thigh.
On examination the wound surface has dirty-gray coating with unpleasant sweet smell.
Wound content resembles raspberry jelly. Skin tissues around the wound are glossy
and turgid. Palpation reveals moderate crepitation in the tissues. What microflora is
the most likely to cause such inflammation?

A. Blue pus bacillus

B. Streptococci

C. Staphylococci

D. *Anaerobic clostridial

E. Anaerobic non-clostridial

163. A 10-year-old girl exhibits a high level of physical development (M + 30). her
body length increased by 10 cm within a year (which is double the norm for her age
group), the number of permanent teeth corresponds with the age norm (20), the
development of her secondary sex characteristics is three years ahead of her age (Ma,
P, Ax, Menarche). Development rate ahead of her biological age can occur due to:

A. Sports training

B. Acceleration

C. Deficient hygienic education

D. *Endocrine disorders

E. Certain components of her diet

164. In the inpatient gynecological unit within a year 6500 women underwent
treatment. They spent there a total of 102000 bed-days there. What indicator of the
gynecological unit work can be calculated based on this data?

A. Bed turnover rate

B. Average bed occupancy rate per year


C. Number of beds by hospital department

D. Planned bed occupancy rate per year

E. *Average length of inpatient stay

165. A 57-year-old woman complains of weakness, dyspnea, loss of appetite, and


liquid feces. She has been suffering from this condition for 2 years. Objectively she
presents with pale skin, subicteric sclerae, and bright red fissured tongue. Lymph
nodes are not enlarged. Pulse – 100/min. BP- 105/70 mm Hg. Liver +3 cm, the spleen
cannot be palpated. Blood test: erythrocytes – 1.2.102/L, Hb- 56 g/L, color index –
1.4. macrocytes, leukocytes – 2,5 . 10°/L, eosinophils – 1%, juvenile – 1%.
metamyelocytes – 1%, band neutrophils – 8%, segmented neutrophils – 47%,
lymphocytes – 38%, monocytes – 4%, reticulocytes – 0.1% platelets – 10010°/L, ESR-
30 mm/hour, indirect bilirubin – 26 mmol/L. What changes can be expected in the
bone marrow puncture material?

A. *Prevalence of megaloblasts

B. Presence of blast cells

C. Increased number of sideroblasts

D. Prevalence of lymphoid tissue

E. Erythroid hyperplasia

166. The burns unit received a patient, who 6 hours ago during a fire received flame
burns. On the patient’s body there is a gray brown area of necrosis that covers 3/4 of
the body perimeter. Occasionally there are small blisters with hemorrhagic contents
and patches of shredded epidermis. What local therapy is necessary in this case?

A. Chemical necrolysis

B. *Decompression necrectomy

C Necrectomy with dermal autograft

D. Necrectomy with xenotransplantation


E. Blister puncture

167. A 45-year-old woman is registered for regular check-ups due to Werlhof disease
(immune thrombocytopenia). Complete blood count: Hb- 100 g/L. erythrocytes – 2.8.
109/L. platelets – 90.0 . 10°/L, leukocytes – 8.4 -10°/L. erythrocyte sedimentation rate
– 13 mm/hour. Examination detects a single small hematoma on the anterior surface
of the thigh, developed after the patient accidentally stumbled on a table. What
treatment tactics should be chosen in this case?

A. Administer thrombocytic mass, continue the treatment in the hematology unit

B. *Continue the supervision by the hospital hematologist

C. Urgent hospitalization into the general care unit

D. Urgently start a hemostatic therapy followed by a planned hospitalization into the


hematology unit

E. Urgent hospitalization into the hematology unit

168. In the process of hiring, a prospective employee has undergone preventive


medical examination and was declared fit to work in this manufacturing environment.
What type of preventive medical examination was it?

A. Scheduled

B. Periodical

C. Comprehensive

D. Specific

E. *Preliminary

169. A 27-year-old woman, a teacher in the elementary school, complains of frequent


stools, up to 3 times per day, with lumpy feces and large amount of mucus, abdominal
pain that gradually abates after a defecation. irritability. Her skin is pale and icteric.
Pulse IS 74/min, rhythmic, can be characterized as satisfactory. Blood pressure is
115/70 mm Hg. The abdomen is soft, moderately tender along the colon on palpation.
Fiberoptic colonoscopy detects no changes. What disease can be suspected?

A. Whipple disease

B. Chronic enteritis

C. Crohn disease (regional enteritis)

D. Chronic non-ulcerative colitis

E. *Irritable bowel syndrome

170. A 6-month-old child on breastfeeding is hospitalized in the inpatient department.


After the child recovers, the doctor recommends the mother to start introducing solid
food to the child’s diet. What products should be introduced to the child’s diet first?

A. Buckwheat porridge

B. Semolina porridge

C. Fermented dairy products

D. Grated apple

E. *Vegetable puree

171. 10 hours after birth a child developed jaundice, hypotonia, hyporeflexia, and
moderate hepatosplenomegaly. Feces and urine are of normal color. Umbilical cord
blood bilirubin is 51 mcmol/L due to unconjugated bilirubin levels. In venous blood:
erythrocytes – 3.5 . 1012/L, Hb- 140 g/L. reticulocytes – 1.5%, bilirubin – 111
mcmol/L, conjugated – 11 mcmol/L, ALT-40 U/L, AST- 30 U/L. Mother’s blood
group is A(II) Rh(-), child’s blood group is A(II) Rh(+). What laboratory test can
confirm the diagnosis?

A. Viral hepatitis markers analysis

B. *Coombs test
C. Erythrocytometry

D. Measurement of erythrocyte osmotic resistance

E. Measurement of glucose 6-phosphate dehydrogenase levels in erythrocytes

172. The body of a 24-year-old woman with suspected poisoning has been found on
the street. Forensic medical examination was requested by an investigator during
examination of the site and the body. According to the Criminal Procedure Code
currently in force in Ukraine, forensic medical examination is required when it is
necessary to determine the:

A. *Cause of death

B. Mode of death

C. Manner of death

D. Mechanism of death

E. Time of death

173. During regular preventive gynecological examination a 30-year-old woman was


detected to have dark blue punctulated “perforations on the vaginal portion of the
uterine cervix. The doctor suspects endometriosis of the vaginal portion of the uterine
cervix. What investigation method would be most informative for diagnosis
confirmation?

A. Hormone testing

B. *Colposcopy, target biopsy of the cervix

C. US of the lesser pelvis

D. Hysteroscopy

E. Curettage of the uterine cavity


174. A 72-year-old man with pneumonia complains of marked dyspnea, chest pain,
severe cough with expectoration, is 39.5-40°C, no urination for a whole day.
Objectively the patient is conscious. Respiratory rate is 36/min. Over the right lower
pulmonary lobe percussion sound is dull: on auscultation there is bronchial respiration
and numerous moist crackles. Blood pressure is 80/60 mm Hg. Heart rate is 120/min.
Heart sounds are muffled, there is tachycardia. What tactics should the family doctor
choose in the management of this patient?

A. Treatment in the day patient facility

B. *Hospitalization into the intensive care unit

C. Hospitalization into the pulmonology unit

D. Outpatient treatment

E. Hospitalization into the neurology unit

175. A parturient woman is 30 years old, Stage 1 of the labor is ongoing. The fetus is
in the cephalic presentation. Auscultation of the fetal heart sounds detects bradycardia.
Evaluation of cardiotocogram yielded the following data: decrease of basal heart rate
down to 90/min., variability – monotonous (2 and less): late decelerations with
amplitude of 50/min. Make the diagnosis and choose the obstetrical tactics necessary
in this case:

A. Fetal distress. Stimulation of uterine contractions

B. Fetal distress. Vacuum extraction delivery

C. *Fetal distress. Urgent cesarean section delivery

D. Fetal distress. Forceps delivery

E. Normal condition of the fetus. Vaginal birth

176. A 42-year-old man, a worker at the meat processing factory, developed an itching
spot on his lower jaw, which gradually transformed into a slightly painful carbuncle 3
cm in diameter, surrounded by a painless swelling that reaches the clavicle.
Temperature is subfebrile, under 37.8°C. The doctor suspects anthrax. What drug
should this man be prescribed for treatment?

A. Interferon alpha

B. Biseptol (Co-trimoxazole)

C. *Penicillin

D. Azidothymidin (Zidovudine)

E. Levomycetin (Chloramphenicol)

177. The dermatologist has an appointment with a 30-year-old man that complains of
severely itching rashes that especially disturb him at night. The rashes developed 2
weeks ago, after he had returned from a travel. Objectively on the lateral surfaces of
his fingers, hands, wrists, elbows. lower abdomen, genitals, and thighs there are paired
papulovesicles, single pustules, and scratch marks. What disease can be suspected?

A. *Scabies

B. Dermatitis

C. Pyoderma

D. Shingles

E. Eczema

178. Clinical trials have proved the “Lipoflavon” drug to be effective for treatment of
unstable angina pectoris in the control group and experimental group of patients.
Neither patients nor researchers knew who belonged to which group. Name this type
of study:

A. *Double blind study

B. Simple blind study

C. Total-blind study
D. Multicenter study

E. Triple-blind study

179. A 39-year-old man suffers from chronic rheumatic heart disease. He complains
of dyspnea during physical exertion, cough with expectoration, and palpitations.
Auscultation detects intensified I heart sound and diastolic murmur; the sound of
opening mitral valve can be auscultated at the cardiac apex. The II heart sound is
accentuated over the pulmonary artery. The patient is cyanotic. X-ray shows dilated
pulmonary root and enlargement of the right ventricle and left atrium. What is the most
likely diagnosis?

A. Patent ductus arteriosus

B. Pulmonary artery stenosis

C. *Mitral stenosis

D. Aortic stenosis

E. Coarctation of the aorta

180. A 43-year-old man complains of a protrusion in the right inguinal region, that
enlarges due to strain. He has been pre with this condition for 6 months. Within this
period the protrusion has grown. Objectively in the right inguinal region an elastic
protrusion 8×5 cm is visible. On palpation it disappears, leaving an empty space 4×4
cm between the pedicles of the Poupart ligament. “Cough push” sign is positive over
this opening. Make the diagnosis:

A. *Right-sided reducible inguinal hernia

B. Right-sided inguinal lymphadenitis

C. Cyst of the right spermatic cord

D. Right-sided reducible femoral hernia


E. Right-sided reducible arcuate line hernia

181. A 56-year-old woman was diagnosed with stage 2 hypertension of the 2nd degree.
She belongs to the group of moderate risk and has bronchial asthma. What group of
drugs is CONTRAINDICATED to this patient?

A. Calcium antagonists

B. Diuretics

C. Imidazoline receptor antagonists

D. *B-blockers

E. Angiotensin-converting enzyme inhibitors

182. In a rural health care area there is an increasing cervical cancer morbidity
observed. The decision is made to conduct a medical examination of the women living
in this locality. What type of medical examination is it?

A. Screening

B. Complex

C. Preliminary

D. *Target

E. Regular

183. A 25-year-old woman complains of fatigue, dizziness, hemorrhagic rashes on the


skin. She has been presenting with these signs for a month. Blood test: erythrocytes 10
109/L, Hb- 37 g/L, color index – 1.1, leukocytes – 12. 109/L. platelets – 42 . 10°/L.
What analysis would be the most advisable diagnosis-making in this case?

A. Splenic biopsy

B. Coagulation studies
C. US of the gastrointestinal tract

D. *Sternal puncture (bone marrow biopsy)

E. Liver biopsy

184. A patient has gradually lost consciousness. The skin is pale and dry. There is a
smell of ammonia from the mouth. Respirations are deep and noisy. Heart sounds are
muffled, pericardial friction rub is present Blood pressure is 180/130 mm Hg. Blood
test Hb- 80 g/L, leukocytes – 12 . 10°/L, blood glucose – 6.4 mmol/L, urea – 50
mmol/L creatinine – 1200 mcmol/L, blood osmolarity – 350 mOsmol/L. No urinary
excretion. Mak the diagnosis:

A. Acute renal failure

B. Acute disturbance of cerebral circulation

C. *Uremic coma

D. Hyperglycemic coma

E. Hyperosmolar coma

185. A 3-year-old child presents with dyspnea that abates in the sitting position.
occasional loss of consciousness and seizures, delayed physical development,
cyanosis, drumstick fingers. Echocardioscopy detects aortic dextroposition,
ventricular septal defect, pulmonary artery stenosis, and right ventricular hypertrophy.
What is the most likely diagnosis?

A. Coarctation of the aorta

B. Ventricular septal defect

C. Transposition of the great vessels

D. Acquired valvular disease

E. *Tetrad of Fallot
186. Increased general morbidity of the local population is observed in the area near a
factory, where the atmosphere is being intensively polluted with sulfurous gas. What
effect does polluted air have on the human body in this case?

A. Selective

B. *Chronic nonspecific

C. Acute specific

D. Acute nonspecific

E. Chronic specific

187. A woman in her early- to mid-thirties lost consciousness 3-5 minutes ago. On
examination: the skin is pale, no pulse over the carotid arteries, no spontaneous
respiration, pupils are dilated; the patient is unresponsive, presents with atony. The
patient’s condition can be determined as:

A. Syncope

B. Comatose state

C. Brain death

D. *Clinical death

E. Natural death

188. During regular medical examination a lyceum student presents with signs of
cheilitis that manifests as epithelial maceration in the area of lip seal. The lips are
bright-red, with single vertical cracks covered with brown red scabs. These clinical
signs are most likely caused by insufficient content of the following in the diet:

A. Ascorbic acid

B. *Riboflavin
C. Retinolt

D. Thiamine

E. Calciferol

189. A 59-year-old man complains of pain in his left eye and left side of his head,
significant vision impairment of the left eye, nausea, and vomiting. Visual acuity of
the right eye is 1.0. Visual acuity of the left eye is 0.03. attempts at correction bring no
improvement. Right eye intraocular pressure – 21 mm Hg, left eye intraocular pressure
– 65 mm Hg. Congestive injection is observed on the sclera of the left eye. The cornea
is thick and swollen. The anterior chamber is shallow, moist, and clear. The pupil is
dilated and unresponsive to the light, the fundus of the eye is not visible. What is the
most likely diagnosis?

A. Endophthalmitis of the left eye

B. Acute iridocyclitis of the left eye

C. Panophthalmitis of the left eye

D. *Acute attack of glaucoma of the left eye

E. Stage II intraocular tumor of the left eye

190. Employees work in conditions of high dust concentration. Certain chemical


(silicon dioxide content) and physical properties of dust aerosols contribute to the
development of occupational dust-induced diseases. What is the main physical
property of dust aerosols?

A. Ionization

B. Electric charge

C. Solubility

D. Magnetization

E. *Dispersion
191. A 13-year-old girl for a month has been complaining of fatigability, dull pain in
her right subcostal area, abdominal distension, and constipations. Abdominal palpation
reveals positive Kehr, Murphy, and Ortner signs, while Desjardins and Mayo-Robson
points are painless. Total bilirubin is 14.7 mcmol/L. predominantly indirect, ALT- 20
U/L, AST- 40 U/L, amylase – 6.3 mmol/L. Echocholecystography shows practically
no contraction of the gallbladder. Make the provisional diagnosis:

A. Chronic hepatitis

B. Acute pancreatitis

C. *Hypokinetic biliary dyskinesia

D. Chronic pancreatitis

E. Hyperkinetic biliary dyskinesia

192. A 35-year-old pregnant woman with degree 1 essential hypertension, developed


edemas and headache at the 33 week of her pregnancy. Objectively her general
condition is satisfactory, blood pressure – 160/100 mm Hg. normal uterine tone. Fetal
heart rate is 140/min., rhythmic. She was diagnosed with daily proteinuria – 4 g/L,
daily diuresis – 1100 mL. Creatinine – 80 mcmol/l, urea – 7 mmol/L platelets – 100.
10°/L. What complication of pregnancy occurred?

A. Severe preeclampsia

B. Renal failure

C. Mild preeclampsia

D. Hypertensive crisis

E. *Moderate preeclampsia

193. A 38-year-old woman complains of weakness, sleepiness, pain in the joints,


weight gain despite low appetite, and constipations. She presents with dry and
thickened skin. puffy and amimic face, narrowed palpebral fissures, thick tongue, and
deep hoarse voice. Her heart sounds are weak, pulse is 56/min. Low levels of free T4
are observed. This patient needs to take the following on a regular basis:

A. *Thyroxine

B. Mercazolil (Thiamazole)

C. Lithium carbonate

D. Furosemide

E. Calcium gluconate

194. A 28-year-old man complains of skin rash and itching on both of his hands. The
condition persists for 1.5 years. The exacerbation of his condition he ascribes to the
occupational contact with formaldehyde resins. Objectively the lesion foci are
symmetrically localized on both hands. Against the background of erythema with
blurred margins there are papulae, vesicles, erosions, crusts, and scales. What is the
most likely pathology?

A. Allergic dermatitis

B. Simple contact dermatitis

C. Erythema multiforme

D. *Occupational eczema

E. Idiopathic eczema

195. A 3-year-old child has been brought to a hospital with complaints of pain in the
legs, fever, and loss of appetite. Objectively: pale skin and mucosa, hemorrhagic rash.
Lymph nodes are enlarged, painless, dense and elastic, not matted together. Bones,
joints, and abdomen are painful. The liver and spleen are enlarged. Hemogram: Hb-
88 g/L, color index – 1.3. platelets – 80 – 10°/L, leukocytes – 25.8. 10°/L, lymphoblasts
– 70%, ESR 52 mm/hour. Make the provisional diagnosis:

A. *Acute leukemia
B. Acute rheumatic fever

C. Infectious mononucleosis

D. Hemorrhagic vasculitis (Henoch-Schonlein purpura)

E. Thrombocytopenic purpura

196. A patient has the second and third degree burns of 15% of the body surface. On
the 20th day after the trauma the patient presents with sharp increase of body
temperature, general weakness, rapid vesicular respiration: facial features are
sharpened, BP is 90/50 mm Hg, heart rate is 112/min. What complication is it?

A. Pneumonia

B. Purulent bronchitis

C. *Sepsis

D. Acute intoxication

E. Anaerobic infection

197. An 18-year-old girl was brought into the gynecology department with complaints
of elevated body temperature up to 37.8°C, sharp pain in her lower abdomen, more
intense on the right, and difficult defecation. Vaginal examination detected a painful
dense elastic formation 5×6 cm in the area of her right ovary Pregnancy test is negative.
What is the most likely diagnosis?

A. Ovarian apoplexy

B. Ectopic pregnancy

C. Appendicitis

D. Ovarian cyst rupture

E. *Torsion of ovarian tumor pedicle


198. On the 15th day after a small trauma of the right foot, the patient developed
indisposition, fatigability, irritability, headache, elevated body temperature, and
sensation of constriction. tension, and twitching in the muscles of the right shin, WL
disease can be suspected?

A. Erysipelas

B. *Tetanus

C. Thrombophlebitis of the popliteal artery

D. Anaerobic gas gangrene

E. Acute thrombophlebitis

199. On the day 4 after the cesarean section a woman developed fever with body
temperature up to 39°C and abdominal pa Pulse – 104/min. She vomited twice. The
patient is sluggish, her tongue is dry and has gray coating. The abdomen is distended
Signs of peritoneal irritation are positive in all segments. Peristalsis cannot be
auscultated. No passage of gas occurs. Uterine fundus is located at the level of the
navel. The uterus is painful on palpation. The discharge is moderate and contains blood
and pus. What is the most likely diagnosis?

A. Parametritis

B. Pelvic peritonitis

C. Progressive thrombophlebitis

D. Metroendometritis

E. *Diffuse peritonitis

200. A 51-year-old man complains of vomiting with blood. He has been drinking
alcohol excessively. Health disorder has been observed since he was 40, when he first
developed jaundice. On examination the skin and visible mucosa are icteric, with a
stellate vascular pattern. The patient is malnourished and presents with abdominal
distension, umbilical hernia, and ascites. The edge of the liver is tapered and painless.
+3 cm, the spleen is +2 cm. Blood test: Hb- 80 g/L, leukocytes – 3. 10°/L. platelets –
85 . 10°/L. What is the cause of portal hypertension in this patient?

A. Budd-Chiari syndrome

B. Hemochromatosis

C. Constrictive pericarditis

D. Thrombosis of the splenic vein

E. *Hepatic cirrhosis

You might also like